You are on page 1of 1072

1

MLE

Q bank, 12th Edition

smle ,2016


If you find any wrong answer kindly send the correction to the following email:
smle2015.2016@gmail.com
or on twitter on this account @smle_sa
to submit the corrections on the next SMLE Qbank.
If you have new question please send it to this link:
http://qus.li/smle

smle ,2016

SECTION 1 : Medicine
Infection ... 4
Allergy & immunology .......60
Cardiology ... 67
Endocrinology ..... 105
Pulmonology .... 139
Rheumatology ..157
Gastroenterology ..... 181
Neurology .. .207
Nephrology .232
Hematology 249
SECTION 2 :Pediatrics .. 278
SECTION 3 : Obs & Gyne .. 408
SECTION 4 : General surgery . 574
SECTION 5 : Orthopedics .. 704
SECTION 6 : ENT .... 755
SECTION 7 : Ophthalmology .. 777
SECTION 8 : Emergency medicine 807
SECTION 9 : Family.medicine & Ethics .... 839
community .... 877
.
SECTION 10 : Psychiatry. 895
SECTION 11 : Anesthesia.. 920
SECTION 12 : Dermatology.. 925
SECTION 13: Basic
Embryology... 952
Anatomy . 956
Physiology.. 989
Microbiology. 992
Biochemistry ... 1006
Genetic.... 1014
Histology.... 1019
Pharmacology.. 1021
smle ,2016

Medicine

smle ,2016

INFECTIOUS DISEASES & ANTIBIOTICS


1.

Diabetic patient developed fever, productive cough and SOB. Labs show high WBC. CXR (picture
was given which showed lower lobe infiltrates + air-fluid level).The drug that will be given to
the patient acts on which of the following?
A. DNA gyrase
B. 30 S ribosome
C. 50 S ribosome
D. Transpeptidase
Answer: C

http://emedicine.medscape.com/article/428135-overview#showall
2. Patient with ventilator associated pneumonia. Culture showed lactose non-fermenting, gram negative motile bacilli producing greenish colony + Oxidase positive. What is the organism?
A. Haemophilus influenzae
B. Streptococcus pneumoniae
C. Klebsiella or other gram negative bacteria
D. Pseudomonas aeruginosa
Answer: D

Reference: http://www.columbia.edu/itc/hs/medical/pathophys/id/2008/utiGNR.pdf

smle ,2016

6
3. Most specific test for TB?
A. PCR
B. Chest X ray
C. Sputum culture
D. PPD
Answer: C
Reference:
http://www.cdc.gov/tb/publications/factsheets/testing/diagnosis.htm
4. A patient with tuberculosis on medication for 3 months. He developed pins and needles sensation
of his lower limbs. Deficiency of which of the following caused his symptoms?
A. Niacin
B. Folic acid
C. Iron
D. Pyridoxine(B6)
Answer: D Vitamin B6 (pyridoxine) supplementation during isoniazid (INH) therapy is necessary in some
patients to prevent the development of peripheral neuropathy. Reference: Pubmed.
5. Best prophylactic against travelers diarrhea :
A. fresh fruit and vegetables
B. peeled fruit
C. daily antibiotic
D. drinks with rice
Answer: B
Reference: uptodate Basic advice for travelers to moderate or high-risk regions for travelers' diarrhea
includes eating only food that has been thoroughly cooked and served hot, fruits that the traveler peels
just prior to eating, and pasteurized dairy products. Beverages should be bottled or disinfected. Bottled
drinks should be requested without ice and should be drunk from the bottle with a straw rather than from
a glass. Hot tea and coffee are usually safe alternatives to boiled water.
Although antibiotics and other agents (namely bismuth salicylate) are effective in reducing the rate of
travelers' diarrhea for individuals traveling from resource-rich to resource-poor areas, we do not routinely
recommend chemoprophylaxis. Use of daily antibiotics is expensive, has potential side effects, can wipe
out normal gastrointestinal flora that may be beneficial, and can promote bacterial resistance.
However, chemoprophylaxis may be a reasonable approach in the setting of an underlying medical condition that would increase the risk of complications from diarrhea or would be severely exacerbated by
dehydration from diarrhea such that the benefits of using antibiotic prophylaxis outweigh its risks. Such
situations include known severe inflammatory bowel disease that could be exacerbated by an episode of
infectious diarrhea; severe vascular, cardiac, or renal disease that would be seriously compromised by
dehydration; or a severe immunocompromised state, such as advanced HIV disease or after a complicated
organ transplant.
If prophylaxis is administered, the options include:
Ciprofloxacin 500 mg once daily
Norfloxacin (not available in the US) 400 mg once daily
Rifaximin 200 mg once or twice daily
Bismuth subsalicylate two tablets chewed four times daily

smle ,2016

7
6. Long scenario about TB and what's the antibiotic to start :
A. Rifampicin, INH, Ethambutol and Pyrazinamide
B. Rifampicin, INH and Ethambutol
C. Rifampicin and INH
D. INH
Answer: A
For initial empiric treatment of TB, start patients on a 4-drug regimen: isoniazid, rifampin, pyrazinamide,
and either ethambutol or streptomycin.
Reference: http://emedicine.medscape.com/article/230802-treatment
Question is not complete, we do not know the result of the CXR
7. A middle-aged man presents with a cough and fever lasting several weeks. Posteroanterior chest
radiograph shows a prominent paratracheal area on the right, lymphadenopathy, a cavitary opacity in
the right upper lobe, and a focal consolidation in the middle lung zone on the right.
CXR shown below. What is the dx?

A. COPD
B. BA
C. Pneumonia
D. TB
Answer: D
8. What is the optimal duration of antibiotic treatment in strep throat?
A. 3 days
B. 5 days
C. 7 days
D. 10 days
Answer: D
Penicillin is the treatment of choice for strep throat. It is usually given in pill or liquid form, two to four
times per day for 10 days. Reference: UpToDate.
http://www.emedexpert.com/conditions/strep-throat.shtml
9. College student have meningitis. What to do as a prophylaxis to dorm friends next?
A. Isolate all contacts for 4 weeks
B. Immunize all contacts
C. Give antibiotic (Penicillin and other similar antibiotics) exact sentence was written
D. Do nothing

smle ,2016

8
Answer: C?
Close contacts of patient with meningococcal meningitis should receive rifampin. Second-line prophylaxis
regimens include ciprofloxacin, ceftriaxone or azithromycin. Reference: First Aid USMLE step 2 CK P189
reference: http://www.medbullets.com/step2-3-neurology/20285/meningitis
10. Young adult came complaining of painless penile ulcer, what is the appropriate investigation to do?
A. Blood culture
B. Excisional biopsy.
C. Swab culture and urinalysis
D. Dark field microscope.
E. CBC & ESR.
Answer: D
Primary syphilis usually begins with a single, painless, well-demarcated ulcer (chancre) with a clean base
and indurated border. Treponema pallidum is identified on darkfield microscopy or direct fluorescent antibody
testing
of
a
chancre
or
lymph
node
aspirate.
Reference:
http://www.aafp.org/afp/2012/0201/p254.html
11. HIV patient ... (symptoms of intestinal obstruction) did intestinal resection. They found tumor white
in color nearly encircling the wall. What is the tumor?
A. hodgkin
B. non Hodgkin
C. Adenocarcinoma
D. plasmacytoma
Answer: C all blood cancers except adenocarcinoma
12. Enteric fever resistance to chloramphenicol, whatsnext?
A. Double chloramphenicol
B. Ciprofloxacin alone
C. Add ciprofloxacin
D. IM ceftriaxone
Answer: B
Resistance patterns have led to a shift toward the third generation cephalosporins, azithromycin, and
fluoroquinolones as empiric therapy for typhoid fever while awaiting the results of antimicrobial susceptibilities
13. post streptococcal infection generalized petechial and plt =15 Management?
A-splenectomy
B-cyclo
C-VIII
D-IVIG
Answer: D
14. patient has UTI organism grows on both antiseptic & detergent
A. E.Coli
B. Proteus
C. Pseudomonas

smle ,2016

9
D. Staph aureus or strep
Answer: C
15. Man eating rice only, he has gingival and tongue lesions. Which of the following deficiency you will
find ?
A. Vitamin A.
B. Vitamin C.
C. Thiamin(B1).
D. Niacin (B3).
Answer: b
In Asia when white rice became more popular than brown rice, people started to develop a nerve damage
disease called beriberi. It can cause hypersensitivity of oral mucosa, burning sensation of the tongue and
loss of taste.
References:http://www.dietobio.com/vegetarisme/en/vit_b1.html+
https://books.google.com.sa/books?id=EBqYie7BGsYC&pg=PA12&lpg=PA12&dq=#v=onepage&q&f=false
16. What is the common cause to make the pt. Retire in KSA?
A. HBV
B. HBC
C. HIV
D. hep a
Answer: C or All?
17. what is the cause to use every year influenza vaccine?
A. resistance of antimicrobial
B. new antigen
C. different type of transmission
D. drift .. So could be new antigen.
Answer: D
http://www.uptodate.com/contents/treatment-and-prevention-of-acute-rheumatic-fever?source=search_result&search=rheumatic+fever&selectedTitle=2~111#H2
18.2 years old presented with fever for one month with the pic, lab shows Pancytopenia, what is the
cause:
A. leishmania
B. leukemia (I choose)
C. malaria
D. brucellosis
Answer:
19. patient has UTI organism grows on both antiseptic & detergent?
A. E.Coli
B. Proteus
C. Pseudomonas
D. Staph aureus or strep
Answer:

smle ,2016

10
Do not know
I guss B because it can protect herself against acidity
20. Immune deficient pt : what vaccine could be given :
A. Measles
B. Rubella
C. Pneumococci
D. varicella
answer: C
21. methicillin-resistance, sensitive to chloramphenicol and aminoglycosides What not to give?
A. Gentamycin
B. Azithromycin
C. Chloramphenicol
D. Vancomycin
E. azithromycin
Answer:
22. (long scenario) report {single strand RNA)?
A. hepatitis a
B. hepatitis b
C. hepatitis c
D. hepatitis d.
Answer:
the structure of hepatitis viruses are as follow:
Hepatitis A virus (non-enveloped single-stranded RNA)
Hepatitis B virus (enveloped double-stranded DNA)
Hepatitis C virus (enveloped single-stranded RNA)
Hepatitis D virus (envelop from HBV, single-stranded RNA)
Hepatitis E virus (non-enveloped single-stranded RNA)
Question is not complete
You have to know which one is +ve sense or -ve sense
http://www.baronerocks.com/index.php/mnemonics/mnemonics-microbiology/73-rna-sense-virusmnemonic
23. 75 years old male, asymptomatic, BM report:
increased lymphocytes
Immunohistochemistry:
Positive CD19, CD56
Treatment?
A. No treatment
B. Rituximab + CVB
C. Rituximab + Prednisolone* (my answer)
D. Cyclophosphamide
It is B or C
http://www.usmle-forums.com/usmle-step-1-forum/1065-cd-markers.html
24. Diarrhea \Shigella species treatment

smle ,2016

11
A. Metronidazole
B. Azithromycin
C. Amoxicillin
D. Ceftriaxone
Answer:
http://www.mayoclinic.org/diseases-conditions/shigella/basics/treatment/con-20028418
25. DM, hypothyroid, irregular menses female, present with recurrent itching & white adherent oral
plaque , +ve mantoux test , she was exposed to TB 4 years ago, immunoglobulin, WBC, RBCs all are
normal ; Dx ;
A. Chronic granulomatous disease
B. Chronic candidiasis
C. DiGeorge syndrome
D. Hyperglobulinemia ( or hypo i don't remember )
Answer: B
answer by exclusion is B
a= TB but the real cause is decrease immunity which lead to candida
C=WBC is normal so it is not the answer
D= immunoglobulin is normal so it is not the answer
26. Giardia. Diagnostic test
A. 3 stool parasite
B. 3 stool cultures
C. concentration test
D. stool immunoassay
Answer: D
http://www.cdc.gov/parasites/giardia/diagnosis.html
The question is not clear if he want sensitivity of other thing
27.Giardiasis treatment
A. Metronidazole
B. Paromomycin
C. Ciprofloxacin
Answer: A
http://www.mayoclinic.org/diseases-conditions/giardia-infection/basics/treatment/con-20024686
28. Gram negative bacteria oxidase, non lactose fermenter which of the following the best is antimicrobial?
A. Ceftriaxone
B. Cefepime
C. Ciprofloxacin
D. SMZ TMP (bactrim)
Answer :
not sure because they all anti pseudomonas
29. To prevent recurrence of UTI , What you prefer of the following circumstances ?
A. Decreased PH ,increased urea or urea(I did not remember),decreased urine osmolarity (I think)*
B. Decreased PH ,increased urea ,increased urine osmolarity
C. Increased PH ,increased urea or urea(I did not remember),decreased urine osmolarity

smle ,2016

12
D. Increased PH ,decreased urea or urea(I did not remember),increased urine osmolarity
Answer :
30. Patient came from Africa with fever, myalgia and arthralgia?
A. Ebola fever
B. Lassa fever
C. Yellow fever
D. Malaria
Answer:
31. 16 yr from Africa (Ginia) with painless neck mass for 5 weeks developed cough, fever , URS:
A. Burkitt lymphoma
B. infectious mononucleosis !!!
C. Hugging lymphoma
D. Lym Dx
Answer :
not sure but i go with (A) because he young from Africa
32. pt have parotitis, pain with eating that radiate to the ear, with nerve transmit pain with eating?
A. 8
B. 9
C. 10
D. 7
Answer: D
33.Patient with end stage liver disease, on central venous line, septic, blood showed ood c/s budding
yeast, what anti-fungal is appropriate at this stage?
A. Caspofungin
B. Flucytosine
C. fluconazole
D. Itraconazole
Answer:
34.HIV PT come with diffuse pasutle in skin and mouth Tt by :
A. Topical steroid
B. Oral AB
C. Topical AB
D. Chemo & radiotherapy
Answer: D?
35.malaria fast diagnostic test what u will see(something like that)
A. malaria antigen
B. malaria antibodies
C. malaria pigments
D. parasite sexual
Answer:
36. Patient with TB, how to prevent her dorm friends from having it:

smle ,2016

13
A. no need they want get it
B. isolate all contact for 4 weeks
C. immunization
D. penicillin and other antibiotics like it.
Answer ?
Household contacts of patients with MDR-TB have a particularly high risk for tuberculosis, 7.8% within 4
years in a study from Lima, Peru.[81] Limited data are available on regimens for the treatment of patients
exposed to MDR-TB. However, if treatment is initiated, at least 2 drugs should be given, and the index
isolate should be susceptible to all drugs used
http://emedicine.medscape.com/article/230802-treatment
37. patient has symptoms of infectious mononucleosis monospot , after 8 day he sudden complaining
of acute abdominal pain , decrease BP ? What will you do?
A. Antibiotic
B. Fluid resuscitation
C. urgent CT abdomen i think 3- Urgent gasto..
D. Barium enema
Answer:
38. A patient who was treated from TB, came to you complaining of eye pain. What is the cause?
A. Isoniazid
B. Rifampicin
C. Ethambutol
D. Pyrazinamide
Answer: C
Reference:http://www.drugs.com/sfx/ethambutol-side-effects.html
39. A patient with signs of TB. What vaccination you would give to his family?
A. MMR
B. DTaP
C. BCG
D. Polio
Answer: C
40. A patient was diagnosed with enteric fever. What is the presentation that he will have?
A. Confusion (or other CNS problems)
B. Maculopapular rash
C. Nausea, vomiting and loose stools
D. Abdominal pain, headache, fever( newly added )
Answer: D
Signs and symptoms of Salmonella typhi (aka Enteric Fever, Typhoid) include "Rose spot" rash (on anterior
thorax, upper abdomen), sustained fever 39 to 40 C (103 to 104 F), abdominal pain precedes diarrhea/
constipation, headache, loss of appetite and cough. Toronto Notes. If not treated the patient may become
delirious. Reference: Mayoclinic
41. What is the best ttt for traveler diarrhea ? 3 times
A. Ciprofloxacin

smle ,2016

14
B. Cipro
C. Amoxicillin
D. metronidazole
Answer: fluoroquinolones, such as ciprofloxacin or levofloxacin
http://www.uptodate.com/contents/travelers-diarrhea-clinical-manifestations-diagnosis-and-treatment
42. UTI case patient resistant to B LACTAMS, sensitive to fluoroquinolones, chloramphenicol, aminoglycosides which drug is contraindicated:
A. gentamicin
B. azithromycin
C. flucloxacillin
D. chloramphenicol
answer: A??
43. Safe vaccine you can give it to immunocompromised patient :
A. BCG
B. HBV
C. Polio salib
D. Polio salia
44. Which vaccine you will give to immunocompromised?
A. Recom. HBV
B. Sabin
C. Salk
D. BCG
Answer: Salk
45. old patient came to ER 4 w of fever cough night sweat... (clear T.B symptom) immediately do!??
A. put pt in negative pressure
B. give anti T.B drug
C. sputum culture
D. chest x ray
Answer:
CXR

46. What is the percentage of complete recovery from HCV?


A. 20
B. 40
C. 60
D. 80%
answer: 80%
Reference: Peer review
47. Pt signs of infectious mononucleosis LNs, when palpating spleen became cold and pale, what to do
next?
A. urgent gastroscopy
B. fluids
C. urgent CT

smle ,2016

15
D. antibiotic
Answer:
48. Patient is presented with hand cellulitis and red streaks in the hand and tender axillary lymphadenopathy. This condition is more likely to be associated with:
A. Malignancy
B. Pyoderma
C. Neuropathy
D. Lymphangitis
Answer : D
Lymphangitis is defined as an inflammation of the lymphatic channels that occurs as a result of infection
at a site distal to the channel
O/E
erythematous and irregular linear streaks extend from the primary infection site toward draining regional
nodes. These streaks may be tender and warm.
http://emedicine.medscape.com/article/966003-clinical#showall
49. Man came to the hospital with cellulitis after he got bitten by a wild cat. What is the most likely
organism ?
a. Pasteurellacaballi.
b. PasteurellaMultocida.
c. Pasteurellacanis.
d. Pasteurellaavium.
Answer: B
Reference:
http://www.ncbi.nlm.nih.gov/pmc/articles/PMC3122494/http://www.ncbi.nlm.nih.gov/pmc/articles/PMC3122494/
http://emedicine.medscape.com/article/224920-treatment

50. Picture of skin with chickenpox, with history of child with malaise and fatigue followed by single
macule then spread all over the body including the face, what is the treatment:
A. Antibiotics
B. Antiseptic
C. Acyclovir
Answer: C
Antiviral medications are recommended for people with chickenpox who are more likely to develop serious disease including
* otherwise healthy people older than 12 years of age
* people with chronic skin or lung disease
* people receiving steroid therapy
* some groups of pregnant women
Acyclovir, an antiviral medication, is licensed for treatment of chickenpox
http://www.cdc.gov/chickenpox/about/prevention-treatment.html
50. history of septic arteritis on cephalexin culture gram+ cocci resistant to ceresin (2gener cephalo)
what next:
A. Vancomycin

smle ,2016

16
B. stop drug
C. continuous
Answer: A
this pt developed MRSA (is any strain of Staphylococcus aureus that has developed, through the process
of natural selection, resistance to beta-lactam antibiotics, which include the penicillins(methicillin, dicloxacillin, nafcillin, oxacillin, etc.) and the cephalosporins)
https://en.wikipedia.org/wiki/Methicillin-resistant_Staphylococcus_aureus
51. Immunocompromised patient which vaccine will give for her brother ?
A. Varicella
B. Pneumococcus
C. influenza
Answer:
52. patient presented to the ER with cough hemoptysis night sweats and malaise.
what is most appropriate initial step in the management?
A. Isolation in negative pressure room
B. start anti TB
C. give OPD appointment after 2 weeks
Answer: B
TB case is usually confirmed by a positive culture for M. tuberculosis. However, in some cases, patients
are diagnosed with TB disease on the basis of their signs and symptoms, even if their specimen does not
contain M. tuberculosis CDC
53. Old lady with sharp chest pain and fever diagnosed with pericarditis what will you do to dx the case,
most accurate test is:
A. Acid fast stain
B. Pericardial biopsy (my answer)
C. Pleural aspiration
Answer: ???
54. Patient after 24 h from eating from restaurant and found gram positive bacilli in food ,what is the
organism ?
A. Salmonella
B. Shigella
C.
B.cerus

answer :
55. 12 y/o with bilateral lower lung infiltration treatment?
A. Ciprofloxacin
B. Azithromycin
C. penicillin
most likely B atypical pneumonia ttt by macrolides class if not available tetracycline

smle ,2016

17
56. Patient has central line what is the most common cause of infection
A. skin opening
B. drug administration
C. contamination from staff hand
Answer: A?
skin colonization is the most common source of cvc infection by microorganism from the patients skin
and occasionally the hands of the healthcare worker.
bacterial migration
migration of microbes from catheter-skin interface extraluminal to the catheter-vessel interface (most
common situation)
Staphylococci (S. aureus, coagulase negatives)
Reference : http://lifeinthefastlane.com/ccc/central-line-infections
uptodate
57. Repeated UTI, US revealed stones, organism with swarming motility, more details I dont remember,
the organism is?
A. Proteus mirabilis*
B. Pseudomonas
C. E coli
Answer:
58.Associated with animal bites?
A. Pasteurella multocida
B. Pseudomonas
C. Eikenella
Answer: A
59.HSV- 2 most appropriate treatments?
A. Acyclovir
B. Lamivudine
C. Ribavirin
Answer:
60. Patient with vesicles in forehead and supraorbital region for one day, what will udo?
A. Antiviral
B. Antiviral and refer to ophthalmic
C. Reassure
Answer: A
Varicella is self-limited in healthy children. A vaccine is available for infants, children, and
adults and is routinely used for disease prevention. Adults should be treated with systemic acyclovir.
Although acyclovir may speed the cutaneous course of zoster, pain control is most important for patients
with this disease.
First aid step2 ck
61. IV drug abuser has HIV +ve. oral thrush and symptoms of pneumonia. x ray and Broncho alveolar
lavage was done. after staining with silver stain. Pneumocystisjiroveci was found. what is the most likely
predictor of her HIV infection?
A. pneumocystis jiroveci infection

smle ,2016

18
B. IV drug use
C. candida
Answer: A
62.Patient on anti TB medication complaining of numbness and paresthesia what is the treatment:
A. Pyridoxine
B. Iron
C. thiamine
Answer:INH(isoniazid) will couse pripheral neuropathy .first aid p184
63.Patient presented with meningitis symptom and he swim in the river what is the organism
A. s. Pneumonia
B. h. Influenza
C. n. meningitis
Answer: not complete
Naegleria would be better answer but between these 3 I would choose
64.Q about susceptibility to fungal and viral infections or repeated fungal & viral infections:
A. T cell deficiency
B. B cell deficiency
C. phagocytes deficiency
Answer:Q not complete
65. Pt on cloxacillin for staph micro reported it is resistant to one of the cephalosporins what to do:
A. Continue cloxacillin
B. Start vancomycin
C. Stop antibiotics
Answer: B
66.Patient has been bitten by a cat, what is the organism that has been transmitted
A. staphylococcus aureus
B. Pasteurella multocida
C. streptococcus pneumoniae
Answer: B :Common bacteria involved in cat bite wound infections include the following:
Pasteurella species
Actinomyces species
Propionibacterium species
Bacteroides species
Fusobacterium species
Clostridium species
Wolinella species
Peptostreptococcus species
Staphylococcus species
Streptococcus species
http://emedicine.medscape.com/article/768875-overview

smle ,2016

19
67. Abdominal pain and fever, then constipation then diarrhea. Culture showed gram ve rod, nonlactose fermenting, oxidase -ve organism that produces hydrous sulphate. Most appropriate treatment? (Salmonella Typhi)
A. 50s subunit
B. DNA gyrase inhibitor
C. Transpeptidase
Answer: B
Reference: Peer Review
68. What two drugs are contraindicated together?
A. Digoxin and Levodopa
B. Sulphate
C. Tetracycline and aluminum
Answer: C
Reference:
http://reference.medscape.com/drug/tetracycline-342550
69. A 6 years of positive HBV mother not taken any vaccine except BCG after delivery. What will you
give him:
A. MMR,OPV,HBV,Varicella
B. HIB, MMR,OPV,HBV,PCV
C. HIB, MMR,OPV,HBV,.
Answer= MMR , OPV , Varicella , PCV, HIB All can be given so choose the answer that dont include HBV
Pediatric question
70. Bee sting since 18 hrs. With swelling and redness, what will you do?
A. Antihistamines
B. Steroids
C. Observe
Answer: A
Local reactions can be life threatening if swelling occludes the airway. initiate invasive measures to secure
the airway if this occurs. Otherwise, the following local care measures suffice:
Provide supplemental oxygen
Diphenhydramine limits the size of the local reaction.
Clean the wound and remove the stinger if present.
Apply ice or cool packs.
Elevate the extremity to limit edema.
*Treatment should include an initial intravenous (IV) bolus of 10-20 mL/kg isotonic crystalloids in addition
to diphenhydramine and epinephrine.
*If the patient has not removed the stinger, it should be removed as soon as possible by the first caregiver
on the scene. Delay increases venom load, so the fastest removal technique is the best. Pinching and
traction is an acceptable technique. Reference:http://emedicine.medscape.com/article/768764-treatment
71. which cause insomnia irritability and restless?
A. TCA
B. tetracycline antidepressant?

smle ,2016

20
C. SSRI
Answer: C
First aid p485
72. patient came from Sudan before two weeks and developed fever headache and vomiting what is
the best diagnostic test?
A. blood culture
B. Stools culture
C. Peripheral blood picture.
Answer C
Malarial infection is suspected. Individuals are generally asymptomatic for 12 to 35 days but can commence symptoms as early as 7 days (depending on parasite species) In most cases, the incubation period
for P. falciparum infection is about 12 to 14 days (range 7 to 30 days); most infections due to P. falciparum
become clinically apparent within one month after exposure. Detection of parasites on Giemsa-stained
blood smears by light microscopy is the standard tool for diagnosis of malaria and remains the most common onsite diagnostic method
References:http://www.uptodate.com/contents/clinical-manifestations-of-malaria
http://www.uptodate.com/contents/clinical-manifestations-of-malaria
http://www.uptodate.com/contents/diagnosis-of-malariahttp://www.uptodate.com/contents/diagnosis-of-malaria
73. Long scenario of patients labs show low hg low platelet with normal reticulocyte which Antibiotic
cause this ?
A. Tetracycline
B. Cloroampincoal
C. cefepime
Answer: B
http://reference.medscape.com/drug/chloramphenicol-iv-chloromycetin-342554
74. A patient with an infection that is resistant to Beta-lactam antibiotics, what antibiotic should be
given?
A. Azithromycin
B. Vancomycin
C. Gentamicin
Answer: B
Reference: Master the boards.
75.long Scenario about old male bedridden on foley's catheter he develop Gram -ve bacteria what is
the organism
A. E.coli
B. pseudomonas aeruginosa
C. strep. Pneumonia
Answer : A
E. coli Short-term catheters are placed for a mean duration of 2-4 days. The usual indications are for
acute illnesses, output measurement, perioperative routine, and acute retention. Approximately 15% of
patients develop bacteriuria, usually with a single organism (E coli).
http://emedicine.medscape.com/article/231574-overview

smle ,2016

21

76. A man came from Africa with some symptoms. Vital signs were provided. What is the diagnosis?
A. Yellow fever
B. Ebola
C. Lassa fever
Answer: ?not complete
Reference: http://lectures.shanyar.com/3rd_Stage/Medicine/Dr._Muhammad_Shaikhani/5._Viral_Hemorrhagic_Fevers.pdf
http://lectures.shanyar.com/3rd_Stage/Medicine/Dr._Muhammad_Shaikhani/5._Viral_Hemorrhagic_Fevers.pdf
http://lectures.shanyar.com/3rd_Stage/Medicine/Dr._Muhammad_Shaikhani/5._Viral_Hemorrhagic_Fevers.pdf

http://lectures.shanyar.com/3rd_Stage/Medicine/Dr._Muhammad_Shaikhani/5._Viral_Hemorrhagic_Fevers.pdf
77.Patient with oral ulcers. Culture showed herpesvirus
A. HSV 2
B. VSV
C. HSV 1
Answer: C
http://emedicine.medscape.com/article/218580-overview

78. Dog bite infection caused by?


A. Viral
B. Bacterial
C. Polymicrobial
Answer: C
major concern in all bite wounds is subsequent infection. Infections can be caused by nearly any group of
pathogens (bacteria, viruses, rickettsia, spirochetes, fungi). At least 64 species of bacteria are found in the
canine mouth, causing nearly all infections to be mixed.Common bacteria involved in dog bite wound
infections include the following:
Staphylococcus species
Streptococcus species
Eikenella species
Pasteurella species
Proteus species

smle ,2016

22
Klebsiella species
Haemophilus species
Enterobacter species
DF-2 or Capnocytophaga canimorsus
Bacteroides species
Moraxella species
Corynebacterium species
Neisseria species
Fusobacterium species
Prevotella species
Porphyromonas species
http://emedicine.medscape.com/article/768875-overview
79. patient can't take BCG vaccine Because he deficiency in
A. IL
B. TNF gamma
C. IFN
Answer:?not clear
UpToDate: Studies among infants demonstrate BCG-associated induction of CD4+ and CD8+ T cells, interferon (IFN)-gamma, interleukin (IL)-2+, tumor necrosis factor (TNF)-alpha+, and polyfunctional CD4+ T
cells
Studies in adults indicate that BCG induces CD4+, IFN-gamma responses, and IFN-gamma and TNF-alpha
secreting CD8+ cells with cytotoxic activity; data on polyfunctional T cells has been conflicting
In infants & adults, BCG works on: CD4 & CD8 T cells, IFN-gamma, IL 2, TNF-alpha
80. HIV patient test to confirm:
A. PCR
B. western blot
C. Elisa
answer: B western blot is confirmatory.
ELisa is for screening. If s negative, other tests are not usually needed.
http://emedicine.medscape.com/article/211316-overview
81. HIV patient, presented with SOB and productive cough.Lung biopsy showed soap bubble like intraalveolar lesions with exudates, small cyst, stained by silver stain.
A. Pneumocystis jiroveci
B. Aspergillus fumigatus
C. Cryptococcus neoformans
Answer: A
http://emedicine.medscape.com/article/225976-overview
First aid p196
82.HIV presents commonly with?
A. opportunistic infection
B. chronic diarrhea
C. generalized lymphadenopathy
Answer: C

smle ,2016

23
The patient may present with signs and symptoms of any of the stages of HIV infection. Acute seroconversion manifests as a flu like illness, consisting of fever, malaise, and a generalized rash. The asymptomatic phase is generally benign. Generalized lymphadenopathy is common and may be a presenting symptom.
http://emedicine.medscape.com/article/211316-clinical
83. Patient had 1.5 cm calcified lesion in the routine chest x ray . He's symptomless. Next action will be.
Answer:
A. observation
B. percutaneous biopsy
C. transbronchial biopsy
answer: C
http://emedicine.medscape.com/article/2139920-overview
Patients with solitary pulmonary nodules are usually asymptomatic. However, solitary pulmonary nodules
can pose a challenge to clinicians and patients. Whether detected serendipitously or during a routine investigation,Patients with early lung cancer, when the primary tumor is less than 3 cm in diameter without
evidence of lymph node involvement or distant metastasis (stage 1A), have a 5-year survival rate of 7080%. Therefore, prompt diagnosis and management of early lung cancer manifesting as a solitary pulmonary nodule is the the best chance for cure.
Because the yield from bronchoscopy is only 10-20% when the nodule is less than 2 cm in diameter, bronchoscopy and transbronchial needle aspiration (TBNA) may be helpful when the lesion is either endobronchial in location or near a large airway.
84. Dental caries caused by which organism?
A. Candida
B. HSV
C. Streptococcus mutans
Answer: C
Caries develop through a complex interaction in which transmissible cariogenic oral microflora, primarily
Streptococcus mutans and lactobacilli, metabolize fermentable dietary carbohydrates, resulting in lactic
acid production.[3] The resultant drop in pH at the tooth surface results in dissolution of the mineral component of the enamel.
http://www.medscape.com/viewarticle/722767_2
85. patient with hemosiderin laden macrophages- what is the diagnosis?
(Answers we're all pulmonary causes)
A. CMV
B. Pneumocystis jiroveci
C. Chronic lung infection
Answer:
Diffuse pulmonary hemorrhage (diffuse interstitial pulmonary disease - DIPD)
86. Drug that cause white patches in mouth. What is the inhaler?
A. Ipratropium
B. Short acting beta2 agonist
C. Steroid
Answer: C
Children on inhaled steroids also have increased incidence of oral candidiasis.

smle ,2016

24
Reference: http://emedicine.medscape.com/article/969147-overview
87. Patient is allergic to sulfa drugs and penicillin and shellfish. She has UTI what antibiotic you will give?
A. Nitrofurantoin
B. Trimethoprim Sulfamethoxazole
C. Amoxicillin
Answer: A
http://emedicine.medscape.com/article/233101-overview

88. A married woman was diagnosed with UTI. Urine culture revealed Staphylococcus Saprophyticus.
What you should ask this patient about?
A. Use of condom and spermicides.
B. Alcohol consumption.
C. Fecal incontinence.
Answer: a :
Sexual intercourse contributes to increased risk, as does use of a diaphragm and/or spermicide.
http://emedicine.medscape.com/article/233101-overview
89. Pt with 1st metatarsal joint pain, redness and erythema High temperatures? What is the cause?
A. staph areus
B. NA monourate crystal
C. Ca pyrophosphate crystal
Answer:B
na monourate crystals , Gout and pseudogout are the 2 most common crystal-induced arthropathies.
Gout (see the image below) is caused by monosodium urate monohydrate crystals
http://emedicine.medscape.com/article/329958-overview
90. Patient has whitish elevated patchy lesion over the dorsal surface of the tongue, it does not remove
after scrubbing the lesion, what is the most likely diagnosis?
A. Dysplasia
B. Neurofibroma
C.
Foreign
body

Answer
91. medical student diagnosed as meningitis, what you do for him ?
A. start antibiotics
B. isolate him for 4 weeks
C. give him influenza vaccine for his colleagues
answer: a
http://emedicine.medscape.com/article/232915-overview
92. Known case of HIV , Have several problems on iris ,Including the word"necrotizing"

smle ,2016

25
Wt the cause?
A. HIV
B. Cytomegalovirus?
C. toxoplasma
Answer:B-CMV http://emedicine.medscape.com/article/215702-overview
93. (long scenario) how does hyperglycemia cause infection to occur?
A. allow bacteria to grow.
B. decrease immune response.
C. impairs phagocytosis
Answer: C
http://www.ncbi.nlm.nih.gov/pubmed/16006275
Studies have demonstrated impairment of host defenses, including decreased polymorphonuclear leukocyte mobilization, chemotaxis, and phagocytic activity related to hyperglycemia.
94. patient have a chronic liver disease and you found a fungal infection which drug you will use
A. fluconazole
B. itraconazole
C. amphotericin B
95. patient has DM & HTN ( I think on captopril which induce dry cough ) presentwith mild ankle edema
and +ve protein in urine what will give instead ofACEI! ,, Im not sure about this Q
A. ARB
B. Thiazide
C. CCB
Answer:
96. patient with secondary syphilis was treated with penicillin 2 hours from the firstdose he developed
fever myalgia and malaise. what is your management?
A. Epinephrine
B. Antihistamine
C. symptomatic management with paracetamol
Answer: C
97.old patient on antiviral treatment for the influenza, intranasal, the mechanism of action of this antiviral is to:
A. Inhibit viral neuraminidase*
B. DNA gyrase
C. Polymerase
Answer:
98. Patient c/o fever ,productive cough , x-ray show right lung opacification and obliteration of right
costophrenic angle what you'll find on examination?
A. crepitations on both lungs
B. absence of vesicular breathing sounds of rt side
C. presence of bronchial breathing
Answer

smle ,2016

26
99. KSA have implemented strong regulations regarding worker health cares, which of the following
diseases if the worker had, he can't work?
A. HeB
B. HeC
C. HIV
Answer:
100. bacteria grow in antiseptic and detergent
A. E. coli
B. klebsiella
answer
All the bacterial contaminants of these antimicrobial substances were Klebsiella species of which all K.
pneumoniae species were multidrug
http://pubs.sciepub.com/ajnr/2/3/3/ http://pubs.sciepub.com/ajnr/2/3/3/
101.

Patient can't take BCG vaccine Because he has deficiency in :


A. IL
B. TNF gamma
Answer: B
102. Patient with hyperpigmented non pruritic papules in the dorsum of the hands notresolved with
antifungal?
A. tinea corporis
B. lichen planus
Answer: B?
lichen planus is hyperpigmented lesions thats doesnt resolved by antifungals but it cause pruritis!!
Treatment: Topical corticosteroids with occlusion or intradermal steroid injections.
source: Toronto notes 2014 D17
103. Vesicles on the eye and forehead?
A. Herpes zoster ophthalmicus.
B. Herpesvirus ophthalmic.
Answer: A
104. The type of Hepatitis B vaccine is ;
A. Recombinant
B. inactivated
Answer:
105. Best method to prevent food poisoning?
A. high cooked food and rewarm
B. other wrong answers
answer: A
106. 19-year-old boy after unprotected sex he developed generalized rash involving hand and feet
A. Chlamydia
B. Syphilis
Answer: B

smle ,2016

27

107. Pt with +ve PPD for the first time and -ve CHEST X-RAY. No signs or symptoms of TB, what to do:
A. Reassure
B. INZ for 6 months
C. Others
Answer: B
source:
http://www.uptodate.com/contents/treatment-of-latent-tuberculosis-infection-in-hiv-uninfected-adults?source=see_link
108.

Treatment of chlamydia:
A. azithromycin
B. doxycycline
C. metronidazole
Answer: A or B
Azithromycin (oral one dose) or doxycycline (oral for 7 days ) + treat all sexuall partners
Reference: step up to medicine
109.
Generalized cervical lymphadenopathy + mild tenderness + low grade fever. What's the most
likely diagnosis?
A. small lymphocytic lymphoma
B. hodgkin's lymphoma
C. EBV
answer: C
EBV (LYMPHOMA IS NOT TENDER)
small lymphocytic lymphoma presented by generalized lymphadenopathy
110.
A.
B.
C.

cervical infection can enter to superior-mediastinum through :


Retro-pharyngeal space>>
Para-pharyngeal space
Carotid sheath

111. Bacteroids in gunshot wound abdomen, what antibiotics:


A. clindamycin
B. metronidazole or cefoxitin)
C. Others options
Answer:
112.diagnosis of pertussis:
A. Nasopharyngeal swab
B. Throat swab
Answer: A
The culture specimen should be obtained during the first 2 weeks of cough by using deep nasopharyngeal
aspiration
http://emedicine.medscape.com/article/967268-overview
113. Hemosiderin laden in alveolar lavage?
A. Due to PCP
B. CMV

smle ,2016

28
Answer:
114. Patient with meningitis and facial nerve palsy, what is the organism ?
A. H. Influenza
B. Streptococcus pneumoni
Answer:
115. Scenario of patient with mastoiditis what is the proper antibiotic:
A. amoxicillin
B. azythro
Answer:
116. after removal of phlegmon they found Enterococcus faecalis ? what antibiotic ?
A. ceftriaxone
B. ciprofloxacin
Answer:
117. girl prick her nail when she cut thr rose. the lesion become ulcerated then transmitted lesion on
same lymphatic drainage, what the organism?
A. Aspergillus
B. sporotrichosis
s
answer :
118. Why patient will be in a risk of Neisseria infection defect in which of the following:
A. classical complement pathway
B. final lytic complement pathway
Answer:
119. Smoker c/o whitish lesion on mouth, not removed by wash, what is Your diagnosis?
A. Thrush
B. Leukoplakia
answer
120. Transmitted of parasite by ingestion of undercooked meat?
A. Schistosoma
B. Taenia saginata /Tenia solium*
Answer:
121. organism undercooked beef?
A. Entamoeba
B. Tinea
Answer: B (Taenia saginata - beef Tenia solium- pork)
122. TB test :
A. IFN y
B. TGN or IGRAs ( some like that )

smle ,2016

29
Answer: B
TB blood tests: TB blood tests (also called interferon-gamma release assays or IGRAs) measure how the
immune system reacts to the bacteria that cause TB. An IGRA measures how strong a persons immune
system reacts to TB bacteria by testing the persons blood in a laboratory.
Two IGRAs are approved by the U.S. Food and Drug Administration (FDA) and are available in the United
States:
1.
QuantiFERONTB Gold In-Tube test (QFT-GIT)
2.
T-SPOT.TB test (T-Spot)

Positive IGRA: This means that the person has been infected with TB bacteria. Additional
tests are needed to determine if the person has latent TB infection or TB disease. A health care worker
will then provide treatment as needed.

Negative IGRA: This means that the persons blood did not react to the test and that latent
TB infection or TB disease is not likely.
IGRAs are the preferred method of TB infection testing for the following:

People who have received bacille CalmetteGurin (BCG). BCG is a vaccine for TB disease.

People who have a difficult time returning for a second appointment to look for a reaction
to the TST.
There is no problem with repeated IGRAs.
http://www.cdc.gov/tb/topic/testing/
123.patient with renal stones and hematuria, most likely organism?
A. Schistosoma haematobium
B. E col
Answer:
124.

Patient transverse blood from Kenya and had anal infection


A. Hbv
B. Syphilis
Answer:
125. Pt come from Africa:
A. Ebola
B. HIV
Answer: incomplete
126. patient had throat infection 2 weeks ago was developed hematuria how to treat:
A. Corticosteroids
B. Thiazide
Answer: C
the diagnosis is post strep glomerulonephritis
During the acute phase of the disease, restrict salt and water. If significant edema or hypertension develops, administer diuretics. Loop diuretics increase urinary output and consequently improve cardiovascular
congestion and hypertension.
For hypertension not controlled by diuretics, usually calcium channel blockers or angiotensin-converting
enzyme inhibitors are useful. For malignant hypertension, intravenous nitroprusside or other parenteral
agents are used.
Other features of therapy are as follows:
Indications for dialysis include life-threatening hyperkalemia and clinical manifestations of uremia

smle ,2016

30
Restricting physical activity is appropriate in the first few days of the illness but is unnecessary once the
patient feels well
Steroids, immunosuppressive agents, and plasmapheresis are not generally indicated
Reference:
http://emedicine.medscape.com/article/240337-treatment#d8
127. organism gram positive cluster? what will be positive?
A. Coagulase
B. Oxidase
Answer:
128. Pt with ulcer on penis?
A. chancer
B. syphilis
Answer:
129. Patient with Sore throat, skin rash , spleen enlarge, What is the cause ?
A. Epstein-bar virus EBV
B. CMV
Answer: A
http://emedicine.medscape.com/article/222040-clinical
130. How to diagnose Giardia lamblia?
A. three consecutive stool analysis
B. three separate stool analysis* (not sure)
Answer:
131. What is the organism gram -ve bacilli, non-lactose fermenter, oxidase +ve
A. Pseudomonas
B. Proteus
Answer:
132. Old man with DM, has redness in calf area, raised and painful, tender:
A. Cellulitis
B. Diabetic neuropathy
Answer:
Erysipelas if demarcated
133.
Patient with damaged valve after a tooth extraction he got infective endocarditis, what is the
organism:
A. strep viridans
B. staph aureus
Answer: A
These account for 50%80% of IE cases. Streptococcus viridans (eg, S. anguis, S. milleri, S. mutans, S. mitior) make up the normal bacterial flora of the pharynx and upper respiratory tract. Tonsillectomy, dental
extraction, and dental cleaning can result in bacteremia and lead to infection.
134.

fever with spot in molar tooth?

smle ,2016

31
A. Measles
B. Rubella
Answer:
135. History of infection, Hb low, WBC high, what is the investigation:
A. Bone marrow biopsy
B. Hb electrophoresis
Answer:
136. patient with meningitis, which combination of antibiotic should be given?
A. vancomycin + ampicillin
B. amoxicillin + gentamycin
Answer:
137. Which of the following organisms is seen in patients with chronic granulomatous disease?
A. C.Difficile
B. Staph aureus
answer B
138. patient in ICU on ventilator develop yeast infection what is the treatment?
A. Fluconazole
B. itraconazole
Answer: A
Amphotericin B is initial therapy
aspergillosis > voriconazole
medscape
http://emedicine.medscape.com/article/213853-treatment

139. positive culture of budding yeast in urine what is the management ? ( repeted )
A. Fluconazole
B. caspofungin
Answer:
A (UpToDate & Medscape) in TB patients were highest among injection drug users, homeless persons,
correctional facility inmates, and alcoholics (35, 22, 16, and 15 percent, respectively)
140. Increase of which of the following prevalence cause reactivation of TB in developed countries?
A. DM
B. HIV
Answer: Bhttp://www.medscape.com/viewarticle/443137_2
141. parasite in soil contamination:
A. tenia saginata
B. ascaris-bancrofti
Answer:
The main species that infect people are the roundworm (Ascaris lumbricoides), the whipworm (Trichuris
trichiura) and hookworms (Necator americanus and Ancylostoma duodenale).

smle ,2016

32
http://www.who.int/mediacentre/factsheets/fs366/en/
142. woman with recurrent UTI , WHY ?
A. Because it cleans itself of the from anus to vulva
B. using of cervical cap for contraception
answer: B
Note : cervical cap usage require spermicidal cream with it, & since spermicide is a risk for recurrent UTI
according to the uptodate , i think it will be the answer.
143.
A farmer presented with 2 weeks history of fever, headache and one more symptom. What is
the most likely diagnosis?
A. Brucellosis
B. Meningitis
Answer: A,
The trick in the duration, bacterial meningitis usually present over several hours
(Cryptococcal meningitis may be present for several weeks- but it's not one of the options). In the other
hand, brucellosis may show up any time from a few days to a few months after the person get infected.
Brucellosis is an infectious disease. People can get the disease when they are in contact with infected
animals or animal products contaminated with the bacteria. Animals that are most commonly infected
include sheep, cattle, goats, pigs, and dogs, among others.
Signs and symptoms include fever, headache, sweats, malaise, anorexia pain in
muscles, joint, and/or back, fatigue,...
Reference: Centers for Disease Control and Prevention.
144. long scenario about pt coming from Africa 3 weeks ago with fever no other +ve points What is
the Dx?
A. Ebola
B. yellow fever
answer:
yellow fever incubation period 3-6 days and in this scenario 3 weeks so we exclude it
ebola from 2 to 21 days so it could be
if malaria present most likely 630 days (98% onset within 3 months of travel
DISEASE
USUAL INCUBATION PE- DISTRIBUTION
RIOD (RANGE)
Incubation <14 days
Chikungunya

24 days (114 days)

Tropics, subtropics

Dengue

48 days (314 days)

Topics, subtropics

Encephalitis, arboviral (Jap- 314 days (120 days)


anese encephalitis, tickborne encephalitis, West
Nile virus, other)

Specific agents vary by region

Enteric fever

Especially in Indian subcontinent

smle ,2016

718 days (360 days)

33
Acute HIV

1028 days (10 days to 6 Worldwide


weeks)

Influenza

13 days

Worldwide, can also be acquired


while traveling

Legionellosis

56 days (210 days)

Widespread

Leptospirosis

712 days (226 days)

Widespread, most common in tropical areas

Malaria, Plasmodium falci- 630 days (98% onset Tropics, subtropics


parum
within 3 months of travel)
Malaria, P. vivax

8 days to 12 months (al- Widespread in tropics and subtropmost half have onset >30 ics
days after completion of
travel)

Spotted-fever rickettsiae

Few days to 23 weeks

Causative species vary by region

Incubation 14 Days to 6 Weeks


Encephalitis, arboviral; en- See above incubation peri- See above distribution for relevant
teric fever; acute HIV; lepto- ods for relevant diseases diseases
spirosis; malaria
Amebic liver abscess

Weeks to months

Most common in developing countries

Hepatitis A

2830 days (1550 days)

Most common in developing countries

Hepatitis E

2642 days (29 weeks)

Widespread

Acute
schistosomiasis 48 weeks
(Katayama syndrome)

Most common in sub-Saharan Africa

145.
A farmer presented with 2 weeks history of fever, headache and one more symptom. What is
the most likely diagnosis?
A. Brucellosis
B. Meningitis
Answer: A
The trick in the duration, bacterial meningitis usually present over several hours ( Cryptococcal meningitis
may be present for several weeks - but it's not one of the options). In the other hand, brucellosis may
show up anytime from a few days to a few months after the person get infected.
Brucellosis is an infectious disease. People can get the disease when they are in contact with infected
animals or animal products contaminated with the bacteria. Animals that are most commonly infected
include sheep, cattle, goats, pigs, and dogs, among others. Signs and symptoms include fever, headache,
sweats, malaise, anorexia pain in muscles, joint, and/or back, fatigue,
Reference: Centers for Disease Control and Prevention.

smle ,2016

34

146.
A case of a patient diagnosed to have cutaneous leishmania or baghdad boil which type of
leishmania ?
A. Kala Azar /
B. Donovan
Answers is Tropical
Cutaneous leishmaniasis (also known as oriental sore, baghdad boil)i s the most common form of leishmaniasis affecting humans caused by (e.g., L. major and L.tropica)
Visceral leishmaniasis (VL), also known as kala-azar caused by (e.g., L. infantum and L. donovani)

147. A child who ate honey develops progressive paralysis symptoms?


A. Botulism
B. C. Perfringens
Answer: A
Honey can contain the bacteria that causes infant botulism so, children less than 12 months old should
not be fed honey. Honey is safe for persons 1 year of age and older.
Reference: http://www.cdc.gov/nczved/divisions/dfbmd/diseases/botulism/
148. Fever and cough then facial nerve then loss ofreflexes?
A. Tetanus
B. Botulism
Answer:B
149. patient with honeymoon cystitis. Which of following could be the organism?
A. Staph.saprophyticus
B. E,coli
Answer: A
Honeymoon cystitis is a bladder infection that results from sexual activity. It occurs when the bacteria
travels in the upward direction from the urethra to the bladder.
http://www.m.webmd.com/a-to-z-guides/tc/understanding-bladder-infections-basic-information?page=2
http://www.urineinfection.net/honeymoon-cystitis-symptoms-and-relief/
150. Positive PPD test but normal X-ray, what will you do?
A. Isoniazid for 6 months
B. Rifampin for 6 months

smle ,2016

35
Answer: a
151. TB case, what is the next appropriate step to get a definitive dx?
A. Sputum smear under microscope
B. Sputum culture
Answer: B
Culture of sputum is the gold standard but can take weeks to obtain , acid fast stain can yield rapid preliminary results but lacks sensitivity
Ref: First Aid USMLE STEP 2 CK
for AFB is the most specific test for TBand allows direct identification anddetermination of susceptibility of the causative organism.
http://emedicine.medscape.com/article/230802-workup#c1
152. Treatment of meningitis?
A.Ampicillin
B.doxycycline
Answer:A
less than 1m amoxicillin + ceftriaxone or genta
1m- adulthood vancomycin + ceftriaxone or cefotaxime
more than 60Y ampicillin + vancomycin + cefotaxime or ceftriaxone
Ref: 1st aid USMLE step 2
http://emedicine.medscape.com/article/232915-treatment#d8
153. Long scenario, bloody diarrhea and RBC in urine after 7 days hx of food poisoning, Treatment?
A. STEROID
B. ANTIBIOTIC
answer: conservative, This is E coli: HUS As the infection progresses, diarrhea becomes watery and then
may become grossly bloody; that is, bloody to the naked eye. E. coli symptoms also may include vomiting
and fever, although fever is an uncommon symptom.
On rare occasions, E. coli infection can cause bowel necrosis (tissue death) and perforation without progressing to hemolytic uremic syndrome (HUS)a complication of E. coli infection that is now recognized
as the most common cause of acute kidney failure in infants and young children.
http://emedicine.medscape.com/article/175569-treatment
154.
Pt with barking cough and 38 temp which of the following symptoms is associated with this
disease?
A. Cyanosis
B. Wheezing
Answer: A
explanation: scenario of barking cough + fever suggestive of croup which is associated with stridor and
possibly cyanosis
http://emedicine.medscape.com/article/303533-overview
155. Patient with acute rheumatic fever show acute cardiac symptoms, what is the treatment?
A. IV penicillin
B. IM steroid
Answer: B

smle ,2016

36
Treatment of ARF consists ofanti-inflammatory therapy, antibiotic therapy, and heart failure management.
Patients with severe carditis (significant cardiomegaly, congestive heart failure, and/or third-degree heart
block) should be treated with conventional therapy for heart failure.
Corticosteroids should be reserved for the treatment of severe carditis.
http://emedicine.medscape.com/article/236582-treatment
156. Patient with fever, pre-auricular swelling, (description of the swelling was provided), what is
the diagnosis?
A. Mumps
B. Pre-auricular lymphadenopathy
Answer: B
157. Bacterial meningitis in LP
A. decrease glucose and increase protein
B. increase glucose and decrease protein
Answer: A
http://emedicine.medscape.com/article/2172226-overview
158. Staph saprophyticus vaginal infection, whats a risk factor for it:
A. Septicidal in condoms
B. douching habit
Answer:
159. vesicles highly suspected roundworms:
A. Ascaris
B. Taeniasaginata
Answer: A
http://emedicine.medscape.com/article/212510-overview#showall
160. Mycobacterium tuberculosis , that is the best culture media ?
A. Blood agar
B. Lwenstein-Jensen medium
https://en.wikipedia.org/wiki/L%252525C3%252525B6wenstein%252525E2%25252580%25252593Jensen_mediumhttps://en.wikipedia.org/wiki/L%252525C3%252525B6wenstein%252525E2%25252580%25252593Jensen_medium
Answer: B
Explanation:
Mycobacterium tuberculosis on Lwenstein-Jensen medium after 6 weeks of cultivation, 37C. Typical
nonpigmented, rough, dry colonies on Lwenstein-Jensen medium. The green color of the medium is due
to the presence of malachite green which is one of the selective agents to prevent growth of most other
contaminants. Unlike many other solid cultivation media used in clinical microbiologyLwenstein-Jensen
medium (or e.g., Ogawa medium) doesn't contain any agar (solid consistence is attained by heat coagulation of the egg albumin).
-While the LJ medium is the most popular means of culturing Mycobacteria, as recommended by the International Union against Tuberculosis (IUAT), several alternative media have been investigated.

smle ,2016

37
Solid media
Egg-based Petragnini medium and Dorset medium
Middlebrook 7H10 Agar
Middlebrook 7H11 Agar

Blood based Tarshis medium

Serum based Loeffler medium

Potato based Pawlowsky medium


Liquid media
Dubos' medium
Middlebrook 7H9 Broth

Proskauer and Beck's medium

Sula's medium

Sauton's medium
161. Painless genital ulcer + lymph nodes enlargement
A. Syphilis
Answer:
Syphilis
Single, painless, well-demarcated ulcer (chancre) with a clean base and indurated border
Mild or minimally tender inguinal lymphadenopathy
REF : http://www.aafp.org/afp/2012/0201/p254.htmlhttp://www.aafp.org/afp/2012/0201/p254.html
162. enterococcus faecalis antibiotic and the patient is allergic to ampicillin, what to give?
A. Vancomycin
Answer: A
Ampicillin is the drug of choice for monotherapy of susceptible E faecalis infection. For most isolates, the
MIC of ampicillin is 2- to 4-fold lower than that of penicillin. For rare strains that are resistant to ampicillin
because of beta-lactamase production, ampicillin plus sulbactam may be used. Vancomycin should be
used in patients with a penicillin allergy or infections with strains that have high-level penicillin resistance due to altered PBPs
http://emedicine.medscape.com/article/216993-treatment
163. What is the best treatment for traveler's diarrhea ?
A. ciprofloxacin
Answer: A
Explanation: Because traveler's diarrhea tends to resolve itself, you may get better without any intervention. It's important to try to stay hydrated with safe liquids, such as bottled water or canned juice. If you
don't seem to be improving quickly, you can turn to several medications to help relieve symptoms such
as Anti-motility agents, however you can use Abx to decrease the duration and severity of the disease.
Antibiotics Used for the Treatment of Travelers Diarrhea:
Source:
http://www.aafp.org/afp/2005/0601/p2095.htmlhttp://www.aafp.org/afp/2005/0601/p2095.html
164. Brucellosis prevention?
A. Pasteurization
answer: A
The best way to prevent brucellosis infection is to be sure you do not consume:
undercooked meat

smle ,2016

38
unpasteurized dairy products
http://www.cdc.gov/brucellosis/prevention/index.html
165. Female pt diagnosed with PID on ceftriaxone with no benefit wt is the organism
A. herpes
answer: chlamydia.
Chlamydia & gonorrhea are the most common causative organisms causing PID. Ceftriaxone is used for
gonorrhea Tx.
http://emedicine.medscape.com/article/256448-overview
166. woman with recurrent UTI, WHY?
A. Because it cleans itself of the from anus to vulva ( my answer )
Answer:
Biologic or genetic factors Women with recurrent UTI have been shown to have an increased susceptibility to vaginal colonization with uropathogens
Behavioral risk factors:
Spermicide use during the past year
Having a new sex partner during the past year
Having a first UTI at or before 15 years of age
Having a mother with a history of UTIs
Pelvic anatomy: Pelvic anatomy may predispose to recurrent UTI in some women,
Postmenopausal women
Urinary incontinence
Presence of a cystocele
Post Voiding residual urine
No associations were found between a history of recurrent UTI and pre- and postcoital voiding patterns,
frequency of urination, delayed voiding habits, wiping patterns, douching, use of hot tubs, frequent use
of pantyhose or tights, or body mass index.
http://www.uptodate.com/contents/recurrent-urinary-tract-infection-in-women#H5
http://www.uptodate.com/contents/recurrent-urinary-tract-infection-in-women - H5
http://www.uptodate.com/contents/recurrent-urinary-tract-infection-in-women - H5
167. Patient on central cath developed infection. Blood gram stain shows
budding yeast?
A. Fluconazole
Answer: A
Reference: Toronto notes
168. Syphilis case, what do you give?
A. Penicillin Benzathine (aka penicillin G)
Answer: A
Penicillin is the only recommended therapy for neurosyphilis, congenital syphilis, or syphilis during pregnancy. Rarely, T pallidum has been found to persist following adequate penicillin therapy; however, there
is no indication that the organism has acquired resistance to penicillin
The following regimens are recommended for penicillin treatment:

smle ,2016

39

Primary or secondary syphilis - Benzathine penicillin G 2.4 million units intramuscularly (IM) in a single dose
Early latent syphilis - Benzathine penicillin G 2.4 million units IM in a single dose
Late latent syphilis or latent syphilis of unknown duration - Benzathine penicillin G 7.2 million units
total, administered as 3 doses of 2.4 million units IM each at 1-week intervals
Pregnancy - Treatment appropriate to the stage of syphilis is recommended.
http://emedicine.medscape.com/article/229461-treatment#d7
169.40 years old man presents with persistent lesion on the forearm that started 3 months ago. He
came from a deserted area that has sandfly infestation. Giemsa stain showed Donovan bodies inside
and outside monocytes. What will you use to treat him?
A. Miltefosine

Answer: A?
This patient has leishmaniasis.
Uncomplicated cutaneous leishmaniasis: Topical application of paromomycin or Intralesional antimony (Sodium stibogluconate, Meglumineantimoniate).
Complicated cutaneous leishmaniasis: Oral systemic therapy; include azoles and miltefosine
Agents with activity against visceral leishmaniasis (VL) include amphotericin B, pentavalent antimonial drugs, Paromomycin (a parenteral aminoglycoside), Miltefosine (the first oral drug for
treatment of VL)
Reference: http://www.uptodate.com/contents/treatment-of-cutaneous-leishmaniasishttp://www.uptodate.com/contents/treatment-of-cutaneous-leishmaniasishttp://www.uptodate.com/contents/treatment-of-cutaneous-leishmaniasis
Additional information ; Topical paromomycin has been shown to be effective against cutaneous leishmaniasis caused by L major ( in Iraq ) and L mexicana.
While Oral miltefosine is used for Treatment of New World cutaneous leishmaniasis.
http://emedicine.medscape.com/article/220298-treatment#d9http://emedicine.medscape.com/article/220298-treatment - d9http://emedicine.medscape.com/article/220298-treatment - d9
http://emedicine.medscape.com/article/220298-treatment - d9http://emedicine.medscape.com/article/220298-treatment - d9
170. Oral leukoplakia that cant be swiped off. Most likely to be?
A. Dysplasia
Answer: its Hyperplasia of squamous cells >> dysplasia >> Carcinoma in situ
http://emedicine.medscape.com/article/853864-overview#showall
171. cholera vibrio antibiotic:
A. Doxycycline
Answer: Doxycycline inhibits protein synthesis and, thus, bacterial growth by binding to 30S and possibly
50S ribosomal subunits of susceptible bacteria.

smle ,2016

40
In areas of known tetracycline resistance, therapeutic options include ciprofloxacin and erythromycin.
Strains resistant to ciprofloxacin have been reported from Calcutta, India.
Pharmacotherapy plays a secondary role in the management of cholera; fluid replacement is primary.
http://emedicine.medscape.com/article/962643-medication#2http://emedicine.medscape.com/article/962643-medication%232
http://emedicine.medscape.com/article/787591-overview%23a5http://emedicine.medscape.com/article/787591-overview%23a5
172. How is giardia diagnosed?
A. 3 different stool sample
Answer: A
Because Giardia cysts can be excreted intermittently, multiple stool collections (i.e., three stool specimens
collected on separate days) increase test sensitivity. Reference: Centers for Disease Control and Prevention
Stool examination may be performed on fresh specimens or after preservation with polyvinyl alcohol or
10% formalin (with appropriate staining). Ideally, 3 specimens from different days should be examined
because of potential variations in fecal excretion of cysts. G intestinalis is identified in 50-70% of patients
after a single stool examination and in more than 90% after 3 stool examinations
http://emedicine.medscape.com/article/176718-workup

smle ,2016

41

173. Lung disease causes clubbing


A. Bronchiectasis

smle ,2016

42
Answer: A
It usually begins in the thumb and index fingers and is most often associated with pulmonary or cardiovascular diseases, including lung cancer, interstitial pulmonary fibrosis, lung abscess, pulmonary tuberculosis, pulmonary lymphoma, congestive heart failure, infective endocarditis, and cyanotic congenital heart
disease . Less frequently, digital clubbing may occur in patients with extrathoracic disease, including inflammatory bowel disease, liver cirrhosis, and gastrointestinal neoplasms
Uptodate
Bronchiectasis:Findings on physical examination are nonspecific and may include the following:
Crackles, rhonchi, scattered wheezing, and inspiratory squeaks on auscultation
Digital clubbing (2-3% of patients; more frequent in moderate-to-severe cases)
Cyanosis and plethora with polycythemia from chronic hypoxia (rare)
Wasting and weight loss
Nasal polyps and signs of chronic sinusitis
Physical stigmata of corpulmonale, in advanced disease
http://emedicine.medscape.com/article/296961-overview

174. The most specific/ best diagnostic test for syphilis?


A. Fluorescent treponemal antibody absorption (FTA-ABS)
Answer: A
Nontreponemal testing is commonly used for serologic screening for syphilis, which
include VDRL, RPR, and TRUST. (These tests must be confirmed with specific
treponemal testing since false positive nontreponemal tests can occur. Nontreponemal
testing is performed initially due to its low cost.)
Specific treponemal tests include
FTA-ABS MHA-TP, TP-PA, and TP-EIA.
Reference:UpToDate
175. Patient with needle sensation after TB drug.
A. Isoniazid
Answer: A
IsoniazidAdverse Effects
>10%
Mild incr LFTs (10-20%)
Peripheral neuropathy (dose-related incidence, 10-20% incidence with 10 mg/kg/d),Loss of appetite
Nausea,Vomiting,Stomachpain,Weakness.
1-10%
Dizziness,Slurredspeech,Lethargy,Progressive liver damage (increases with age; 2.3% in pts > 50 yo)
Hyperreflexia
<1%
Agranulocytosis,Anemia,Megaloblasticanemia,Thrombocytopenia,Systemic lupus erythematosus,Seizure
http://reference.medscape.com/drug/isoniazid-342564#4
176. Schistosoma Antibiotics:
A. Praziquantel
Answer: A
Praziquantel.

smle ,2016

43
Add glucocorticoid if acute schistosomiasis or neurologic complications develop. Reference: Toronto
notes
177. patient with lymphadenopathy , splenomegaly and fever:
A. infectious mononucleosis
answer: A
infectious mononucleosis by EBV , early sign is a fever, lymphadenopathy. Later finding include hepatomegaly and splenomegaly. Reference: medscape
178. A woman with jaundice and high liver enzymes. Her husband has +veHep B surface antigen but
she does not have +ve markers for Hep A, B, C. What will you do now?
A. Check for anti Hep B core antibody (IgM)
Answer: A
Because it will become +ve early.
nterpretation of Hepatitis B Serologic Markers [5, 9]
HBsAg

AntiHBs

Anti-HBc
IgM

Anti-HBc
IgG

HBeAg

AntiHBe

HBV
DNA

Susceptible to infection

Immune due to natural


infection

-/+

-*

Immune due to vaccination

Incubation

Acutely infected

-/+

Chronically infected

-/+**

-/+

-/+

(-) is undetectable; (+) is detectable; (-/+) means may be detectable

*Nondetectable with non-PCR method

**May be positive in 10%-15% patients with reactivation of infection


http://emedicine.medscape.com/article/2109144-overview#a2
179. Postherpetic neuralgia treatment
A. Antiviral
Answer: TCA (Amitriptyline or Nortriptyline), Antiviral aims to shorten the clinical course, prevent complications, prevent the development of latency and/or subsequent recurrences, decrease transmission, and
eliminate established latency. (Medscape)

smle ,2016

44
Reference: Master the board

180. bacteria associated with ventilator- pneumonia ?


A. Pseudomonas
181. wet cutaneous leishmania transmitted by?
A. sand-fly
Answer: A
There are 3 main forms of leishmaniases visceral (also known as kala-azar and the most serious form of
the disease), cutaneous (the most common), and mucocutaneous. Leishmaniasis is caused by the protozoan Leishmania parasites which are transmitted by the bite of infected female phlebotomine sandflies
Reference: http://www.who.int/mediacentre/factsheets/fs375/en/
Also you can look Davidson's Practice & principles of Medicine ed. 22nd ; p362, Leshmaniasis under epidemiology and transmission subtitle.

182. monospot test +ve what is the diagnosis ?


A. infectious mononucleosis
Answer: A
The monosopt test will be + in epstein barr virus
https://www.nlm.nih.gov/medlineplus/ency/article/003454.htmhttps://www.nlm.nih.gov/medlineplus/ency/article/003454.htm
183. Which type of hepatitis have available vaccine
A. Hep B and A
Answer: A
Hepatitis A and B
184. Pt traveled, came back with large watery diarrhea, dx:
A Giardasis???
Answer: A
Travelers diarrhea but there a lot of causes that cause watery (E.coli,salmonla,giardasis)

smle ,2016

45
Reference: first aid
185. I don't remember the scenario but in culture : Methicillin sensitive , gram+
Answer: MRSA
186. cocci in cluster , what's the Abx?
A.Cloxacillin
the organism is staph arus so best ttt is penicillin but resistance common to pencillin so better start
penicillinase resistant beta lactam abs
reference: master the board
187. Male with Painless genital ulcer, What you will order ?
A.darkfield microscopy
Answer: A
REFERNCE: MERCK MANULE
188. Patient not know to be HIV came with lab result that confirm that he has HIV CD count less that
200 and WBC count he developed fever and cough what is the organism which is gram positive:
A. actinomycosisisraelii (that's what I chose)
Answer:
I don't remember the other but no pneumocytes in the choices
189.Trip overseas & got bloody diarrhea
A. amoebic dysentery
Answer:
190. Foul smelling diarrhea, bloating, swelling of abdomen
A. Colistrudium ?
Answer:
191. A scenario about a patient who deals with flowers and got pricked by rose thorn,presented with
redness at the prick site. which of the following is the cause:
A. fungus Sporothrix schenckii .
Answer: A
Explanation :Diagnosis is sporotrichosis ttt by itraconazole
192. A man came from india with diarrhea and then developed rash:
A. post kala azar dermal leishmaniasis
Answer: A ?
193.OM by strep pyogenes, on antibiotics amoxicillin, add what/?
A. Clavulanic acid
Answer:
Medscape: Amoxicillin is the antibiotic of choice unless the child received it within 30 days, has concurrent
purulent conjunctivitis, or is allergic to penicillin; in these cases, clinicians should prescribe an antibiotic
with additional beta-lactamase coverage The recommendations support the use of amoxicillin as the firstline antimicrobial agent of choice in patients with AOM. The group recommended increasing the dose
used for empiric treatment from 40-45 mg/kg/day to 80-90 mg/kg/day because of concerns about increasingly resistant strains of S pneumoniae, which are theoretically susceptible to this higher dose ..The

smle ,2016

46
recommendations for second-line therapy were more controversial, despite their reasonableness from a
scientific viewpoint. Stressing the importance of documenting true clinical failure of therapy after at least
3 days of treatment with high-dose amoxicillin, the working group suggests tympanocentesis for identification and susceptibility testing of the etiologic bacteria to guide alternate antibiotic therapy. So depends
on the scenario, and the duration to keep patient on amoxicillin for longer time or to give augmentin
194. A patient had repeated attacks of reddish rash and plaques in mouth. What is the diagnosis?
A. Candidiasis
Answer:
Pseudomembranous candidiasis. The typical adherent white plaques may be removed by wiping firmly
with a tongue blade or gauze.
195. Fever, diarrhea bloody and stool microscopic showed ( pic ) Based on the organism above which
drug supposed to be given ?
A. Metronidazole

Answer
196. HIV patient has velvety skin rash:
A. Kaposi sarcoma
answer :
197. Red back of the hand with previous prick sensation and fever, what is the dx?
A. Cellulitis
Answer:
198. Antiviral inhaled how it work
A. Neuraminidase inhibitor
Reference: http://www.comed.uobaghdad.edu.iq/uploads/lectures/lippincott%2030.pdf
199. Patient has traveled then got inflammation and took amoxicillin, he developed lymphadenopathy and skin rash, what is the proper investigation for this patient
A. EBVmonospot test
B. Brucellosis
answer: EBV infectious mononucleosis.
200. Positive PPD skin test for adult man without any sign of TB infection what you will be do ?
A. Chest X ray if negative INH for 9 m
Explanation:Everyone with a reactive PPD test should have a chest x-ray to
exclude active disease.After active tuberculosis has been excluded with a chest x-ray, patients should
receive 9
months of isoniazid.
201. Case of giardiasis,
A. in biopsy sickle shape parasite
Reference: http://www.medscape.com/viewarticle/442755_5

smle ,2016

47

202. patient taking carbamazepine developed generalized rash and peeling of epidermis:
A. Steven Johnson syndrome
Answer: A
http://emedicine.medscape.com/article/1197450-overview#a5http://emedicine.medscape.com/article/1197450-overview - a5
203.patient get beta lactamase organism , on amoxicillin , what to add :
A. clavulanic acid
answer
a
reference: https://en.wikipedia.org/wiki/Beta-lactamase

204. Elderly with back bone pain, bone biopsy positive acid fast bacilli ,the organism?
A. Mycobacterium tuberculosis
Answer: A
205. Where do latent herpes virus stay?
A. Macrophages.
Answer : sensory neuron ?
206.treatment of enteric fever?
A. Ciprofloxacin
207. Infectious Mononucleosis test?
A. Monospot.
Answer: A
208. A patient had repeated attacks of reddish rash and plaques in mouth. What is the diagnosis?
A. candidiasis?
Answer:
Pseudomembranous candidiasis. The typical adherent white plaques may be removed by wiping firmly
with a tongue blade or gauze.
209. Characteristic features of enteric fever?
A. Nausea.
B. Vomiting.
c. fever
Answer: c
210. Young patient with recurrent bacterial infections ( I believe encapsulated organisms ), history
of arthritis treated with IVIG , lymph node biopsy ( central ,, depletion ) In investigation ) Immunoglobulin within normal range , low lymphocytes on CBC ) >>> what is the disease ( not the Dx the pathophysiology of the disease
A. adenosine ADA deficiency ( SCID )
Reference: essential nalsone

smle ,2016

48
211. Red back of the hand with previous prick sensation and fever, what is the dx?
A. Cellulitis
Answer:
212. Signs and symptoms of 1st syphilis?
A. Painless genital ulcer
Answer: A
Reference:http://www.cdc.gov/std/syphilis/stdfact-syphilis-detailed.htm http://www.cdc.gov/std/syphilis/stdfact-syphilis-detailed.htm
213. lung infection:
A. Iv ceftriaxone
answer: CAP treatment guidelines 2007:
Setting

Circumstances

Treatment

Outpatient

Previously well, no antibiotics Macrolide ( azithro, clarithro,


use in last 3 months
erythromycin) OR doxycycline
Comormbidities, Antibotics use Flouroquinolones (levofloxacin,
in last 3 mo ( use different class ) moxifloxacin) OR B-lactams (eg.
Ceftriaxone)+ Macrolide

Inpatient

Ward

Flouroquinolones OR B-lactam+
macrolide

ICU

B-lactam+ (flouroquinolones or
macrolide )

Source: Toronto Notes 2015 page ID8; table 10


For further reading in hospital acquired pneumonia look Toronto Notes 2015 page ID8; table 11
treatment of CAP
Inpatients, non-ICU treatment
A respiratory fluoroquinolone (moxifloxacin, gemifloxacin, or levofloxacin [750 mg])
Refhttp://www.uptodate.com/contents/image?imageKey=ID%2F71665&topicKey=ID%2F7027&source=see_linkhttp://www.uptodate.com/contents/image?imageKey=ID%25252F71665&topicKey=ID%25252F7027&source=see_link
214. patient with cough .and vomiting after cough fever for 3 wk
A. Pertussis vaccine
Answer: The best way to prevent pertussis (whooping cough) among babies, children, teens, and adults is
to get vaccinated.
Source: http://www.cdc.gov/pertussis/about/prevention/index.html

smle ,2016

49
215. meningitis case fever, headache ,nuchal rigidity ,And rash(pic) what is the most complication?
A. deafness
Answer: ?
Meningitis complications:
Disseminated intravascular coagulation (DIC; blood-clotting disorder)

Encephalitis.

Persistent fever.

Seizures.

Syndrome of inappropriate antidiuretic hormone (SIADH; causes fluid build-up)

Long-term complications:
Behavioral and personality changes

Vision loss (partial or total)

Cerebral palsy

Hearing loss (partial or total)

Learning disabilities or mental retardation

Paralysis (partial or total)

Speech loss (partial or total)

Reference : http://www.healthcommunities.com/meningitis/complications.shtml
216.

Sore throat, gingivitis, papule pustule with erythematous base what is Dx. May be HSV?

It could be more like Strep pyogenes and could be HSV, it depends on age as strep pyogenes is common
in children ( 3-12yrs according to Toronto notes 2015) but HSV common in all ages and usually accompanied by other URTI symptoms like runny nose, cough conjunctivitis.
References: http://emedicine.medscape.com/article/228936-clinical
http://www.webmd.com/genital-herpes/pain-management-herpes
(Old Answer)
Viral infections The most common viral infections are herpes simplex virus type1(HSV-1) and 2 (HSV-2)
and varicella-zoster virus. HSV is the most common viral infection of the oral/facial area. It has two subtypes: type 1, which affects the oral cavity; and type 2, which affects the genitals. Primary herpetic gingivostomatitis is most commonly observed in children from 7 months to 4 years of age but can also be found
in adolescents or young adults. Children are often infected with HSV by their own parents if these have
recurrent herpes lesions. The primary infection may be asymptomatic but can manifest as severe gingivostomatitis, in which the gingiva are painful, inflamed and ulcerated. Fever and lymphadenopathy are classic
features and affected individuals experience difficulty in chewing. The incubation period of the virus is 1
week, and healing occurs after approximately 10 to 14 days.. Recurrent herpes infections can be found
intra- and extra-orally. Intraoral herpes infection manifests as a group of painful ulcerations involving the
gingiva and hard palate. The infection can be clinically diagnosed and confirmed by isolating the virus.
Varicella-zoster virus causes chickenpox, primarily in children, and later reactivation of the virus in adults
causes herpes zoster (shingles). Both can involve the gingiva, presenting as vesicle lesions that burst leaving fibrin-covered lesions. This infection is readily diagnosed from the intense associated pain and unilateral lesions, which generally heal after 1-2 weeks.

smle ,2016

50
http://cdn.intechopen.com/pdfs/20292/InTech-Etiology_of_gingivitis.pdfhttp://cdn.intechopen.com/pdfs/20292/InTech-Etiology_of_gingivitis.pdf
217. What is the treatment of choice for Kawasaki?
Answer:
Aspirin (5mg/kg for 14 days) and IVIg (400mg/kg for 4 days).
Reference: CanadaQbank, Davidson's practice & principles of medicine 22nd ed. P1116 under Kawasaki
title.
218. which of the following antibiotics is DNA gyrase and it works on what organism?
Answer: fluoroquinolone = pseudomonas
Fluoroquinones represent an important class of antimicrobial which work through inhibition of DNA gyrase.
Reference: http://www.pharmacology2000.com/Antibacterial/dnagyr1.htm
219. Splenectomy case, what vaccines should be given afterwards or prior to spleen removal?
Answer: Spleen acts as a Macrophage to Encapsulated organisms, of those organisms the ones we should
vaccinate with are as follows:
pneumococcal vaccine
meningococcal vaccine
Hemophilus influenza vaccine
Reference: Toronto Notes 2015 page GS53 under splenectomy; long-term complications.
220. Rheumatic fever acute management?
Answer: 1- Oral penicillin 2- Amoxicillin 3- Intramuscular penicillin, single dose 4- Cephalexin 5- Aspirin
http://www.uptodate.com/contents/treatment-and-prevention-of-acute-rheumatic-fever?source=search_result&search=rheumatic+fever&selectedTitle=2~111#H2
220. Infective endocarditis (migratory arthritis)
Answer: Duke criteria for diagnosis of infective endocarditis
Major: at least 2 separate + blood culture for a typical organism , Evidence of endocardial vegetations (
via echo or new murmur )
Minor ( fever>38.5 , vascular and immunological phenomena as septic emboli , janeway lesion , osler's
nodes , roth spot
The presence of 2 major or 1 major + 3 minor or 5 minor, diagnosis endocarditis
Ref: First Aid USMLE STEP 2 CK
221. meningitis finding in CSF due to bacteria
Answer:
Appearance: Clear, cloudy, or purulent
Opening pressure: Elevated (>25 cm H 2 O)
WBC count: >100 cells/L (>90% PMN)
Glucose level: Low (< 40% of serum glucose)
Protein level: Elevated (>50 mg/dL)
Reference : http://emedicine.medscape.com/article/2172226-overview
Also you can look Toronto Notes 2015 page P62.
222. A patient presented with macular papular rashand fever. (case of rubella)

smle ,2016

51
Answer:
Clinical Features: Rash (pink, maculopapular rash 1-5 d after start of symptoms. The rash starts on face
and spreads to neck and trunk), Prodrome of low grade fever and
occipital/retroauricular nodes.STAR complex (sore throat, arthritis, rash) and Positive serology for rubella
IgM.
Management:
For infected patients: Symptomatic Rx.
For Prevention: MMR vaccine.
For rubella-exposed pregnant women: Serologic testing,
Reference: Toronto Notes
223.
Q about patient having some infection and having allergy from Penicillin. What other medications will you give?
Answer: Tetracyclines (e.g. doxycycline), quinolones (e.g. ciprofloxacin), macrolides (e.g. clarithromycin),
aminoglycosides (e.g. gentamicin) and glycopeptides (e.g. vancomycin) are all unrelated to penicillins and
are safe to use in the penicillin allergic patient. B-lactams antibiotics should be avoided including cephalosporins.
Reference: http://www.nhstaysideadtc.scot.nhs.uk/Antibiotic%20site/penhypers.htm
224. parasite infection in under cooked food
Answer: Trichinellosis/Trichinosis
Infection is initiated by ingestion of viable larvae in raw or undercooked meat. Digestive action liberates
the
larvae.
http://emedicine.medscape.com/article/787591-overview#a5http://emedicine.medscape.com/article/787591-overview - a5
225. Ebola Question.
Answer: http://www.who.int/mediacentre/factsheets/fs103/en/http://www.who.int/mediacentre/factsheets/fs103/en/
226. What is the anti-influenza medication that is given intranasal?
Answer:
Live Attenuated Influenza Vaccine (Nasal spray flu vaccine) provide protection against four flu viruses: an
influenza A (H1N1) virus, an influenza A (H3N2) virus and two influenza B viruses. Reference:
http://www.cdc.gov/flu/about/qa/nasalspray.htm
227. Infection in venous lines ? With needle insertion. (Something like that.)
Answer: For the purpose of this guideline,
short-term catheters are defined as those devices that are in
situ for less than 14 days.
Most CRBSIs emanate from the insertion site, hub, or both
[9]. For long-term cathetersparticularly tunneled catheters
the catheter hub is a prominent source of microbes causing
bloodstream infection [10]. In order of prevalence, the 4 groups
of microbes that most commonly cause CRBSI associated with
percutaneously inserted, noncuffed catheters are as follows: coagulasenegative staphylococci, S. aureus, Candida species, and
enteric gram-negative bacilli. For surgically implanted catheters and peripherally inserted CVCs, they are
coagulase-negative staphylococci, enteric gram-negative bacilli, S. aureus, and P. aeruginosa [8].

smle ,2016

52
Management: Vancomycin is recommended for empirical therapy in
heath care settings with an increased prevalence of methicillinresistant
staphylococci; for institutions with a preponderance
of MRSA isolates that have vancomycin MIC values 12 mg/mL,
alternative agents, such as daptomycin, should be used.
Empirical coverage for gram-negative bacilli should be
based on local antimicrobial susceptibility data and the severity
of disease (e.g., a fourth-generation cephalosporin, carbapenem,
or b-lactam/b-lactamase combination, with or without
an aminoglycoside).
For empirical treatment of suspected catheter-related
candidemia, use an echinocandin or, for selected patients, fluconazole
(A-II). Fluconazole can be used for patients without
azole exposure in the previous 3 months and in health care
settings where the risk of C. krusei or C. glabrata infection is
very low.
Reference:http://www.idsociety.org/uploadedFiles/IDSA/Guidelines-Patient_Care/PDF_Library/Management%20IV%20Cath.pdf
For further reading click on the link ( updated American guidelines 2009)
228. Bacteria sexual like behavior?
Conjunction
Answer: Bacterial conjugation.
http://www.who.int/mediacentre/factsheets/fs366/en/
http://www.who.int/mediacentre/factsheets/fs366/en/
Also look Lippincott's illustrated reviews of microbiology 2nd ed. Page 61 under gene transfer title.
229. CGD is a primary immunodeficiency that affects phagocytes of the innate immune system and
leads to recurrent or persistent intracellular bacterial and fungal infections
Most infections in CGD are caused by Staphylococcus aureus.
Ref ;http://emedicine.medscape.com/article/1116022-clinical#showall
230. Vancomycin plus a third-generation cephalosporin Ceftriaxone or cefotaxime.
Ref
:http://www.uptodate.com/contents/image?imageKey=ID%2F71968~ID%2F60707&topicKey=ID%2F1290&source=see_linkhttp://www.uptodate.com/contents/image?imageKey=ID%25252F71968~ID%25252F60707&topicKey=ID%25252F1290&source=see_link
According to age

smle ,2016

53

231.Watery discharge no itching ?


Answer: Watery viral conjunctivitis, allergic conjunctivitis, eye allergies, dry eyes, eye injury, dacryocystitis
Toronto: VIRAL CONJUNCTIVITIS serous discharge, lid edema, follicles, may be associated with rhinorrhea

pre auricular node often palpable and tender

initially unilateral, often progresses to the other eye

mainly due to adenovirus Dacryocystitis

pain, swelling, redness over lacrimal sac at medial canthus

epiphora, crusting, fever digital pressure on the lacrimal sac may extrude pus through the punctum

in the chronic form, epiphora may be the only symptom


232. hepatitis A infection diagnosis (easy)
Hepatitis A :http://www.who.int/mediacentre/factsheets/fs328/en/
Serologic markers for the different phases of acute and chronic hepatitis B virus infection
http://www.uptodate.com/contents/image?imageKey=GAST%2F60627~GAST%2F60827&topicKey=GAST%2F3680&rank=1~150&source=see_link&sear
ch=Hepatitis+Bhttp://www.uptodate.com/contents/image?imageKey=GAST%25252F60627~GAST%25252F60827&topicKey=GAST%25252F3680&rank=1~150&source
=see_link&search=Hepatitis+B
ref : UPTODATE
233. C.difficile diagnosis (easy)
Reference: http://emedicine.medscape.com/article/186458-overview
234. Clinical manifestations of Clostridium difficile infection
http://www.uptodate.com/contents/clostridium-difficile-infection-in-adults-clinical-manifestations-anddiagnosis http://www.uptodate.com/contents/clostridium-difficile-infection-in-adults-clinical-manifestations-and-diagnosis

smle ,2016

54
235.
Laboratory diagnosis of C. difficile infection requires demonstration of C. difficile toxin(s) or detection of toxigenic C. difficile organism(s) [30]. We favor use of PCR for diagnosis of C. difficile on a single
stool sample
Ref: http://www.uptodate.com/contents/clostridium-difficile-infection-in-adults-clinical-manifestationsand-diagnosis#H15443551 http://www.uptodate.com/contents/clostridium-difficile-infection-in-adultsclinical-manifestations-and-diagnosis - H15443551
236.Case of tonsillitis <<< antibiotic
Answer: GAS hx( 20%, peak age =3-12yrs ): moderate to severe sore throat, fever, malaise and headache, nausea and vomiting, abdominal pain, other URTI symptoms usually absent. In addition, on P/E
feverish. Viral (80%, all ages): hx of often mild sore throat, conjunctivitis, watery rhinorrhea, hoarseness, diarrhea with flu-like symptoms.
Management: If GAS then antibiotics against sterp i.e. Amoxicillin or erythromycin if allergic to penicillin.
Supportive: hydration and painkillers for the fever and sore throat.
Prophylactic tonsillectomy if recurrent.
In viral sore throat no need for antibiotics only supportive.
237. 2 Q about meningitis how to interpretation of types of meningitis and how to treat children with
meningitis?
Answer: for CSF findings in different types of meningitis look table 24 in Toronto Notes 2015 on page P62.
Management: it depends on the pathogen,,
If the age is 0-28d then most common pathogens are GBS, E.coli, Listeria or other gram ve so we give
Amoxicillin+ cefotaxime.
If the age is 28-90d; pathogens overlap between that seen in neonates and older children, so we give
Amoxicillin+ cefotaxime +/- vancomycin.
In those older than 90d; most common bacterial pathogen is S.pneumoniae & N. meningitidis.. we give
cefotaxime +/- vancomycin.
In viral meningitis management is mainly supportive unless HSV we give acyclovir.
Reference: Toronto Notes 2015 page P62.
source:http://emedicine.medscape.com/article/961497treatment#d10
http://emedicine.medscape.com/article/961497-treatment - d10
238. patient come from Africa , I forgot the symptoms :/ , what to do :
Answer: blood culture b.stool culture
Answer: ?
239.Motile, urease and oxidase negative what is the best treatment?
Answer :
salmonela
Salmonella gastroenteritis is usually a self-limiting disease. Fluid and electrolyte replacement may be
indicated in severe cases.
http://emedicine.medscape.com/article/228174-treatment

240.Side effect of the medication used to treat infection ( you saw Donovan body )?
Answer :
ACTUALLY DONOVAN BODIES seen in 2 diseases one of them is visceral leishmaniasis

smle ,2016

55
amphotericin B - side effect Fever, shaking, chills, flushing, loss of appetite, dizziness, nausea, vomiting,
headache, shortness of breath, or fast breathing may occur 1 to 3 hours after the infusion is started.
http://www.webmd.com/drugs/2/drug-13565/amphotericin-b-injection/details#side-effects
Miltefosine - side effect vomiting,nausea,diarrhea,decreased appetite,dizziness,motion sickness,headache,weakness,stomach or abdominal pain,general feeling of being unwell (malaise),fever,drowsiness,etching, and testicular pain
http://www.rxlist.com/impavido-side-effects-drug-center.htm
recommended for visceral leishmaniasis:

Liposomal amphotericin B alone, given as a single dose (currently recommended as the


drug of choice by the Kala-Azar elimination programme of India)

Liposomal amphotericin B in a single dose, in combination with 7 days of oral miltefosine or 10 days of paromomycin

Miltefosine plus paromomycin for 10 days

Amphotericin B deoxycholate: 0.75-1 mg/kg/day via infusion, daily or on alternate days


for 15-20 doses

Miltefosine orally for 28 days or paromomycin intramuscularly for 28 days

Pentavalent antimonials: 20 mg Sb5+/kg/day intramuscularly or intravenously for 30


days in areas where they remain effective: Bangladesh, Nepal, and the Indian states of Jharkhand, West
Bengal, and Uttar Pradesh
http://emedicine.medscape.com/article/220298-treatment#d11
other disease granuloma inguinal
treatment could cause Hepatotoxisity
and the treatment plan Recommended dosing is as follows:

Azithromycin 1 g PO once a week or 500 mg/day for at least 3 weeks or until all lesions
have completely healed have side effect hepatotoxitiy

Doxycycline 100 mg PO twice a day for at least 3 weeks or until all lesions have completely healed should be avoided in pregnancy, especially after 15 weeks gestation because of the risk
of maternal hepatitis and brown discoloration of the infants deciduous teeth and inhibition of bone
growth. It should also avoided in patients who are breastfeeding, since prolonged exposure beyond 3
weeks may also cause decreased bone growth

Erythromycin base 500 mg PO 4 times a day for at least 3 weeks or until all lesions have
completely healed have side effect hepatotoxitiy

Ciprofloxacin 750 mg PO twice a day for at least 3 weeks or until all lesions have completely healed associated with damage to fetal cartilage. When given to breastfeeding women, observe
for diarrhea in the infant since it can cause pseudomembranous colitis.

Trimethoprim-sulfamethoxazole 1 double strength (160 mg/800 mg) tablet PO twice a


day for at least 3 weeks or until all lesions have completely healedis a risk of cardiovascular defects
when given in the first trimester, and it is associated with preterm delivery, low birth rate, and miscarriage. Sulfonamides are also associated with serious kernicterus in patients with G-6-P deficiency, and,
when given in the third trimester of pregnancy, there is a higher risk of neonatal hyperbilirubinemia.
241.Typical presentation of coxsackie B virus?
Answer :
They are divided into groups A and B:

Coxsackievirus A:

smle ,2016

56

Usually affects skin and mucous membranes.

Causes herpangina and HFMD.

There are a number of different viruses within the group. The most common causes of
HFMD are Coxsackievirus A16 (CA16) along with the closely related enterovirus 71 (EV71).

Coxsackievirus B:

Usually affects the heart, lungs, pancreas and liver.

Causes Bornholm disease, hepatitis, myocarditis and pericarditis.


Coxsackieviruses of both types are a leading cause of aseptic meningitis. They may also cause nonspecific febrile and upper respiratory tract illnesses.
http://patient.info/doctor/coxsackievirus-infection
And More than 90% of coxsackieviruses infections are asymptomatic or cause nonspecific febrile illnesses
http://emedicine.medscape.com/article/215241-clinical
242.Taenia saginata
http://emedicine.medscape.com/article/999727-clinical
243.Derma description of a lesion that is loose and easy to fall defining of ( scales, other skin lesions) ?
Ehlers-Danlos Syndrome ???http://www.medicinenet.com/ehlers-danlos_syndrome/article.htm
244.Treatment of herpes zoster
Management
Episodes of herpes zoster are generally self-limited and resolve without intervention; they tend to be
more benign and mild in children than in adults.
Conservative therapy includes the following:

Nonsteroidal anti-inflammatory drugs (NSAIDs)

Wet dressings with 5% aluminum acetate (Burrow solution), applied for 30-60 minutes
4-6 times daily

Lotions (eg, calamine)


Primary medications for acute zosterassociated pain include the following:

Narcotic and nonnarcotic analgesics (both systemic and topical)

Neuroactive agents (eg, tricyclic antidepressants [TCAs])

Anticonvulsant agents
Steroid treatment for herpes zoster is traditional but controversial. Typically, a substantial dose (eg, 4060 mg of oral prednisone every morning) typically is administered as early as possible in the course of
the disease and is continued for 1 week, followed by a rapid taper over 1-2 weeks.
Antiviral therapy for herpes zoster may decrease the length of time for new vesicle formation, the number of days to attain complete crusting, and the days of acute discomfort. Usually, the earlier antiviral
medications are started, the more effective they are in shortening the duration of zoster and in preventing or decreasing the severity of PHN. Ideally, therapy should be initiated within 72 hours of symptom
onset.
Oral treatment with the following has been found beneficial:

Acyclovir

Famciclovir

Valacyclovir
Hospital admission should be considered for patients with any of the following:

Severe symptoms

Immunosuppression

smle ,2016

57

Atypical presentations (eg, myelitis)

Involvement of more than 2 dermatomes

Significant facial bacterial superinfection

Disseminated herpes zoster

Ophthalmic involvement

Meningoencephalopathic involvement
Prevention and treatment of postherpetic neuralgia
Prompt treatment of acute zoster and its associated pain (eg, with antiviral therapy) can prevent the development of PHN. Once PHN has developed, various treatments are available, including the following:

Neuroactive agents (eg, TCAs) [6]

Anticonvulsant agents (eg, gabapentin, [7] pregabalin)

Narcotic and nonnarcotic analgesics, both systemic (eg, opioids) and topical
http://emedicine.medscape.com/article/1132465-overview
245.Patient given yellow fever vaccine, started vomiting and SOB.. Cant remember the rest of Q
http://www.rxlist.com/yellow-fever-vaccine-drug/side-effects-interactions.htm
246.Typical presentation of coxsackie B virus
repeated Q same as Q number 244
247.What the organism for someone sky with diver ?
?????
248. GE salmonella which AB274- Another same Q what is the mechanism ??
After ingestion, infection with salmonellae is characterized by attachment of the bacteria by fimbriae or
pili to cells lining the intestinal lumen. Salmonellae selectively attach to specialized epithelial cells (M
cells) of the Peyer patches. The bacteria are then internalized by receptor-mediated endocytosis and
transported within phagosomes to the lamina propria, where they are released. Once there, salmonellae induce an influx of macrophages (typhoidal strains) or neutrophils (nontyphoidal strains).
http://emedicine.medscape.com/article/228174-overview#a5
249.Treatment of type of bacteria " fragilis
http://emedicine.medscape.com/article/233339-treatment
250.Cumulative ?
251.Question about Herpes diagnosis
herpes zoster
http://emedicine.medscape.com/article/1132465-workup
herpes simplex
http://emedicine.medscape.com/article/218580-workup
252.Question about HPV infectious mono
http://emedicine.medscape.com/article/219110-overview
253.Gram + cocci what the organism

smle ,2016

58
could be any one of them :
Enterococcus
Streptococcus
Pneumococcus
Staphylococcus
https://www.uptodate.com/contents/table-of-contents/infectious-diseases/gram-positive-cocci
254.male pt has diarrhoea after he back from Indonesia and ate from their foods,What is the organism?
travelars diarrhoea
The most common culprit is a bacteria called E.coli.
http://www.webmd.com/digestive-disorders/travelers-diarrhea
255.After 2 weeks msn come gimrom sudan gmhe developed abdominal pain, lethargy?
256.male after traveling present with abdominal pain, dysentery and bloody diarrhea causative organism?
amebic infection
http://www.medicalnewstoday.com/articles/171193.php
257.patient in college diagnosed with meningitis proved and she receiving treatment How to prevent
the spread to the roommate ?
Preventive treatment with antibiotics is sometimes recommended for the close contacts of an individual
who is ill with meningococcal
http://medical-dictionary.thefreedictionary.com/Menegitus
258.true fever vs Factitious fever :
Fever produced artificially by a patient. This is done by artificially heating the thermometer or by selfadministered pyrogenic substances. An artificial fever may be suspected if the pulse rate is much less
than expected for the degree of fever noted.
http://medical-dictionary.thefreedictionary.com/factitious+fever
259.How can differentiate factitious fever?
same answer as Q 262
260.Whose lactose fermenting oxidase neg green ..etc and whose not ( many questions for case the
result of the culture described so you have to choose which one is the bacteria
Non-Lactose Fermenters:
"If you dont lac-toes you can work as a Sneaky SPY"
Salmonella
Shigella
Proteus
Yersinia
Fast Lactose Fermenters:
"If you lac-toes at least you have a KnEE"
Klebsiella Pneu

smle ,2016

59
Ecoli
Enterobacter Cloacae
Slow Lactose Fermenters:
"Lac-toes or not, one should move slow in a Serene City"
Serratia marcescens
Citrobacter
http://www.usmle-forums.com/usmle-step-1-mnemonics/46676-lactose-fermentation-bacteria-mnemonic.html
261.Patient with infection , culture and sensitivity show Methicillin sensitive organism , which antibiotic
you will select :
A. Piperacillin
B. Oxacillin
Answer :
pipercillin
http://www.medscape.com/viewarticle/587892
262.Pt with antibodies of toxoplasma ( IgM ) .. To confirm diagnosis ?
The diagnosis of toxoplasmosis is confirmed with the demonstration of T gondii organisms in blood,
body fluids, or tissue. T gondii may be isolated from the blood via either inoculation of human cell lines
or mouse inoculation.
http://emedicine.medscape.com/article/229969-workup

263.side effect of vancomycin


>10%
Bitter taste (PO)
Erythematous rash on face and upper body (IV; red neck or red man syndrome; related to infusion rate)
Hypotension accompanied by flushing (IV)
Nausea and vomiting (PO)
1-10%
Chills (IV)
Drug fever (IV)
Eosinophilia (IV)
Rash (IV)
Fatique (PO)
Peripheral edema (PO)
Urinary tract infection (PO)
Back pain (PO)
Headache (PO)
Reversible neutropenia (IV)
Phlebitis (IV)
<1%
Nephrotoxicity
Ototoxicity (especially with large doses)
Stevens-Johnson syndrome

smle ,2016

60
Thrombocytopenia
Vasculitis
http://reference.medscape.com/drug/vancocin-vancomycin-342573#4
264.intranasal influenza vaccine ----> the name of the vaccine
Live Attenuated Influenza Vaccine [LAIV] (The Nasal Spray Flu Vaccine)
http://www.cdc.gov/flu/about/qa/nasalspray.htm
265. Q. Painless genital ulcer?.
could be :
Syphilis (primary)
Lymphogranuloma venereum
Granuloma inguinale (donovanosis)
depend on scenario and choices
http://www.aafp.org/afp/2012/0201/p254.html
266. Young boy with pain in his knee, aspiration of fluid reveal yellowish and turbid appearance, Diagnosis?
Answer :
most properly joint infection depend on the age and fluid
joint aspiration types:

Fracture or ligament injuries may show blood in the fluid.

Meniscus injury or osteoarthritis may show straw colored or pale yellow fluid.

Presence of crystals may mean gout or pseudogout.

Turbid fluid or presence of pus may mean there is an infection.

Gram staining and culture tests of the fluid can help in detecting the microorganism
causing infection.
http://www.epainassist.com/joint-pain/knee-pain/knee-aspiration
267. N. Meningitides meningococcemia, prophylaxis for family:
give prophylaxis vaccination dose
http://emedicine.medscape.com/article/1165557-treatment#d11
268- HIV attacks?
A. B cells
B. T cytotoxic cells
C. Macrophages

smle ,2016

61

ALLERGY AND IMMUNOLOGY


1. A patient with signs and symptoms of Atopy. Which cell produce these mediators?
A. Mast cells!
B. Nk cells
C. Macrophage
D. B cells
Answer: A!
Reference: https://www.uptodate.com/contents/clinical-manifestations-of-food-allergy-an-overview?source=search_result&search=allergy&selectedTitle=2~150

2. Patient got rapid swelling response after a bee sting what type of hypersensitivity?
A. 1
B. 2
C. 3
D. 4
Answer: A
Reference: https://www.uptodate.com/contents/drug-allergy-classification-and-clinical-features?source=machineLearning&search=type+i+hypersensitivity&selectedTitle=1~150&sectionRank=1&anchor=H13#H13

smle ,2016

62

Pathology is similar to other immunoglobulin E (IgE)mediated allergic reactions.


http://emedicine.medscape.com/article/768764-overview#a5 http://emedicine.medscape.com/article/768764-overview - a5
3. Bee sting since 18 hrs. With swelling and redness, what will you do?
A. Antihistamines
B. Steroids
C. Observe
Answer: A
Reference: Toronto Notes 2016 (32E), Page: 229 (ER48)
Local reactions can be life threatening if swelling occludes the airway. Initiate invasive measures to secure the airway if this occurs. Otherwise, the following local care measures suffice:

Provide supplemental oxygen

Diphenhydramine limits the size of the local reaction.

Clean the wound and remove the stinger if present.

Apply ice or cool packs.

Elevate the extremity to limit edema.


*Treatment should include an initial intravenous (IV) bolus of 10-20 mL/kg isotonic crystalloids in addition to diphenhydramine and epinephrine.
*If the patient has not removed the stinger, it should be removed as soon as possible by the first caregiver on the scene. Delay increases venom load, so the fastest removal technique is the best. Pinching
and traction is an acceptable technique.
Reference: http://emedicine.medscape.com/article/768764-treatmenthttp://emedicine.medscape.com/article/768764-treatment
4. man got a bee sting then his wife trying look for the epinephrine what it
gonna inhibit?
A. leukotriene release from macrophages
B. cross reactivity with the cardiac..
C. inhibit immunocomplex formation
D. Widespread histamine release.
Answer: D, here in the link below the same Q (Q10)but there is extra choice and it is the Right Answer:
https://quizlet.com/8362963/immuno-block-4-practice-exam-flash-cards/
(Inhibit histamine and tryptase, missed option)

smle ,2016

63
Epinephrine maintains blood pressure(by agonist alpha and beta-adrenergic receptors), antagonizes the
effects of the released mediators, and inhibits further release of mediators.
Reference: http://emedicine.medscape.com/article/135065-treatment#d9
5. Itching only & CASE OF LADY WITH COMPLAINT OF pursuits ..physical examination not towards
specific infection What to do ?
A. Referral to sexual transmitted disease
B. Re-evaluate if symptomatic with inflammatory manifestation?
C. She is okay not need to come
D. Give ttt for all possible infections
Answer:?
6. Case is allergic to sulfa , shellfish , penicillin what u will give him ?
A. Nitrofurantoin
B. Tm/smz
C. Penicillin
D. Amoxicillin
Answer: A
7. Treatment of pyodermagangrenosum
A. Methotrexate.
B. Antibiotics.
C. Systemic Steroids.
Answer: C
Reference: http://www.uptodate.com/contents/pyoderma-gangrenosum-treatment-and-prognosis?source=search_result&search=pyoderma+gangrenosum&selectedTitle=2~68#H2054609

No specific therapy is uniformly effective for patients with pyodermagangrenosum. In patients with an
associated, underlying disease, effective therapy for the associated condition may be linked to a control
of the cutaneous process as well.
Topical therapies include gentle local wound care and dressings, superpotent topical corticosteroids,[17]
cromolyn sodium 2% solution, nitrogen mustard, and 5-aminosalicylic acid.
Systemic therapies include corticosteroids, cyclosporine,[19, 20, 21] mycophenolatemofetil,[22, 23, 24]
azathioprine,[25] dapsone
REF : MEDSCAPE http://emedicine.medscape.com/article/1123821-treatment http://emedicine.medscape.com/article/1123821-treatment

smle ,2016

64

8. Pyodermagangrenosum is associated with which one of these diseases?


A. Ulcerative colitis.
Reference: https://www.uptodate.com/contents/pyoderma-gangrenosum-pathogenesis-clinical-features-and-diagnosis?source=search_result&search=Pyoderma+gangrenosum&selectedTitle=1~68

Answer: A
PG is more common in UC (512%) than CD (12%).
Reference:http://www.ncbi.nlm.nih.gov/pmc/articles/PMC327372/table/T1/
9. What is the most common pathogen in patient with chronic granulomatous disease?
A. Aspergillus species then staphylococcus aureus
Answer: A
https://www.uptodate.com/contents/chronic-granulomatous-disease-pathogenesis-clinical-manifestations-and-diagnosis?source=search_result&search=chronic+granulomatous+disease&selectedTitle=1~122

10. Immune deficient pt : what vaccine could be given :


A. Measles
B. Rubella
C. Pneumococcus
D. varicella
Answer:C
Q: 66, Page: 98, Brattons Family Medicine Board Review, 5th edition
11. wheal with erythematous base itching lymph node enlargement periorbital swelling hepatosplenomegaly ?

smle ,2016

65
A. Rheumatic arthritis
B. Angioedema
C. Cholinergic urticaria ( not sure): itching more with urticarial
Answer:
12. Clear presentation of Sjogren Syndrome, asking about the complication:
A. Pulmonary fibrosis
B. Malabsorption
C. Lymphocytic tissue infiltration
Answer: C
Morbidity associated with Sjgren syndrome is mainly associated with the gradually decreased function
of exocrine organs, which become infiltrated with lymphocytes.
Ref:http://emedicine.medscape.com/article/332125-overview#a7
13. peanut allergy mechanism of action
Answer:
Immunoglobulin E (IgE) mediated The discovery of IgE and its relation to allergic reactions provided a
major step in the understanding of the pathogenesis of food allergy
REF :http://www.uptodate.com/contents/pathogenesis-of-food-allergy#H4
Reference: https://www.uptodate.com/contents/clinical-manifestations-of-food-allergy-an-overview?source=search_result&search=allergy&selectedTitle=2~150

14. What types of hypersensitivity rapid onset of :


Answer:
If they mean hypersensitivity reaction so it's: hypersensitivity I (immediate) (IgE)
Reference: https://www.uptodate.com/contents/clinical-manifestations-of-food-allergy-an-overview?source=search_result&search=allergy&selectedTitle=2~150

smle ,2016

66

15. Thrombocytopenia and recurrent infection (+ Eczema??), Male and uncle has the same condition
Answer: WiskottAldrich syndrome
Reference: https://www.uptodate.com/contents/wiskott-aldrich-syndrome?source=search_result&search=Wiskott%E2%80%93Aldrich+syndrome&selectedTitle=1~62

16. Patient complaining of hematuria and cough with saddle nose?


Answer: Wegeners Granulomatosis
Reference: Toronto Notes 2016 (32E), Page: 1302 (RH19)
17. Question about chronic granulomatous disease.?
18. Goodpasture syndrome >>>>> types of GN : Rapidly Progressive GlomeruloNephritis Type I (RPGN
Type 1)
Reference: Toronto Notes 2016 (32E), Page: 718 (NP23)
19. X-LINKED agammaglobulinemia ?

Reference: [Conrad Fischer] Master The Board USMLE Step 2 CK, (2nd E), Page: 45
20. What most commonly cause itching?

smle ,2016

67

CARDIOLOGY
1.

What is first thing to do in MI ?


A- ECG
Answer: A The first goal for healthcare professionals is to diagnose in a very rapid manner whether the
patient is having an STEMI or NSTEMI because therapy differs between the 2 types of myocardial infarction. Reference:
http://emedicine.medscape.com/article/155919-treatment
2.

A Female C/O leg pain when she walks 300 Meters. Relief by rest:
A- Claudication
B- DVT
Answer: A Toronto Claudication.pain with exertion: usually in calves or any exercising muscle group.relieved by short rest: 2-5 min, and no postural changes necessary .reproducible: same distance to elicit
pain, same location of pain, same amount of rest to relieve pain
3.

Patient HTN and hyperlipidemia with chest pain and when he is coming to hospital take drug relieve his symptoms which drug :
a- nitric oxide
b- digoxin
answer: Nitrates.
4.

patient has history of MI suddenly he became breathlessness and harsh systolic murmur heard
what the cause :
A.PE
B.rupture aortic cusp
C.tricuregurge
D. rupture .......
Answer :-Rupture of papillary muscle ~>MR (harsh systolic )

smle ,2016

68
https://books.google.com.sa/books?id=ANbZecMvdyIC&pg=PA107&lpg=PA107&dq=myocardial+infarction+harsh+systolic&source=bl&ots=Zc4qWUx2t6&sig=6Vaup9OPqC3FhVYwI7ElgI7b2bA&hl=en&sa=X&redir_esc=y#v=onepage&q&f=true
5.

Which heart disease is common in down syndrome :


a- VSD
b- atrioventricular septal defect
c- coarctation of aorta
answer:B.
6.

best initial screening test for pt suspected with coarctation of aorta


a-echo/Doppler
b-CT cardiac
c-MRI cardiac
d-cardiacangio
answer : Chest X-Ray (best initial test according to Master the Boards, First Aid, Kaplan)
UpToDate: In children with coarctation, echocardiography often provides adequate anatomic and hemodynamic information for the surgeon or interventional cardiologist without the need for a further imaging study. However, MRI or CT is generally used as a complementary diagnostic tool in adolescent and
adult patients, and provides important anatomic data prior to intervention.
In adults, cranial MR angiography (or CT angiography) is also appropriate to search for intracranial aneurysms.
Master the boards CK
7.

What is the commonest cause of HTN in adolescent :


A-idiopathic
B -renal-.....
Answer: idiopathic

smle ,2016

69
8.

Patient on warfarin 7 mg presented with melena, INR was very high. What will you do?
a. givevit. K
b. b. lower the dose of warfarin
answer: stop warfarin + Vit K
9.

Young pt with HTN discrepancy , what is the tx ?


A- Trans-aortic stenting
B- Thiazides
answer : ?
common cause B
of HTN at adolescent is essential .
10. Pt with long hx of uncontrolled HTN , he presented to you with headache and 160/90 BP , what
you will see in his kidneys :
A-Decrease sclerosis
B-Increase hyalinization of arterioles
Answer: B
hyaline arteriosclerosis is associated with aging , long standing HTN and DM
11. Pt with migraine and HTN Best medication to ttt HTN?
a) BB
b) ACEI
c) CCB
Answer:A
Source: Angiotensin II receptor blockers (ARBs) and angiotensin converting
enzyme (ACE) inhibitors , have been shown to provide effective migraine
prophylaxis. These two classes of drugs are recommended by ( The Seventh
Report of the Joint National Committee on Prevention, Detection, Evaluation,
and Treatment of High Blood Pressure )for the treatment of hypertension and
thus may be reasonable options for the hypertensive patient with migraine
headache
12. Elderly pt with right leg swelling,pitting edema hx of knee swelling and pain, what is the next?
A.Echocardiogram
B.Doppler
C.Chest x-ray
Answer:
13. Patient with murmur in left sternal border change with stand or sitting what is Dx:
a- pulmonary stenosis
b- aortic stenosis
c- cardiomyopathy
answer:innocent murmur
14. patient with chest pain for 6 hours what you will give :
a- tPA
b- aspirin

smle ,2016

70
answer:b. Aspirin improves survival & lowers mortality in patients with acute coronary syndrome and
should be administered immediately upon arrival to the hospital.
Reference: Medescape
15. (long scenario) case of endocarditis with negative bacterial culture, there is diastolic murmur radiate to the left axilla. What is the most likely diagnosis?
a.SLE
b.Rheumatoid arthritis.
c. Acute myocarditis.
Answer: A
Libman-Sacks endocarditis (otherwise known as verrucous, marantic, or nonbacterial thrombotic endocarditis) is the most characteristic cardiac manifestation of the autoimmune disease systemic lupus erythematosus.One cohort study reported that pure mitral regurgitation was the most common valvular
abnormality, followed by
aortic regurgitation, combined mitral stenosis and regurgitation, and combined aortic stenosis and regurgitation.
Reference:http://emedicine.medscape.com/article/155230-overview#showall
Blood culture-negative infective endocarditis (IE)
:http://www.uptodate.com/contents/epidemiology-microbiology-and-diagnosis-of-culture-negativeendocarditis
16. best exercise to patient with HTN :
a- weight loss
b- aerobic exercise
answer: b
anaerobic increase blood pressure
17. pregnant lady with DVT what is the best investigation to diagnose?
a-d-dimer
b-duplex to calf muscle
c-CT-angio
answer:
18. Pt had MI , he was given sublingual nitrate and analgesia , the MOA of the analgesic that was
given ?
Answer: morphine bind to Mu receptor, side effect of sublingual .....
19. Which of the following is side effect of morphine:
a- dry cough
b- nausea , vomiting
c- tachypnea
d- anxiety
answer: B
20. Man wants to increase awareness about stroke prevention , what will he do
A. HTN campaign in mole
Answer:
Repeated, HTN is the most important factor for stroke prevention.

smle ,2016

71

21. Which of the following is most likely to be the presentation of a patient with early STEMI?
A. Troponin of 0.12 with T inversion in V1-V4
B. Pathological Q wave with subsided chest pain
C. Presence of chest pain with 0.3 mg elevation in ST segment in leads 2,3, AVf
D. ST depression in 2,3, AVf
Answer: C?
1. Normal ECG prior to MI
2. Hyperacute T wave changes - increased T wave amplitude and width; may also see ST elevation
3. Marked ST elevation with hyperacute T wave changes (transmural injury)
4. Pathologic Q waves, less ST elevation, terminal T wave inversion (necrosis)
o (Pathologic Q waves are usually defined as duration 0.04 s or 25% of R-wave amplitude)
5. Pathologic Q waves, T wave inversion (necrosis and fibrosis)
6. Pathologic Q waves, upright T waves (fibrosis)
22. (long scenario) man with chest pain and abnormal EKG. Which one of the following will be elevated?
a. ESR.
b. M2 Protein.
c. C-RP.
d. Creatinine.
Answer: C?
References:http://circ.ahajournals.org/content/113/6/e72.full
http://circ.ahajournals.org/content/123/10/1092.full
http://circ.ahajournals.org/content/107/3/499.figures23. According to the modified criteria of rheumatic fever which is considered as a minor criteria ?
a. Carditis
b. Fever
c. Arthritis
Answer:

smle ,2016

72
24. Patient come with precordial pain, ECG ST segment elevation, patient given aspirin and nitrate,
but no relieve of pain what next step you will do?
A. Give morphine IV (umm alqura)
Answer:
25. 27 Y/O female , recurrent palpitations ECG :
Answer: Supraventricular tachycardia
http://www.practicalclinicalskills.com/mobile/ekg/supraventricular-tachycardia.aspx
26. MI patient within 6 hours what is the most expected complication :
A. PE
B. Arrhythmia
Answer : B Arrhythmia is the most expected complication of MI within 6 hours( especially VF)
Reference: FA step 1
27. Patient with chest pain and ST changes, you will find elevation in:
A.ALT
B.AST
C.Troponin
Answer:C
28. infective rheumatic fever. ! Rash ,
A. Reticulocyte count
Answer:
29. Pt with decreased lower limbs pulses and intercostal notching in x-ray. What is the diagnosis?
A- Coarctation of Aorta
Answer: A
Reference: http://emedicine.medscape.com/article/895502-clinical
30. What lower blood pressure the most ??
A-Weight loss
B-Salt restricted diet
C- Exercise
Answer: A
*Weight loss helps to prevent hypertension (range of approximate systolic BP reduction [SBP], 5-20 mm
Hg per 10 kg).
*Reduce sodium intake to no more than 100 mmol/d (2.4 g sodium or 6 g sodium chloride; range of approximate SBP reduction, 2-8 mm Hg)
*Engage in aerobic exercise at least 30 minutes daily for most days (range of approximate SBP reduction, 4-9 mm Hg)
31. Which murmur is a Crescendo-decrescendo murmur?
A-Systolic Ejection (Aortic stenosis)
B-Physiologic murmur
C-Innocent murmur

smle ,2016

73
Answer: A
Classic crescendo-decrescendo systolic murmur of aortic stenosis begins shortly after the first heart
sound.
Reference: http://emedicine.medscape.com/article/150638-overview
32. Patient has HTN and BPH. .WHICH DRUG CAN USE.?
A.PRAZOSIN
Answer: A
Reference: www.drugs.com
33. Patient with chest pain .. Pic of ECG ..NO ST ELEVATION LAP RESULT, HIGH TROPONIN,
HIGH LDH, HIGH ASPARTAT
Answer : Increase LDH, AST ,Troponin in skeletal muscle injury and MI (NSTEMI)
So answer according the pic.
Reference: http://www.ayubmed.edu.pk/JAMC/PAST/14-4/Rehan%
34. Ischemic heart pt with lap result of high lipid .. What the next order ?
A. TFT
B. LFT
Answer: B
recommend treating all patients with CVD with high-dose statin therapy
In 2012, the US Food and Drug Administration revised its labeling information on statins to only recommend liver function testing prior to initiation of statin therapy and to only repeat such testing for clinical
indications
uptodate
35. A.HIGH RISK UNSTABLE anginaA.low risk
B.NSTEMI
C.STEMI
Answer: B
NSTEMI indicate myocardial necrosis marked by elevation in troponin I and CK-MP without ST-segment
elevation on the ECG
Ref: First Aid USMLE STEP 2 CK
36. 69 y/o female on antiarrhythmic and she developed hyperpigmentation . Which drug cause that ?
answer:cordarone

37. Patient with heart failure and AF, you added digoxin, what is the benefit:
A. decrease heart failure
B. slow ventricular rate
C. decrease ventricular efficacy
answer: B

smle ,2016

74
Uptodate

38. long scenario asking how amlodipine cause LL edema


a-Increase capillary oncotic pressure ?
b-Decrease plasma ...
c-Decrease cardiac output
Answer:
The pathophysiology of edema with CCB therapy has no relationship to salt and water retention. In fact,
CCBs are intrinsically natriuretic -- a process related to a direct tubular effect of CCBs. This latter phenomenon is the probable explanation for the occasional reports of polyuria in patients receiving CCB
therapy. However, CCB-related edema can occur with pre-existing volume expanded forms of edema, in
which case the edema can be severe. CCB-related edema is caused by preferential arteriolar or precapillary dilation without commensurate dilation in the venous or postcapillary circulation.
39. patient with murmure in midsternal not radiation high pitched first sound with decrescendo:crescendo ?
A. ejection diastol
B. ejection systolic
C. physiologic
D. innocent
Answer: Systolic ejection ( not sure 100%) maybe some points missed in the questions because answer
overlapping. Systolic ejection murmurs may be functional or organic.The most common functional systolic ejection murmur in adults is probably a variant of Still's murmur, the so-called innocent murmur of
childhood. It is a short, buzzing, pure, medium-pitched, nonradiating, midsystolic murmur heard best
along the upper left sternal border.
40. Patient with damaged valve after a tooth extraction he got infective endocarditis, what is the organism:
a- strep viridans
b- staph aureus
answer:A
41. Elderly heavy smoker with pain on walking and o/e loss of hair and pallor of feet whats dx?
Answer: PVD
Ref:
http://www.healthline.com/health/peripheral-vascular-disease#Overview1
*ref: Toronto note 2014,C44
42. Infective endocarditis (migratory arthritis)

smle ,2016

75
Answer: Duke criteria for diagnosis of infective endocarditis
Major: at least 2 separate + blood culture for a typical organism , Evidence of endocardial ( via
echo or new murmur )
Minor ( fever>38.5 , vascular and immunological phenomena as septic emboli , janeway lesion ,
osler's nodes , roth spot )
The presence of 2 major or 1 major + 3 minor or 5 minor , diagnosis endocarditis
Ref: First Aid USMLE STEP 2 CK
43. food should be avoided in high cholesterol
A- organic meat
B- egg white
C- avocado
D- chicken
Answer: A
Ref:http://www.webmd.com/cholesterol-management/foods-to-avoid-for-high-cholesterol
44. women complain of buttock pain ,In angiogram which artery most likely occluded?!
a.Internal iliac
b.external iliac
c.femoral
d.pudendal
Answer: A
Ref:http://emedicine.medscape.com/article/461285-clinical
45. Coarctation of aorta, what can cause:
a.
coronary artery disease
b.
aortic dissection
Answer: B
Complications of untreated patient include: HTN, stroke, aortic aneurysm, aortic dissection, premature
coronary artery disease, HF, brain aneurysm or hemorrhage. The most common complication in adult is
HTN and CAD.
Ref:http://www.mayoclinic.org/diseases-conditions/coarctation-of-the-aorta/basics/complications/con-20031772
46. Patient on diuretics developed palpitation. Due to the disturbance of which of these electrolytes
lead to his presentation?
a.
Na
b.
K
c.
Cl
d.
Ca
Answer: B Hyperkalemia: usually asymptomatic but may develop nausea, palpitations, muscle weakness,
muscle stiffness, paresthesias, areflexia, ascending paralysis, and hypoventilation. Can be caused by K+sparing diuretics such as Spironolactone, Amiloride and Triamterene.
Reference: Toronto Notes 2014., c53
47. Which of these anti HTN medications decrease afterload and preload?
a.ACEI
Answer: A

smle ,2016

76
ACE inhibitors have the following actions:
Dilate arteries and veins by blocking angiotensin II formation and inhibiting bradykinin
metabolism. This vasodilation reduces arterial pressure, preload and afterload on the
heart.
Ref: http://cvpharmacology.com/vasodilator/ACE
48. patient with mitral valve disease and murmur radiating to the axilla, right ventricle is enlarged.
How will you manage?
A- Mitral valve replacement
B- Medical treatment
C- PCI
Answer: A
Its Mitral Regurgitation.Valve replacement is indicated when the heart starts to dilate. Do not wait for
left ventricular end systolic diameter (LVESD) to become too large because the damage will be irreversible.
*Toronto note 2014 ,c39
Surgery if: acute MR with CHF, papillary muscle rupture, NYHA class III-IV CHF, AF,
increasing LV size or worsening LV function, earlier surgery if valve repairable (>90%
likelihood) and patient is low-risk for surgery
Surgical Options
Valve repair: >75% of pts with MR and myxomatous mitral valve prolapse
annuloplasty rings, leaflet repair, chordae transfers/shorten/replacement
Valve replacement: failure of repair, heavily calcified annulus
49. Prophylactic antiarrhythmic therapy:
a.Procainamide
b.Lidocaine
c. Amiodarone
Answer: C
Dont give the patient any anti-arrhythmic medication unless he has an arrhythmia. Reference: Step up
to Medicine.
C is the best answer compared to A and B (if not post MI)
The question might be similar to this (Post MI):
In this case the answer will be Metoprolol, which is a beta blocker (Prophylaxis for arrhythmias after MI:
Beta blocker).
Reference: Lippincott Illustrated Reviews Pharmacology.
*Reference: (davidsons Essentials of medicine )

smle ,2016

77
Treatment with amiodrone will reduce the risk of recurrence following successful cardio version , but if
associated with ischemic heart disease B.Blockers will be the first line

50. Why are we concerned about anti hypertensive agents in elderly patients?
a.
Hypotension
b.
Hypokalemia
c.
CNS side effect
Answer: A
Reference: http://www.ncbi.nlm.nih.gov/pubmed/11574742 http://www.ncbi.nlm.nih.gov/pubmed/11574742
51. Cardiac patient on Aspirin, no new complaints. He is having low platelet (less than 10) for the last
6 months. What is your management?
Answer: ?the diagnosis is rare and called aspirin induced thrombocytopenia
Stop aspirin and find the cause (Answered by a senior cardiologist consultant)
52. right coronary artery dominance" is explained as right coronary artery giving branch to:
a.
Circumflex
b.
Anterior Descending
c.
Posterior Descending
d.
Marginal
Answer: C
Reference :Toronto note C2,2014
*right coronary artery give branches to :acute marginal branches ,AV nodal artery , posterior descending
artery .
53. A patient presented with frothy red sputum, flushed cheeks, etc (long scenario). What is the diagnosis?
A. mitral stenosis
Answer: A
Reference: Master the boards + Medical diagnosis and Management by Mohammad Inam Danish
Reference: Toronto note 2014 ,C39
symptoms of MS: SOB on exertion ,orthopnea, fatigue ,palpation ,peripheral edema, malar rash
54. A male patient came to the ER complaining of palpitations, tachycardia ... ECG shows deep S wave
in lead (?) and tall R wave in lead (?) ,Dx ?
a.
LBBB
b.
RBBB
Answer: ?

smle ,2016

78
LBBB:
QRS duration >120 msec
V1 and V2: W pattern and wide deep slurred S wave
V5 and V6: wide QRS complex with M pattern or rabbit ear pattern
RBBB: the opposite of LBBB
QRS duration >120 msec
V1 and V2: wide QRS ( more than 3 small squares) with ear rabbit pattern or M shape pattern
V5 and V6: wide and deep/slurred
LVH:
S inV1 + R in V5 or V6 > 35mm above age 40, (>40 mm for age 31-40, > 45 mm for age 21-30)
R in aVL>11mm
R in I + S in III >25mm
RVH:
Right axis deviation
R/S ratio > 1or qR in lead V 1
RV strain pattern: ST segment depression and T wave inversion in leads V1-2
Reference: Toronto Notes

55. long standing trip, swelled LL, no pain, high D-Dimer management?
A- aspirin
B- LMWH
C- warfarin
D- unfractionated heparin with warfarin
Answer:D
Admitted patients may be treated with a LMWH, fondaparinux, or unfractionated heparin (UFH). Warfarin 5 mg PO daily is initiated and overlapped for about 5 days until the international normalized ratio
(INR) is therapeutic >2 for at least 24 hours.
For admitted patients treated with UFH, the activated partial thromboplastin time (aPTT) or heparin activity level must be monitored every 6 hours while the patient is taking intravenous (IV) heparin until the
dose is stabilized in the therapeutic range. Patients treated with LMWH or fondaparinux do not require
monitoring of the aPTT.

smle ,2016

79
Platelets should be monitored. Heparin or LMWH should be discontinued if the platelet count falls below 75,000. Fondaparinux is not associated with hepatin-induced thrombocytopenia (HIT).
Reference: http://emedicine.medscape.com/article/1911303-treatment
56. patient symptoms of rheumatic fever with heart involvement and history of sore throat was not
treated..Treatment ?
A. Aspirin and steroid
Answer: give penicillin to eliminate any residual streptococcus infection +high dose aspirin + corticosteroid (prednisolone )
And long term prophylaxes with penicillin should be given
Reference : Davidsons
57. what is diagnosis?

A.
B.
C.
D.

SVT
AF
VT
VF

Answer: A
same ECG that shows SVT in UQU cardiology section<< couldnt find it, so Iuploaded another ECG
strip.Read about other arrhythmias.
Ref: ECG made easy
-The atria depolarize faster than 150 b/min
-P wave superimposed on the T wave of the preceding beats
58. patient known case of aortic stenosis had syncope. What is the cause?
a. systemic hypotension
Answer: A
Syncope from aortic stenosis often occurs upon exertion when systemic vasodilatation in the presence
of a fixed forward stroke volume causes the arterial systolic blood pressure to decline. It also may be
caused by atrial or ventricular tachyarrhythmias.
Syncope at rest may be due to transient ventricular tachycardia, atrial fibrillation, or (if calcification of
the valve extends into the conduction system) atrioventricular block. Another cause of syncope is abnormal vasodepressor reflexes due to increased LV intracavitary pressure (vasodepressor syncope).
Reference: http://emedicine.medscape.com/article/150638-clinical
Refrence: davidsons Essentials of medicine
-syncope in AS usually occur on exertion when cardiac output fails to rise to meet demand because of
severe outflow obstruction ,causingafall in B/P
59. Which one of these can cause LBBB:
a.
Aortic stenosis
b.
PE
c.
Cardiomyopathy

smle ,2016

80
Answer: A (Most common of these choices)
Arteriosclerotic coronary artery disease is the most common adult cardiovascular disease. As a result, it
is the most common cause of left bundle branch block. Hypertension, aortic valve disease and cardiomyopathies continue to be important but less common etiologic disorders. Reference: Journal of the American College of Cardiology.
60. Patient is hypertensive. In addition to antihypertensive medications the patient was given phytosterol. What is the mode of action of phytosterol?
a.
Decrease plasma triglycerides
b.
Decrease plasma cholesterol
c.
Inhibit fatty acid synthesis
d.
Decrease de novo synthesis of cholesterol
Answer: B
Phytosterols are plant sterols structurally similar to cholesterol that act in the intestine to lower cholesterol absorption. Reference: Pubmed
Reference :http://my.clevelandclinic.org/services/heart/prevention/nutrition/food-choices/phytosterols-sterols-stanols
Phytosterols (referred to as plant sterol and stanol esters) are a group of naturally occurring compounds
found in plant cell membranes. Because phytosterols are structurally similar to the bodys cholesterol,
when they are consumed they compete with cholesterol for absorption in the digestive system. As a result, cholesterol absorption is blocked, and blood cholesterol levels reduced.
61. A Patient had an MI and was treated for it, after that he developed chest pain that worsen with
movement and taking deep breath. On examination there was distant heart sounds and pericardial rub. What is the most ECG changes associated with this condition?
A. ST changes
B. PR prolongation
Answer: A case of pericarditis.
ECG: initially diffuse elevated ST segments depressed PR segment, the elevation in the ST segment is
concave upwards >> 2-5 d later ST isoelectronic with T wave flattening and inversion.
Reference: Toronto Notes.
http://emedicine.medscape.com/article/156951-overview
62. Which medication will delay the surgery for chronic aortic regurgitation?
a.
Digoxin
b.
Verapamil
c.
Nifedipine
d.
Enalapril
Answer: C
Nifedipine is the best evidence-based treatment in this indication.
ACE inhibitors are particularly useful for hypertensive patients with AR.
Beta blockers may be indicated to slow the rate of aortic dilatation and delay the need for surgery in
patients with AR associated with aortic root disease. Furthermore, they may improve cardiac performance by reducing cardiac volume and LV mass in patients with impaired LV function after AVR for AR.
Reference: 3rd Edition UQU>Cardiology> Q3
In severe aortic valve insufficiency, the excess in afterload increases burden on the left side of the heart.
Theoretically, any medication that can reduce afterload could be expected to improve left ventricular

smle ,2016

81
function and decrease regurgitant backflow from the aorta. This would provide a temporizing measure
by which surgical intervention can be postponed. One study showed that the use of nifedipine in asymptomatic patients with severe aortic regurgitation who had normal LV function could delay the need for
surgery by 2-3 years. This result may also be expected with the use of similar vasodilating agents.
Reference: Medscape.
63. long scenario I don't remember exactly but there is upper limb hypertension and low or absent
lower limb pulse What is Dx?
A. Coarctation of aorta
answer : A
64. mid diastolic murmur in left sternum ?
A- Mitral stenosis
B- mitralreguarge
C- aortic stenosis
D- aorticregurage
answer A
MS: mid diastolic to late diastolic , rumbling
MR: Holosystolic pansystolic , blowing radiate to left axilla
AS: Midsystolic , harsh radiated to carotid artery
AR: early diastolic
Reference :
http://emedicine.medscape.com/article/155724-clinical#b4
65. Pt HTN came with uric acid 200 you prescribe antihypertensive drug for him after 1 week uric acid
400 and gout what is the drug ?
Answer: Thiazide diuretics
Reference :
http://www.uptodate.com/contents/diuretic-induced-hyperuricemia-and-gout
66. Aspirin inhibit which product formation
A. Thromboxane A2
answer: Thromboxane A2 (TXA2) is a type of thromboxane that is produced by activated platelets and
...Aspirin irreversibly inhibits platelet cyclooxygenase 1 preventing the formation of prostaglandin H2,
and therefore thromboxane A2
reference:
https://en.wikipedia.org/wiki/Mechanism_of_action_of_aspirin
67. ASD :
**Fixed wide s2 splitting ( two q about it )
http://emedicine.medscape.com/article/162914-clinical#b2
68. Which murmur is associated with Mitral stenosis ?
A-Mid systolic
B- Holo systolic
C- Mid diastolic
D- early systolic

smle ,2016

82
Answer : C- Mid diastolic
Reference :
http://emedicine.medscape.com/article/155724-clinical#b4
69. Which antilipid medications decrease LDL and Triglycerides, and increase HDL?
a.Statin
b.Fibrate
Answer: A
Statins block the production of cholesterol in the liver itself. They lower LDL and triglycerides, and have a
mild effect in raising HDL.
fibrates are not effective in lowering LDL cholesterol
Reference :
http://www.webmd.com/heart-disease/guide/heart-disease-lower-cholesterol-risk?page=4
http://www.medicinenet.com/fibrates/article.htm
70. 29 yrs old female , came for her annual check up , her father was dx with dyslipidemia one year
ago and she is anxious about she will have the same thing , he lab were all normal except for high
triglycerides , what will you give her ?
Answer: 6 week trial of diet and exercise if very high not respond start fibrate
71. ECG finding of ostiumsecundum atrial septal defect
A- LT axis deviation
B- RBBB
C- LVH
D- Delta wave
Answer: B
Toronto: ECG: RAD, mild RVH, RBBB

72. A patient on Digoxin and Lisinopril or CCB??. Most probable electrolyte imbalance that will affect
his ECG?
A. K
B. Na
C. Mg
Answer: A
Digoxin side effect.

smle ,2016

83
73. reversible cause of stroke?
A- Hypertension
B- Obesity
C- Smoking
Answer: A
I dont know
74. Pt with migraine and HTN Best medication to ttt HTN?
a)
BB
b)
ACEI
c)
CCB
Answer:A
Beta blockers Beta blockers were originally developed to treat high blood pressure. In addition,beta
blockers reduce the frequency of migraine attacks in 60 to 80 percent of people
Reference;
http://www.uptodate.com/contents/migraine-headaches-in-adults-beyond-the-basics
75. X-ray of pericardial effusion
answer

76. what factor cause thrombosis ?


Answer:
Factor V Leiden mutation or activated protein C resistance*
Prothrombin 20210 mutation
Hyperhomocysteinemia
FVIII, FIX, FXI, FVII, VWF

77. what medication dissolve thrombus ?

smle ,2016

84
Answer:
STREPTOKINASE , ALTEPLASE , RETEPLASE , TENECTEPLASE , LANOTOPLASE
Ref: uptodate
78. Mid diastolic rumble with opening snap and dysphagia. Where is the lesion?
a)Left atrium
b) Aortic arch
c) Left ventricle
answer: A
reference:
http://doknotes.wikidot.com/mitral-stenosis
79. mitral stenosis cause enlargement in which chamber
a-left atrium
b- right atrium
c- left ventricle
d- right ventricle
Answer:
Left Atrium
Dysphagia from left atrium (LA) pressing on the esophagus
Hoarseness (LA pressing on laryngeal nerve)
80. obese male pt presented to family physician complaining of chest pain for 2 days ..ECG normal
..what to do next ?
A-Treadmill
b- Coronary angio
c- 24 hrs monitoring
Answer: A
81. What is the best to dx coarctation of aorta ?
A-Cardiac MRI
B- echo
Answer:
Postnatal diagnosis The clinical diagnosis of coarctation of the aorta is based upon the characteristic
findings of systolic hypertension in the upper extremities, diminished or delayed femoral pulses (brachial-femoral delay), and low or unobtainable arterial blood pressure in the lower extremities. The diagnosis is confirmed by noninvasive imaging methods, particularly echocardiography.
Ref: http://www.uptodate.com/contents/clinical-manifestations-and-diagnosis-of-coarctation-of-theaorta#H17
82. What valve lesion you'll find in acute infective endocarditis ?
A- Mitral stenosis
B- mitralregurgue
C- aortic stenosis
D- aorticregurge .
Answer: B-mitral regurge
*Mitral valve is most frequently involved ( 1st aid USMLE step 1 )

smle ,2016

85
In the past, rheumatic heart disease with mitral stenosis was the most common valvular defect in patients with IE. Recently, the most common predisposing lesions are mitral regurgitation, aortic valve disease (stenosis and regurgitation), and congenital heart disease Mitral valve prolapse is a risk factor for
IE, primarily when regurgitation is present.
http://www.ncbi.nlm.nih.gov/pmc/articles/PMC2726828/
83. Pt with heart valve lesion developed endocarditis ..what's the organisms ?
Answer: streptococcus virdins
they have the potential of causing endocarditis, in particular in individuals with damaged heart valves.
They are the most common causes of subacute bacterial endocarditis. (wiki)
84. Anomaly between aortic arch and pulmonary trunk ?
A. Patent ductus arteriosus .
Answer: A

85. Enzyme that get elevated in MI ?


A. Creatine phosphokinase
Note : no troponin in question
Answer: Creatine Kinase-MB,
Serial measurements of CK-MB isoenzyme levels were previously the standard criterion for the diagnosis
of myocardial infarction. However, sensitivity and specificity are not as high as they are for troponin levels. The trend has favored using troponins and they are now considered to be the criterion standard for
defining and diagnosing myocardial infarction, according to the American College of Cardiology.
Reference:http://emedicine.medscape.com/article/155919-workup#c9

smle ,2016

86

86. smokers obese patient can't exercise with family history of MI came with vague chest pain . But
ECG is normal Next step ?
A- 24 hours ECG
B- exercise with ECG
C- perfusion cardiac scan
Answer : C
When this is the answer:
Patients who cannot exercise to a target heart rate of >85% of maximum:
COPD
Amputation
Deconditioning
Weakness/previous stroke
Lower-extremity ulcer
Dementia
Obesity
Reference : Master The Board
87. With thyrotoxicosis, what is the most common arrhythmia:
A. SVT
B. VT
C. VFib
D. WPW
Answer: SVT ( toronto note mentions AF as most common)

smle ,2016

87

88. An ECG with ST elevation in V1-V5. :


A. Extensive anterior MI
B. High lateral MI
Answer: Anterior or anteroseptal MI

smle ,2016

88

89. A patient with sudden chest pain, diaphoresis, best test to order
A. Echo
B. ECG
C. Cardiac enzymes
Answer: ECG ( first investigation in chest pain is ECG, Master the board)
90. hypertensive patient on ACEI but not controlled BP, what to add?
A-furosemide
B-thiazide
C-beta blocker
Answer: B
Answer: we recommend therapy with a long-acting ACE inhibitor or ARB in concert with a long-acting
dihydropyridine calcium channel blocker. Combination of an ACE inhibitor or ARB with a thiazide diuretic
can also be used but may be less beneficial. ACE inhibitors and ARBs should not be used together.
Ref: uptodate
http://www.pharmacology2000.com/Cardio/antihyper/antihype.htm
91. A 72 y male disoriented and hallucinating and disorganized thinking had aorto popliteal graft and
symptom fluctuates in the 2 days what the cause
A.multi infarct dementia
B.mania
C.dementia
D.delirium
Answer: D (postoperative delirium)
Older patients have a high incidence of post-anesthesia delirium and thus should have a mental state
examination before and after surgery. (Canada Q bank )
92. best initial screening test for pt suspected with coarctation of aorta
A. echo/doppler
B. ct cardiac
C. mri cardiac
D. cardiac angio

smle ,2016

89
Answer:A Echo
http://www.uptodate.com/contents/clinical-manifestations-and-diagnosis-of-coarctation-of-the-aorta
93. Pt with retrosternal chest pain for 3 days increase in the last 24 hrs relieved by sublingual nitro
Dx?
A.Unstable angina
B. Pericarditis
C. MI
Answer: The duration of chest pain here is more than 3 days so unlikely to be a cardiac cause.
I suggest Non Cardiac cause eg. esophageal spasm that will relieved by nitro .
94. Pt had hx of AF and prior stroke, How to prevent this pt from second stroke?
A. Dipyridamole
B. Warfarin
Answer : B. Warfarin
Secondary prevention of Ischaemic stroke ( Davidsons 21st edition P1188 , box 26.62)
Anticoagulants: There is no benefit to be gained in the routine use of anticoagulants after acute strokeexcept in the presence of atrial fibrillation.
treating 1000 pt for a year prevents about 80stroks
Antiplatelets: in pt with Ischaemicstrok , aspirin , colpidogrel or combination of aspirin
and dipyridamle reduce the risk of recurrent strok , MI , vascular death. treating 1000 pt
for a year prevents about 10 stroks
Statins: reduce the risk of recurrent stroke,MI and vascular death
treating 1000pt for a year prevents about 17stroks
blood pressure lowering :B.p lowering even in the( NR range) reduce the risk of recurrent stroke , MI , , vascular death .
treating 1000 pts for a year prevent 22 strokes
95. woman underwent lung lavage hemosiderin laden macrophages seen indicates.. ?
A - Heart failure
answer A
heart failure lead to diffuse alveolar hemorrhage syndromes
http://www.uptodate.com/contents/the-diffuse-alveolar-hemorrhage-syndromes?source=see_linkionName=Bronchoalveolar+lavage&anchor=H12#H12
96. early detection of ST segment elevation MI :
a-Typical chest pain with some ECG changes
b-typical chest pain with some ecg changes
Answer: typical chest pain with some ECG changes
97. character of MS murmur:
A-diastolic low pitched rumbling
Reference: talley
Turbulent flow produce the characteristic low pitched mid diastolic murmur. Davidsons
21steditionP. 615)

smle ,2016

90

98. something
down e fix split s2 +harsh systolic murmur + biventricular hypertrophy +......... dx?
a)ASD
b)VSD
Answer:ASD
Wide fixed splitting of 2nd heart sound , systolic flow murmur over the pulmonary valve, CXR show enlargement of the heart and pulmonary artery ,ECHO shows RV dilation , RV hyperatrophy and pulmonary
artery dilation .
In VSD : ECG show bilateral ventricular hyperatrophy
Davidsons 21stedition P. 632-633)
99. The most effective medications are called anticoagulants and include
Answer: Warfarin, dabigatran, and 3.rivaroxaban
100.traveling women 18hr in flight Develope LL edema what will you do:
a-Compression ultrasonography
b-Angiography
c-Other two option ?
Answer: A
101. Most specific for coronary artery disease
A. Cholesterol
B. Triglycerides
C. Hdl
D. LDL
Answer: LDL
102. Dyslipidemia on tx, developed muscle weakness, whats the drug hes taking?
A. Statins

smle ,2016

91
Answer:
103. Indication of Defibrillator (pacemaker) is injury in ?
A.SA node
B.AV node
Answer:
104. Scenario of PT come with friction rub what is your next action
a. Echo and cardiology consultant
b. X ray
c. Pericardiocentesis
Answer A pericardial friction rub is specific for pericarditis. Echo is part of its initial approach. If the patient is unstable, Pericardiocentesis will be needed
X-ray doesn't have a strong rule in diagnosing pericarditis
Extra note: Echocardiography is recommended in all cases of pericarditis. Any form of pericardial inflammation can induce pericardial effusion.
Medscape
105.without tilling the dx) then asked about Mode of inheritance ?
a)
X linked
b)
AD
c)
AR
d)
Mitochondrial
Answer: B
Source:http://ghr.nlm.nih.gov/condition/ehlers-danlos-syndrome#genes
How do people inherit Ehlers-Danlos syndrome? The inheritance pattern of Ehlers-Danlos syndrome varies by type. The arthrochalasia, classical, hypermobility, and vascular forms of the disorder have an autosomal dominant pattern of inheritance. The dermatosparaxis and kyphoscoliosis types of Ehlers-Danlos
syndrome, as well as some of the rare, less well-characterized types of the disorder, are inherited in an
autosomal recessive pattern.

cardiac case ..
106. Patient with MI what is the drug that decrease the mortality rate:
a) Captopril
b) digoxin
NO B blocker in the choices
107. Patient diagnosed with rheumatic heart disease and mitral regurg. what is the treatment of
choice:
a) oral penicillin and aspirin (I choose this one)
b) IM penicillin once every month
c) corticosteroids
d) IV penicillin
answer:

smle ,2016

92
108. Hypertension in child ?
A. More than 120/70
B. More than 140/90
C. More than 90th percentile
D. More than 95th
Answer:
109.SVT scenario what most important to test?
A- TSH
Answer :
Electrolyte levels - Should be checked because electrolyte abnormalities can contribute to paroxysmal
supraventricular tachycardia (paroxysmal SVT)
CBC help to assess anemia
TSH to asses hyperthyroidism
110.patient elderly k/c of DM and HTN with hx of previous TIA and ECG showed Atrial fibrillation controlled on digoxin, whats next for him ?
A- Anticoagulate
Answer: A
CHAD2 score
http://www.fpnotebook.com/cv/exam/ChdScr.htm
111.lung curve for cardiac notch at what level of costal cartilage:
A- 3
B- 4
C- 5
D- 6
Answer B
112. Acute onset chest pain + image coronary angio not so clear ..
A-Tpa
B-Pci
C- Heparin
Answer:
113. Mitral stenosis ECG :
A-Bifid p wave left axis deviation
B-Elevated p wave right axis deviation
Answer: B
The ECG in mitral stenosis is often normal early in disease. The most common finding is left atrial enlargement (p-mitrale), however this finding disappears if the patient enters atrial fibrillation. Right heart
strain may produce findings of right axis deviation and right ventricular hypertrophy on ECG. In pure mitral stenosis, left ventricular hypertrophy would be absent.
http://www.healio.com/cardiology/learn-the-heart/cardiology-review/mitral-stenosis/introduction-andetiology
114. sign of pericardial tamponade:

smle ,2016

93
A- pericardial rub
B- HTN
C- muffled heart sound
Answer: c
1Becks Triad
Hypotension
Increased JVP
Muffled heart sounds
Classic Quartet of Tamponade
Hypotension
Increased JVP
Tachycardia
Pulsusparadoxus
Toronto note 2014
Chest radiographic findings suggestive of mitral stenosis include left atrial enlargement (eg, double
shadow in the cardiac silhouette, straightening of left cardiac border due to the large left atrial appendage
115.The best for lifestyle Tt for HTN PT :
a. Na restriction less than 6
b. Give to him K
c. Run every day 1.5 Km
d. Do some thing4 time per week
in order of effect on BP reduction: weight loss saturated fat Exercise regularlysalt intake
alcohol. ((Step-Up to Medicine, 4E , p439 ))
116.Crescendo-decrescendo murmur best heard
A. at midsternal
B. Innocent
C. Systolic ejection
D. Physiologic
Answer:
117.Pain management in MI:
A- aspirin
B- morphine
Answer: B
118.about cyanotic heart disease.
A- TOF
B- ASD
C- VSD
D- PDA
Answer: A
119.pt has HTN control on his medications , developed albuminuria , what should you add to his HTN
medications :

smle ,2016

94
A-ACEI
Answer: ACEI ??
120.12 y.old girl HTN:
A- 120/80
B-40/99
C- above 90 percentile
D- above 95 percentile
Answer:above 95 percentile
121.which of the following lead to dilatation of aorta?
a)achalasia
b)barrett
answer??
122.60 years old male, c/c chest pain, he was brought to ER of tertiary center. ECG showed St elevation in leads II, V1,V2, V3, after initial management in the ER, next step is?
A-Heparina
B-Streptokinase
C-tPA
D- Stenting of coronary artery
Answer: D
Reference:kumar&clarks clinical medicine , eight edition .page 738 + 739
123. Pt on diuretics and digoxin, feels palpation and muscle weakness, mostly because a disturbance
in?
A-K
B-Ca
C-Na
answer : A
Reference:kumar&clarks clinical medicine , eight edition .page 791
124. Clear case of dessler syndrome ask about ecgchange ?
A. St segment elevation in all lead
answer : A
Reference:kumar&clarks clinical medicine , eight edition .page 774
125. ECG with T inverted in some leads otherwise normal. Labs: tropI high Ck high What Dx ?
A-Low risk unstable angina.
B-High risk unstable angina.
C- NSTEMI.
D- Acute ST elevation MI.
Answer: B
Reference:kumar&clarks clinical medicine , eight edition . page 734
126. Most common risk factor for CAD
A. smoker and 50 y m

smle ,2016

95
B. dyslipidemia and obesity
Answer: B
Reference:kumar&clarks clinical medicine , eight edition . page 727
127.pt after 2 weeks post MI c/o unilateral leg swelling ,pale,loss of hair other leg is normal .Diagnosis?
A-acute arterial emboli
B-acute arterial thrombosis
C- DVT
D- dissecting artery
Answer: A
Reference:kumar&clarks clinical medicine , eight edition . page 786
128.coarctation of aorta with ?
a) down syndrome
b) Turner syndrome
c) Edouard syndrome
d) Patau syndrome
Answer: B
Reference:kumar&clarks clinical medicine , eight edition . page758
129.patient with chronic Afib, came with ??? what is the next step?
The question is not clear.
130. A medication that will decrease cardiac output and decrease peripheral resistance?
A. - Carvedilol
B. - Hydralazine
C. CCB
answer: C
Reference:kumar&clarks clinical medicine , eight edition . page783
131.Genetics of hereditary hypercholesterolemia)
A. apo c 11
B. apo b100
answer: LDLR , apoB , pcsk9
reference: http://www.ncbi.nlm.nih.gov/m/pubmed/17380167
132.Dm , osteoarthritis on NSAID came for regular check up .. Bp found to be 160/something
Invest:Inccr +_ urea IncbicrbDec k
nlna What is the cause of htn :
A. Essential
B. NSAID induced
C. Primary hyperaldosteronism
answer: NSAID induced
Reference:kumar&clarks clinical medicine , eight edition . page778

smle ,2016

96
133. Acute onset chest pain + image coronary angio not so clear ..
A-Tpa
b-Pci
C- Heparin
the q not clear
134. Which antihypertensive drug can cause severe hypertension when stopped abruptly :
A- Clonidine
B-Thiazide
C-Beta blocker
Answer: A
Reference:http://www.ncbi.nlm.nih.gov/m/pubmed/39949/
135. ACS which enzyme to order after 1 hour
Answer: myoglobin
Reference:kumar&clarks clinical medicine , eight edition . page 734
136. Acute MI pt will present with?
a- high troponin I with ST elevation in V2-V6
b- Ongoing chest pain with high ST elevation in lead II - III and avF
c- ST elevation with no chest pain
d- ST depression with .....:/
answer: B
Reference:kumar&clarks clinical medicine , eight edition . page 737
137.Heart defect result from malrotation and truncus arteriosus
a)Transposition of great arteries ( my answer)
Answer: A
138. What is the initial management of hypotension induced lumbar puncture?
A- IV fluid
B- Bed rest
the correct answer is a
139. Which of the following is clinically suggest of early onset st elevation MI?
A- elevated troponin and long inverted T wave in lead 1,2 avf
B- ST elevation of .3 mm in lead v1-v6 with chest pain
C- St elevation without chest pain
The answer is B
140.Or with MI he is on nitroglycerin and morphine . After 20 minutes he worse with raised JVP and
no lung finding. BP drop.
A. Ruptured
B. Arrhythmia
C. RV infarction
Answer: A
C it would be the most correct answer if it was RV FAILURE

smle ,2016

97
Reference:kumar&clarks clinical medicine , eight edition . page 739
141. Pt came e Hx of HF & pulmonary edema. Tx?!
A. Diuretics
Answer: A
Reference:kumar&clarks clinical medicine , eight edition . page 719
142. A 50-year-old male diabetic and hypertensive, unable to maintain an erection, started on Phosphodiesterase type 5 inhibitors. Which one of the following drug classes should be avoided?
A. Steroids
B. Antibiotics
C. Narcotics
D. Nitrates
Answer: D
Reference:http://www.ncbi.nlm.nih.gov/m/pubmed/163875/66
143.A 68-year-old diabetic, started 10 days ago on Amlodipine 10mg for hypertension, now he is complaining of gross ankle edema, on examination, JVP was not raised, by auscultation the chest no
base crepitations
Labs: Na, K and Ca all within normal range
What is the most likely cause of his edema?
A. Na and water retention
B. Increased capillary hydrostatic pressure
C. Decreased oncotic pressure
Answer: B
144.pt on estroststinhasb high TGA ( isolate other test normal)what durg give
A. fenofibrate
B. statin
answer: A
The American Heart Association sets out four main categories of triglyceride levels:

healthy: below 100 milligrams per deciliter of blood (mg/dL)

borderline high: 150 to 199 mg/dL

high: 200 to 499 mg/dL

very high: 500 mg/dL and above.

Triglyceride-lowering drugs have been available for some time. The main ones are niacin and a
class of drugs called fibrates. But the effects of these drugs are somewhat controversial. For example,
two large clinical trials found no benefit from niacin therapy in preventing heart disease events, even
though niacin both lowered triglyceride levels and raised the level of heart-healthy HDL (good) cholesterol. Not only did niacin not lower the risk of heart disease, it also caused troublesome side effects,
such as flushing of the skin and an increased risk of infections. So, there is reason to be wary about taking niacin to lower triglyceride levels.

A large clinical trial of fenofibrate (Tricor, other names), one of the fibrate drugs, showed that it
also reduced triglyceride levels, but didnt lower the risk of heart disease. However, fenofibrate did
seem to help a subgroup of people in the trial who had particularly high triglyceride levels and low HDL
levels.
http://www.health.harvard.edu/blog/a-promising-new-treatment-for-high-triglycerides-201507298160

smle ,2016

98

145.Description of congenital heart disease there is decrease vascularization of the lungs of ray ,, cardiac shadow ( they described boot shaped but not in a direct way )
a) Tetralogy of fallot was my answer
Answer: A
Reference : illustrated textbook of paediatrics , fourth edition . page 312
146. Scenario of PT come with friction rub what is your next action
a. Echo and cardiology consultation
b. X ray
c. Pericardiocentesis
Pericarditis:
ECG: initially diffuse elevated ST segments depressed PR segment, the elevation in the ST segment is
concave
upwards 2 5 d later ST isoelectronic with T wave flattening and inversion
CXR: normal heart size, pulmonary infiltrates
echo: performed to assess for pericardial effusion
Thi question is answered in upcoming questions below
147. Pt with rheumatic fever on aspirin developed heartburn what you will add
a. Misoprostol
b. Durg from ( h2 blocker )
answer: proton pump inhibitor
Aeffective but may causes intolerable side effects
B less effective
Reference:kumar&clarks clinical medicine , eight edition . page 511
148. Which type of the angina present when go to bed
Note: Prinzmetal (variant) angina
Reference: http://www.m.webmed.com/heart-disease/guid/heart-disease-angina
149.Tx of acute myocarditis?!
A. Ab
B. Immunoglobulin
C. Steroid
ANSWER C ??
Answer: A
Reference:kumar&clarks clinical medicine , eight edition . page767
150. 50 Y\O male presented to ER with severe respiratory distress, no other history obtained, investigations were done to him CXR: pulmonary edema with infiltration ECG: he was having Atrial fibrillation, his pulse rate was 125 what investigation you will do to him:
A. Echocardiography
B. spiral CT
C. V\Q match
D. Arteriogram
The answer is: A answered by cardiologist ,and senior medical resident

smle ,2016

99

151. Scenario of PT come with friction rub what is your next action
A-Echo and radiology consultant
B-X ray
C- Pericardiocentesis
Answer: ECG then chest xray
Reference:kumar&clarks clinical medicine , eight edition . page775
152.type of exercise is recommended for coronary artery disease.
A. isometric
B. isotonic
C. yoga
D. dynamic
Answer is isotonic
Reference sle made easy
153. Pt had sign of heart failure (orthopnea ,pnd and ankle swelling) what is the pathophysiology ?
With tables!!!
The question is not clear
154. MI patient has big thrombus in left coronary artery what is drug that cause thrombolytic action?
Answer: streptokinase ,tpa
155.Which is thrombolytic ?
A. Streptokinase
Answer: A
156. In patients with an ostium secundum defect, electrocardiogram (ECG) results usually demonstrate the following:
A. Left -axis deviation
B. Right ventricular hypertrophy
Answer: B
http://emedicine.medscape.com/article/348121-overviewhttp://emedicine.medscape.com/article/348121-overview
157. Patients with HTN and he has hypercalcemia RX
A. Furosemide
Answer: A
Reference:kumar&clarks clinical medicine , eight edition . table at page783
158. Patient 24 hours with diarrhea and vomiting ,Bp supine 120/80, Bp. 80/40. What is the cause ?
A. Decrease intravascular
Answer:
159. What is the most common cause of secondary hypertension?
A. renal disease
B. cushing

smle ,2016

100
C. conn
D. pheochromocytoma
Answer: A
Reference:http://www.uptodate.com/contents/evaluation-of-secondary-hypertension
160. patient has history of endocarditis and underwent to extract his teeth the doctor what will do
before extraction
A. 2 g amoxicillin before
B. 2 g clindamycin before
Answer: A
Reference: http://emedicine.medscape.com/article/1672902-overview#a3
161.ASD changes on ECG
Reference: http://emedicine.medscape.com/article/348121-overview#a1
162. 10 year old child with rheumatic fever without heart lesion, prophylaxis :
A. for 10 years ,
B. for 15 ....
Answer: lifelong prophylaxis
Reference : illustrated textbook of paediatrics , fourth edition . page 322
163. Pt CXR show notching in inferior ribs , what is diagnosis :
A. coarctation of aorta.
Answer:
164. healthy woman travelled for 18 hours, examination were completely normal, investigation done
and D dimer was 350, they give the normal value less than 250 what is the appropriate management:
A. Heparin
B. LMWH and heparin
C. Unfractionated heparin
Answer:
165. Pt with angina take pill under tongue , this pill work by?
a)oxidized to nitric acid (something like that)
answer : Nitroglycerin forms free radical nitric oxide (NO) which activates guanylate cyclase, resulting in
an increase of guanosine 3'5' monophosphate (cyclic GMP) in smooth muscle and other tissues. These
events lead to dephosphorylation of myosin light chains, which regulate the contractile state in smooth
muscle, and result in vasodilatation
166. What is the recommended management for a patient with carotid bruit with occlusion 60% of
the left carotid artery?
A. aspirin daily
B. Angiography
C. carotid endarterectomy
D. none of the above
Answer:A

smle ,2016

101

167.Pt with chest pain diagnosed with STEMI , what the other finding you can detect in his ECG :
A. inverted T wave ?!
answer:
168.pt has HTN control on his medications , developed albuminuria , what should you add to his HTN
medications :
A. ACEI
Answer:
169. HF patient what to order next ?
Answer:
170. Cyanotic heart disease?
A. coarctation of the aorta
B. Truncus arterio
answer: b
https://en.wikipedia.org/wiki/Cyanotic_heart_defect
171.Came with MI CPR done the pt died what is the result of post Mortem finding?
Answer:
172. Pt with a family history of multiple sudden cardiac death pt has Marfan's syndrome features
What might be the cause of death?
A. Ruptured aortic aneurysm
Answer:
173. Which is more at risk for CAD
a-25 male , smoker , sedentary life
b-35 female, type 1DM, obese
c-55 male , hypertension, high cholesterol
d-50 male , obese, DM
answer:
174. Cardiac case ..his brother die when walk Dx?
Answer: cardiomyopathy
175.heart sound that supports pulmonary HTN.
Answer:
Cardiovascular examination in patients with PAH often reveals the following findings:
The pulmonic component of the second heart sound is usually increased, which may demonstrate fixed
or paradoxic splitting in the presence of severe right ventricular dysfunction; occasionally, the second
heart sound may be palpable
A pulmonic regurgitation (Graham Steell murmur) may be apparent
A murmur of tricuspid regurgitation can be present, and a right ventricular lift (heave) may be noted,
Jugular venous pulsations may be elevated in the presence of volume overload, right ventricular failure,

smle ,2016

102
or both; large V waves are often present because of the commonly present severe tricuspid regurgitation
Right-sided S3 gallop
Reference: medscape
176. Elderly Patient presented with chest pain he is a smoker with positive family history of coronary
artery disease and he is dyslipidemic he is also over weight and can not tolerate exercise his ECG
is normal what will do:
A. Stress echo
B. Stress test
C. Resting reperfusion scan
Answer : Angiography
177. Patient has paroxysmal nocturnal dyspnea orthopnea and dyspnea with a history of mitral stenosis ... What is the most likely diagnosis:
A. Acute respiratory distress syndrome
B. Left heart failure
Answer : Right heart failure :

178. Man with chest pain while exercises ,hx of chest pain in rest and exercise. Ejection systolic murmur at left sternal , not radiating , ecg> left atrial enlargement with no specific st changes . Dx
A. pul stenosis ,
B. restrictive cardiomyopathy
C. hypertrophic cardiomyopathy.
Answer:
179.Rheumatic heart Dx prophylactic for 10 yrs old boy with no cardiac involvement?
A. 3m
B. 6m
C. 6 yrs
Answer : 10 yrs
180.Patient presented with chest pain ECG showing ST segment elevation in lead I ,aVL,V5 and V6 ?
A. Lateral ischemia*
B. Ant schema
C. Post schema
D. Lateral ischemia*
Answer :

181.Case of cardio change in ECG show , inversion T and ST change , what is caus ?

smle ,2016

103
a)ischemia
b)( MI - infarction )
Answer:
182. What is condition should to take attention regarding prevention to Coronary artery disease
a)pt 25 age with DM1 and HTN
b)pt 55 age with DM and HTN
c)pt 55 ( or50) with DM and high cholesterol
d)pt 25 with DM and >> something
Answer: ?
183. about truncus arteriosus &Bulbuscordis , cause:
a-VSD
b-ASD
c-tetralogy of fallot
Answer : if there is TOGA choose it , if not choose TOF
184.Which one risky (more) to stroke ?
A- 55 , male, HTN , obese
B- 50, male , DM, hypercholesterolemia
C- 22,female,smoker
Answer: A
Hypertension Hypertension, which promotes the formation of atherosclerotic lesions, is the single
most important treatable risk factor for stroke
Uptodate
185. ECG picture
A. AF
B. VT
C. SVT ?
Answer:
186.hypertensive women 160/90 what will advice her:
A. Reduce salt less than 6g
B. Walk 1.3km 4 times per week
C. Brisk walking 30min 3 time per week
D. Somthing
Answer: reduce salt less than 6g
187. Mid diastolic rumble with opening snap and dysphagia. Where is the lesion?
a)Left atrium
b)Aortic arch
c)Left ventricle
answer:?
http://doknotes.wikidot.com/mitral-stenosis
188.Pt chest pain radiate to left shoulder and jaw whats best analgesics?

smle ,2016

104
(all answers are painkillers medications,morphine was one of the answers).
Answer: Narcotic analgesics (eg.Morphine) remain the drug of choice in many UK hospitals,They not
only relieve the sensation of severe pain but also reduce the effective and physiologic reaction to pain
and thus reduce patient anxiety
poor detailed question in exam.
189. Down syndrome fix split s2 +harsh systolic murmur + biventricular hypertrophy +......... dx?
a)ASD
b)VSD
Answer: atrioventricular septal defect
190.A case of rheumatic fever he had a severe arthritis and mitral regurgitation what is your SHORT
management?
A/steroid and high dose aspirin
B/daily steroid and aspirin
C/IM penicillin
D/monthly
Answer:
191.(long scenario), child with pansystolic (holosystolic) murmur. what is the cause?
a- ASD
b- VSD
Answer: B
192. Isosorbide dinitrate side effect?
Headache Medscape
193.Africa with HT diagnosed now protein+2 Wtttt
A)Diuretic
B)ACEI
Answer:?
194. Man loses his consciousness during playing football ( syncopal attack) , he had history aortic stenosis. how can you explain this case: (no low cardiac output not mentioned in the choices!)
A)Cardiac arrhythmia.
Answer: A might be right because Aortic stenosis is common cause of left bundle branch block
195. Q/pt present with Inferior MI which artery affected?
A. ST elevation in II, III
B. and aVF
C. ST elevation or depression
in V4R can help in differentiating a RCA from a RCX occlusion.
This part of the heart muscle lies on the
diaphragm and is supplied of blood bij the right coronary artery (RCA) in 80%
of patients. In the remaining 20% the inferior wall is supplied by the ramus
circumflexus(RCX).

smle ,2016

105
196. professor present with headache sometimes resolve with panadol vital sign given his bp 170/100
what is the type of his HTN
1. essential
2. Secondary
3. Malignant
4. pain causing
Answer:?
197.relative contraindication of methergine ?
A)HTN
B)stable angina
C- unstable angina
Answer:
198. ST elevation at lead 2 , 3 , and AVF , that represent which cardiac parts?
inferiorMI
199.hypertensive male on furosemide have dehydration ,Hypokalemia 3, hyponatremia 123 What will
you give him?
A--Normal saline 5 kcl , 20cc /hr
B -normal saline 40 KCl, 80cc /hr
c-.45 , 5kcl, 20cc/hr
d- .45 , 40 kcl, 80cc/hr
answer:
200. 22 year old male patient presents with chest pain increase when he lying flat ,Ecg shows diffuse
ST-Elevation in all leads wt is ur diagnosis?
a-myocardial infarction
b-pericarditis
c-Infective endocarditis
Answer: B
201.patient with MI Blood pressure 80/65 what is your action ecg shows (bradycardia) i think complete heart block :
a-dopamine/ norepinephrine
b-iv fluid bolus /sub
c-cutaneous pacemaker my answer
answer:
202.very obese smoker patient complain of chest pain how to test for
myocardial perfusion :
a-exercise stress test
b-24hrsholter monitor
c-dobutamine stress test
answer:
203.which of the following is major criteria of Infective endocarditis ?

smle ,2016

106
a-positive blood culture
b-echo shows valvular
Answer: B its the answer coz we need 2+ve blood cultures
204.Which one is criteria of TOF ?
A-Pulm outflow stenosis ..
Answer:
205.2 Y old ejection systolic murmur cause: ??
Answer:
206.3 y/o k/c of cardiac what you make:
A-Eco
B-ecg
C-catheterization
D-observation
Answer: A??

ENDOCRINOLOGY
1.

A female in her 20s, has DM and hypothyroidism and has white plaques in her mouth and rash on
her back. No hx of immunodeficiency, no travel hx.+vehx of contact with tb with +ppd. Whats
your diagnosis ?
A. Chronic candidiasis
B. Chronic granulomatous disease

smle ,2016

107
answer :

dysplasia?? ( repeated in different part)


2. Pt with constipation increase weight,thinning of hair ?
A- Hypothyroidism
Answer: AHypothyroidism
3. Female presents with weight loss . What clinical finding you will see?
A- buffalo hump
B- skin hyperpigmentations
C- cutaneous stria
Answer : B - skin hyperpigmentations because of adrenal insufficiency
4. In DKA which ketone is predominantly found in urine?
A- Acetone
B- Acetoacetate
C-Beta-hydroxybutyrate
Answer: B, dipstick detect acetoacetate but not beta-hydroxybutyrate
http://fitsweb.uchc.edu/student/selectives/TimurGraham/Ketoacidosis_DKA.html

smle ,2016

108
http://bestpractice.bmj.com/best-practice/monograph/162/diagnosis/step-by-step.html
5. Why DM decrease wound healing
A- Decrease immunity
B- Increase bacteria due to increase glucose
C- Decrease phagocytosis
Answer: C ( I found that because of atherosclerosis )

Uptodate: Over 100 known cytologic factors contribute to impaired wound healing in patients with diabetes.These include decreased or impaired growth factor production, angiogenic response, macrophage
function, collagen accumulation, epidermal barrier function, quantity of granulation tissue, keratinocyte
and fibroblast migration and proliferation, number of epidermal nerves, bone healing, and abnormal
balance between the accumulation of extracellular matrix components and their remodeling by matrix
metalloproteinases.

6. young female came to your office complaining of swelling in front of her parotid gland. No tenderness, no secretion coming out, What is the most likely diagnosis?
a.
Parotid CA
b.
Sialadenitis
c.
Mumps
d.
L.N enlargement
Answer: a
the most common presentation of parotid ca. is a painless, asymptomatic mass; >80% of patients present because of a mass in the posterior cheek region. Some people infected with the mumps virus have
either no signs or symptoms or very mild ones. When signs and symptoms do develop, they usually appear about two to three weeks after exposure to the virus and may include: Swollen, painful salivary
glands on one or both sides of your face(parotitis). Reference: Medscape and myoclinic
7. Hypothyroidism pt on dose of thyroxine 75 .. Missed the dose 2 days because he does not have the
drug , lab result show high TsH and normal t4 .. What dose should be taken ?
a-25
b-50
c-75
d-100
Answer: 75 or 100
in cases where the TSH is only mildly raised, the patient is not symptomatic and the serum free T4 is normal, the diagnosis is sub-clinical hypothyroidism. levothyroxine: 1 microgram/kg/day orally (usual dose
50-75 micrograms/day), adjust dose in increments of 25 to 50 micrograms to normalise TSH
8. Female patient with high prolactin What to exclude

smle ,2016

109
A. thyroid disease
B. pituitary tumors
Answer: Pituitary tumors
A. Causes of hyperprolactinemia
1. Prolactinoma
Most common cause of hyperprolactinemia
Most common type of pituitary adenoma (up to 40%)
2. Medications (e.g., psychiatric medications, H2 blockers, metoclopramide, verapamil, estrogen)
3. Pregnancy
4. Renal failure
5. Hypothyroidism
Reference : step up to Medicine
9. Pt came with cough and she takes an anti-cholesterol medication (statins), she started it 3 weeks
ago, What should the doctor monitor?
A. Liver function test
Answer: a
explanation: LFT should be carried out before and within 4-6 weeks of starting statin therapy (1). Thereafter at intervals of 6 months to 1 year - earlier if clinical features of hepatotoxicity; also at the first review at 4-6 weeks - enquire about adverse effects such as itching, rash, myalgia, arthralgia, insomnia (1)
if satisfactory lipid control and no evidence of adverse effects then review again at 4-6 months,
then 6-12 monthly
if unsatisfactory lipid control then measurements should be repeated 6 weeks after dosage adjustments are made until the desired lipid concentrations are achieved (2)
however NICE state that LFTs only need to be measured on three occasions:
baseline liver enzymes should be measured before starting a statin. Liver function
(transaminases) should be measured within 3 months of starting treatment and at 12
months, but not again unless clinically indicated
people who have liver enzymes (transaminases) that are raised but are less than 3 times
the upper limit of normal should not be routinely excluded from statin therapy
treatment should be discontinued if serum transaminase concentrations rise to, and persist at, 3x normal range
10. Pt k/c of DM with uncontrolled blood sugar.. With figure shows high at the 6 am what to do?
a- long acting night insulin
b- short acting night insulin
c- long acting morning insulin
d- short acting morning insulin
Answer: A

smle ,2016

110

Explanation:
-We should increase the long acting night insulin as the patient is sleeping for hours and it wouldn't be
ideal to wake him/her up daily in the middle of the night to take medications.
- Cortisol among other counter-regulatory hormones such as epinephrine, glucagon, or growth hormone
are anti-insulin in the sense that they elevate Blood glucose, Cortisol will work mainly during the early
hours of the morning peaking especially around 6 AM.
- Adjusting the long acting insulin night dose should be able to adjust this physiologic hyperglycemia
The Dawn Effect: In the dawn hours, many people experience a release of counterregulatory hormones.
These hormones, such as human growth hormone (HGH), help with the growth, maintenance, and repair of our bodies while we sleep. They also make muscle tissue and the liver more resistant to the effects of insulin, meaning that more insulin is needed to maintain normal blood glucose levels. If there
isnt enough insulin to counteract the resistance, the result is morning hyperglycemia.
Re: http://www.todaysdietitian.com/newarchives/111412p18.shtml
11. Single Small thyroid nodule, investigations revealed an Increase in iodine uptake, what is the best
treatment ?
A. Conservative
B. Antithyroid drug
C. Iodine radiotherapy
Answer: c
http://bestpractice.bmj.com/best-practice/monograph/34/treatment.html
Rx of toxic nodule is Radiotherapy in the case scenario if:
If the patient is well and asymptomatic choose A
If the patient is symptomatic mild to moderate choose B
If the patient is severely symptomatic consider C or Surgical removal
Explanation:
Radioactive Iodine uptake test:
Treatment of Graves disease that is best will depend on many factors. Antithyroid drugs and RAI or a
mix of both often are preferred.
http://emedicine.medscape.com/article/924550-overview#a1
12. Patient with pheochromocytoma And high catecholamine in urine Initial medical management
A- ACEI
B-aldosterone blocker
Answer: alpha blocker : phenoxybenzamine

smle ,2016

111

All patients with pheochromocytoma need to undergo preoperative alpha-adrenergic blockade; we


suggest phenoxybenzamine as the first line drug other alpha blocker medication : ( prazosin ,
terazosin , doxazosin ) after alpha blocker is given we give beta blocker but never start beta blocker
before the alpha blocker surgical resection with early ligation of venous drainage is the treatment of
choice in pheochromocytoma. Reference: uptodate
13. K/c of DM pt on Glipizide want to go for elective surgery you want to control his blood
sugar during the surgery what to add ?
A. Insulin **
B. Metformin
C. Sulfonylurea
Answer: A
Patients with type 2 diabetes who take oral hypoglycemic drugs or noninsulin injectables are advised to
hold their oral hypoglycemic and noninsulin injectable drugs on the morning of surgery.For patients who
develop hyperglycemia, supplemental short or rapid-acting insulin may be administered subcutaneously
(typically every six hours), based on frequently (every one to two hours) measured glucose levels which
are often obtained on capillary "fingerstick" samples .
Reference: http://www.uptodate.com/contents/perioperative-management-of-blood-glucose-in-adultswith-diabetes-mellitus?source=preview&search=%2Fcontents%2Fsearch&anchor=H20#H20
14. A male patient known case of HTN on ACEi but with poor control, which drug you'll add?
a.
Thiazide
b.
Furosemide
c.
Vasodilator
Answer: A
15. Cushing case which skin manifestation is associated with it
A) Vitiligo
B)Telangiectasia
C)Acropathy
D)Something Derma
Answer:B
16. A patient presented with high parathyroid hormone and high calcium what is the diagnosis ?
a.primary parathyroid
b. secondary parathyroid
Answer: A
High ca + high PTH = primary hyperparathyroid

smle ,2016

112
Reference: medlineplus medical encyclopedia
17. Case of DKA with metabolic acidosis. What is the early mechanism to restore blood pH?
A. Excretion of CO2 through lungs.
B. Excretion of lactic acid through kidneys.
Answer: A
Reference:https://en.wikipedia.org/wiki/Acid%E2%80%93base_homeostasis
18. Old man with DM, has redness in calf area, raised and painful, tender:
a- cellulitis
b- diabetic neuropathy
answer: A
19. 50 years old patient with third reading of persistent hypertension wasn't started on medication
yet, lab shows high Na of 147 and low k of 3 other parameters were normal What's most likely the
diagnosis?
a.Essential hypertension
b.Hyperadrenalism
Answer:B
20. old patient with poor control of DM1, complains of SOB and hemoptysis . x:ray show lung consolidation . culture show non septate fungal hypha . what is the diagnosis?
A. Aspergillus
B. Candida
C. Zygomycetes
Answer: a
Opportunistic infection especially associated with diabetes; other predisposing factors are neutropenia,
corticosteroid therapy, iron overload and mucocutaneous trauma.
Large, non-septa hyphae with 90 degree angle branching and non-parallel walls, angioinvasive causing
tissue necrosis and hemorrhage
http://www.pathologyoutlines.com/topic/lungnontumormucor.html
21. Why is inulin used to measure GFR?
A- Freely filtered by glomeruli
Answer: A
Recall that all of the plasma that is filtered and only the plasma filtered is cleared of inulin so that if one
were to measure the clearance of inulin, it would equal the amount of plasma filtered in a minute, the
glomerular filtration rate. Therefore, the clearance of inulin is equal to the glomerular filtration rate,
the volume of plasma filtered in one minute.
Reference: http://www.austincc.edu/emeyerth/clearancehtm.htm
22. 20 yrs old girl her parent have dyslipidemia and she denies if have dyslipidemia lipid profile showing high triglyc,cholesterol,high LDL ,low HDL what is the best to check next ?
A. GH
B. TSH
C. FSH
D. ACTH

smle ,2016

113
Answer: TSH to rule out hypothyroidism or GH to rule out acromegaly

http://www.uptodate.com/contents/lipid-abnormalities-in-thyroid-disease
23. Best Type of carbohydrate for Diabetic pt & " Which type of Carbohydrates is better for diabetic patient":
A. Polysaccharides
Answer: Polysaccharides, a type of carbohydrate that includes starch and cellulose, may benefit people
with diabetes because they help retard absorption of glucose
http://www.sciencedaily.com/releases/2009/07/090728172604.htm
24. patient for surgery known case of DM2 on glimepiride you will shift patient to which drug during
and after surgery?
A-insulin
B-metformin
Answer: A ,
On the morning of surgery hold oral hypoglycemic agent because Sulfonylureas will increase the risk for
hypoglycemia,and metformin will increase the risk for lactic acidosis.
If the patient develop hyperglycemia give SC insulin, and correction insulin is given until the patient is
eating and either can resume oral agent.
Reference : uptodate "perioperative management of blood glucose in adult with DM"
25. Diabetic patient has history of weakness and dizziness. what anti-diabetes can cause it?
A. Sulfonylurea (glipizide)
B. Metformin
C. Thiazolidinediones (rosiglitazone)
D. Insulin
Answer:GLP-1

D (Symptoms are consistent with hypoglycemia)


Sulfonylurea side effects: Hypoglycemia Weight gain.
Metformin can cause unusual tiredness or weakness. Glucagon-like peptide (GLP)-1 analogue:
Exenatide or Liraglutide causes muscle weakness.
Pioglitazone, rosiglitazone and troglitazone can cause myalgia and muscle weakness.
References: Toronto Notes + Mayoclinic + Pubmed

smle ,2016

114
26. What is the mechanism of action of metformin on the cellular level?
A. decrease Muscle uptake of glucose
B. increase Muscle gluconeogenesis
C. Enhance muscle use of fatty acid oxidation
Answer :METFORMIN Sensitizes peripheral tissues to insulin
increases glucose uptake
Decreases hepatic glucose production by
simulation of hepatic AMP-activated
protein kinase (AMPK)
27. Most common cause of DKA in adult:
A. Missing insulin
B. Dietary
C. Increase physical activity
Answer: ?
The most common precipitating factor is infection, followed by noncompliance with insulin therapy.
Reference:http://www.aafp.org/afp/2005/0501/p1705.html
The 2 most common precipitating events are inadequate insulin therapy and infection. http://bestpractice.bmj.com/best-practice/monograph/162/basics/aetiology.html
28. Diabetic patient who is allergic to sulfa drugs, on metformin but its not controlled. What will you
add to control his diabetes?
A. chlorpropamide
B. glyburide
C. rosiglitazone
Answer: C or Gliptin?

smle ,2016

115

Gliptin (DPP-4 inhibitor) is a better choice (Answered by an endocrine consultant).


Chlorpropamide (sulfonylurea)
Glyburide (sulfonylurea)
Rosiglitazone (thiazolidinedione)

29. Obese patient recently diagnosed to have DM II. He is following a diabetic diet regimen and he
exercises regularly. When he came to you in the next visit... His blood sugar was high and he gained 5
kgs... He was also complaining of thirst and hunger, what would you give him:?
A. Metformin
Answer: A
30. What causes polyuria in DM?
A. Increase glucose in urine
B. Increase glucose in serum
C. Increase ketones in serum
Answer: A

smle ,2016

116

31. Young patient has hypertension, high Na and low K. What is the treatment?
A. Spironolactone
Answer: A
This patient has hyperaldosteronism. Spironolactone is a potassium sparing diuretic.
32. about pathophysiology of DM 1 I Don't remember the options
Answer:

33. 70 years old female patient with osteoporosis what is the treatment ?
A. ESTROGEN

smle ,2016

117
B. BIOPHOSPHATE
answer is B
- Bisphosphonates (e.g. alendronate) is first line
-HRT: second-line treatment (unless for vasomotor instability as well)
Toronto notes 2015 p 516
34. 52 years old woman recently diagnosed to have DM came with high ketones and hyperglycemia.
She was treated for it but she developed DKA again. The doctor is confused whether she have type 1
or 2 DM. What test should be ordered?
a.
Insulin
b.
HbA1c
c.
C-peptide
Answer: C
C-peptide is commonly used in preference to insulin measurement when assessing -cell function in clinical practice. In patients on insulin, C-peptide measurement must be used as exogenous insulin will be
detected by insulin assays.
Reference: http://www.ncbi.nlm.nih.gov/pmc/articles/PMC3748788/
35. What hormone increases body cells sensitivity and response to insulin?
a.
Leptin
b.
Lipase
Answer: A
Reference:http://www.ncbi.nlm.nih.gov/pmc/articles/PMC3602983/
36. 50 years old woman with no issues except for HgA1c 7.3, LDL and triglyceride are high. What is the
next test you want to order?
a.
TST
b.
LFT
Answer: B
In diabetic patients with clinical CVD or over age 40 years, statin therapy should be added to lifestyle intervention regardless of baseline lipid levels. Reference: UpToDate
All patients started on statins should have their AST and ALT tested as a matter of routine monitoring,
even if no symptoms are present.
Reference: Master The Board USMLE Step 2 CK
37. What is the best anti-diabetic regime that resembles the normal physiology?
a.
Lispro&glargine
b.
NPH &glargine
c.
Lispro& NPH
Answer: A
Glargine is better than NPH
Single injection of insulin glargine leads to a smooth 24-hour timeaction profile with no undesirable
pronounced peaks of activity. In clinical trials, this profile has been associated with at least equivalent, if
not better, glycemic control than other traditional basal insulins and a significantly lower rate of overall
and nocturnal hypoglycemia.
http://www.ncbi.nlm.nih.gov/pmc/articles/PMC1993975/

smle ,2016

118
38. DM on metformin and gliclazide .. she is not well controlled .. which drug u will add ?
A- Acarbose
B- pioglitazone
Answer: B
Reference: kumar and clarks
39. A patient with signs and symptoms of Cushing syndrome. What is the best next investigation?
a.
Brain MRI
b.
Adrenal scan
c.
Adrenal MRI
Answer: A
In adults, 80% of CS is due to ACTH-dependent causes and 20% due to adrenal causes. Since the majority
of patients with ACTH-secreting tumors have a pituitary lesion (often very small), a MRI of the pituitary
gland with gadolinium enhancement is always the initial approach.
References: http://www.ncbi.nlm.nih.gov/pubmed/18209870
40. Patient known case of Diabetes Type 2 suffer from recurrent hypoglycemia. Which drug responsible?
a. Sulphonylureas
Answer: A
Reference: Davidson's.
41. papillary thyroid ca mostly associated with which of the following ?
A-hrthle cell
Answer: A
Most common thyroid malignancy (80%)
Radiation exposure
Lymphatic Spread to cervical nodes
Prognosis: excellent- slow growth. 20 years survival
Tx: surgery resection or radiotherapy with to I131 if mts.
Histology: papillary pattern, psammoma bodies, clear nuclei orphan Annie eyes nuclei, intranuclear
cytoplasmic inclusions
Medscape:
another subclassification of Hrthle cell neoplasms has been proposed, namely the papillary variant of
Hrthle cell cancer (ie, Hrthle cell papillary thyroid carcinoma), in addition to Hrthle cell cancer and
adenoma. Clinically, tumors in this group tend to behave like papillary thyroid carcinoma; however, they
are more indolent, with a propensity for lymph node metastasis rather than hematogenous spread.
Hrthle cell carcinoma is a variant of papillary cell carcinoma.
42. (long scenario) 55 year old known diabetic patient came for checkup. What is the earliest effect of
Diabetes Mellitus on the kidney ?
A. Hydronephrosis with protein excretion.
B. Hydronephrosis with protein excretion.
C. Sclerosis with protein excretion.
D. Sclerosis with protein excretion.
Answer: A? Hyperfiltration with normoalbuminuria or protein excretion.
(I think they meant by hydronephrosis = hyperfiltration).

smle ,2016

119

References:
Toronto Notes +http://www.pathophys.org/ckd/http://emedicine.medscape.com/article/238946overview#showall
43. What is the difference between type I&II DM?
a.
Endogenous insulin secretion
b.
Weight
Answer: A

44. Case of polyuria, polydipsia and weight loss. Na: . What is the diagnosis?
A. DI
Answer: A (Depends on the case)

smle ,2016

120
Diabetes Insipidus (DI) clinical features: Passage of large volumes of dilute urine, polydipsia, dehydration; hypernatremia can develop with lack of access to water or impaired thirst mechanism. Reference:
Toronto Notes.
45. pt with DKA he start to breath rapidly to buffer his acidosis through
a- oxygen
b- carbon monoxide
c- carbon dioxide
d- nitrogen
Answer C
46. Which of the following is considered to be beneficial in DM and hypertensive patients?
a.ACEI
Answer: A
Hypertensive patients with DM and tight BP control at <150/85 mmHg by use of ACEI or B-blocker reduced risk of diabetic complications and death related to DM and
reduced risk of end-organ damage.
Reference: Toronto Notes.
47. Patient with 3 reading high BP all investigation normal except high Na What dx?
A-primary hyperaldosteronism
B.essential hypertension
C.Secondary hypertension
Answer: Primary hyperaldosteronism.
Mild serum hypernatremia in the 143-147 mEq/L range and mild hypomagnesemia from renal magnesium wasting are other associated biochemical findings in established primary aldosteronism
Reference:http://emedicine.medscape.com/article/127080-workup#c8
48. Young adult having episodic palpitation and fear and tightness . Btw the attack she feel fatigue?
What investigation ?
A- urine catecholamines test
Answer:A
Pheochromocytoma is a nonmalignant lesion of the adrenal medulla autonomously over producing catecholamines despite a high blood pressure.
Symptoms :
Hypertension that is episodic in nature
Headache
Sweating
Palpitations and tremor
Diagnostic Testing
Best initial tests:
High plasma and urinary catecholamine
Plasma-free metanephrine and VMA
Most accurate test:
CT or MRI of the adrenal glands
Metastatic disease is detected with an MIBG scan

smle ,2016

121
Treatment
Phenoxybenzamine (alpha blockade) first to control blood pressure.
Without alpha blockade, patients blood pressure can significantly rise intraoperatively.
Propranolol is used after an alpha blocker like phenoxybenzamine.
Surgical or laparoscopic resection. Metastatic disease cannot be treated with surgery.
Metastatic disease is treated medically.
Reference : Master The Board
49. DM type 1 which of these confirm dx ?
a. acetone
Answer: A
50. acetone + high glucose
A. DKA
B. hyperosmolar
Answer: A
51. diabetic patient come with fatigue and drowsiness , which drug group responsible for that :
Answer: sulphonylurea
52. case of hyperthyroidism clear but with bradycardia
A. Hypothyroidism
B. Hyperthyroidism
C. Goiter
D. Neoplasm
Answer:C
53. Patient with fracture, constipation and abdominal pain, high serum CA:
A. Hyperparathyroidism
Answer: A
source: Master the boards USMLE step 2 CK
54. man want to lose weight have hx of DM he is on regular exercise & low calorie diet but his weight
not decreasing what will you advise him:
A. Intense exercise
B. Use medication to loss wt
C. Wt bearing exercise
D. Low calori
Answer: B use medication to loss wt
55. female DM on metformin and sulfonylurea can't tolerate them what you will do?
A. Insulin
B. Glitazone
Answer: B

smle ,2016

122
56. a girl with type 1 DM diagnosed 3 years ago , she is 12 years did not show sign of puberty .. What
should she repeat annually :
A. growth hormone
B. celiac testing
C. ophthalmology exam
Answer: A
57. Q about. Cretinism? What wou will give?
A. Iodin
B. Thyroxine
Answer: B
58. Pt with hyperthyroidism on treatment. c/o recurrent infections. What is the
Drug?
A. Methimazole
Answer : A
59. Q. HTN on treatment now has gout, what drug?
A. Hydroxy Thiazide
Answer : A
60. A scenario of a patient with HTN came with headache and anxiety and have 3 previous vistas of
High blood pressure and treated for it then we need to stop because of the patient had a hypotension
what you will do
A. 24 urine metanephrines (pheochromocytoma )
B. TSH
C. ACTH
Answer: A
61. Diabetic and hypertensive, was given a drug then experienced cough, improved after changing the
drug to hydrochlorothiazide what to add:
A. Atenolol
B. Amlodipine
C. Losartan
D. ACEI (I forgot the name)
Answer: C
62. Diabetic woman with hypothyroidism developed oral white plaque (thrush) for long time with recurrent rash Ppd test positive with old exposure What's the diagnosis :
A. Chronic mucocutaneous candida infection <disease associated with DM,
hypothyroid
B. IgM deficiency Digeorge disease
C. Chronic granulomatous ....
Answer: Chronic mucocutaneous candida infection
63. DKA pt if fluids corrected rapidly. What the complication?

smle ,2016

123
A. brain edema
Answer: A
64. Old pt, diabetic dehydrated, lab high Na low k, high bicarbonate, ketones in urine, Dx ?
A. metabolic syndrome
B. diabetic ketoacidosis
C. lactic acidosis
Answer: B
65. What is the characteristic of Fragile X syndrome ?
A. Dilated Pulmonary
Answer :
66. Female c/o clear fluid from nipple. Breast examination was normal. Brain MRI: pituitary mass. This
mass secrets ?
A. Prolactin
Answer: A
67. DM with high BP and deteriorating renal what to give?
A- ACEI (lisinopril)
Scientists have made great progress in developing methods that slow the onset and progression of kidney disease in people with diabetes. Drugs used to lower blood pressure can slow the progression of kidney disease significantly. Two types of drugs, angiotensin-converting enzyme (ACE) inhibitors and angiotensin receptor blockers (ARBs), have proven effective in slowing the progression of kidney disease.
Many people require two or more drugs to control their blood pressure. In addition to an ACE inhibitor
or an ARB, a diuretic can also be useful. Beta blockers, calcium channel blockers, and other blood pressure drugs may also be needed.
An example of an effective ACE inhibitor is lisinopril (Prinivil, Zestril), which doctors commonly prescribe
for treating kidney disease of diabetes. The benefits of lisinopril extend beyond its ability to lower blood
pressure: it may directly protect the kidneys' glomeruli. ACE inhibitors have lowered proteinuria and
slowed deterioration even in people with diabetes who did not have high blood pressure.
An example of an effective ARB is losartan (Cozaar), which has also been shown to protect kidney function and lower the risk of cardiovascular events.
http://www.medicinenet.com/script/main/mobileart.asp?articlekey=101334&page=6
68. 42 years old female complaining of amenorrhea, night sweat and flushing for the last 6 months.
What is the most likely diagnosis?
A. Hypothyroidism
B. Hyperprolactinemia
C. Congenital adrenal Hyperplasia
D. Pheochromocytoma
Answer : A
Primary hypothyroidism will lead to decreased level of T3, T4 and increase level of TSH. TSH and FSH
both of them have the same alpha unit, so when level of TSH is increased it will go and attach to FSH
receptor and will work like FSH, this will lead to appear of menopause symptoms.

smle ,2016

124
69. Adult, collapse, high glucose , Ketone in urine, what's the diagnosis ?
A. DKA
Answer: A
70. Case of congenital adrenal hyperplasia ;
A. give steroid
Answer: A
71. Military soldier want to deceive his boss, got hypoglycemic symptoms, what did he use?
A. Insulin before exercise
B. Metformin before exercises
Answer : A
72. Pt heard about the new anti-DM medication Incretin on tv, asking about the mechanism of action of the new drug?
A. Inhibit liver gluconeogenesis
B. Increases the insulin secretion
C. Decreases insulin resistance
Answer: A
73. What type of insulin is used in DKA?
A. Regular
B. Glargine
C. NPH
Answer:A
74. Well controlled DM, BP 3 readings 138/82, what to do next?
A- Nothing
Answer: A * or if there is Reassess yearly *
For patients with diabetes, renal impairment or established cardiovascular disease a lower target of
130/80 mmHg is recommended.
But the treatment should be started if the BP 140-159/90-99
Treated with ACEI or Angiotensin II receptor blockers

smle ,2016

125

Reference: Kumar and Clarks Clinical Medicine


75. Anti diabetic drug taken by ladies with PCOS:
A. Metformin
Answer: A
Insulin sensitizers (e.g., metformin alone or with clomiphene citrate) may be used for PCOS patient to
reduce insulin resistance, control weight, and facilitate ovulation.
Reference: Beckmann, Obstetrics and Gynecology
76. 46 y/o male with no risk factors for diabetes and with normal BP and normalfasting glucose and
normal HbA1c (I forget what the level, but for sure itwasn't in the prediabetic or diabetic range).
When will the next screen be?
A. In 6 months
B. In 12 months
C. In 36 months
Answer: C * If they mean screen for type 2 diabetes *

smle ,2016

126
Type 2 diabetes screening should be performed in adults of any age who are overweight or obese, and
who have one or more diabetes risk factor
Testing should begin at age 45
If test is normal? Repeat it at least every 3 years
Reference: http://www.ndei.org/uploadedFiles/Common/NDEI/Treatment_Guidelines/ADA%202015%20Summary%20PDF.pdf
77. pt Dx with DM1 which will confirm the Dx:
A. DR3
B. DR4
C. DR7
D. DR5
Answer: A or B * both are right *
Reference: Kumar and Clarks Clinical Medicine, Toronto
Notes
78. diabetic with painful back swelling has multiple discharges:
A. Cellulitis
B. Lymphoid
C. something Lymphadenitis Carbuncle
Answer:
79. known case of DM1 , last insulin time not known ,
was playing football , lose his consciousness for minutes
, what will u give him :
A. D5 with
B. ringer lactate with ..
C. subcutaneous insulin

Answer: A
The incidence of hypoglycemic episodes in patients with type 1 diabetes mellitus (DM1) is frequent. Hypoglycemia occur as a complication of therapy for diabetes. IV dextrose (25 g of 50 percent glucose [dextrose]) can be administered to treat hypoglycemia in patients with impaired consciousness and established IV access
Reference: http://www.uptodate.com/contents/management-of-hypoglycemia-during-treatment-ofdiabetes-mellitus?source=search_result&search=hypoglycemia&selectedTitle=4~150
80. Patient presented with short stature, he has growth hormone deficiency as the lab shows low GH
what else you would measure :
A. Somatomedin C
B. Glucose
C. Prolactin
Answer: A

smle ,2016

127

81. A 30-year-old male, diabetic otherwise healthy with persistent one hour post prandial hyperglycemia after lunch and dinner:
A. Milk
B. Meat
C. Diet cola
D. Potato
Answer: D
Foods with a high glycemic index (greater than 70), such as cold cereals, bread, potatoes, rice, and snack
chips, tend to raise blood glucose the fastest, with a significant peak occurring in 3060 minutes. For
these types of foods, it is best to bolus 1520 minutes prior to eating. This will allow the insulin peak to
coincide as closely as possible with the blood glucose peak.
Reference: http://www.health.harvard.edu/diseases-and-conditions/glycemic_index_and_glycemic_load_for_100_foods , http://www.diabetesselfmanagement.com/managing-diabetes/blood-glucose-management/strike-the-spike/
82. Patient presented with acne, upper body obesity and hypertension , what is the test to diagnose
this patient ?
A. Anabolic stress test
B. Urine metanephrine
Answer: A
Cushings syndrome
83. Stones multiple, high Ca:
Answer: hyperparathyroidism
Hypercalcaemia: If the GFR is normal, hypercalcaemia almost invariably leads to hypercalciuria. The
common causes of hypercalcaemia leading to stone formation are:
Primary hyperparathyroidism
Vitamin D ingestion
Sarcoidosis
Reference: Kumar and Clarks Clinical Medicine
84. Breast cancer metastasis to the lung before mastectomy pt is depressed ,increase urination ,
thirsty all the time , lab provided Na low , urine osmo lowDiagnosis , what is appropriate to give:
A. Psych polydipsia
B. SIADH
C. Diabetes insipidus

smle ,2016

128
Answer: A

Reference:
one high yield

step

85. diabetic controlled on medication developed (eye down and out) dropped upper eyelid limited
movement of eye medially and up and down
A- Right oculomotor palsy
B- right facial palsy
Answer: A
86. T score of -3.5?
A. Osteoporosis
Answer: A
87. Diabetic pt on metformin
and gliclazide not control
what to add :
A. acarbose
B. Pioglitazone

smle ,2016

129

B*
was

Answer:
if
there
no
basal

insulin*
Reference:
Up-

todate
88. Pt has osteoporosis on ca and vit D But no improvement why?
A. hypomagnesemia
Answer:
89. Melanocyte stimulating hormone released from?
A. intermediate lobe of pituitary!
Answer: A
In most vertebrates, melanocyte-stimulating hormone (MSH) peptides are secreted specifically by the
intermediate lobe of the pituitary gland
Reference: http://global.britannica.com/science/melanocyte-stimulating-hormone
90. Most common cause of secondary hypertension
a) Kidneys disease
b) Cons syndrome
Answer: A
Reference: Toronto Note
91. Pt with small lung ca and undergoing
chemo , developed ... (I don't remember),but lab values "low blood Na" and
"low urine osmolality" the treatment is ?

smle ,2016

130
a) Desmopressin*
b) Nothing
Answer: A
Acute symptomatic hyponatremia occurs in <48 hours when fluid intake is rapid and far above normal
levels, resulting in cerebral edema. Chronic hyponatremia may be asymptomatic due to CNS adjustment
to low sodium levels.
Patients with chronic symptoms and no or mild hyponatremia should be treated with fluid restriction.
For those with dilute urine (urine osmolality <200 mOsm/kg H2O), fluid restriction should begin at 1 to
1.5 L/day. However, for patients with a urine osmolality >200 mOsm/kg H2O, fluid restriction to 0.8 to
1.5 L/day is recommended.
Regular patient weights are often used to determine water intake diurnally. Differences in diurnal
weight gain in PPD patients can be much greater than those in controls (2.2% versus 0.6%).
Concomitantly giving loop diuretics to enhance water excretion (e.g., furosemide) may be necessary. Furosemide preferentially causes water excretion over sodium excretion
Reference: https://online.epocrates.com/diseases/86541/Psychogenic-polydipsia/Treatment-Approach
92. 35 y/o pt diagnosed with DM .. you advise him. To do aerobic exercise How much minutes per
week?
A. 120
B. 180
C. 150
Answer: C
Reference: http://www.diabetes.org/food-and-fitness/fitness/types-of-activity/what-we-recommend.html?referrer=https://www.google.com.sa/
93. Patient increase foot size 39 >> 41.5 and increase size of hand and joint which hormone?
a) Thyroxine
b) Prolactin
c) ACTH
d) Somatotropin hormone known as Growth Hormone
Answer: D
94. 42 obese with normal HA1C plus normal glucose level then to screen again >
A. 3 years or 36 Months
Answer: A
C * If they mean screen for type 2 diabetes *
Type 2 diabetes screening should be performed in adults of any age who are overweight or obese, and
who have one or more diabetes risk factor
Testing should begin at age 45
If test is normal? Repeat it at least every 3 years
Reference: http://www.ndei.org/uploadedFiles/Common/NDEI/Treatment_Guidelines/ADA%202015%20Summary%20PDF.pdf
95. Hypoglycemia is more with
A. glyburide
B. acarbose

smle ,2016

131
Answer: A
96. What's the Anion gap Na= 138/?
97. rapid ttt of hyperthyroid
A. Beta blockers
Answer:A
-blockers often necessary for symptomatic treatment prior to definitive therapy (TORNTO NOTE )
98. Pt with 3 reading of high BP did not start on meds yet Showing labs all normal except high K what
is the most likely cause?
A. essential hypertension
B. Hyperaldosteronism
C. pheochromocytoma
D. dont remember but irrelevant
Answer: -

99. man was has


cholecystectomy now
complaining of
unilateral parotid
swelling ,he has hx of mumps, facial nerve intact ,no decrease in salivation ,Lab was show cloudy saliva?
A. sarcoidosis granuloma
B. sialadenitis
C. parotid cancer
D. mumps
Answer: B
Reference: http://emedicine.medscape.com/article/882358-clinical
100. SE Female irritable + loss of wt+ bradycardia=>
A. Hypothyroid
B. Hyperthyroid
C. nodule

smle ,2016

132
D. thyroid cancer
Answer: -

101. DKA
management?
AFluids.
B- Insulin infusion.
answer: A
Managing diabetic ketoacidosis (DKA) in an intensive care unit during the first 24-48 hours always is advisable. When treating patients with DKA, the following points must be considered and closely monitored:
Correction of fluid loss with intravenous fluids
Correction of hyperglycemia with insulin
Correction of electrolyte disturbances, particularly potassium loss
Correction of acid-base balance
Treatment of concurrent infection, if present
Ref.:http://emedicine.medscape.com/article/118361-treatment
102. Incretin role in dm
Increase insulin sensitivity Stimulate insulin secretion (the answer ) Be careful it increase the secretion
the play with phrasing you get confuse
103. In case of thyroid crisis what's the initial step
A. Radioactive
B. thyroid Iodine
C. thyroid Propranolol
Answer: C
Treatment is urgent. Propranolol in full doses is started immediately together with potassium iodide,
antithyroid drugs, corticosteroids (which suppress many of the manifestations of hyperthyroidism) and
full supportive measures. Control of cardiac failure and tachycardia is also necessary.
Reference: Kumar and Clarks Clinical Medicine
104. Diabetic and hypertensive male with erectile dysfunction doctor prescribed phosphodistrase
inhibitor , what he should avoid :

smle ,2016

133
A. steroid
B. Abx
C. Morphine
D. Nitrat
Answer: D
Reference: Toronto Notes
105. patient with symptoms of hypothyroidism and excessive thirst and urination brain MRI report
contain a lot of details which finally suggest pituitary tumor? you put her on thyroxine what drug you
will add:
A. Insulin
B. Vasopressin
C. Somatostatin
D. Analogues
Answer: C
* Diabetic insipidus*
For other pituitary tumor complication:
For apoplexy: rapid corticosteroid administration surgical decompression
For prolactinoma: dopamine agonists (e.g. bromocriptine)
For Cushings: serotonin antagonist (cyproheptadine), inhibition of cortisol production (ketoconazole)
For acromegaly: somatostatin analogue (octreotide) bromocriptine
Endocrine replacement therapy
Reference: https://neurosciences.uvahealth.com/services/pituitary-tumors/conditions-treatments/pituitary-adenomas, Medscape,Toronto Notes
106. Treatment of gestational DM is?
Answer: insulin
1st line of treatment: Diet, exercise and glucose monitoring if uncontrolled insulin therapy
Reference: BMJ
107 . Which antidiabetic medication cause weight gain?
A. Sulfonylurea
Answer: A
Reference: Toronto Notes
108. female patient treated for hyperlipidemia investigation shown increase in CPK? what is the cause
?
A. atorvastatin
B. niacin
Answer: A
Reference: https://www.atrainceu.com/course-module/1552780-76_statins-controlling-cholesterolmodule-5
109. patient presents with bilateral exophthalmos and eyelid injection and intermittent ptosis ,auscultation shows thyroid bruit , normal vital sign ,,there is some important information?
A. retinal vein ..

smle ,2016

134
B. hyperthyroidism
Answer: B *Graves' disease*
Clinical Features:
Signs and symptoms of thyrotoxicosis
Diffuse thyroid goiter thyroid bruit secondary to increased blood flow through the gland
Ophthalmopathy: proptosis, diplopia, conjunctival injection, corneal abrasions, periorbital puffiness,
lid lag, decreased visual acuity if Graves' (plus signs of hyperthyroidism: lid retraction, characteristic
stare)

Reference: Toronto Notes


110. Patient missed his insulin injection. Most likely to be found in urine analysis?
A- Ketones.
B- Proteins.
Answer : A
111. thyroid disease associated with papillary ca :
A. Hashimoto
B. Riedel thyroiditis
Reference: American thyroid association
112. osteoporosis prevention..
phosphate +vit D +ca Diet
113. 61 y female known case of osteoarthritis, came for regular check up, not taking ca supplements
nor high ca diet, She is a high risk of osteoporosis What is the best initial thing before deciding the appropriate mx ?
a)
DECADE SCAN
b)
oral ca, vit D, bisphosphonate
c)
TSH,Ca,vit D
d)
Intranasal calcitonin, CA, vit Dr

smle ,2016

135
Answer : b
114. pregnant has Gestational DM .. , what is her risk for future ?
A. DM type 2
B. DM type 1
Answer : A
Reference: master the board P466
115. High density bone in dexa With scheduled total hip replacement
A. osteoporosis
B. osteomalacia
C. osteoarthritis
D. paget disease
Answer: ??????
116. Diabetic on metformin+glib , uncontrolled
Answer: "No insulin in choices "

117. A Lot of Q about the best type of insulin to give/side effects of each of them:
Know the anti-diabetic medications well.
118. there is q about hyperaldosteronism
Answer:

smle ,2016

136

1 Hyperaldosteronism (hyporeninemic)

Adrenal hyperplasia (idiopathic) or Conns syndrome

HTN, hypokalemia, alkalosis ,hypernatremia


2 Hyperaldosteronism
Reduced blood flow to JGA -> Increased renin secretion
Increased plasma renin

119. Lipid profile of a patient shows high level and patient is on simvastatin What to add?
Answer:
UpToDate: Thus, in patients who do not achieve a particular LDL-C goal on statin therapy alone, we suggest not adding a non statin lipid-lowering medication for primary prevention. That is, the patient should
be maintained on statin therapy as his/her only lipid-lowering medication.
If the patient has high triglyceride level we can add a fibric acid derivative but with caution increased
risk of myositis
if the patient has persistent high LDL we can add ezetimibe although it has no mortality benefit but it is
an LDL lowering agent
MTB
120. Head trauma with low urine osmolality?
Diabetes insipidus (DI)
o DI is rare, with an estimated 1 case per 100,000 hospital admissions. Posttraumatic DI occurs in 216% of all cases. The most common etiologies of posttraumatic DI include severe closed head injury,
frequently with basilar skull fractures; craniofacial trauma; thoracic injury; postcardiopulmonary arrest; and intraventricular hemorrhage in neonatal patients. DI frequently is associated with cranial
nerve injuries. The usual onset is 5-10 days following trauma.
o Characteristic features of DI include polyuria, low urine osmolality, high serum osmolality, normal
serum glucose, and normal to elevated serum sodium. Urine output usually is greater than 90
mL/kg/d, with a specific gravity of less than 1.010 and an osmolality of 50-200 mOsm.
Reference:http://emedicine.medscape.com/article/326123-clinical#b5
121. Q about young boy present with Abd pain and vomiting??
DKA **
Answer: very short question cant decide but the answer depends on the clinical scenario,
Common causes of abdominal pain include constipation, gastrointestinal (GI) infections, infections outside of the GI tract, and colic.
Less common GI conditions (ie, inflammatory bowel disease, pancreatitis, cholecystitis, intraabdominal
abscess, dietary milk protein allergy, malabsorption, and Meckel's diverticulum)
and conditions outside of the GI tract (ie, diabetic ketoacidosis, painful crisis with sickle syndromes,
Henoch Schnlein purpura (IgA vasculitis), tumors, urolithiasis, ovarian torsion, testicular torsion, and
some toxic ingestions) can present with abdominal pain.
Reference:http://www.uptodate.com/contents/causes-of-acute-abdominal-pain-in-children-and-adolescents?source=preview&search=%2Fcontents%2Fsearch&anchor=H30#H30
please read here for more specific classification according to age:
http://www.uptodate.com/contents/image?imageKey=EM/65488&topicKey=EM%2F6454&source=outline_link&search=%2Fcontents%2Fsearch&utdPopup=true

smle ,2016

137

122. Question about Addison disease:


Answer:
123. In addition to anti- HTN , what to advice ?
Restrict Na to 9mg ,Walking ( such meters)
124. Diabetic patient on metformin and another drug but still blood sugar not controlled. He is allergic
to sulfa, what to add?
Answer: ?
Metformin should be used as initial therapy for type 2 diabetes unless contraindications exist. Second-line agents include sulfonylureas, DPP-4 inhibitors, GLP-1 receptor agonists, meglitinides, and -glucosidase inhibitors. TZDs are no longer recommended because of potential increases in cardiovascular
risk and fracture risk.
The incretin agents are a reasonable second choice if the main problem is PPG
elevation and if hypoglycemia and weight gain are concerns. If FPG and postprandial glucose are elevated and if cost is a concern, sulfonylureas are a reasonable second- line agent.]
If a patient has symptomatic hyperglycemia, poor control despite two to three oral agents, or an A1C >
8.5%, insulin should be considered.
Reference:http://m.clinical.diabetesjournals.org/content/30/2/72.full
125. Scenario about diabetic what is the Tx
Metformin
Reference: master the board P125
126. primary hyperparathyroidism what will be the lab results:
A- calcium and PTH both high
Primary hyperparathyroidism:
high Ca
high PTH
low phosphate
Reference: master the board P 117
127. primary hyperparathyroidism what will be the lab results(repeated Que)
calcium and PTH both high
128. patient with metabolic acidosis with high anion gap (aspirin toxicity)
Activated charcoal and Alkaline diuresis with extra KCl
Reference: master the board P535-536

smle ,2016

138

129. Type of HTN in Adolescent:


Essential
130. Patient diagnosed DM and complained on medication but he complain of
hypoglycemia more than 3 times ..... Cause ,?

132. What come with Turner syndrome ?


A.Hypothyroid
B.DM
C.Addison's
Answer: A

smle ,2016

139
Reference:medscape
132. Bisphosphonate which drug make bone clast ???
133. patient with symptoms of hyperthyroidism and her T4 is high and low TSH ... pathophysiology
TSH is low because of ve feedback from increased level of circulating T3 and T4
Reference: Toronto note
134. Diabetic patient on metformin has sulphonylurea allergy need to be on another extra drug for
control which one you choose(repeated Que)
135. x-ray of lower back , investigation showed high ca+ , what will u do :
A. Dexa
B. Parathyroid
Answer : B

136. What is the preferable imaging to diagnose pituitary microadenoma?


A) Contrast with enhanced brain CT
B) Brain CT
C) Contrast with enhanced brain MRI
D) Unenhanced brain MRI
Answer : C
Reference: UpToDate , Usmle 2 first aid , medscape

smle ,2016

140
137. Pheochromocytoma ..what is the initial tx ?
A. Alpha antagonist .
Answer : A
Reference: master the board P124
138. Patient diagnosed DM and complained on medication but he complained of hypoglycemia more
than 3 times Cause ,?(repeated Que)
A. Honeymoon?????
http://www.diabetes.co.uk/blood-glucose/honeymoon-phase.html

PULMONOLOGY
1.

An asthmatic who needs daily short acting beta 2 inhalers, oralsteroids and daily spirometry monitoring of PFTs. What is his asthma stage?
a.Mild intermittent
b.Mild persistent

smle ,2016

141
c.Moderate
d.Severe
Once the patient on oral steroid he/she classified as a severe asthma.
Reference: 1.http://www.med.umich.edu/1info/FHP/practiceguides/asthma/EPR-3_pocket_guide.pdf
2. Toronto note
2. Bilateral pneumonia treatment ??????? not sure
Answer : virus pneumonia or aspiration pneumonia
viral pneumonia : antiviral
aspiration pneumonia : clindamycin
3.

most common cause of chronic cough in adult?


a GERD
b -postnasal drip
c asthma
Reference: http://www.uptodate.com/contents/chronic-cough-in-adults-beyond-the-basics?source=outline_link&view=text&anchor=H3#H3
4.

What is the most specific investigation for TB ?


A. Sputum culture
B. PPD
Answer:
5. Cancer of lung with high keratin ?
Squamous cell carcinoma
Reference:1. http://www.ncbi.nlm.nih.gov/pmc/articles/PMC3155291/
2.http://misc.medscape.com/pi/iphone/medscapeapp/html/A279960-business.html#a3
6.

Calcified lesion in the upper lung?


A. Bronchoscope biopsy
B. Percutaneous biopsy
C. Thoracotomy
D. Follow up with serial x-ray
if there is HRCT it is the right answer
1.
http://www.clevelandclinicmeded.com/medicalpubs/diseasemanagement/hematology-oncology/pulmonary-nodules/
2.http://www.ncbi.nlm.nih.gov/pmc/articles/PMC2883201/#!po=27.6316
7. Most common cause of obstructive sleep apnea ?
Obesity is the most commonly identified cause of obstructive sleep apnea. Patients present with daytime somnolence and a history of loud snoring.
Other symptoms include:
Headache
Impairedmemoryandjudgement Depression
Hypertension

smle ,2016

142
Erectiledysfunction
Bullneck
The most accurate test is polysomnography (sleep study) which shows multiple episodes of apnea. Arrythmias and erythrocytosis are common.
Reference: master the board Page 164
8.

Tb case, what is the next appropriate step to get a definitive dx?


A. Sputum smear under microscope
B. Sputum culture
The presence of acid-fast-bacilli (AFB) on a sputum smear or other specimen often indicates TB disease.
Acid-fast microscopy is easy and quick, but it does not confirm a diagnosis of TB because some acid-fastbacilli are not M. tuberculosis. Therefore, a culture is done on all initial samples to confirm the diagnosis.
(However, a positive culture is not always necessary to begin or continue treatment for TB.) A positive
culture for M. tuberculosis confirms the diagnosis of TB disease.
Reference: CDC
9. Pt have lesion in right upper lung look like calcium how to treated ?
Answer:
The first step in the evaluation of a pulmonary nodule is to look for a prior x-ray.
Finding the same pulmonary nodule on an x-ray done years ago may save you
from doing any further workup. If no prior x-ray is available, then consider
whether this patient is high or low risk for lung cancer.

In low-risk patients, <35 years of age and nonsmokers with


calcified nodules, you may follow
the patient with chest x-rays or chest CT every 3 months for 2
years. Stop the follow-up if after
2 years there is no growth.

High-risk patients >50 years of age with a smoking history and


a nodule are likely to have bronchogenic cancer. The best diagnostic procedure is open-lung biopsy and removal of the nodule
at the same time.
Reference : 1.Kaplan Internal Medicine 2013
2.Master the board page156-157
10. Pt with barking cough and 38 temp which of the following symptoms is associated with this disease ?
Answer:Croup usually begins with nonspecific respiratory symptoms (ie, rhinorrhea, sore throat, cough).
Fever is generally low grade (38-39C) but can exceed 40C. Within 1-2 days, the characteristic signs of

smle ,2016

143
hoarseness, barking cough, and inspiratory stridor develop, often suddenly, along with a variable degree
of respiratory distress. Symptoms are perceived as worsening at night, with most ED visits occurring between 10 pm and 4 am. Symptoms typically resolve within 3-7 days but can last as long as 2 weeks.
Spasmodic croup (recurrent croup) typically presents at night with the sudden onset of "croupy" cough
and stridor. The child may have had mild upper respiratory complaints prior to this, but more often has
behaved and appeared completely well prior to the onset of symptoms. Allergic factors may cause recurrent croup due to respiratory epithelial changes from the viral infection.
Reference: http://emedicine.medscape.com/article/962972-clinical
11. Cancer of lung with high keratin ?
Answer: Non small cell cancer
Answer: Sq C C
According to the book it'sseq cell carcinoma:
https://books.google.co.uk/books?id=KZazAQAAQBAJ&pg=PA419&lpg=PA419&dq=Cancer+of+lung+with+high+keratin&source=bl&ots=7y0dnGMLzf&sig=grrvlAQfdvP3ch5lupGVw5OiLbY&hl=ar&sa=X&ved=0ahUKEwi2sq6
FhfHNAhWjLMAKHddZCEw4ChDoAQgrMAE#v=onepage&q=Cancer%20of%20lung%20with%20high%20keratin&f=false
12. Calcified lesion in the upper lung?
A. Bronchoscope biopsy
B. Percutaneous biopsy
C. Thoracotomy
D. Follow up with serial x-ray
Answer: D
In asymptomatic pts with calcified nodule, we follow the pt with serial x ray for about 3months, if it is
not changed without Sx development, the calcification is mostly
benign.
The first step in the evaluation of a pulmonary nodule is to look for a prior x-ray. Finding the same pulmonary nodule on an x-ray done years ago may save you from doing any further workup. If no prior xray is available, then consider whether this patient is high or low risk for lung cancer.
-In low-risk patients, <35 years of age and nonsmokers with calcified nodules, you may follow the patient with chest x-rays or chest CT every 3 months for 2 years. Stop the follow-up if after 2 years there is
no growth.
-High-risk patients >50 years of age with a smoking history and a nodule are likely to have bronchogenic
cancer. The best diagnostic procedure is open-lung biopsy and removal of the nodule at the same
time.(USMLE step 2CK)
13. Asthma on montelukast and Bronchodilator has dry cough every day came to ICU, what to give
for long term?
Answer: high-dose inhaled corticosteroid plus a leukotriene receptor antagonist plus an oral corticosteroid. Consider omalizumab for patients who have allergies.Medscape
Treatment Asthma is managed in a stepwise fashion of progressively adding more types of treatment if there is no response.
Step 1. Always start the treatment of asthma with an inhaled short-acting beta agonist (SABA) as
needed. Examples of SABA are: Albuterol Pirbuterol Levalbuterol

smle ,2016

144
Step 2. Add a long-term control agent to a SABA. Low-dose inhaled cortico- steroids (ICS) are the best
initial long-term control agent. Example of ICS are: Beclomethasone,budesonide, unisolide, uticasone,mometasone, triamcinolone Alternate long-term control agents include: Cromolynandnedocromiltoinhibitmastcellmediatorreleaseandeosin- ophil recruitment eophylline Leukotrienemodiers:montelukast,zarleukast,orzileuton(bestwith atopic patients)
Step 3. Add a long-acting beta agonist (LABA) to a SABA and ICS, or increase the dose of the ICS. LABA
medications are salmeterol or formoterol.
Step 4. Increase the dose of the ICS to maximum in addition to the LABA and SABA.
Step 5. Omalizumabmay be added to the SABA, LABA, and ICS in those who have an increased IgE level.
Step 6. Oral corticosteroids such as prednisone are added when all the other therapies are notsucient to
control symptoms. Adverse Effects of Systemic Corticosteroids ey should be used as a last resort because of very harsh adverse e ects such as: Osteoporosis Cataracts Adrenalsuppressionandfatredistribution Hyperlipidemia,hyperglycemia,acne,andhirsutism(particularlyin women) inningofskin,striae,andeasybruising (reference: master the board)
14. cystic fibrosis mode of inheritance?
Answer: autosomal recessive (AR)
15. A patient is coughing bloody frothy sputum. He has pulmonary edema, + hepatojugular reflux and
lower limb edema. Capillary pressure is 3 times more than oncotic pressure. What is the type of
edema ?
A. Venous
B. Arterial
C. Interstitial
D. Capillary
Answer: C
Pulmonary edema in heart failure patient caused by increase of pulmonary venous pressure lead to pulmonary venous distention and transudation of fluid. Also, lead to pulmonary capillaries rupture.
Extra information:

Reference: Rapid Review Pathology, 4th edition


16. According to the new classification of lung cancer, which of the following is considered carcinoma
in-situ?
A. Adenocarcinoma less than 2 cm.
B. Atypical hyperplasia
Answer: A :
Adenocarcinoma in situ (AIS) with no invasive features is a localized, small (3 cm) adenocarcinoma with
growth restricted to a non invasivelepidic pattern and an absence of papillary or micropapillary patterns
or intraalveolar tumor cells. Reference: UpToDate.

smle ,2016

145

17. A male patient who is a smoker, developed symptoms. Ca: High. CXR showed solitary nodule.
What is the most likely diagnosis?
A. Squamous Cell Carcinoma SCC
B. Adenocarcinoma
Answer: A
High Ca > Paraneoplastic of SCC.
http://www.cancerresearchuk.org/about-cancer/coping-with-cancer/coping-physically/calcium/high-calcium-in-people-with-cancer
18. Asymptomatic Patient. Chest X ray shows a unilateral calcified nodule on the upper zone of his
lung?
A. Adenoma
B. Granuloma
C. Hamartoma
D. SCC
Answer: c

The most common cause of nodule calcification is granuloma formation, usually in the response to
healed infection.
Reference:http://radiopaedia.org/articles/calcified-pulmonary-nodules
Granuloma is account 25% and 15% of all benign causes, respectively. Active granulomatous infections
include tuberculosis, coccidioidomycosis, histoplasmosis, cryptococcus, and aspergillosis. Hamartomas
comprise an additional 15% of benign lesions
Reference:http://www.clevelandclinicmeded.com/medicalpubs/diseasemanagement/hematology-oncology/pulmonary-nodules/
19. Patient with shoulder pain and pleurisy. Which part of the pleura causes radiation of the pain to
shoulder:
a.
Visceral
b.
Mediastinal
c.
Costal
d.
Anterior
Answer: B
Visceral pleura: insensitive to pain due to autonomic innervation.
Parietal pleura:
Costal and peripheral parts of diaphragmatic pleura are referred along intercostal nerves to thoracic and abdominal wall.
Mediastinal and central diaphragmatic pleural pain referred to root of neck and over shoulder
(Dermatomes C3-C5). Reference: Lippincott's Concise Illustrated Anatomy: Thorax, Abdomen &
Pelvis.
20. Most common cause of excessive sleepiness in the daytime is?

smle ,2016

146
A. Narcolepsy
B. Obstructive Sleep Apnea (OSA)
Answer: B
The most common causes of excessive daytime sleepiness are sleep deprivation,
obstructive sleep apnea, and sedating medications.
Reference: American Family
Physician Journals.
21. case of pneumonia, what ur finding on the auscultation?
A.dispred crackles
B.bronchial breath sound
C.absence of vesicular breath sound
Answer: B
(short textbook of medical diagnosis and management, INAM DANISH, page 344)
In lobar pneumonia: Bronchial breath sound+crackles
http://www.turner-white.com/pdf/hp_jan02_pulmonary.pdf
22. Bilateral pneumonia treatment
answer :( not clear question )
23. A patient with lung cancer. Lab results: low PTH and High Calcium. What is the reason?
a. PTH related peptide for lung ca
Answer: A
Reference: kumar and clarks p:556
24. Patient with bronchiectasis. What else beside medical treatment can benefit this patient?
a. Chest physiotherapy
Answer: A
Reference: Toronto Notes
25. Best drug to decrease bronchial secretion in COPD?
a. Ipratropium
Answer: A
Reference: Master the board.
26. A patient with chronic retrosternal pain, cough and metallic taste in mouth. What is the most
likely diagnosis?
a.GERD
Answer: A
heartburn (pyrosis) and acid regurgitation (together are 80% sensitive and specific for reflux) sour
regurgitation, water brash, sensation of a lump in the throat (bolus sensation) and frequent belching.
Usually a clinical diagnosis based on symptom history and relief following a trial of pharmacotherapy
(proton pump inhibitor (PPI): symptom relief 80% sensitive for reflux) . NB. 24-h pH monitoring is the
most accurate test, but rarely required.
PPIs are the most effective therapy. Reference: Toronto Notes.
27. Definitive diagnosis of TB.
Answer : culture is done on all initial samples to confirm the diagnosis. (However, a positive culture

smle ,2016

147
is not always necessary to begin or continue treatment for TB.) A positive culture for M. tuberculosis confirms the diagnosis of TB disease.
http://www.cdc.gov/tb/publications/factsheets/testing/diagnosis.htm
28. Most common cause of acute bronchiolitis:
A. RSV
B. Adeno
C. Parainfluenza
Answer: A
Reference: http://emedicine.medscape.com/article/961963-overview
29. .A patient developed rhinorrhea and itching immediately after moving into new apartment. There
was Molds spores in the apartment
Answer: allergic Rhinitis
30. Rhinorrhea, cough and conjunctivitis etiology?
A- Rhinovirus
B- Adenovirus
Answer: B
The most common cause of rhinorrhea and sinusitis is Rhinovirus. But since there is also conjunctivitis, then Adenovirus is more appropriate.
Reference: http://emedicine.medscape.com/article/302460-clinical
http://emedicine.medscape.com/article/211738-clinical
31. In which lobe does lobar pneumonia become ( m79ora)???
1.
Right mid
2.
Right upper
3.
Right lower
4.
Left upper
answer:Right lobe of lower lung
Ref https://books.google.com.sa/books?id=-CB3jViGHv0C&pg=PA32&dq=most+common+site+in+pneumonia&hl=ar&sa=X&ved=0ahUKEwiWu_zn94DKAhXCvhQKHckOAccQ6AEILzAC#v=onepage&q=most%20common%20site%20in%20pneumonia&f=false
32. Which of the following medication decrease mucus production in a patient with emphysema?
A. Cromolyn sodium
B. Steroids
answer : Anticholinergic agents will dilate bronchi and decrease secretions. They are very effective in
COPD.
Cromolyn sodium (no benefit in COPD)
Ref : Master the board
33. Organism causing pneumonia after intubation in ICU ?
Answer:
-Pseudomonas,. S. aureus (from http://www.ncbi.nlm.nih.gov/pmc/articles/PMC4056625/)
-Gram negative bacilli and S.aureusFrom Washington book
34. What of the following decrease the recurrence of asthma of the

smle ,2016

148
following?
a. Salbutamol
b. Aminophylline
c. Ipratropium
d. Montelukast
Answer A
Medications upon discharge An asthmatic attack has not fully resolved even when symptoms have
abated. Residual airflow obstruction due to airway inflammation may last for several days. Thus, in addition to short-acting beta agonists to be used as needed, the patient will need glucocorticoids to treat the
inflammation and prevent recurrent symptoms. Inhaled glucocorticoids Treatment with regular inhaled glucocorticoids constitutes an important method to prevent recurrent asthma attacks after discontinuation of oral glucocorticoids and to prevent the potential decline in lung function associated with
any future severe asthma exacerbation
REF :http://www.uptodate.com/contents/treatment-of-acute-exacerbations-of-asthma-inadults?source=search_result&search=asthma&selectedTitle=3~150
Or LOW Dose cortisone
http://www.ncbi.nlm.nih.gov/pmc/articles/PMC2121160/
35. What of the following decrease the inflammation in asthma of the following? Lukiutrina
Answer:
36. hemosiderin deposition in macrophage in lung in
a-CMV
b-Pneumocystisjiroveci
c-Chronic lung infection
Answer C: http://www.ncbi.nlm.nih.gov/pmc/articles/PMC2610638/
37. Pt had URTI ,2 weeks later developed orthopnea , severe pulmonary edema
-What is the dx .
A.Infective endocarditis
B.acute pericarditis
C.acute myocarditis
D.acute bronchitis
Answer C
Patients with myocarditis have a clinical history of acute decompensation of heart failure,( e.g. tachycardia, gallop, mitral regurgitation, edema ).
In viral myocarditis, patients may present with a history of recent (within 1-2 wk) flu like syndrome
of fevers, arthralgias, and malaise or pharyngitis, tonsillitis, or upper respiratory tract infection.
http://emedicine.medscape.com/article/156330-clinical#b1
38. Pt healthy pt with no symptoms x ray is normal ,has negative hx of tuberculin test now has positive test ..?
A. Reassure
B. give rifampicin and izo
C. give izo for 6 months
The pt has latent TB ,so the treatment will be:
* 6 month or 9-month isoniazid daily,
* 3-month rifapentine plus isoniazid weekly,

smle ,2016

149
* 3- or 4-month isoniazid plus rifampicin daily,
* 3 or 4-month rifampicin alone daily.
http://www.who.int/tb/challenges/ltbi/en/
39. patient have cough and sob x-Ray show consolidation in right upper lob what ttt?
Answer: Antituberculosisdrugs.TBmay be found in any part of the lung, but upper lobe involvement is
most common.
Reference: http://emedicine.medscape.com/article/230802-workup#c12
40. patient with asthma exacerbation ,Which drug will decrease the mucous secretion more the bronchodilation :
A-oral steroids
B- ipratropium
C- leukotriene
Answer A
Corticosteroids reduce the mucus secretion by inhibiting the release of secretagogue from macrophages.
Reference: http://www.ncbi.nlm.nih.gov/pubmed/3026210
41. A middle-aged man presents with a cough and fever lasting several weeks. Posteroanterior chest
radiograph shows a prominent paratracheal area on the right, lymphadenopathy, a cavitary opacity in the right upper lobe, and a focal consolidation in the middle lung zone on the right.
CXR shown below. What is the dx?
A-COPD
B-BA
C-Pneumonia
D-TB

Answer: D
42. pt with lung nodule and high ca& parathyroid
hyperparathyroidism - lung ca
Answer:
PTH-RELATED PROTEIN : squamous cell carcinomas (lung)
Toronto Notes 2016 ,page 1275 ,Table 29. Paraneoplastic Syndromes

smle ,2016

150
Kumar &ClarksEighthEdition,page 858
43. patient with pertussis best swap
A- Nasal swab
B- Nasopharyngeal
C- Tracheal
Answer B
The culture specimen should be obtained by using deep nasopharyngeal aspiration or by holding a flexible swab (Dacron or calcium alginate) in the patient posterior nasopharynx for 15-30 seconds or until a
cough is produced.
http://emedicine.medscape.com/article/967268-workup#c9
44. lung found some material in the macrophage ( i can't remember biopsy of the material )
a. (PCP ) pneumocystis cariniijinra
b. CMV
c. Bacterial
depend on the conten
45. pt present with sx of brochictasis what is the best advice?
A. stop smoking
Answer: A
http://emedicine.medscape.com/article/296961-treatment?src=refgatesrc1#d8
46. Smoker + hemoptysis what to do (not specified first or best)?
A.Chest x-ray
B.Chest CT
C.Ppd
D.Coagulation profile
Answer: A cause he is symptomatic Toronto notes
47. Pt with diarrhea and cxr showing bilateral infiltrates(pneumonia). Which organism responsible
A. Legionella
Answer:a
Kumar &ClarksEighthEdition ,page 837
48. 2 qs about croup
49. Q about. TB. Numbness
A. Pyridoxine
http://emedicine.medscape.com/article/124947-overview#a4
50. scenario about horner syndrome asking about the site of tumor :
Answer: Aon the lung apical .
Pancoast tumor (tumor in the apex of the lung, most commonly squamous cell carcinoma)
http://emedicine.medscape.com/article/1220091-overview#a4
51. asthmatic pt on inhaled corticosteroids , asthma becomes more sever what should you add :

smle ,2016

151
A. LABA
Answer: A
The stepwise management of asthma(step 3 Inhaled corticosteroids and long-acting inhaled 2
agonist) Kumar &ClarksEighth Edition,page830)
52. Patient with ventilator associated pneumonia. Culture showed lactose non-fermenting, gram negative motile bacilli producing greenish colony and oxidase positive what is the organism:
A. E. coli
B. Pseudomonas
C. Klebsiella
D. Proteus
Answer B
http://www.columbia.edu/itc/hs/medical/pathophys/id/2008/utiGNR.pdf
53. PPD was +ve , to prevent false +ve , what to do?!
A. Repeat it
B. Do X-ray
C. Do Mantoux test
Answer B
http://emedicine.medscape.com/article/1947912-overview#a6
54. X-ray of the lung showed opacification with air fluid level? What is the moa of Abx?
Opacification with air level <<abscess
http://radiopaedia.org/articles/lung-abscess
55. Elderly, asthmatic, what is the best induction
A. Propofol
B. ketamine
Answer : A
propofol is considered to be the agent of choice for induction of anesthesia in asthmatics.
http://www.ncbi.nlm.nih.gov/m/pubmed/11050961
56. long scenario about patient presented dry cough after being diagnosed with HTN what is the
cause:
A. furosemide
B. ACEI (they mentioned the drug name )
answer :B
The most common side effect is a mild dry cough due to their effect on bradykinin.(Captopril,
Enalapril, Lisinopril )Kumar &ClarksEighth Edition ,page 782
57.
non small cell lung cancer has 4 risk factors which are stage of the disease , condition of the patient and male sex
Answer:

smle ,2016

152
58. In emphysema alpha one antitrypsin which part is affected:
A. Interstitial
B. Centroacinar
C. Peripheral
D. subseptal or something like this
Answer: A
http://emedicine.medscape.com/article/298283-overview#a4
59. pt develop cough during exercise :: which medication want to give her before exercise .
Short-acting inhaled beta2-agonists (bronchodilators) stop symptoms
60. Old male with recurrent episodes of cough with sputum and hemoptysis:
A- bronchiectasis(my answer)
B- Tb)
Answer:A
Toronto Notes 2016 page1258
61. Old patient with small cell lung cancer treated by chemotherapy on
examination there is crepitation on the lung no LL swelling lab result showed
hyponatremia what is your advice ?(case of paraneoplastic syndrome (siadh ).
A- IV furosemide
B- Fluid restriction
C- Desmopressin
The answer is B
62. Which one of these lung cancers is associated with anti epithelial cell receptor therapy ?
A- Adenocarcinoma
B- Squamous Cell Carcinoma
C- Small Cell Carcinoma
Answer: A
Kumar &ClarksEighth Edition,page861
63. 64.Old woman has HT its not controlled even with multi drugs .. She sleeps afternoon alot and
feels fatigue most of the time .. What is the cause of her resistance HT ?
a) Obstructive sleep apnea
Answer: A
Toronto Notes 2016page1045
64. Skin manifestation associated with cystic fibrosis:
A- seborrheic dermatitis
B- dermatitisherpetiformis
answer: b present with celiac disease, other two answers were irrelevant
65. embryonal alveolar from what?
A. saccule
I think the Q is not complete!!

smle ,2016

153
http://www.ncbi.nlm.nih.gov/m/pubmed/19175284/
66. Patient brought to ER comatosed with cherry red skin ?
A. co posing
answerA
http://cursoenarm.net/UPTODATE/contents/mobipreview.htm?27/29/28112?source=related_link#H66
67. asthmatic controlled on albuterol PRN, now she got pregnant,
and she started to have daily symptoms & night ... ?
A. Inhaled steroid + LABA
I think the Q is not complete!!
The drugs that have been used for many years in asthma
therapy have now been shown to be safe and without teratogenic
potential. These drugs include SABA, ICS, and theophylline; there is
less safety information about newer classes of drugs such as LABA,
antileukotrienes, and anti-IgE. If an OCS is needed, it is better to use
prednisone rather than prednisolone because it cannot be converted to
the active prednisolone by the fetal liver, thus protecting the fetus from
systemic effects of the corticosteroid.
68. patient presented to the ER with cough hemoptysis night sweats and malaise. what is most appropriate initial step in the management?
A.isolation in negative pressure room
B.start anti TB
C.give OPD appointment after 2 weeks
Answer: B, TB case is usually confirmed by a positive culture for M. tuberculosis. However, in some
cases, patients are diagnosed with TB disease on the basis of their signs and symptoms, even if their
specimen does not contain M. tuberculosis CDC
Kaplan LNs internal medicine page201
69. COPD patient was on oral steroid and there was improvement 17%in breathing , which medication will u put him on :
A. theophylline
B. amitriptyline inhaler or oral
answer :
Prednisolone 30 mg daily should be given for 2 weeks, with measurements of lung function before and
after the treatment period. If there is objective evidence of a substantial degree of improvement in airflow limitation (FEV1 increase >15%), prednisolone should be discontinued and replaced by inhaled corticosteroids (beclometasone 40 g twice daily in the first instance, adjusted according to response).
Kumar &ClarksEighthEdition ,page 816
70. An Elderly pt presented to ER due to decreased level of consciousness lethargy
Pco2 20 mmhg
K2
Ph 7.2 I can't remember the rest of the labs and choices
What does she have?

smle ,2016

154
A. high anion gap metabolic acidosis with primary respiratory alkalosis
answer:A
Toronto Notes 2016 ,page710,1252
71. pt with chest infx was treated with oral AB For 4 weeks later came complain from rt lung effusion
what dx?
a) Parapneumonic effusion empyema
b) TB
c) Lung ca
Answer:A
Kumar &ClarksEighthEdition ,page 837
Definition
pus in pleural space or an effusion with organisms seen on a Gram stain or culture (e.g. pleural fluid is
grossly purulent)
positive culture is not required for diagnosis
Etiology
contiguous spread from lung infection (most commonly anaerobes) or infection through chest wall
(e.g. trauma, surgery)
72. Old male with recurrent episodes of cough with sputum and hemoptysis
A. Bronchiectasis
B. Tb
Answer A
Bronchiectasis((chronic cough, purulent sputum (but 10-20% have dry cough), hemoptysis (can be massive)) Toronto Notes 2016 ,page1258
73. what you'll seen on physical examination of pt with croup ??
A- presence of inspiratory sounds
B- presence of expiratory wheeze
Answer: A but in severe both of them
The physical presentation of croup has wide variation. Most children have no more than a "croupy"
cough and hoarse cry. Some may have stridor only upon activity or agitation, whereas others have audible stridor at rest and clinical evidence of respiratory distress. Paradoxically, a severely affected child
may have , stridor secondary to a greater degree of airway obstruction. The child with croup typically
does not appear toxic.
The child's symptoms can range from minimal inspiratory stridor to severe respiratory failure secondary
to airway obstruction.[14] In mild cases, respiratory sounds at rest are normal; however, mild expiratory
wheezing may be heard. Children with more severe cases have inspiratory and expiratory stridor at rest
with visible supra sternal, intercostal, and subcostal retractions. Air entry may be poor. Lethargy and agitation are due to marked respiratory difficulty and, hence, hypoxia and increasing hypercarbia. Respiratory arrest may occur suddenly during an episode of severe coughing.
http://emedicine.medscape.com/article/962972-clinical#b33
74. Case of asthma sever , cough every week , he took neublazer Steroid , wt next management ?
A. add long acting B agonist
B. ibrapritom
Answer: A

smle ,2016

155
add long acting B agonist
The stepwise management of asthma(step 3 Inhaled corticosteroids and long-acting inhaled 2 agonist)
Kumar &ClarksEighth Edition,page830)
75. Lung tumor with keratinization, what type of cells?
A. Epithelial
B. Squamous
Answer:
76. 12yo bilateral lower lung infiltration ttt?
A. Ciprofloxacin
B. Azithromycin
C. penicillin
most likely B atypical pneumonia ttt by macrolides class if not available tetracycline
boy referral due to having recurrent chest infections & has brother die at 6yrs as same chest infection
sister normal all immunoglobulins low T Cell function good.
77. 61yo male patient with bilateral lung base infiltrate (x-ray chest) cough ,diarrhea,Temp. 38.7c
Answer:
78. (long scenario ) ,what is the microorganism ?
A. legionella pneumonia
B. mycoplasma pneumonia
Answer:
79. ptpreseneted with pneumonia symptoms for 2 weeks i think the gram stain negative :
A. mycoplasma pneumonia
Answer:
80. patient presented to the ER with cough hemoptysis night sweats and
malaise ?what is most appropriate initial step in the management
A.isolation in negative pressure room
B.start anti TB
C.give OPD appointment after 2 weeks
Answer: B TB case is usually confirmed by a positive culture for M. tuberculosis. However, in some cases,
patients are diagnosed with TB disease on the basis of their signs and symptoms, even if their specimen
does not contain M. tuberculosis CDC
81. typical history of pneumonia. x ray was done lower lobe consolidation was
found culture shows gram positive cocci arranged in clusters. catalase and
coagulase positive. what is the most appropriate AB for this infection?
A.oxacillin
B.penicillin G
C.amoxicillin
Answer: A

smle ,2016

156
82. Status asthmaticus on drug inhibit cholinesterase what is the drug?
A. Ipratropium
Answer:
83. COPD developed , DM, developed Acute closure glaucoma; which
treatment.
Answer: (...systemic ophtha, local ..Acetazolamide )
84. asthmatic with 3/week of frequency on short acting and last sever attack was 3 months ago What
the appropriate management ?
A. short
B. Long
C. ipratropium
D. Dexamethasone
This is the option not missing any info dexamethasone
85. positive PPD skin test for adult man without any sign of TB infection what you will be do ?
Answer: Chest X ray if negative INH for 9 m
86. asthmatic controlled on albuterol PRN, now she got pregnant, and she started to have daily symptoms & night ... ?
A. Inhaled steroid + LABA
Answer:
87. boy referral due to having recurrent chest infections & has brother die at 6yrs as same chest infection sister normal all immunoglobulins low T-cell function good..
A. X-LINKED agammaglobulinemia
Answer:
88. 61yo male patient with bilateral lung base infiltrate (x-ray chest) cough, diarrhea,Temp. 38.7c
(long scenario ) ,what is the microorganism ?
A. legionella pneumonia
B. mycoplasma pneumonia
answer:
89. patient with positive ppd , never was +ve before what is next step no x-ray findings?
A. isoniazid and rifampin 6 months
B. rifampin 3 months
Answer: isoniazid ..
90. if there is no iso then rifampin adult patient came to ER cant talk agonic what are you going to do
?
A. look for object in his mouth
B. give him oxygen
answer:
91. After delivery shortness of breath at night. What findings in the x-Ray support diagnosis?

smle ,2016

157
A) Increase in mediastinal width.
B) Increase shadowing
C) Cardiothoracic increase.
Answer: PE
92. Patient with hemoptysis, at first it was blood tinged then it appeared bright red blood what
should the next investigation be?
A) Chest x-ray
B) ppd
Answer:
93. upper limb edema , intercostals vein engorgement , lesion in right lung ,
compression in which side :
A. Ant mediastinum
B. Post mediastinum
C. Rt hilum
D. Median mediastinum
Answer: A ??
94. 3 years old with a father known to have pulmonary TB his PPD 10mm what does he have ?
A. Strong +ve
B. Weak +ve
answer:
95. case of asthma sever , cough every week , he took neubelizer Steroid , what next management ?
A. add long acting B agonist
B. Ipratropium
Answer: add long acting B agonist
96. pt e chronic interstitial lung dis in biopsy see small non necrotizing granul in alveolar membrane
Dx?
A. TB
B. Arthrosis
C. hypersensitivity pneumonitis
Answer: hypersensitive pneumonitis
97. Obstructive sleep apnea. Most effective treatment
A. BMI less than 30
B. Cpap
Answer:
98. female came from 18 hours flight and she feel leg pain what you will take :
a- warfarin
b- LMWH
c- Unfractionated heparin and warfarin
answer: Unfractionated heparin and warfarin. Reference: uptodate.

smle ,2016

158
99. Prostate cancer patient with Recurrent DVT. Best prophylaxis is:
a- LMWH.
b- unfractionated heparin.
c-LMWH short term therapy followed by warfarin.
Answer: A
100.Pt with small lung ca and undergoing chemo , developed ... I don't remember But lab values : low
blood Na, And low urine osmolality
The treatment: ?
a)Desmopressin
b) Nothing
answer: A ??
101. female non smoker with nodule by ct found calcium and fat
a. hamartoma
B. mystheoma
C. no adeno
Answer: A
http://emedicine.medscape.com/article/356271-overview
102. smoker with hilar mass what suspect
A. lymph node
B. squamous cell ca
C. adenocarcinoma
Answer: B
http://emedicine.medscape.com/article/279960-overview
103. 4 weeks back pt had mycoplasma pneumonia what will be very high in LP?
A. protein
B. WBC
C. glucose
Answer:
104.Low ph, bicarb, co2?
A. Compensated metabolic acidosis
answer: NON COM
105.bronchial cancer mets to sympathetic plexus what the sign ?
A. Ptosis dilated pupil
Answer: the answer is horners syndrome ptosis ,anhidrosis,miosis
Toronto Notes Neoplasm (R28)
http://www.uptodate.com/contents/superiorpulmonarysulcuspancoasttumors

smle ,2016

159

RHEUMATOLOGY
1. blue sclera + multiple fracture?
Answer: osteogenesis imperfecta
2. patient with symptoms of gout , which medication will help?
Answer: inhibit the xanthine oxidase
3. treatment of chronic pain?
Answer: Physiotherapy, NSAIDs, Acetaminophen, Antidepressants, Anticonvulsants, Muscle relaxant and
Opioids
REFERENCE: https://www.asra.com/page/46/treatment-options-for-chronic-pain
4. 60 year old female with distal phalangeal joint swelling and shoulder pain and knee pain , x
ray showed narrow joint space and osteophytes?
A. Rheumatoid arthritis
B. Osteoarthritis
Answer: B
REFERENCE: Toronto notes
5. giant cell arteritis sequelae:
A- Complete loss of vision
B- Destructive arthritis
Answer: A
If left untreated may lead to permanent visual loss
REFERENCE: Kaplan Internal Medicine
6. passive and active joint restriction in all directions of ROM is caused by a frozen shoulder or
glenohumeral synovitis?
Restriction in internal rotation suggests an impingement syndrome due to rotator cuff tendinitis, Inability to perform active abduction suggests a rotator cuff tear or a frozen shoulder.
REFERENCE: http://emedicine.medscape.com/article/328253-clinical
7. patient complaining of hip pain after long periods of using the hip it keeps him awake at night
and have prolonged hours of stiffness in the morning?
A. osteoporosis
B. osteoarthritis
Answer: B
8.

TX of osteoporosis?
A. bisphosphonate
Answer: A

smle ,2016

160

9. Case of rheumatic fever, what is the treatment?


Answer: Penicillin and high dose of aspirin
REFERENCE: http://www.uptodate.com/contents/acute-rheumatic-%20fever-treatment-%20and-prevention
10. Young boy with pain in his knee, aspiration of fluid reveal yellowish and turbid appearance,
Diagnosis?
A. septic arthritis
Answer: A
11. Lytic femur lesion & osteoporosis skull?
A. Paget disease
Answer: A
http://emedicine.medscape.com/article/334607-overview#a3
12. Pt took her anti osteoporosis drug then had a severe retrosternal pain?
Answer: Dont remember exact drugs, but belongs to bisphosphonate.
There are four bisphosphonates currently approved for use in Canada: alendronate (Fosamax ), etidronate (Didrocal ), risedronate (Actonel ) and zoledronic acid (Aclasta). Also available are: Actonel DR,
Fosavance (Fosamax with vitamin D) and several generic versions.

Gastrointestinal side effects, such as abdominal pain, constipation or diarrhea, gas, esophagus ulcer, or stomach bloating, may occur. Esophageal ulcers or swelling may be severe enough to cause
bleeding and has resulted in hospitalization, even death. Standing or sitting upright for 30 minutes
after taking oral bisphosphonates helps prevent the drug causing esophageal irritation. Call your
doctor immediately if you experience any of the following side effects.

Allergy symptoms, such as itching, hives, swelling of mouth or hands, tightness in chest, or trouble
breathing.

Unusual or severe stomach or throat pain.


13. unilateral knee swelling and pain, knee tap labs: cloudy yellow color, mucoid, WBC - pmn 15
(normal less than 200), lymphocytes 80%, what is dx?
A. Gout not sure
B. Septic arthritis
C. RA
Answer: B
14. How would you manage aggressive RA?
A. Methotrexat
B. vtCyclosporin
Answer: A
15. when diagnosing patient with SLE, what's most important?
A. Age

smle ,2016

161
B. sex " I think it's right"
C. joint affection
answer: C
REFERENCE: http://www.rheumtutor.com/2012-slicc-sle-criteria/
16. Behcet disease is more common in?
A. Children
B. young men
C. old women
D. equal in men & women
Answer: D
REFERENCE: http://www.vasculitis.org.uk/about-vasculitis/behcets-disease
Men and women are probably affected with equal frequency, but it is sometimes more severe in men. It
may occur at any age, but is diagnosed most commonly in adults in their 20s and 30s
17. which one of these is specific and found in SLE?
A. cystoid bodies on fundoscopy
B. anti RNP
C. severe Raynaud
Answer: anti-dsDNA and anti-Sm are specific (95-99%)
18. Something about prognosis of SLE?
A. Sex
B. Arthritis
C. nephritis
Answer: C
19.Polymyalgia rheumatica. Aid Dx?
A. proximal muscle weaknesses
B. proximal muscle tenderness
Answer: B
20. Long history of big toe pain with ve birefringent crystals, what is the management?
A. Colchicine
B. Allopurinol
C. NSAID
answer: C
NSAID treatment of acute attack , Allopurinol treatment of chronic disease
REFERENCE: http://emedicine.medscape.com/article/329958-treatment#d8
21. Female patient with skin thickening over the forearm, with Raynauds phenomenon. [CASE
OF SCLERODERMA] Which of the following would help you the most?
A. Anti Scleroderma - SCL 70 [100% in Dermatology]
B. Anti Centromere

smle ,2016

162
C. SS-A
D. SS-B
Answer:
A if its diffuse
B if it's limited
Limited systemic sclerosis
Skin sclerosis restricted to hands, face, neck
3rd to 4th decade
Pulmonary hypertension common
CREST Syndrome:
Calcinosis: calcium deposits on skin
Raynauds phenomenon
Esophageal dysfunction: acid reflux
Sclerodactyly: tightening of skin on digits
Telangiectasia: superficial dilated blood vessels
anti-topoisomerase 1/anti-Scl-70: specific but not sensitive ( for diffuse systemic sclerosis )
anti-centromere: favours diagnosis of CREST ( for limited systemic sclerosis)
REFERENCE: Toronto notes 2014
22. female with arthritis and butterfly rash on the face. ANA is positive. What other
marker you would order?
A. Anti Double stranded DNA
B. Anti Centromere
C. SS-A
D. SS-B
Answer: A
treatment for rheumatoid arthritis uses ( something about macrophages and TNF and IL)
23. What can recur in patients using the treatment mechanism?
A. inflammatory bowel disease
B. TB
C. gram -ve pneumonia
D. rheumatoid arthritis
Answer: C
Biologics work for more people because they target specific parts of the immune system to reduce inflammation in the joints. They have fewer side effects than older drugs. Any drug that suppresses the
immune system carries risks, though. Side effects can include:

severe infections, such as lung infections

liver damage

reduced ability to make new blood cells

nausea

smle ,2016

163

pain or swelling at the injection site

24. Hypertensive PT. with acute gout what to give?


A. NSAID
B. Allopurinol
C. Colchicine
D. Intra Articular steroid
Answer: D ( in patient with comorbid condition cannot use nsaid or colchicine use intra articular steroid
injection!! )
25. Major Jones criteria of rheumatic fever ?
http://emedicine.medscape.com/article/236582-clinical
26. pt with recurrent oral and genital ulcers and arthritis when the pt taken some type of IM vaccine develop sterile abscess at the site of injection. What is the most likely Dx?
A- Behcet disease
Answer: A
Behet disease is characterized by a triple
-symptom complex of recurrent oral aphthous ulcers, genital ulcers, and uveitis. The disease is named
after the Turkish dermatologist HulusiBehet, who identified it in a patient in 1924 and published a description of the disease in 1937.
REFERENCE: http://emedicine.medscape.com/article/329099-overview

27. wheal with erythematous base itching lymph node enlargement periorbital swelling hepatosplenomegaly?
A. Rheumatic arthritis
B. Angioedema
C. Cholinergic urticarial (not sure)
D. itching more with urticaria
Answer: B>> answered by rheumatologist
28. Osteoporosis prevention?
A. biphosphate + vit.D + ca
Answer:
If premenopausal; vit.D + ca
If postmenopausal; biphosphate + vit.D + ca
answered by rheumatologist
29. HX of septic arthritis management?
A. aspiration and AB
B. iv ab
answer: A
30. Case of rheumatic fever, what is the treatment?
A. Penicillin and high dose of aspirin

smle ,2016

164
answer: A
REFERENCE: http://www.uptodate.com/contents/acute-rheumatic-fever-treatment-and-prevention
31. Polymyalgia rheumatica, what supports your dx?
A. muscle pain
B. muscle stiffness
C. Others
Answer: A
REFERENCE: http://emedicine.medscape.com/article/330815-overview
32. Patient is diagnosed with rheumatoid arthritis and is on aspirin he developed symptoms of
heartburn which is relieved for a little while with antacids what will you give him?
A. Misoprostol
B. H2 blocker

Answer: A
full-dose misoprostol (200 g four times daily) reduces the risk of NSAID-induced ulcer complications by 40%
omeprazole was superior to ranitidine and misoprostol
33. Pt HTN came with uric acid 200 you prescribe antihypertensive drug for him after 1 week uric
acid 400 what is the drug ?
A. Thiazide diuretics
Answer: A
Answer explained in Q152
34. Patient known case of behcet came with arthritis resistant to steroid what to do?
A. Cholacine
Answer: A
35. A 61 y female known case of osteoarthritis, came for regular check up, not taking ca supplements nor high ca diet, she is a high risk of osteoporosis What is the best initial thing before deciding the appropriate mx?
A. DECADE Scan
B. oral ca, vit.D, bisphosphonate
C. TSH, Ca, vit.D
D. Intranasal calcitonin, CA, vit.D
Answer: B
36. Case of arthritis in first metatarsal joint, dx? (symptoms of gout)
A. gout
B. RA
C. osteoarthritis
Answer: A
37. Patient presented with knee swelling and pain they did x ray and aspiration and found negative birefringence needle like what are you going to discharge the patient with?

smle ,2016

165
A. Allopurinol
B. NSAID
answer: B
38. Characteristics of Ehlers Danlos syndrome (without tilling the dx) then asked about Mode of
inheritance?
A. X linked
B. AD
C. AR
D. Mitochondrial
Answer: B & C
REFERENCE: http://ghr.nlm.nih.gov/condition/ehlers-danlos-syndrome#genes
39. How do people inherit Ehlers-Danlos syndrome?
The inheritance pattern of Ehlers-Danlos syndrome varies by type. The arthrochalasia, classical, hypermobility, and vascular forms of the disorder have an autosomal dominant pattern of inheritance. The
dermatosparaxis and kyphoscoliosis types of Ehlers-Danlos syndrome, as well as some of the rare, less
well-characterized types of the disorder, are inherited in an autosomal recessive pattern.
40. Scenario of SLE (Rash type), case of?
A. SLE
Answer: A
41. Question about latent SLE?
42. Headache when combing hair, tender vessel on temporal area, sudden loss of vision "clouding in one eye what next action?
A. oral prednisolone for 3 months
B. immediate cortisone eye drops B
Answer: A
REFERENCE: http://emedicine.medscape.com/article/332483-treatment
43. Old man with joint pain worsens on movement. X-ray of wrist shows narrow joints in small
joints. What's the dx?
A. Osteoarthritis
B. Rheumatoid arthritis
answer: A
REFERENCE: https://www.hss.edu/osteoarthritis-diagnosis.asp
44. Typical symptoms of RA, what's the dx?
A. RA
Answer: A
45. What is the single most important thing for the prognosis of SLE?
A. renal involvement
B. arthritis
C. WBCs

smle ,2016

166
Answer: A
46. Pt complain of pain swelling of big toe what's the Dx?
A. Gout
Answer: A
47. Patient come with back pain when awake from sleep he had stiffness for almost 30 minutes and
it's resolved he take i think paracetamol or other thing in examination there is a paraspinal muscle
stiffness When you did a spine CT you found mild lumbar stenosis what your management?
A. epidural steroid injection
B. physiotherapy
C. ibuprofen
D. bed rest
Answer: B and C
REFERENCE: http://www.niams.nih.gov/health_info/spinal_stenosis/
http://patient.info/doctor/lumbar-spinal-stenosis
48. treatment of gestational diabetes?
A. Insulin
B. metformin
C. glipizide
Answer: A
49. Behcet's disease?
A. Vasculitis
B. Ulcer
C. Test
Answer: A
50. temporal arteritis pt. "clear scenario" this pt. is in higher risk of: CAD, Blindness, brain tumor or
other option I forgot?
A. blindness
Answer: A
51. Patient present with Hip and shoulder pain, ESR high ( polymyalgia rheumatica case) in addition
to symptom what else can be?
Answer: ( proximal muscle tenseness )
#TORONTO .... Signs and Symptoms
constitutional symptoms prominent (fever, weight loss, malaise)
pain and stiffness of symmetrical proximal muscles (neck, shoulder and hip girdles, thighs)
gel phenomenon (stiffness after prolonged inactivity)
physical exam reveals tender muscles, but no weakness or atrophy
52. Osteoporotic Patient on medication that work by atp-analogue?
A. Etidronate
answer: A
Bisphosphonates are the most common medications prescribed for osteoporosis treatment

smle ,2016

167
53. SLE patient and she is vegetarian, complaining of fatigue & tiredness, what u will find?
A. Low iron low TIBC *total iron binding capacity
B. low iron high TIBC
C. high iron high TIBC
D. high iron low TIBC
Answer: B
54. PT. develop gout what cause?
A. Thiazide
B. HTN (( hypertension ))
C. CHF (( congestive heart failure ))
Answer: B and A >> most accurate A
Both may cause gout .. C is the result of gout
REFERENCE: http://www.medscape.com/viewarticle/757006
55. Risk factors for overproduction of uric acid and gout include?
Genetics The occurrence of gout is partly genetic, contributing to about 60% of variability in uric
acid level.[8] Three genes called SLC2A9, SLC22A12, and ABCG2 have been found commonly associated
with gout, and variations in them can approximately double the risk. Gender and age It is more common in men than in women and children.
Weight Being overweight increases risk.
The consumption of coffee, vitamin C, and dairy products, as well as physical fitness, appear to decrease the risk.[15][16][17] This is believed partly due to their effect in reducing insulin resistance.
Alcohol consumption Drinking too much alcohol keeps the body from removing uric acid.
Diet Eating high-purine foods can lead to gout flares or make them worse. These foods include anchovies, asparagus, beef kidneys, brains, dried beans and peas, game meats, gravy, herring, liver, mackerel, mushrooms, sardines, scallops, and sweetbreads (animal glands).
Lead exposure.
Other health problems A common cause of gout in older people is the kidneys being unable to
eliminate waste products from the body. Other problems that may contribute to gout include: high
blood pressure, hypothyroidism (underactive thyroid gland), psoriasis, hemolytic anemia, or some cancers, and some inherited disorders in which the enzyme helping control uric acid is not present or
doesnt work properly.
Metabolic syndrome, a combination of abdominal obesity, hypertension, insulin resistance, and abnormal lipid levels.
Medications A variety of medicines may increase your risk for gout. They include:
o Diuretics (water pills): a low dose of hydrochlorothiazide does not seem to increase the risk
o Salicylate-containing drugs, such as aspirin
o Niacin, a vitamin also known as nicotinic acid or vitamin B3
o Cyclosporine, a drug used to suppress the immune system in the treatment of some autoimmune diseases and to prevent rejection of transplanted organs
REFERENCE: http://www.mayoclinic.org/diseases-conditions/gout/basics/risk-factors/con-20019400
56. In gout what substance would be high?
A. PRPP
B. xanthine
Answer: A

smle ,2016

168
There are at least three different inherited defects that lead to early development of severe hyperuricemia and gout: glucose-6-phosphatase (gene symbol: G6PT) deficiency; severe and partial hypoxanthineguanine phosphoribosyltransferase (HGPRT, gene symbol: HPRT) deficiency; and elevated 5'-phosphoribosyl-1'-pyrophosphate synthetase (PRPP synthetase) activity
REFERENCE: http://themedicalbiochemistrypage.org/gout.php
57. wheal with erythematous base itching lymph node enlargement periorbital swelling hepatosplenomegaly?
A. Rheumatic arthritis
B. Angioedema
C. Cholinergic urticarial ( not sure)
D. itching more with urticarial
Answer: B>> answered by rheumatologist

58. Treatment of acute gout?


A. Indomethacin
Answer: A
#TORONTO Treatment of acute gout
NSAIDs: high dose, then taper as symptoms improve
corticosteroids: IA, oral, or intra-muscular (if renal, cardiovascular, or GI disease and/or if NSAIDs
contraindicated or failed)
colchicine within first 12 h but effectiveness limited by narrow therapeutic

59. Old man with joint pain worsens on movement. Xray of wrist shows narrow joints in small joints.
What's the dx?
A. Osteoarthritis
B. Rheumatoid arthritis
Answer: A
REFERENCE: https://www.hss.edu/osteoarthritis-diagnosis.asp
http://www.merckmanuals.com/professional/musculoskeletal-and-connective-tissue-disorders/joint-disorders/osteoarthritis-oa

smle ,2016

169
60. unilateral knee pain , swelling , middle age I think Joint aspirate
Wbc: nl Neut : 80% Did not mention crystals?
A. RA
B. Gout
C. septic arthritis
D. Pseudogout?
Answer: C
Symptoms
The classic picture is a single swollen joint with pain on active or passive movement. The knee is involved in about 50% of the cases, but wrists, ankles, and hips are also commonly affected.[6]
Septic arthritis may present as polyarticular arthritis in about 15% of patients.[7]
It is more common in patients
61. patient irrigate and weakness in lower and upper extremities, what is the cause?
A. deficiency in vit.D
B. deficiency in vit.A
C.

deficiency in B1

D. deficiency in B3
Answer: C
Vitamin B1 Depression, irritability, poor concentration, memory problems, confusion, numbness and
tingling in the hands and feet, loss of appetite, muscle weakness, sleep disturbances, fatigue, abdominal and chest pains, shortness of breath, rapid pulse, ankle swelling

62. 70 year old female patient with osteoporosis, what is the treatment?
A. estrogen
B. bisphosphonates
Answer: B
REFERENCE: (1st Aid p:320)

63. rheumatoid arthritis with the loss of bones in joint? What is the cause?
A. substance released by synovial cell
B. synovial fluid pressure

smle ,2016

170
C.

something prostaglandin?

Answer: A, TNF-alpha thats going to be released


REFERENCE: http://www.medscape.com/viewarticle/457990

64. adult with right toe tenderness for 1 week and he have ear pain also (investigation included).
What is the best treatment for him?
A. Cortisone
B. NSAIDs
C.

Allopurinol

Answer: B (For acute attack. Allopurinol can be given after attack resolution)
Treatment of acute gout attack does not differ substantially in patients with or without clinically apparent tophi, although the presence of tophi is an indication further initiation of long-term uratelowering therapy after attack resolution to prevent or reverse chronic gouty arthropathy)
REFERENCE: http://emedicine.medscape.com/article/329958-treatment#showall
http://www.uptodate.com/contents/treatment-of-acute-gout

65. A patient with right knee osteoarthritis, presented with swelling of the right knee. On examination the right knee is swelled with no change in temperature. What will you do?
A. Aspiration of the knee fluid
B. Bilateral Knee X-Ray and Ibuprofen
Answer: B (Because there is no change in temperature. Its normal for osteophyte to cause swelling
of the knee in OA)
REFERENCE: http://emedicine.medscape.com/article/330815-clinical

66. What is the treatment of temporal arteritis?


A. oral steroid
B. topical steroid
Answer: A (The universally accepted treatment of giant cell arteritis (GCA) is high-dose corticosteroid therapy. The major justification for the use of corticosteroids is the impending danger of blindness in untreated patients.
Few studies exist regarding dosing protocols for corticosteroids in GCA. It is generally agreed that
most patients with suspected GCA should be started on oral prednisone 40-60 mg/day, with a temporal artery biopsy performed within 1 week).

smle ,2016

171
REFERENCE: http://emedicine.medscape.com/article/332483-treatment

67. Raynaud phenomena. Which antibody will be positive?


A. Anti-SCL-70
Answer: A (If its scleroderma case the answer will be A
ANA is ordered for Raynaud phenomenon, although it's not specific. The anti-centromere antibodies
are ordered for CREST syndrome) (R for Raynaud)

68. Characteristic finding in Behet disease?


Answer: ? (Leukocytoclastic vasculitis, multi-system disorder presenting with ocular involvement
(uveitis), recurrent oral and genital ulceration, venous thrombosis, skin and joint involvement, more
common in Mediterranean and Asia, average age 30s, M>F) REFERENCE: Toronto Notes

69. A patient with symmetrical joint involvement, complaining of morning stiffness that is relieved
with movement, MCP and PIP are involved as well. What is the diagnosis?
A. RA
Answer: A

70. Boutonniere deformity?


A. Flexion of PIP joint & hyperextension of DIP
Answer: A

71. A patient presented with hip and shoulder pain. ESR: high (Polymyalgia Rheumatica case). In addition to these symptoms. what else can be there in the history?
A. Proximal muscle weakness
B.
Answer: B (Muscle weakness is not a feature of PMR)

72. 60 year old man with a fractured thoracic vertebra. T Score = -2.6
What is the diagnosis?
A. Osteoporosis
B. Established osteoporosis
C. Osteopenia
Answer: B (Dexa scan interpretation)

smle ,2016

172

73. 37 year old presented with back pain. On examination there was tenderness when palpating
paraspinal muscles, neurovascular exam was normal. What is the treatment?
A. Physiotherapy
Answer: A

74. patient with gout. What drug should be avoided?


A. thiazide
B. furosemide
C.

ACEI

Answer: A
Hyperuricemia is a relatively common finding in patients treated with a loop or thiazide
diuretic and may over a period of time, lead to gouty arthritis. Furosemide can elevate
uric acid levels as well.
REFERENCE: UpToDate.

75. A patient with signs and symptoms of renal and respiratory involvement . What is the diagnosis?
A. Glomerulonephritis
B. Wegener's granulomatosis
Answer: B
*Incomplete question but you should to keep one thing in your mind if they mentioned there is a
history of sinusitis the diagnosis will be 100% Wagners
REFERENCE: Master the boards A, First Aid step 1
Acute GN Early symptoms of acute GN include:
puffiness in the face (edema)
urinating less often
blood in your urine (dark, rust-colored urine)
extra fluid in your lungs, causing coughing
high blood pressure

smle ,2016

173
76. about a patient who presented with gout. Inhibition of which enzyme will treat this disease?
A. PRPP synthase
B. Adenosine deaminase
C.

Xanthine oxidase

D. orotate phosphoribosyltransferase
Answer: C (The enzyme xanthine oxidase catalyses the oxidation of hypoxanthine to xanthine and
then to uric acid, which plays a crucial role in gout)
REFERENCE: Pubmed (Thats why we give Xanthine Oxidase inhibitors such as Allopurinol)

77. A patient with SLE with rash on her cheeks, etc. What will you advise her?
A. avoid sun exposure as much as she can
Answer: A
REFERENCE: Medical diagnosis and Management by Mohammad Inam Danish

78. Patient with arthritis and rash on the face. ANA is positive. What should you do next?
A. Anti DNA
Answer: A
#TORONTO .. Investigations
blood work: ANA (sensitivity 98%, but poor specificity >> used as a screening test, ANA titres are not
useful to follow disease course)
anti-dsDNA and anti-Sm are specific (95-99%)
anti-dsDNA titer and serum complement (C3, C4) are useful to monitor treatment response in patients
who are clinically and serologically concordant
anti-dsDNA increases and C3 and C4 decrease with disease activity
antiphospholipid Ab (anti-cardiolipin Ab and SLE anticoagulant), may cause increased risk of clotting
and increased aPTT
79. Patient is concerned about osteoporosis as her mother had it, what you will do?
A. give vit.D, calcium
B. give estrogen postmenopausal
Answer: A
80. treatment of juvenile rheumatoid arthritis?
Answer: #Medscape:
Pharmacologic management consisting of nonsteroidal anti-inflammatory drugs (NSAIDs), disease-modifying antirheumatic drugs (DMARDs), biologic agents, and intra-articular and oral steroids. For systemic
JIA with active systemic features and varying degrees of synovitis, initial treatment for most patients
should consist of anakinra with systemic glucocorticoids. For systemic JIA without active systemic fea-

smle ,2016

174
tures and with varying degrees of active synovitis, initial treatment should be methotrexate or leflunomide for an active joint count higher than 4, with a change to abatacept, anakinra, a tumor necrosis factor (TNF)- inhibitor, or tocilizumab if disease activity continues after 3 months; for patients with 4 or
fewer active joints, NSAID monotherapy or intra-articular glucocorticoid injections should be initial
treatment
#Toronto:
1st line drug therapy: NSAIDs, intra-articular corticosteroid
2nd line drug therapy: DMARDs (methotrexate, sulfasalazine, leflunamide), corticosteroids (acute management of severe arthritis, systemic symptoms of JIA, topical eye drops for uveitis), biologic agents
81. Case of juvenile arthritis & ttt of JRA:
Answer: Oligoarticular juvenile idiopathic arthritis (JIA) is usually responsive to intra-articular
glucocorticoids. Methotrexate and other immunosuppressive drugs are recommended for children with disease that extends to involve five or more joints or require repeat injections. Biologic agents are typically reserved for patients with uveitis and are also used in some patients
with extended oligoarticular JIA.

Systemic JIA

Mild-to-moderate disease nonsteroidal anti-inflammatory drug (NSAID) monotherapy as the


initial treatment in children with possible sJIA who have mild-to-moderate, non disabling symptoms on presentation and no evidence of macrophage activation syndrome (MAS)

Moderate-to-severe disease For patients whose initial symptoms include high fevers, other
systemic manifestations including serositis and possible early MAS, and/or moderate-to-severe
polyarthritis, we suggest adding one of the biologic agents that inhibit interleukin (IL)-1 or IL-6,
such as anakinra, canakinumab, or tocilizumab, rather than the nonbiologic disease-modifying
antirheumatic drug (DMARD), methotrexate. NSAIDs can be continued in conjunction with other
agents if needed for pain control.

REFERENCE: Uptodate

82. Case of rheumatoid arthritis:


Answer: Rheumatoid arthritis is a chronic inflammatory condition. its symptoms develop gradually and may include joint pain, stiffness, and swelling. The condition can affect many tissues
throughout the body, but the joints are usually most severely affected. The cause of rheumatoid
arthritis is unknown.

A person with well-established rheumatoid arthritis typically has or has had at least several of
the following:

Morning stiffness that lasts at least one hour and that has been present for at least six weeks
Swelling of three or more joints for at least six weeks

smle ,2016

175
Swelling of the wrist, hand, or finger joints for at least six weeks
Swelling of the same joints on both sides of the body
Changes in hand x-rays that are characteristic of rheumatoid arthritis
Rheumatoid nodules of the skin
Blood test positive for rheumatoid factor and/or anti-citrullinated peptide/protein antibodies
(ACPA)

Not all of these features are present in people with early RA, and these problems may be present in some people with other rheumatic conditions.

In some cases, it may be necessary to monitor the condition over time before a diagnosis of
rheumatoid arthritis can be made with certainty.

Laboratory tests Laboratory tests help to confirm the presence of rheumatoid arthritis, to differentiate it from other conditions, and to predict the likely course of the condition and its response to treatment.

Rheumatoid factor An antibody called rheumatoid factor is present in the blood of 70 to 80


percent of people with rheumatoid arthritis. However, rheumatoid factor is also found in people with other types of rheumatic disease and in a small number of healthy individuals.

Anti-citrullinated peptide/protein antibody test Blood tests for ACPA are more specific than
rheumatoid factor for diagnosing rheumatoid arthritis. Anti-ACPA antibody tests may be positive very early in the course of disease. The test is positive in most patients with rheumatoid arthritis.

REFERENCE: uptodate

#TORONTO , RH8

83. adolescent male with swollen parotid and salivary gland with dry eye and dry mouth, labs HLA,
ANA and RF are positive which of the following is appropriate treatment?
A. Physostigmine
B. Artificial eye and saliva drops
Answer: B
REFERENCE: medscape .. Treatment for Sjgren syndrome is largely based on symptoms

84. Pseudogout > pyrophosphate crystal

smle ,2016

176
Answer : PSEUDOGOUT DIAGNOSIS A healthcare provider can confirm or rule out a diagnosis of
pseudogout by performing an examination and tests. In many patients, a sample of joint fluid is obtained in order to determine whether calcium pyrophosphate dihydrate (CPP) crystals are present
and to exclude arthritis due to other causes, such as gout or joint infection.
REFERENCE: uptodate

85. Chronic Gout > allopurinol


Answer : Allopurinol inhibition of uric acid production is in large part due to inhibition of xanthine
oxidase (xanthine dehydrogenase) by both the native drug and the active metabolite oxypurinol. Allopurinol and oxypurinol are pyrazolo-pyrimidine analogs of the purine bases hypoxanthine and xanthine, respectively>
REFERENCE: uptodate

86. About antiphospholipid from common


REFERENCE: First aid USMLE step 1

87. Acute management of gout ----------------- NSAID


Answer : INITIAL TREATMENT CHOICES The choice of medications depends upon the comorbidities that are present, the effectiveness of past treatments, patient preferences for use, and
the experience of the clinician with joint injection. Despite wide use in the treatment of acute
gout attacks, the various anti inflammatory agents have only infrequently been compared with
placebo or with each other in randomized trials. We take the following approach to the initial
treatment of acute attacks of gout (algorithm 1):
We treat most patients able to take an oral medication with a nonsteroidal antiinflammatory drug
(NSAID). (See 'NSAID therapy' below.)
An oral low-dose colchicine regimen may be used in patients who are able to take an oral medication but who have contraindications to NSAIDs (eg, moderate or more severe chronic kidney disease
[CKD], active peptic ulcer disease, or a history of NSAID-intolerance). (See 'Colchicine therapy' below.)
In patients with contraindications to the use of both NSAIDs and colchicine, we prefer intraarticular, oral, or parenteral glucocorticoids, depending upon the number of involved joints, the experience of the clinician with joint injection techniques, and the need, if present, for parenteral rather
than orally administered therapy. (See 'Glucocorticoids' below.)

REFERENCE: http://www.uptodate.com/contents/treatment-of-acute-gout

88. Which of the following is the most specific for Rheumatoid arthritis?

smle ,2016

177
A. HLA-DR4.
B. Rheumatoid factor.
C.

CRP.

D. Anti-cyclic citrullinated peptide. (Anti-CCP)


Answer: D (Anti CCP is the most specific for RA)
#TORONTO , Investigations
blood work
RF sensitivity ~80% but non-specific; may not be present at onset of symptoms
anti-CCP: sensitivity ~80% but more specific; may precede onset of symptoms
increased disease activity is associated with
89. Elderly female complaining of depression, bilateral shoulder and hip pain. Normal blood
workup.
A. polymyalgia rheumatica
B. fibromyalgia
Answer: B (Normal blood workup)

smle ,2016

178
90. Unwanted effect in an osteoporosis case, which medication was the cause?
Answer: ?
>> Warfarin and steroids can cause osteoporosis
91. pt. with recurrent oral and genital ulcers and arthritis when the pt taken some type of IM
vaccine develop sterile abscess at the site of injection. What is the most likely Dx ?
A. Behcet disease
Answer: A (Behet disease is characterized by a triple-symptom complex of recurrent Oral aphthous ulcers, genital ulcers, and uveitis. The disease is named after the Turkish dermatologist Hulusi Behet, who
identified it in a patient in 1924 and published a description of the disease in 1937).
REFERENCE: http://emedicine.medscape.com/article/329099-overview
92. The first initial treatment of osteoarthritis in middle aged patient is?
A. Intra articular steroids
B. Oral steroids
C. Stairs exercise
D. Muscle strengthening
Answer: D
REFERENCE: http://www.aafp.org/afp/2000/0315/p1795.html

93. Pt does not complain of anything, has sudden knee swelling, What is the best thing to do?
A. Aspirate
Answer: A
94. Pt with 1st metatarsal joint pain, redness and erythema, High temperatures, What is the
cause?
A. Staph areus
B. NA monourate crystal
C. Ca pyrophosphate crystal
Answer: ? (B came with gout or A because of high temp)
REFERENCE: http://worldwidescience.org/topicpages/a/acute+gouty+arthritis.html
95. Low back pain in the morning that resolves in 30 minutes, two cases, what to do?
A. Physiotherapy to strengthen muscles
Answer: A
96. It is a case of arthritis?
Osteoarthritis Hand
Cartilage loss with narrowing of interphalangeal joints
B: Bouchard nodes (osteophytes proximal interphalangeal joints)
H: Heberden nodes (osteophytes distal interphalangeal joints
97. A patient came to you complaining of morning stiffness of the PIP and DIP that decreases
and goes away with activity. On x-ray you observed a bone growth. What is the name of that
growth?
A. Heberden node
Answer: ? (depends on the X-ray given)

smle ,2016

179

98.

symmetric joint pain and swelling worse at morning, Dx?


A. rheumatoid arthritis
Answer: A
REFERENCE: http://www.webmd.com/rheumatoid-arthritis/guide/diagnosing-ra
99. A patient with SLE with rash on her cheeks, etc. What will you advise her?
A. avoid sun exposure as much as she can
Answer: A
REFERENCE: Medical diagnosis and Management by Mohammad Inam Danish
100. patient with OA which type of exercise is the best?
A. high repetition and
B. low repetition and
answer: A ?
101. Which of the following is a primary cause of osteoporosis?
A. Age
Answer: A
Primary type 1: most common in postmenopausal women,
due to decline in estrogen, worsens with age
Primary type 2: occurs after age 7, seen in females and males at 2:1 ratio,
Possibly due to zinc deficiency
REFERENCE: Toronto Notes
102. old lady with osteoarthritis and risk for osteoporosis , what you will given?
A. calcium, TSH, dihydroxy vit.D
B. bisphosphonate, vit.D, calcium
Answer: B
103. 70 year old smoker woman with low vit-D and osteoporosis. Which of the following has the
highest risk for osteoporosis?
A. Smoking
B. Age
C. Vit.D
Answer: B
104.

Pt does not complain of anything, has sudden knee swelling? What is the best thing to do?

smle ,2016

180
Answer: ?
Treatment will depend on the cause of the swollen knee, but the most common ways to reduce the
swelling are:
1)ICE: can be used to slow down the blood flow and therefore reduce swelling and pain. It is important
to use it properly otherwise it can make things worse, see the Ice Therapy section for more details and
the ice wraps section for the best ways to apply ice
2)Compression: Tubigrip and knee braces can be used to provide compression to the knee which helps
reduce swelling. Click the links to find out more
3)Medication: Non-steroidal anti-inflammatories e.g. ibuprofen may be prescribed to reduce the knee
swelling
4)Aspiration: Water on the knee can be drained by your doctor with a needle, but it does sometimes
come back
5)Cortisone Injections: Cortisone is a steroid hormone that suppresses the immune system, reducing inflammation and pain
6) Elevation: Keeping the leg elevated, ideally with the knee higher than the level of the heart can help
treat a swollen knee as gravity draws the fluid down away from the knee
7) Rest: Reducing your activity levels helps to take pressure of the knee which can reduce swelling otherwise the knee keeps getting irritated
105. Patient is concerned about osteoporosis as her mother had it, what you will do?
A. give vit.D, calcium
B. give estrogen postmenopausal
Answer: A

106. similar scenario 37 years old presented with back pain. On examination there was tenderness when palpating paraspinal muscles, neurovascular exam was normal. What is the treatment?
A. Physiotherapy
Answer: A
107. 2 questions has the same idea with the same options about polymyalgia rheumatica (they
mention that ESR was high):
A. PMR
Answer: A
108. Most common cause of itching?

smle ,2016

181
A. eczema
B. bile salt can't recall the rest
Answer: A, I think it is eczema
109. 70 years old female patient with osteoporosis what is the treatment ?
A. ESTROGEN
B. BIOPHOSPHATE
Answer: B (For the treatment of osteoporosis in postmenopausal women, we suggest oral bisphosphonates as first-line therapy. We prefer oral bisphosphonates as initial therapy because of their efficacy)
REFERENCE: UPTODATE
110. Patient with back pain that improves with walking, you find that it is muscle strain, how will
you treat him?
A. Physiotherapy
B. Surgery
Answer: A
111. Adolescent male with swollen parotid and salivary gland with dry eye and dry mouth. labs
HLA, ANA and RF are positive. which of the following is appropriate treatment?
A. Physostigmine
B. Artificial eye and saliva drops
Answer: B

112. well circumscribed lesion on erythematous base, arthritis?


Rheumatological disease
Subacute cutaneous lupus erythematosus can present in an annular form on sun exposed surfaces or in
a papulosquamous form
Subacute cutaneous lupus erythematosus, Annular erythematous plaques have central clearing, often
mimicking annular psoriasis when associated with scales.
REFERENCE: http://www.aafp.org/afp/2001/0715/p289.html

smle ,2016

182

113. Rheumatoid arthritis with the loss of bones in joint, What is the cause?
A. substance released by synovial cell
B. synovial fluid pressure
Answer: A, TNF
114. bilateral shoulder and hip stiffness and pain what is Dx?
A. polymyalgia rheumatica
B. OA
Answer: A Polymyalgia rheumatica (PMR) is characterized by aching and morning stiffness in the
shoulders, hip girdle, neck, and torso in patients over the age of 50
REFERENCE: Uptodate: http://goo.gl/ZZPjlI
115. patient with numbness of index finger when he scissor, What is Dx?
A. OA
B. Ducyptus
Answer: Trigger finger, tenosynovitis
116. q about latent sle:
https://en.wikipedia.org/wiki/Undifferentiated_connective_tissue_disease
117. Case of juvenile arthritis & ttt of JRA:
Answer: Oligoarticular juvenile idiopathic arthritis (JIA) is usually responsive to intra-articular glucocorticoids. Methotrexate and other immunosuppressive drugs are recommended for children with disease
that extends to involve five or more joints or require repeat injections. Biologic agents are typically reserved for patients with uveitis and are also used in some patients with extended oligoarticular JIA.
systemicJIA
Mild-to-moderate disease nonsteroidal anti-inflammatory drug (NSAID) monotherapy as the
initial treatment in children with possible sJIA who have mild-to-moderate, non disabling symptoms on presentation and no evidence of macrophage activation syndrome (MAS)
Moderate-to-severe disease For patients whose initial symptoms include high fevers, other
systemic manifestations including serositis and possible early MAS, and/or moderate-to-severe
polyarthritis, we suggest adding one of the biologic agents that inhibit interleukin (IL)-1 or IL-6,
such as anakinra, canakinumab, or tocilizumab, rather than the nonbiologic disease-modifying

smle ,2016

183
antirheumatic drug (DMARD), methotrexate. NSAIDs can be continued in conjunction with other
agents if needed for pain control.
REFERENCE: uptodate
118. Henoch schonlein purpura:
A. IgA Vasculitis
Answer: A

GASTROENTEROLOGY
1. Which disease involves antibiotic in treatment regimen?
a.Crohn
b.Ulcerative colitis
c.Celiac
d.Whipple
Answer: D
Reference: http://emedicine.medscape.com/article/183350-treatment
2. Splenectomy, what vaccin he should took ?
Answer: pneumococcal vaccination
Reference: (master the board )
3. Enal or small cell cancer stage III with bone pain, what is the immediate action?
A. MRI only
B. Radiotherapy
C. Iv steroid and MRI
D. No immediate action
Answer: A
4. Symptomatic pt. With Positive HBV antigen?
A. Acute hepatitis
B. Chronic hepatitis
C. Active carrier
D. Non active carrier
Answer: A/C ??
Reference: http://www.cdc.gov/vaccines/pubs/surv-manual/chpt04-hepb.html
Transient HBsAg positivity (lasting <18 days) might be detected in some patients during vaccination.
hepatitis B surface antigen (HBsAg)
This tests for the presence of virus. A "positive" or "reactive" HBsAg test result means that the person is
infected with the hepatitis B virus, which can be an "acute" or a "chronic" infection.

hepatitis B surface antibody (HBsAb)

A "positive" or "reactive" HBsAb (or anti-HBs) test result indicates that a person has successfully responded to the hepatitis B vaccine or has recovered from an acute hepatitis B infection.

smle ,2016

184

hepatitis B core antibody (HBcAb)

A "positive" or "reactive" HBcAb (or anti-HBc) test result indicates a past or present infection, but it
could also be a false positive.
5. Primary biliary cirrhosis options were about pathophysiology:
Answer: intrahepatic - T lymphocyte mediated attack on small intralobular duct
6. Abdominal solid mass ( renal i guess but not sure ) confirmed by?
A. CT
B. MRI
C. US
Answer: I think A (CT scan) >> read about it
7. Women coming with elevated indirect bilirubin:
a.Rotor syndrome
b.Crigler Najjar
c.Dubin Johnson
Answer: B
Differential diagnosis:
Unconjugated (indirect) hyperbilirubinemia: Hemolytic, physiologic (newborns), Crigler-Najjar,
Gilbert syndrome
Conjugated (direct) hyperbilirubinemia:
1- Biliary tract obstruction: gallstones, cholangiocarcinoma, pancreatic or liver cancer, liver fluke
2- Biliary tract disease: 1sclerosing cholangitis, 1biliary cirrhosis
3- Excretion defect: Dubin-Johnson syndrome, Rotor syndrome
Mixed (direct and indirect) hyperbilirubinemia: Hepatitis, cirrhosis
8. Peptic ulcer patient with Anemia, what you will do regarding his anemia?
a.
Oral iron supplement
b.
I.M iron
c.
Blood Transfusion
Answer: B
Reference: uptodate
9. Which one of the following can cause liver cirrhosis?
Answer: ?
Fatty liver (alcohol, metabolic syndrome)
Chronic viral hepatitis (B, B+D, C; not A or E)
Autoimmune hepatitis
Hemochromatosis
1-antitrypsin deficiency
Primary biliary cirrhosis
Chronic hepatic congestion
1.cardiac cirrhosis (chronic right heart failure, constrictive pericarditis)
2. hepatic vein thrombosis (Budd-Chiari)
Idiopathic

smle ,2016

185
Rare: Wilsons disease, Gauchers disease
Reference: Toronto Notes.
10. Most common hepatitis is?
A- HBV
B- HCV
C- HDV
D- HEV
Answer: A
Reference: Medscape:
Whereas HAV is the most common cause of acute hepatitis in the United States, HCV is the most common cause of chronic hepatitis.
The most common type is HCV, and according to the answer of gastro consultant that HBV is the commonest type in KSA.
11. Which common feature of IBS?
A-diarrhea
B-constipation
C-vomiting
answer : A or Abdominal Pain relieved by defecation
Reference: http://www.ncbi.nlm.nih.gov/m/pubmed/21929652/
12. How to differentiate large ovarian cyst from ascites?
A. Dull anteriorly resonant laterally
B. Resonant anteriorly and dull laterally
Answer:A (midabdominal dullness and lateral tympany)
Reference:
https://books.google.com.sa/books?id=qW4IKSfM8x8C&pg=PA472&lpg=PA472&dq%23v=onepage&q&f=false

13. Asymptomatic patient, known case of chronic gastritis, has positive occult blood stool and his
Hb=9, You will manage him by?
a.
IM iron
b.
Oral iron
c.
Erythropoietin

smle ,2016

186
d.
Blood transfusion
Answer: A (IV iron would be more appropriate answer)
14. Similar question in another exam with different choices:
Elderly man on NSAIDs developed dyspepsia. Endoscopy showed gastritis.
Labs showed iron deficiency anemia with Hb= 9. What is the treatment?
A. IV iron
B. IM iron
C. Erythropoietin
D. Oral Iron
Answer: A
Oral ferrous sulfate associated with a significantly higher risk of GI side effects than IV iron.
Acquired malabsorption for iron with autoimmune atrophic gastritis or Helicobacter pylori infection.
Reference: http://www.uptodate.com/contents/treatment-of-the-adult-with-iron-deficiency-anemia
If the patient tolerated orally you should start with oral iron then IV iron if he doesn't tolerate! (hematologist consultant)
15. Patient with Crohn's disease. Most relevant and associated with CD?
A. Positive Family History
B. Smoking
C. Alcohol
Answer: A
Positive family history is the largest independent risk factor for CD and UC
Reference: (Kumar)
16. Pt. with HBA after 3 week we take bipsy, what is show?
A. Normal architecture
B. Fibrosis something
C. Another something
Answer: A
17. Asymptomatic pt. with Positive HBV antigen?
A. Acute hepatitis
B. Chronic hepatitis
C. Active carrier
D. Non active carrier
Answer: D
INACTIVE carrier, asymptomatic, carry HBV antigen more than 6m and HBVeAB
18. What gene is related to Celiac Disease?
Answer: ?
HLA-DQ2 (chromosome 6) found in 80-90% of patients compared with 20% in general
population; also associated with HLA-DQ8.
Reference: (Toronto Notes)
The risk of developing celiac disease is increased by certain variants of the HLA-DQA1 and HLA-DQB1
genes.
Reference: (https://ghr.nlm.nih.gov/condition/celiac-disease#genes)

smle ,2016

187

19. Old patient that presented with abdominal pain from time to time starts in the left mid abdomen
radiates to the back, whenever he have the attacks, he lies down on that side and bend his body position like a baby, no vomiting, diarrhea or weight loss, what is the diagnosis?
A- duodenal ulcer
B- gastric ulcer
C- chronic pancreatitis
D- mesenteric thrombosis
Answer: C
20. Female pt diagnosed with IBD on ceftriaxone with no benefit, wt is the organism?
A. herpes
B. can't recall others
Answer: Pseudomonas aeruginosa
Ceftriaxone does not have useful activity against Pseudomonas aeruginosa. It is generally not active
against Enterobacter species and its use should be avoided in the treatment of Enterobacter infections
even if the isolate appears susceptible because of emergence of resistance. Some organisms, such as
Citrobacter, Providencia, and Serratia have the ability to become resistant through development of
cephalosporinases (these enzymes hydrolyze cephalosporins and render them inactive)
21. Characteristic of perforated duodenal ulcer?
A - Mid epigastric pain
B - Steatorrhea
C - Melena
Answer: A
Symptoms include sudden, severe abdominal pain, a rapid heartbeat, and a low body temperature. Pain
may radiate to one or both shoulders, and the abdomen may become rigid. The abdominal pain is usually sudden, sometimes producing collapse or syncope. Localization is usually epigastric, but it quickly
becomes generalized.
Reference: uptodate
22. Patient with watery diarrhea I think:
If asking about the most common cause: so choose the ( rotavirus)
Reference: http://www.medicinenet.com/rotavirus/article.htm
23. About which marker indicate chronic hepatitis B?
A.IgM
B.HBsAg
Answer:
The diagnosis of chronic HBV infection is based on persistence of HBsAg for more than six months, IgG
anti-HBc is positive, while IgM anti-HBc is negative.
Reference: medical diagnosis and management book ( moh. Danish)
http://www.uptodate.com/contents/overview-of-hepatitis-b-virus-infection-in-children-and-adolescents?source=preview&language=en-US&anchor=H6&selectedTitle=1~150#H6
24. Peptic ulcer medication cause erectile dysfunction and decrease lipido?
A. Cimetidine

smle ,2016

188
Answer: A
Reference: http://www.webmd.com/erectile-dysfunction/guide/drugs-linked-erectile-dysfunction
25. Female using NSAID for her dysmenorrhea developed epigastric pain, most likely Dx?
Answer: gastritis
26. What does it mean when the patient has positive Hbs Ag?
Answer:
HBsAg: This tests for the presence of virus. A "positive" or "reactive" HBsAg test result means that the
person is infected with the hepatitis B virus, which can be an "acute" or a "chronic" infection. Infected
people can pass the virus on to others through their blood and infected bodily fluids.
Reference: https://www.cdc.gov/hepatitis/hbv/pdfs/serologicchartv8.pdf
27. If the patient has Hbs Ag +ve and IgM +ve, what do you treat the patient with?
A- Interferon
B- Lamivudine
Answer:
#Toronto: treatment options: interferon, tenofovir, entacavir, lamivudine, adefovir
#Medscape:
interferon a and etacavir ~> 1st line agents
Lamivudine, adefovir: historical, 2nd, 3rd line agent
Adeponectin (a fat cell hormone that increase insulin sensitivity): medscape, Resveratrol, L-carnitine
28. Hereditary chronic pancreatitis is?
A- autosomal dominant
B- polygene
Answer: A
Reference: https://www.uptodate.com/contents/hereditary-pancreatitis

29. What is the oncogene for pancreatic ca?


a.
Ki-Ras
b.
Myc
Answer: A
There are abnormalities in the structure and/or function of several oncogenes and growth factors in human pancreatic cancer, notably the EGF receptor and its ligand TGF alpha, c-erb B-2 proto-oncogene, Kiras oncogene and the tumour suppressor gene p53.
Reference: http://www.ncbi.nlm.nih.gov/pubmed/1964102
30. Celiac disease affects which of the following locations?
A. Distal SB
Answer: ?
Affect the upper small bowel (Duodenum and proximal jejunum) more than lower small bowel.
Reference: WebMed website and digestive disease center

smle ,2016

189
31. Female patient complaining of constipation, weight gain and fatigue. What is the most likely diagnosis?
A. Hypothyroidism
Answer: A
32. Scenario; pt. came with melena and bleeding take NSAID for 3 weeks
what is the cause?
Answer: NSAID use is associated with an increased risk of gastric or duodenal ulcer by inhibition of prostaglandins that leads to symptomatic chronic ulceration ,also inhibit platelet function, and their use has
been associated with GI bleeding from throughout the GI tract.
Reference: http://www.uwgi.org/guidelines/ch_07/ch07txt.htm
33. Old patient that presented with abdominal pain from time to time starts in the left mid abdomen
radiates to the back, whenever he have the attacks, he lies down on that side and bend his body position like a baby, no vomiting or diarrhea or what weight loss, what is the diagnosis?
a- duodenal ulcer
b- gastric ulcer
c- chronic pancreatitis
d- mesenteric thrombosis
Answer: C
35. Case of guillain barre syndrome, what you will find in LP ?
Answer: A- high protein
Reference: http://emedicine.medscape.com/article/315632-workup#c13
36. Pt with constipation, increase weight, thinning of hair?
A- Hypothyroidism
Answer: A
37. Hx of infection, Hb low, wbc high, what is the investigation?
A- Bone marrow biopsy
B- Hb electrophoresis
Answer: !
I think A >> bone marrow biopsy
38. Paracetamol overdose after 24 hrs what happened to liver?
Answer: depletes the liver , natural antioxidant glutathione
antacid cause constipation & aluminium hydroxide
39. Liver biopsy taken at level of intercostal space:
a- Fifth
b- Seventh
c- Ninth.
d- Eleventh.
Answer: B

smle ,2016

190
Proper identification of the biopsy site is of paramount importance. To choose the site, start with percussion over the right upper quadrant. The biopsy site is usually located in the seventh or eighth intercostal space in the midaxillary line. The site can be further confirmed with either routine ultrasonography or a bedside portable ultrasound machine.
Reference: medscape
40. Patient history of travail c/o bloody diarrhea?
A. Amebiasis
Answer: A
Reference: http://www.healthline.com/health/amebiasis#Causes3
41. What Antacid causes constipation?
A. Aluminum hydroxide
Answer: A
42. What is most likely diagnosis in patient came for regular check and found to have HBsAg? no
more details;
a-Acute
b-Chronic
c-Carrier inactive
Answer: A
43. A 6 years of positive HBV mother not taken any vaccine except BCG after delivery. What will you
give him?
a-MMR,OPV,HBV,Varicella
b-HIB, MMR,OPV,HBV,PCV
c-HIB, MMR,OPV,HBV,.
Answer = MMR , OPV , Varicella , PCV, HIB All can be given so choose the answer that dont include HBV
44. Women not oriented and confused her husband have HBV Ig titer is -ve Direct bilirubin is high
(55) what will you do to confirm Dx?
Answer: IgM
45. Which type of hepatitis have available vaccine?
A - Hep B
Answer: A
Hepatitis A and B
46. What is the treatment of choice for Traveler's diarrhea?
a.Ciprofloxacin.
b.Amoxicillin.
c.Metronidazole.
Answer: A
The most common organism is enterotoxigenic E. coli and it is treated with fluoroquinolones (e.g.
Ciprofloxacin)
Reference: Uptodate

smle ,2016

191
Initial treatment: rehydration, the Abx only will decrease the duration of symptoms; 1st line: antibiotics
include fluoroquinolones, such as ciprofloxacin or levofloxacin or ofloxacin; 2nd line: azithromycin; 3rd
line: Rifaximin
47. Triple therapy for gastric ulcer in 10 yo boy; Ppi + metro + ?
Answer: treatment of PU is the same for adults and children
PPI-based triple therapy regimens for H pylori consist of PPI, amoxicillin, and clarithromycin for 7-14
days
48. Oral leukoplakia that cant be swiped off. Most likely to be?
a-Dysplasia
Answer: its Hyperplasia of squamous cells >> dysplasia >> Carcinoma in situ
I am not sure about dysplasia , according to medscope : The plaques may show hyperorthokeratosise or
hyperparakeratosise
49. Kwashiorkor disease is due to which kind of diet?
a-High carb, low protein diet
b-Low carb, high protein
Answer: A
Reference: http://emedicine.medscape.com/article/1104623-clinical#b5
Kumar Clarks Clinical Medicine,7th , P 215
50. Pt presents with black vomiting after ingesting large amount of medication (suicidal attempt).
What is the most likely medication?
Answer: Iron toxicity
#initial features are characterized by
- Nausea.
vomiting (the vomit may be grey or black in colour).
abdominal pain.
- diarrhoea.
Reference: Kumar Clarks Clinical Medicine,7th , P 944
51. How to dx hepatitis B ?
Answer :
HBsAg, hepatitis B core antibody (anti-HBc), and hepatitis B surface antibody (anti-HBs)
Reference: http://www.uptodate.com/contents/diagnosis-of-hepatitis-b-virus-infection?source=see_link#H9

smle ,2016

192

Kumar Clarks Clinical Medicine,7th , P 337


52. Pt with absence seizure the doctor wants to start him on sodium valproate. what test should be
done before starting the tx?
A) Liver function ,
B) creatinine ,
C) urea ..
Answer: A
Side effects of VPA include nausea, vomiting, hair loss, easy bruising, and tremor
VPA is associated with weight gain, obesity, insulin resistance, and the metabolic syndrome VPA can also
cause thrombocytopenia and other coagulation disturbances and has also been associated with subclinical hypothyroidism with mild to moderate elevations in thyrotropin (TSH) levels
VPA has also been linked to the polycystic ovarian syndrome. A number of case reports have linked VPA
to Fanconi syndrome in children with severe disability
VPA-exposure in utero is associated with major malformations and other adverse effects, including neurodevelopmental abnormalities. VPA should be avoided in pregnancy when possible
Approximately 5 to 10 percent of patients developed ALT elevations during long term VPA therapy; most
of the time these abnormalities are asymptomatic and can even resolve with continuation of the drug.
In addition, there are more serious forms of hepatotoxicity that can occur with VPA:
_VPA-related hyperammonemic encephalopathy (VHE)
_Acute hepatocellular injury with jaundice can occur
****Although routine monitoring of hepatic function has not been shown to permit early identification
of serious toxicity or improve outcome, many physicians choose to obtain liver function tests (LFTs) once
or twice a year in patients who are clinically asymptomatic. The FDA recommends checking LFTs prior to
initiating treatment and at frequent intervals thereafter, especially during the first six months
Reference: UPTODATE
53. Gastroenteritis ddx:
Answer:
DIFFERENTIAL DIAGNOSIS The differential diagnosis of acute viral gastroenteritis includes other
causes (infectious and non-infectious) of acute diarrhea [12,14]. Diarrhea that lasts over a week in an
individual with a history of travel, hiking, or oral-anal sexual activity should prompt evaluation for protozoa such as Giardia and cryptosporidium [9]. Recent antibiotic use or hospitalization should prompt consideration of Clostridium difficile infection. Common foodborne illnesses (eg, Staphylococcus aureus)
need to be considered, particularly when the incubation period is shorter than is typical for viral illness

smle ,2016

193
(ie, within 8 to 16 hours). The presence of alarm symptoms or signs should prompt further investigation
for an alternate diagnosis.
Causes of chronic diarrhea that may less commonly masquerade as acute viral gastroenteritis include:
colorectal cancer, irritable bowel syndrome, inflammatory bowel disease, microscopic colitis, malabsorption syndromes, post-cholecystectomy related diarrhea, medication-induced diarrhea, laxative
abuse, and chronic infections. Patients with acute viral gastroenteritis may also present with isolated
vomiting without prominent diarrhea. Clinicians should consider adverse effects of medications and
acute vestibular disorders in the differential diagnosis of these patients.
Reference: uptodate
54. Treatment of HCV?
Answer:
#chronic :
Pegylated Interferon + ribavirin.
#Acute :
Interferon has been used in acute cases to prevent chronic disease. Needle-stick injuries must be followed and treated early if there is evidence of HCV viraemia, usually re-tested for at 4 weeks.
Reference: Kumar Clarks Clinical Medicine,7th , P 340 , 341
55. 40 year old patient complaining of weight loss, nausea, lethargy and jaundice. When he was
asked about similar attack in the past, he mentioned 4 episodes during the past two years. What is
the most Likely diagnosis?
a. Acute pancreatitis.
b.Cancer head of pancreas.
c.Peptic ulcer disease.
d.Chronic pancreatitis.
Answer: B
The most characteristic sign of pancreatic carcinoma of the head of the pancreas is painless obstructive
jaundice.
Reference: http://emedicine.medscape.com/article/280605-clinical
Toronto Notes 2015,GS51
56. 27 year old smoker who was studying in a foreign country for two years and lived in a student
housing. He returned home two months ago. The patient complains of 4 day mid epigastric pain, what
is the most likely diagnosis?
a. Viral Hepatitis.
b. H. pylori infection.
c. Acute Pancreatitis.
d. Myocardial infarction.
Answer: B
Epigastric pain is the most common symptom of both gastric and duodenal ulcers. It is characterized by
a gnawing or burning sensation and occurs after meals classically, shortly after meals with gastric ulcer and 2-3 hours afterward with duodenal ulcer.
Reference: http://emedicine.medscape.com/article/181753-overview
57. Enteric fever best diagnosed in the first week of presentation by?
A-blood culture
B-stool culture

smle ,2016

194
C-multiple something?
D-bone marrow>>not sure
Answer: A
Reference: Kumar Clarks Clinical Medicine,7th , P 144
58. ASA high dose with GIT complication how to treat or avoid it?
A. Cimetidine
B. Misoprostol
Answer: PPI (the best) if not in the choices H2 blocker (cimetidine)
Proton pump inhibitors the best drugs to protect those at high risk from serious gastrointestinal
events + reduce dyspepsia.
H2 blockers less effective as gastroprotective agents.
prostaglandin E2 analogues has limited effect by their tendency to cause nausea and diarrhoea.
Reference: Kumar Clarks Clinical Medicine,7th , P 518
59. First sign of portal HTN?
A- splenomegaly
B- hepatomegaly
C- ascites
Answer : C
60. Celiac pt, what is safe for him?
A. Rice
Answer : A, rice is safe for celiac patients
Reference: http://wheat.pw.usda.gov/ggpages/topics/Celiac.vs.grains.html
gluten free diet: avoid barley, rye, wheat
- oats allowed if not contaminated by other grains
rice and corn flour are acceptable
iron, folate supplementation (with supplementation of other vitamins as needed)
Reference: Toronto Notes 2015, G18
61. Aspirin works in COX will lead to :
A. Decrease Thromboxane
B. Leukotriene
C. and others i can't remember
Answer: A
Irreversibly inhibit acetylates COX inhibiting TXA2 synthesis inhibiting platelet aggregation

Reference: Kumar Clarks Clinical Medicine,7th , P 443 + 753


62. aspirin toxicity lead to?
A. Resp. Alkalosis followed by metabolic acidosis
B. metabolic acidosis followed by Resp. Alkalosis

smle ,2016

195
C. Metabolic alkalosis followed by resp. acidosis
Answer: A
Reference: http://emedicine.medscape.com/article/1009987-overview
63. Food poisoning , investigation shows gram positive cocci?
A. Staph aureus
B. Shigella
C. Salmonella
Answer :A
Reference: Kumar Clarks Clinical Medicine,7th , P 123 ,130,131, 133

64. Best treatment for primary biliary cirrhosis is?


A. azathioprine
B. Ursodeoxycholic acid
Answer: B
Ursodeoxycholic acid (improves bilirubin and aminotransferase) + Malabsorption of fat-soluble vitamins (A, D and K) (when deficiency is detected and prophylactically in the jaundiced patient) + Bisphosphonates (for osteoporos) + colestyramine (helpful for Pruritus)
Reference: Kumar Clarks Clinical Medicine,7th , P355

65. pt
distended
abdomen
A. liver
B. Heart
Answer: A

with bilateral Lower limb edema +


abdomen + + O/E small vessels on

cirrhosis
failure

Reference:
P329

Kumar Clarks Clinical Medicine,7th ,

smle ,2016

196
66.

Case about IBD Which cell type responsible about ulceration in intestine ?
a) B CELL
b) T CELL
Answer: b
Reference: Kumar Clarks Clinical Medicine,7th , P287

67.

Pt on NSAID with signs of ulcer, what you will do?


A. Triple therapy
B. Urea breath test
C. Immediate endoscope
Answer: B
68. Child postmeal complain of nausea, abdominal distention, cramps, she
oriented and cooperative. this symptom after he came from school 1
month ago. Vital sign at admission normal but after that will be decrease BP
Normal temperature , How to reach diagnosis?
A. history of food intolerance
B. pus and stool examination
C. Stool culture
Answer : A
69. Rota confirmatory diagnosis?
A- Serum antibody
B- Stool antigen
C- Stool leukocyte
Answer: B
The diagnosis can be established by PCR for genome detection or ELISA for the detection of rotavirus
antigen in faeces and by electronmicroscopy of faeces. Histology of the jejuna mucosa in children shows
shortening of the villi with crypt hyperplasia and mononuclear cell infiltration of the lamina propria
Reference: Kumar Clarks Clinical Medicine,7th , P112

70. Man present with epigastric pain for 3 month worse after eat diagnosed as h
pylori triple treatment given to him what is the best indicator for his improvement?
A. endoscope
B. ph
C. blood test for h pylori
D. clinical improve
Answer: 13C , urea breath test or stool test for H. pylori (for active Hp) 6 weeks after end of treatment
to be sure eradication is successful
Reference: Kumar Clarks Clinical Medicine,7th , P112

smle ,2016

197
71. 19 year old female, presenting with abdominal pain, diarrhea, bloating, improved with defecation, diagnosis?
A. IBD
B. IBS
C. Celiac
Answer: B
pain or discomfort associated with two or more of the following:
1 improvement with defecation
2 onset associated with a change in frequency of stool
3 onset associated with a change in form (appearance) of stool
Reference: Kumar Clarks Clinical Medicine,7th , P312

72. 30 years with chronic diarrhea and ataxia and abnormal movements. Jejunal Biopsy showed: T.
Whipple infection. Management?
A. Short term antibiotics
B. Steroids
C. Long term Antibiotics
Answer: C
Treatment is with antibiotics which cross the bloodbrain barrier, such as trimethoprim and sulphamethoxazole daily for 1 year. This is often preceded by a 2-week course of streptomycin and penicillin or
ceftriaxone. Treatment periods of less than a year are
associated with relapse in about 40%.
Reference: Kumar Clarks Clinical Medicine,7th , P281

73. Pt with anemia, high bilirubin, positive direct and indirect coombs, photo of smear showing
spherocytosis, wts the dx:
A. AIHA
B. spherocytosis
Answer: A
Dirct coombs test specific for Extravascular Hemolysis
Reference: Kumar Clarks Clinical Medicine,7th , P416
Toronto Notes 2015,H12

74. Young female came with jaundice, disoriented, tremor. Husband has HBV, lab -ve. LFT slightly
elevated What lab to order next?
A. serum copper
B. serum level of ceruloplasmin
C. HBV core
Answer: B

smle ,2016

198
75. Long scenario about patient with peptic ulcer disease you give him PPi + amoxicillin , what can
you add to them ?
A- clarithromycin
Answer: A
Reference: Kumar Clarks Clinical Medicine,7th , P216

76. Question about splenic sequestration ( with lobar infiltrate in lung )??Incomplete question

77. Patient presented to primary health care with his annual health visit, In lab result: HBsAg is (+ve)
& quot; this is the only test that written in this question, The patient has?
A- Acute HBV
B- Chronic HBV
C- Chronic HBV Carrier
Answer: C
patients are usually discovered incidentally on blood tests, such as when they are screened for donating
blood for transfusion or when attending genital medicine or antenatal clinics. Patients with inactive
chronic hepatitis have HBsAg in their serum.
Reference: Kumar Clarks Clinical Medicine,7th , P338

78. Case about pancreatitis what is the treatment? + 79. Pt with abdominal tenderness and high
amylase, lipase what is the management?
#Acute pancreatitis :
Nasogastric suction.
Oxygen administration.
Prophylactic antibiotics.
Analgesia ( Pethidine and tramadol are drugs of choice).
Little oral feeding. (Total parenteral nutrition has been associated with a high risk of infection).
Anticoagulation with a low molecular weight heparin for DVT prophylaxis.
#chronic pancreatitis :
In patients with alcohol-related chronic pancreatitis long term abstinence is likely to be of benefit.
Reference: Kumar Clarks Clinical Medicine,7th , P379, 381

80. How to diagnose Giardia lamblia?


A- three consecutive stool analysis
B- three separate stool analysis
Answer: B
multiple stool samples (daily x 3 d) for microscopy, stool antigen used occasionally, since the parasite may be excreted at irregular intervals
occasionally small bowel aspirate or biopsy

smle ,2016

199
Reference: Kumar Clarks Clinical Medicine,7th , P162
Toronto Notes 2015, G18

81. Patient with bilateral abdominal mass?


A. Polycystic kidney disease
Answer: A
cystic renal disease present as solitary or multiple renal cysts in the kidneys
Reference: http://emedicine.medscape.com/article/244907-overview
Kumar Clarks Clinical Medicine,7th , P642

82. Patient presented with severe vomiting. his labs showed hypocalcaemia.
what is your management?
A. furosemide
B. hydration
Answer: B
Loss of water and electrolytes, should be treated by replacement of the loss. If possible, this should be
with oral water and sodium salts
Reference: Kumar Clarks Clinical Medicine,7th , P660

83. Patient with AST very high, the cause?


A. Alcohol
Answer: A
Aspartate aminotransferase (AST) is primarily a mitochondrial enzyme (80%; 20% in cytoplasm) and is
also present in heart, muscle, kidney and brain. High levels are seen in hepatic necrosis, myocardial infarction, muscle injury and congestive cardiac failure
Reference: Kumar Clarks Clinical Medicine,7th , P324
84. MVA when to do diagnostic peritoneal lavage?
A- Hypotensive patients.
B- All MVA patients.
C- Unconscious patients with severe head injury.
D- Conscious patients with abdominal pain.
Answer : A
DPL can be used to evaluate both blunt and penetrating abdominal trauma in patients who are hemodynamically unstable or who require urgent surgical intervention for associated extra-abdominal injuries.
DPL can rapidly confirm or exclude the presence of intraperitoneal hemorrhage. Thus, the patient with a
closed head injury, the unstable patient who has been in a motor vehicle accident, or the patient with a
pelvic fracture and potential retroperitoneal hemorrhage can be appropriately triaged to emergency
laparotomy
Reference: http://emedicine.medscape.com/article/82888-overview
85. The cause of Crohn's disease?

smle ,2016

200
A. unknown (sure 100%)
Answer: A
86. HELLP syndrome?
A- Hypertension,........, low enzyme
B- Hypertension,........, high enzyme
C- Hemolysis , Elevated liver enzyme , low platelet
Answer: C
87. Patient with IBD, mild to moderate, what is the mainstay of treatment?
A- Surgery
B- Antibiotics
Answer:
Read about it
Reference: http://www.mayoclinic.org/diseases-conditions/pseudomembranous-colitis/symptomscauses/dxc-20169332
88. Case of lactose intolerance?
Answer: http://www.medicinenet.com/script/main/mobileart.asp?articlekey=7809
89. What hepatitis can be prevented by vaccination?
A. A
B. B
C. C
D. D
Answer: B
90. Pt came for general check up , u found + HBsAg , ( insufficient data ?! )
A. Acute hepatitis
B. Acute hepatitis carrier
C. Chronic hepatitis
Answer: I think C
91. Pt c/o liver cirrhosis and ascites, now he c/o wt loss, what should u do?
A. CEA
B. CA125
C. abdominal Us
D. Alphabetic Protein
Answer: I think C
92. Mountain climber who has hypoxia, which of the following liver zones is most affected by hypoxia?
A- Central of acini zone II
B- Peripheral of acini zone II
C- Sinusoidal
Answer:
Zone 1 encircles the portal tracts where the oxygenated blood from hepatic arteries enters.

smle ,2016

201
Zone 3 is located around central veins, where oxygenation is poor. >>most susceptible to hypoxic injury

Zone 2 is located in between.


93. MVA, femur fracture, high ALP, what would confirm that this elevation in ALP is not Hepatic in
origin?
A. Glucose 6 phosphatase
B. kinase
C. ALT liver
D. GGT live
Answer: D
94. A patient had weight gain, because she cant taste the food or smell. Examination is normal, she
was seen by neuro and psychiatry with no diagnosis. Whats her Diagnosis?
A.Meningioma
B.Aneurysm
C.Malingering
Answer: C
The most common goals of people who malinger in the emergency department are obtainingdrugs and
shelter. In the clinic or office, the most common goal is financial compensation and choices.
A &; B can be diagnosed easily.
Reference: http://emedicine.medscape.com/article/293206-clinical
95. Elderly with abdominal tenderness and dilated bowel loops?
I think it is a case about intestinal obstruction, read about it.
96. Vit B3 deficiency?
I think it is a case about pellagra, read about it.
97. Pt male 50s i think with chronic epigastric abd pain, last week lost 7 kg, +ve stool occult blood;
A. Chronic pancreatitis
B. Chronic cholecystitis
C. Chronic gastritis
Answer: i think due to significant wt loss, should think about malignancy, but the case is not clear
98. Man in acitamphinicol drink 2 glass of win every week LFT (high) and bilirubin (high) dx?
A. Alcoholic hepatitis
B. Drug induced hepatitis
C. Whipple
Answer: A
99. Young girl with jaundice (and I think liver failure) her color turned from yellow to green, why?
A. Hypoxia
B. I don't remember the rest
Answer:

smle ,2016

202
100. Less complicated of NSAID drugs in PUD?
101. A patient came to you with severe diarrhea and vomiting for several days the patient is untalkative and you can't take detailed history from him
bp in standing 80/60 , bp in sitting 120/80, what cause of fluid loss?
A decrease extracellular fluid
B decrease extracellular glucose
C decrease intracellular fluid
D diluted hyponatremia
Answer: I think it is A
102. 80 year old with symptom of vit b12 deficiency, What to do first?
A. Vitamin b12
B. Reticulocyte count
Answer: I think it is A
103. Pt with history of GERD, required aspirin for the treatment of rheumatoid
Arthritis, what medication youre going to add next?
Answer: Misoprostol
104. Senario consistent with sialadenolithiasis (the pain in the submandibular
Area) they are asking about the location of obstruction?
A. Submandibular bcz the pain in the involved area
B. Sublingual
C. Parotid
D. Submental
Answer: A
105. What is maximum normal 2 h postprandial blood glucose?
A. 8
B. 9
C. 10
D. 11
Answer: A in mmol/L
106. During colonoscopy of patient who has fragile thin surface of the colon with multiple blood
dots , there was no previous chronic disease, what is the most likely diagnosis?
A. Ulcerative colitis
B. Mesenteric schema

Answer:
107. Case of pain in mandibular region increase the swelling while eating and subside after it (Swelling in the horizontal line of the mandible):

smle ,2016

203
A. Ncancer
B. caliculai
Answer: B
108. Young male complaining of epigastric discomfort, he tried Over-the-counter (OTC) medication
to relieve this discomfort, he noticed some improvement but experienced constipation. Which OTC
drug most likely he tried?
A) Calcium carbonate
B) Sodium carbonate
C) Aluminum hydroxide
D) Something aluminum
Answer: A
109. Patient presented with severe vomiting on exam he looks ill with dry mucous membrane capillary refill 3 seconds and he cries with tears
what is the appropriate management?
A. severe and treat with dextrose
B. moderate and treat with ORS
C. moderate and treat with pedialyte (I chose this on as he cries with tears so its not severe but im
not sure)
D. this is a 5% dehydration
Answer:
110. 25 years teacher,, complaining of abd pain, fatigue, on exam there was
icting , palpable liver 1cm,, also 2 student complaining from same complain?
A- HAV
B- HBV
C- HCV
Answer: A
111. Pt with bilateral flank swelling, dull, move with shifting?
Answer: ascites
112. Female e jaundice, Her husband is HBV +ve , they did for her HBS antigen -ve HAV -ve HCV ve.
What to do next?!!
HBC antigen ?!!
113. HBsAg +ve
A. acute hepatitis
B. chronic hepatitis
C. acute carrier
D. chronic carrier

Answer:

smle ,2016

204
114. 25 years teacher, complaining of abd pain, fatigue, on exam there was icting, palpable liver
1cm, also 2 student complaining from same complain:
A. HAV
B. HBV
C. HCV
Answer: A
115. Type of HBV?
A. DNA
Answer: A
116. Anti obesity medication how its work ( apt -Lipes )
http://www.uptodate.com/contents/weight-loss-treatments-beyond-the-basics
117. Which antidiabetic medication cause weight gain?
Answer: Sulfonylurea
118. Treatment of gestational DM is?
Answer: insulin
119. Young obese want to lose weight, advice;
A. Yoga
B. Bicycle
C. weight lifting
Answer:
120. Diagnosis of Peptic ulcer disease:
121. Diagnosis of GERD:
122. Unwanted side effects of anticholinergic?
A) diarrhea
B) blurred vision
C) excessive salivation
D) urinary incontinence
Answer: B
123. Obese man with low calorie diet and intensive exercise, could not lose weight, he has DM &
HTN, best way to lose weight?
A. Surgery
B. Medication
C. decrease calories
D. increase intensity of exercise
INH is not mentioned
Answer: B >>metformin?

smle ,2016

205
124. Treatment of plummer vinson syndrome?
A. iron
Answer: A
125. Young boy presented with diarrhea sometimes bloody, weight loss, arthritis, anemia. the diagnosis is?
A. Crohns
B. UC
C. Celiac
Answer:
126. A 70-year-old male with history of dysuria, frequency and urgency, DRE revealed tenderness
but no masses, Temp is high, What is the most likely diagnosis?
A) Cystitis
B) Acute prostatitis
C) Rectal abscess
Answer: B
127. Preventable cause of gastroenteritis by vaccine?
Answer: Rotavirus
128. 12-years with hepatic failure admitted to ICU, his skin was yellow, now become green in color,
what is this indicates? ( not mentioned obstructive jaundice in the choices!)
A)Oxidation of bilirubin
B)Impending death
Answer: A
129. Patient treated for duodenal ulcer. Now complains of breast enlargement and
decrease sexual desire. Which drug?
Answer: cimetidine
130. Pt with history of gastric ulcer, Which of the following are used for pain control not causing gastric irritation?
A. aspirin
B. Ibuprofen
C. Coex Mb ?
D. Indomethacin
Answer:
131. Patient treated for duodenal ulcer. Now complains of breast enlargement and decrease sexual
desire. Which drug?
Answer: Ranitidine (H2 blocker)
Reference: http://www.drugs.com/sfx/ranitidine-side-effects.html
132. Lady come to the clinic to regular check up with everything is normal with table of direct and
indirect bilirubin (within the normal range) what is your diagnosis?

smle ,2016

206
a. Rotor syndrome
b. Crigler Najjar
c. Dubin Johnson
d. Gilbert syndrome
Answer:
read about these syndromes
133. Barrett's esophagus;
A. Adenocarcinoma
B. SCC
Answer: A
134. A patient with unilateral parotid swelling, hes postcholecystectomy. Saliva was cloudy (I think),
culture from parotid saliva was negative;
a. Sarcoid granulo
b. Bacterial
c. Cancer
d. Sjogren syndrome
Answer: B
135. Asymptomatic 40 y female patient direct bilirubin 5 indirect 9 what is your diagnosis?
A. gilbert disease
B. rotor syndrome
C. dubin johnson syndrome
Answer: I think A but read about these syndromes
136. Young boy eat battery of remote control what is your immediate action?
137. Young lady with abdominal pain, bloating, what treatment to give?
A- TCA
B- Antispasmodic
Answer: B
138. College student complains of generalized pain relieved by defecation. No blood or mucus. Diagnosis?
A) IBS
Answer: A
Diagnostic criteria: 3d in 3m of episodic abdominal discomfort that is two or more of 1- relieved by defecation, 2-change in stool frequency or consistency 3- change in stool appearance
139. y male 6m hx of diarrhea mixed with blood + wt loss + no family hx of inflammatory disease,
then pt had intermittent fever, in sigmoidoscopy see fragile mucosa of rectum and spot blood, Dx?
A. bacterial dysentery
B. UC
C. ischemic colitis
Answer: B

smle ,2016

207

140. Medication for resistant hiccups?


Answer:
Chlorpromazine is the most thoroughly studied and appears to be the drug of choice.
Reference: Medscape
141. Aphthous ulcer:
Answer: IBD
142. Kayser fleischer ring, What treatment?
A. Penicillamine
B. Deferoxamine
Answer: A
Treatment of Wilson disease, 4 drugs available;
1. penicillamine chelates copper, poorly tolerated
2. trientine chelates copper
3. zinc impairs copper excretion in stool/decreases copper absorption from gut
4. tetrathiomolybdate preferred if neurological involvement
screen relatives
liver transplant in severe cases
143. Patient treated for duodenal ulcer. Now complains of breast enlargement and
decrease sexual desire. Which drug?
Answer: cimetidine
144. 35 y/o female, used ampicillin 1 week ago for UTI, now presented with Hx of fever, hypotension
and tachycardia;
A. pseudomembranous colitis
B. toxic megacolon
Answer: I think it's sign of septic shock

smle ,2016

208

NEUROLOGY

smle ,2016

209

smle ,2016

210

smle ,2016

211

1.patient with Lt. Hemiplegia, MRI at T2 show hyperdense area in the Rt. Side of the brain. Which of
the following will worsening the pt prognosis?
A. blood glucose &lt 6.5
B. BP &lt 140
Answer: A
2. Patient complain of headache (worst headache in life) CSF result (blood)
A. brain abscess
B. brain infection
C. epidural hematoma
D. ruptured berry aneurysm
Answer: D
3. Which antiviral used in Parkinsonism:
A. ribavirin
B. zuvedamin
C. ganciclovir
D. amantadine
Answer: D
4. A 6 year old girl, brought by parents to ER with history of falling from height... Not talking but crying, withdrawal from pain, open her eye only in response to Doctor talking... Calculate GCS
A. 9
B. 10
C. 11
D. 12
Answer: B crying=3

5. Loss of sense of smell which lobe affected ?

smle ,2016

212
A. Frontal
B. Occipital
C. Parietal
D. Temporal
Answer: D
Temporal lobe : senses of smell and sound, as well as processing of complex stimuli like faces and scenes
Frontal lobe : conscious thought damage can result in mood changes, social
differences, etc. The frontal lobes are the most uniquely human of all the brain structures.
Parietal lobe : plays important roles in integrating sensory information from various senses, and in the
manipulation of objects portions of the parietal lobe are involved with visuospatial processing
Occipital lobe : sense of sight lesions can produce hallucinations
Limbic system : emotion, memory, emotional expression, resolve conflict from
frustration. &quotOdors often trigger emotional reactions and memories&quot (Elaine &amp Katja,
2015).
Insula&quot island inside&quot pain, taste, hunger, visceral functions, social emotions, time perception and awareness &quotconnects to the cortex and the limbic systems&quot (Blanc et. al.,2014) .

6. Epilepsy stimulated by
A. glutamate
Answer : A
7. scenario about extremities numbness and stomatitis ?
Answer: Vit B12 deficiency
8. Epileptic patient with gingival bleeding and white gain which medication:
A. phenytoin
answer A
http://reference.medscape.com/drug/dilantinphenytekphenytoin343019#
http://emedicine.medscape.com/article/1076264overview#
9. Pt came to ER with dilated pupils, diaphoresis , what's the cause?
A. Sympathomimetic
B. Organophosphate
C. Anticholinergic
Answer : A
10. SE of nitroglycerin:
A. Throbbing headache
Answer :A
11. Status epilepticus lorazepam given , still , Rx ?
A. IV phenobarbital
B. IV phenytoin
C. Oral carbamazepine
Answer B

smle ,2016

213

12. Least one cause tardive dyskinesia :


A. clonazpine,
B. risperidone,
C. haloperidol,
D. Chlorpromazine
Answer: A
13. Case of migraine what next step :
A. MRI
B. blood culture
C. more history and physical ex)
Answer: C
14. Case male with convulsion then loses his consciousness he is on antidepressants drug and drug for
congestive heart failure O/E all normal ECG shows dysrhythmia Which drug can cause this:
A. quinine
B. digitalis
C. SSRI
D. TCA
ANSWER D
15. Pt with MVA suspected increase ICP . What CN to check for?
A. optic
B. trochlear
Answer: A
16. How you test trochlear nerves :
A)Adduction and downward
B)Abduction and downward
Answer: A
17. diabetic controlled on medication developed (eye down and out) dropped upper eyelid limited
movement of eye medially and up and down
A. Right oculomotor palsy
B. right facial palsy
Answer: A
18. Pain and numbness in left thumb and index, of sensation over the dorsum 1/3 of left hand, to confirm the dx?
A. Nerve conduction studies
B. CT
C. MRI
Medscape:

smle ,2016

214
Ultrasound potentially can identify space occupying lesions in and around the median nerve, confirm
abnormalities in the median nerve (eg increased cross sectional area) that can be diagnostic of CTS, and
help guide steroid injections into the carpal tunnel.
Electrophysiologic studies, including electromyography (EMG) and nerve conductions studies (NCS), are
the first line investigations in suggested carpal tunnel syndrome (CTS)
19. hemiplegic pt come with abnormality in the eye( may be nystagmus)where is the lesion in the
brain :
A. Pons
B. medullary oblongata
C. internal capsule
D. midbrain
20. numbness and pain on thumb and index , confirmatory test :
A. compression test
B. phalen's test
C. tinels test
Answer: A
21. numbness of the lateral side of hands and fingers in computer programmer female that is confirmed by phalen test , in which position would u splint the hand:
A )Dorsiflexion
Answer: A
22. Type of cerebral palsy lower paralysis more than upper ?
A. Diplegia
B. Quadriplegia
C. hemiplegia
Answer A
23. Elderly Patient presented with chest pain palpitation and SVT with normal vital signs including O2
sat. air entry was bilateral which of the following is most helpful in establishing the diagnosis:
A)Pulmonary arteriography
B) Chest CT
C) TSH
Answer : Brain MRI
24. Neurofibromatosis gene 1 located in :
A. Chromosome 17 q 22:11
B. Chromosome 17 p 22:11
Answer: A
25. Ascending paralysis with areflexia post URT infection
A. Guillain Barre syndrome
Answer: A

smle ,2016

215

26. Young male had a history of two seizures in the last 30 minutes, No history of diabetes no history
of head trauma, at initial presentation had another seizure. What is the best to give him now?
A. phenytoin
B. diazepam
C. phenobarbital
Answer: B
27. Pt. Overdosed a medication and presented Comatose , dilated pupil, hyperreflexia: what meds:
A)Erdophnium
B) SSRI*
C) TCA
D)Something
Answer: B
28. Question about parkinson location (damage) in the brain
A. Substantia nigra
answer is Ahttp://www.pdf.org/about_pd
29. Glove neuropathy cause?
A. Vit b12
B. Vit b6
Answer: A

30. Old male Hemiplegia, CT showed ischemia, examination normal except for Hemiplegia, now he is
stable doing physiotherapy.. Taking hydrochlorothiazide...2 yrs ago had a history of acute
gastric
ulcer... What to give him:
A. Do nothing
B. tPA
C. Aspirin
D. Warfarin
Answer: C
31. Scenario and ask about the type of headache pt had stress at school:
A. Tension
32. Young female having symptoms of optic neuritis and other neuro symptoms (Case of MS) what is
the best diagnostic test (MRI)
answer :
Multiple sclerosis (MS) is diagnosed on the basis of clinical findings and
supporting evidence from ancillary tests, such as magnetic resonance imaging (MRI) of the brain and spinal cord and cerebrospinal fluid examination.
http://emedicine.medscape.com/article/1146199workup
33. patient complains of headache, he lost his wife recently, crying a lot and socially isolated, sleep is
not affected,i can't remember the duration ?

smle ,2016

216
A. Major depressive episode
B. Conversion
C. dysthymia
Answer: A
34. Def of status epilepticus ?
http://www.uptodate.com/contents/convulsive-status-epilepticus-in-adults-classification-clinicalfeatures-and-diagnosis
35. Drug that interfere with OCP ?
Antiepileptic ? Not sure
http://www.mckinley.illinois.edu/handouts/pill_interactions_drugs.html
36.weakness ... deficiency of ?
A. Vit b1
B. Vit b3
fatigue . muscle tenderness numbness are 3 symptoms of B1 THiamine Defic
37. After MVA a patient can't bring the spoon to feed himself which type of cerebral lobe is affected:
A. Temporal
B. Parietal
C. Occipital
D. cerebellum
they said which part of cerebral lobe !!!
It is a problem of coordination.
References:
http://www.healthline.com/health/movementuncoordinated#
Overview1
AlQassim MCQs > Q 342
38.Patient complain of headache band like distribution ?
A)Cluster
B)Migraine
C)Tension

answer : C
39. clear case of absence seizure then asked what is going to happen to this patient if given fentanyl :
A)glutamate receptors activation

B) seizure activity due to toxic neurotransmitters release


answer : fentanyl induced epileptiform activity on the electrocorticogram
Reference: http://www.epilepsy.com/information/professionals/diagnosistreatment/Proceduresepil
epsypatients/ generalanesthetics4
40. What is the following drug causes erectile dysfunction :
A. sartoline

smle ,2016

217
B. olanzapine
Answer: A
41. Schizophrenic, haloperidol 15 mg, presented Comatose, muscle rigidity, ruled eyes,no fever :
A. Neuroleptic malignant syndrome*
B. Tardive dyskinesia
C. Dystonia
D. Conversion
Answer :c
42. Headache when combing hair, tender vessel on temporal area, sudden loss of vision" or
clouding in one eye what next action?
A. oral prednisolone for 3 months
B. immediate cortisone eye drops
Answer: A
43. Pt with difficulty in swallowing she has frontal baldness and cataract (other symptoms that I can't
remember) Her mother has the same condition
A. Myotonic muscular dystrophy
Answer: A

44. Case of Guillain barre syndrome (hx of diarrhea 3w past ,then develop ascending symmetrical LL
paralysis... )
Answer: Guillain barre syndrome
45. ptn with increase of ICP complain of vomiting , tinnitus , nausea , headache and blurred vision ,
the doctor order for him CT scan , what the cranial nerve will discover to know the diagnosis before
doing imaging study ?
A)Optic
B)Facial
C)Trochlear
D)Oculomotor
Answer: A
46. Women traveled 18 hrs after landing She couldn't put her feet back on the shoes The best Dx test
?
A)MRI pelvis
B)CT pelvis & abdomen
C)Ct abdomen
D)Compression US
answer :???
47. case about deafness and paralysis of tympanic muscle and other muscle i can not recall it
A. trigeminal
B. facial

smle ,2016

218
Answer: the q not clear mean The tensor tympani is a muscle within the ear,Innervation of
the tensor tympani is from the tensor tympani nerve, a branch of the mandibular division of
the trigeminal nerve
https://en.wikipedia.org/wiki/Tensor_tympani_muscle
if the q mean tympanic membrane so, the nerve is facial N.
48. patient with neck pain and headache in occipital area for months ,, there is a limitation in movement ? What is Young female has severe attack of headache anxiety and palpitation she also have lost
weight and her skin looks (i forgot the word) which test will order:
A. Brain MRI
B. Urine catecholamine
C. TSH
answer :B
49.Case of urine incontinence.. pt urinate without sense or stressor dx ?
a)reflex incontinence
b)overflow
c)stress
Answer:B overflow

50. Patient has history of meningitis before 4 weeks , came againe to hospital , what is the lap result
will increase?
a) protein
b) leukocytosis
c) glucose
Answer:A

51. Elderly patient already diagnosed with Alzheimer's disease developmentpsychotic symptoms
what is drug of choice?
A)olanzepine
B)resperidone
C)Haloperidol
Answer: B
52. Neck pain radiating interscapular and numbness?
A.Vertebral collapse
B.Polymyalgia rheumatic
Answer: A
53. Blindness. Numbness that comes and goes (MS). Diagnostic test?/
A. MRI
54. older patient with memory loss and change of behavior ?
A. Alzheimer
Answer: A

smle ,2016

219
55. Young male had a history of two seizures in the last 30 minutes, No history of diabetes no history
of head trauma, at initial presentation had, another seizure. What is the best to give him now?
A. phenytoin
B. diazepam
C. phenobarbital
Answer : B
56. Which of Alzheimers drugs is hepatotoxic?
A. Tacrine
B. Donepezil
C. Rivastigmine
Answer : A
58. loss of sensation in the anterior aspect of leg and weak plantar flexion What is the nerve root?
A. L5
B. S1
Answer: B
59. Patient with drop foot, loss of sensation in 1st & 2nd metatarsal joints, what's the damaged
nerve?
A Common peroneal
B Deep Peroneal*
3 Tibial n.
4 Femoral n
Answer: B
60. 65 years old male, presenting with peripheral neuropathy which progressed to weakness (Subacute combined degeneration), labs shows Macrocytic anemia, Diagnosis?
A. Vit. B12 Deficiency
Answer: A
61. 32 years old male presenting with left sided headache associated with nausea, vomiting, photophobia, aura, lasting for 12 hours occurring 45 times per month, what's your best modality of investigation?
A)CBC
B)ESR
C)History and examination
D) MRI
history and Physical Examination* (My answer, this is a trick question,
Migraine is diagnosed clinically, MRI isn't necessary, but CT might be important to rule out
Brain lesions but not in the options)
62. Causative organism of meningitis ???
A. Neisseria meningitides

smle ,2016

220
63.patient after tooth extraction he feel numbness in his left anterior tongue what nerve is sensory
supply to this area :
A. lingual ( right answer)
B. infralobular
Answer: A
64. CSF LP showes + IgG and irregular band under the gel : MS, spinal tumor?
Elevation of IgG levels in the cerebrospinal fluid (CSF) of patients with inflammatory diseases of the central nervous system (multiple sclerosis [MS], neurosyphilis, acute inflammatory polyradiculoneuropathy, subacute sclerosing panencephalitis) is due to local central
nervous system (CNS

65. Nerve can cause problems in eye and ear ?


A. oculomotor 3
B. trochlear 4
C. facial 7
D. abducent 6
Answer: C
66. Nerve responsible for the tonsils
Answer:lesser palatine nerve from maxillary division of trigeminal
67. Nerve responsible for the sinuses
Answer : maxillary division of trigeminal
68.Man with abnormal teeth order and marginated white patch on his tonge with ulcer Dx:
A. Toxic
B. ingestion
C. Excessive
D. grow of tonge cells
NO Oral thrush , either Aphthous ulcer
69. Retinal edema between which layer ?
https://en.wikipedia.org/wiki/Macular_edema
Headache , pain in Rt eye ,Halos around light :
A. Hyphema
B. digoxin
C. toxicity
Answer: C
70. Question abort migraine meds?
A. Triptan
71. Case presented with st elevations MI for 6 hours no neurological symptoms after cpr/19 the patient died in autopsy you well find :

smle ,2016

221
A. Brain abscess
B. Intraventricular hemorrhage
C. Red cells in the hippocampal area
(Necrosis in the area associated with midcerbral infarct ( i think
72. the nerve that supply the largest part of tongue is
hypoglossal nerve supplies the anterior 2/3 of the tongue
answer: ant of tongue supply
sensory =lingual from trigeminal while taste by chora tympani from facial
73. Girl with BMI 16 and fine hair all over body
A. Anorexia Nervosa
Answer : A
74. Young female having symptoms of optic neuritis and other neuro symptoms ((Case of MS) what is
the best diagnostic test (MRI)
answer
Multiple sclerosis (MS) is diagnosed on the basis of clinical findings and supporting evidence from ancillary tests, such as magnetic resonance imaging (MRI) of the brain and spinal cord and cerebrospinal fluid
examinationhttp://emedicine.medscape.com/article/1146199workup
75. Clinical dementia?
A. Vit b12 def
76. patient diagnosed as MG Come To ER with weakness & severe fatigability she is on pyridostigmine
what initial step you do?
A. Add other drug
B. Plasmapheresis
(it is used as an adjunct therapy to pharmacological therapy in myasthenic crisis
77. temporal arteritis pt "clear scenario" this pt is in higher risk of :
A)CAD
B) Blindness,
C) brain tumor
Answer: B
78. Alzheimer disease or lowes bodies CT brain changes ?
The initial criteria for CT scan diagnosis of Alzheimer disease includes diffuse cerebral atrophy with enlargement of the cortical sulci and increased size of the ventricles. A multitude of
studies indicated that cerebral atrophy is significantly greater in patients with Alzheimer disease than in patients who are aging without Alzheimer disease. Ref: http://emedicine.medscape.com/article/336281overview#a2
79. administering pyridoxine and supportive care?
Pyridoxine used as vitamin B6 dietary supplement. The classic clinical syndrome for vitamin
B 6 deficiency is a seborrhoeic dermatitis like eruption, atrophic glossitis with ulceration , angular
cheilitis , conjunctivitis , intertrigo , and neurologic symptoms of somnolence , confusion, and

smle ,2016

222
neuropathy . The elderly and alcoholics have an increased risk of vitamin B 6 deficiency, as
well as other micronutrient deficiencies
80. patient with lower limb weakness and sensation also angular stomatitis ?
A. Vit B1
B. Vit B3
Answer : B not sure
81. early sign of hypomagnesemia :
Answer:

*Positive Chvostek sign and Trousseau sign


*Hyperactive deeptendon reflexes
*Loss of deep tendon reflex is the hypermagnesemia
Clinical manifestations are anorexia, nausea, vomiting, lethargy, weakness, personality
change, tetany (eg, positive Trousseau or Chvostek sign or spontaneous carpopedal
spasm, hyperreflexia), and tremor and muscle fasciculations.
Ref. The Merk Manual
82. Old guy with Uneven teeth.. ?
A. Neurofibromatosis
83. pt with neck pain and occipital headache , no history of trauma , there is a limitation in neck
movement on examination , weakness in upper shoulder What is the diagnosis ?
A. Acervical
B. spondylosis
C. meningitis??
Answer: C
84. Loss of memory ,no loss of conscious ,..ect Finding, Enlarge ventricle, atrophy of... What is the diagnosis:
A. Alzheimer dementia
B. Multi Infarct
Answer: A

85. case of neuroparalysis and numbness in all extremity ..


Answer:
86. case of neurosis what is investigation perform ?
A. urinalysis
B. urine Culture
Answer:
87. At serum magnesium levels less than 1mEq/L, patients develop the following symptoms:
Tremor
Hyperactive deep tendon reflexes
Hyperreactivity to sensory stimuli
Muscular fibrillations

smle ,2016

223
Positive Chvostek and Trousseau signs
Carpopedal spasms progressing to tetany
Vertical nystagmus
Mental status changes may become evident and may include irritability, disorientation,
depression, and psychosis. Cardiac arrhythmias and reversible respiratory muscle failure can also occur in severe hypomagnesemia
88. Female complaining of hair growth no seizure for 7 years wants to stop the drug?
A ) Stop the drug after 6 months (tapering)
B) Continue the drug indefinitely
C) Continue till 10 years
Answer: A
Reference: Peer review
89. Patient with exophthalmos and swollen lids and you can feel its pulse, TFT normal. What's your
diagnosis.
A. Hyperthyroid
B. Cellulitis
C. Cavernous sinus thrombosis
answer C
Ref :http://emedicine.medscape.com/article/791704clinical#
showall
It can be B according to other info if provided in the Q
90. In GuillainBarr syndrome, which cell will be affected?
Answer: ?
Schwann cells
GuillainBarr syndrome Rapidly progressive limb weakness that ascends following GI/ upper respiratory infection.

91. patient is having seizure since 35 minutes , he had diazepam I.V. but no benefit , what will you do?
Answer:

smle ,2016

224

92. Pt had head trauma , he has nausea, vomiting and decreased level of consciousness , ICP was suspected and CT was arranged , what cranial nerve examination can confirm the Dx ?
A.Optic
B.Oculomotor
C.Trochlear
Answer: A?
Looking for Papilloedema which is swelling of the optic disc
93. Fever and cough then facial nerve then loss of reflexes?
A.Tetanus
B. Botulism
Answer:B
buti think it is most going with GuillainBarr syndrome ?!
http://www.cdc.gov/nczved/divisions/dfbmd/diseases/botulism/
94. which of the following drugs causes hair growth :
A. phenytoin
B. phenobarbital
C. valproic acid
D. carbamazepine
answer: a

95. treatment of cluster headache :


A)100% oxygen
answer: A
100% oxygen by mask is the abortive therapy for cluster headache.
Reference: Master the board

smle ,2016

225

96. old pt with recent memory loss and poor self care and social withdrawal , what to give him ?
A. Neostigmine
B. Rivastigmine
Answer: b

97. patient with status epilepticus treatment:(I don't think there was intubation as one of the options)
Answer:

98. patient with seizure, what is the treatment


A. Phenobarbitone
B. Phenytoin
Answer:
Toronto:
Antiepileptic Drugs
Generalized onset and partial onset seizures:felbamate, lamotrigine, levetiracetam,refinamide,
topiramate, valproate, zonisamide
partial seizures (simple partial, complex partial, and secondarily generalized seizures): carbamazepine, gabapentin, lacosamide, oxcarbazepine, phenobarbital, phenytoin, pregabalin,
primidone, tiagabine, vigabatrin (note: these drugs may exacerbate generalized seizures)
absence seizures:ethosuximide Disorders of Consciousness

99. treatment of chronic pain ?


Answer :If they mean chronic pain syndrome:
Medscape:
Initially, pain may respond to simple over the counter analgesics, such as paracetamol, ibuprofen, aspirin, or naproxen.
Antidepressant: Amitriptyline (Elavil) and nortriptyline (Pamelor) are the tricyclic antidepressants (TCAs)
most frequently used to treat chronic pain. The selective serotonin reuptake inhibitors (SSRIs) fluoxetine
(Prozac), paroxetine (Paxil), and sertraline (Zoloft), as well as the selective serotonin/norepinephrine

smle ,2016

226
reuptake inhibitor (SNRI) duloxetine (Cymbalta), are commonly prescribed by many physicians. Other antidepressants, such as doxepin, desipramineprotriptyline, and buspirone, also can be used.
Anticonvulsant: Certain antiepileptic drugs (eg, the gamma aminobutyric
acid [GABA] analogue gabapentin and pregabalin [Lyrica]) have proven helpful in some cases of neuropathic pain.
Analgesics: Oxycodone,Fentanyl, Acetaminophen.
NSAID: ibuprofen, naproxen, diclofenac, indomethacin.
100. Old lady living alone for 5 years. She has memory problem and looks pale.
A. B12 deficiency
Answer: A
This patient is pale, this is a hint for anemia, vitB12 deficiency can cause memory problems plus anemia. Thats why if you have old patient with memory problems, before you say
he has dementia, you should exclude: Hypothyroidism and vitB12 deficiency.Reference:
Stepup to Medicine.
101. 3rd Cranial palsy ?
Answer:Out and down
102. What is the most accurate test for carpal tunnel syndrome:
A. Tinel
B. Compression test
C. Durkan's carpal test
D. Phallens test
Answer:b
B ??due to higher specificity than phalen and tinel tests in uptodate (DURKAN)
The manual carpal compression test is performed by applying pressure over the transverse
carpal ligament, and it is deemed positive if paresthesia occur within 30 seconds of applying
pressure. The average sensitivity and
specificity of the manual carpal compression test is 64 and 83 percent
http://www.uptodate.com/contents/carpaltunnelsyndromeclinicalmanifestationsanddiagnosis?source=outline_link&view=text&anchor=H2#H2

103. Medication for resistant hiccups:


Answer:?
Chlorpromazine is the most thoroughly studied and appears to be the drug of choice.
Reference: Medscape
104. Patients with epilepsy, which of the following receptors most likely is stimulated?
A. Protein G
B. glutamate
C. serotonin
D. kinase

smle ,2016

227
Answer: B, some antiepileptic drugs are glutamate receptor antagonists works specifically
on glutamate receptors, some other AEDs (such as topiramate) work on glutamate receptors and other targets.
Ref: http://www.epilepsysociety.org.uk/howantiepilepticdrugswork#.
VmyX0K2e1D8
105. whenexamin Trochlear nerve we will said to pt to move his eye
A. Medial upward
B. Medial downward
C. Lateral upward
D. Lateral downward "
Answer: B
106. Gingival hypertrophy side effect
A. Carbamazepine
B. Phenytoin
C. Phenobarbital
Answer : B
Gastrointestinal side effects including gingival hyperplasia (in as many as 50% of treated
patients) have been reported. The gingival hyperplasia associated with phenytoin is occasionally severe enough to merit surgical removal.
Reference: http://www.drugs.com/sfx/phenytoinsideeffects. html
107. multiple sclerosis Present to ER. what to give ?
Answer:
Medical management goals that are sometimes achievable in the emergency
department are to relieve symptoms and to ameliorate risk factors associated with an acute exacerbation. In patients with fulminant MS or disseminating acute encephalitis, management involves the following:
Stabilize acute life threatening Conditions Initiate supportive care and seizure precautions Monitor for
increasing intracranial pressure
Consider intravenous steroids, IV immunoglobulin (IVIG), or emergent plasmapheresis. One study suggested that plasmapheresis may be superior to IV steroids in patients with acute fulminant MS.[79] The
2011 American Academy of Neurology (AAN) plasmapheresis guideline update states that plasmapheresis is possibly effective and may be considered in acute fulminant demyelinating CNS disease.[5] Identification and control of known precipitants of MS exacerbation include the following:
Aggressively treat infections with antibiotics In patients with a fever, normalize the body temperature
with antipyretics, as even small increases in temperature can strongly affect conduction through partially
demyelinated fibers Provide urinary drainage and skin care, as appropriate. medscape.
108. Cafe au lait spots, to diagnose neurofibromatosis:
A. Presence of axillary freckling
Answer: A
Toronto: diagnosis of NF1 requires 2 or more of:
6 caf aulait spots (>5 mm if prepubertal, >1.5 cm if postpubertal)
2 neurofibromas of any type or one plexiform neurofibroma

smle ,2016

228

2 Lisch nodules (hamartomas of the iris)


optic glioma
freckling in the axillary or inguinal region
a distinctive bony lesion (e.g. sphenoid dysplasia, cortical thinning of long bones)
a first degree relative with confirmed NF1

109. A patient with myasthenia gravis presents with myasthenic crisis What is the cause?
A. anticholinesterases
B. pyridostigmine
Answer: ?
Numerous medications may exacerbate MG, including quinidine, procainamide,adrenergic antagonists, calcium channel antagonists (verapamil, nifedipine, felodipine),
magnesium, antibiotics (ampicillin, gentamicin, streptomycin, polymyxin, ciprofloxacin, erythromycin), phenytoin, gabapentin, methimazole,interferon
and contrast media.
Reference: Pubmed
110. A girl with migraine. She doesnt want to take prophylactic medicine?
A. biofeedback
B. sumatriptan
C. ergotamine
D. propranolol
Answer: A
111. Patient developed dysphagia. On examination there was deviation of the uvula to the left side.
Which nerve is affected?
A. Lt. Vagus
B. Rt. Vagus
C. Hypoglossal
D. Glossopharyngeal
Answer: B
In glossopharyngeal nerve (sensory) involvement, there will be no response when touching
the affected side. With vagal nerve damage, the soft palate will elevate and pull toward the
intact side regardless of the side of the pharynx that is touched. If both CN IX and X are
damaged on one side (not uncommon), stimulation of the normal side elicits only aunilateral
response, with deviation of the soft palate to that side no consensual response is seen.
Touching the damaged side produces no response at all. Reference: Disorders of the Nervous SystemDartmouth.
112. Loss of sensation over the maxilla and mandible:
A. trigeminal
Answer: A
113. Treatment for trigeminal neuralgia:
A)Carbamazepine

smle ,2016

229
B)Prednisolone
C)Naloxone
Answer: A
Carbamazepine and oxcarbazepine are considered first line therapy in trigeminal neuralgia
(TN). Reference: Toronto notes
http://emedicine.medscape.com/article/1145144treatment#d9
114. A patient with facial nerve involvement. Presents with loss of taste sensation of the anterior 2/3
of tongue. There is loss of function of stapedius as well. At what point is the injury?
options includes different points of facial nerve course
Answer: Facial canal between geniculate ganglion and nerve to stapedius muscle.
Reference:http://www.ncbi.nlm.nih.gov/books/NBK385/
115. Patient developed nausea and vomiting then developed cranial nerve palsies and bilateral
symmetrical progressive LL paralysis. What is the most likely diagnosis?
A. Tetanus
B. Botulism
C. Lead poisoning
Answer: B
Botulism is an acute neurologic disorder that causes potentially life threatening
neuroparalysis due to a neurotoxin produced by Clostridium botulinum. The 3 main clinical presentations: Infant botulism, Foodborne botulism and Wound botulism.
Signs and Symptoms:
occur 648h after ingestion
difficulty with convergence, ptosis, paralysis of extraocular muscles
dilated, poorly reactive pupils
other autonomic dysfunction: jaw weakness, dysarthria, dysphagia.
spreads to trunk and limbs
abdominal cramps with nausea and vomiting
symmetric weakness with paralysis and absent/decreased deep tendon reflexes
anticholinergic symptoms: dry mouth, constipation, urinary retention
rarely respiratory distress, potentially advancing to respiratory failure
Reference: Toronto Notes and Medscape.
116. Which of the following toxins has many uses?
A. Botulinum
B. Tetanus
Answer: a
117. What the scientific term of chewing ice?
A. Pagophagia
Answer: A, is a form of the disorder pica involving the compulsive consumption of ice or
iced drinks.
118. patient is seizure sence 35 mint , he take diazepam I.V but not effective , what will you do ?
Phenytoin is not in the choices

smle ,2016

230
answer:
Seizures occurring continuously for at least 30 minutes, or two or more seizures occurring without full recovery of consciousness between attacks.
Firstline
therapy = benzodiazepinesdiazepam, lorazepam, or midazolam.
Lorazepam has a relatively longer duration of seizure suppression.
Secondline
therapy = phenytoin and/or phenobarbital
Phenytoin:Rapid administration may cause hypotension and cardiac dysrhythmias this may be avoided with fosphenytoin .
Phenobarbital: Anticipate sedation, respiratory depression and hypotension.
Drug Induced coma (pentobarbital, midazolam, propofol) or general anesthesia, if resistant to above
Reference : first aid of emergency
119. What is the first structure you will hit after lumbar puncture?
A. Interspinous
B. Ligamentumflavum
C. Anterior spinal ligament
D. Posterior spinal ligament
Answer: A
Skin
Fascia and SC fat
Supraspinous ligament
Interspinous ligament
Ligamentumflavum
Epidural space and fat (epidural anesthesia needle stops here)
7Dura
Reference: USMLE step 1 + Wikipedia

120. What is the drug of choice for absence seizure?


a.Phenytoin
b.Carbamazepine
c.Ethosuximide
Answer: C
Reference: Master the Boards.

121. Old patient presented with agitation, urinary incontinence, confusion and impaired short term
memory.Long term memory is intact. CT shows temporal and hippocampal atrophy. Which of the following genes might be affected?
a.13
b.15

smle ,2016

231
c.18
d.X
Answer: 19?
Alzheimers Disease:
Early onset, (age 30 to 60):3 major genes for autosomal dominant AD have been identified amyloid precursor protein (chromosome 21), presenilin 1 (chromosome 14) and presenilin 2 (chromosome 1).
Late onset (mid60s and later):The E4 polymorphism of apolipoprotein E is a
susceptibility genotype(E2 is protective) .The APOE gene is located on the long (q) arm of
chromosome 19.
Reference:Toronto Notes
OTHER Reference :
Early onset
Alzheimer's disease occurs in people age 30 to 60 and represents less
than 5 percent of all people with Alzheimer's. Implicated chromosomes: 21, 14, and 1.
Most people with Alzheimer's have the late onset form of the disease, in which
symptoms become apparent in the mid60s and later. The causes of late onset
Alzheimer's are not yet completely understood, but they likely include a combination of genetic, environmental, and lifestyle factors that affect a person's risk for developing the disease.
Reference:
https://www.nia.nih.gov/alzheimers/publication/alzheimersdiseasegeneticsfactsheet
121. How you can confirm Brain stem death?
a.Absence of doll eye
b.Active coughing & gag reflex
c.One inactive pupil
Answer: A
The process for brain death certification includes 3 Performance of a complete neurological
examination.Components of a complete neurological examination are:
iii. Absent corneal, oculocephalic (doll's eye), cough and gag reflexes. The corneal reflex
may be altered as a result of facial weakness.
Reference:http://www.ncbi.nlm.nih.gov/pmc/articles/PMC2772257/
122. Patient came with severe pain involving the forehead to nose ( trigeminal distribution) what is
the diagnosis ?
A. trigeminal neuralgia
Answer:A
Trigeminal neuralgia is an idiopathic disorder of the fifth cranial nerve resulting in severe,
overwhelming pain in the face. Attacks of pain can be precipitated by chewing, touching the
face or pronouncing certain words in which the tongue strikes the back of the front teeth.
Patients describe the pain as feeling as if a knife is being stuck into the face. There is no
specific diagnostic test. Treat with oxcarbazepine or carbamazepine.
Baclofen and lamotrigine have also been effective. If medications do not control the pain,
gamma knife surgery or surgical decompression can be curative.
Reference : Master The Board

smle ,2016

232
123. which of this drug cause seizure ?
A. isoniazid
B. ethambutol
C. ripaficin
D. pyrazinamide
Answer: A
http://reference.medscape.com/drug/isoniazid342564#4
124. Q about. Numbness. What drug?
A. Isoniazid
Answer :A
125. reversible cause of stroke?
A- Hypertension
B- Obesity
C- Smoking
Answer: A
126. patient with pinpoint pupil antidote ?
A. Naloxone.
Answer:A
127. patient with dilated pupil, tachycardia , what is the cause ?
A. Sympathemmtic
Answer A
128. most common tumor intracranial in adult ?
A. Hemangioblastoma
B. Ependymoma
C. Schwannoma
Answer: B
Gliomas are the most prevalent type of adult brain tumor, accounting for 78 percent of malignant brain tumors.
Meningiomas are the most common benign intracranial tumors
Pituitary adenomas are the most common intracranial tumors after gliomas, meningiomas
and schwannomas.
So: the most common brain tumors are (in sequence)
1.
Glioma
2.
Meningioma
3.
Schwannoma
4.
Pituitary adenoma
Gliomas include: Astrocytomas, Ependymomas, Glioblastomamultiforme, Medulloblastomas,
and Oligodendrogliomas.
In children: Medulloblastoma
American Association of Neurological Surgeons http://www.aans.org/patient%20information/conditions%20and%20treatments/brain%20tumors.aspx

smle ,2016

233

129. Q-Mainstay treatment for parkinson's disease?


1. Dopamin agonists
2. MAO inhibitors
3. Anticholinergics
4. Levodopa*
answer : 4
-1st aid step 2

NEPHROLOGY
1- pt with depigmentation and problems in her eyes (I can't recall them), what can be associated with
it?
A-renal
Answer:
2- why is inulin used to measure GFR?
Answer: Active biological, Passive free in glumuras
Recall that all of the plasma that is filtered and only the plasma filtered is cleared of inulin so that if one
were to measure the clearance of inulin, it would equal the amount of plasma filtered in a minute, the
glomerular filtration rate. Therefore, the clearance of inulin is equal to the glomerular filtration rate, the
volume of plasma filtered in one minute.
Reference: http://www.austincc.edu/emeyerth/clearancehtm.htm
3-case of UTI methicillin sensitive?
A-Cloxacillin
B-indole positive
C-E coli
Answer:
4- DM with high BP and deteriorating renal what to give?
A- ACEI (lisinopril)
Answer: A
Scientists have made great progress in developing methods that slow the onset and progression of kidney disease in people with diabetes. Drugs used to lower blood pressure can slow the progression of kidney disease significantly. Two types of drugs, angiotensin-converting enzyme (ACE) inhibitors and angiotensin receptor blockers (ARBs), have proven effective in slowing the progression of kidney disease.
Many people require two or more drugs to control their blood pressure. In addition to an ACE inhibitor
or an ARB, a diuretic can also be useful. Beta blockers, calcium channel blockers, and other blood pressure drugs may also be needed.

smle ,2016

234
An example of an effective ACE inhibitor is lisinopril (Prinivil, Zestril), which doctors commonly prescribe
for treating kidney disease of diabetes. The benefits of lisinopril extend beyond its ability to lower blood
pressure: it may directly protect the kidneys' glomeruli. ACE inhibitors have lowered proteinuria and
slowed deterioration even in people with diabetes who did not have high blood pressure.
An example of an effective ARB is losartan (Cozaar), which has also been shown to protect kidney function and lower the risk of cardiovascular events.
Reference: http://www.medicinenet.com/script/main/mobileart.asp?articlekey=101334&page=6
5- case of pyelonephritis, what is the next step?
A- Admit and give antibiotics
B-Do investigations
C- Give him antibiotics at home
#Answer: scenario not clear ??
Dysuria with:
Flank or costovertebral angle tenderness
High fever
Occasionally with abdominal pain from an inflamed kidney
Urinalysis shows increased WBCs. Imaging studies (CT or sonography) are done to determine if there is
an anatomic abnormality causing the infection.
Treat with:
Ceftriaxone, ertapenem
Ampicillin and gentamicin until culture results are known
Ciprofloxacin (oral for outpatient)
Admit if severe or non-resolving or renal obstruction
Ref erence: Master the board & Toronto
#Answer: According to the severity
- Mild cases :OPD treatment Fluroqunolone: Ciprofloxacin is the 1st line
- Sever Cases : Admission + IV Antibiotics Fluroqunolone + 3rd or 4th generation Cephalosporins: Cefttriaxone, Cefepime or Carbapenem" can be used according to the case
Reference: USMLE STEP 2 CK FA 9th edition P221
6- Pt on diuretic develop palpation, which electrolyte lead to this:
A- Na
B-K
C - cl
D - ca
Answer: B
Both HyperK and HypoK
7- You give pt ACI for treating hypertension , what you'll add?
Answer: we recommend therapy with a long-acting ACE inhibitor or ARB in concert with a long-acting
dihydropyridine calcium channel blocker. Combination of an ACE inhibitor or ARB with a thiazide diuretic
can also be used but may be less beneficial. ACE inhibitors and ARBs should not be used together.
Reference: uptodate
http://www.pharmacology2000.com/Cardio/antihyper/antihype.htm
8-patient in ICU ,ESRD develops fever, culture shows yeast on blood What is
The ttt?

smle ,2016

235
Answer:
It is candidemia
Reference: http://www.uptodate.com/contents/treatment-of-candidemia-and-invasive-candidiasis-inadults?source=outline_link&view=text&anchor=H2629268750#H2629268750
9- pt presented with Hypotension, his phosphatase in normal level, after one day hir phosphate level
decrease, what organ damage?
A-liver
B-kidney
C-lung
Answer: I think B
10-patient had throat infection 2 weeks ago was developed hematuria, how to treat?
A-Corticosteroids
B-Thiazide
Answer:
the diagnosis is post strep glomerulonephritis
During the acute phase of the disease, restrict salt and water. If significant edema or hypertension develops, administer diuretics.
Loop diuretics increase urinary output and consequently improve cardiovascular congestion and hypertension.
For hypertension not controlled by diuretics, usually calcium channel blockers or angiotensin-converting
enzyme inhibitors are useful.
For malignant hypertension, intravenous nitroprusside or other parenteral
11-patient presented with severe vomiting. his labs showed hypocalcemia. what is your management?
A-furosemide
B-hydration
Answer :
if hypocalcemia: Ca& Mg Supplements + Treat the underlying cause
if hypercalcemia: IV Hydration
Reference: USMLE STEP 2 CK FA 9th edition P472&473
12-patient with bilateral abdominal mass:
A-Polycystic kidney disease.
Answer: A
Reference: http://emedicine.medscape.com/article/244907-overview
13-Old pt complaining of frequent urination, but difficulty initiating urine. Palpation shows distended
bladder. Type?
A- overflow
B- urge
C- stress
Answer: A
14-HTN, fluid overload, azotemia
A-DM

smle ,2016

236
B-nephropathy
C-bilateral renal artery stenosis
Answer: B
15-Renal failure with azotemia
Answer: Bilateral renal artery stenosis
16- pt with rt kidney 14 Cm and left kidney 7 cm.. Arteriography: renal artery
stenosis, what to do?
A. CT angio
B. Ct abdomen
C. biopsy
Answer: A
17-Non opaque radiography?
Answer: Uric acid
18-To decrease UTI recurrence?
A decrease urea, decrease PH, increase osmolarity
B - increase urea, decrease PH, increase osmolarity
Answer : B
Biochemical properties are normally important in making bacterial survival difficult in urine: acid pH,
high urea content, and high osmolality.
Reference: https://www.auanet.org/education/adult-uti.cfm
19-Pt c/o severe uncontrolled HTN, Renography not sure, showed Lt renal artery stenosis, next
step is to?
A-Venography
B-IVP
C-CT angiography
D-Renography
Answer: D Renal Ultrasonography
20-pt with long hx of uncontrolled HTN , he presented to you with headache and 160/90 BP , what you
will see in his kidneys?
A) Decrease sclerosis
B) Increase hyalinization of arterioles
Answer: B
intimal thickening and luminal narrowing of the large and small renal arteries and the glomerular arterioles, and Glomerulosclerosis: both focal global (involving the entire glomerulus) and focal segmental
sclerosis
Reference: http://www.uptodate.com/contents/clinical-features-diagnosis-and-treatment-of-hypertensive-nephrosclerosis?source=preview&search=%25252Fcontents%25252Fsearch&anchor=H11268824#H2
21-Old pt with Back pain, dysuria , frequency. what is the next investigation?
A- PSA

smle ,2016

237
B- ALP
Answer: A
22-boy with hematuria and SNHL, his father has end stage renal disease and SNHL, wt DIAGNOSIS?
(sensorineural hearing loss)
A-alport syndrome
Answer: A
23-Patient presented with hematuria and he diagnosed to have post streptococcal glomerulonephritis, what is the treatment?
A-Antibiotics
B-Cortisone
Uptodate: Management is supportive and is focused upon treating the volume overload that causes the
clinical complications of PSGN. These general measures include sodium and water restriction, and diuretic therapy.
Answer: A according to these choices
Reference: Master TheBoared 3rd edition P353
24-The most common cause of renal failure is?
A-diabetes mellitus
B-hypertension
Answer: A
25-The most common cause of secondary hypertension is?
A-kidney disease
B-pheochromocytoma
C-conn
D-Cushing
Answer: A
26-case of fanconi syndrome?
Answer:
a type of RTA a disease of the proximal renal tubules in which glucose, AA, uric acid, P and bicarbonate
are passed into the urine, instead of being reabsorbed. It may be inherited, or caused by drugs or heavy
metals.
clinical features of proximal renal tubular acidosis are:
Polyuria, polydipsia and dehydration
Hypophosphatemic rickets (in children) and osteomalacia (in adults)
Growth failure
Acidosis
Hypokalemia
Hyperchloremia
Other features of the generalized proximal tubular dysfunction of the Fanconi syndrome are:
Hypophosphatemia/phosphaturia
Glycosuria
Proteinuria/aminoaciduria

smle ,2016

238
Hyperuricosuria
- treatment :
mainly consists of the replacement of substances lost in the urine. mainly fluids and electrolytes.
Dehydration due to polyuria must be prevented by allowing free access to water; treat dehydration with either oral or parenteral solutions.
Metabolic acidosis due to the loss of bicarbonate is corrected by 3-10 mg/kg/d of sodium bicarbonate in divided doses.
Diuretic, as 1-3 mg/kg/d of hydrochlorothiazide, may be necessary to avoid volume expansion,
need to augment potassium supplementation in the form of potassium bicarbonate, citrate, or
acetate.
Phosphate and vitamin D supplementation are necessary, 1-3 g/d of supplemental phosphate.
Reference: https://en.wikipedia.org/wiki/Fanconi_syndrome
http://emedicine.medscape.com/article/981774-treatment#d6
27-child with decreased UOP, tea colored urine, generalized swelling. next investigation?
Answer: Urine Analysis
Reference: Master The Boared 3rd edition P353
28- Patient with DM, how to prevent progressive renal disease?
Answer: ACEI
Reference: Toronto NP29
29-HTN pt with decreased GFR;
A- Bilateral renal artery stenosis
B- DM nephropathy
Reference: A
30-Prostatic ca marker;
A-a feto
B- alk
C-...
*No PSA
Answer :
only the use of PSA and its isoforms can be recommended in prostate cancer. Below we present a more
detailed discussion of the use of these measurements.
Biomarkers Currently Being Explored for Prostate Cancer table 8
(Human kallikrein 2 (hK2), Insulin-like growth factor (IGF-1), insulin-like growth factor binding protein
(IGFBP-3),Molecular urine markers PCA3, Alpha-methylacyl-CoA,racemase (AMACR),(GSTPi), Methylation pane, Telomerase activity)
Reference: from Laboratory Medicine Practice Guidelines, Use of Tumor Markers in Testicular, Prostate,
Colorectal, Breast, and Ovarian Cancers book
31-Polycystic (renal) disease;
A. AR
B. x-linked
Answer: the term polycystic kidney disease is reserved for one of two hereditary conditions:

smle ,2016

239
Autosomal dominant polycyctic kidney disease (ADPKD), common type.
Autosomal recessive polycystic kidney disease (ARPKD), previously called infantile polycystic kidney disease.
32-Best investigation to measure GFR?
A. Inuline
B. 24 urine creatinine collection
C. other options were irrelevant
Answer: serumecreatinine
Reference: uptodate
33-NEPHRITIC SYNDROME?
A-HTN
B- Hypobilirubinemia
C- Edema
D- hyperlipidemia
Answer: A
HTN is common in nephritic while edema is common in nephrotic
Reference: FA, step 1
Hypertension is present in 50 to 90 percent of patients and varies from mild to severe
Reference: uptodate
34 -pt with bilateral flank pain for 6 months and there is gene 16 mutation , what is the disease?
A. Adult polycystic kidney disease
B. pcos
Answer: A
35-Pt presented with right flank pain, no dysuria or penile discharge?
Answer: renal colic
36-Pt c/o of loin pain & hematuria and inherited autosomal dominant what the diagnosis?
A. polycystic kidney disease
Answer: A
Reference: http://www.uptodate.com/contents/polycystic-kidney-disease-beyond-the-basics?source=outline_link&view=text&anchor=H1#H1
37-ABG case of pt presented with vomiting ?
A. Metabolic alkalosis
Answer: A
38-Patient 3 weeks after URTI develop rash, knee pain, and hematuria. What's the Dx?
A- Henoch-Schnlein purpura ( specific type of hypersensitivity vasculitis)
Answer: A
Henoch-Schnlein purpura IgA vasculitis characterized by a tetrad of clinical manifestations:
Palpable purpura in patients with neither thrombocytopenia nor coagulopathy 95-100%
Arthritis/arthralgia especially involving the knees and ankles 60-84%
Abdominal pain usually diffuse, with acute-onset 35-85%
Renal disease proteinuria, hematuria

smle ,2016

240
In one half to two thirds of children, an upper respiratory tract infection (URTI) precedes the clinical onset of HSP by 1-3 weeks
Reference: http://www.uptodate.com/contents/henoch-schonlein-purpura-immunoglobulin-a-vasculitis-clinical-manifestations-and-diagnosis
39-IgA nephropathy;
Reference: https://en.wikipedia.org/wiki/IgA_nephropathy
40-Adult presented with edema and proteinuria. low albumin, Normal urea and creatinine?
A-nephrotic syndrome
B-interstitial nephritis
C-nephritic syndrome
Answer: A
Serum creatinine will be in the normal range in uncomplicated nephrotic syndrome, such as that occurring in minimal-change nephropathy. In children, the serum creatinine level will be lower than it is in
adults. The normal adult serum creatinine level is approximately 1 mg/dL, whereas that of a child aged 5
years will be about 0.5 mg/dL. Values higher than this in children indicate reduced kidney function.
the serum albumin level is classically low in nephrotic syndrome
Reference :medscape
41-First symptoms of hypomagnesaemia?
A-muscle paralysis?
B-Hypotension
C-loss of deep muscle reflex
D-respiratory depression
Answer: D
Neuromuscular manifestations, including neuromuscular hyperexcitability (eg, tremor, tetany, convulsions), weakness, apathy, delirium, and coma
Weakness of the respiratory muscles is a major concern in critically ill patients with hypomagnesemia,
and may be a variable in the genesis of respiratory failure
Cardiovascular manifestations, including widening of the QRS and peaking of T waves with moderate
magnesium depletion, and widening of the PR interval, diminution of T waves, and atrial and ventricular
arrhythmias with severe depletion.
Abnormalities of calcium metabolism, including hypocalcemia, hypoparathyroidism, parathyroid
hormone (PTH) resistance, and decreased synthesis of calcitriol.
Hypokalemia.
Magnesium depletion is also associated with several other disorders, such as insulin resistance and the
metabolic syndrome and hypertension
Reference: uptodate
42-High K+ , wide QRS:
A-give ca gluconate
Answer: A
43-55 y old female has HTN, fluid overload and azotemia. what is the diagnosis?
A-Tubular acidosis (was one of the choices) ??

smle ,2016

241
Answer:
44- Pt drink some material made from ethanol use for freezing what the complication?
A-Rapid progressive glomerulonephritis
B-pyelonephritis
C-ATN
Answer: C
I think they meant antifreeze (ethylene glycol)
First it has nurological SX
Then cardiopulmonary SX
Then Renal SX (ATN)
45-elderly with acute urine retention:
a.transurethral prostatectomy
b.partial prostatectomy
c.foley catheter and urine culture
Answer: C
In men, AUR is most often secondary to benign prostatic hyperplasia (BPH) . In men with BPH, risk factors for developing AUR include advanced age,
Other causes of outflow obstruction in men include constipation, cancer (prostate or bladder), urethral
stricture, urolithiasis, phimosis, or paraphimosis, Most patients with suspected acute urinary retention
(AUR) will have a bladder ultrasound that will confirm the diagnosis. However, in patients whose history
and physical examination strongly suggest a diagnosis of AUR, it is reasonable to proceed with catheterization without a bladder ultrasound, which is both diagnostic and therapeutic.
ACUTE MANAGEMENT: The initial management of acute urinary retention (AUR) is prompt bladder decompression by catheterization.
Reference: uptodate
46-Patient presented with non traumatic acute urinary retention. How will you treat him/her?
1. Foley catheter
Answer: A
Reference: UpToDate
47-pt use diuretics and he developed muscle weakness and diarrhea what is the cause,?
A-hyper k
B-hypo k
C-hyper Na
In most patients with hypokalemia, the cause is apparent from the history (eg, vomiting, diarrhea, diuretic therapy).
Reference :uptodate
48-12 -years with hepatic failure admitted to ICU, his skin was yellow .now become green in color ,
what is this indicates :( not mentioned obstructive jaundice in the choices!)
A-Oxidation Of bilirubin.
B-Impending death .
Answer: A

smle ,2016

242
49-patient diagnosed as MG, Come To ER with weakness & severe fatigability, she is on pyridostigmine, what initial step you do?
EVALUATION AND MANAGEMENT There are a number of general concepts common to the treatment
of patients with myasthenic crisis or severe exacerbations of myasthenia gravis. These include the following:
Admit to intensive care unit
Frequently monitor respiratory muscle strength
Electively intubate if clinical evaluation or tests of respiratory muscle strength suggest impending
respiratory failure; temporarily stop anticholinesterase medications for intubated patients
Begin rapid therapy with plasma exchange or intravenous immune globulin (IVIG)
Begin immunomodulating therapy with high-dose glucocorticoids; consider azathioprine, mycophenolate mofetil, or cyclosporine if glucocorticoids are contraindicated or previously ineffective
Initiate weaning from mechanical ventilation when respiratory muscle strength is improving, but
only after starting treatment with plasma exchange or IVIG
Reference: uptodate
Add other drug
Plasmapheresis it is used as an adjunct therapy to pharmacological therapy in myasthenic crisis
50-Which cells forming the filtration layer in the kidney?
A-mesengil
B-podocytes
C-partial
Answer: B
The healthy kidney filters metabolic byproducts into the urine but prevents the passage of albumin and
other larger essential molecules. This selective filtration occurs across the glomerular capillary wall:
The glomerular capillary wall, through which the filtrate must pass, consists of the following three layers:
Fenestrated capillary endothelium, extensively coated with a layer of poly anionic glycosoaminoglycans and glycoproteins.
Glomerular basement membrane (GBM), containing heparin sulfate and other anionic glycosoaminoglycans.
Podocytes (or epithelial cells), which are attached to the GBM by discrete foot processes. The
pores between the foot processes (slit pores) are closed by a thin membrane called the slit diaphragm, which functions as a modified adherens junction and may also be permeated by anatomical pores.
51-Patient on diuretics with diarrhoea. What type of electrolyte imbalance?
A-Hypokalaemia.
B-Hyperkalaemia.
Answer : A
52-Hx and investigation All normal except Hyponatremia and crepitation on Ex, what would you give?
A.IV NS
B.furosemide
Answer: B

smle ,2016

243

53-Case with: ph 7.2 \ pCo2 : decreased below normal range \ bicarbonate decreased below normal
range. Dx ?
A-Compensated Metabolic acidosis
B-Uncompensated Metabolic acidosis
C-Compensated respiratory acidosis
D-Uncompensated Metabolic acidosis
Answer: B
54-According Henoch schonlein purpura , which of the following carries bad prognosis?
A-Renal failure
B-Hepatic failure
Answer: A
anion gap if corrected Na 138 >> choose the closest number to 20
55-Patient with vomiting and diarrhoea. What type of electrolyte imbalance?
A- Hypernatremia.
B- Hyperglycemia.
C- Hyperkalemia.
Answer: A
Usually due to water loss in excess of sodium loss, eg: diarrhoea , vomiting , burn.
Reference: Oxford clinical medicine
http://www.uptodate.com/contents/image?imageKey=NEPH%2F69879&topicKey=NEPH%2F2376&source=see_link&utdPopup=true
56- Patient presented with color soda urine since one week, during examination congestion of throat
with cervical lymphadenopathy with fever, What's the cause?
A-Acute glomerulitis
B-Iga nephropathy
C-Acute cystitis
Answer: B
Reference: uptodate
Approximately 40 to 50 percent present with one or recurrent episodes of visible hematuria, usually following an upper respiratory infection. This has sometimes been called "synpharyngitic hematuria."
These episodes can be provoked by bacterial tonsillitis, or by other viral upper respiratory infections
57- Young female with malar rash, photophobia, joint pain, hematuria?
A-lupus nephritis
B-HSP
Answer: A
Reference: http://www.uptodate.com/contents/overview-of-the-clinical-manifestations-of-systemiclupus-erythematosus-in-adults?source=machineLearning&search=sle&selectedTitle=1~150&sectionRank=1&anchor=H91779658#H91779658
58-Old male pt presented with urgency and frequency symptoms, what's the first thing that u will do?
A-PSA
B-trans rectal US ....
Answer: PSA

smle ,2016

244
Benign prostatic hyperplasia (BPH) is a common disorder that increases in frequency progressively with
age in men older than 50 years .
The clinical manifestations of BPH are lower urinary tract symptoms (LUTS) that include storage symptoms (increased daytime frequency, nocturia, urgency, and urinary incontinence), voiding symptoms (a
slow urinary stream, splitting or spraying of the urinary stream, intermittent urinary stream, hesitancy,
straining to void, and terminal dribbling), and irritative symptoms (frequent urination, urinary urgency).
serum prostate specific antigen Prostate cancer can cause obstructive symptoms, although the presence of symptoms is not predictive of prostate cancer [26]. Measurements of serum PSA may be used as
a screening test for prostate cancer in these men with BPH, preferably in men between the ages of 50 to
69 years and before therapy for BPH is discussed.
59-Patient with HTN and he has hypercalcemia, RX?
A. Furosemide
Answer:
60-wheal with erythematous base itching lymph node enlargement periorbital swelling hepatosplenomegaly?
A-Rheumatic arthritis
B-Angioedema
C-Cholinergic urticarial
D-itching more with urticarial
Answer: A, extraarticular manifestation of RA
61-about chronic granulomatous disease?
62-female patient swallow 1 liter of car_anti_freeze water (ethylene glycol ), what is going to happen?
A-interstitial nephritis
B-End stage renal disease
Patients with large ingestions of methanol or ethylene glycol may present with mild CNS effects, such as
inebriation and sedation, similar to ethanol intoxication. Ethanol coingestion may exacerbate such
symptoms. Coma, seizures, hyperpnea (Kussmaul-Kien respirations), and hypotension all suggest a substantial portion of the parent alcohol has been metabolized to its toxic byproducts.
An afferent pupillary defect is an ominous sign of advanced methanol poisoning. Eye examination in
methanol poisoning may also reveal mydriasis, a retinal sheen due to retinal edema, and hyperemia of
the optic disk.
It is important to emphasize that the onset of methanol or ethylene glycol toxicity is delayed when ethanol is coingested. The possibility of concomitant ethanol and toxic alcohol ingestion must always be considered, particularly in alcoholics, who may ingest alcohol in any form.
Ethylene glycol metabolism can lead to cranial nerve palsies and tetany (thought to result from oxalateinduced hypocalcemia). Oliguria and hematuria may occur. Cerebral herniation and multisystem organ
failure are common preterminal events in patients with profound poisoning.
Answer could be any organ failure (brain , lung , liver or kidney)
63-Urge incontinuance principle management?
A-Medical

smle ,2016

245
B-Surgical
C-Medical and surgical
D-Bladder training and physio
Answer: C , medical & surgical
64-A patient with renal function test abnormalities. Tests show beads on string appearance. What is
the diagnosis?
A. renal artery disease
B. fibromuscular dysplasia
Answer: B
The string-of-beads appearance is considered pathognomonic for medial fibroplasia on diagnostic angiography
Reference: http://emedicine.medscape.com/article/417771-overview#a
65-Pt had pyelography showing rt kidney 7 cm and left kidney 16 or 14 cm, arteriography shows rt renal stenosis, Next management?
A- arteriography
B- Lt kidney percutaneous biopsy
C- CT angiography
D- CT abdomen
Answer : C (not sure)
66-Urine incontinence, bladder palpable during examination, what type of urine does the patient has?
A- Stress.
B- Overflow.
C- Reflux.
D- Urgency.
Answer: B
67-case of urine incontinence..pt urinate without sense or stress,, dx?
A-reflex incontinence
B- overflow
C-stress
Answer: B

68-A patient with dyspnea, pallor, edema of legs and itching. What
is the diagnosis?
A. Scabies
B. renal failure
Answer: B
69- Breast cancer patient who receives many medications (cyclophosphamide, fluorouracil, ..etc). In
order to avoid the adverse effect of having hemorrhagic cystitis which of the following will be given to
this patient?

smle ,2016

246
A-Aldesleukin
B-Mesna
Answer: B
Mesna: Protects the bladder from damage that may be caused by some cancer medicines.

70-scenario with urine analysis only which shows:


PH of urine normal, chloride high, bicarbonate low
and other mentioned in labs but within normal limit, what's dx?
A-metabolic acidosis
B-metabolic alkalosis
C-rep. Alkalosis
D-rep. acidosis
Answer:
71-Normal pH and high Bicarbonate?
Respiratory acidosis with compensatory metabolic Alkalosis

72-Most sign renal cancer;


A-Cachexia
B-Hematuria
C-Abdomenal mass
Answer: B
The classic triad of RCC (flank pain, hematuria, and a palpable abdominal renal mass) occurs in at most 9
percent of patients; when present, it strongly suggests locally advanced disease.
Hematuria is observed only with tumor invasion of the collecting system. In an early series, hematuria
was observed in almost 40 percent of patients.
Reference: http://www.uptodate.com/contents/clinical-manifestations-evaluation-and-staging-of-renalcell-carcinoma
73-NF1 gene response for ?
A - Neurofibromatosis type 1 (NF1)
Answer: A
NF1 is due to mutations in the NF1 gene, located at chromosome 17
Reference: uptodate
74-Smoking increases risk of?
A- BLADDER CANCER
Answer: A
Smoking is the most important risk factor for bladder cancer. Smokers are at least 3 times as likely to get
bladder cancer as nonsmokers. Smoking causes about half of the bladder cancers in both men and
women
Reference: http://www.cancer.org/cancer/bladdercancer/detailedguide/bladder-cancer-risk-factors

smle ,2016

247
75-A patient presented with hemoptysis and signs of nephropathy. Biopsy of the lung showed presence of anti-GBM antibodies. What is the most likely diagnosis?
A-Goodpasture syndrome
Answer: A
Frank hemoptysis suggests Goodpasture syndrome (glomerulonephritis and pulmonary hemorrhage associated with anti GBM antibody) but this also can be a prominent feature of systemic vasculitis
Reference: Diseases of the Kidney and Urinary Tract textbook edited by Robert W. Schrier
NB. Wegners was not included in the choices
Reference: http://emedicine.medscape.com/article/981258-overview
76-DKA case with dehydration sign..Hx of wt. loss last 2 week, In ER given IV fluid and slow infusion
insulin.. What is more to concern? (ER)
A. Cerebral edema
B. Renal Failure
C. Hypoglycemia
Answer: A
77-What is the cause of HTN?
A-Essential
B-NSAID induced
C-Primary hyperaldosteronism
Primary (essential) hypertension
For most adults, there's no identifiable cause of high blood pressure. This type of high blood pressure,
called primary (essential) hypertension, tends to develop gradually over many years.
Secondary hypertension
Some people have high blood pressure caused by an underlying condition. This type of high blood pressure, called secondary hypertension, tends to appear suddenly and cause higher blood pressure than
does primary hypertension. Various conditions and medications can lead to secondary hypertension, including:

Obstructive sleep apnea


Kidney problems
Adrenal gland tumors
Thyroid problems
Certain defects in blood vessels you're born with (congenital)
Certain medications, such as birth control pills, cold remedies, decongestants, over-thecounter pain relievers and some prescription drugs
Illegal drugs, such as cocaine and amphetamines
Alcohol abuse or chronic alcohol use

78- ph : 7.2 , HCO3 : 25 , co2 : 60


A. Metabolic acidosis
Answer:

smle ,2016

248
79-Effect of HTN on kidney?
A-Decrease glumer
B-Hydraulic of vessels
Answer: B
80-patient with history urethral discharge culture (negative )and severe unilateral knee pain what is
the diagnosis ?
Answer: Reiter's syndrome
81-filling defect seen with acoustic shadow in the renal US?
A. Clot
B. Tumor
C. Uric acid
D. Papillary necrosis
Answer:
If hypo >>uric
If hyper >> papillary
82-organ responsible for multiorgan failure? Unclear question!
A. heart
B. lung
C. kidney
D. liver
Answer:
83-Na: 135 Cl: 100 HCO3: 12, AG?
A. 23
B. 10
Answer: A
AG = Na - (Cl + HCO3)
Range of normal AG: 10 2
Reference: http://emedicine.medscape.com/article/2087291-overview
84-Calculate anion gap with corrected NA 138?
Answer: anion gap = Na - ( Cl + HCO3 )
anion gap should not be corrected to calculate anion gap it will lead to false elevated calculation
85 -Case with acid base imbalance (Low ph, bicarb, co2) , including values Dx?
A. Compensated metabolic acidosis
Answer:
86-Post infection the abx was formed (low c3)
87-Tx of SLE pt with URTI
Reference: http://www.clevelandclinicmeded.com/medicalpubs/diseasemanagement/nephrology/hypokalemia-and-hyperkalemia/
88- Minimal change nephrotic syndrome, histopathological finding?
Answer:

smle ,2016

249
histopathology of minimal change nephrotic syndrome
The glomeruli appear normal on light microscopy in patients with and there are no complement or immunoglobulin deposits on immunofluorescence microscopy. Glomerular size is usually normal by standard methods of light microscopy, although enlarged glomeruli may be observed
The characteristic histologic lesion in MCD is diffuse effacement (also called "fusion") of the epithelial
foot processes on electron microscopy. More specifically, there is retraction, widening, and shortening
of the foot processes. The spaces between flattened podocyte foot processes are reduced in number
and support the burden of plasma filtration, which may play a role in the excess albumin load into the
urinary space. The degree of effacement does not correlate with the degree of proteinuria. Foot processes regain a normal appearance with remission of proteinuria
89-what is the effect of high blood pressure on the kidney?

90- early sign of hypomagnesemia?


A- Loss of deep tendon reflex
Answer:
Clinical manifestations are anorexia, nausea, vomiting, lethargy, weakness, personality change, tetany
(eg, positive Trousseau or Chvostek sign or spontaneous carpopedal spasm, hyperreflexia), and tremor
and muscle fasciculations
Reference: The Merk Manual
At serum magnesium levels less than 1 mEq/L, patients develop the following symptoms:
Tremor
Hyperactive deep-tendon reflexes
Hyperreactivity to sensory stimuli
Muscular fibrillations
Positive Chvostek and Trousseau signs
Carpopedal spasms progressing to tetany
Vertical nystagmus
Mental status changes may become evident and may include irritability, disorientation, depression, and psychosis. Cardiac arrhythmias and reversible respiratory muscle failure can also occur
in severe hypomagnesemia
91-long scenario with urine analysis only which shows: (PH of urine normal, chloride high, bicarbonate
low)"and other mentioned in labs but within normal limit", what's dx?
a)metabolic acidosis
b)metabolic alkalosis
c)rep. alkalosis
d)rep. acidosis
Answer:
Normal pH and high Bicarbonate >> Respiratory acidosis with compensatory metabolic Alkalosis

smle ,2016

250

HEMATOLOGY
1-In hemolytic anemia which enzyme will be noticed?
A. erythropoietin
B. bilirubin
Answer: B
Standard blood studies for the workup of suspected hemolytic anemia include the following:
Complete blood cell count
Peripheral blood smear
Serum lactate dehydrogenase (LDH) study
Serum haptoglobin
Indirect bilirubin
Reference :http://emedicine.medscape.com/article/201066-workup
2-In factor VII deficiency what is the Lab abnormality we will detect?
a- increased pt
b- increased ptt
Answer: A
Explanation: prothrombin time (PT) to measure the functioning of factors I, II, V, VII, and X
partial prothrombin time (PTT) to measure the functioning of factors VIII, IX, XI, XII, and von Willebrand
factors
The prothrombin time (PT) is prolonged in factor VII (FVII) deficiency and the international normalized
ratio (INR) is elevated. The activated partial thromboplastin time (aPTT) is within the reference range in
isolated factor VII deficiency. ( http://emedicine.medscape.com/article/960592-workup )
A normal aPTT and a prolonged PT in a patient with a lifelong history of a tendency
for mild or severe bleeding is consistent with the diagnosis of factor
VII deficiency or the presence of an inhibitor to factor VII.
-Bleeding time is usually within the reference range .
-With no significant clinical bleeding but a prolonged PT and a normal aPTT, the patient has either mild
factor VII deficiency or is taking oral anticoagulants.
http://emedicine.medscape.com/article/209585-workup
3-best diagnosis tool for thalassemia is :
Answer : Haemoglobin electrophoresis is the gold standard
Source : uptodate
4-Patient with osteoarthritis, you found that he developed anemia which type it would be:
a- normochromic normocytic
b- microcytic hypochromic ( if with NSAiD)
c- macrocytic hyperchromic
Answer:A
5-What factor cause thrombosis ?

smle ,2016

251
answer :
Risk factors (causes) for the development of venous thrombosis :
Inherited : Factor V Leiden mutation, Prothrombin gene mutation
Protein S & C deficiency ,Antithrombin (AT) deficiency
Acquired : Malignancy, Surgery, Trauma,Pregnancy ,OCP
- Hormone replacement therapy, Polycythemia vera, IBD
-Antiphospholipid antibody syndrome, Nephrotic syndrome
Reference : UPTODATE
6-ITP case management
Answer : Glucocorticoids ( Prednisone)
Reference: uptodate
7-Pt with past hx of hodgkin lymphoma .. But cured completely .. Presented with back pain Examination and evaluation show paraspinus Edema and fluid collection -ve brucella titer and tuberculin test
,, what the cause ?
A. brucellosis
B. breast cancer
C. recurrent hodgkin lymphoma
answer: B
8-Pt with high aptt what mechanism of action of that drug?
" unfractionated heparin" Antithrombin
Answer:
low dose heparin : inactivates factor Xa and inhibits conversion of prothrombin to thrombin
high dose heparin : inactivates factors IX , X , XI , XII and thrombin and inhibit conversion of fibrinogen
to fibrin .
Aspirin inhibit which product formation Thromboxane A2
9-pt with low hemoglobin and low MCV, which of the following will confirm the dx?
A. Iron level and TIBC
Answer: Peripheral blood smear, if not in the options, then choose A
10-A case of low platelets, low RBCs and low WBCs. What is the diagnosis?
a-.Iron def anemia
b-.Aplastic anemia
Answer: B
Aplastic anemia (AA) is characterized by diminishedor absent hematopoietic precursors in the bonemarrow, most often due to injury to the pluripotentstem cell (Decrease RBCs, WBCs, Platelets).
Reference: Uptodate.
11-A patient with decrease in factor Va. The etiology is due to:
a.Inherited
b.Immune
c.Infection
Answer: A

smle ,2016

252
12-Asymptomatic patient, known case of chronic gastritis, has positive occult bloodstool and his Hb=9.
You will manage him by?
a.IM iron
b.Oral iron
c.Erythropoietin
d.Blood transfusion
Answer: A
(IV iron would be more appropriate answer)
13-a known case of sickle cell disease presented with unilateral lower limb pain since (short period:acute) . Vital signs: Tachycardia . Fever 38.3 . the range of movement is intact with no signs of inflammation over the limb. what is the diagnosis:
a:Vaso-occlusive Crisis
b_Osteomylitis
Answer: A
Findings secondary to vaso-occlusion.
a. Painful crises involving bonebone infarction causes severe pain. This is the
most common clinical manifestation.
Bone pain usually involves multiple sites (e.g., tibia, humerus, femur). It may
or may not be bilateral.
The pain is self-limiting and usually lasts 2 to 7 days.
b. Handfoot syndrome (dactylitis).
Painful swelling of dorsa of hands and feet seen in infancy and early childhood
(usually 4 to 6 months).
Often the first manifestation of sickle cell disease.
Caused by avascular necrosis of the metacarpal and metatarsal bones.
precipitated by infection , fever , dehydration , pregnancy and alcohol
Reference : step up to medicine
14-hemolytic anemia , comb positive , what type of hypersensitivity ?
answer: Type 2
(ANOTHER Q combs positive what's the Dx)
example of type II hypersensitivity is the ABO blood incompatibility where the red blood cells have different antigens, Wiki
15-Patient has been diagnosed with Lymphoma in the past and has received full course of chemotherapy. Now complaining of painless facial swelling, cough and flushing, what is the diagnosis?
A. Superior vena cava obstruction
B. Inferior vena cava obstruction
C. Some type of facial tumor
Answer: A. SVC obstruction http://radiopaedia.org/articles/superior-vena-cava-obstruction
Superior vena cava (SVC) obstruction can occur from extrinsic compression, intrinsic stenosis or thrombosis. Malignancies are the main cause and is considered an oncologic emergency.
Clinical features of acute superior vena cava obstruction include: facial and neck swelling, facial flushing,
bilateral upper extremity swelling, neurological signs, dyspnoea, headache and cough.
16- Long case of boy bleeding epistaxis and ecchymosis with long lab results showing anemia thrombocytopenia and leukopenia, what is the diagnosis:

smle ,2016

253
a- IDA
b- aplastic
c- hypoplastic
d- hemolytic
answer: B
17-SCA patient presents with bloody urine. What is the cause?
a.Recurrent UTI
Answer: ?
Typically, the hematuria is mild and self-limited in SCD. As a rule, the hematuria originates from the
left kidney; this has been attributed to the greater length of the left renal vein and compression of the
left renal vein between the aorta and superior mesenteric artery (ie, the nutcracker phenomenon)
.Hematuria can also be secondary to papillary necrosis.The renal medulla contains the vasa rectae,
that is, the renal tubules and blood vessels located therein. In SCD,increased blood viscosity contributes
to ischemia and eventual infarction that involves the renal microcirculation. Medullary ischemia and infarction cause papillary necrosis.
Renal medullary carcinoma is an uncommon cause of gross hematuria.
It can result from other problems such as urinary infections, renal stones or glomerulonephritis
References
http://emedicine.medscape.com/article/1957952-overview#a7
https://scinfo.org/problem-oriented-clinical-guidelines/specific-problems-hematuria-and-nephropath
18-Pt with vit b 12 deficiency what gastric cell type will be affected
A. Cheif cell
B. Parietal cell
Answer: Parietal cell deficiency as its responsible for intrinsic factor synthesis which is required for vit.
B12 to be absorbed in terminal ileum
Autoimmune metaplastic atrophic gastritis A major component of PA (pernicious anemia) is chronic
atrophic gastritis, which is associated with autoantibodies directed against gastric parietal cells in approximately 90 percent of patients with PA.
http://www.uptodate.com/contents/etiology-and-clinical-manifestations-of-vitamin-b12-and-folatedeficiency?source=machineLearning&search=megaloblastic+anemia&selectedTitle=3~91&sectionRank=1&anchor=H2125777#H2125777
19- Elderly came with leukocytosis... What supports the diagnosis of CML
Answer: The diagnosis of CML is suspected based on the results of a simple blood test The test may
show an abnormally high white blood cell count in blood samples examined under a microscope, less
mature white blood cells, normally found only in bone marrow, are seen
Ref: Medical council of canada exam
on CML---- increase WBC and decreased Leukocyte alkaline phosphatase
on Leukemoid reaction -----increase WBC and Increase Leukocyte alkaline phosphatase
20-patient has splenomegaly and teardrop RBC on blood film:
A. ITP
B. Myelofibrosis

smle ,2016

254
Answer: Myelofibrosis
Ref: http://www.pathologystudent.com/?p=607
21-Sickle cell disease patient with multiple gallbladder stones. what is the Best thing to do:
A:hydroxyurea
B:Cholecystectomy
Answer: B
If the patient does not have symptoms, no treatment is usually necessary. If there is recurrent or severe
pain from gallstones, the gallbladder may need to be removed. Minimally invasive procedures (using
laparoscopy) reduce possible complications
Reference: https://umm.edu/health/medical/reports/articles/sickle-cell-disease
22-to increase the HbF in sicklers, give:
A- Deferoxamine
B- Penicillamine
C- Folic acid
Answer:hydroxyurea, butyrate, decitabine
23-70 years old man complaining of back pain. Calcium: High, IgA,IgM and IgG: Low (i.e. three types of
immunoglobulins are low). What is the diagnosis? (They didnt give any other details)
Answer: ?
Multiple myeloma
Reference:http://www.cancernetwork.com/hematologic-malignancies/non-secretory-myeloma-clinician%E2%80%99s-guide#sthash.nmvYh621.dpuf
Monoclonal Gammopathy of Unknown Significance (MGUS) is the least aggressive subclass of plasma
cell dyscrasia. Patients have only a small abnormal protein spike (<3.0 gm/dl and <2.0 gm/dl for IgG and
IgA and IgM, respectively), minimal or no bone marrow involvement (<10% plasma cells), no bony involvement, normal blood counts and usually normal levels of unaffected antibodies. The urine is usually
free of monoclonal protein, however, it is not unusual to find small amounts of monoclonal light chains
in the urine of MGUS patients. Patients with MGUS have a 20-25% chance of developing multiple myeloma or a related lymphoproliferative disorder. The remainder lead a normal life.
Reference: https://www.clevelandclinic.org/myeloma/mm-pt.htm
24-Patient with past history of Hodgkin lymphoma treated with chemotherapy and radiation; and now
in remission for 10 years. Presented now with back pain. Examination and evaluation showedparaspinusedema and fluid collection. She tested negative for Brucella titer and tuberculin skin test was
negative. What is the most likely cause?
A. Brucellosis
B. BREAST CANCER
C. Recurrent Hodgkin lymphoma
CORRECT ANSWER: B
REFERENCE: Toronto Note 2016 Hematology Section &Medscape
EXPLANATION:
Long term complication of Hodgkin lymphoma is solid tumor (breast cancer, which can metastasize to
bone) due to radiotherapy.

smle ,2016

255
25-Patient with lap result showingincreasedaPTT and prolonged bleeding time. What is the most likely
factor deficient?
A. Factor VII (7)
B. Factor VIII (8)
C. Factor IX (9)
D. VON WILLEBRAND (VWF)
ANSWER: D
REFERENCE: Toronto Note 2016 Hematology Section &Medscape
EXPLANATION:
In patients with factor VII deficiency, the PT is increased (designating abnormality in the extrinsic pathway), while aPPT (which used to measure intrinstic pathway) is normal. Both hemophilia A (factor VIII
deficiency) and hemophilia B (factor IX deficiency) have increased aPPT and normalbleeding time.
von Willebrand diseases (vWD), which results from deficiency in vWF will have increase aPPT and prolonged bleeding time.
26-Hodgkin lymphoma with no fibrosis and eosinophils,Reed Sternberg cell, histiocytes. Which type of
HL is this?
A. Nodular sclerosing Hodgkin lymphoma (NSHL)
B. MIXED-CELLULARITY HODGKIN LYMPHOMA (MCHL)
C. Lymphocyte-depleted Hodgkin lymphoma (LDHL)
D. Lymphocyte-rich classical Hodgkin lymphoma (LRHL)
ANSWER: B
REFERENCE: Medscape
EXPLANATION:
Nodular sclerosingHL (NSHL) is the most common subtype of HL. Morphologically, it shows nodular pattern. There is also fibrosis that divides the LN in nodules. There is thickening in the capsule. Reed-Sternberg cells is present.
Mixed cellularity HL (MXHL) is the second most common subtype of HL. Reed-Sternberg cells is also present here, where they have large, with bilobate, double or multiple nuclei, and a large, eosinophilic nucleolus. It is the most common histological type found in HIV patients.
Lymphocytic-depleted HL (LDHL) is the least common subtype of HL. The infiltrate in LDHL is diffuse and
often appears hypocellular. Large numbers of Reed-Sternberg cells and bizarre sarcomatous variants are
present.
Lymphocytic-rich classical HL (LRHL) Reed-Sternberg cells of the classical or lacunar type are observed,
with a background infiltrate of lymphocytes.
27-Which type of anemia is associated with hyposplenism (Functional Asplenia)?
A. Beta Thalassemia
B. Spherocytosis
C. SICKLE CELL ANEMIA
D. Glucose-6-phosphate dehydrogenase deficiency (G6PD Def.)
ANSWER: C
REFERENCE: Medscape &Toronto Notes 2016 Hematology Section
EXPLANATION:
SCD is the most severe and most common form. Affected individuals present with a wide range of clinical
problems that result from vascular obstruction and ischemia. Although the disease can be diagnosed at
birth, clinical abnormalities usually do not occur before age 6 months, when functional asplenia develops.

smle ,2016

256
Functional asplenia results in susceptibility to overwhelming infection with encapsulated bacteria. Subsequently, other organs are damaged. Typical manifestations include recurrent pain and progressive incremental infarction.
Splenic sequestration crises:
It is uncommon in adults due to asplenia from repeated infarction
28-What is genetic defect (mutation) in Beta Thalassemia?
A. Insertion mutation
B. MISSENCE OR NONSENSE MUTATION
C. Deleting mutation
D. Frameshift mutation
ANSWER: B
REFERENCE: Medscape
EXPLANATION:
Mutations in globin genes cause thalassemias. Beta thalassemia affects 1 or both of the beta-globin genes.
(Alpha thalassemia affects the alpha-globin gene[s].) These mutations, by causing impaired synthesis of
the beta-globin protein component of Hb, result in anemia.
Peripheral smear in beta-zero thalassemia minor showing microcytes (M), target cells (T), and poikilocytes.The genetic defect usually is a missense or nonsense mutation in the beta-globin gene, although
occasional defects due to gene deletions of the beta-globin gene and surrounding regions also have been
reported.
29-How to differentiate between hypersplenism and aplastic anemia?
Answer:
Aplastic Anemia
o It is characterized by diminished or absent hematopoietic precursors in the bone marrow,
most often due to injury to the pluripotent stem cell. major causes: Idiopathic, Cytotoxic drug
& radiation,Drug reaction (Anticonvulsants: carbamazepine Antibiotics: sulfonamides,
chloramphenicol, NSAIDsphenylbutazone, indomethacin , anti-thyroidmethimazole,
propylthiouracil). Toxic chemical, viral infection (EBV, HIV, other herpes) , Immune disorders.
Hypersplenism:
o It is condition in which the spleen becomes increasingly active and then rapidly removes the
blood cells. It can result from any splenomegaly. It is most common with splenomegaly secondary to portal hypertension and haematological disorders
Reference :http://www.researchgate.net/publication/237841754_Hypersplenism_Review_article
30-A patient working in a new place up the hill. Recently, he had raised hemoglobin. No past medical
history. What is the reason?
A. Sleep apnea
B. INCREASED PRODUCTION OF ERYTHROPOIETIN.
C. Carbon monoxide poisoning
D. Polycythemia Vera
ANSWER: B
REFERENCE: Toronto Notes 2016 Hematology Section

smle ,2016

257
EXPLANATION:
One of the causes of erythrocytosis [increase in the number of RBCs: Hb>18.5 mg/dL or Hct>52% (males);
Hb>16.5 mg/dLor Hct>47%(females and African males)] is high altitude, which is a normal physiological
reaction to poor tissue oxygenation from high altitude.
31- Patient has HbF(5%) and HbA2 (5%) are present. What is the diagnosis?
A. BETA THALASSEMIA MINOR
B. Beta Thalassemia major.
C. Alpha Thalassemia.
D. Sickle cell anemia
ANSWER: A
REFERENCE: Toronto Notes 2016 Hematology Section
EXPLANATION:
Beta thalassemia minor (trait):
o

HbF non-specific: 50% have slight increase.

HbA2 specific: increased to 3.5-5% (normal 1.5-3.5%)

Beta thalassemia major


o

HbA 0-10% (normal >95%)

HbA2>2.5%

HbF 90-100%

32- Which condition has increased osmotic fragility?


A. Hereditary elliptocytosis
B. HEREDITARY SPHEROCYTOSIS
C. Glucose-6-phosphate dehydrogenase deficiency (G6PD Def.)
D. Autoimmune hemolytic anemia
ANSWER: B
REFERENCE: Toronto Notes 2016 Hematology Section &Medscape
EXPLANATION:
Blood film (shows spherocytes), osmotic fragility (increased), molecular analysis for spectrin gene
The most sensitive test for HS is the incubated osmotic fragility test, which is performed after incubating
RBCs for 18-24 hours under sterile conditions at 37C. Osmotic fragility testing with RBCs that have not
been incubated may demonstrate hemolysis of HS cells in some patients but is not reliable. This is especially true of newborns; as fetal RBCs are generally more resistant to osmotic hemolysis. Incubated osmotic fragility test results usually are abnormal.
33. A patient with vit B12 deficiency. what is the type of gastric cell that will be affected?
A. Chief cell
B. Parietal cell
Answer: B
Patient with pernicious anemia is usually directed against the intrinsic factor or parietal cell themselves.

smle ,2016

258
34- Which of those diseases is most common in the Mediterranean Sea Area?
A. BETA THALASSEMIA
B. Alpha Thalassemia
C. Sickle cell anemia
D. Glucose-6-phosphate dehydrogenase deficiency (G6PD Def.)
ANSWER: A
REFERENCE: Toronto Notes 2016 Hematology Section&Medscape
EXPLANATION:
Beta thalassemia is more common in Mediterranean Sea Area; whereas Alpha Thalassemia is more common in South East Asia & Africa.
Sickle cell anemia, although common in Mediterranean Sea Area, is most common in sub-Saharan Africa
and India.
35-What type of anemia is associated with rheumatoid arthritis?
A. Normocytic normochromic anemia
B. MICROCYTIC HYPOCHROMIC ANEMIA
C. Macrocytic hypochromic anemia
D. Aplastic anemia
ANSWER: B
REFERENCE:Toronto notes 2016 Hematology Section&Master the Board
EXPLANATION:
Anemia resulting from rheumatoid arthritis iscategorized as anemia of chronic diseases, which is a subtype of microcytic hypochromic anemia.
36-What is the most common symptom of factor 8 deficiency?
A. Bleeding from gum and tongue
B. Iliopsoas bleeding
C. HEMARTHROSIS
D. Hematuria
ANSWER: C
REFERENCE:https://www.cdc.gov/ncbddd/hemophilia/course/page995.html
37-Chemotherapy can cuse which type of anemia?
A. Normocytic normochromic anemia
B. Microcytic hypochromic anemia
C. Macrocytic hypochromic anemia
D. APLASTIC ANEMIA
ANSWER: D
REFERENCE: Toronto notes 2016 Hematology Section&Master the Board
EXPLANATION:
Chemotherapeutic agents will suppress the bone marrow, leading to failure of hematopoiesis and anemia.
38-Elderly patient presented with typical symptoms of multiple myeloma. X-ray showed lytic lesions.
Serum protein electrophoresis (SPEP) showed positive M protein and hypercellular bone marrow. What
other findings can be found in the blood?

smle ,2016

259
A. Increase peripheral blood B cell
B. ROULEAU FORMATION
C. Reticulocytosis
D. Lymphocytosis
ANSWER: B
REFERENCE: Toronto Notes 2016 Hematology Section&Medscape
EXPLANATION:
Perform a complete blood count (CBC) to determine if the patient has anemia, thrombocytopenia, or
leukopenia. The CBC and differential may show pancytopenia. The reticulocyte count is typically low. Peripheral blood smears may show rouleau formation.
39-Elderly man on NSAIDs developed dyspepsia. Endoscopy showed gastritis.
Labs: iron deficiency anemia with Hb= 9. What is the treatment?
A. IV IRON
B. IM iron
C. Erythropoietin
D. Oral Iron
ANSWER: A
REFERENCE: Toronto notes 2016 Hematology Section
EXPLANATION:
The first line in treating iron deficiency anemia is oral supplements. However, for this case, the patient is
using NSAIDs, and he is having dyspepsia, so oral ferrous sulfate will be associated with more GI side
effects than IV iron.
Thus, we will opt to treat with IV ferrous sucrose.
Other indication for IV ferrous sucrose is when the patient cannot tolerate oral supplements, or they have
problems with absorbing iron.
40-How to monitor the response to iron treatment?
A. Ferritin
B. Hematocrit
C. RBC
D. RETICULOCYTE COUNT
ANSWER: D
REFERENCE: Toronto notes 2016 Hematology Section
EXPLANATION:
Monitoring response:
reticulocyte count will begin to increase after one wk
Hb normalizes by 10 g/L per wk (if no blood loss)
Iron supplementation required for 4-6 mo to replenish stores.
41-Acquired malabsorption for iron with autoimmune atrophic gastritis or Helicobacter pylori infection?
Reference:http://www.uptodate.com/contents/treatment-of-the-adult-with-iron-deficiency-anemia

smle ,2016

260
42-A male patient presented with symptoms. Labs showed 80% blasts with 20% auer rods. What is the
diagnosis?
A. Chronic myeloid leukemia (CML)
B. Acute myeloid leukemia (AML)
C. Chronic lymphocytic leukemia (CLL)
D. Acute lymphocytic leukemia (ALL)
ANSWER: B
REFERENCE: Toronto notes 2016 Hematology Section
EXPLANATION:
Circulating blasts with Auer rods (azurophilic granules) are pathognomonic for AML.
43-Patient on warfarin 7 mg presented with melena. INR was very high. What will you do?
Answer:?
The treatment of excessive anticoagulation is based on the level of the INR, the presence or absence of
bleeding, and clinical circumstances. Reversal of Warfarin sodium anticoagulation may be obtained by
discontinuing Warfarin sodium therapy and, if necessary, by administration of oral or parenteral vitamin
K1.
Reference:http://www.drugs.com/pro/warfarin.htm
Reference: Toronto Notes.

44-A patient known case of anemia on medications. Later he came complaining of dark stools. What is
the medication?
A. FERROUS SULFATE
B. Folic acid
C. Iron dextran
ANSWER: A
REFERENCE:Toronto notes 2016 Hematology Section
https://www.nlm.nih.gov/medlineplus/ency/article/007478.htm
45- What is the diagnostic test for Sickle cell anemia?
A. Complete blood count (CBC)
B. Peripheral blood smear
C. HB ELECTROPHORESIS
D. Bone marrow aspiration
ANSWER: C
REFERENCE:Toronto notes 2016 Hematology Section&Medscape
EXPLANATION:

smle ,2016

261
Hemoglobin electrophoresis differentiates individuals who are homozygous for HbS from those who are
heterozygous. It establishes the diagnosis of SCD by demonstrating a single band of HbS (in HbSS) or HbS
with another mutant hemoglobin in compound heterozygotes.
In children with normocytic hemolytic anemia, if results of electrophoresis show only HbS with an HbF
concentration of less than 30%, the diagnosis is sickle cell anemia. If HbS and HbC are present in roughly
equal amounts, the diagnosis is HbSC disease
46-68 old guy with painless, palpable cervical lymph nodes and B-symptoms. Microscopically,the biopsied lymph nodes showed Reed-Sternberg cells. What is best management for this patient?
A. CHOP
B. CHOP-R
C. ABVD
D. ABVD+XRT
ANSWER: D
REFERENCE: Toronto notes 2016 Hematology Section
EXPLANATION:
The painless, palpable cervical lymph nodes, alongside the microscopic findings of Reed-Sternberg cells,
all indicate the diagnosis of HODGKIN LYMPHOMA. There is only one lymph nodes group involved, so it is
stage I (localized disease). Treatment is ABVD+XRT
B symptoms 1) Fever || 2) Night sweats || 3) Weight loss
NHL:
Local disease (stage I and Il): local radiation and small dose/course of chemotherapy
Advanced disease (stage III and IV, OR any "B" symptoms): combination chemotherapy with CHOP-R.
HL:
Stage I and Il: small course of chemotherapy followed bylocal radiation (ABVD+XRT).
Stage III and IV OR anyone with "B" symptoms: ABVD
C = Cyclophosphamide
H = Hydroxydaunorubicin (Doxorubicin, or Adriamycin Antibiotics)
O = Oncovirin (Vincristine, Vinca-alkaloids)
P = Prednisone
R = Rituximab ( antibody against CD20)
A = Adriamycin (Doxorubicin, or Hydroxydaunorubicin)
B = bleomycin ( Antibiotics)
V = Vinblastine (Vinca-alkaloids)
D = Dacarbazine ( Alkylating agent)
XRT = Radiotherapy
47-Reed-Sternberg cells is found in which malignancy?
A. Multiple myeloma
B. Hodgkin lymphoma
C. Acute myeloid leukemia
D. Myelodysplastic syndrome
ANSWER: B

smle ,2016

262
REFERENCE: Toronto notes 2016 Hematology Section
EXPLANATION:
The histological hallmark of HL is the presence of ReedSternberg cells, large malignant lymphoid cells of
B cell origin.
48- Sickle cell anemia false positive test because:
A. HIGH PROTEIN LEVEL
B. Protein c
C. Protein d
ANSWER: A
REFERENCE:http://education.questdiagnostics.com/faq/FAQ99
EXPLANATION:
A positive test is consistent with sickle cell trait (hemoglobin A/S), sickle cell anemia (hemoglobin S/S),
hemoglobin S in combination with another hemoglobin variant, or hemoglobin C Harlem.
Positive results also occur in polycythemia, multiple myeloma, or cryoglobulinemia.

49- Patient known case of sickle cell anemia came with hepatosplenomegaly and low platelets. What is
the treatment?
A. Splenectomy
B. BLOOD TRANSFUSION
ANSWER: B
REFERENCE: Medscape
EXPLANATION:
Transfusions are not needed for the usual anemia or episodes of pain associated with SCD.
Urgent replacement of blood is often required for sudden, severe anemia due to acute splenic sequestration, parvovirus B19 infection, or hyperhemolytic crises.
Transfusion is helpful in acute chest syndrome, perioperatively, and during pregnancy. Acute red cell exchange transfusion is indicated in the following situations:
For anemic crisis with splenic sequestration, give early red cell transfusions because the process can rapidly progress to shock. Do not allow hemoglobin (Hb) levels to rise to more than 10 g/dL, since the spleen
may disgorge trapped cells, which can create a relative polycythemia and increased blood viscosity.
Children who experience a single sequestration event frequently have recurrences. Surgical splenectomy
or a short-term transfusion regimen has been suggested for this complication.

smle ,2016

263
50- What is HELLP syndrome?
A. Hemolysis + Elevated Liver enzymes + Low Platelets
ANSWER: A
REFERENCE: Medscape
EXPLANATION:
HELLP syndrome, named for 3 features of the disease (Hemolysis, Elevated Liver enzyme levels, and Low
Platelet levels), is a life-threatening condition that can potentially complicate pregnancy.
Although the idea is controversial, some propose that HELLP is a severe form ofpreeclampsia, which, in
turn, is defined as gestational hypertension accompanied by proteinuria after the 20th week of gestation.
Others believe that HELLP syndrome is an entity of its own. Although the cause of HELLP syndrome is
unknown, certain risk factors, including a maternal age of older than 34 years, multiparity, and European
descent, have been described.
51-A patient present with fatigue, palpitation, SOB and pallor, Hgb 9. Shown is the peripheral film. What
is the type of anemia?
A. MEGALOBLASTIC ANEMIA
B. B. Hypochromic microcytic
C. C. Sickle cell
D. D. G6PD deficiency
ANSWER: A
REFERENCE: Toronto Notes 2016 Hematology Section
EXPLANATION:
Note that RBCs are as large as the neutrophil and lymphocyte. In sickle cell anemia, the RBCs will be sickled. In G6PD deficiency, blood film will show bite cells and Heinz bodies.
52-post-transfusion fever how to prevent?
A. Transfusion filter Depleted leukocyte
B. Premedication with antipyretics
C. REMOVAL OF PLASMA
ANSWER: C
REFERENCE: Toronto Notes 2016 Hematology Section
EXPLANATION:
A small fraction of FNHTRs are not eliminated by third generation filtration (post-storage). Cytokine accumulation during storage may play a primary role in this setting, since it has been demonstrated that the
risk of a transfusion reaction increases with the age of the unit transfused, and that removal of plasma is
more effective than post-storage leukoreduction in preventing reactions, especially severe reactions to
platelets.
53- 45-year-old patient presented with abdominal pain and fatigue.He has a family history of hemochromatosis. What is the next step in investigation?
A. Ferritin
B. TRANSFERRIN SATURATION
C. Genetic testing HFE mutations
D. Liver biopsy
ANSWER: B

smle ,2016

264
REFERENCE: Toronto Notes 2016 Hematology Section
EXPLANATION:
Transferrin saturation corresponds to the ratio of serum iron and total iron-binding capacity (TIBC). The
screening threshold for hemochromatosis is a fasting transferrin saturation of 45-50%. If transferrin saturation is greater than 45%, the presence of the C282Y or H63D mutation may be evaluated to confirm the
diagnosis of hemochromatosis.
Hemochromatosis is suggested by a persistently elevated transferrin saturation in the absence of other
causes of iron overload. This is the initial test of choice.
54-A patient present with fatigue, palpitation, SOB and pallor, his Hgb 9. Shown is the peripheral film.
What is the type of anemia?
A. Megaloblastic anemia
B. Hypochromic microcytic
C. Sickle cell
D. G6PD
Answer: A
Note that RBCs are as large as the neutrophil and lymphocyte.
55- 40 years old man, Hb: low, MCV: high, No megaloblast. He has no past medical history. What is the
most likely diagnosis?
A. ALCOHOLIC ANEMIA
B. Lead poisoning
C. Vitamin B12 deficiency
D. Anemia of chronic disease
CORRECT ANSWER: A
REFERENCE: Toronto Notes 2016 Hematology Section
EXPLANATION:
Macrocytic hypochromic non-megaloblastic anemia Differentials:
1. Liver disease
2. Alcoholism
3. Reticulocytosis
4. Hypothyroidism
5. Myelodysplasia.
56-What the hematological parameter should be investigated before performing lumbar puncture?
A. Hemoglobin level
B. WBC with differentials
C. PLATELETS
D. Hematocrit
ANSWER: C
REFERENCE: Toronto Notes 2016 Neurology Section
EXPLANATION:
Lumbar puncture is CONTRAINDICATEDin:
Mass lesion causing increased ICP Brain CT should be done when suspecting brain mass
Infection over LP site

smle ,2016

265

Suspected epidural abscess


LOW PLATELETS (<50.000)
Treatment with anticoagulants ( INR or prolonged aPPT)
Uncooperative patient

57-Which of the following can be found on smear in sickle cell disease?


a.Bite cells
b.Howell-Jolly bodies
c.Acanthocyte
d.Spherocyte
Answer: B
Reference: Master the Boards
Bite cells , heinz bodies : G6PD
Howell-Jolly bodies : SCD , post spleenectomy , neonate , megaloplastic anemia
Acanthocyte : Sever liver dis , anorexia , post spleenectomy
Spherocyte : herditery spherocytosis , immune hemolytic anemia , post transfusion .
Ref: TORONTO NOTES .
58-Old female live alone change her dietary habit what is the cause of anemia?
A. Fe deficiency
Ref: TORONTO NOTES .
59-about. Old lady with forgetfulness and numbness?
A. B12 deficiency
60-patient with anemia, thrombocytopenia, splenomegaly and hepatomegaly investigation ?
A. Bone marrow biopsy
61-pt known case of g6pd low hg what will u do?
A-blood transfusion
B-folic acid
C-reassure
Most individuals with (G6PD) deficiency do not need treatment. hemolysis is self-limited and often .resolves in 8 to 14 days. Transfusions Are rarely indicated. Splenectomy is usually ineffective
NOTHING REVERSES THE HEAMOLYSIS , avoid oxidant stress
The ttt of G6PD :
Folic acid , avoid triggers , blood transfusion in sever cases .
62-Patients with chronic hemolysis or non-spherocytic anemia should be placed On
A. daily folic acid supplements
63-Pt was anemic, being given blood transfusion. Developed fever, chills, burning at site of IV line,
what to do:
A. stop transfusion and give crystalloids
Answer:A
Ref: TORONTO NOTES

smle ,2016

266
64-what vitamin increase metabolize or absorption of iron :
A.
vitamin c
B.
vitamin B
C.
vitamin D
answer :A
Please note the following:

Although the supplements work best on an empty stomach, you may want to take them with
food so that they don't upset your stomach.

You shouldn't take iron supplements with milk, caffeine, antacids, or calcium supplements.
These can decrease the amount of iron that is absorbed.

Try to take your iron supplement with vitamin C (for example, a glass of orange juice) to increase
absorption.
https://my.clevelandclinic.org/health/diseases_conditions/hic_Anemia/hic_oral_iron_supplementation
Absorbtion is 5-10% enhanced by citiric acid , ascorbic acid , vit C , Reduced by polyphenols , phytate ,
calcium , soy protein .
65-Sickler with VOC dehydrated and his hemoglobin is 3.5 what the next step in management ?
A- PRBcs transfusion
B- analgesia and IV fluids
Answer : IV fluid to rehydration then blood transfusion
https://www.nhlbi.nih.gov/files/docs/guidelines/sc_mngt.pdf
66-which is feature of VWD:
Answer: prolonged Bleeding time

Ref:TORONTO NOTES .

smle ,2016

267
67-Most common Mendelian inheritance ?
A. Thalassemia
https://en.wikipedia.org/wiki/Thalassemia
68-High d-dimer ..acute case What would you give this pregnant woman?
A. Unfractionated heparin and warfarin
B. LMWH
C. Warfarin
D. Aspirin
ANSWER IS A
Ref: TORONTO NOTES .
69-women (complain ) lab .. high platelets. .Treatment
A. Plateletpheresis ??
70-which type of anemia associated with chemotherapy and Radiotherapy
Answer: Aplastic anemia
Ref : MASTER THE BOARD
71-cavernous sinus thromboembolism sinus more common? (need to be check)
A- ophthalmic
B- maxillary
C- sphenoid
Answer: C Most commonly with ethmoid
https://en.wikipedia.org/wiki/Cavernous_sinus_thrombosis
CST most commonly results from contiguous spread of infection from a nasal furuncle (50%), sphenoidal
or ethmoidal sinuses (30%) and dental infections (10%).[3] Less common primary sites of infection include tonsils, soft palate, middle ear, or orbit (orbital cellulitis)
72-Child with mild jaundice ,splenomegaly and echogenicity in the gallbladder .what is the type of
anemia?!
A.Sickle cell anemia
B.Thalassemia
Answer: A Toronto notes
73-Cervical LN enlargement, hepatosplenomegaly, circumoral edema, acute presentation..what is the
diagnosis?
A. Lymphoma
B. Angioedema
Answer : b ?
https://en.wikipedia.org/wiki/Angioedema
http://emedicine.medscape.com/article/135208-clinical#b2
74- Polycythemia vera with blurred vision And headache What the cause of these sx?
a) Hypovolemia
b) Hyperviscosity

smle ,2016

268
Answer: B
Ref: MASTER THE BOARD
75-Case of iron deficiency anemia lab showed typical microcytic with high TIBC , mechanism :
Decrease RBCs more than serum volume )
76-Burkitt's lymphoma associated with ?
A- Epstein Barr Virus &quot;EBV
B- T Lymphocyte Infiltration
Answer : A
Associated with EBV , HIV
Ref: TORONTO NOTES
77-Scenarios about vegetation with lap showing microcytic hypochromic anemia what's the dx
Answer : IDA
78-Long history about DIC, lab showing fragmented RBC, low platelets. which antibodies is target :
A. Cardiolipins
B. ADAMTS13
C. Glycoprotein
Answer : ?
79-16 years old boy known case of sickle cell anemia presented to with painful right hip pain for several weeks (this was the scenario and it was for several weeks ). what is the most likely diagnosis:
A. avascular necrosis
B. still's disease
C. tumor
..osteomyelitis was not in the choices
80-Pts take methotrexate for something present with tiredness with labe result, High MCV , What to
give?
A. Iron
B. Folinic acid
C. Folic acid
ANSWER : C
81-Elderly, asymptomatic, with 90% lymphoblast.. what to do for him?
A- Observation
B- Chemo
Answer : B
Ref : TORONTO NOTES
82-male patient from ghenea with a neck mass they took biopsy and it shows starry appearance
A. Burkitt lymphoma
Ref: http://www.pathologystudent.com/?p=2283

smle ,2016

269
83-A 60-year-old male with history of lower back pain , constipation thirstLow hemoglobin , Low WBC
, Low PLT , High caand Lower spinal X-ray pic was attachedWhat is the next appropriate to be ordered:
A. DEXA
B. Protein electrophoresis
C. parathyroid hormone
Answer: B
Ref:MASTER THE BOARD
Serum protien electrophoresis showes an IgG60% , igA 25%
84-which type of anemia have high A2 :
a) SCD
b) Thalassemia
c) Spherocytosis
Answer: b
Ref : https://en.wikipedia.org/wiki/Hemoglobin_A2
85-Craving ice pt with anemia?
a)Pagophagia
Answer: A
Source:http://www.mayoclinic.org/diseases-conditions/iron-deficiency-anemia/expert-answers/chewing-ice/faq-20057982
86-microcytic normochromic result in patient with pallor
a-IDA
b- hemolytic
c-folate
Answer : A MASTER THE BOARD
87-purpura ( immune,, henoch purpura)
A. lesion in MM
88-patient with multiple blood transfusion and jaundice + osmotic fragility test
A. spectrin-ankyrin binding deficiency (spherocytosis)
89-75 years old male, asymptomatic, BM report:increased lymphocytes, Immunohistochemistry: Positive CD19, CD56 .. Treatment?
A. No treatment
B. Rituximab + CVB
C. Rituximab + Prednisolone* (my answer)
D. Cyclophosphamide
Answer : ??
90-65 years old male, presenting with peripheral neuropathy which progressed to weakness (Subacute combined degeneration), labs shows Macrocytic anemia, Diagnosis?
A- Vit. B12 Deficiency

smle ,2016

270
91-young pt presents with cough and chest pain, cxr showed infiltrate, CBC: anemia with high retic
(10%) and leukocytosis, Dx?
A. sickle cell anemia
answer: A (sickle cell anemia presenting with acute chest syndrome supported by clinical and radiological evidence)
92-young pt with continuous bleeding post-op, coagulation showed high APTT, otherwise normal,
which factor is deficient?
A. factor VIII
https://en.wikipedia.org/wiki/Haemophilia_A
93-young male patient, asymptomatic, for routine check up, lab showed microcytic hypochromic anemia, Dx?
A: thalassemia trait (I CHOSE IT)
?Iron def. anemia (since he is young, not female, I excluded it)
94-Most common cause of death in sickle cell anemia?
a)Aplastic crises
b)Sequestration crises
c)Acute chest syndrome
d)Parvovirus b19
Answer:C Acute chest syndrome
https://www.nhlbi.nih.gov/health/health-topics/topics/aplastic/signs
95-long scenario with paragraph pt have low hemoglobin and macrocytic anemia he treated now
what does brown line mean if the blue are hemoglobins ?
a-reticulocyte
b-hematocrit
c-RBCs
d-MCHC
((look like picture in attachment ?? ))
96-What is the reliable method to diagnose ALL:
A/ lymph node involvement
B/anemia and thrombocytopenia
C/ bone marrow blast cell
97-Patient with fatigue and weakness , lab results showing :
Hemoglobin : decreased
MCH : decreased
RDW : 13:
reference OM ALQORA
98-ttt of plummer vinson? Treatment has been expanded.
Taking iron supplements may improve the swallowing problems.
If supplements do not help, the web of tissue can be widened during upper endoscopy. This will allow
you to swallow food normally

smle ,2016

271

99-pt from africa presented with symptoms of burkitt lymphoma what gene associated with :
A. ABL
100-24 years old female, has 3 LAN at the same side of diaphragm with no distant metastasis, Diagnosed with Hodgkin's Lymphoma. What's the stage?
A- 4
B- 3
C- 2
D- 1 * (not sure but it's not 3 or 4)
101- Crescentic cell cause
A. Hemolytic anemia
102- patient referred from cardiac unit due to severe decrease in platelets < 10,000. ( case of heparin
induced thrombocytopenia) What is the treatment :
A. Platelets transfusion .
B. Argatroban
C. IVIG
D. Steroid.
Others treatment also can be: lepirudin
Answer: B
Treatment of HIT :
Discontinue and avoid all heparin products immediately
Platelet transfusions should be avoided in HIT, as they may increase the thrombogenic effect
Patients with HIT are at high risk for thrombotic events and should be treated with alternative anticoagulants, typically a direct thrombin inhibitor (DTI).
Direct thrombin inhibitor (DTI) argatroban (Acova) for prophylaxis and treatment of thrombosis in patients with HIT. DTI bivalirudin is approved for use in patients who are undergoing percutaneous coronary intervention (PCI) and have, or are at risk for, HIT or HIT with thrombosis (HITT).
Reference : Medscape
103-CML associated with which translocation :
A- t (14; 18)
B- t ( 11;14)
C- t (8; 14)
D- t (9; 22).
Answer : D
CML is almost invariably associated with an abnormal chromosome 22 known as the Philadelphia chromosome, often abbreviated as Ph, Ph(1), or Ph1[1,2]. The Philadelphia chromosome t(9;22)(q34;q11)
results in the formation of a unique gene product (BCR-ABL1)
104-Vegetarian with numbness and soak and gloves distribution
A. B6 deficiency
B. B12 deficiency
Answer: B

smle ,2016

272

105-for Hodgkin lymphoma What the gene response


A. BRC-A
Answer:
Activation of the transcription factor NF-B is common in classical HL.
REF : goljan : Rapid Review Pathology
106. case of pt with hemochromatosis what will be increased
A. Ferttin
B. Ceruloplasmin
C. Irrelevant
Answer: A
107. In thalassemia
A. 4 abnormal beta chain genes, normal 2 alpha chain genes
B. 4 normal beta chain genes, abnormal 2 alpha chain genes
C. 2 abnormal beta chain genes, normal 4 alpha chain genes
D.2 normal beta chain genes, abnormal 4 alpha chain genes
108-mechanism of action of heparin is
Heparin binds to the enzyme inhibitor antithrombin III (AT
109-14 year old girl patient ectric (jaundice) sore throat , there is blood film shows (spherical shape
RBC) what is your diagnosis?
A. G6PD deficiency
B. spherocytosis
C. c-sickle cell anemia
110-case of lymphoma , treatment :
a-Chop r
b-Abvd
111-How to stop bleeding in VWD ?
A- Fresh frozen plasma
B- Vit. K
C- Platelets Transfusion
D- something irrelevant !!
Answer : Neither DESMOPRESSIN nor Factor VIII concentrate were in the
choices !!!
112-Hx of infection, Hb low, WBC high what is the investigation :
a. Bone marrow biopsy
b. Hb elect
Answer: B (SCA)
Ref: http://emedicine.medscape.com/article/205926-workup#c1

smle ,2016

273
113-case scenario of young pt has sudden onset of dyspnea, decreased MCV , Normal WBC , Increased
Platelets , peripheral smear shows Microcytic hypochromic anemia :
A. Iron deficiency anemia- thalassemia
the answer: is IDA
Reference:http://emedicine.medscap
114-sickle cell disease with hip pain?
answer: AVN (similer casehttp://gait.aidi.udel.edu/educate/sickle.htm )
115-question about splenic sequestration ( with lobar infiltrate in lung )
probably a combined splenic sequestration + acute chest syndrome ? incomplete Q
116-Blood film attached, asking for diagnosis:
a- leishmaniasis
b- malaria
c- lymphoma
d- leukemia
answer: according to the slide
117-Leukemia bleeding is characterized by:
Multiple choices about abnormal bleeding test results.
Answer:?!
118-B6 & B12 deficiency
B6 deficiency:
Overt deficiencies of vitamin B6 are probably rare. Marginal deficiencies may be more common, manifested as nonspecific stomatitis, glossitis, cheilosis, irritability, confusion, and depression.
The classic clinical picture of cobalamin deficiency, mentally sluggish, shiny tongue (atrophic glossitis),
and a shuffling broad-based gait. hematologic changes (eg, macrocytic anemia with oval macrocytes and
increased neutrophil lobulation) and neurologic abnormalities classic picture of subacute combined degeneration of the dorsal (posterior) and lateral spinal columns, neuropathy which is symmetrical and affects the legs more than the arms. It begins with paresthesias and ataxia associated with loss of vibration and position sense, and can progress to severe weakness, spasticity, clonus, paraplegia, and even
fecal and urinary incontinence.
Ref: uptodate
119-spoon shaped nail ?
Answer: iron deficiency
120-Scenario about hereditary spherocytosis and its lab findings ?
Answer:
Medscape: The classic laboratory features of hereditary spherocytosis (HS) include the following: Mild to
moderate anemia, Reticulocytosis, Increased MCHC, Spherocytes on the peripheral blood smear, Hyperbilirubinemia, Abnormal results on the osmotic fragility test
Red blood cell (RBC) morphology in HS is distinctive yet not diagnostic. Anisocytosis is prominent, and
the smaller cells are spherocytes. Unlike the spherocytes associated with immune hemolytic disease and

smle ,2016

274
thermal injury, HS spherocytes are fairly uniform in size and density. Spherocytes are characterized by a
lack of central pallor, decreased mean corpuscular diameter, and increased density.
121-cases about IDA
Answer: iron deficiency anemia
ttt:
1-Oral iron therapy: initial therapy, gastrointestinal side effects are extremely common and may result in
poor adherence.
2-Parenteral iron: for those unresponsive to or intolerant of oral iron, for pt with IBD, gastric bypass surgery.
3-BT: for pt who are hemodynamically unstable because of active bleeding.
http://www.uptodate.com/contents/treatment-of-the-adult-with-iron-deficiency-anemia
122-case of multiple myeloma
Answer:
-cancer of plasma cells in the bone marrow.
-Criteria for diagnosis:
- A bone marrow aspirate or biopsy showing that at least 10% of the cells are plasma cells or the presence of a plasma cell tumor (called a plasmacytoma), plus at least one of the following two features:
- Evidence of damage to the body as a result of the plasma cell growth, as severe bone damage, kidney
failure, anemia, or high calcium in the blood, and/or
- Detection of one of the following findings: 60% plasma cells in the bone marrow; free light chain ratio
of 100 or more (provided involved FLC level is at least 100 mg/L); or MRI showing more than one lesion
(involving bone or bone marrow).
variety of symptoms:
-bone pain in the back or chest, or less commonly, the arms and legs. The pain is usually triggered by
movement and is absent at night, except when changing positions, The bone loss and erosions can lead
to osteoporosis and fractures pathological fracture
-High blood calcium levels : hyperCa symptoms loss of appetite, nausea, vomiting, frequent urination,
increased thirst, constipation, weakness, confusion, stupor, or coma.
-Anemia: paleness, weakness, and fatigue.
-Impaired kidney function: Occasionally, kidney failure is the first sign of MM.
-hyperviscosity syndrome The symptoms may include bleeding from the nose and mouth, blurred vision,
neurologic symptoms, and heart failure.
-Neurologic symptoms radiculopathy causing numbness or tingling, pain, or muscle weakness.
- Generalized symptoms The generalized symptoms of MM include an increased susceptibility to infections (especially during chemotherapy) and weight loss.
Dx: Blood and urine tests for monoclonal protein , Bone marrow examination (aspiration and biopsy)
http://www.uptodate.com/contents/multiple-myeloma-symptoms-diagnosis-and-staging-beyond-thebasics#H3498669556
123-clear question about aplastic anemia
Answer:
Aplastic anemia is pancytopenia of unclear etiology. Any infection or cancer
can invade the bone marrow, causing decreased production or hypoplasia.
Other causes of pancytopenia are:
Radiation and toxins such as toluene, insecticides (DDT), and benzene
Drug effect: sulfa, phenytoin, carbamazepine, chloramphenicol, alcohol,

smle ,2016

275
chemotherapy
SLE
PNH
Infection: HIV, hepatitis, CMV, EBV
B12and folate deficiency
Thyroid-inhibiting medications such as propylthiouracil (PTU) and
methimazole
Presentation/Diagnostic Tests
Patients present with the fatigue of anemia, infections from low white cell
counts, and bleeding from thrombocytopenia. Aplastic anemia is confirmed
by excluding all the causes of pancytopenia. The most accurate test is a bone
marrow biopsy.
Treatment
Besides supportive therapy such as blood transfusion for anemia, antibiotics for infection, and platelets
for bleeding, you should treat any underlying
cause that is identified. A true aplastic anemia is treated with allogeneic bone
marrow transplantation (BMT) if the patient is young enough and there is a
matched donor.
When the patient is too old for BMT (above age 50) or there is no matched
donor, the treatment is antithymocyte globulin (ATG) and cydosporine.
Tacrolimus is an alternative to cyclosporine.
Large vessel thrombosis of
the mesenteric and hepatic
veins is the most common
site of thrombosis.
Aplastic anemia acts as
an autoimmune disorder
in which the T cells
124-question about autoimmune hemolysis
Answer:
Autoimmune Hemolysis
Look for other autoimmune diseases in the history,such as SLE or rheumatoid arthritis. Other clues are a
history of chronic lymphocytic leukemia (CLL),
lymphoma, or medications such as penicillin, alpha-methyldopa, quinine, or
sulfa drugs.
Diagnostic Testing
The LDH, indirect bilirubin level, and reticulocyte count will be elevated as
with all forms of hemolysis. The haptoglobin level can be decreased in both
intravascular and extravascular forms of hemolysis.
Peripheral smear: May show spherocytes
Most accurate diagnostic test: Coombs test
Treatment
Best initial therapy: Steroids,such as prednisone
If the case describes recurrent episodes of hemolysis, splenectomy is the
most effective therapy. Rituximab works on both IgG and IgM.
The antibodies found in Coombs test are also called warm antibodies,
which are IgG. Only IgG antibodies respond to steroids and splenectomy.

smle ,2016

276

125-women (complain )lab .. high platelets ..Treatment?

over-the-counter, low-dose aspirin to reduce blood clotting

prescription medications to lower the risk of clotting or to reduce platelet production in the bone
marrow

plateletpheresis,
126-about Hemolytic anemia dx with peripheral blood smear shows ?
microspherocytosis
Answer: hemolytic anemia (read about it)
127-Sickle cell anemia dx (in hx there is typical feature of hand foot syndrome:
Answer: SCA (Repeated 2 times in exam)
128-anemia + glossitis + paresthesia:
129-pernicious anemia
130-Clinical dementia ?
Vit b12 def
131- Case of thalassemia >> splenomegaly +frontal protru
132-what is papanicolaou smear ? ( Choices: how many sample and how many area ?)
http://www.uptodate.com/contents/screening-for-cervical-cancer?u
133-Hemolytic anemia:
Unconjugated bilirubin
134-question about ( autoimmune hemolysis)

135-Patient with high Ca and low Iga Igm what is the diagnosis (this Q was already submitted by one
of our colleagues but the answer in the (gathered smle) had multiple myeloma and an explanation to
a subtype of it. In the exam they offer you both multiple myeloma and it's subtype as choices and you
have to choose between the two
136-Question about Hodgkin's lymphoma
137-Fanconi anemia :
autosomal recessive
138-young patient came with abd pain and tenderness in LUQ and splenomegaly , febrile,
139-Von willebrand disease > pathophysiology
FACTOR VIII

smle ,2016

277

140-2 cases of anaemia. (interpretation)


141-Pt. with severe IDA Hg=10 range they put (120something) what to do first
blood transfusion . ( no iron trial in other choices)
142- 31years old male presented with recent tiredness and dizznes he give history of change of his
bowel habit since awhile, in that he change his diet to gaurge ( something I don't know type of food )
on lap he had hypochromic microcitic anemia and iron deficiency What could be the cause : Change in
his bowel habit Change in diet His age I don't remmber it was irrelevant hyperglobulinemia, not sure
143-Peripheral blood smear about leukemia I forgot it
144- Sickle cell having pain in the hand ( Vasoocclusive crisis ) what is the most probable diagnosis:
splenic sequestration
answer : need more details read about sickle cell crisis
145. Hemolytic anaemia patient with schistocyte normal apt and Bt what is management
A. acyclovir.
B. antibiotic if case of hus tx supportive
146. spectrin-ankyrin binding deficiency
Answer: (spherocytosis)
147. Child given-heparin blood +FTP the dz is
A. DIC with thromboses
reference: OM ALQORA
148. Which bleeding disorder presents with prolonged BT & aPTT?
Answer: ? VWD

149. False positives may occur in patients with erythrocytosis, hyperglobulinemia, extreme leukocytosis
or hyperlipidemia. Coarse flocculation may occur in these samples due to elevated levels of total serum
protein. These patient samples may be washed in normal physiologic saline and centrifuged to minimize
these problems. 2. False positives or false negatives may occur in patients with severe anemia (lt15%
hematocrit). 4. False positives or false negatives may occur in patients with a recent blood transfusion.
150. Agranulocytosis
A. carmphicanol treatment

smle ,2016

278

151. Hemolytic anemia , comb positive , what type of hypersensitivity ?


answer: Type 2
(ANOTHER Q combs positive what's the Dx)
example of type II hypersensitivity is the ABO blood incompatibility where the red blood cells have different antigens, Wiki
152. Examination and evaluation show para-spinous Edema and fluid collection -ve brucella titer and
tuberculin test , what the cause ?
A. Brucellosis
B. breast cancer
C. recurrent hodgkin lymphoma
Answer : Recurrent Hodgkin's Lymphoma ?
(I tried searching for this question, but couldnt find anything proper to answer it.)
153. Maculapapular rash , plt low , blood smear showed fragmented RBC , sciocyte The antibodies was
targeting?
A. G glycoproteins Annex

answer :
154-Cause of low sickle solubility test :
A. anemia,
B. presence of hemoglobin c
Answer :

Pediatrics
smle ,2016

279

1- The most common parotid tumor in pediatrics?


Answer: ?
Pleomorphic adenoma then hemangioma.
Reference:http://www.ncbi.nlm.nih.gov/pubmed/10680869
*Mucoepidermoid if they asked about the most common malignant tumor.
2- Which of the following is the most common heart abnormality to get infective endocarditis?
a. Tetralogy of fallot
Answer: A
In children, cyanotic heart disease is still the most common cause of endocarditis, and the risk does not
diminish
after
surgical
repair
as
prostheses
carry
their
own
risk.
Reference:http://www.ncbi.nlm.nih.gov/books/NBK2208/
Bacterial endocarditis can occur with many heart defects but is most common in aortic valve lesions, a
patent ductus arteriosus (unrepaired), tetralogy of Fallot, ventricular septal defects, coarctation of the
aorta, and mitral valve prolapse with mitral regurgitation.
Reference: Bacterial Endocarditis by Ernest G. Brookfield, M.D.
3- What is the most common congenital abnormally cause infective endocarditis ?
A.ASD
B.VSD

smle ,2016

280
C.Tetralogy of fallot
Answer: C (C because according to the references the cause of IE is unrepaired cyanotic congenital heart
disease
reference: nelson essential + webs)
4- Baby with greasy looking rash on face.
a.Seborrheic dermatitis
Answer: A
5- Child presented with petechiae and his platelets is 15, otherwise healthy. What will you do for him?
a. Splenectomy
b. IVIG
c. Observations
Answer:C
Observation vs. pharmacologic intervention highly debated; spontaneous recovery in >70% of cases
within 3 months.
Treatment with IVIg or Prednisone if mucosal or internal bleeding, platelets <10, or at risk of significant
bleeding.
Life-threatening bleed: additional platelet transfusion emergency splenectomy
Supportive: avoid contact sports and ASA/NSAIDs. Reference: Toronto Notes.
6- 4 weeks old boy with acute onset forceful non bilious vomiting after feeding. On abdominal examination: There is olive mass at epigastric area. What is the 1st investigation should you do?
a. PH monitoring
b. Abdominal US
Answer: B
(The diagnosis of choice is US and the most accurate test is an upper GI series)
Hypertrophic pyloric stenosis: When the vomiting persists, other clinical and biochemical findings may
occur such as dehydration, hypochloremic alkalosis and unconjugated jaundice. Late clinical manifestations include weight loss and visible gastric peristaltic activity with a palpable pyloric olive. Prompt US
diagnosis is important as these late findings make the infants suboptimal candidates for surgery. Reference:Illustrated
7- What is the most common site for mump?
a. Parotid
Answer: A
Reference:http://reference.medscape.com/article/966678-clinical
8- Typical case of Poststreptococcal glomerulonephritis (PSGN). What will you do to confirm diagnosis?
Answer:?
Occurs 1-3 wk following initial primary GAS infection of pharynx or skin.
Diagnosis is confirmed with elevated serum antibody titers against streptococcal antigens (ASOT, antiDNAseB), low serum complement (C3) Reference: Toronto Notes
If it Glomerular involvement occurred in less than a week after URTI its due to IgA Nephropathy
9- The most common chromosomal abnormality in a new infant:
a. Down syndrome (Trisomy 21)
Answer: A

smle ,2016

281
Most common abnormality of autosomal chromosomes.
Reference: Toronto Notes.
10- Child came with wheezing and cough, diagnosed to have asthma and his dr. prescribed beclomethasone space inhaler or nebulizer?? twice daily. you will be worried about:
a. Growth retardation
b. Extraocular problem
Answer: A
Reference: 3rd Edition UQU > Pediatrics > Q 58
NB. Corticosteroids inhalers can lead to oral thrush (yeast infection of the mouth).
11- Child presented with Asthma exacerbation. The patient did not respond to Beta agonist. What is
your next step?
a. Aminophylline
b. Systemic steroid
Answer: B
Reference: First aid USMLE step 2CK
12- Cystic fibrosis
A.
chromosome 7 long q arm
B.
chromosome 7 short p arm
C.
chromosome 7 short q arm
D.
chromosome 7 long p arm
Answer: A

about CF chromosome
Answer: chromosome 7 called CFTR *autosomal recessive, CFTR gene found on the long (q) arm of
chromosome 7 resulting in a dysfunctional chloride channel on the apical membrane of cells. Reference:
Toronto Notes
13- Child was having rhinorrhea and then developed episodes of cough followed by vomiting. Which of
these vaccination may prevent him from having this disease?
a. DTaP
Answer: A
14- A case of pertussis ,, How to diagnose of pertussis?
A.
Nasopharyngeal swab
Answer: A
Reference: Nelson essential

smle ,2016

282

15- Child is complaining of severe throbbing unilateral headache, aggravated by light. What is the most
likely diagnosis ?
a. Migraine
b. Cluster Headache
c. Stress Headache
Answer: A
16- Tetralogy of fallot findings ?
a. Ventricular Septal Defect (VSD) + Overriding of the aorta + Pulmonary Stenosis + Right Ventricular
Hypertrophy (RVH)
Answer: A
Tetralogy of Fallot (TOF) is a cyanotic congenital heart disorder that encompasses four anatomic features:
right ventricular hypertrophy, ventricular septal defect (VSD), overriding aorta, and right ventricular (RV)
outflow
obstruction.
(See
'Anatomy'
above.)
The pathophysiologic effects of TOF are largely dependent upon the degree of: RV outflow obstruction.
The clinical presentation :, intermittent hypercyanotic (tet) spells, crescendo-decrescendo harsh systolic
ejection
murmur,
and
a
single
second
heart
sound.
The diagnosis of TOF is typically made by echocardiography, which can usually delineate the location
and number of VSDs, the anatomy and severity of RV outflow tract obstruction, the coronary artery and
aortic arch anatomy, the presence of any associated anomalies, and the hemodynamic abnormalities associated
with
the
anatomical
defects.
Treatment is by surgical repair. This usually consists of complete intracardiac repair typically during the
neonatal or infant period. Occasionally, an aortopulmonary shunt is used palliatively before complete
repair.
The most common long-term complications of complete repair are progressive pulmonary regurgitation
and
RV
failure,
atrial
arrhythmias,
and
ventricular
arrhythmias.
References :
http://www.uptodate.com/contents/pathophysiology-clinical-features-and-diagnosis-of-tetralogy-offallot
http://bestpractice.bmj.com.ezp.uod.edu.sa/best-practice/monograph/701/highlights/summary.html
17- Mother brought her 2 years old child to the ER with history of upper respiratory tract infection for
the last 3 days with mild respiratory distress. This evening the child started to have hard barking cough
with respiratory distress. On examination: RR 40/min, associated with nasal flaring, suprasternal & intercostal recessions. What is the most likely diagnosis?
a. Viral Pneumonia
b. Bacterial Pneumonia
c. Bronchiolitis
d. Acute epiglottitis
e. Laryngotracheobronchitis (croup)
Answer: E
Reference: 3rd Edition UQU > Pediatrics > Q 27
18- A case of TOF. How does it appear on X-Ray and echo?
Answer:?

ECG: RAD, RVH

smle ,2016

283

CXR: boot shaped heart, decreased pulmonary vasculature, right aortic arch (in 20%)
Reference: Toronto Notes
19- Turner Features:
a. Thick skin neck
Answer: A
Short stature, low set mildly deformed ear, triangular face, flat nasal bridge, epicanthial fold, web neck
with or without cystic hygroma, shield like chest with wide internipple space, puffiness of hands & feet,
internal malformation mainly coarctation of aorta in heart, horseshoe kidney. Reference: Nelson.
20- Child known case of DM 1, lost his consciousness at school. The last insulin injection is unknown.
a. Take him to the hospital
b. IV ranger lactate
c. IM Glucagon
d. Insulin
Answer: C
Because the patient could have hypoglycemic attack so give him shot of glucagon to increase his blood
sugar.
Reference: http://www.healthofchildren.com/G-H/Hypoglycemia.html
21- A child with the history of repeated infections, failure to thrive and anemia. His older brother also
has same condition. what is the most likely diagnosis?
A.
Nutritional anemia
B.
Leukemia
C.
Lymphoma
D.
Haemoglobinopathy
Answer: D
Repeated infection, Failure to thrive, Anemia and Family history of same condition make Hemoglobinopathy the best answer. Reference :http://www.uptodate.com/contents/overview-of-the-clinical-manifestations-of-sickle-cell-disease?source=machineLearning&search=haemoglobinopathy&selectedTitle=7%7E150&sectionRank=1&anchor=H13#H19
22- A child was on clindamycin developed abdominal pain and watery diarrhea.
a.
Clostridium difficile
Answer: A
http://www.uptodate.com/contents/clostridium-difficile-infection-in-children-clinical-features-and-diagnosis
23- During delivery, when the doctor cut the umbilical cord, bleeding doesnt stop. Which of the following factors is deficient in this case ?
A.
Factor X
B.
Factor XI
C.
Factor XII
D.
Factor XIII
Answer: D

smle ,2016

284
The bleeding diathesis in inherited factor XIII (FXIII) deficiency is severe in most patients. Bleeding from
the stump of the umbilical cord within the first days to weeks of life is a characteristic sign.
24- When does girls get puberty as compared to boys?
A.
1 to 2 yrs before
B.
2 to 3 yrs before
C.
Same time when boys do
D.
After boys
Answer: A

Average age for girls: 10 and a half years old.

Average age for boys: 11 and a half to 12 years old.

25 6 year old child with fever , malaise, lymphadenopathy and hepatosplenomegaly. Shown is the
CBC and bone marrow aspiration slide. What is the most likely diagnosis?
WBC

30,000
(4,500-11.000)

Hgb

Platelet

50,000
(150,000-400,000)

Peripheral Blast Cells

25%

a. Sickle sequestration.
b. Leukemia.
c. Malaria.
d. Leishmaniasis.
Answer: B
The diagnosis of acute lymphoblastic leukemia (ALL) is made when at least 30% lymphoblast (FrenchAmerican-British [FAB] classification) or 20% lymphoblast (World Health Organization [WHO] classification) are present in the bone marrow and/or peripheral blood. In addition, slides should be stained with
myeloperoxidase (MPO) (or Sudan black) and terminal deoxynucleotidyl transferase (TdT), unless another
method is used, such as flow cytometry. Bone marrow samples should also be sent for flow cytometry
and cytogenetics. Approximately 15% of patients with ALL have a t(9;22) translocation (ie, Philadelphia
[Ph] chromosome).

smle ,2016

285
Reference: http://emedicine.medscape.com/article/207631-workup#c11
26- Child came with Rt abdominal pain , jaundice, palpable tender liver, Dx ?
a.
HAV
Answer: A
Hepatitis A virus (HAV) spread via the fecal-oral route. The majority of patients who acquire the illness
have
had
personal
contact
with
an
infected
person.
The incubation period for HAV is 15 to 50 days. HAV RNA can be detected in stools at least one week
before the onset of histological and biochemical evidence of hepatitis, and it can be detected for at least
33 days after the onset of disease. In neonates and younger children, HAV RNA can be detected in stools
for
several
months.
In
the
childcare
setting,
HAV
infection
in
children
is
typically
an
acute,
self-limited
illness
Symptomatic patients may present with abrupt-onset fever, abdominal pain, malaise, and jaundice. Common examination findings are hepatomegaly and clinical jaundice with marked elevation of serum transaminases (usually >1000 units/L). IgM anti-hepatitis A virus serology is the test of choice for diagnosis.
The diagnosis of acute HAV infection is made by the detection of anti-HAV IgM in a patient with the
typical clinical presentation. Serum IgM anti-HAV is the gold standard for the detection of acute illness.
This antibody is positive at the onset of symptoms, peaks during the acute or early convalescent phase of
the
disease,
and
remains
positive
for
approximately
four
to
six
months
Hepatitis A vaccine is part of the recommended childhood and adolescent immunization schedule in the
United States. It is recommended for all children at one year of age (ie, 12 to 23 months), and also for
specific high-risk groups, including international travelers and patients with chronic liver disease
Post-exposure prophylaxis for individuals with recent exposure to HAV may be accomplished with the
HAV
vaccine
or
immune
globulin.
HAV infection in children is usually a minor and self-limited infection requiring no specific therapy. The
usual supportive measures for fever and diarrhea may be undertaken. Patients rarely require hospitalization except for those who develop fulminant hepatic failure. Children with HAV-related hepatic failure are
candidates
for
liver
transplantation.
Reference: up to date
27- An adolescent boy came to the clinic with unilateral gynecomastia. No other complaints. Everything
was normal. How will you manage?
a.
Reassure
b.
Give hormonal therapy
c.
Breast US
Answer: A
Gynecomastia (transient development of breast tissue) is a common self-limited condition seen in 50%
of male during puberty (but any discharge from nipple or fixed mass should be investigated). Reference:
Toronto Notes.
28- Neonate born in home by a midwife presented with umbilical bleeding. What does he need?
A.
Vit K injection
Answer: A
Vitamin K is one of the routine management of the healthy newborn infant
Prophylactic vitamin K1 is given to newborns shortly after birth to prevent VKDB, previously referred
to
as
hemorrhagic
disease of the newborn. In our practice, a single intramuscular dose of 1 mg is administered,

smle ,2016

286
http://www.uptodate.com.ezp.uod.edu.sa/ contents/overview-of-the-routine-management-of-thehealthy-newborn-infant?source=search_result&search=neonate+K+injection&selectedTitle=2%7E150
29- Turner syndrome chromosomes?
A.
45 XO
Answer: A
Reference: Toronto note
30- Child with fever, malaise, LNs enlargement & mouth ulcers. What is the diagnosis?
A.
Herpes simplex virus infection type 1 (HSV 1)
Answer: A
Reference: Toronto Notes.
30- A boy with rickets (Picture was provided). What is the deficient vitamin?

a.
Vit D
Answer: A
Reference: Toronto notes
31- Which of the following diseases has Mendelian mode of inheritance?
a.
Alpha thalassemia
Answer: A
Autosomal recessive Mendilian inheritance example? Thalassemia*
32- Minimal probability of a baby getting beta thalassemia from carrier parents?
A:0% ,
B:25%,
C:50%,
D:75%,
E:100%
Answer:B
both
parents
are
carrier
:25%
Unaffected
50%
Carrier
25% affected
Explanation:
If both members of a couple are carriers (or heterozygotes) for this mutation, each of their offspring has
a 25% chance of being affected (Nelson Essentials of Pediatrics 7th Ed, 2015)

smle ,2016

287

33- A child presented with fever and coryza, then watery diarrhea.
A.
Adenovirus
B.
Rotavirus
Answer: A
Adenovirus: http://emedicine.medscape.com/article/211738-overview
Rotavirus: http://emedicine.medscape.com/article/803885-clinical
35- Child that throws a ball at you and draws a straight line and stacks few cubes on each other
(they didnt mention the number of cubes). What is the age?
A.
12 months
B.
14 months
C.
18 months
D.
24 months
Answer: D
NB. Drawing straight lines begins at the age of 2 years. Reference: illustrated textbook.

36- A child that can raise his head slightly when prone and smiles. He turns his head 180 degrees and
has head lag when you pull him to sit. How many old is he?
A.
4 weeks
B.
8 weeks
C.
12 weeks
D.
16 weeks
Answer: B

smle ,2016

288

37- Which of the following is most typically seen in 4 years old baby?
A.
Print name.
B.
Stand on one foot briefly.
C.
Copy triangle and square.
D.
Toilet trained.
Answer: D

38- Child can set without support, cruises around furniture, uses chair to stand, say dada, crawl stairs.
What is the age of this child ?
A.
8 months.
B.
10 months.
C.
12 months.
D.
15 months.
Answer: B

N.B the
nearest answer is 10 m (at 9m can sits , crawls , say dada & Mama)
39- Roll from prone to supine and vise versa + crab with only two fingers. What is the milestone.
Answer: the fact that baby can grasp objects with two fingers (the so called pincer grasp, a fine motor
milestone) means he/she is about 10 month old. Reference: Illustrated
40 -2Q about developmental milestone" 2 times baby can smile at which age :
a. 2 months
b. 4 months
c. 6 months
Answer: A
social Smile 2 months
41- A 2 years old child can run and play ball. Socially active but he refused to share his toys with other
children. What you will say to his parents?

smle ,2016

289
a.
Delay motor
b.
Social impairment
c.
Well developed and normal response
Answer: C
2 y child can run , play alone , and the cooperative playing start at 4y .
42- 5 months old infant his parents were not able to bring him for his 4 months vaccination. What will
you do?
a.
Arrange for 4 months vaccination
b.
Give missed vaccination and next appointment
c.
Give vaccine together during next appointment
Answer: B
Give missed vaccination (4 months) and arrange for the a next appointment (for the 6 months vaccination) Children who have received some of their vaccines shots and then fallen behind schedule can catch
up without having to start over. Reference: http://www.medicinenet.com/childhood_vaccination_schedule/page3.htm
43- A child who came for 6 month vaccination appointment, his family report he had an anaphylaxis
shock at 4 months vaccination: what vaccine to give and what not to give? Or do an allergy test first?
Or reassure and give them all?
Answer: Allergy test
Answer: http://www.uptodate.com/contents/allergic-reactions-to-vaccines
44- 14 years old boy comes to your clinic with swollen lips. He has similar episodes since 3 or 5 years.
Deficiency of which of the following caused his presentation?
a.
Factor D
b.
Anaphylatoxin inhibitor
c.
C1 esterase inhibitor
Answer: C
Hereditary angioedema is an autosomal dominant disease caused by low levels of the plasma proteins C1
inhibitor (C1-INH).
Reference: Medscape
45- Image of cells for a 2 years old with pancytopenia and something else. What is the most likely
diagnosis?
a.
Malaria
b.
leishmaniosis
c.
Leukemia
Answer: ? Depends on the image.

46- Mode of inheritance of neurofibromatosis.


Answer: Autosomal dominant
Explanation:
Neurofibromatosis type 1 (NF1) and type 2 (NF2) are neurocutaneous disorders inherited as autosomal
dominant genetic syndromes

smle ,2016

290
47- 10 years old girl presented with fatigability, diarrhea and glossitis. What is the diagnosis?
a.
Vitamin B12 deficiency
Answer: A
Reference: Textbook of Oral Medicine
Other source : manual of pediatric hematology and oncology
48- 6 years old boy presented with gingivitis, petechiae and rash. What is the diagnosis?
a.
Vitamin C deficiency
Answer: A
Explaination : Vit C deficncy cause impaired collagen synthesis symptoms occurs after 3 months of deficiency, which includes ecchymoses, bleeding gum, petechiae,coild hair,hyperkeratosis and impaired
wound healing. Its common is severely malnourished and alcohol abusers , Tx supplementation.
Source: uptodate : overview of water-soluble vitamins
49- 15 years old has DM with dehydration?
a.
DKA
Answer: A
Explanation: DKA is the 2nd most common presentation of type1 diabetes and it presents with polyuria,
vomiting ( which lead to dehydration ) other symptoms : abdominal pain drowsniss and lathergy and fruity
smelling
breath.
Source: uptodate : clinical features and diagnosis of diabetic ketoacidosis in children
50- Infant suffer from groin rash that spare folds?
Answer:?
Irritant diaper dermatitis: Shiny, red macules/patches, no flexural involvement. Reference: Toronto
Notes.
51- A child with rheumatic heart disease allergic to Penicillin. What prophylaxis should be given before
a procedure?
A.
IV amoxicillin
B.
IV vancomycin + IV gentamicin
C.
Oral vancomycin + gentamicin
D.
Oral amoxicillin
Answer: B
Most probable, the answer is B. Since Amoxicillin is type of penicillin and gentamicin generally not given
PO.
Reference: http://www.cps.ca/documents/position/infective-endorcarditis-guidelines
Patients with rheumatic heart disease and valve damage require a single dose of antibiotics 1 hour before
surgical and dental procedures to help prevent bacterial endocarditis. Patients who had rheumatic fever
without valve damage do not need endocarditis prophylaxis. Do not use penicillin, ampicillin, or amoxicillin for endocarditis prophylaxis in patients already receiving penicillin for secondary rheumatic fever
prophylaxis (relative resistance of PO streptococci to penicillin and aminopenicillins). Alternate drugs recommended by the American Heart Association for these patients include PO clindamycin (20 mg/kg in
children, 600 mg in adults) and PO azithromycin or clarithromycin (15 mg/kg in children, 500 mg in adults).
Reference: http://emedicine.medscape.com/article/891897-treatment
52- Whats the most common minimum side effect of DTP vaccine?
a.
low grade fever

smle ,2016

291
b.
erythema over the injection site
Answer:B
Explanation: mild local and systemic reactions are the most common adverse reaction for the vaccine and
it includes low grade fever and mild redness and tenderness at the site of injection , both of them are
correct
but
the
redness
is
the
minimal
side
effect
Source: uptodate : Diphtheria, tetanus, and pertussis immunization in infants and children 0 through 6
years of age Diphtheria, tetanus, and pertussis immunization in children 7 through 18 years of age
53- 7 years old boy developed Flu after receiving Flu vaccine. His father asked you about the reason.
How will you reply?
a.
Live attenuated vaccine has small risk of infection
Answer: A
Uptodate
:
seasonal
influenza
vaccination
in
children
In a study of the effectiveness of a school-based immunization program, students who received LAIV had
significantly higher rates of symptoms of influenza-like illness (other than wheezing) "
54- Boy presented with unilateral nasal obstruction and foul smelling. What is the diagnosis?
a.
Foreign body
Answer: A
Source: http://emedicine.medscape.com/article/763767-overview#a4
55- What is the inheritance mode of fanconi anaemia?
Answer: Autosomal recessive
Source
:
journal
of
http://jmg.bmj.com/content/40/1/1.full

medical

genetics

56- Patient diagnosed with congenital adrenal hyperplasia. What is the next step?
Answer:
IV hydrocortisone
Initial management is hydration , long term is Oral steroids
Source: uptodate :Treatment of classic congenital adrenal hyperplasia due to 21-hydroxylase deficiency
in infants and children
57- In Atrial septal defect (ASD) there will be:
A.
fixed S2 split
Answer: A
Reference: Master the boards
58- Child with thumb sign on lateral Xray. What is the diagnosis?
a.
Epiglottitis
Answer: A.
Reference:uptodate
59- DTaP vaccine is against what?
a.
Whooping cough, tetanus, diphtheria
b.
Pertussis, tetanus, measles
c.
Rubella, tetanus diphtheria
d.
Whooping cough, rubella, diphtheria
Answer: A

smle ,2016

292
60- Bilateral parotid swelling.
Answer:?
Reference: http://www.ncbi.nlm.nih.gov/pmc/articles/PMC3528147/
local disease

mumps - more commonly children


than adults
parotitis
uveoparotid fever
sialectasis - especially if related to eating
Sjogren's syndrome
tumour infiltration

systemic disease

sarcoidosis
tuberculosis
alcoholism
myxoedema
cushing's disease
diabetes/insulin resistance - about 25% of patients with overt or latent diabetes have bilateral asymptomatic enlargement of the parotid
glands
liver cirrhosis
gout
bulimia nervosa
HIV in children may cause bilateral parotid enlargement

61- Child presented with recurrent nasal congestion, rhinorrhea, sneezing, tearing eyes,..What is the
diagnosis?
A.
allergic rhinitis
Answer: A
Allergic rhinitis is characterized by paroxysms of sneezing, rhinorrhea, nasal obstruction, postnasal drainage, and itching of the eyes, nose, and palate with a pattern of allergic triggers. More insidious effects of
the disorder include fatigue, irritability, reduced performance at school and work, and depression
Definitive diagnosis would require specific IgE reactivity during skin-prick or in vitro testing, but a therapeutic trial may be ordered on the basis of a presumptive clinical diagnosis.
Treatment consists of allergen avoidance where possible and pharmacotherapy (antihistamines, corticosteroids,
cromoglicate,
decongestants,
leukotriene
receptor
antagonists).
Intranasal corticosteroids remain the single most effective class of medications for treating allergic rhinitis.
References:
http://www.uptodate.com/contents/allergic-rhinitis-clinical-manifestations-epidemiology-and-diagnosis
http://bestpractice.bmj.com.ezp.uod.edu.sa/best-practice/monograph/232/highlights/summary.html
62- What is the causative organism of croup or typical symptoms of croup (laryngotracheobronchitis)?
A.Parainfluenza virus
Answer: A
Croup is usually caused by viruses. Bacterial infection may occur secondarily. Parainfluenza virus type 1
is the most common cause of croup; other causes include respiratory syncytial virus and influenza virus.
Croup most commonly occurs in children 6 to 36 months of age. Most cases occur in the fall or early
winter.

smle ,2016

293
Treatment :Orally administered corticosteroids are the mainstay for all levels of severity, combined with
nebulised epinephrine (adrenaline) in moderate to severe croup to provide temporary relief of the symptoms
of
upper-airway
obstruction.
References:
http://www.uptodate.com/contents/croup-clinical-features-evaluation-and-diagnosis?source=search_result&search=croup%2C+clinical+features&selectedTitle=1~51
http://bestpractice.bmj.com.ezp.uod.edu.sa/best-practice/monograph/681/highlights/summary.html
64-child with croup what is the best initial investigation to diagnosis?
a-c x-ray
b-pharyngeal swab
c-?
d-?
Answer: x-ray
Explination
:
see
the
reference
References:
http://www.uptodate.com/contents/croup-clinical-features-evaluation-and-diagnosis?source=search_result&search=croup%2C+clinical+features&selectedTitle=1~51
65- Rheumatic fever prophylaxis?
a.
IM penicillin monthly
Answer: A
An injection of 0.6-1.2 million units of benzathine penicillin G intramuscularly every 4 weeks is the recommended regimen for secondary prophylaxis for most US patients. Administer the same dosage every
3 weeks in areas where rheumatic fever is endemic, in patients with residual carditis, and in high-risk
patients.
References: http://emedicine.medscape.com/article/891897-treatment#showall
66- 11 years old patient with rheumatic fever and cardiac involvement. For how long he will require
prophylaxis?
a.
5 years
b.
6 years
c.
10 years
d.
15 years
Answer: D
Reference:uptodate
67- Which of the following congenital heart disease is the least associated with infective
endocarditis?
a.
ASD
b.
VSD
c.
PDA
d.
Pulmonary stenosis
Answer: A
Similar question: http://gradestack.com/Dr-Bhatia-Medical/Infective-endocarditis-is/0-30423177-15646-sf
Valvular aortic stenosis 13.3 percent
Coarctation of the aorta 3.5 percent
Primum atrial septal defect 2.8 percent

smle ,2016

294
Ventricular septal defect (VSD) 2.7 percent
Tetralogy of Fallot (TOF) 1.7 percent
No child with secundum atrial septal defect, patent ductus arteriosus (PDA), or pulmonic
stenosis had IE after surgery.
68- 8 years old boy with petechiae all over his body. Lab results: low platelets and high creatinine
level. what is the diagnosis?
A.
ITP
B.
TTP
Answer: TTP?
HUS if no fever or altered mental status
HUS = Microangiopathic hemolytic anemia + thrombocytopenia + renal failure
TTP= HUS + fever and/ or altered mental status.
69- A boy came to your clinic with yellow discoloration of the eyes noticed 3 days back and hepatomegaly. His liver enzymes are increased. What is the diagnosis?
A.
Hepatitis A
B.
Hepatitis B
C.
Hepatitis C
D.
Hepatitis D
Answer: A
References: http://www.uptodate.com/contents/overview-of-hepatitis-a-virus-infection-in-children
70- What can increase fetal hemoglobin in sickle cell anemi (no hydroxyurea in the options)
A.
Folic acid
Answer: ?
5-azacytidine, butyric acid, erythropoietin and hydroxyurea

Reference: Advances in Pediatric


71- (long scenario) baby with cavernous hemangioma and have pleural effusion. ( he have other
findings ). What you will find in this baby?
A.
Pulmonary hemangioma.
Answer:? - References:
http://www.uptodate.com/contents/tufted-angioma-kaposiform-hemangioendothelioma-and-thekasabach-merritt-phenomenon?
http://bestpractice.bmj.com.ezp.uod.edu.sa/best-practice/monograph/1041/diagnosis/differential.html
http://emedicine.medscape.com/article/1218120-clinical?src=refgatesrc1#b4
72- A child is always using abnormal sitting habits (W-Setting), what is the effect on the bones?
Answer:
Internal Femoral Torsion (Femoral Anteversion), it's the most common cause of intoeing > 2 years of
age.

smle ,2016

295
Management: Observation, takes 1-3 years to resolve. Surgery only if significant at > 10 years of age.
Reference: Pediatrics Kaplan for Step 2.
73- What is the Triple Antitoxoid?
A.
Tetanus, Diphtheria, Whooping cough
B.
Tetanus, Diphtheria, TB
C.
Diphtheria, Pertussis, Colorectal CA
D.
Diphtheria, Tetanus, Rabies.
Answer: A
References
http://www.who.int/vaccine_safety/initiative/tools/DTP_vaccine_rates_information_sheet.pdf
74- 2 weeks neonate passed unformed stool. What will you do?
A.
Prescribe formula milk.
B.
Give oral rehydration solution.
C.
Prescribe Lactose-free milk.
Answer:B
References http://www.babycenter.ca/a82/diarrhea-in-babies
75- (long scenario) child have 1mm defect in muscular atrial septum. What you will do?
A.
Surgical repair.
B.
Catheter repair.
C.
Reduce after load.
D.
Watchful waiting.
Answer: D
80-100% spontaneous closure rate if ASD diameter <8 mm.
Reference: Toronto Notes.
76- 7 month baby with you discover that the baby has VSD and Asymptomatic otherwise healthy what are you going to do?
A.
Close observation
B.
Surgery
C.
F/u after 6 month
Answer: C: f/u after 6 months
- No intervention is usually required for patients with small defects. These patients are typically asymptomatic and have a reasonable expectation of spontaneous closure or decrease in the size of the defect
over time.
-Patients who continue to have a murmur, but are otherwise asymptomatic and growing well at the 8- to
10-week visit, are seen again by the pediatric cardiologist at approximately 12 months of age.
-If the murmur persists at the 12-month and the patient remains asymptomatic and clinically stable, no
further intervention is required. Echo follow-up is typically performed at three years of age for patients
with membranous defects. In those with a muscular defect, no echo is required if the patient remains
asymptomatic.

smle ,2016

296
-Asymptomatic patients with residual small defects are usually followed every two to five years for overall
assessment. http://www.uptodate.com/contents/management-of-isolated-ventricular-septal-defectsin-infants-and-children

77- A baby 8 month old breastfeed for 6 month normally. He devolved vomiting and jaundice after
fruit juice. What component in the juice the baby is allergic to?
A.
Glucose
B.
Fructose
C.
Sucrose
D.
galactose
E.
phenylalanine
Answer: B
Hereditary fructose intolerance due to deficiency of Fructose-1,6-biphosphate aldolase B in the Liver.
Reference: http://reference.medscape.com/article/944548-overview
Hereditary galactosemia is among the most common carbohydrate metabolism disorders and can be a
life-threatening illness during the newborn period.[1, 2, 3] First described in a variant patient in 1935 by
Mason and Turner, galactose-1-phosphate uridyltransferase (GALT) deficiency is the most common enzyme deficiency that causes hypergalactosemia.[4] Removing lactose largely eliminates the toxicity associated with newborn disease, but long-term complications routinely occur.
Reference: http://emedicine.medscape.com/article/944069-overview
78- What is the most common cause of acute bronchiolitis?
A.
Respiratory Syncytial virus (RSV)
B.
Adenovirus
C.
parainfluenza
D.
Mycoplasma pneumonia.
Answer: A. Reference: FA for the USMLE step 2 CK Bronchiolitis is typically caused by a virus. Respiratory
syncytial virus (RSV) is the most common cause. http://www.uptodate.com/contents/bronchiolitis-andrsv-in-infants-and-children-beyond-the-basics
79. To which part of body it can go ? (continuation of the previous question )
A.
Spleen.
B.
Bladder.
C.
kidney.
D.
Liver.
Answer: D
RSV may be recovered from extrapulmonary tissues, such as liver, cerebrospinal fluid, or pericardial fluid
Reference: uptodate
80- What is the gene responsible for neurofibromatosis?
Answer: ?

smle ,2016

297
The NF1 (peripheral neurofibromatosis or von Recklinghausen disease) gene is cytogenetically located on the long (q) arm of chromosome 17, band q11.2 (17q11.2). The NF1 gene encodes for a cytoplasmic protein called neurofibromin 1, which is a ras -GTPaseactivating protein that suppresses tumor
growth, primarily by inhibiting ras activity.
The gene for NF2 (bilateral acoustic neurofibromatosis or central neurofibromatosis) is cytogenetically located on the long (q) arm of chromosome 22, band q12.2 (22q12.2). The NF2 gene codes for the
protein neurofibromin 2, also called merlin or schwannomin. Reference: Medscape.
81- NF1 gene responsible for?
A. Nureofibromatosis
Answer:A
Nf1 Gene located on chromosome 17q11.2
And coding for a tumor suppressor gene
Responsible for Neurofibromatosis type 1
Reference : ESSENTIAL NELSON OF PEDIATRICS
82- A child came to you with Caf au lait spots in face and neck. Which of the following features can
strengthen your diagnosis?
A.
Port-wine stain.
B.
Axillary freckling.
Answer: B
Clinical diagnosis requires the presence of at least 2 of 7 criteria to confirm the presence of NF1. The 7
clinical criteria used to diagnose NF1 are as follows, in the absence of alternative diagnoses:

Six or more caf-au-lait spots or hyperpigmented macules =5 mm in diameter in prepubertal children and 15 mm postpubertal

Axillary or inguinal freckles (>2 freckles)

Two or more typical neurofibromas or one plexiform neurofibroma

Optic nerve glioma

Two or more iris hamartomas (Lisch nodules), often identified only through slit-lamp examination
by an ophthalmologist

Sphenoid dysplasia or typical long-bone abnormalities such as pseudarthrosis

First-degree relative (eg, mother, father, sister, brother) with NF1


Reference:http://emedicine.medscape.com/article/1177266-overview
NF2: meningioma, schwannoma, glioma, neurofibroma, posterior subcapsular lenticular opacities accompanied by external signs: hearing loss, ringing in the ears, and balance problems associated with vestibular
nerve lesions, visual deficits and cranial nerve palsies. Reference: Medscape.
83- Breastfeeding after delivery should start:
A.
Immediately
B.
6 hr
C.
8hr
D.
24 hr
Answer: A
Reference:https://www.betterhealth.vic.gov.au/health/healthyliving/breastfeeding-when-to-start
84- (long scenario) Child brought by his father in wheelchair complaining of knee swelling and history
of falling on his knee. What is the best investigation ?

smle ,2016

298
A.
Joint Aspiration.
B.
X-ray.
Answer: B
A plain radiograph of the affected joint should be performed to rule out fractures, periostitis, avascular
necrosis, bone tumors, and bone dysplasias. Reference: Uptodate
85- A case of ambiguous genitalia. Which hormone you would like to check?
Answer: 17-hydroxyprogesterone (Hormone) should be measured promptly in all infants with non palpable gonads presenting with genital ambiguity to exclude congenital adrenal hyperplasia (CAH) due
to 21-hydroxylase deficiency. This is the most common cause of genital ambiguity and can lead to lifethreatening adrenal insufficiency within the first weeks of life. Reference: Uptodate
86. 26 years old female G1P1 brought her two weeks old baby who cries a lot. He is on breastfeeding
since birth, he stop crying at night when she gave him formula milk. On examination the baby looks
normal except for increase gurgle sound. What is the most likely diagnosis?
A.
Paralytic ileus.
B.
Lactose malabsorption.
C.
Increase bowel gases.
D.
Breast milk jaundice.
Answer: B or C
http://www.webmd.com/digestive-disorders/tc/lactose-intolerance-symptoms https://www.breastfeeding.asn.au/bfinfo/lactose.html
87- 5 years old girl with uncomplicated cystitis. what is the management?
A.
Oral amoxicillin
B.
IV cephalosporin
C.
IM ceftriaxone
D.
Sodium
Answer: A
This is an oral therapy for infection with susceptible organisms. Amoxicillin inhibits bacterial cell-wall synthesis by binding to penicillin- binding proteins. The addition of clavulanate inhibits beta-lactamase producing
bacteria.
88- Pediatric patient with classical symptoms of Diabetes + elevated blood glucose . What will you do
next?
A.
Urine dipstick
B.
Genetic testing
C.
HbA1c
Answer: A
Urine dipstick to pick up DKA early and to prevent further complications.
Reference: http://guidelines.diabetes.ca/browse/Chapter34
89- Case of croup ,what is symptoms associated :
A- Cyanosis

smle ,2016

299
B- wheezing
C- dysphonia
Answer: A. cyanosis
90- pediatric patient come with barking cough what is the best dignosis ?
Answer: chest x ray ---> show steeple sign in croup patient
91- Child presented with bronchiolitis. What is your management?
A.
Give Oxygen
Answer: A
92- A child presented with Croup. What to give in ER?
A.
epinephrine
Answer: A
- Refer to the table at the end of Pediatrics section
93- Epiglottitis case.
A.
intubate
Answer: A
Refer to the table at the end of Pediatrics section
94- Case of drooling of saliva?
Answer: Epiglotitis
Causes

Nausea
Foods
Emotional

Central
nervous

Drugs

Familial

of

drooling
Developmental
Teething

Physiological

system
Mental
Oropharyngeal
Esophageal
Gastroesophageal

and

muscular

and
dysautonomia
Wilson
Rett

(Riley-Day

stimuli
disorders
retardation
lesions
lesions
reflux
chemicals
syndrome)
disease
syndrome

95- Pediatric case of bronchiectasis what is the most important ?


Answer: Physiotherapy or home oxygen or steroid
96- 7 years old girl was brought by her mother, she developed pubic hair and her height 70th percentile
and weight 50th percentile. On examination there are no signs of puberty except pubic hair. Abdominal,
chest, cardiac and renal examination were normal. What is the most likely diagnosis?
A.
Congenital Adrenal hyperplasia
B.
45X (Turner syndrome).
C.
Premature Adrenarche.

smle ,2016

300
D.
Normal puberty.
Answer: C
Premature adrenarche is when these changes begin early, before age 8 for girls and age 9 for boys.
Reference: http://www.medscape.com/viewarticle/759350_3
97- Malnourished baby with fair coiled hair and abdominal distension. What is the most likely diagnosis?
A.
Kwashiorkor
B.
Marasmus
Answer: A
- See the table below for more information:

98 -treatment of choice for kawasaki?


Answer: Guidelines by the American Heart Association (AHA) and the American Academy of Pediatrics
(AAP) are available for the treatment of patients who fulfill the diagnostic criteria for Kawasaki disease
(KD) and for those who do not
the recommended initial therapy includes intravenous immune globulin (IVIG; 2 g/kg) administered as a
single infusion over 8 to 12 hours and aspirin (initial dose of 30 to 50 mg/kg daily divided into four doses).
IVIG : to reduce risk of coronary artery aneurysm
Aspirin : reduce risk of thrombosis
at risk of coronary arteries aneurysm within the first week of illness in about one third of affected children
http://www.uptodate.com/contents/kawasaki-disease-initial-treatment-and-prognosis?source=search_result&search=kawasaki&selectedTitle=2%7E150
99- Pediatric blunt trauma with duodenal coiled spring sign. what will you do?
Answer: Duodenal hematoma:
important clues to the diagnosis include the history of upper abdominal trauma, persistent vomiting, and
air-fluid levels in both the stomach and duodenum (the so called double bubble) in a plain erect abdominal x ray film. If doubt still exists, contrast study of the upper gastrointestinal tract will show partial
to complete obstruction of the duodenum, with a stacked coins or coiled spring appearance caused
by oedematous mucosal folds proximal and distal to the point of maximum obstruction
In the pediatric age group, duodenal injury from a blunt abdominal trauma resulting in an intramural
hematoma is rare. This case illustrates the characteristic delayed presentation of a duodenal hematoma
following a blunt injury from a motor vehicle accident and the associatedsequelae of delayed pancreatitis.

smle ,2016

301
Total parenteral nutrition and gastric decompression provide an effective conservative treatment of the
gastric outlet obstruction associated with this injury. Surgical intervention is only reserved for those patients who continue to show the clinical and radiological signs of complete high obstruction despite conservative
management
for
three
weeks.
- For more details, See: http://www.ncbi.nlm.nih.gov/pmc/articles/PMC3088392/
100- Baby with tachypnea, cough, hemoptysis and bilateral lung infiltrates. what is the treatment ?
A.
Steroid
B.
Antibiotic
C.
Surgery
Answer: B
Reference: Hemoptysis in Children; See: http://medind.nic.in/ibv/t10/i3/ibvt10i3p245.pdf
101- Child with meningitis what to do immediately?
A.
Antibiotics
B.
Lumbar puncture

Answer: A
Reference: Toronto notes.
102- Child sucking his thumb what to do?
Answer: The American Dental Association says most children can safely suck their thumb without damaging the alignment of their teeth or jaws until their permanent teeth begin to appear. (Permanent
teeth don't usually start to erupt until around age 6.)
103- Which of the following is a cyanotic heart disease in children?
A.
ASD
B.
VSD
C.
Tetralogy of fallot
Answer: C
Reference: Illustrated textbook of paediatrics
104- A child with developmental delay and constipation. Labs: low Na , low K and low ..?? what is the
most likely diagnosis ?
A.
CAH
B.
Congenital chloride diarrhea
C.
*other choices include syndromes
Answer:?

smle ,2016

302
Bartter syndrome?
Reference: http://emedicine.medscape.com/article/238670-overview#showall
105- A child with normochromic normocytic anemia + splenomegaly. Blood smear was attached showing clear spherocytosis. which of the following will be abnormal?
A.
plt
B.
retic
C.
wbc
D.
mcv
Answer: B
106- What is the most common intra-abdominal tumor in children?
A.
hepatoma
B.
rhabdomyosarcoma
C.
ewing tumor
D.
Wilms tumor
Answer: D
Reference: http://www.ncbi.nlm.nih.gov/pubmed/15321038
107- What is the single most important risk factor for cerebral palsy?
A.
prematurity
B.
birth weight less than 1.5 kg
C.
prenatal asphyxia
D.
genetic mutations.
Answer: A
Prematurity is the single most important risk factor for cerebral palsy
- Reference: http://emedicine.medscape.com/article/238670-overview#showall
108- A child with flat buttocks. What investigation will you do (celiac)?
A.
Antibody
B.
Intestine biopsy
Answer: A
Antibody testing, especially immunoglobulin A anti-tissue transglutaminase antibody (IgA TTG), is the
best first test, although biopsies are needed for confirmation; in children younger than 2 years, the IgA
TTG test should be combined with testing for IgG-deamidated gliadin peptides.
Reference: http://emedicine.medscape.com/article/171805-workup

Set of tables for explantation:

smle ,2016

303

Reference: Toronto Notes

smle ,2016

304

Vaccinations

Reference: Ministry of Health

smle ,2016

305

Comparison between Croup, Tracheitis and Epiglottitis:

Reference: Toronto Notes


109- child with bad smell and tooth is good :::: tonsillitis with crept
Answer: Tonsilloliths
Tonsils The tonsils may be involved in the pathogenesis of bad breath in a small percentage of cases
(perhaps 3 percent). Tonsillectomy based solely upon a complaint of bad breath should be avoided.
Some patients complain of small stones on their tongue or tonsils when they cough that have a foul odor
(and often lead patients to assume that they must have terrible breath). These stones are "tonsilloliths"
that form in crypts of the tonsils.
Reference : uptodate
110 - child with chronic diarrhea , endoscopy showed sickle shaped parasite adherent to the bowl wall
, what is it?
A.
Giardia
B.
Entemebea histolytica
Answer: A

smle ,2016

306
Biopsy specimens from duodenum are often teeming with sickle-shaped Giardia trophozoites,which are
tightly bound by the concave attachment disc to the villus surface of the intestinal epithelial cells
-Reference : medscape
111 -scenario a child didn't take HBV VACCINE what u will give him
Answer: If the vaccine is not given after birth the baby may start the course of hepatitis B vaccines (in
combination with other childhood vaccines) beginning at six weeks, then at four and six months of age.
112 -2Q about coarctation of Aorta (straight forward)
Coarctation of aorta: radio femoral delay
Baby with Deference in the Bp in upper and lower Extremities
Answer: A clinical diagnosis of coarctation of Aorta is made if there is an absent or delayed femoral pulse
(when compared with the brachial pulse). if theres a murmur may be associated with other cardiac
defects, such as PDA, aortic stenosis, or ventricular septal defect (VSD).
http://www.uptodate.com/contents/clinical-manifestations-and-diagnosis-of-coarctation-of-theaorta?source=search_result&search=coarctation+of+Aorta&selectedTitle=1~126
113 -Formula milk comparing to Breast milk contain more ..what
Answer: It contains more protein 1.5-1.9 g , Carbohydrate 7-8.6 g , sodium 0.65-1.1 mmol , Calcium 0.882.1 mmol , phosphorus 0.9-1.8 mmol and Iron 8-12.5 umol
Reference: illustrated textbook of pediatrics page 206 - Table 12.2
114 -breast feeding mother known history of seizure with phyntoin ask about breast feed ?
A.
Reassurance
B.
feeding after 8 hrs
Answer: A. Reassurance
Breast-Feeding Considerations: Phenytoin is excreted in breast milk; however, the amount to which the
infant is exposed is considered small. The manufacturers of phenytoin do not recommend breast-feeding
during therapy. phenytoin, carbamazepine and valproate are probably safe http://www.uptodate.com/contents/phenytoin-drug-information?source=see_link&utdPopup=true
115 -child with jaundice, elevated direct bilirubin
Answer (Two notes 1 and 2):
1- toronto notes P72 (check the picture below)

smle ,2016

307

2- http://www.pathophys.org/neonatal-hyperbilirubinemia/
116 -Case of gonorrhea eye infection in a newborn.. What to give?
Answer: gonorrhea eye infection= copious, purulent discharge, swelling of the lids and conjunctivae, corneal involvement common (risk for perforation). Appear one to two week afterbirth uni or bilateral = drug
of choice IV Ceftriaxone 25-50 mg/kg not exceed 125 mg.
Erythromycin (0.5 percent) ophthalmic ointment
Tetracycline (1 percent) ophthalmic ointment; tetracycline ophthalmic ointment is not available in the
United States
http://www.uptodate.com/contents/gonococcal-infection-in-the-newborn
117 -2 month boy present with 2 cm of hemangioma in the back .. Wt is the ttt?
A.
close f/u
B.
excision
C.
bblocker
Answer: A
The vast majority of infantile hemangiomas do not require any medical or surgical intervention.] Treatment options for clinically significant hemangiomas include the following:

Laser surgery

Surgical excision

Medication
http://emedicine.medscape.com/article/1083849-overview

Confirmed by nelson p727

118 -mother came to you that her child compline of spells for seconds

smle ,2016

308
Answer: absence (petit mal): usually only seen in children, unresponsive for 5-10 s with arrest of activity,
staring, blinking or eye-rolling, no post-ictal confusion; 3 Hz spike and slow wave
activity on EEG
Seizure Mimics
Benign paroxysmal vertigo
Breath holding
Hypoglycemia
Narcolepsy
Night terror
Pseudoseizure
Syncope
TIA
Tic
Reference : toronto note and nelson p679
119 -20 days infant diagnosis as meningitis, his culture show gram negative bacilli. Which of following
could be the organism?
A.
hemophiles influenza
B.
E.coli
C.
neisseria meningitides
Answer: B. E.coli
Note: group B streptococci (GBS) are the most commonly identified causes of bacterial meningitis, implicated in roughly 50% of all cases. Escherichia coli accounts for another 20%. Thus, identification and treatment of maternal genitourinary infections is an important prevention strategy.[5] Listeria monocytogenes
is the third most common pathogen, accounting for 5-10% of cases; it is unique in that it exhibits transplacental transmission.[6]
- N . Meningitides it is gram negative diplococcus
- H . Influenza it is gram negative coccobacili
Reference : nelson p381
120 -young patient complains of scrotal pain, examination is normal, US normal , urine analysis show
pyuria . what is the next step :
A.
Refer to surgery
B.
Give him azithromycine and cefxime
Answer: B. Give him azithromycine and cefixime
This is a case of epididydimitis. This patient has epididymitis. In males 14-35 years of age, the most common causes are Neisseria gonorrhoeae and Chlamydia trachomatis. The recommended treatment in this
age group is ceftriaxone, 250 mg intramuscularly, and doxycycline, 100 mg twice daily for 10 days. A single
1-g dose of azithromycin may be substituted for doxycycline. In those under age 14 or over age 35, the
infection is usually caused by one of the common urinary tract pathogens, and levofloxacin, 500 mg once
daily for 10 days, would be the appropriate treatment.

smle ,2016

309
But testicular torsion cause severe pain without urinary symptoms diagnosis by Color Doppler ultrasonography will show a normal-appearing testis with decreased blood flow. And it need ergent surgical
intervention
121 -Patient with Ostium secundum atrial septal defect What you will see in his ECG
A- prolonged PR interval
B-left axis deviation
C- right axis deviation
Answer: C
Confirmed by nelson p539
An ECG demonstrates sinus rhythm, often with evidence of right atrial enlargement manifested by tall,
peaked P waves (usually best seen in leads II and V2) and prolongation of the PR interval. The QRS axis is
slightly directed to the right (+100), and the precordial leads reveal right ventricular enlargement of the
so-called volume overload type that is characterized by an rSR' pattern in leads V3 R and V1 with normal
T waves.
The QRS duration may be mildly prolonged because of right ventricular dilation. This mimics the finding
in right ventricular conduction delay.A significant proportion (20-40%) of children with secundum atrial
septal defect may not have abnormal ECG findings.[14] Uncommonly, a patient with a secundum atrial
septal defect may demonstrate a superior QRS axis with right ventricular enlargement, mimicking findings
observed in the ECG of a patient with an ostium primum atrial septal defect.
http://emedicine.medscape.com/article/890991-workup#c6

122 -best way to take urine sample:::


A.
midstream
B.
foley catheter
C.
bag
Answer: A. Midstream clean catch mid-stream urine sample means that you dont collect the first or last
part of urine that comes out. This reduces the risk of the sample being contaminated with bacteria from:

your hands

the skin around the urethra (tube that carries urine out of the body)

But
in
general
collection
of
urine
sample
depend
on
the
age.
Midstream is the best in toilet trained patient. Supra pubic bladder aspiration is the best in neonate and young children. Bag never used nowadays. Edited and Confirmed. Reference: Alhowasi
6th edition 15
123- Picture of growth chart all parameters was low what is the Dx?
A.
A-Genetic
B.
B- GH Def
Answer: depends on the chart

If the weight and height proportionally small >> Chromosomal genetic

If the weight fall more that the height FTT >> due to chronic illness , lack of intake

if height fall more than the weight short stature >> endocrine .
- Reference : uptodate

smle ,2016

310

124- pt with respiratory sx..between attack ..he looks apathetic .what u to investigate?
A.sputum culture
b.sweet chloride.
Answer: Not clear QQ!
125- Which of the following influenza vaccines is given intranasally?
Answer: Live attenuated influenza vaccine
http://www.cdc.gov/flu/about/qa/nasalspray.htm
126- Young patient swallowed pins, what are you going to do. ( serial x-rays)
Answer: The peak incidence of foreign body ingestion is between the ages of 6 months and 3 years. 2
Most ingested foreign bodies pass through the gastrointestinal tract without difficulty, especially once
they have reached the stomach. Children with foreign body ingestion typically do not require laboratory
testing. Laboratory studies may be indicated for workup of specific complications, such as potential infection Chest/abdominal radiography Most foreign bodies ingested by children are radiopaque (in contrast to inhalation, in which most are radiolucent).
http://adc.bmj.com/content/84/2/165.full
http://emedicine.medscape.com/article/801821-workup
127 - apgar score :
Answer:
Rapid scoring system that helps evaluate the need for neonatal resuscitation.
Each of 5 parameters:Appearance (blue/pale, pink trunk, all pink) ,Pulse (0, < 100, > 100),Grimace with
stimulation (0, grimace,grimace and cough), Activity (limp, some, active), Respiratory effort (0, irregular,
regular) is assigned a score of 02 at 1 and 5 minutes after birth.
n Scores of 810: Typically reflect good cardiopulmonary adaptation.
n Scores of 47: Indicate the possible need for resuscitation. Infants should
be observed, stimulated, and possibly given ventilatory support.
n Scores of 03: Indicate the need for immediate resuscitation.
Reference: first aid
128 -Pneumococcal conjugate 13 what type of vaccination ??
Answer:
Pneumococcal conjugate vaccine (called PCV13) is 23 Strain Polysaccharide that protects against 13 types
of pneumococcal bacteria.
PCV13 is routinely given to children at 2, 4, 6, and 1215 months of age. It is also recommended for
children and adults 2 to 64 years of age with certain health conditions, and for all adults 65 years of age
and older. Your doctor can give you details.
http://www.cdc.gov/vaccines/hcp/vis/vis-statements/pcv13.html
129- Q down syndrome have bilateral ventricular enlargement
Answer:

smle ,2016

311
Children with Down syndrome are at a much higher risk for congenital heart disease. As a comparison:
the incidence of congenital heart disease in the general population is 0.8 percent. The incidence of congenital heart disease in children with Down syndrome is between 40-60 percent.
Some heart defects can be left alone with careful monitoring while others require surgery to correct the
problem.
The following types of heart defects in children with Down syndrome are discussed below.
o
Atrioventricular Septal Defects (AVSDs) These are the most common in children with Down syndrome.
o
Ventricular Septal Defects (VSDs)
o
Atrial Septal Defects
o
Patent Ductus Arteriosus
o
Tetralogy of Fallot
http://downsyndrome.nacd.org/heart_disease.php
130. baby with recurrent infection tb, aspergillosis all type of infection with history of brothers
death at 3 year with same pt give?
A. Influenza
B. BCG
C. Varicella
D. Polio
Answer: A. IM INFLUENZA
This baby is immunodeficiency patient so he need annual influenza vaccine ..
See
the
link
for
farther
inf.
http://emedicine.medscape.com/article/973120-overview#a1
131- 2 year old complains of papule on the foot no itching pink pale not respond for antifungal?
A. Granuloma .

(Answer: not clear question and not found answer!!)

132- A 7 y child and an U & L respiratory tract infection since birth with generalized joint pain
aches his uncle and brother have the same condition labs show high creatinine and BUN and
+ve urine protein. repeated
A. idiopathic fibrosis
B. Autoimmune
Answer: B. Autoimmune (** With broad DDx like HSP , WG , SLE and also glomerulonephritis)
133- a child have came to take the 6 month vaccination the mother mention on 4th month
vaccination he developed an anaphylactic What to do now ? repeated with different answer!
A. Do allergen test
B. Give vaccine without DTP
C. Give vaccine with reassurance
D. Give vaccine and observe for 1h
Answer: B (First answer was A, and now B!, needs further checks)
Children who have severe responses should not be given further vaccination
especially
DTP.
http://reference.medscape.com/drug/infanrix-daptacel-diphtheria-tetanus-toxoids-acellular-pertussisvaccine-343172

smle ,2016

312

134- Nine month child .. Take all vaccine regularly what vaccine should be given ?
Answer: 9
months vaccines : Measles , Meningococcal conjugate quadrivalent (MCV4) Ref: according to saudi national vaccination schedule ->
135- Neonate with deafness , rash .. What the cause ?
A.
A.Rubella
B.
B.measles
C.
C.CMV
Answer: A
Rubella infection Manifestation:
1.Blueberry Muffin rash due to extramedullary hematopoiesis
2. cataracts
3. Salt and Pepper retinopathy
4. Radiolucent bone disease (long bones)
5. UGR, glaucoma, hearing loss, pulmonic stenosis, patent ductus arteriosus, lymphadenopathy, jaundice,
hepatosplenomegaly, thrombocytopenia, interstitial pneumonitis, diabetes mellitus
Reference: https://pedclerk.bsd.uchicago.edu/page/torch-infections
136- 6 month baby can't sit , hypotonia , crossed lower limb .. Which vaccine should be modified ?
A .change opv vaccine to ipv
Answer:
In
this
case
regarding the
hx
given the
Dx
is
cerebral palsy
So no need to modified the vaccine and there is no contraindications for vaccination in these group of
patient
http://www.phac-aspc.gc.ca/publicat/cig-gci/p02-02-eng.php
137- 12 yo , mild jaundice, splenomegaly , echogenic shadow of gall bladder
A.
SCA
B.
Thalassemia
C.
hereditary spherocytosis
Answer:C. hereditary spherocytosis
http://emedicine.medscape.com/article/206107-clinical

138- 15 kg child .. What is the daily water requirement ?


Answer: 15Kg = 1250 ml.
Source:http://faculty.olin.edu/~jcrisman/Service/KWTWebNews/Nutrition/fluid.htm
** Holliday Segar fluid requirement calculation:
1 to 10 kg: 100ml\kg
11 to 20 kg : 1000 ml + 50 ml\kg, for each Kg over 10kg.
over 20Kg: 1500 ml + 20 ml\kg for each kg over 20kg.
1Kg = 2.2lbs.
139- What is characterised about x fragile syndrome?
A. Obesity
B. Macrogonadisim
C. Smal
Answer: A & B are both correct (BUT B is more common, hence, a closer answer)

smle ,2016

313
Fragile X syndrome (FRAX)
1 in 200 in frequency.
Most common cause inherited mental retardation.
FEATURES :
A)Characteristic craniofacial finding (large head,prominent forehead, jaw,ear)
B)Characteristic neurobehavioral profile including (mental retardation, autism spectrum disorder, pervasive developmental disorder )
C)Macro-orchidism
D)Mild connective tissue disorder including (joint laxity,patulous eustachian tubes ,mitral valve prolapse)
Reference : ESSENTIAL NELSON OF PEDIATRICS
140-typical case of turner what another finding you find?
A.Cardiovascular abnormality .
Answer: A
141- Turner syndrome
FEATURES :
A) Characteristic facial appearance (low set mildly malformed ears,triangular face, flattened nasal bridge
epicanthal fold )
B) Webbing of the neck with or without cystic hygroma
C) shield like chest with widened internipple distance
D) Internal malformations may include congenital heart defect (coarctation of the aorta is most common
anomaly Followed by bicuspid aortic valve, post stenotic aortic dilation with aneurysm may develop )and
renal anomaly (horseshoe kidney )
E) short stature is cardinal feature
F) hypothyroidism
Reference : ESSENTIAL NELSON OF PEDIATRICS
142- Autosomal recessive disease Both parents are carrier and phenotype normal what the chance they
have a kid with a disease ?
A.25
Answer: A
Reference: Illsturated text book
143-What come with Turner syndrome ?
A.Hypothyroid
B.DM
C.Addison's
Answer: A
(reference is nelson essential of pediatric)
144- A baby 6 month show regurgitation after every meal he esophagus ph is low he is normally developing what is the Rx?
A.
Close follow up
B.
Surgical fundal
C.
Esophageal manometry
Answer: A

smle ,2016

314
(no treatment is required may be A because I think this is a case of normal physiological gastroesophageal
reflux which happen in baby younger than 8months & presented with effortless regurgitation but otherwise the baby is normal
Reference: nelson
145-Child ingested a caustic material he present to ER crying drooling what to do 1st ?
A.
Maintain airway
B.
Activated charcoal
Answer: A
I think because activated charcol is contraindicated in causatic material ingestion ,And Because of the risk
of rapidly developing airway edema, the patients airway and mental status should be immediately assessed
and
continually
monitored.
Reference: http://emedicine.medscape.com/article/813772-treatment#d10
146-A baby with bilateral renal agenesis: (oligohydramnios)
Answer: with bilateral renal agenesis: (oligohydramnios which is a sign for the disease during prenatal
diagnosis)
Reference: http://emedicine.medscape.com/article/983477-overview
147- A child alert, anterior fontanelle depressed, how much dehydration:
A.
5-9
B.
>9
Answer: A
Mild 5% : normal fontanelle
moderate 6-10% : Sunken slightly
severe >10 % : Sunken significantly
http://www.utmb.edu/pedi_ed/CORE/Fluids&Electyrolytes/page_09.htm
http://emedicine.medscape.com/article/801012-clinical#b3
148- A child with hepatosplenomegaly, current infection. Brother died at 3 years with septic shock.
How to give vaccination?
A.
Give all.
B.
Dont give until 3 years.
C.
Dont give live vaccines.
D.
Dont give killed vaccines.
Answer: C
149-patient can't take BCG vaccine Because he deficiency in
A.
IL
B.
TNF gama
C.
INF gama
Answer:C
reference: https://www.drugs.com/pro/bcg-vaccine.html
150-child ride tricycle can't copy square what is the age
Answer: 3 years

smle ,2016

315

151- child can know color but with difficulty in making square
A.2 y
B.3 y
C.4 y
D.5 y
Answer: C
reference: https://www.cdc.gov/ncbddd/actearly/pdf/parents_pdfs/milestonemomentseng508.pd
152- Baby wave his hand bye bye which developmental milstone reflect ???
abcAnswer: Social and emotional milestone, 12 months
reference: http://www.cdc.gov/ncbddd/actearly/milestones/milestones-1yr.html
153- what is the age of child should be know few word ?
A.6 month
B.8 month
C.12 month
D.24 month
Answer: 24month
154-child can support his head when sit and loving when stare to him or cooing
A. 4wk
B. 8wk
C.12month
Answer: B
155- Developmental milestones Q: 5 words, hop on one leg ?
A.48 m
B.36 m
C.24 m
Answer: A
Reference: http://www.clinicalexam.com/pda/peds_ref_developmental_milestones.htm#9_months
156-Milestone, baby pull him self to stand crawl without difficulty, which age ?
A. 8 months
B. 10 months
C.12 months
D.14months
Answer: B and to be more specific 9 months
Reference:
https://www.cdc.gov/ncbddd/actearly/pdf/parents_pdfs/milestonemomentseng508.pdf
157- ER Child can roll over, sit triploid, attempt to take object

smle ,2016

316
Which month?
A.
6
B.
9
C.
2
Answer is A 6 months

158- Child tells stories runs and plays father -role.. What's the age?
1234Answer: 5 y
reference: https://www.cdc.gov/ncbddd/actearly/pdf/parents_pdfs/milestonemomentseng508.pdf
159- Baby setting in mothers lap unsupported, when the doctor spoke the baby turned around and
laughed and babbled to the doctor, babys age?
A:2m
B:4m
C:6m
Answer: C
160- 18 months baby says baba mama what you will do for him
A.
Developmental assessment
B.
Bone age
ANSWER: A
161- Baby can walk when he held by one hand and good pincer grasp but he can not put things in the
bottle What is his age:
A.9m
B. 12 m (Answer: B)
C. 15 m
162- milestone baby can hold his head and when he looks at his flying hand he laughs and coos
Answer: 4m
163- milestone said baba and walk holding furniture and a lot of other features
A.
12M
B.
10M
Answer: B
164- adolescent male with swollen parotid and salivary gland with dry eye and dry mouth, labs HLA,
ANA and RF are positive which of the following is appropriate treatment?
A.
Physostigmine
B.
Artificial eye and saliva drops
Answer: B this is case of sjogrens syndrome
- up to date
http://www.uptodate.com/contents/treatment-of-dry-eye-in-sjogrens-syndrome?source=outline_link&view=text&anchor=H3377156#H3377156

smle ,2016

317

165- 9 month old baby cannot sit by himself he is fisting his hand and crossing his leg Most likely his
presentation of?
A.
Normal child
B.
CP
C.
Down syndrome
Answer:B
166- child with high fever 2 wk and abdominal distention and wt loss
A.bm
Answer: ???
167-Child with typical symptoms of epiglottitis. (Tripod position) Diagnosis?

168- 16 years old female . Fever and Chronic diarrhea for 10 months, Post meal periumbilical pain,
Sometimes blood mixed with stool?
A.
A.crohn
B.
B.chronic pancreatitis
Answer: A
169- baby was playing with his father which suddenly his father looks the watch is not working baby
become agitated and refuse food what you will do :
A.
upper Gi endo
Answer:A
)Battery
Ingestion)
http://www.emedicinehealth.com/script/main/mobileart-emh.asp?articlekey=58824&page=7
169-the most part in impaction of foreign body in the :
A.
left bronch
B.
right bronch

smle ,2016

318
C.
bifurcation
Answer: b
170- boy with glomerulonephritis after week he developed hemoptysis :
A.
heno choinlin purpra
B.
good pasture syndrome.
C.
rapid deterotion
Answer:
http://emedicine.medscape.com/article/240556-overview

171-Qs about osteosarcoma


Answer:
http://www.uptodate.com/contents/osteosarcoma-epidemiology-pathogenesis-clinicalpresentation-diagnosis-and-histology?source=search_result&search=osteosarcoma&selectedTitle=1~105
172- bacterial meningitis in 14 month child , Gram positive cocci, what is the management?
A.
amoxicillin
B.
amoxicillin and gentamicin
C.
ceftriaxone and vancomycin
D.
vancomycin
Answer: Ceftrixone and vancomycine C
Answer:
http://www.fpnotebook.com/mobile/neuro/ID/BctrlMngtsMngmnt.htm

173- 3 year old child with UTI admitted what investigation to be done ?
-US
-Cystoscope
Answer: US
http://www.uptodate.com/contents/urinary-tract-infections-in-infants-older-than-one-month-andyoung-children-acute-management-imaging-and-prognosis

Added Qs 7th update


174- child with fever, general swelling and dark colored urine which best evaluate this pt:
a- US
b-RFT
c-urine culture
d-urine specimen
Answer: D
this Q there is hematuria we will do Urine analysis to confirm hematuria and to look for microscopic
examination and also will give us a hint about the origin of hematuria ( is it glomerular or non glomerular
) then if it is glomerular hematuria requires more extensive evaluation (renal ultrasound; CBC; complement levels; antinuclear antibody (ANA), antineutrophilic cytoplasmic antibody (ANCA), and ASO titers;
hepatitis
B
screen;
and
possibly
renal
biopsy).
Reference
:
The
Washington
Manual
of
Pediatrics
..

smle ,2016

319
175- Child history of hip pain x-ray of hip shows effusion What is the next step of management?
a-Aspiration
b-Antibiotic
c-US
Answer:A
Reference
:
The
Washington
Manual
of
Pediatrics
http://eradiology.bidmc.harvard.edu/LearningLab/musculo/Easter.pdf

&

176- Case about neonate algorithm for resuscitation very long case i couldn't remember
Answer;
;
177- Mother and her child visited the pediatrician for 6 months vaccination, however the mother stated
that her child was hospitalized after receiving the 4 months vaccination he developed anaphylaxis ,
what
is
the
right
thing
to
do?
A-Test
the
child
for
which
antigen
is
he
allergic
from
B-Give
him
steroid/antihistamine
post
the
vaccination
C-Vaccinate
him
and
discharge
home
D-Vaccinate
him
and
hospitalize
the
child
for
1
hour
Answer
:
A
178Children
diagnosed
A-BP
Above
B-BP
Above
Answer
sBP and/or dBP>= 95th percentile for
(Toronto

to

be
90th
95th

hypertensive

:
sex , age
note

height

on

>=

:
percentile
percentile
B
3 occasions
)

179- What is the injection that is routinely given to new-born to inhibit haemorrhage:
A-Vitamin
K
B-Vitamin
C
C-Vitamin
D
D-Vitamin
E
Answer : A
(toronto note )
As the neonate's colonic flora has not adequately colonized, E.coli is not present in sufficient quantities
to make enough vitamin K to produce clotting factors II , VII , IX and X and protein C and S . without such
factors, the newborn is more likely to have bleeding from GI tract , belly button and urinary tract ..
To prevent Vit K deficient bleeding ( hemorrhagic disease of the newborn ) , a single intramuscular dose
of
vitamin
K
is
recommended
..
Reference : master the board +toronto note )
180- Child with URTI is complaining of bleeding from nose, gum and bruising the treatment is:
A-Prednisolone
B-IVIG
Answer
:
A

smle ,2016

320
Explanation: ITP (idiopathic thrombocytopenic purpura) can occur in anyone at almost any age, but these
factors increase your risk:
1. Women are about twice as likely to develop ITP as men are.
2. Recent viral infection. Many children with ITP develop the disorder after a viral illness, such as mumps,
measles or a respiratory infection.
- Common medications used to treat idiopathic thrombocytopenic purpura include:

Corticosteroids.The first line of therapy for ITP is a corticosteroid, usually prednisone, which can
help raise your platelet count by decreasing the activity of your immune system. Once your platelet count is back to a safe level, you can gradually discontinue taking the drug..

Intravenous immune globulin (IVIG). If you have critical bleeding or need to quickly increase your
blood count before surgery, you may receive medications, such as immune globulin, given intravenously.

Thrombopoietin receptor agonists. The newest medications approved to treat ITP are romiplostim (Nplate) and eltrombopag (Promacta). These drugs help your bone marrow produce more
platelets, which helps prevent bruising and bleeding. Possible side effects include headache, joint
or muscle pain, dizziness, nausea or vomiting, and an increased risk of blood clots.

Biologic therapy. Rituximab (Rituxan) helps reduce the immune system response. It's generally
used for people with severe ITP, and in those who corticosteroids don't help. Possible side effects
include low blood pressure, fever, sore throat and rash
Soure:http://www.mayoclinic.org/diseases-conditions/idiopathic-thrombocytopenic-purpura/basics/treatment/con-20034239
181- Cellulitis occurring about the face in young children (6-24 months) and associated with fever and
purple
skin
discoloration
is
MOST
often
caused
by
A- group A beta hemolytic streptococci
Reference: Toronto note

12 update

smle ,2016

321

182- 12 years old with myopia, pectus exavatum, congenital heart disease, Height > 90th percentile
weight <50th percentle, what's your diagnosis?
A- Marphan's Syndrome*
Answer
:
A
Reference : http://www.glogster.com/faithbreen18/marfan-syndrome/g-6lkpl0ukh0id80elm3hu1a0
183- 5 years old girl presenting with hepatospleenomegally, pale, decreased level of conscious, V\S
indicating shock
labs: Hb: 5 Platlets: 65
what's your important next investigation?
A- Abdomen US
B- Reticulcytes count* (patient has spleenic sequestration)
C- BM biopsy
Answer
:
B
Acute splenic sequestration crisis ASSC is characterized by a precipitous drop in hemoglobin concentration, tender splenomegaly, thrombocytopenia, and reticulocytosis . Effective circulating blood volume is
reduced, sometimes leading to hemodynamic instability. Fever, leukocytosis, splenic rupture, and death
may occur. Mortality has been estimated at 15%.We present a case of ASSC with sickle cell anemia who
suddenly developed tender splenomegaly, high fever, acute anemia, thrombocytopenia, leukocytosis,
jaundice, hypoxia, and tachycardia. He was thought to have sepsis, but this diagnosis was ruled out after
microbiologic
evaluation
and
invasive
testing.
Reference : http://www.medscape.com/viewarticle/475043

184- 5 years old, with recurrent infection, greasy diarrhea, slow growing, he's jaundiced with positive
sweat chloride test?
A- Cystic Fibrosis

185- 4 weeks old infant, mother happy he never cries. on examination: Jaundice + Umbilical hernia +
Distended Abdomen + Coarse face features + Bulging frontal fontanel. Diagnosis?
A- Congenetal Hypthyroidism*
B- Gilbert's Syndrome
C- Cerebral Palsy

smle ,2016

322
D- Rickets
Answer:A
Reference: http://emedicine.medscape.com/article/919758-overview
186- Recived antibiotics and went home . Now improving but The culture then was ....... N.meningitdis
What you will do ?
A.
rifambicin for 7 days
B.
one dose ceftriaxone IM ( correct ).
C.
Tell family to come to hospital
D.
.......
Nose
carrier
Answer:B
Antibiotics should be started as soon as possible. Ceftriaxone is one of the most commonly used antibiotics for meningococcal meningitis. Penicillin in high doses is almost always effective, too.
If the patient is allergic to penicillin, chloramphenicol may be used. Sometimes corticosteroids may be
used,
especially
in
children.
People in close contact with someone who has meningococcal meningitis should be given oral rifampcin
or
ciprofloxacin
is
an
alternative
drug
to
prevent
infection.
Reference:
https://www.nlm.nih.gov/medlineplus/ency/article/000608.htm
& Danish Book ..
187- New born with meningitis organism + cocci??

188pneumocyte
95% of the alveolar surface
http://www.ncbi.nlm.nih.gov/pubmed/3285521

type

features?

189- Child his teacher noticed decrease concentration with 15-10 secounds up ward staring of eyes .
His unaware of that what is treatment of choise ?
A.
carbamezabine
B.
ethosuxamide ( correct )
C.
...............
D.
lamotrigene
E.
Answer:B
Absence seizure

smle ,2016

323
http://www.uptodate.com/contents/childhood-absenceepilepsy?source=outline_link&view=text&anchor=H383785509#H383785509
190- 11 years old child obese with abdominal stria what is investigation you do :
A.
Adrenal CT
B.
morning & evening urine cortisol.
Answer:B
The case is most likely Cushing then, first we establish the presence of hypercortisolism
The best initial test is 24 hr urine cortisol(more specific). If this is not in the choices, then 1 mg overnight
dexamethasone
suppression
test
Master
the
boards:
USMLE
step
2
CK
3rd
edition
page
No.
133
191- Sickler PT with recurrent cholecystitis what is management:
A.
cholecystectomy
192- Case of SCA . What is the most important on long term ?
A.Penicilline
B.Hydroxyurea
C.
D.
Answer:A
Nelson
Essentials
of
Pediatrics
7th
Step-up to pediatrics page No.239

edition

page

No.

193- Guillain-Barr syndrome prognosis (it was a child I think) ?


1/from uptodate-- 90% of children are symptom-free or no disability despite residual symptoms
2/Time to full recovery varies from days to as long as several months
Step-up to pediatrics page No.143

519

194- Child came with rhinorrhea, cough, respiratory distress, which vaccine can prevent this disease?
The symptoms and sings are not sufficient to make dx, and to decide which vaccine we choose ,however
there
are
some
DD
:
*Croup : barking cough, croyza, inspiratory stridor, peripheral cyanosis, CXR shows steeple sing and commonly
caused
by
parainfluenza
(
no
vaccine
for
parainfluenza)
*Pertussis
:
caused
by
bordetella
pertussis.
Pt.
presents
with
:
catarrhal
stage:
congestion
+
rhinorrhea
(14days)
- Paroxysmal stage : whooping cough + inspiratory whooping followed by vomiting (14-30 days)
Convalescent
stage
:
decrease
of
coughing
(14days)
DTaP
vaccine
has
decreased
incidence
*Bronchiolitis
caused
by
RSV
Pr.
Present
with
wheezing
+
apnea
We can prevent such a disease by giving RSV specific monoclonal antibody(palivizumab) + all children
older than 6 months should take influenza vaccine to prevent influenza associated diseases
*Pneumonia
we
give
-Hib
and
s.pneumonia
vaccination
-influenza
vaccine
Palivizumab
&Nelson Essentials of Pediatrics 7th edition pages: 354-364

smle ,2016

324
195- Baby with delayed sitting, labs showed normal calcium, low PO4, what is the diagnosis ?
Vitamin D deficiency
Nelson

Essentials

of

Pediatrics

7th

edition

page

No.

603

196- GBS i think , Distal progresses paralysis upper and lower limb ,What will you find on CSF?
A.
increased
protien
B.
decreased
glucose
C. other
Answer:A
Lumber puncture in patients with GBS classically shows elevated protein and normal WBC count in CSF
called
albuminocytologic
dissociation
Step-up
to
pediatrics
page
No.143
197- Pathogenesis of jaundice in newborn From a mother has - blood group And the newborn has +o
http://emedicine.medscape.com/article/797150-overview#a5
198- Child with recurrent URTI with pseudomonas, and atypical organism. Whats the cause:
A.
CF
B.
low CD4
Answer:A cystic fibrosis
Illustrated textbook of pediatrics 4th edition page No.295
199- Pt with red points in palms and hands, buttocks, fever. Turned into vesicles
A.
herpes simplex (not sure about this question)
Answer:
herpes simplex virus if the presentation were (Sore Throat Dysuria Vaginal Discharge Vesicles and Bullae
Fever and Rash Lymphadenopathy), Primary illness lasts 10 to 20 days with recurrences in 50% to 80% of
patients.
Or may be it is henoch-schonlein purpura if there is hx. Abdominal pain with Melana or haematemesis,
ankle
&
knee
joint
pain
and
haematuria
Illustrated
textbook
of
pediatrics
4th
edition
page
No.339
Nelson
Essentials
of
Pediatrics
7th
edition
pages
No.
336-338
&
380
200- child wih oral and tonisllar ulcurs and vesicals, fever. Dx:
Herpangia
Answer:Herpangia
Illustrated textbook of pediatrics 4th edition page No.255
201- Which one of these disease likely to exhabit cynosis in later life ?
A.
6 year with coartcation of aorta
B.
VSD
C.
Trancus arteriosis
D.
ASD
I think all options are wrong b/c the VSD, ASD, and coarctation of aorta are Acynotic heart diseases. And
Symptoms
of
TA
occur
within
the
first
few
days
of
life
The
best
answer
is
TOF

Master the boards: USMLE step 2 CK 3rd edition pages No. 459-467

smle ,2016

325

202- Scale for questioning parents about development of their child


With only questioning them?
Developmental
milestones
Illustrated textbook of pediatrics 4th edition page No.32

203- To confirm down syndrome ?


choroinc villous sampling
Reference:Toronto note
204- Long case about child 2 months with respiratory symptoms his brother dies from pneumonia and
her sister doesnt any medical hX. Investigations. ShowT cell 0 low all IG Dx
Not clearQQ
205- x-linked a gamma globulinemia
A.
cd 19 and 20
B.
Cd40
Answer:A
http://emedicine.medscape.com/article/1050956-overview)

mutation

206- Baby 18 month old with Delay speech other exam normal
(hearing,development
Read from Toronto note p24
207- fallot tetralogy present with a. VSD
Read from Toronto note p19
208- Early sign of puberty in Male?
a) Hair in the face
b) Hair in the gentalia
c)Hoarseness of voice
d)Enlarged peins
Answer:D
Males

onset:
age
9-14
yr
old
usual
sequence
1.
testicular
enlargement
2.
penile
enlargement
3. pubarche: axillary and facial hair, body odour, mild acne 4. growth spurt: occurs later in boys

209- Child with episodic of Cyanotic lip with cold extremities , what is the diagnostic invistigation :
Echocardiography
Reference:Toronto note

smle ,2016

326
210- newborn baby complain of (many signs that indicate distress likes tachypnea) pt was unwell on
examination left side no breath sound and the heart sound heard in right side whats Dx
a-situs inversus
b-pneumothorax
c-bowl hernia
d-another not related answer
Answer:C.
infants with CDH most often present with respiratory distress in the first few hours or days of life. Physical
findings include a barrel-shaped chest, a scaphoid-appearing abdomen (because of loss of the abdominal
contents into the chest), and absence of breath sounds on the ipsilateral side. In patients with a left-sided
CDH, the heartbeat is displaced to the right because of a shift in the mediastinum.(uptodate
211- read about diseases comes with turner syndrome
212- child with barking cough and another sign indicate respiratory infection what cause of this pt
NO croups or laryngotracheitis in answers
A.
pertussis
B.
epiglottitis
213- Baby one month with total bilirubin 200 and direct 80. What's the cause?
A.
Gilbert
B.
Crigler najjar
both occur within (onset less than 24 hours)
C.
Choledocal cyst
D.
Abo incompatibility
https://en.wikipedia.org/wiki/Choledochal_cysts
214- Female Baby 18 month anemia. Dx?
A.
Homozygous b thalassemia
B.
Homozygous a thalassemia
C.
Carrier a thalassemia
Answer :
215- Child with typical absence seizure, what would happen if he were given fentanyl?
A.
Demyelination
B.
Increase stimulation of excitatory GABA,
not sure
(Repeated but leave it for more choices)
216- Infant + RVS chest infection, ttt?
A.
Ribavirin
Torrento note
217- Bronchiolitis:
self-limiting disease with peak symptoms usually lasting 2-3 wk
mild to moderate distress
supportive: PO or IV hydration, antipyretics for fever, regular or humidified high flow O2
severe distress
as above intubation and ventilation as needed

smle ,2016

327
consider rebetol (Ribavirin) in high risk groups: bronchopulmonary dysplasia, CHD,
congenital lung disease, immunodeficient
218- 4 years old child c/o fever and maculopapular rash associated with auricular and occipital lymph
node, he only received his birth vaccines. Dx?
A.
Mumps
B.
Measles
C.
Rubella
D.
Chicken pox
Answer:C
Rubella
Rubivirus
14-21
d
7
d
before
and
after
eruptions
Droplet
Appearance:
pink,
maculopapular
Timing:
1-5
d
after
start
of
symptoms
Prodrome
of
low
grade
fever
and
occipital/retroauricular
nodes

219- Neonate + prolonged bleeding after circumcision, aPTT high, PT, BT and plt are normal, condition
is most likely due to deficiency in?
A.
V
B.
VII
C.
VIII
D.
X
Answer:C
Illustrated textbook
220- Child c/o bilateral knee swelling, fever, pharyngitis, all at the same time I think, high ESR no
other labs, what Dx?
A.
Juvenile RA,
B.
Septic arthritis,
C.
Acute rheumatic fever,
D.
Infectious mono
Answer:C
Illustrated textbook
221- Child c/o fever, bloody stool, and tenesmus, abd exam showed abd distention, Dx?
A.
Ascaris
B.
Amebiasis
C.
Geirdiasis
Answer:B
Reference:Nelson essential
222- Neonate developed cyanosis ( in 2nd or 3rd week after delivery) and there is finding on auscultation I cannot remember exactly but they didnt mention about machinery murmur the qs about the
management

smle ,2016

328
A.
NSAID
B.
steriod
C.
Prostaglandin E1
Answer:C
223- 11 years old child had sever diarrhea 3 weeks ago, now the child presented with bilateral lower
limbs weakness and numbness, DX?
A.
Polio
B.
GBS
C.
Muscular dystrophy
Answer:B
Reference:nelson essential
224- 6 years old child, came for pre school checkup, on exam he looks normal except for grade III heart
murmur along the sternal border no specific location or timing of the murmur provided,
no thrill, the murmur accentuated with supine position, DX?
A.
Still's murmur
B.
VSD
C.
ASD
D.
Innocent?
Not sure if it was in the choices
Answer:A
Reference: nelson essential
225- Child presented to clinic, very tall and very thin not the exact wording of the question Exam
showed cardiac abnormality and pectus excavatum, dont remember other details. Dx?
A.
Marfan**
B.
CHARG syndrome
Don't remember others
Answer:A
Reference named:Nelson essential
226- The most common disorders differential sex in boys
Not clear QQ
227- Scenario of child with guillain barre syndrome had viral gastroenteritis 3 weeks ago ask about
prognosis?
My answer : residual weakness
228- Pediatric patient is complaining of red eye and fever,later he develops pink rash on the face which
spreads to upper and Lower limbs there is also white papule in the mouth, what is the diagnosis?
A)rubella
B)meningococcal rash
I think there is a mistake according to characteristics of the rash should be a red-rose colored lesion to be
the
answer
:
A
but if it is white lesion the answer will be measles
229- Child with left sided abdominal mass , other features? !

smle ,2016

329
Wilms tumor the nearest answer
230 Child with epiglotitis ( whit swollen epi..) with its s&s , what is ttt?
A.
Vancomycin,
B.
ceftriaxon ,
C.
others
Answer:A
Illustrated textbook
In uptodate: ttt is combination of ceftriaxon and vancomycin.!
231- 11 yrs old female with jaundice and splenomegaly?
A.
SCA ,
B.
Spherositosis
Answer:B
232- Child hematuria. Hearing loss?
A. Alport syndrome
Answer:A
Alport syndrom::disease that damages the tiny blood vessels in your kidneys. It can lead to kidney disease
and kidney failure. It can also cause hearing loss and problems within the eyes
Blood in the urine (hematuria), the most common and earliest sign of Alport syndrome
Protein
in
the
urine
(proteinuria)
High
blood
pressure
(hypertension)
Swelling in the legs, ankle, feet and around the eyes (called edema)
233- A child with gum bleeding, erythema papules in mouth. Swab show multinucleated gaint cell.
Which organism
A. Coxacke vi (my answer but not sure)
B. Staph
C. Herpes
Answer
:
Actually
both
A
&
C
have
affect
on
gum
and
cause
gingivostomatitis
http://www.healthline.com/health/gingivostomatitis#Symptoms3
and
also
both
cause
vesicles
papule
on
an
erythematous
base
(
Toronto
Note
)
.
And about multinucleated giant cell we find a few unclear study that said Coxackie virus has multinucleated
giant
cell
!
And
clear
knowledge
about
multinucleated
giant
cell
in
herpes
So
if
there
is
more
details
in
Q,
we
can
sure
about
answer
But
most
likely
is
C
In the absence of cutaneous lesions , the oral lesions of HFMD( coxsackie viruses ) may be mistaken for
aphthous ulcers, Herpes simplex gingivostomatitis or oral varicella lesions. However, the oral erosions in
HFMD are usually smaller, more uniform and asymptomatic unlike those in herpetic gingivostomatitis
which are painful and coalesces, and those of varicella usually last longer and always crust. Unlike HFMD,
both varicella and herpes lesions will also show multinucleated giant cells in Tzanck smears. Herpangina,
another self limiting disease in children due to multiple types of coxsackie viruses and echoviruses and
characterized by acute febrile illness with headache, sore throat, dysphagia, anorexia, occasionally stiff

smle ,2016

330
neck, and small yellowish-white vesicles/ulcers with erythematous areola distributed irregularly over posterior oropharynx (anterior faucial pillars, tonsils, uvula, or soft palate), closely mimics HFMD. However,
absence of skin lesions and characteristic distribution of oral lesions in herpangina are diagnostic. The skin
lesions of HFMD can be distinguished from Herpes simplex associated erythema multiform by the skin
lesions
which
are
round/oval,
grey
and
targetoid
.
http://medind.nic.in/ibv/t10/i4/ibvt10i4p345.pdf
for
more
reading
about
thease
:
http://emedicine.medscape.com/article/1079920-overview#a1
234- child with facial swelling hypoalbuminemia, high cholestrol ?
Nephrotic
Answer
:

syndrome.
A

235- child with gradual developing of cyanosis and ejection systolic murmur of left upper sternal ?
TOF.
A harsh systolic ejection murmur (SEM) is heard over the pulmonic area and left sternal border.
Reference
:
http://emedicine.medscape.com/article/2035949-clinical#b3
left
upper
sternal
=
Pulmonic
area
236- child with morning stiffness and joint pain of wrist and ankle ?
Jeuvenile rheumatoid arthritis
Answer
:
A
Reference
:
http://www.kidsgetarthritistoo.org/about-ja/juvenile-arthritis-early-signs-and-symptoms.php
237- child bilious vomiting and constipation since birth diagnosis ?
Rectum biopsy.
238- 1 month Baby come with abdominal distension and constipation since brith what you do next :
a. Xray
b. biobsy
c. rectal manometrey
Answer
:
B
This
is
Case
of
Hirschsprung
disease
:
This is a distal obstructive disorder of the colon. Abdominal distension, delayed passage of meconium,
and bilious emesis are hallmarks of the disorder. Obstructive series demonstrates diffuse dilation of small
bowel and, possibly, proximal colon. The gold standard for diagnosis is a rectal suction biopsy (RSB)

smle ,2016

331
demonstrating absence of submucosal ganglion cells. Barium enema may help delineate the location of
the transition zone and length of aganglionic colon but can also appear normal, especially in the infant <3
months
of
age.
Reference
:
The
Washington
Manual
of
Pediatrics
.
239- child with non bolious vomiting and olive mass in diagnosis ?
A.
Sonography
240- pt post delivary by one week preseneted with orthopnea and increase jvp symptoms of heart
failure the xray will show :
A.
bilateral infiltration
B.
thoracocardiac rasio <50%
Answer:A
not B because in heart failure there is cardiomegaly so the cardiothoracic ratio is more than 50 %
and
in
Investigations
of
HF

CXR:
cardiomegaly,
pulmonary
venous
congestion
Reference : Toronto Note ..
241- child with eczma use topical steroid not affective what we add:
A.
oral antibiotic
B.
topical antiobiotuc
C.
sulfa somyhing
D.
tacrolimus
Answer:D
Reference
:
https://nationaleczema.org/eczema/treatment/topical-calcineurin-inhibitors/
242- boy 14 years with follow up asymptomatic by examiation\ arrhythmia and (pic of ECG which show
irregular rate and t elevation )!! What's Dx :
The diagnosis according to the picture came in Q ,, But the DDx of irregular rate arrhythmia

Narrow
complex
irregular
tachycardia
:
A.
Atrial
fibrillation
B.
Ectopic
atrial
tachycardia
(irregular)
C.
Atrial
flutter
with
variable
AV
conduction

Wide
complex
irregular
tachycardia
:
A.
Ventricular
fibrillation/fast
polymorphic
ventricular
tachycardia
B.
Torsade
de
pointes
C.
Atrial
fibrillation
with
WPW
D.
Ectopic
atrial
tachycardia,
irregular,
with
aberrancy
Reference
:
The
Washington
Manual
of
Pediatrics
.
243- child with meningitis treated with antibiotics see report what is the cause?

smle ,2016

332

244- Cardiac defect communication between aortic arch and pulmonary vessels
PDA
Answer:A. PDA
Patent
Ductus
Arteriosus
patent vessel between descending aorta and left pulmonary artery (normally, functional closure
within
first
15
h
of
life,
anatomical
closure
within
first
days
of
life)
Reference
:
Toronto
Note
..
245- OPV and IPV common mechanism of action
OPV induces intestinal immunity against poliovirus reinfection, which explains its effectiveness in controlling the wild-type poliovirus circulation . In addition, OPV persists in the pharynx for 1 to 2 weeks and
is excreted in the feces for several weeks or longer after administration . Consequently, the vaccine virus
can be transmitted to contacts and results in their immunization . However, in rare cases, vaccine-associated paralytic poliomyelitis (VAPP) can occur in these contacts as well as in those vaccinated .
Mucosal immunity is induced by IPV and eIPV but to a lesser extent than with OPV . IPV and eIPV inhibit
pharyngeal acquisition of poliovirus and, to a lesser extent, intestinal acquisition.
Reference
:
https://www.drugs.com/mmx/ipv.html
246- Child with head trauma, admitted to ICU and recieved fluid after hours she started to have high
urine output 100ml/hr. What would be the cause?
A. Nephrogenic DI
B. Central DI
C. SIADH
D.
Answer : C
SYNDROME
OF
INAPPROPRIATE
ANTIDIURETIC
HORMONE
(SIADH)
Anti-diuretic hormone (vasopressin) level is inappropriately high despite low serum sodium and osmolality

Etiology
Meningitis, encephalitis, pneumonia, tuberculosis, AIDS, mechanical ventilation, brain trauma, head
trauma, neurosurgery, prolonged nausea, vomiting, ethanol intoxication, pregnancy, medication side effects.

Clinical
Presentation
Hyponatremia (Serum sodium <135), in the setting of euvolemia or hypervolemia with decreased urine
output
and
inappropriately concentrated urine (Urine Osm >100 mOsm/kg and Urine Sodium >30 mEq/L).
Diagnosis cannot be in the setting of hypothyroidism, adrenal insufficiency, renal insufficiency or diuretic
use.
Reference
:
The
Washington
Manual
of
Pediatrics
.

smle ,2016

333

247- Regarding PALS, which is true in pediatric rescitation?


A. 30 compression 2 breath
B. 15 compression 1 breath
C. 10 compression 2 breath
D. 5 compression 1 breath
Answer:B
Answer
:
15
compression
2
breath
Reference : https://www.resus.org.uk/resuscitation-guidelines/paediatric-advanced-life-support / & Oxford of Pediatric .
248- GCS of a 10 yrs child, continously crying and screaming not responding to his name, doesn't move
but respond to pain by ...'I forgot the scenario ?
A. 11
B. 10
C. 9
D. 8
249- Child with renal disease, his family are afraid that he become like his father on dialysis. He is also
known to have SNHL (sensorineural hearing loss), what is the dz?
A. Alport
Answer:A
Reference: http://emedicine.medscape.com/article/238260-overview#a1
250- Child with vomiting presented dehydrated and a small mass was felt in the epigastric area ... what
is his acid-balance?
A. Hypochloremic metabolic alkalosis
B. Metabolic acidosis
Answer:A
This
child
has
Hypertrophic
Pyloric
Stenosis
(
HPS)
The classic biochemical abnormality in HPS is hypochloremic, hypokalemic metabolic alkalosis.
Reference:
http://emedicine.medscape.com/article/929829-overview#a1
251- Neonate continously crying, US should donut sign, how to manage?
Answer: The neonate may have intussusception, endoscopic pneumatic reduction
252- 15 years old male come for routine check up labs Hb: 10 MCV: 69 MCH: 20 WBC: Normal Diagnosis?
A- IDA
B- Beta thalasemia trait*
C- Anemia of chronic Disease
D- SCD
Depends on these issues,answer is A
But We need more details

smle ,2016

334

253- 5 years old girl presenting with hepatosplenomegaly, pale, decreased level of conscious, V\S indicating shock labs: Hb: 5 Platelets: 65 what's your important next investigation?
A- Abdomen US
B- Reticulcytes count* (patient has spleenic sequestration)
C- BM biopsy
Answer:B
Reference: http://www.medscape.com/viewarticle/475043
254- pediatric patient with abdominal distention and severe foul smelling flatus?
A- Giardia lambila*
Answer:A
Reference:
http://emedicine.medscape.com/article/999282-clinical#b1
255- Neonate came e decrease feeding & activity + fever , o/e baby is hypotensiveDx?
Answer: Septic shock

smle ,2016

335
256- child presented with convulsion after GI:
A- salmonella
B- shigella
answer: B Seizures. Generalized seizures have been reported occasionally among young children with
shigellosis, and usually resolve without treatment. Children who experience seizures while infected with
Shigella typically have a high fever or abnormal blood electrolytes (salts), but it is not well understood
why the seizures occur http://www.cdc.gov/shigella/general-information.html#definitions-symptoms
257- 4 y\o child developed resistance or not respond to chloramphenicol in salmonilla infection you
will :
A- repeated dose of chloramphenicol
B- ciprofluxocin
C- IM ceftrixone
Answer is: C reference: http://emedicine.medscape.com/article/2281
treatment we cannot give Bfluoroquinolones to a child he is 4 years
258- Child brought to you with complain of yellwish occlusive area of the teeth what u will give :
A.
Antiseptic mouth wash
B.
Fluride supplement
Answer: B. fluride
259- child 2-4 mths ? Vominting ? Wt height 50 th centile , wt to do
Answer: Reassure
260- Case of 2 months baby , his parent feeds him with bottle of milk while he sleeping , developed
with plaque and discoloration of his teeth : Nursing bottle carries?!
can't recall other answers ?
261- 10 years old baby boy diagnosed with type 1 diabetes , presented to emergency department with
3 days hx of not feeling well and losing weight,Investigations showing high blood sugar and ketone in
the urine. What is best initial thing to do:
A. Electrolyte replacement
B. Fluid replacement
C. Insulin
Answer:B
Provide oxygen and advanced airway management in patients with diabetic ketoacidosis (DKA), if
needed.[7]
Monitor
the
patient.
Provide isotonic intravenous fluids (eg, isotonic sodium chloride solution or lactated Ringer solution).
Perform
fingerstick
glucose
testing.
Consider empiric naloxone if altered mental status is present.
262- years old girl has type 1 DM , her weight 40 kg ( below 50th percentile ) and his height 150 cm (
below 95th percentile ) , she has no signs of secondary sexual characteristics of puberty, you want to
perform annual screening in clinic for :
A.
ophthalmology
B.
growth hormone
C.
Ct to renal

smle ,2016

336
Answer: B
263- Mother changes her baby diaper many times a day Labs all within the normal except NA low ?
what is Dx?
A.
Acute proximal renal tubular acidosis
B.
Acute distal renal tubular acidosis
C.
Congenital chloridorrhea
Answer: C
264- 10 year old with ASS:
A.
VSD
Not clear
265- Overweight child with increase in height and weight more than other children and BMI 30 what
you will do ?
A.
Follow up after 12 m
B.
life's style modification
C.
No treatment
D.
Anti obesity medication
Answer:A
266- baby diagnosed with Cystic fibrosis. ..he has + sweat chloride test his brother is normal. to confirm
diagnosis of cystic fibrosis....
A.
CTRF gen in parent
B.
CTRF in sibling
C.
Chloride test. .parent
D.
Chloride test in sibling
Answer:A
https://www.nhlbi.nih.gov/health/health-topics/topics/cf/diagnosis
267- in CPR for infant you'll use :
A.
Atropine
B.
Epinephrine
Answer:B
http://www.merckmanuals.com/professional/critical-care-medicine/cardiac-arrest/cardiopulmonary-resuscitation-in-infants-and-children
268- Di George syndrome
A-cervical cyst
B-ectopic thymus
C-ectopic parathyroid
D-fistula
Answer: B
http://primaryimmune.org/about-primary-immunodeficiencies/specific-disease-types/digeorge-syndrome/
269- child had constipation after few days he develop bloody urine and lower abdominal pain
a) autoimmune hemolytic anemia
b) UTI

smle ,2016

337
Answer:A
http://www.merckmanuals.com/home/blood-disorders/anemia/autoimmune-hemolytic-anemia
270- Preterm baby c/o SOB X-ray showed gross ground appearance + air bronchogram This is due to ?!
A- Pneumonia
B- Low surfactant
ANSWER B
http://emedicine.medscape.com/article/409409-overview
271- 46 y/o came to the clinic with her 1w old baby, complaining of ( symptoms of hyperthyroidism
recently developed):
a) Hypothyroidism
b) postpartum thyoditis
c) Thyroid tumer ?? (I can't remember the other choices)
Answer: B
272- about phenyleketonuria which enzyme is defecient?
phenylalanine hydroxylase (PAH)
http://emedicine.medscape.com/article/947781-overview
273- Dehydrated child 25 kg , maintenance is
A-1500
B-1750
C-1625
ANSWER C
http://www.slideshare.net/golwalkar/maintainance-replacement-fluid-therapy-pediatrics-ag
274- Long history of child with history of infectious mononeclouses ( upper rispartory tract infection
with hepatosplenomegaly) what is your next step ?
A- CBC
B- Monospot test
Answer is B
explanation:
the
child
has
past
history
of
of
infectious
mononucleosis
and
he
is
suffering
from
URTI
So
I
believe
here
the
aim
is
to
detected
the
level
of
antibodies
by
monospot
test
the
read
about
monospot
:
http://www.cdc.gov/epstein-barr/laboratory-testing.html
275- Problem in the phagocytosis in a child with normal immunoglobulin
Which protein defected give problem in oxygenation :
A.
Cytochrome 460
B.
Lysosomal
C.
Something with nitric
D.
Decrease in NADH
Answer:D
Chronic granulomatous disease (CGD) is a rare (1:250,000 births) disease caused by mutations in any
one of the five components of the nicotinamide adenine dinucleotide phosphate (NADPH) oxidase in

smle ,2016

338
phagocytes. This enzyme generates superoxide and is essential for intracellular killing of pathogens by
phagocytes.
276- 5 years old child presented with constipation, polyuria , growth retardation You found na 130 , k
3.1 , cl normal , hco3 above 40 high , the defect in :
A.
Nacl chanel excretion or absorption not sure
B.
H ion absorption
C.
H ion excretion (the answer? )
D.
Chloride absorption or excretion not sure
277- hx of child with multiple osteochondroma he presented in the imaging with huge mass from the
left hip with calcification
A.
Chondrocytoma
B.
Ewing carcinoma
C.
I don't remember it's type of multiple osterochondroma
Answer:
A
according
to
these
options
in this study Ewing sarcoma has never been reported superimposed on an osteochondroma
http://www.healio.com/orthopedics/journals/ortho/2014-4-37-4/%7B060e9345-37e1-45d7-b8c97ee71ce6502d%7D/ewing-sarcoma-superimposed-on-a-previous-osteochondroma-in-multiple-osteochondromatosis
https://ojrd.biomedcentral.com/articles/10.1186/1750-1172-3-3
278- child with hx of cancer on chemotherapy he had insertion of venous line , he develop gram negative sepsis , What could the organism
A.
E-coli
B.
N. Mengitis
C.
Psoudomans
Answer:C
http://www.ncbi.nlm.nih.gov/pmc/articles/PMC3916372/
Increasing antimicrobial resistance among gram-negative enteric bacteria and opportunistic gram-negative pathogens (e.g., Pseudomonas, Acinetobacter, Burkholderia spp.), also raises the risk of mortality
among infected children by delay of effective antibiotic treatment and/or from increased virulence that
is observed in some multidrug-resistant organisms. Such organisms are most commonly identified in children hospitalized for prolonged periods with persistent indwelling devices such as intravascular catheters
or tracheostomies,49 and in oncology and other immune-suppressed patients who have had multiple
courses of broad-spectrum antibiotics
279- child receive his hep A first dose He present to the clinc to receive his second dose mother give
you hx of sickle cell crises 3 weeks ago and he recived blood transfusion upon that ,otherwise he is fine
now examination normal what you gonna do ?
A.
Give him the scheduled vaccine
B.
Give him after 3 months
C.
Give him after 6 months
D.
Do hep A serology level
Answer:A
http://www.medscape.com/viewarticle/720175_8

smle ,2016

339
280- infant sit in tripod position , role from prone to supin , reach object
A.
4 mon
B.
6mon
C.
12mon
D.
24 mon
Answer:B
http://2.bp.blogspot.com/-QYxe36daT4o/UFvu_fHzN9I/AAAAAAAAABQ/e46pvQmRZcY/s1600/developmental+milestones.jpg
281- Asthmatic child with prodrome sx : (Not clear!)
A.
venral conjunctivitis
B.
bacterial conjunctivitis
282- Infant recently shifted to cow&#39;s milk presented with abdominal pain and diarrhea :
A.
continue
B.
free-milk diet
Answer: stop cow milk
http://www.espghan.org/fileadmin/user_upload/guidelines_pdf/Diagnostic_Approach_and_Management_of_Cow_s_Milk.28.pdf
Infants
Up
to
Age
12
Months
If the diagnosis of Cow milk protein allergy is confirmed, then the infant should be maintained on an
elimination diet using a therapeutic formula for at least 6 months or until 9 to 12 months of age. Infants/children with severe immediate IgE-mediated reactions may remain on the elimination diet for 12
or even 18 months before they are re challenged after repeated testing for specific IgE
283- Child with iron toxicity what best to do ?
A.
syrup ipecac
B.
gastric lavage ( no activated charcoal in choices ) .
Answer: http://emedicine.medscape.com/article/1011689-treatment
Ipecac-induced
emesis
is
not
recommended.
Gastric lavage is not recommended because iron tablets are relatively large and become sticky in gastric
fluid,
making
lavage
unlikely
to
be
of
benefit.
Whole bowel irrigation has been used to speed the passage of undissolved iron tablets through the GI
tract, although there is no convincing evidence from clinical studies that it improves the outcome. A polyethylene glycol electrolyte solution (eg, GoLYTELY) may be administered orally or nasogastrically at a
rate of 250-500 mL/h for toddlers and preschoolers and 2 L/h for adolescents. Continue irrigation until
the
repeat
radiographic
findings
are
negative
or
rectal
effluent
is
clear.
Deferoxamine is the iron-chelating agent of choice. Deferoxamine binds absorbed iron, and the irondeferoxamine
complex
is
excreted
in
the
urine.
284- Child with growth retardation in addition to GH , what you going to check : A.
somatomedin
Answer:
http://emedicine.medscape.com/article/920446-workup#showall

Thyroxine
(T4)
and
thyroid-stimulating
hormone
(TSH)

Serum
electrolytes

CBC
count
and
sedimentation
rate

smle ,2016

340
IGF-1 and IGFBP-3: Both IGF-1 and the binding protein IGFBP-3 are growth hormone (GH) dependent.
Karyotype
285- 8 years old girl brought by her parents regarding widing gait. Hip examination show trenderburg
gait and + signs ( i dont remmeber it but start with C ) all labs are normal x-ray show wide hip ..dx?
A-rickets
B-achonodro somting &quot; i choose it&quot;
Answer
http://www.aafp.org/afp/2009/0201/p215.html
286- child present with rash and vesicles on trunk , chest &amp; upper lime with fever? ; not sure&quot;
With one of the following antibodes will confirm your dx?
A- mumps anti somting
B- VZV antibodies
Answer:B
Because diagnosis is chicken pox
287- child come with painless witish patches in later side of tounge and posterior palate...dx?
A- Metal ingesion
B-tounge- lu somthing &quot; i choose it&quot;
Answer:
http://www.aafp.org/afp/2010/0301/p627.html#sec-1
ORAL
HAIRY
LEUKOPLAKIA
The condition is characterized by white, hairy appearing lesions localized to the lateral margins of the
tongue, in a unilateral or bilateral fashion. It is caused by Epstein-Barr virus infection
288- young girl her height is 167 and her BMI is 24 she classified as :
A- Normal
B- obese
C- morbide obesity
D-underweight
Answer:A
http://apps.who.int/bmi/index.jsp?introPage=intro_3.html
289- Young pt with symptoms of GBS ascending paralysis. Lab csf high protein What other features you
will find?
A.
high cell count csf
B.
cranial nerve association
C.
fever
Answer:
http://emedicine.medscape.com/article/315632-overview
The weakness may progress over hours to days to involve the arms, truncal muscles, cranial nerves, and
muscles
of
respiration.

290- pt young nephritic syndrome , guarding and tenderness abdomen.


A.
peritonitis
B.
appendicitis

smle ,2016

341
C.

pancreatitis

291- Can't sit without support , coos and laughs


A.
6 months
B.
3 months
C.
1 month
D.
9 months
Answer:B
http://2.bp.blogspot.com/-QYxe36daT4o/UFvu_fHzN9I/AAAAAAAAABQ/e46pvQmRZcY/s1600/developmental+milestones.jpg
292- child with sickle cell anemia , bilateral hip pain . Dx ?
A.
Avn (Avascular necrosis)
Answer:A
293- child brought with diarrhea and abdominal pain . Distended abdomen lab otherwise normal. Dx ?
A.
IBD
B.
carbohydrates intolerance
A lot of differential diagnosis here!!
294- 6 years old with blood pressure above 95th percentile - +ve FHx of HTN - radial pulse intact femoral
pulse is absent ( cause )
A.
Essential hypertension
B.
Renal artery stenosis
C.
Coarcotation of the aorta
Answer: C
reference:
Master
the
board
:USMLE
step
2CK
according
to
Presentation
:
Severe
CHF and
respiratory
distress within
the
first
few
months of life
Differential

pressures

andpulses

Reduced
pulses
in
the
extremities due to narrowing

lower

between
extremities

theupper
and

and

hypertension

lower
in

extremities
the

upper

295- Child with croup. What will you find in chest auscultation ?
A. Wheezing
B. Crepitation
C. Gasp ?
D. Silent chest ( not sure about this option)
Answer:A
A lot of reference say wheezing
296- Child presented with .. Examination: heave ,, ECG: shows RBBB ,, ECHO: Rt ventricle motion abnormality and Rt ventricle hypertrophy What is the most likely cause?
A.mitral prolapse
B. ASD

smle ,2016

342
C. VSD
D. Coarctation of aorta
Answer:C
297- Surgeon want to treat child diagnosed with PDA. During procedure, the surgeon is at risk to injure?
A- Left Phrenic Nerve
B- Vagus Nerve
C- Left Recurrent Laryngeal Nerve
Answer:C
Complications of operative PDA closure include bleeding, injury to the recurrent laryngeal nerve, disruption of the thoracic duct with resultant chylothorax, and injury to the vagus nerve. The more serious
complications of surgical closure include ligation of the left pulmonary artery, ligation of the descending
aorta, or ligation of other arterial structures within the chest; the results of such mishaps are catastrophic.
Other developments to watch for include incomplete closure or recanalization, residual shunting, and
vocal cord paralysis.
298- child can Know the colour when you point at them, ride tricycle but cannot copy square what is
his age ?
A- 2 y
B- 3
C- 4
D- 5
Answer: C
299- 10 years old child with difficulty to eat for 2 years , multiple time aspiration in the past two weeks
, on Total parenteral nutrition 1000 calorie and protein daily with lab result attached (low albumin and
anaemia ) what is your action ?
A- continue same TPN dose
B- continue TPN with increase dose to 2000
C- insert gastrostomy tube, same calorie
D- insert gastrostomy tube decrease calorie to 80
ANSWER B
300- What is the type of murmur in atrial septal defect ?
A- Fixed splet in s2
B- Ejection systolic clic
answer is A
reference:
Master
the
board
Patients are usually asymptomatic except for

:USMLE
fixed wide

step
splitting

of

2CK
S2.

301- neonate with gradual cyanosis, in the beginning there is parasternal murmur without cyanosis,
there is right ventricular hypertrophy , right axis deviation, on x-ray there is small heart with pulmonary
vessel (something I didnt recall) what is the diagnosis ?
A- Transposition of great vessels
B- Tetralogy of fault
C- acyanotic heart disease
answer is A

smle ,2016

343
reference:
Master
the
board
:USMLE
step
Transposition of the great vessels is the most common cyanotic heart
of newborns. Tetralogy of Fallot is the most common cyanotic heart disease of childhood.

2CK
disease

302- 4 month child which developmental milestone ?


A- try to crawling
B- roll over from side to side
C- sit without support
D- complete fixation of the head
answer C
Reference: first aid USMLE CK2
303- neonate with bilious vomiting , seed stool diharrea he passed meconium in day 2 what is the cause
?
A- hischbrug disease
B- allergy to formula milk
answer B
304- child with bronchiolitis what is the treatment ?
A.
ribavirin
B.
acyclovir
C.
other antiviral
answer is A
Reference:
first
aid
USMLE
CK2
Treatment is primarily supportive; treat mild disease with outpatient management using fluids and nebulizers
if
needed.
Hospitalize
if
signs
of
severe
illness
are
present.
Treat inpatients with contact isolation, hydration, and O2. A trial of aerosolized albuterol may be attempted;
albuterol
therapy
should
be
continued
only
if
it
is
effective.
Corticosteroids
are
not
indicated.
Ribavirin is an antiviral drug that has a controversial role in bronchiolitis treatment. It is sometimes used
in
high-risk
infants
with
underlying
heart,
lung,
or
immune
disease.
RSV prophylaxis with injectable poly- or monoclonal antibodies (Respi- Gam or Synagis) is recommended
in winter for high-risk patients 2 years of age (eg, those with a history of prematurity, chronic lung disease,
or
congenital
heart
disease).
305- initial management of juvenile RA,
A- Intra articular corticosteroid
B- Paracetamol
ANSWER NSAID Ithink
JVA first line >> both NSAID + INTRA ARTICULAR CORTICOSTEROIDS
Reference:
brattons
family
medicine
NSAIDs are the first-line treatment for JRA. Clinical improvement may not be seen for up to 1 month.
Methotrexate is often used with NSAIDs, particularly for systemic and polyarticular JRA. Corticosteroids
are used orally for severe, life-threatening, systemic JRA and intra-articularly for pauciarticular JRA
306- child with red urine ,constipation,Urine analysis increase RBCs ,WBC ,protein:
A- UTI

smle ,2016

344
B- HSP
C- Hemolytic uremic
D- post streptococcus glomurenephritis
ANSWER A
307- Child with arthritis , fever, epistaxis, gingival bleeding, results plt is low , hgb is low.. Appropriate
investigation?
A- Bone marrow aspiration
B- Electrophoresis
C- Anti dsDNA
ANSWER A
Resources: nelson essential of pediatrics textbook +Boston children's hospital organization
308- six year old girl with pan-systolic murmur Any of
A- VSD
B- MR
C- TR
ANSWER A
nelson
essential
of

pediatrics

textbook

309- five years lumping with pain over five month no hx of trauma or fever dd ?
slipped femoral head TYPICAL SCENARIO
12 Y +OBESE+ Cannot bear weight + hip externally rotated>>slipped capital femoral epiphysis
REFERENCE : BRATON FAMILY MEDICINE
http://orthoinfo.aaos.org/topic.cfm?topic=a00052
American
association
of
family
medicine
Chicago university website
310- which situation the hepatitis c positive mother should not breastfed her infant ;
A- Lack of hepatitis c vaccine
B- Cracked nipples
ANSWER B
American
association
of
pediatrics
Breastfeeding
is
NOT
contraindicated
with
the
following
conditions:
mothers who are infected with hepatitis C virus (persons with hepatitis C virus antibody or hepatitis C
virus- RNA-positive blood)
311- child was diagnosed with DM type 1 presented with frequent hypoglycemic attack at different
time during the day, he was diagnosed 6 month back, and he is complinced to his diet and treat, the
most likely cause of his symptoms is:
A- brittle diabetes
B- down phenomena
C- smogy phenomena
D- honey moon period
ANSWER A
reference: http://www.uptodate.com/contents/the-adult-patient-with-brittle-diabetes-mellitus
Brittle diabetes mellitus (or labile diabetes) is a term used to describe particularly hard to control type 1
diabetes
.

smle ,2016

345
Those people who have brittle diabetes will experience frequent, extreme swings in blood glucose levels
,
312- adolescent with fever, headache, meningism after swimming in a river; causative organism ;
A- Streptococcus
B- H. Influanza
C- N. Meningitidis
D- Naegleria fowelri
ANSWER D
primary amebic meningoencephalitis (PAM). The ameba is commonly found in warm freshwater (e.g.
lakes,
rivers,
and
hot
springs)
and
soil
http://www.cdc.gov/parasites/naegleria/
313- 7 days baby presented with vomiting and fever ,culture shows catalase positive ,gram positive in
chain ,beta hemolytic .Mother had hx of infection before delivery .the tx is :
Ampicillin.
Combination
of
Ampicillin.and
gentamicin
Nelson
essential
of
pediatrics
314-most causes of FTT :
A- psychosocial
B- cystic fibrosis
C- asthma
ANSWER A
Nelson essential
315- 8 month child presented with high fever 39.5 for 2 days examination is normal BP is 120\80 HR is
120 what is the organism
A- E.coli
B- H.influnz
C- Listeria monocytogense
D- Streptococcus pneumonia
The answer is D (the most common organism in this age group for meningitis )
316- Long scenario about child with dysmorphic features, developmental delay seizure disorder on anticonvulsant therapy other details
A- IPV instead of OPV
B- Deter DTP
C- Deter all live vaccine
D- Deter all vaccine
the answer is B
http://www.cdc.gov/vaccines/hcp/admin/contraindications-misconceptions.html#modalIdString_CDCTable_0
317- inflammation there is internal rotation with 30 degree fixed flexion of left hip
DDH
Nelson essential page 741-74

smle ,2016

346
318- when to give both heparine and FFP:
DIC and thrombosis
ANSWER IS A Reference : om ALOQRA
This Q comes in pedia section which one of pediatric hematologic disorders treated with heparin blood
plus FFP
319- pediatric with holosystolic murmer in left 3rd intercostal space , with hyper dynamic heart and
loud S2 whats the dx:
A- VSD
B- ASD
C- PDA
ANSWER A
Nelson essential
320- 3Y , UTI , febrile what to do:
answer: US
If I'm suspecting UTI from case senario >-- suprapupic aspiration and culture
As
treatment
>-antibiotics
As other investigation to evaluate >---VCUG AND DEMSA for scaring detect
Nelson 414-415
321- child with anemia a picture showed spherocytosis , what investigation you will do?
A- electrophoresis
B- osmotic fragility test
C- G6pd levels
answer is: B
http://www.seattlechildrens.org/medical-conditions/heart-blood-conditions/ hereditary-spherocytosissymptoms/
http://www.austincc.edu/mlt/clin1/hematology_review1.pdf
322- young child presented with painful lesion in the back of her mouth and soft palate:
ANSWER : herpangina
http://www.medscape.com/viewarticle/457481_4
323- newborn scrotal swelling tenderness , both testis what is the next investigation :
A- illumination
B- wait till pt cough or cry
C- watch voiding
ANSWER A
Nelson essential 227
324- development child walk alone & build three cup ?
A.
18 months
Answer:A
https://www.verywell.com/developmental-milestones-three-months-to-five-years-1449132
325- newborn with congenital adrenal hyperplasia present with ;
A-Hirsutism

smle ,2016

347
B- Infantile acne
C- Abdominal striae
D- Dehydration
If
the
newborn
was
male
ANSWER
D
If the newborn is female then she may present in adolescence with hirsutism http://emedicine.medscape.com/article/919218-clinical#b1
326Child
say
few
words
a)
b)
Answer:A
http://www.m.webmd.com/a-to-z-guides/baby-talk-your-babys-first-words
https://www.uptodate.com.ezp.uod.edu.sa/contents/image?imageKey=PEDS%2
topicKey=PEDS%2F14 339 source=see_link

what

age
12m
24m

58598

327- child after hx of URTI developed ascending muscle weakness ?


a) guilian barre syndrome
Answer: A
source:http://emedicine.medscape.com/article/315632-clinical
328- Child with severe rheumatic fever & cardiac involvement what to give for short period?
a) iIM penicillin monthly
b) large dose aspirin + Oral steroid
Answer: B
http://emedicine.medscape.com/article/891897-treatment#d8
329- child etc ... Can Not take the object in 2 fingers .. How old is he?
a) 4 months
b) 6 months
c) 9 months
Answer: B
Source: http://connectability.ca/2011/03/21/practical-strategies-for-developing-fine-motor-skills/
330- 2 months baby brought to hospital Parents complain that the baby is spitting what you will do?
a) Reassurance
b) CT abdomen
Answer: a
331- infantile colic:
A) Decreased peristalsis.
B) Increased gases.
Answer
:
https://www.uptodate.com.ezp.uod.edu.sa/ contents/infantile-colic-clinical-features-andsis?source=outline_link&view=text&anchor=H1#H1
332- The most common drug used to treat juveniles rheumatoid arthritis is:

smle ,2016

B
diagno-

348
A. Paracetamol
B. Penicillamine
C. Systemic steroid
D. Aspirin
Answer: B
333- IDA in 2 year old child hg 9 what to give?
A-Oral
B-IV
C-Transfusion
Answer: A
334- complication of screening for down syndrome ?
A- rupture of membrane
B- abortion
Answer: B
Reference: uptodate
335- child come with severe symptom of croup, the doctor gave epinephrine then relive but after time
again symptom? What will do?
1- repeated epinephrine
2- steroid
Answer:A
336- Adolescent with recurrent swelling of lips gene affected?
a) SERPING1 gene
Answer: A
Source: Mutations in the SERPING1 gene cause hereditary angioedema type I and type II. The SERPING1
gene provides instructions for making the C1 inhibitor protein, which is important for controlling inflammation. C1 inhibitor blocks the activity of certain proteins that promote inflammation.
http://ghr.nlm.nih.gov/condition/hereditary-angioedema
337- baby with 5 min after birth assessing (HR 120 ,breath irregular and grasping, acrocynotic , cough
and
grimace
,
flexing
all
limbs
not
moving
)
apgar
score
:
A. 6
B.
7
C.
8
D.
9
Answer:C
338mother
wants
to
screen
her
fetus
A.
chorionic
villus
sampling
at
B.
amniotic
fluid
with
something
C. amniotic fluid with 2 other things at 16 week
Answer:B

for
at

thalassemia
:
16
week
15
week

339- baby presented with abdominal bloating and constipation , inv shows increase ca+ . I forgot the
choices

smle ,2016

349
Answer:
http://emedicine.medscape.com/article/920955-clinical#b4
340- 4 years old boy was found on the floor of the kitchen , cyanosed , has peanut bean on his hand :
Answer: aspiration
341- full term baby on breast milk and gaining weight, presented 10 days after birth with yellowish
discoloration,
indirect
bilirubin
19.5,
direct
2.7,
what
to
do
next
:
A.
blood
exchange
B.
phototherapy
Answer:phototherapy http://www.uptodate.com/contents/jaundice-in-newborn-infants-beyond-thebasics
342Answer:

baby

was
jaundiced
Oxidation

then

become
the

of

greenish

:
bilirubin

343- baby with unknown numbers of paracetamol pills ingestion presented after 10 or 4 hours what to
do:
A.
N.ascetylecystin
B.
Drug
level
Answer:
Drug
level
http://www.rch.org.au/clinicalguide/guideline_index/paracetamol_poisoning/
see
paracetamol
ingestion
flowchart
http://emedicine.medscape.com/article/820200-overview
Serum paracetamol concentration at (or as soon as possible after) 4 hours post ingestion determines the
need
for
N-acetyl
cysteine
(NAC)
administration.
There is no benefit in measuring paracetamol concentration earlier than 4 hours post ingestion.
It is safe to wait for the paracetamol concentration to decide on the need for NAC in all cases that present
within 8 hours of ingestion AND where a paracetamol concentration result will be available for interpretation
within
8
hours
of
ingestion.
Patients who present 1 > hours after a toxic ingestion 122>( mg/kg) or after an ingestion in association
with symptoms of toxicity (RUQ pain or tenderness, nausea, vomiting) should be commenced on NAC
immediately. The decision to continue or cease NAC is then based on the paracetamol concentration.
344baby
Answer: ORS

with

diarrhea

no

vomiting

how

to

manage

345- 1 month old baby, has constipation science birth , best diagnostic investigation?
manometry
Answer:
full
thickness
intestinal
biopsy
.
http://emedicine.medscape.com/article/937030-overview#a8
346scenario
not clear QQ
347A.
B.

monozygot

smle ,2016

about

twin

renal

tubular

presentation (twin
Cephalic.
transverse.

A/twin

acidosis

B)

will

in

be

dengioras

child:

in :
Cephalic
Cephalic

350
C.
Answer: cephalic cephalic

Breech

Cephalic

348- child parents Have TB- You read a PPD result after 48 hours. It showed 10 mm induration. What
does that indicate?
a. negative
b. weakly positive
c. strongly positive
Answer: C
349- The person's medical risk factors determine the size of induration the result is positive (5 mm, 10
mm, or 15 mm).
Five mm or more is positive in:
HIV
-positive person
Recent contacts of active tuberculosis cases
Persons with nodular or fibrotic changes on Chest X-ray consistent with old healed TB
Organ transplant recipients and other immunosuppressed patients who are on cytotoxic immune-suppressive agents such as cyclophosphamide or methotrexate.
Patients on long term systemic corticosteroid therapy (> than six weeks) and those on a dose of prednisone 15 mg/day or equivalent.
End stage renal disease
Ten mm or more is positive in:
Recent arrivals (less than five years) from high-prevalence countries
Injectable drug users
Residents and employees of high-risk congregate settings (e.g., prisons, nursing homes, hospitals, etc.)
Mycobacteriology lab personnel
Persons with clinical conditions that place them at high risk (e.g., diabetes, prolonged
corticosteroid therapy, leukemia, end-stage renal disease, chronic malabsorption syndromes, low body
weight, etc.)
Children less than four years of age, or children and adolescents exposed to adults in high-risk categories
Infants, children, and adolescents exposed to adults in high-risk categories
Fifteen mm or more is positive in:
Persons with no known risk factors for TB. (Reactions larger than 15 mm are unlikely to be due to
previous BCG vaccination or exposure to environmental mycobacteria).
Reference: Pubmed
350sabin
(polio
vaccine)
IV
A.
Increase
Intestinal
B.
Attack
on
anterior
horn
to
Answer: A

&

oral
both
do:
IgA
secretion
prevent
viral
attachment

351child
with
fever.
Rash.
Grysh
macul
in
moth
dx???
Answer:
measles
consist of bluish-gray specks against an erythematous background . http://emedicine.medscape.com/article/1079920-overview#a6

smle ,2016

351
352- baby brought by his parents to ER complaining of SOB & drooling saliva ................... diagnosis :
A.
croups.
B.
epiglottitis.
Answer: B
353- normal 10 y/o child , his parents worry about height of child ............. ? what the most thing to ask
??
can't
remember
choices
Answer:
354- Child with nephrotic syndrome on steroid for 3-6wks or 6 months have vaccine ?
A.
Give
the
vaccine
B.
Stop
the
steroids
C.
3month
and
give
appointemnt
Answer:C
Killed or inactivated vaccines do not represent a danger to- immunocompromised persons and generally
should
be
administered
as
recommended
for
healthy
persons
Steroid therapy usually does not contraindicate administration of live-virus vaccines when such therapy is
short term (less than 2 weeks); low to moderate dose; long-term, alternate-day treatment with shortacting preparations; maintenance physiologic doses (replacement therapy); or administered topically
(skin or eyes), by aerosol, or by intra-articular, bursal, or tendon injection. The exact amount of systemic
corticosteroids and the duration of their administration needed to suppress the immune system of an
otherwise healthy child are not well defined. The immunosuppressive effects of steroid treatment vary,
but many clinicians consider a dose equivalent to either 2 mg/kg of body weight or a total of 20 mg/day
of prednisone as sufficiently immunosuppressive to raise concern about the safety of immunization with
live-virus vaccines. Corticosteroids used in greater than physiologic doses also may reduce the immune
response to vaccines. Physicians should wait at least 3 months after discontinuation of therapy before
administering a live-virus vaccine to patients who have received high-dose, systemic steroids for greater
than
or
equal
to
2
weeks..
https://www.cdc.gov/mmwr/preview/mmwrhtml/00023141.htm
355- 2 year child didn't complete his vaccination ( 10 months is the last one ) present with fever bilateral
swelling pediauricular , unable to swallow , dysphasia , enlarged tonsils and spleen and lymph nodes...(
no
Infectious
Mononucliosis
nor
mump
in
the
choices):
Answer:
Diphtheria
Streptococcus
pharyngitis
356What
is
the
milestone
of
4
A.
40
60
B.Count
to
4
C.
tell
D.
Say
mama
Answer
:
tell
a
http://www.cdc.gov/ncbddd/actearly/milestones/milestones-4yr.html

year

357A.
B.
C.

in

smle ,2016

which

vaccine

is

contraindicated

child

HIV

?
word
and
story
baba
story

pt
OPV
varicella
MMR

352
C.
Answer:OPV
Confirmed by CDC

HBV

358- Boy has a cat developed itching for a month with Red eye and watery with discharge No lymphadenopathy
and
general
exam
normal:
A.
Cat
scratch
Dermtitis.
B.
allergic
conjunctivitis.
Answer: B
359- Boy with elevated WBCs very high? Low hemoglobin Low MCV Low reticulocytes What is the diagnosis?
A.
Iron
deficiency
B.
B
Thalasemia
trait
C.
Sickle
D.
Anemia
of
chronic
disease
Answer:D
It's nearest answer
360- 7 year girl developed pubic hair And axillary hair and complain from acne and breast develop NOT
enlargement of clitoris what dx?
a) central cause of puberty (something same this)
b) gonadotropin releasing tumor
Answer:
Etiology of PRECOCIOUS PUBERTY
usually idiopathic in females (90%), more suggestive of pathology in males (50%)
central (GnRH dependent)
hypergonadotropic hypergonadism; hormone levels as in normal puberty
premature activation of the HPG axis
differential diagnosis: idiopathic or constitutional (most common in females), CNS disturbances (tumors,
hamartomas, post-meningitis, increased ICP, radiotherapy), NF,primary severe hypothyroidism
peripheral (GnRH independent) hypogonadotropic hypergonadism
differential diagnosis: adrenal disorders (CAH, adrenal neoplasm), testicular/ovarian tumor,gonadotropin/hCG secreting tumor (hepatoblastoma, intracranial teratoma, germinoma),exogenous steroid administration, McCune-Albright syndrome, aromatase excess syndrome,rarely hypothyroidism (Van WykGrumbach syndrome).
361- 5 year old child present with odor of adult and pubic hair what investigation will do?
Answer:
investigation: initial screening tests: bone age, serum hormone levels (estradiol, testosterone, LH,
FSH,TSH, free T4, DHEA-S, 17-OH-progesterone)secondary tests: MRI head, pelvic U/S, -hCG, GnRH,
and/or ACTH stimulation test
Confirmed by nelson p652
362- Breastfeeding mother treated with HCV by interferon more than one year what the risk of breastfeeding on infant ?
A.
Nipple crickeld
B.
Mother with anemia

smle ,2016

353
C.
Infant complain of oral candidacies
D.
Not follow up of infant immunization
could be A: nipple crickeld ...
http://www.hepatitisaustralia.com/living-with-hepatitis-c/women-and-hepatitis-c
363- treatment of streptococcus glomerulonephritis in children with edema and HT ?
A.
High dose of antibiotic
B.
Diuretic for edema
C.
Diuretic for htn
both B and C acceptable but I think B is the correct answer because we need to remove excess fluid
(edema)
to
correct
hypertention.
from Medscape: http://emedicine.medscape.com/article/239278-treatment#d9
364- fluid required to give for neonate 10 KG, loss 5% of his body fluid?
A.
1000
B.
1500
C.
2500
Answer:B
we have to calculate the fluid deficit and maintenance. fluid deficit = wt x percentage of loss x 10 ml of
fluid.
maintenance
=
first
10
of
child
wt
=
100
ml/kg
second
10
=
50
ml/kg
addition
kg
=
20
ml/kg
so in this case: fluid deficit = 10 x 5 x 10 = 500 ml and the maintenance = 10 x 100 = 1000 . so total fluid
required
is
1500
ml/day
Check nelson page: 124 for more understanding.
365- child with pain when chewing and headache, he complain loss of weight and decrease appetite ?
(Temporomandibular joint dysfunction) ? what is the complain ?
A.
Hearing loss
B.
Facial palsy
Answer:A
366- High height low weight dm1 child, what to check next
A.
GF
B.
TSH
Answer:A
367- mother bring her first & only infant was fatigute not move or gaze from light direct on his eye
what the Dx?
A.
infantile botulism
http://emedicine.medscape.com/article/961833-clinical
368- 6y old boy fever, sore throat, developed pink maculopapular rash all over and pericarditis , what's
the Dx?
A.
still' disease
B.
Kawasaki Disease
Answer:B

smle ,2016

354

369- what is the condition that baby will have bone age more than chronological age?
A.
hypothyroid
B.
chronic kidney disease
C.
congenital adrenal hyperplasia
D.
Reckitt
Answer:C
370- milestone baby can hold his head and when he looks at his flying hand he laughs and coos
-If he asked about the age it would be 2 months ((Toronto notes,2015,p5 \ first aid USMLE
step2,ED8,p361..))
And nelson essential p20
371- milestone said baba and walk holding furniture and a lot of other features
A. 12M
B. 10M
Answer: B
By Nelson essential p20
372- Say mama/baba at 9-11 m but it not specific meanwhile he walk throw the object
at 12m . ((First Aid for the USMLE Step 2 CK, 8Ed ,p361 ))
373- 1 month Baby come with abdominal distension and constipation since birth what you do next:
a. Xray
b. biopsy
c. rectal manometry
I think the diagnosis is Hirschsprung's disease and the barium enema is the imaging study of choice ((First
Aid for the USMLE Step 2 CK ,8Ed,p368 ))
Answer: it's could be A
374- One month boy came for vaccine. His older sister (6 years old) had renal transplant and now is on
immunosuppressive medication. Which vaccine is contraindicated? (Doesnt say for the baby or for the
girl)
a- MMR
b- Oral polio
c- Salk polio
d- Influenza
Answer:
B
Oral polio b/c of the risk of spread OPV for immunocompromised girl
Explanation: VAPP "vaccine associated paralytic poliomyelitis" usually occurs among young infant OPV
recipients and among direct contacts of OPV recipients. The overall risk is about 1 case per 900,000 first
dose OPV. Adults may be at higher risk than young infants and children. Persons with B cell immunodeficiency carry the highest risk with an estimated VAPP rate of 2 per 1000 vaccines
Which means immunocompromised direct contacts carry the highest risk for OPV complications
Source: uptodate: poliovirus vaccination
375- Most common intra-abdominal tumor in infants?
a- Wilms tumor

smle ,2016

355
b- Neuroblastoma
Answer: B
Explanantion: neuroblastoma is the most common intra-abdominal malignancy of infancy.
Wilms tumor is the most common childhood abdominal malignancy with a median age of 3.5 years. Source
: Medscape : neuroblastoma : http://emedicine.medscape.com/article/439263-overview#a5
Wilms
tumor
:
http://emedicine.medscape.com/article/989398-overview
376- pediatric pat k/c of Asthma since age 2 years he is now asymptomatic with rare uses of albuterol
inhalers, he came for counseling, he had contact sport participation, what is best question to ask to
know his response:
A- Are keeping up with your freinds
B- How frequent do use inhaler
C- presence night symptoms (cough)
answer : C
according to uptodate : Symptom assessment Symptoms over the past two to four weeks should be
assessed at each visit. Assessment should address daytime symptoms, nighttime symptoms, use of short
acting inhaled beta agonists to relieve symptoms, and difficulty in performing normal activities and exercise. Several quick and validated questionnaires, like the Asthma Control Test, have been published
Both B and C can be correct according to the above , but B is not specific for SABA , it could be steroid and
it's not one of the routine question you ask to asses response , so the most probable answer is C.
Source:
Uptodate:
an
overview
of
asthma
management
377long
term
treatment
of
sickle
cell
is:
Afolic
acid
Bpenicilline
C-hydroxurea
Answer : A
According to the uptodate , pencilline is recommended daily up to the age of 5.
Hydroxyurea
only
used
for
a
short
time
to
avoid
complications
"Folic acid supplementation is well established in the treatment of chronic hemolytic anemia. Although it
is proposed that folate in anemia raises hemoglobin levels and helps provide a healthy reticulocyte response, the use of folic acid in patients with SCD is not well supported by the primary literature."
Both folic and penicillin are used as long term but folic acid use can go until adulthood even though data
is not clear weather folic acid is required for management
378- 12-hour newborn developed jaundice. Which investigation will you order?
A.
Hb electrophoresis
B.
Osmotic fragility test
C.
G6PD screening
D.
G6PD
Answer:G6PD
According to uptodate.; "Most common causes of jaundice in the 24 hours of life are hemolytic anemias"
The DDx include ABO incompatibility, which is the most common cause, other differential includes spherocytosis and G6PD. Refer to uptodate for further DDx pathogenesis and etiology of unconjugated hyperbilirubinemia
in
the
newborn.
According to uptodate : "In pathological severe jaundice ,jaundice in first 24 H, or suspicion of hemolytic
disease require subsequent measurement(s) of Total Biluribin and further evaluation to determine the

smle ,2016

356
etiology
of
jaundice.
Initial tests that should be obtained
areBlood
type
and
direct
Coombs
test
Complete
blood
count
and
smear
Reticulocyte
count
- Glucose-6-phosphate dehydrogenase (G6PD) measurement, if clinically appropriate - Direct or conjugated
bilirubin
ETCOc
(if
available)"
Source: uptodate: evaluation of unconjugated hyperbilirubinemia in term and late preterm infants
Why not physiologic jaundice
He didn't mention this choice
And normal physiologic jaundice of the newborn typically appears between the 2nd and 5th days of life
and clears with time.
379- 8 years old boy with petechiae all over his body. Lab results: low platelets and high creatinine
level. what is the diagnosis?
a. ITP
b. TTP
Answer:
Explanation : Both ITP and TTP are diseases of platelet, but the median age of TTP is 42 and its a disease
of adults, extremely rare cases are reported for children, so this choice is probably excluded.
ITP is a disease of children that is usually self-limited and can be present with a wide spread petechiea and
it usually occurs 1 month after viral infection but renal involvement is not common.
A strong DDx that would be the probable answer is HUS. The patient will present with the triad of anemia
,
thrombocytopenia
and
renal
failure.
Sources:
uptodate
:
thrombocytopenia
in
children
Medscape
:
http://emedicine.medscape.com/article/202158-overview
http://emedicine.medscape.com/article/201181-overview
380- Newborn just delivered vaginally presented with respiratory distress, on examination: left side
silent and heart sounds heard on right side , what's the most likely diagnosis ?
1- diaphragmatic hernia
2- spontaneous pneumothorax
3- dextrocardia
Answer:A. CDH
Infants with congenital diaphragmatic hernia (CDH) most often develop respiratory distress in the first
few hours or days of life.Physical examination will reveal a barrel-shaped chest, a scaphoid appearing
abdomen because of loss of the abdominal contents into the chest, and absence of breath sounds on the
ipsilateral side. In most patients with CDH (because the lesion is on the left side), the heartbeat is displaced to the right because of a shift in the mediastinum. " + the most common type in diaphragmatic
hernia
is
left
posterior
(
left
silent
chest)
*In Spontaneous pneumothorax, there will probably be a hint that the newborn were on a ventilator, and
the
most
cardiac
finding
is
tachycardia
not
shifting
of
heart
sound.
*Dextrocardia
will
not
present
with
silent
chest
(
no
breath
sounds
)
Source:
Uptodate
:
congenital
diaphragmatic
hernia
in
the
neonate
Pneumothorax / physical exam section
381- 5 year old presented with abdominal pain + constipation, labs show low K, low Na , low Cl, most
likely diagnosis ?
1- bartter syndrome

smle ,2016

357
2-gitelman syndrome
3-congenital adrenal hyperplasia
4- congenital chloride diarrhea
Answer is bartter syndrome.
382- Sanrio about baby when birth weight was 3.5 kg after one week 3.1 kg
A.
Give Abotics
B.
Oral replacement therapy
C.
Reassurance
D.
Do routine investigation and send home
Answer:C
Explanation: " Term neonates may lose up to 10 percent of their birth weight in the first few days of life
and
typically
regain
their
birth
weight
by
10
to
14
days".
So
normally
is
weight
after
birth
can
reach
to
3.15
kg.
Source: uptodate: normal growth patterns in infants and prepubertal children.
383- Asthmatic boy on muntelukast present to ER, he has symptoms everyday exacerbated by exercise
What
to
give
for
maintenance
A.
oral
steroid
daily
with
long
acting
when
needed
B.
inhaled
steroid
twice
with
short
acting
when
needed
C.
Muntelukast
with
long
acting
Answer: B
384- y.o amenorrhea ,Short stature ,HTN,broad neck ,Also parent short stature dx:
AFamilial
BTurner
Answer: B
Explanation:
- Familial short stature usually occur in healthy children with no delay in bone age and normal growth
velocity
that
correlates
with
their
parents.
- The mentioned features are typical for turner, which is a result of a chromosomal defect; the presence
of
a
short
stature
in
one
of
the
parent
might
be
coincidental.
- If the short stature runs in the family that is always due to short stature of one of the parents with no
other complains think of the autosomal dominant ( achondroplasia : Dwarfism )
Source: short stature
385thymic (some thing)

Digeorge

syndrome

386- 4 y.o child , language development


a) Can say 50-100
b) 2 phrase / words sentences
c) Use past tense
d) Tell a story
Answer:D
Explanation " according to uptodate the child starts to tell stories at the age of 4, and later on he/she
would be able to retells stories with clear beginning , middle and end.

smle ,2016

358
Source uptodate: developmental behavioral surveillance and screening in primary care, Table ( developmental
milestones
4
through
8
yrs
A
)
387- young boy presented with diarrhea some time bloody, Wight loos, arthritis, anemia the diagnosis
is
:
A.
Chrons
B.
UC
C.
Celic
Answer: A
Both UC, and CD can present with the above features. Since CD is more common than UC. + some of the
the extra-intestinal manifestations are more common in CD, it would be the probable diagnosis
Quoting from uptodate " CD tends to present In late childhood or adolescence and its the predominant
form
of
IBD
after
the
age
of
8"
"
arthritis
is
twice
as
common
in
CD
as
compared
to
UC"
Source : uptodate: clinical features and diagnosis of inflammatory bowel disease in children and adolescents
3888
year
child
come
with
30
BMI
what
can
you
do
for
him?
A.
Surgery
for
obesity
B.
Give
him
drug
for
obesity
C.
Reassure
and
Live
style
Answer:
C
Explanation: For patients in the initial stages of obesity treatment (stages 1 and 2 above), we suggest that
the provider of primary care perform a brief clinical intervention, using the behavioral strategies, nutritional
goals,
and
exercise
goals.
Source
uptodate:
management
of
childhood
obesity
in
primary
care
sitting
Anti-obesity drugs can be useful adjuncts to diet and exercise for obese adults with a BMI greater than
30 kg/m2, who have failed to achieve weight loss goals through diet and exercise alone.
Source uptodate : drug therapy of obesity
389- Child presented with cyanosis and murmur ( case of the transposition of great vessel )
Answer:
VSD
http://emedicine.medscape.com/article/900574-clinical#showall
390- 4 years old SCD w/ recurrent VOC what is best management for
future improvement ?
A- Hydroxyurea
B- Multiple blood transfusions
Answer: A
391- Fragile x syndrome associated with
Answer: Large protruding ear, long face, high arched palate, hyperextensible
finger joint, double joint thumbs, flat feet, soft skin, post pubescent
macroorchidism , hypotonia, language deficit, autism, ADHA, strabismus,
seizures, fertility.
392- pediatric patient with abdominal distention and severe foul smelling flatus?

smle ,2016

359
A- Giardia lamblia*
393- child come with case of immunodeficiency with presented with lump in the groin and lap was given
what
is
the
diagnosis
??
there's
CH50
in
Q
A.
severe
combined
immune
deficiency
B.
chronic
granulomatous
disease
Answer:A
394- child presented with something in the eye with lab result of incense of WBC what is the diagnosis
A.
leukemia
B.
neuroblastoma
Answer:???
Not
clear
Q
But
:- According leukemia we will have high WBC and may papilledema in case of CNS involvement . - Regarding neuroblastoma may we have low WBC (Anemia or other cytopenias suggest bone marrow
involvement.) + eye manifestation will be Opsoclonus or Horner syndrome ) + Abdominal mass . Ref :First Aid step 2 CK
395- 2 y child had mass in flank lead to displace the collection system Dx?
A.
wilms
B.
neuroblastoma
Answer:B- neuroblastoma
-neuroblastoma
abdominal
mass
which
CROSS
MID
LINE
+
other
S&S
- wilms tumor usually we have Asymptomatic abdominal mass which Does Not CROSS MID LINE.
396- hemangioma in the lift eye in an infant and it needs to be resected so it doesn't affect the vision,
when
will
u
do
that?
A.
immediately
B.
2
weeks
C.
3
months
D.
6
months
Answer: immediately
Reference:
https://www.vbiny.org/photo-gallery/hemangiomas/eyelid-eyebrow

397child
9
y
..Rx
of
severe
avascular
necrosis?
Answer: ??!
Medical
and
surgical.
398child
e
recurrent
UTI
investigation
?
A.
u/s
..
B.
x-ray
C.
Ct
.
Answer:A
usually
start
by
US
then
Voiding
cystourethrography
(VCUG))
Ref
:Nelson
Ultrasonography
of
the
bladder
and
kidneys
is
recommended
for
infants
with
febrile
UTIs
to
exclude
structural
abnormalities or detect hydronephrosis. Voiding cystourethrogram (VCUG) is indicated if the ultrasound

smle ,2016

360
is
abnormal
(hydronephrosis,
scarring,
or
other
findings
to
suggest
obstruction or congenital abnormality). Vesicoureteral reflux is the most common abnormality found and
is ranked from grade I (ureter only) to grade V (complete gross dilation of
the ureter and obliteration of caliceal and pelvic anatomy) (see Chapter 167). A technetium-99m DMSA
scan can identify acute pyelonephritis and is most useful to define renal scarring as a late effect of UTI
399A
child
Answer: ALL
Reference:

present

with

s&s

of

Leukemia

with

CALLA

+ve?

Dx?

http://www.medicinenet.com/script/main/art.asp?articlekey=39568

400child
with
Painful
limping,
Dx?
Answer:
SCFH
12 Y +OBESE+ Cannot bear weight + hip externally rotated>>slipped capital femoral epiphysis
Answer:
SCFH

401- child with blue dot in testis and


A.
Incarcerated
B.
testicular
Answer:
testicular
Reference: Toronto note
402Answer:

smle ,2016

HUS

painful

mass

appendix
appendix

vs

in

inguinal

area?
hernia
torsion
torsion

TTP?

361

403- pedia case neonat has jaundice admitted due to physiological jaundice then discharge , still appear
jaundiced
what
is
case
?
duodenal
atresia
(Prolonged physiological jaundice associated with hypothyroidism and GIT obstruction )
Answer:

404- baby has sickle cell anemia and receive blood transfusion , what about vaccination ?
A.
take
vaccine

B.
don't
give
vaccine
Answer: A
Reference: http://www.mayoclinic.org/diseases-conditions/sickle-cell-anemia/basics/treatment/con20019348
405pediatric
case
croup
,
from
where
take
swap
A.
nasopharyngeal
B.
nasal
C.
pharyngeal
Answer:
nasopharyngeal

406- child in the hospital play and come to his parents say stories , draw head and hands and legs

what
is
the
age
of
this
A.
B.
C.
Answer:5
407A.
B.
C.

Child

smle ,2016

after

trauma

in

perineum
Penile
Prostate

region

showed

urine

?
swab
swab
swab
swab

child:
3yrs
4yrs
5yrs
years

extravasation
urethra
urethra
Ureter

362
D.
Answer: penile urethra

Bladder

408- 12 years old girl has type 1 DM , her weight 40 kg ( below 50th percentile ) and his height 150 cm
( below 95th percentile ) , she has no signs of secondary sexual characteristics of puberty, you want to
perform annual screening in clinic for :
A. ophthalmology
B. growth hormone ?!
C. Ct to renal
Answer: ??
409- 2y old baby only says mama and baba his brother with hx of delay speech until 3y.Speech and
language delay may be due to : hearing loss ,global developmental delay ,anatomical deficit eg cleft palate, environmental deprivation , familial. A hearing test and assessment by a speech and language therapist are the initial steps.
Source :illustrated 46) )baby presented with abdominal bloating and constipation , inv shows increase
ca+ .
I
forgot
the
choices
answer :
410- baby on antibiotic developed watery diarrhea , what is most likely organism :
A. c.perferingas
B. c.difficle
C.
rota
virus
answer :B
Refernce: http://www.uptodate.com/contents/antibiotic-associated-diarrhea-caused-by-clostridiumdifficile-beyond-the-basics

smle ,2016

363

411- parent brought their baby to your clinic , you noticed testicular asymmetry and was tender on
palpation , the testes were palpable in scrotum bilaterally , next step :
A.
transillumination
B.
surgery
C.
watch it while the baby is crying and coughing
answer :maybe C
412- baby came to u with sore throat , u obtain culture and send the baby home , the culture grows
meningococcus , u call the parents and they say their baby is asymptomatic , what will u do :
A.
10 days oral ampicillin
B.
one dose IM ceftriaxone sorry I forgot the rest of choices
answer :??
413- baby with 5 min after birth assessing ( HR 120 ,breath irregular and grasping , acrocyanotic , cough
and grimace , flexing all limbs not moving ) apgar score :
A.
6
B.
7
C.
8
D.
9
answer : B

414- baby presented with abdominal bloating and constipation , inv shows increase ca+ . I forgot the
choices
answer
:
not clear QQ
415- child admitted with sore throat and bilateral knee pain?
A.
Juvenile rheumatoid arthritis
B.
Rheumatoid arthritis
C.
Other choices I did not remember it
Answer
??
416- Osgood schlatter disease

smle ,2016

364
A.
Influenza
Answer:?? http://www.mayoclinic.org/diseases-conditions/osgood-schlatter-disease/basics/definition/con-20021911
417- child eating a lot of milk but he does not meat, MCV hypochromic microcytic anemia , How will
you manage this child?
A.
Oral vitamins + iron(I think)*
B.
Trial of iron then then observe
C.
Folic acid
D.
Iron
answer :D
EXPLANATION:
Exclusive milk can cause Iron deficiency anemia , the treatment is by iron of not receiving fortified cereals/meat/meat
alternatives
.
References :Toronto note
418- Child can walk without support ,crawling ,build 3 cubes point to something he interested in , so
what is the age of the child ?
A.
9month
B.
15 month*
C.
2
years
Answer: B
419- Child came by his parents , they said that their son have recurrent lip swelling(angioedema)so
deficiency of which ?
A.
Deficiency of C1 inhibitor*
B.
Deficiency
Anaphylactoid
inhibitor
Answer: A
Reference: uptodate & e-medicine & medscape. Com & master the board 3
420- Parent came to your clinic with their obese child with BMI 33 ,So they are afraid of having their
child a disease , they wanted you to do lipid profile , after taking history you decided to do a lipid profile
but why ?
A.
Because his parents wish or need this test to be done
B.
Because the child eating French fries daily
C.
Because there is early family history of cardiac disease*
Answer: C
Reference: e-medicine & medscape.com.
421- Which of the is mostly associated sign with croup:
A.
dysphonia
B.
Cyanosis
Answer :B
In
case
of
mild
to
moderate
is
In
sever
condition
Reference: is uptodate

smle ,2016

hoarseness
is

of

the

voice
cyanosis

365

422- 2 month old has diarrhea and his mother is worried from dehydration what will you advise the
mother:
A.
Change milk
B.
Oral
rehydration
solution
Answer:B
E-medicine
.medscape.com
Uptodate
423- A child presented with diaper rash with satellite lesion he was given local creams and steroid but
didn't work:
answer
:
Local
antifungal
Explanation : due to candida infection , they use sertaconazole cream (new alternative for diaper dermatitis
candidiasis
treatment
&
ciclopirox
more
safe
and
effective
Reference:
e-medicine
&
medscape.com
&
uptodate
424- child has itching and all student in his class got the same infection:
A.
Sarcoptes scabiei
Answer:
A
Scabies , it's skin infection caused by mite sarcoptes scabiei contagious condition , transmitted through
person
to
person
by
direct
contact
Reference: uptodate
425- What is the most common cause of facial cellulitis in pediatrics?
Answer:
Facial cellulitis in pediatric practice is now largely due to (1) trauma/loss of skin integrity with secondary
infection,
(2)
dental
problems,
or
(3)
severe
sinusitis.
These three underlying causes accounted for 89% of the cases of facial cellulitis seen at our hospital during the past decade. Although the numbers did not reach statistical significance, there was a trend toward
an increasing number of facial infections due to severe sinus disease. Facial cellulitis accompanying group
B streptococcal sepsis in newborns also was not seen during the past 10 years
http://www.medscape.com/viewarticle/439427_4
426- child c/o unilateral nasal foul smelling discharge for two weeks ,what is your treatment?
A.
x-ray of the head
B.
antibiotics
Answer: A
Explanation: due to determine the kind of the foreign body , if appear as radiopaque that's mean it's
metal
object
Then
you
assess
how
to
remove
it.
The role of plain x-ray is very limited as the condition need to and only can be confirmed under direct
visualisation
with
endoscope
It may be very helpful in primary care setting as this bony concretion will demonstrate some degree of
opacity
thus
aid
the
working
diagnosis
And
demonstrate
the
exact
lesion

smle ,2016

366

http://www.ncbi.nlm.nih.gov/pmc/articles/PMC4170447/
http://www.medscape.org/viewarticle/743408_2
427- hydrops fetalis in thalassemia case :
A.
Normal 2 beta abnormal 4 alpha
B.
abnormal 2 beta normal 4 alpha
C.
Normal 4 beta abnormal 2 alpha
D.
Abnormal 4 beta normal 2 alpha
Answer:A
Uptodate
It's
alpha
thalassemia
major
,absent
of
4
alpha
Because there is no alpha globin chain production, affected fetuses have great difficulty in synthesizing a
functional hemoglobin molecule. Gamma chains accumulate and form gamma-4 tetramers (hemoglobin
Bart's); hemoglobin Bart's binds oxygen, but cannot release it to tissues because its affinity for oxygen is
much greater than that of hemoglobin A. Hemoglobin electrophoresis will show greater than 80 percent
hemoglobin Barts while the remaining hemoglobin will be of embryonic origin, with a small component of
hemoglobin H (beta-4 tetramers)
428- most sexual differentiation in male :
A.
micropenis
B.
hypospadias
Answer:B
Common condition is hypospadias
http://www.urologyhealth.org/urologic-conditions/hypospadias
429- 11 months old infant , dark foul smell stool mixed with blood and mucus , what will u do investigation :
A.
Something like technetium scan .
B.
Ultrasound
answer : B
Reference:
http://emedicine.medscape.com/article/930708-overview
430- newborn with umbilical hernia.what should tell the mother ?
A.
Reassure Should treat it before school
answer : A
Reference:
http://www.mayoclinic.org/diseases-conditions/umbilical-hernia/basics/treatment/con20025630
431- baby with vomiting and diarrhea , looks ill , cries with tears , capillary refill 3 sec , vital signs were
normal almost :
A.
moderate dehydration give something 10 ml infusion
B.
moderate dehydration give
C.
sever dehydration give IV fluid
D.
sever dehydration give another something (couldn't remember the exact sentences but these
were the choices )

smle ,2016

367
Answer:C
http://www.ahcmedia.com/articles/9698-pediatric-dehydration-assessment-and-oral-rehydrationtherapy
432- years old baby presented with severe lower limb pain , growth parameters under the 5th percentile , low Hg :
A.
Osteomyelitis
B.
Vaso Occlusive crisis
Answer:B
http://emedicine.medscape.com/article/205926-overview#a1
433- baby with blood jelly stool ( was case of intussusception ) what will u do :
A.
US
B.
Barium enema
Answer:A
Ultrasound. A diagnostic imaging technique that uses high frequency sound waves and a computer to
create images of blood vessels, tissues, and organs. Ultrasounds are used to view organs as they function
and
to
assess
blood
flow
through
various
vessels.
Barium enema can be used for treatment as well http://www.stanfordchildrens.org/en/topic/default?id=intussusception-90-P02002
434- 8 years old girl , parent complains that she looks older than her classmates , wt and height above
the 95th percentile , otherwise normal :
A.
reevaluate after 12 months
B.
obesity medications
C.
life style modification
D.
surgical intervention .
Answer:A
435- 15 yo female , no period still , on examination slight breast buds with wide spaced areola , fine
pubic hair on labia majora , (not sure if they mention a normal growth parameters in the question and
unfortunately
I
forgot
the
choices
answer
:
436- newborn after 2weeks c/o bilateral conjunctivitis ,chest infiltration , lung crepitation, what is organism ??
A.
chlamydia
Answer:A
https://pedclerk.bsd.uchicago.edu/page/chlamydia-newborn
437- what is the features that seen in pt with congenital adrenal hyperplasia?
A.
hirsutism,
B.
Dehydration
answer
:
http://emedicine.medscape.com/article/919218-overview
438developmental
answer :

smle ,2016

score

that

depend

solely

on

parents

hirsutism

information

368

439- child ate honey then he develop flaccid paralysis :


A.
clostridium botulinum
answer
clostridium
http://emedicine.medscape.com/article/961833-overview

:
botulinum

440- child his height and weight below normal besides growth hormone what you will order:
A.
somatomedin C
B.
aldosterone
C.
insulin
D.
testosterone
Answer:C
441- Say few words at the age of which ?
A.
24
B.
12m
Answer:B
442- what u will seen on physical examination of pt with croup ??
A.
presence of inspiratory sounds
B.
presence
of
expiratory
answer
:
Stridor
with
http://emedicine.medscape.com/article/962972-overview

wheeze
inspiration

443- newborn girl ( i did not remember the complain but there is enlargement of clitoris ( I think
this
is
congenital
adrenal
hyperplasia
)
ask
about
treatment
?
A. fluid
B.Hydrocortisone
answer : it's CAH and the treatment is long-term glucocorticoid or aldosterone replacement
http://emedicine.medscape.com/article/919218-treatment
444- Boy 10 yrs BMI 30 , wt &ht above 95 percentile what is the best managem ?
A.
Revert to surgery
B.
Lifestyle modification
C.
Re evaluate after 12 m
answer
:
Lifestyle
modification
http://www.nhlbi.nih.gov/health-pro/guidelines/current/cardiovascular-health-pediatric-guidelines/full-report-chapter-10
445- 18 months old girl delivered premature her wt was 2.6kg .she is healthy but last 2 days c/o irritability and fatigue .mother shift feeding from breast feed to cow milk feeding at age of 9 month , she
has hypochromic microcytic anemia ,
this patient complain due to ?
A.
premature
B.
cow milk feeding
C.
bone marrow defect
answer : B

smle ,2016

369
https://en.wikipedia.org/wiki/Iron-deficiency_anemia#Diet
446- read about epstien bar virus?
447- Read about kawasaki disease?
448- Read about mononucleosis?
449- viral gastroenteritis prevented by which vaccine :
A.
Rota
Answer: A
450- 25 kg child maintenance 24h :
The
Maintenance
Half
800
ml
given
in
1st

..the

vaccine

is
rest

is

given

1600
in

2nd

16

ml
h

451- child develop petechiae in leg hx of URTI LAB : normal CBC expect plt was low next management?
A. plt transfusion
B. immunoglobulin
answer: B ITP
answer: Its ITP 1st choice in treatment is corticosteroids then immunoglobulin
http://emedicine.medscape.com/article/803885-overview
452- type of pneumococcal conjugate 13 :
A. live
B. conjugate
C. inactive
answer: B
Pneumococcal polysaccharide vaccine
Vaccine description
Target disease Streptococcus pneumoniae
Type
Conjugate vaccine
https://en.wikipedia.org/wiki/Pneumococcal_conjugate_vaccine
453- child complains of abdominal pain since 2 weeks , diarrhea occasionally bloody. malaise. what is
the most appropriate test for the diagnosis?
A.abdominal CT
B.barium enema
C.abdominal US
Answer: A ??
454- 3 years old came to the ER with dry cough, he was crying and hoarseness of voice was present,
what is your management?
A. oxygen mask
B. nebulized cool mist
Answer: B

smle ,2016

370
Nelson

Essentials

of

Pediatrics

7th

edition

Page

No.

355

455- child presented to the emergency room with non productive cough and inspiratory stridor. what
is the worst sign we should worry about?
A.bluish lip color
B.expiratory stridor
Answer: A
Step up to pediatrics pages No. 268-269
456- child vaginal bleeding + breast +mass in pelvic?
A-ovarian teratoma
B-granulosa theca
C-yolk sac tumour
Answer: B Patients usually present with precocious pseudopuberty (70-80%) and have secondary sex
characteristics at a very early age. These may include increased linear growth, breast enlargement, clitoral
enlargement, pubic hair development, increased vaginal secretions, and vaginal bleeding.
457- baby diagnosed with Cystic fibrosis. ..he has + sweat chloride test his brother is normal, to confirm
diagnosis of cystic fibrosis?
A- CTRF gen in parent
B- CTRF in sibling
C- Chloride test. .parent
D- Chloride test in sibling
Answer: A suppose to be chloride testing in child
Cystic fibrosis is a recessive disorder, which means that both parents must pass on the defective gene for
any of their children to get the disease. If a child inherits only one copy of the faulty gene, he or she will
be a carrier. Carriers don't actually have the disease, but they can pass it on to their children.
http://learn.genetics.utah.edu/content/disorders/singlegene/cf/
458- Purulent discharge from newborn's eye, what's the organism?
A.
gonorrhea ( by student get 100% in OPH )
Most
likely
Dx.
is
neonatal
conjunctivitis
Could be Gonococcal infection neisseria gonorrhoeae .( within first 48h of life )
Or could be chlamydia trachomatis eye infection ( at 1-2 weeks of age )
Illustrated
textbook
of
pediatrics
4th
edition
page
No.
175
459- 3 years child with UTI . What investigation should done before Tx?
A.US urine sample
B.urethral culture
Answer: A

smle ,2016

371

460- Q about Cerebral palsy with typical feature patient had spastic paralysis of all limbs except upper
limbs had less paralysis. What type of CP the baby had:
A- mixed
B- Diplegic
C- hemiplegic
D- Quadriplegic
Answer: B http://www.michigancerebralpalsyattorneys.com/about-cerebral-palsy/

smle ,2016

372

smle ,2016

373

461- pediatric pt k/c of Asthma since age 2 years he is now asymptomatic with rare uses of albuterol
inhalers, he came for counseling, he had contact sport participation, what is best question to ask to
know his response:
A- Are keeping up with your friends
B- How frequent do use inhaler
C- presence night symptoms (cough)
Answer: C
462- Have you had difficulty sleeping because of your asthma symptoms (including cough)?
463- Have you had your usual asthma symptoms during the day (cough, wheeze, chest tightness or
breathlessness)?
Has your asthma interfered with your usual activities (housework, work/school etc)

smle ,2016

374
464- young boy presented with diarrhea sometimes bloody , Weight loss , arthritis ,anemia the diagnosis is :
a. Crohns
b. UC
c. Celic
answer: B
Illustrated textbook of pediatrics 4th editionn page No. 236
465- child young present with his family (pic of baby his age is months head tilted to one side) he was
normal on birth with normal not complicated delivery ,when you try stretch sternocleidomastoid he
was crying all reflexes and movement normal?
A- Cervical rib
B- Infant Torticollis
Answer: B
http://www.childrenshospital.org/conditions-and-treatments/conditions/torticollis/symptoms-andcauses

466- child mild persistent asthma visit ER once every month he is on ventolin what to add to his medication?
Answer: Low dose CS according to Asthma TTT scale
Nelson essential p 277
467- Child with dehydration, depressed anterior fontanel, and decreased skin turgor. What is the percentage of dehydration?
A- 5
B- 10
C- 15
D- 20
Answer:B
http://www.uptodate.com/contents/image?imageKey=PEDS%2F76198&topicKey=PEDS%2F6142&source=see_link
468- deceleration in fetal assessment?
A- Good prognostic factor
B- Bad prognostic factor
answer: B
Deceleration means decreased HR in Fetal

smle ,2016

375
469- baby recently 6 months ago diagnosed w/ DM type 1 compliant to medication and diet start to
have episodes of hypoglycemia not in particular time what is the cause ?
A- honeymoon period
B- Dawn phenomenon
C- Sigmoid phenomena
Answer: A
http://www.diabetes.co.uk/blood-glucose/honeymoon-phase.html
470- baby can sit roll from prone to supine and back play handle object but can't pick things b/ 2 fingers
age
4 months
6 months
8 months
answer: B
Reference
:
http://2.bp.blogspot.com/QYxe36daT4o/UFvu_fHzN9I/AAAAAAAAABQ/e46pvQmRZcY/s1600/developmental+milestones.jpg
471- baby after operation has loss of gag reflex in left side, normal uveal movement what nerve injured
?
A- Glossopharyngeal
B- Vagus
Answer:A
Glossopharyngeal nerve lesions produce difficulty swallowing; impairment of taste over the posterior
one-third of the tongue and palate; impaired sensation over the posterior one-third of the tongue, palate,
and pharynx; an absent gag reflex; and dysfunction of the parotid gland.
472-What is more present in Cow milk than breast milk
A-proteins
B-fat
C-carbs
D-calories
Answer:A

smle ,2016

376
473- non pruritic pink eruption of the right foot no scales no history of infection
Answer : Granuloma annulare
The
scenario
is
not
enough
complete
to
sure
about
So
read
about
granuloma
http://emedicine.medscape.com/article/1123031-overview
https://www.youtube.com/watch?v=HgDxCOEq4mg

the

answer
annulare

474- newborn presented with vomiting and enlarged clitoris ,lab was included showing hypokalemia
and hyponatremia. what is the most likely diagnosis
A- congenital adrenal hyperplasia
Answer : A if its hyperkalemia not hypo.
In CAH Genitalia are ambiguous in girls , boy do not initially exhibit any abnormality but begin to loss their
finding
sexual
futures
as
they
age
.
Inappropriate facial hair , fertilization and menstrual abnormalities are seen .
Hyponatremia
,
hypochloremia
,
hypoglycemia
and
hyperkalemia
are
seen
Reference : Master the board ..
475- Read about viral hemorrhagic disease?
476- days neonate ,, with pale stool , high conjugate ,, diagnosed with neonatal jaundice and treated
with fluorescence what is the cause
child complaining of bilateral knee pain after that he developed purple rash over lower limb Dx??
A.
HSP

1.
HSP
HSP can be diagnosed if at least two of the following are present (adapted from
American
College
of
Rheumatology
criteria):

smle ,2016

377

Age
<
20
years
Purpuric rash without thrombocytopenia, usually in dependent areas such as legs and buttocks

Abdominal
pain
or
bloody
diarrhea
indicating
bowel
wall
ischemia
Biopsy (e.g., skin, kidney) showing granulocytes in the walls of blood vessels
Other common findings include arthritis and arthralgias in large joints; glomerulonephritis, which occurs
in one-third of cases and usually resolves but can result in renal failure (kidney biopsy may be required);
and
orchitis
Reference
:The
Washington
Manual
of
Pediatrics
477- Dehydration child 8 month tttt
ORS oral rehydration solution

478- What is buline ???


479- clear case of DKA in a child what the initial management?
A - electrolytes replacement
B - Fluid replacement
C - Bicharb
D - Insulin infusion
Answer:B

smle ,2016

378

480- 4 days baby present with bilious vomiting , he had abdominal distention , poorly feed on examination abdominal distention (no other findings in examination in the q)
A - Mid Volvulus
B - Allergic to formula
Answer:A
481- child with symptoms of DKA ,, ABG ph 7.24 ,, Pco2 lower than normal ,, HCO3 lower than normal,
What is it?
A - compensated metabolic acidosis
B - Compensated metabolic alkalosis
C - Uncompensated metabolic acidosis
D - Uncompensated metabolic alkalosis
Answer: C

482- 15 month baby LP show gram + cocci in chain what Abx?


A- Vancomycin alone
B- Ampicillin alone
C- Ceftriaxone and vancomycin
D- Ampicillin and gentamicin

smle ,2016

379
Answer:C
bacterial meningitis is a medical emergency: do not delay antibiotics for CT or LP

empiric
antibiotic
therapy
age <6 wk: ampicillin + cefotaxime IV OR ampicillin an aminoglycoside IV; add
vancomycin
if
suspect
S.
pneumoiae

6
wks-3
mo:
ampicillin
+
cefotaxime
+
vancomycin

age
>3
mo:
vancomycin
+
cefotaxime
OR
ceftriaxone
IV
add ampicillin IV if risk factors for infection with L. monocytogenes present: age >50,
alcoholism,
immunocompromised
steroids in acute bacterial meningitis: dexamethasone IV within 20 min prior to or with first dose of
antibiotics

continue
in
those
patients
with
proven
pneumococcal
meningitis
not recommended for patients with suspected bacterial meningitis in some resource-limited countries

not
recommended
for
neonatal
meningitis
Reference : Toronto Note
483- 4 month baby , sleep all the time , not cry , has jaundice and umbilical hernia
A - Hypothyroidism
B - Congenital adrenal hyperplasia
Answer: Most likely its A
484- adolescent (12-14) with headache like a band , Other characters, he has stress in school?
A.
tension headache
Answer: A
485- newborn is irritable and sweating, chest is clear, vitals was provided HR 300 beat/min , what is
your action:
A.
cardiac dextroversion
B.
vagal massage
C.
digoxin
Answer:B
haemodynamically
stable,
consider
the
following:

Vagal
manoeuvres:
ice
bag
to
face
for
1520s
or
unilateral
carotid
massage
or
Valsalva
manoeuvre.
Do
not
compress
orbits.

Adenosine:
50100micrograms/kg
initially,
as
rapid
IV
push.

DC
shock:
synchronized
countershock
1J/kg
should
be
reserved
for
the
haemodynamically
unstable.
Intubation
and
appropriate
analgesia
and
sedation
are
required.

Other
drugs:
amiodarone,
procainamide,
fl
ecainide.
Reference
:
Oxford
Handbook
of
Pediatrics
486- 90% of children in a village have cretenisim iodine supplement???
487- read ttt of hirschsprung disease and see picture for x-ray 2 Q about ttt

smle ,2016

380

Abdominal radiograph demonstrating small bowel obstruction and megacolon in infant with Hirschsprung's Disease.
To
read
about
management:
http://emedicine.medscape.com/article/929733-treatment
488- neonate with duodenal obs ? What the sign ? Bubble !!!
characteristic finding of duodenal obstruction is the double-bubble image of an air-filled stomach proximal
to
an
air-filled
first
portion
of
the
duodenum.
Reference: http://emedicine.medscape.com/article/932917-workup#c6
489- Perth's disease .(4-8 y)

smle ,2016

381
http://orthoinfo.aaos.org/topic.cfm?topic=a00070
490- child ingested iron what is treatment :
A.
Gastric lavage acute mangment
B.
active charcoal not bind to iron salt deferoxamine.
C.
late mangment
Answer:B
Reference: http://emedicine.medscape.com/article/1011689-treatment#d7
491 - boy goes Camping when he&#39;s back he came with constitutional symptoms lymph nodes enlargement and took penicillin developed rash what does he have
A.
I chose mononucleosis I think it's right ??
492- A child present with s&symptoms of Leukemia with CALLA +ve?
Dx? ALL
Reference:http://www.ncbi.nlm.nih.gov/m/pubmed/2521388/
493- child with Painful limping, Dx?
DDx:
Ankle
Fracture
Ankle
Injury,
Soft
Tissue
Appendicitis
Brain
Abscess
Catscratch
Disease
Erythema
Multiforme
Femur
Fracture
Foot
Fracture
Gout
and
Pseudogout
Heavy
Metal
Toxicity
Hemophilia,
Type
A
Hemophilia,
Type
B
Hip
Fracture
in
Emergency
Medicine
Inflammatory
Bowel
Disease
Insect
Bites
Knee
Fracture
Legg-Calve-Perthes
Disease
in
Emergency
Medicine
Mechanical
Back
Pain
Nongenital
Warts
Pediatrics,
Meningitis
and
Encephalitis
Pelvic
Fracture
in
Emergency
Medicine
Rheumatic
Fever
in
Emergency
Medicine
Rheumatoid
Arthritis
Sickle Cell Anemia
494- child come with case of immunodeficiency with presented with lump in the groin and lap was
given what is the diagnosis ?? there is CH50 in
a. severe combined immune deficiency
b. chronic granulomatous disease
Answer:A

smle ,2016

382
495- Child can roll over, sit triploid, attempt to take object Which month?
A. 6
B. 9
C. 2
answer : A
496- infantile colic
A) decreased peristalsis
B) increased gases
Answer: B by exclusion.
https://en.wikipedia.org/wiki/Baby_colic
497- Boy fight with 2 boys what system activated?
A. Sympathetic
B. Parasympathetic
Answer: A
http://www.health.harvard.edu/search?q=Understanding+20the+Stress+20response
498- pediatric patient complain of cough he control it by leukotrienes when he visited the primary physician 6 month ago. now he develops cough for 4 days after exercise they give him albuterol the the
symptoms were relieved what you gonna give him:
A- luktranse + short act b-2 agonist
B- oral steroid + short act b-2 agonist
C- long act b-2 agonist + short act b-2 agonist
Answer:A
http://www.mayoclinic.org/diseases-conditions/exercise-induced-asthma/basics/treatment/con20033156
499-Children diagnosed to be hypertensive :
a) BP Above 90th percentile
b) BP Above 95th percentile
answer : A
Fourth Report introduced a new category, prehypertension, which is diagnosed when a childs average BP
is above the 90th percentile but below the 95th. Any adolescent whose BP is greater than 120/80 mm Hg
is also given this diagnosis, even if the BP is below the 90th percentile. This classification was created to
align the categories for children with the categories for adults from the recommendations of the Seventh
Report of the Joint National Committee on Prevention, Detection, Evaluation, and Treatment of High Blood
Pressure.
Stage I hypertension is diagnosed if a childs BP is greater than the 95th percentile but less than or equal
to the 99th percentile plus 5 mm Hg. Stage II hypertension is diagnosed if a childs BP is greater than the
99th percentile plus 5 mm Hg. It may be categorized as prehypertension if the BP is between 90th to 95th
percentile.
500- least cardiac anomaly associated with infective endocarditis?
A. ASD
501- chiLd drink cow milk his hemoglobin low and MCV low which type of anemia he will has:
-Iron deficiency anemia

smle ,2016

383
Answer: Iron deficiency anemia
502- milestone > baby healthy run to the doctor play a role model as his father cant complete a sentence can't eat with spoon
Answer: 10 months ??
503- child sit and support his head , laughing and cooing :
A- 4
B- 6
C- 8
D- 16
Answer:C
504- case of fanconi syndrome
Answer:http://emedicine.medscape.com/article/981774-overview
505- case (young or kid) bilateral knee pain then rash starts on legs, thighs and buttocks = typical
Answer: HSP
506- child with sickle cell, what is the lifelong treatment to prevent infections?
A- penicillin and immunization.
https://www.cdc.gov/ncbddd/sicklecell/healthyliving-prevent-infection.html

507- child with pain and swelling in his hands and foot (sickle cell disease),. forgot the question..
508- child with vesicles in his oral mucosa, what is the diagnosis?
A- herpes simplex type 1.
http://emedicine.medscape.com/article/218580-clinical#b4
509- child who's lethargic and losing his concentration, Hgb is 10.5, what to give?
A- IM iron.
B- oral ferrous sulphate.
C- fortified cereal.
Answer: B
the patient is symptomatic, and the initial treatment for IDA is oral Ferrous Sulfate
http://www.fpnotebook.com/hemeonc/peds/pdtrcanm.htm
510- What is the injection that is routinely given to newborn to inhibit haemorrhage:
a) Vitamin K
b) Vitamin C
c) Vitamin D
d) Vitamin E
answer: A
511- Child with URTI is complaining of bleeding from nose, gum and bruising the treatment is:
a) Prednisolone

smle ,2016

384
b) IVIG
answer : a ??
512-Cellulitis occurring about the face in young children (6-24 months) and associated with fever and
purple skin discoloration is MOST often caused by
a) group A beta hemolytic streptococci
b) Haemophilus influenzae type B
c ) streptococcus pneumoniae
d) staphylococcus aureus
e) pseudomonas
answer : A ??
513- 4 years old brought by his parents, height < 5th percentile, they ask if he will remain short. what
you will do initially :
a) Parental height
answer : i think check Somatmedin C
514- 8 years old girl presented with fever, numerous bruises over the entire body and pain in both legs.
Physical examination reveals pallor and ecchymosis and petechiae on the face, trunk and extremities.
Findings on complete blood count includes a haemoglobin of 6.3 g/dl, white cell count of 2800/mm3
and platelet count of 29,000/mm3. Which of the following would be the MOST appropriate treatment?
answer : Emergency treatment

Although rare, severe bleeding can occur with ITP. Emergency care usually includes
transfusions of platelet concentrates, intravenous corticosteroid (methylprednisolone)
and intravenous immune globulin. http://www.mayoclinic.org/diseases-conditions/idiopathicthrombocytopenic-purpura/diagnosis-treatment/treatment/txc-20201329
515- Child with mild trauma develop hemarthrosis, in past history of similar episode DX ?
a) Platelets dysfunction
b) Clotting factor deficiency
Answer : B hemophilia
516-years old his parents has TB as a pediatrician you did PPD test after
72 hr you find a 10mm induration in the child this suggest
a) Inconclusive result
b) Weak positive result
c) Strong positive result
answer: C
http://www.webmd.com/a-to-z-guides/tuberculin-skin-tests?page=3
517- Child with aspirin intake overdose ...what kind of acid base balance:
a) Metabolic alkalosis
b) Metabolic acidosis
c) Respiratory alkalosis
d) Respiratory acidosis

smle ,2016

385
answer: B
Acid-Base Disorders in Salicylate Toxicity Adults: Metabolic acidosis AND Respiratory alkalosis
Children: Metabolic acidosis
If fasting=>starvation ketosis may develop
Salicylates directly stimulate the respiratory center to cause hyperventilation (respiratory alkalosis) which
is dose-dependent. This stimulation is much more pronounced in adults than in children.
http://www.anaesthesiamcq.com/AcidBaseBook/ab8_6c.php
Initial respiratory alkalosis followed by metabolic acidosis
the most common abnormality, especially in adults, is a mixed acid-base disturbance (a primary respiratory alkalosis plus a primary metabolic acidosis)
518- Child with high-grade fever for 5 days and sore throat, on examination there was tonsillitis and
white patches on the gingiva. No LN enlargement, ASO is negative. The most likely causative organism
is:
a) Coxsackie virus.
b) Herpes simplex virus.
c) EBV.
answer : B
References:
http://www.entnet.org/content/tonsillitis
http://patient.info/doctor/tonsillitis-pro
519- child with generalized swelling, long scenario of nephritic syndrome you are suspecting minimal change nephropathy, what you will find in the biopsy?
(options are long "2 lines long for each")
answer: -ve EM , focal fusion , loss of foot process
In children with frequently relapsing MCD, some involuted glomeruli may be present. These lesions are
small and sclerotic but retain their podocyte and parietal epithelial cell constituents. The presence of these
glomeruli
is
related
to
the
duration
of
the
disease.
The most common tubular lesion is protein and lipid droplets in epithelial cells due to increased reabsorption. The presence of areas of tubular atrophy and interstitial fibrosis should raise the suspicion of FSGS.
Immunohistology
These studies usually do not demonstrate significant glomerular deposition of immunoglobulins or complement components in patients with MCD. Some biopsy specimens may be positive for low-level IgM
deposits
not
accompanied
by
mesangial
dense
deposits.
Electron
microscopy
Retraction of the epithelial foot processes is observed consistently in patients with MCD. This is, at times,
erroneously described as foot-process fusion and results from disordered epithelial cell structure with
withdrawal of the dendritic process)
520- child with cola colored urine, which test should you perform first?
A- urinary microscopy.
B- renal function test.
C- renal biopsy.
Answer: A- urinary microscopy.
Reference: Http://www.uptodate.com/contents/blood-in-the-urine-hematuria-in-children-beyond-thebasics?source=outline_link&view=text&anchor=H1#H1:

smle ,2016

386
521- child with delay in walking, on examination there is bowing in his legs, labs showing normal ca+2
normal phosphor and elevated alkaline phosphatase, what is the diagnosis?
A- Rickets.
Answer : A
calcipenic rickets is often but not always associated with low serum calcium levels, while phosphogenic
rickets is characterized by low serum levels of phosphorus. Early on in the course of rickets, the calcium
(ionized fraction) is low. However, this level is often within the reference range at the time of diagnosis,The phosphorus level is invariably low for age, unless recent partial treatment or recent exposure to
sunlight has occurred. Alkaline phosphatase levels are uniformly elevated.
Reference:
http://www.uptodate.com/contents/overview-of-rickets-in-children?source=outline_link&view=text&anchor=H1#H1
http://emedicine.medscape.com/article/985510-workup#c8
522- CPR in child according to American heart association in presence of 2 rescuer:
a) 15 compression and 2 ventilation
b) 30 compression and 2 ventilation
answer: A
523- child with oral and tonsillar ulcers and vesicles, fever. Dx:
A.
Herpangina
Answer : Though herpangina can be asymptomatic, symptoms usually associated are high fever and sore
throat. A small number of lesions (usually 2 - 6) form in the back area of the mouth, particularly the soft
palate or tonsillar pillars. The lesions progress initially from red macules to vesicles and lastly to ulcerations which can be 2 4 mm in size. The lesions heal in 7 10 days.
Reference: http://www.medicinenet.com/script/main/mobileart.asp?articlekey=125686&page=2
524- Neonate was on breast feeding after 2 weeks has irritability and decrease feeding diagnosed with
meningitis .. What is the organism ?
A- Neisseria
B- listeria
C- streptococcal pneumonia ( i chose listeria )
Answer: B
GEL
G: GBS
E: E.coli
L: Listeria
http://emedicine.medscape.com/article/1176960-overview#a5
525- Child presented to Er with hx of testicular pain , what is next step?
1 - u/s
2- surgery counseling
Answer:A
Reference: https://www.auanet.org/education/acute-scrotum.cfm
526- Neonate take immunoglobulin from his mother this is
A- Active Artificial immunity
B- Passive artificial immunity
C- Active neutral immunity

smle ,2016

387
D- Passive neutral immunity
ANSWER : D
527- What measurement you should take to relieve an infant abdominal colic?
A- antispasmodic drugs
B- increase bottle feeding
C- warm baths *
D- prevent child abuse
ANSWER : C mooost likely as all the choices is not recommended .. See the link below
Reference: http://www.medicinenet.com/script/main/mobileart.asp?articlekey=325&page=4
528- days neonate present with lethargy , irritability , fever , signs of meningitis which organism is
causative :
A-listeria monocytogenes
B-strept pneumonia
C-staph aureus
D-N-meningitidis
ANSWER : A
529- What is most common heart lesion in Down syndrome ?
VSD
Reference: http://emedicine.medscape.com/article/943216-overview
530- neonate with sign of sepsis which empirical antibiotic :
A-ampicillin
B-gentamicin
C-cefotaxime
ANSWER A
Ampicillin is a beta-lactam antibiotic that is bactericidal for susceptible organisms, such as group B Streptococcus (GBS), Listeria, nonpenicillinase-producing Staphylococcus, some strains of Haemophilus influenzae, and meningococci. Some publications recommend ampicillin (in combination with gentamicin) as
first-line therapy for suspected sepsis in the newborn.
531- child in the school K/c of DM loss of copiousness , last insulin dose not known , what should you
do:
A-give IV Dextrose
B-SC insulin
C-urgent transfer to hospital
ANSWER : A
532- Most common cause of death in sickle cell anemia :
A- Aplastic crises
B- Sequestration crises
C- Acute chest syndrome
D- Parvovirus b19
ANSWER : C
Reference:
Http://www.nytimes.com/health/guides/disease/sickle-cell-anemia/complications.html

smle ,2016

388
533- Child presented with brun in the upper right extremity with bluster what is the degree of the burn
A- 2nd degree more than 15 %
B- 2nd degree less than 15 %
C- 3rd degree more than 15 %
D- 3rd degree less than 15 %
ANSWER B
Reference: http://www.stanfordchildrens.org/en/topic/default?id=second-degree-burns-partial-thickness-burns-90-P01757&sid=
http://www.emtresource.com/emergencies/burns/rule-of-nines/
534- Best antibiotic for breast feeding is ?
A- Chloramphenicol
B- Azithromycin
C- Cimetidine
D- Ciprofloxacin
ANSWER : B
this antibiotic is generally considered safe to take while breastfeeding. Zoloft: Hale says about 15 percent
of breastfeeding moms are taking an antidepressant, and Zoloft is considered the best choice by many
experts,
since
not
much
of
the
drug
passes
into
mom's
milk.
535- case : mother come to doctor because worry About her child may be had dehydration what is
most tool exam of child ?
1-history
2- clinical
3- C.T
4- ultrasound
answer: A
Reference: Obtaining a complete history from the parent or caregiver is important because it provides
clues to the type of dehydration present. http://emedicine.medscape.com/article/801012-clinical
536- Child after trauma in perineum region showed urine extravasation
A- Penile urethra
B- Prostate urethra
C- Ureter
D- Bladder
ANSWER : A
Retrograde urethrogram reveals the extravasation of contrast material from the urethra into the penile
soft tissues, indicating urethral injury
537- Child with bluster in trunk what dx ?
A- herpes simplex i think
B- varicella
C- impetigo
Answer:C
Bullous
impetigo:
Occurs
mainly
in
children
<
2
yr
Painless, fluid-filled blistersmostly on the arms, legs, and trunk, surrounded by red and itchy skin
which,
after
breaking,
form
yellow
or
silvery
scabs

smle ,2016

389
Reference:
Excellent
Table
for
pediatrics
Rashes-https://www.dropbox.com/s/v6jhyba9ppdkt1t/Rash%20in%20Infants%20and%20Young%20Children%20-%20Pe...s%20%20Merck%20Manuals%20Professional%20Edition.pdf?dl=0
538- Child with his purge disease what to do ?
Answer: Cognitive-behavioral therapy (CBT) is the treatment of choice for bulimia nervosa.
Other
therapies
that
may
be
helpful
include
the
following:
Interpersonal
psychotherapy
(IPT)
Nutritional
rehabilitation
counseling
Family
therapy
Group therapy
539- 10 years old child with diarrhea what is the correct about oral rehydration solution ?
A.
start with 50 in first 4 hour then maintenance 100 per day
B.
start with 50 in first 4 hours then maintenance 50 per day
C.
start with 100 in first 4 hours then maintenance 100 per day
answer : A
540- modified duke's criteria :
1 major 2 minor+ PLUS .
1major 3 minor
Answer: 2 Major OR 1 Major and 3 Minor OR 5 Minors

541- Thump sign what treatment ? (thumb sign )


Answer:
epiglottitis case
Tx :
1- securing airway

smle ,2016

390
2- start Abx empiric until result of the culture and sensitivity come
( 3rd generation cephalosporin e.g. Ceftriaxone ) + antistaphylococcal agent against MRSA e.g. Clindamycin )
Reference : Uptodate
http://www.uptodate.com/contents/epiglottitis-supraglottitis-treatment-and-prevention?source=search_result&search=epiglottitis&selectedTitle=2~44
542- Child high indirect bilirubin?
Answer : It could be crigler najjar syndrome or gilbert syndrome
543- Painless genital ulcer + lymph nodes enlargement
A- primary syphilis
B- secondary syphilis
Answer : A
Ref.:
http://www.aafp.org/afp/2012/0201/p254.html
Single, painless, well-demarcated ulcer (chancre) with a clean base and indurated border with Mild or
minimally
tender
inguinal
lymphadenopathy
544- X- ray picture ( identical to this picture) , with history of 8 years male with Lt hip, appear externally
rotated and there are decrease in range of motion ,decrease in the abduction of the hip.
What is your diagnosis :
a.
Developmental dysplasia of the hip
b.
Legg-calve perthes disease .
c.
Slipped capital femoral epiphysis
Answer:?!
545- HCV infant, mother asking about breastfeeding?
A- Treat the baby then breastfeed.
B- Continue breast feeding.
C- Stop breastfeeding.
answer: B
Ref.: http://www.cdc.gov/breastfeeding/disease/hepatitis.htm
546- Croup case what will you hear by pulmonary auscultation?
Answer: Auscultation reveals prolonged inspiration and stridor. Rales also may be present, indicating
lower airway involvement. Breath sounds may be diminished with atelectasis.
http://www.msdmanuals.com/professional/pediatrics/respiratory-disorders-in-young-children/croup
547- month old with wide anterior fontanel, large protruded tongue, diagnosis?
A- Congenital adrenal hypothyroidism
Answer : Congenital hypothyroidism (cretinism)
548- Easy Q about cystic fibrosis "sweat test" ?
Sweat test is a diagnostic test of CF ( confirm ).
Sweat test and genetic test , it's caused by mutation in both copies of gene for the cystic fibrosis transmembrane conductance regulator (CFTR) protein .

smle ,2016

391

549- 16 years old boy known case of sickle cell anemia presented to with painful right hip pain for
several weeks (this was the scenario and it was for several weeks ). what is the most likely diagnosis:
A- avascular necrosis
B- stills disease
C- tumor.
Osteomyelitis was not in the choices
Answer: Avascular necrosis
550- 7 days baby weight 3.02 was 3.5 Mom is concern about her baby weight, although she feed him:
A-reassure the mom it's normal
B-routine test and reassure
Answer: A
Reference:
http://www.ncbi.nlm.nih.gov/pmc/articles/PMC3091615/
It is expected that newborns will lose some weight in the first 5-7 days of life. A 5% weight loss is considered normal for a formula-fed newborn. A 7-10% loss is considered normal for breastfed babies. Most
babies should regain this lost weight by days 10-14 of life.
551- 13 Y/O child limping & pain, radiological finding destructed femur head, high WBC, diagnosis:
A) Septic arthritis of the hip
B) legg calve perthes
Answer: A
WBC is normal in legg calve perthes disease. Septic arthritis causing painful joint with fever & high WBC.
Radiologic features include:
X-rays may be normal in the very early stage of the disease
joint effusion may be seen
juxta-articular osteoporosis due to hyperaemia
Narrowing of the joint space due cartilage destruction in the acute phase

smle ,2016

392
Destruction of the subchondral bone on both sides of a joint
if left untreated, reactive juxta-articular sclerosis and, in severe cases, ankylosis will develop
http://radiopaedia.org/articles/septic-arthritis
++nelson essential
552- Child with sudden abdominal pain bloody diarrhea, management?
A) Radio decompression
B) Immediate surgery
Answer: Most preferable TTT is to start with radio decompression. NOTE: if patient is in a poor condition
or recurrence or there are sighns of impending perforationaaDO NOT use the previous line of treatmentaaUSE
abdominal
exploration.
*nelson essentials of pediatrics.
According to the age of pt, for patient from 5 months to 3 years, most common cause of bloody stool is
intussusception and they usually respond to non-operative management by enema decompression.
http://emedicine.medscape.com/article/930708-treatment#d1
Intestinal malrotation is suspected with the sudden onset of melena in combination with bilious emesis
in a previously healthy, non distended baby.
Bloody diarrhea and signs of obstruction suggest volvulus, intussusception, or necrotizing enterocolitis,
particularly in premature infants. Acute bloody diarrhea should be considered a medical emergency.
http://emedicine.medscape.com/article/1955984-clinical#b1
Pediatric surgeon: The only definitive treatment for malrotation is surgical in nature
http://emedicine.medscape.com/article/930313-treatment#d7
553- Fever, flu, then developed rashes, which virus ?
Measles
554- Child with yellow and cavities in his teeth, what to give?
A) Antiseptic mouthwash
B) Fluoride
Answer: B
http://www.medicinenet.com/cavities/page3.htm
555- child with diarrhea, vomiting, abdominal Pain, and bloating. What is the possible organism:
A.
Entamoeba histolytica
B.
Giardia lamblia
Answer:B
Nelson essential of pediatric
556- (Case of Intussusception) Child came with colicky abdominal pain, vomiting, bloody stool. US
showed doughnut sign. What is the most important step in management of this case ?
Aurgent
surgery
referral
B- NGT decompression
C- IV Fluid resuscitation
D- Barium Enema
Answer : C- IV Fluid resuscitation (Ref. Master the board)
557- Which of the following is associated with Burkitts Lymphoma ?
A- EBV

smle ,2016

393
B- HIV
C- coxsackie
D- HBV
Answer : A- EBV
Nelson essential of pediatric
558- Child c/o petechial rash all over the body . O/E palpable spleen . Hx of URTI, dx?!
-ITP
-HSP
Answer:A
Toronto:
HSP:
vasculitis of small vessels often have history of URTI 1-3 wk before onset of symptoms clinical
triad: 1) palpable purpura, 2) abdominal pain, 3) arthritis
skin: palpable, non-thrombocytopenic purpura in lower extremities and buttocks, edema, scrotal swelling
joints: arthritis/arthralgia involving large joints associated with painful edema
GI: abdominal pain, GI bleeding, intussusception
renal: microscopic hematuria, IgA nephropathy, proteinuria, HTN, renal failure in <5%
ITP:
50% present 1-3 wk after viral illness (URTI, chicken pox)
sudden onset of petechiae, purpura, epistaxis in an otherwise well child Suggestions
clinically significant bleed in only 3% (severe bleed more likely with platelet count <10) with <0.5% risk
of intracranial bleed no lymphadenopathy, no hepatosplenomegaly labs: thrombocytopenia with normal RBC, WBC bone marrow aspirate only if atypical presentation (1 cell line abnormal, hepatosplenomegaly) differential diagnosis: leukemia, drug-induced thrombocytopenia, HIV, infection (viral), autoimmune (SLE, ALPS)
EVB:
classic triad: febrile, generalized non-tender lymphadenopathy, pharyngitis/tonsillitis (exudative)
hepatosplenomegaly periorbital edema, rash (urticarial, maculopapular, or petechial) more common after inappropriate treatment with -lactam antibiotics any -itis (including arthritis, hepatitis,
nephritis, myocarditis, meningitis, encephalitis
559- Neonate came e decrease feeding & activity + fever , o/e baby is hypotensive Dx?!
A.
Septic shock
Illustrated textbook
560- Preterm baby c/o SOB X-ray showed gross ground appearance + air bronchogram This is due to ?!
A.
Pneumonia
B.
Low surfactant " (ARDS)
Answer:B
Illustrated textbook
561- Child e sx . O/E there's strong pulse in the U.L and absence in LL . Dx?!
A.
Coarctation of aorta
answer:A
562- Daily fluid requirement for child 10 kg .....

smle ,2016

394
A.
1000 ml /day
answer:A
Manual of clinical pediatrics al howasi.
563-Dehydrated child 25 kg , maintenance is
....1600
Notes: 10*100=1000, 10*50=500, 5*20=100 ~> 1600H
Answer: as mention above
Nelson essential of pediatrics 6th edition.
564- Case scenario , they mentioned mother height & father ht. And they asked about excepted Ht of
the child?!
(Paternal
height(inches)
+
maternal
height
/2
-2.5
for
girls
Paternal
height(inches)
+
maternal
height
/2
+2.5
for
boys
565- Baby with oral thrush and ask about Dx (9)?
A-oral candidiasis
Answer: A
566- Baby can sit without support , he can grasp says mama , wave bye bye to the dr. Which developmental milestone he has defect in ?
Gross motor
Fine motor
Sit ~> gross motor
Grasp ~> fine motor
Say mama ~> speech and language
567- picture of child with macular rash all over the body with Hx of fever and carditis, what's the Dx:
A-kawasaki
Answer: A
https://www.merckmanuals.com/professional/pediatrics/miscellaneous-disorders-in-infants-andchildren/kawasaki-disease-kd
568- years old girl with uncomplicated cystitis, how to treat:
A.
oral ampicillin
B.
iv cephalosporin
C.
IM ceftriaxone
D.
iv ...
Medscape: For parenteral therapy in a patient who is not allergic to cephalosporins, initial treatment may
consist of a single dose of ceftriaxone (75 mg/kg IV/IM q12-24h). If the patient has cephalosporin allergy,
initial treatment may be with gentamicin (2.5 mg/kg IV/IM as a single dose)
Toronto: oral cephalixen if outpatient, IV Gentamycin or ampicillin if inpatient.
Uptodate: For E. coli coverage, we suggest a second-generation (eg, cefuroxime, cefprozil) or third-generation cephalosporin (eg, cefdinir, cefixime, cefpodoxime, ceftibuten) for empiric therapy because of
increasing rates of E. coli and other pediatric uropathogen resistance to trimethoprim-sulfamethoxazole
(TMP-SMX), amoxicillin-clavulanate, and first-generation cephalosporins]. However, depending upon local resistance rates, these agents may be acceptable alternatives to second- or third-generation cephalosporins. Second- and third-generation cephalosporins have excellent activity against E. coli and most

smle ,2016

395
other gram-negative uropathogens. However, they are not effective in treating gram-positive uropathogens (eg, Enterococcus)
569- a mother noticed her daughter pulling her hair while studying or with stress and anxiety ( I think
the diagnosis is trichotillomania ) , what is the Rx:
A.
lithium?! X
Answer
:
SSRIs
or
clomipramine
or
Cognitive-behavioral
therapy
-Reference: https://www.merckmanuals.com/professional/psychiatric-disorders/obsessive-compulsiveand-related-disorders/trichotillomania
570- child with long history of enuresis, what is the most important investigation to be done:
A.
VCUG
B.
urinalysis
Answer:B
http://www.mayoclinic.org/diseases-conditions/bed-wetting/basics/tests-diagnosis/con-20015089
571- infant with maculopapular rash over his face with purple discoloration, what is the causative organism:
A.
GBS
B.
strep.pneumonia
C.
staph.aureus
D.
RSV
Answe:
572- child, obese, had left hip pain and limbing with x-ray showing slept femoral hip, no Hx of trauma,
Dx:
A.
slipped capital femoral head
B.
fracture
C.
osteomyelitis
Answer:A
Reference : http://www.seattlechildrens.org/medical-conditions/bone-joint-muscle-conditions/hip-conditions-treatment/hip-disorders/slipped-capital-femoral-epiphysis/
573- young sickler patient had Hx of tiredness and fatigue within 10 hrs, drop in Hb and palpable liver
and spleen 6 cm below the costal margin, had 3 previous similar episodes. What you will do:
A.
splenectomy
B.
start hydroxyurea
C.
reticulocyte count
D.
regular blood transfusions**********
Answer:D
If
hemoglobin
drop
>
than
2
or
showing
s/s
of
anemia:
transfusion
If
recurrence
>3
times:
splenectomy
- Reference : https://www.cdc.gov/ncbddd/sicklecell/treatments.html
574- Child development under 50 percentile, and delay teeth growth, lab normal except for Ca "low",
management?
A) Calcium
Answer: A

smle ,2016

396

575- Child less than 50 percentile, polyuria, constipation, low Na, K, Cl ,where is the primary defect ?
A. Na,Cl Channel
B. K Channel
C. H+ reabsorption
D. H+ secretion
Answer: A
Reference : http://emedicine.medscape.com/article/238670-overview
576- A case of bartter syndrome ; Chloride is passively absorbed along most of the proximal tubule but
is actively transported in the TALH and the distal convoluted tubule (DCT). Failure to reabsorb chloride
results in a failure to reabsorb sodium and leads to excessive sodium and chloride (salt) delivery to the
distal tubules, leading to excessive salt and water loss from the body.
http://emedicine.medscape.com/article/238670-overview#a2
577- infant periumbilical hernia , what you will do ?
A- put plastic in mid abdomen
B- reassurance
C- do hernia repair before start school
Answer: reassurance
578- calculates the deficit for child case?
579- treatment of juvenile rheumatoid arthritis?
answer: Pharmacologic management consisting of:
nonsteroidal anti-inflammatory drugs (NSAIDs),
disease-modifying antirheumatic drugs (DMARDs),
biologic agents,
intra-articular and oral steroids
source: http://emedicine.medscape.com/article/1007276-treatment
580- ECG finding of ostium secundum atrial septal defect
A- LT axis deviation
B- RBBB
C- LVH
D- Delta wave
answer: B
Reference : http://www.metealpaslan.com/ecg/asden.htm

smle ,2016

397

581- Q about meningitis how to interpretation of types of meningitis and how to treat children with
meningitis?
source: http://emedicine.medscape.com/article/961497-treatment#d10

582- about cyanotic heart disease.


A- TOF
B- ASD
C- VSD
D- PDA
Answer: A
source: Master the boards: USMLE step 2, Second Edition, pediatric page 412
583- baby sit briefly, crawl , move object from hand to hand , but no pencil grasp.
A- 4 month
B- 6 month
C- 7 month
B - 9 month
answer: B (Toronto notes 2014 P5)
584- baby thirsty with tachycardia, sunken eye.. Volume loss:
A.1%

smle ,2016

398
B.&lt;3%
C.5-9%
D.9%
Answer : D
Symptom Degree of Dehydration
Mild (&lt; 3% body
weight lost)
Moderate (3-9% body
weight lost)
irritable
Mental status Normal, alert Restless or fatigued,
Heart rate Normal Normal to increased Tachycardia or
Quality of pulse Normal Normal to decreased Weak, thready,
Breathing Normal Normal to increased Tachypnea and hyperpnea
Eyes Normal Slightly sunken Deeply sunken
Fontanelles Normal Slightly sunken Deeply sunken
Tears Normal Normal to decreased Absent
Mucous
membranes
Skin turgor Instant recoil Recoil &lt; 2 seconds Recoil &gt;2 seconds
Moist Dry Parched
Capillary refill &lt; 2 seconds Prolonged Minimal
Extremities Warm Cool Mottled, cyanotic
http://emedicine.medscape.com/article/801012-clinical#showall
++ Toronto note 2014 p6
585- baby ride tricycle draw circle but can't draw square :
A.3 years
B.4 years
answer A
illustrated textbook
586- baby said hi when he entered the clinic, imitates his mother, feeds his doll, refers to himself ME
and say eye:
A.12 months
B.15 months
C.18 months
D. 24 months
answer : D
illustrated textbook
587-Baby with cellulitis and purple rash:
A.
Staph aureus
B.
Strep pneumococcus
C.
Group A beta hemolytic strep
answer C ?
In individuals with normal host defenses, the most common causative organisms are group A streptococci

smle ,2016

399
(GAS) and S aureus. Group B Streptococcus cellulitis occurs in infants younger than 6 months, because
their immune responses are not fully developed, and it may also be seen in adults with comorbidities
such
as diabetes or liver disease. For infantile cellulitis, presentations may include sepsis.
http://emedicine.medscape.com/article/214222-overview#a4
588- 1st Q, child developed generalized edema with fever and dark urine, all labs normal except low
calcium and low albumin, what is dx?
A- Minimal change disease
B- Mesangial
C- nephropathy
Answer: since the child is edematous and has dark urine and feverish, he is going more with nephropathy.
( Toronto notes p83)
589- Child spitting milk, growing fine..what is the dx what will u do next?
most likely everything is just fine. Almost all babies have gastroesophageal reflux (GER), usually just called
reflux. "Seventy percent of infants under 3 months will spit up three times a day, and it's even perfectly
normal for them to be spitting up as often as 10 or 12 times,"
590- A child that can raise his head slightly when prone and smiles. He turns his head 180 degrees and
has head lag when you pull him to sit. How many old is he?
a. 4 weeks
b. 8 weeks
c. 12 weeks
d. 16 weeks
Answer : B
591- Child vaccine, his sister recently kidney transplant, which vaccine should he avoid ?
A. OPV
B. Measles
Answer : A
592- Pencil grasp develops at?
Answer: 9 months
Nelson essential of pediatrics 6th edition
593- Diarrhea in a kid what's the mechanism ( decreased absorption , membranous coating )
Answer:??not clear
594- Asystole first treatment in a child?
A- Epinephrine
B- Atropine.
Answer: A ( Epinephrine remains the drug of choice for asystole in children. Atropine is not indicated.)
Ref.: http://www.aafp.org/afp/1999/1015/p1743.html
595-Calculate Glasco coma scale: child crying and confused i respond when calling his name+ withdraw
to pain=>
A.
9
B.
10

smle ,2016

400
C.
D.

11
12

Answer:
596- 4 month child with developmental milestone ?
A- try to crawling
B- roll over from side to side
C- sit without support
D- complete fixation of the head
answer is B
597- Neonate with bilious vomiting , seed stool diarrhea he passed meconium in day 2 what is the cause
?
A- hirschberg disease
B- allergy to formula milk
answer :A
598- Child with bronchiolitis what is the treatment ?
ribavirin
acyclovir
other antiviral
answer : A
599- Coxsackie virus in pediatric (Hand foot disease)?
Answer
Not clear QQ
600- viral gastroenteritis prevented by which vaccine :
A-Rota vaccine
Answer : a
Rotavirus vaccine
A vaccine that helps protect children against gastroenteritis caused by the rotavirus is now part of the
routine childhood vaccination schedule.

smle ,2016

401
This vaccine is given as a liquid that is dropped into a baby's mouth. It is given in two doses, with the first
given at two months and another at three months. Two possible side effects of the vaccine are diarrhoea
and irritability, but these are usually mild and short-lived.
http://www.nhs.uk/Conditions/Rotavirus-gastroenteritis/Pages/Prevention.aspx
601- child his height and weight below normal besides growth hormone what you will order:
A.
somatomedin c
B.
aldosterone
C.
insulin
D.
testosterone
answer : A
602- Preventing child from drinking before bed, encourage to go to toilet before bed, all these measures
to help child with?
A.
Enuresis
603- Child above 90th percentile in height, long scenario and cardiac abnormality with fatigueetc ?
A.
Marfan syndrome
http://emedicine.medscape.com/article/1258926-overview
604- Child with epilepsy on anticonvulsant What you will change in his vaccines
A.Change OPV to IPV
B. DTP
C. Remove all vaccines
D. Remove all live vaccines
answer : B
source: http://www.vaccines.gov/basics/safety/should/
https://www.healthychildren.org/English/safety-prevention/immunizations/Pages/Immunizations-ForPreterm-Babies.aspx
605- Kid with RF has PSGN with casts, you will find ?
Poststreptococcal glomerulonephritis (PSGN) is usually diagnosed based upon:
The clinical findings of acute nephritis include hematuria with or without red blood cell casts, variable
degrees of proteinuria, edema, and hypertension.
Documentation of a recent GAS infection includes either a positive throat or skin culture or serologic
tests (eg, anti-streptolysin [ASO] or streptozyme test).
Although a low C3 and/or CH 50 (total complement) level are consistent with a diagnosis of PSGN, these
complement components may also be decreased in other forms of glomerulonephritis, including membranoproliferative glomerulonephritis.
http://www.uptodate.com/contents/poststreptococcal-glomerulonephritis
606- Case of lady delivered macrosomic baby, What is the reliable method of diagnosing postpartum
hemorrhage ?
A- Visual assessment of blood loss
B- maternal pulse
C- hemoglobin
D- creatinine
answer : B

smle ,2016

402
http://www.uptodate.com/contents/overview-of-postpartum-hemorrhage#H100822618
607- child with hydrocephalus progressively increasing in between the 3rd and 4th ventricle - which
area is blocked ?
I chose cerebral aqueduct
Answer : Non-communicating hydrocephalus also called "obstructive" hydrocephalus occurs when
the flow of CSF is blocked along one or more of the narrow passages connecting the ventricles. One of
the most common causes of hydrocephalus is "aqueductal stenosis." In this case, hydrocephalus results
from a narrowing of the aqueduct of Sylvius, a small passage between the third and fourth ventricles in
the middle of the brain.
http://www.ninds.nih.gov/disorders/hydrocephalus/detail_hydrocephalus.htm
608- case of lactose intolerance
Answer :
http://www.medicinenet.com/script/main/mobileart.asp?articlekey=7809
609- infant + symptoms of meningitis,
What is the next?
A. LP
B. IV antibiotics
Answer: A
http://www.uptodate.com/contents/initial-therapy-and-prognosis-of-bacterial-meningitis-inadults?source=outline_link&view=text&anchor=H7#H7
610- 1 month Baby come with abdominal distention and constipation since birth what you do next :
a.
Xray
b.biopsy
c.
rectalmanometery
Answer:A
http://www.aafp.org/afp/2006/0201/p469.html
6118
years
child
come
with
a.
Surgery
b.
Give
him
c. Pressure and Lifestyle
Answer:C
First step is behavior and lifestyle change

30

BMI
drug

what
for

can

you

do

for

for

him
obesity
obesity

612- child presented with cyanosis and murmur (case of transposition of great vessels)
http://emedicine.medscape.com/article/900574-overview
613- child come with case of immunodeficiency with presented with lump in the groin and lap was
given
what
is
the
diagnosis
??
there's
CH50
in
Q
Answer:
a.
severe
combined
immune
deficiency
b.
chronic
granulomatous
disease
CH50 is a screening test for total complement activity. Levels of complement may be depressed in genetic

smle ,2016

403
deficiency, liver disease, chronic glomerulonephritis, rheumatoid arthritis, hemolytic anemias, graft rejection,
systemic lupus erythematosus, acute glomerulonephritis, subacute bacterial endocarditis and cryoglobulinemia.
Elevated complement may be found in acute inflammatory conditions, leukemia, Hodgkin's Disease, sarcoma,
and
Behcet's
Disease.
614- Child present to ER with fever and sore throat for one week . Now he has paroxysmal cough and
cyanosis
at
end
of
cough
?
A.
Epiglottitis
B.
Sinusitis
C.
Croup
D.
Bronchitis
Answer: it could be croup or The case going with pertussis (whooping cough) !!
http://www.cdc.gov/pertussis/clinical/features.html
615- Child present with crying and lobulated breathing + retraction of intercostal muscle what to do
A.
Prepare for intubation
B.
Clam the baby down
Answer:B!!!!!
616- Child present with fever and sore throat what of the following suggest viral cause?
rhinorrhea and mucus secretion
617- A child fall down the stair and his head hit the floor. On examination, he was alert and oriented,
not having any neurological deficits, he had only bleeding from his right ear, on further ear exam, the
eardrum was obviously ruptured. What is the most likely the source of the bleeding?
A.
Subdural
Hemorrhage.
B.
Skull
base
fracture.
C.
Subarachnoid
hemorrhage.
D.
other
choices
I
can
not
remember.
Answer:most likely is B
618- A child suddenly hit a hard object, he sustained a forehead wound, where will this wound be
drained?
A.
Pre
auricular
lymph
nodes.
B.
Retroauricular
lymph
nodes.
C.
Occipital
lymph
nodes.
D. Submental lymph nodes.
Answer:
619- Six day infant irritable poor feeding twitching of muscles
A. blood culture,
B.LP
C.
IV
Answer:B
http://www.uptodate.com/contents/meningitis-in-children-beyond-the-basics

smle ,2016

bulging

fontanelle

antibiotic

404
620- Baby hypotension severe vomiting and watery diarrhea what is the electrolyte abnormality
A.
L
NA,
B.
H
NA,
C.
H
K
,
Dhyperglycemia
Answer: sodium + potassium are low so the answer is A low NA
http://rehydrate.org/diarrhoea/tmsdd/2med.htm
621- baby born on 35th week of gestation. his mother bring him for vaccination. which of the following
is
true?!
A)
give
the
vaccine
at
time
as
schedule
B)
delay
his
vaccination
2
months
C)
give
vaccine
at
time,
but
half
the
dose
answer: A
keep in mind that if preterm babies get the infections that vaccines can prevent, they have a greater
chance of having disease-related problems. All of the available vaccines are safe when given to preterm
and low birth weight babies. Any side effects associated with the vaccines are similar in both full-term
and preterm babies.
622- A 6 weeks old baby pale, jaundice on examination there is palpable spleen 2 cm below the costal
margin. lab shows total bilirubin =205 mg\dl, direct bilirubin = 60 mg/dl, positive direct & indirect
coombs
test.
peripheral
blood
smear(attached
photo
shows
spherocytosis)
A.
spherocytosis
I.
B.
gilbert
disease
J.
C.
ABO
incompatibility
D.
Crigler
Najjar
syndrome
Answer
:
should
be
Autoimmune
623sign
of
duodenal
A.
bird
beak
B. double babble appearance
Answer:B
http://emedicine.medscape.com/article/408582-overview

atresia
appearance

in

x-ray
R.

624- child came complaining of pallor & fatigue. he has short stature & multiple cafe au lait spots over
his
skin.
lab
shows
(
WBC=
normal,
RBC=
low,
Hb=
low,
retc=
)
A.
iron
deficiency
anemia
B.
SCA
C.
fanconi
Answer:C
http://emedicine.medscape.com/article/911900-overview
625- parent bring their child after mid night (3 a.m.) child complain of barking cough, dyspnea, fever &
inspiratory stridor. sp o2 = 2% in room air. which of the following symptoms are of concern ?!
A.
fairing
of
ala
nasi
U.
B.
expiratory
stridor
V.
C.
blue
color
of
lips
Answer:C

smle ,2016

405

626- which of the following congenital heart disease is secondary to failure of spiral rotation of the
heart
septum
?!
A.
transposition
of
great
artery
B.
ASD
C.
VSD
D.
PDA
Answer:D
627- A 18 month old child present to OPD having diarrhoea & flatulence. looking pale , below 25th percentile for weight. baby was completely normal at age 12 month when he weaned from breastfeeding.
A.
celiac
disease
B.
cystic
fibrosis
C.
hirschsprung
disease
Answer:
A?
628- long scenario for cerebral palsy). on examination there is crossing of lower limb when child suspend by the axilla. which type of CP does the pt have
A.
Hemiplegia
B.
Diplegia
C.
Quadriplegia
D.
Athetoid
Answer:C

629-what is central treatment of kawasaki?


A-IVIG (my answer based on decrease aneurysm from 25%to 2%} ++
B-aspirin
Answer:A
630- Diet causes Kawashioker?
A.
Low protein
B.
High Carbs
Answer: A

smle ,2016

406
631- Mother brought her child, 9 months, fisting hand and crossed legs, she mentioned that after his
birth he didn't cry:
A.cerebral palsy
B.Down syndrome
Answer: A
632- scenario of marfan :
A.
X linked
B.
AD
C.
AR
Answer : B

Autosomal Dominant
o
Description

only need to get the abnormal gene from one parent in order for you to inherit the disease.
o
Examples

Syndactyly

Polydactyly

Marfan's syndrome

Cleidocranial Dysostosis

Hereditary Multiple Exostosis

Achondroplasia

MED

Metaphyseal chondrodysplasia (Schmid and Jansen types)

Kniest dysplasia

Malignant hyperthermia

Ehlers-Danlos syndrome

Osteogenesis imperfecta (types I and IV)

Osteochondromatosis/Multiple Hereditary Exostosis

Osteopetrosis (tarda, mild form)


http://www.orthobullets.com/basic-science/9024/inheritance-patterns-of-orthopaedic-syndromes
633- 5 years old boy presented with sore throat, he was discharged
home, culture was done showing Group A meningococcus, the
physician called the family to inform them he finds that the child is
asymptomatic, the best treatment is:
A.penicillin
B.single dose ceftriaxone
Answer: B
ceftriaxone or cefotaxime) is the drug of choice for the treatment of meningococcal meningitis and septicemia. Penicillin G, ampicillin, chloramphenicol, fluoroquinolone, and aztreonam are alternatives therapies (IDSA guidelines).
634- When does the jaundice occurs in gilbert syndrome?
Pregnancy Inter current illness*( i think it is the right)
Others .No stress in options
635- A 6 year old girl, brought by parents to ER with history of falling
from Height ... Not talking but crying, withdrawal from pain, open her

smle ,2016

407
eye only in response to Doctor talking... Calculate GCS
A) 9
B) 10
C) 11
D) 12
Answer: B crying=3
636 -7 month present with respiratory sx ??
bronchlitis
637- baby missed vaccine developed bilateral parotid swelling what is
the likely complication
A-Hearing loss
B-Encephalitis
C-Sterility
D-Facial nerve paralysis
Answer: B
Encephalitis and meningitis
http://www.who.int/biologicals/areas/vaccines/mmr/mumps/en/
638- cat bite Child ,, In Site of bite developed infection ,,What is the bacteria ?
A.
.........
B.
pasteurella
(
correct
Answer: B
http://www.uptodate.com/contents/soft-tissue-infections-due-to-dog-and-cat-bites?source=outline_link&view=text&anchor=H3#H3
639- How to diagnose Giardia lamblia?
A- three consecutive stool analysis
B- three separate stool analysis
Answer:B
http://emedicine.medscape.com/article/176718-workup

smle ,2016

408

OBS & GYNE

smle ,2016

409
1- Which of the following muscles will be affected during normal vaginal delivery?
Answer: ?
A.
coccygeal
B.
pubococcygeal
C.
ischial...
(it depends on the degree of the perineal tear)
1st degree: vaginal mucosa affected only (skin and subcutaneous tissue but the perineal muscles remain Intact).
2nd degree: involvement of perineal body muscles which includes: (not anal sphincter)
bulbocavernous
superficial transverse perineal muscle
pubococcygeus muscles
3rd degree: involvement of the external anal sphincter and/ internal anal sphincter.
4th degree: extent through the anal mucosa.
Reference: UpToDate

2- Lactating mother complaining of breast tenderness, hotness and redness, diagnosed to


have bacterial mastitis. What will you recommend for her?
A. Continue breast feeding, hot compressor and antibiotic.
B. Discontinue breast feeding and give antibiotic to mother and baby.

Answer: A
Heat or ice packs, continued breastfeeding/pumping in Pts who are no longer breastfeeding,
antibiotics (dicloxacillin/cephalexin, amoxicillin/Calvunate, Azithromycin, clindamycin) (erythromycin if penicillin-allergic).
Reference: Toronto notes and 3rd Edition UQU > Obstetrics and Gynecology > Q 385 +First Aid
step 2 .
3- Multigravida in labor with 60% effacement and dilated cervix (5 cm). After 1 hour she still
has 60% effacement but the cervix dilates to 6 cm. What will you do for her?
A. expectant management
B. oxytocin
C. cervix ripening

smle ,2016

410
D. artificial rupture of membranes

Answer: A
Obs/gyne consultant.
4- Swelling at the labia majora for 6 month. It was aspirated and now it relapsed. What is
your management?
A. Aspiration
B. Marsupialization

Answer: B
It is a Bartholin cyst, it needs to be drained with a simple incision and drainage but if it continued to recur, then Marsupialization should be done. Reference: Master the board.
If It Asymptomatic cysts :No therapy +/- warm soaks. Reference :1st Aid step 2 .
5- Primigravida week 16. She is RH negative. What is your next step?
a.
US
b.
Anti-D Rh immunoglobulin
c.
Rh antibody titer
Answer: C
Rh antibody titer during the initial prenatal visit if shes RH Unsensitized patients do not yet have antibodies to Rh positive blood. The goal is to keep it that
way:
so any time that fetal blood cells may cross the placenta, anti-D Rh immunoglobulin (RhoGAM)
are given.
Prenatal antibody screening is done at 28 and 35 weeks. Patients who continue to be unsensitized at 28 weeks should receive anti-D Rh immunoglobulin prophylaxis.
At delivery, if the baby is Rh positive, the mother should be given anti-D Rh immunoglobulin
again.
The patient is considered sensitized if she has a titer level more than 1:4.
If the titer is less than 1:16, no further treatment is necessary.
If it reaches 1:16 at any point during the pregnancy, serial amniocentesis should be done. Serial
amniocentesis allows for evaluation of the fetal bilirubin level. Reference: Master the board.
2nd edition P461
6- 42 years old female complaining of amenorrhea, night sweat and flushing for the last 6
months. What is the most likely diagnosis? Same Q in P71 qustion# 61
a.
Hypothyroid
b.
Hyperprolactinemia
c.
Congenital adrenal Hyperplasia
d.
Pheochromocytoma
Answer: c?
7- Multigravida 34 week, her baby is breech, what you will do for her?
a.
expectant delivery
b.
CS
smle ,2016

411
c.
External Cephalic Version
Answer: A
Expectant delivery until 36 week.
You should not perform ECV before 36 weeks, because the baby can turn into cephalic spontaneously. Reference: Master the board.
8- What is the most common cause of vaginal bleeding?
Answer: anovulation
The most common specific causes in adult women who are not known to be pregnant are:
Anovulatory (dysfunctional uterine) bleeding > most common
Complications of an early, undiagnosed pregnancy
Submucous myoma
(In patient who has abnormal bleeding during the reproductive age group Pregnancy or a complication of pregnancy must first considered. ) Kaplan USMLE step 2
9- Which of the following non hormonal supplements will decrease the hot flashes in postmenopausal women?
A. Black Cohosh
B.
Paroxetine
C.
Bromocriptine
Answer: B
SSRIs/SNRLs, gabapentin, clonidine. Reference: first Aid USMLE step 2
10- Why postmenopausal women develop osteoporosis?
A. decrease progestin
B. increase FSH
C. decrease Estrogen

Answer: C
Estrogen deficiency can lead to excessive bone resorption accompanied by inadequate
bone formation. In the absence of estrogen, T cells promote osteoclast recruitment,
differentiation, and prolonged survival via IL-1, IL-6, and tumor necrosis factor (TNF)
alpha.
Reference: MEDSCAPE
11- In polycystic ovarian syndrome, which of the following will be found on blood test?
a.
FSH:LH 1:3
Answer: A
The LH-to-FSH ratio is usually greater than 3.
Reference: MEDSCAPE
12- Mom wants to know if her baby is having thalassemia or not. How you will investigate her
antenatally?
Answer:

smle ,2016

412
Screening tests for high risk population: CBC (MCV and MCH), Hb electrophoresis or high performance liquid chromatography (HPLC)
Confirmatory tests: Chorionic villus sampling (CVS): between 10-12 weeks, Amniocentesis: between 15-16 weeks to term.
Reference: Toronto Notes

Screening for Hematologic Assessment: ((ACOG))

smle ,2016

413

14- 21 years old Female with negative pap smear. You should advise her to repeat pap smear
every:
a.
6 months
b.
12 months
c.
18 months
d.
no repeat t age 21 years or no more than 3 years after becoming sexually active.
Then every 3 years after 3 normal consecutive test.

smle ,2016

414
Women > 30 years old who have three con
Answer: 3 year
Annual pap smear screening starting asecutive normal tests screening (1 / 3 years).
Screening should be discontinuing for women > 60-70 years who have had 3 or more normal
Pap smear.
Reference: 3rd Edition UQU > Obstetrics and Gynecology > Q 201 and Essential of obstetrics
and gynecology
15- Woman with endometriosis. What is the best way to diagnose?
laparoscopy
Answer: A
Laparoscopic Identifications of endometriotic nodules or endometrioma is the definitive way of
making the diagnosis .
16- How ectopic pregnancy occurs at the cellular level?
A. Disappearance of zona pellucida.
B.
Fertilization at ampulla tube.
C.
Persistence of Zona pellucida.
D. Fast division of blastomere.
Answer: A
As cilia degenerate the amount of time it takes for the fertilized egg to reach the uterus will increase. The fertilized egg, if it doesn't reach the uterus in time, will hatch from the non-adhesive zona pellucida and implant itself inside the fallopian tube, thus causing the pregnancy.
Reference: Wikipedia and Clinical reproductive medicine & surgery book - textbook of clinical
embryology
17- Nursing mom want to conceive but not at the coming two years. What will you recommend for her?
A.
B.
C.
D.

vaginal ring
combined OCP
progestin injection
patch

Answer: C
18- 20 years old sedentary female complaining of amenorrhea for the last 6 months and her
BMI is 20.
a.
Prolactinoma
b.
Anorexia
c.
depression
Answer: C
19- Lactating lady who didn't take the MMR. What will you advise her to do?
a.
Take the vaccine and stop feeding for 72 hour
b.
It is harmful for the baby

smle ,2016

415
c.
She can take the vaccine
Answer: C
Reference: 3rd Edition UQU > Obstetrics and Gynecology > Q 39
vaccines given to a nursing mother do not affect the safety of breastfeeding for mothers or infants and that breastfeeding is not a contraindication to MMR vaccine.
Reference: CDC
20- A patient with ectopic pregnancy of 2.5*3.0 size. hCG is 5000. The patient is stable. What
will you do?
A.
wait and watch
B.
laparotomy
C.
laparoscopy
D.
D&C
Answer: C
Reference: master the bored p448
21- A couple came to your clinic. They are trying to conceive for the last 3 months with no
success. The girl had appendectomy before marriage. She also has an aunt who is her uncles
wife (not blood related) with down syndrome. What should be done?
a.
try some more
b.
clomiphene
c.
laparoscopy
d.
semen analysis
Answer: A
22- A girl who hit puberty few months back and complains of spotting in between her periods. What will you tell her?
A.
She has PCOS
1She needs to take OCPs
B.
C.
If tests were normal its not a disease
Answer: B Toronto note GY5
23- A 50 years old lady came with signs and symptoms of menopause. What picture describes
his report best?
A.
increased LH and FSH
B.
decreased FSH and LH
C.
increased FSH decrease LH
D.
increase LH decrease FSH
Answer: A
Master the boared p482
24- A patient with cervical carcinoma. What viruses are thought to be major culprits?
1

smle ,2016

416
A.
HPV 43 and 44
B.
HPV 16 and 18
C.
HPV 6 and 11
Answer: B (Torento note GY46 gynacology)
25- A long scenario of a lady with vaginal infection, has strawberry cervix. What is the organism?
A. Trachomatis
B. Bacterial vaginosis
C. Gonorrhea
D. Trichomonas vaginalis
Answer: D (First aid of usmle 2013 p170)
26- A woman with vaginal infection that grows gram negative diplococci. What is the organism involved?
A.
N.gonorrhoeae
B.
HSV
C.
Candida
Answer: A
Reference: Toronto Note GY28 gynacology
27- What is the role of metformin in PCOS?
A.
Decrease glucose level
B.
Decrease insulin resistance
C.
Anti-androgenic
D.
Menstrual regulation
Answer: B
The hallmark mark of PCOS is insulin resistance.
28- Pregnant with monochorionic twins in week 27. One of them died, what to do?
Answer: delivery
If fetal assessment after 26 weeks of gestation suggests impending death rather than demise of
one twin of a monochorionic pair, we suggest prompt delivery of both twins rather than expectant management given the high risk of neurologic impairment in the surviving co-twin.
Reference: http://www.uptodate.com/contents/twin-pregnancy-prenatal-issues?source=search_result&search=monochorionic+twins+one+f+them+die&selectedTitle=1%7E150#H24
29- Pregnant lady with UTI, which Antibiotic is contraindicated?
Tetracycline
Answer: A
Source: http://emedicine.medscape.com/article/452604-treatment
Some antibiotics should not be used during pregnancy, because of their effects on the fetus.
These include the following:
Tetracyclines (adverse effects on fetal teeth and bones and congenital defects)
Trimethoprim in the first trimester (facial defects and cardiac abnormalities)
smle ,2016

417
Chloramphenicol (gray syndrome)
Sulfonamides in the third trimester (hemolytic anemia in mothers with glucose-6-phosphate dehydrogenase [G6PD] deficiency, jaundice, and kernicterus)
Certain antibiotics should be avoided during pregnancy. For example, tetracycline such as
doxycycline and minocycline can damage a pregnant woman's liver and discolor a developing
baby's teeth.
Antibiotics generally considered safe during pregnancy:
Amoxicillin, Ampicillin, Clindamycin, Erythromycin, Penicillin and Nitrofurantoin.
30- Hormonal replacement therapy prevents which of the following?
A. Postmenopausal symptoms
B. Osteoporosis
C. Coronary artery disease
D. Stroke
Answer: A
Indications of HRT: primary indication is treatment of menopausal symptoms (short-term).
It can be use to prevent/treat osteoporosis (long-term) and premature ovarian failure.
Reference:http://www.guidelines.co.uk/obstetrics_gynaecology_urology_mm_hrt#.VkoeS8p_qM
31- Patient known case of PCOS and wants to get pregnant.
clomiphene citrate
Answer: A
infertility
medical induction of ovulation: clomiphene citrate, human menopausal gonadotropins (HMG
[Pergonal]), LHRH, recombinant FSH, and metformin
metformin may be used alone or in conjuction with clomiphene citrate for ovulation induction
ovarian drilling (perforate the stroma), wedge resection of the ovary
bromocriptine (if hyperprolactinemia)
Reference: Toronto Notes
32- A patient with premature rupture of membranes for more than 18 hours (long scenario
with a lot of details). Which of the following give the patient high risk for GBS infection?
A. Rupture of membranes for more than 18 hours.
B. Family hx of GBS infection.
Answer: A
Group B strep infection is more common in African Americans than in whites. There are also
maternal risk factors that increase the chance of transmitting GBS to the newborn leading to
early onset disease:
Labor or membrane rupture before 37 weeks gestation
Membrane rupture more than 18 hours before delivery
Urinary tract infection with GBS during pregnancy
Previous baby with GBS infection
Fever during labor
smle ,2016

418
Positive culture for GBS colonization at 35-37 weeks
33- What is the best way to know the date of pregnancy?
A. LMP
B. Ultrasound
C. Fundal height
Answer: B
The baby can be measured as early as 5 or 6 weeks after the mothers last menstrual period.
Measuring the baby using ultrasound is most accurate in early pregnancy. It becomes less accurate later in pregnancy. The best time to estimate gestational age using ultrasound is between
the 8th and 18th weeks of pregnancy.
The 3 basic methods used to help estimate gestational age (GA) are menstrual history, clinical
examination, and ultrasonography. The first 2 are subject to considerable error and should only
be used when ultrasonography facilities are not available.
Reference: Medscape.
34- A pregnant woman who has a child with down syndrome. Shes concerned about having
another child with down syndrome. What is the best test to rule out down syndrome in the
second trimester?
A. Amniotic fluid sample
B. Chorionic villous sampling
C. Triple investigation
Answer: A, obs/gyne consultant
Second trimester screening tests include:
Triple investigation is done between 15- 20 weeks. Its sensitivity is about 65% for trisomy 21.
Patients with positive screen should be offered U/S or amniocentesis for confirmation.
Quadruple screencan improve the detection rate for Down syndrome to about 81%.
Second trimester confirmatory test: Amniotic fluid sample.
*See the tables at the end of OB/GYN section.
References: Toronto Notes and American Family Physician Journalshttp://www.aafp.org/afp/2007/0901/p712.html
35- 46 years old woman comes with amenorrhea for 6 months and flushes at night that disturbs her sleep. What is the best investigation to make your diagnosis?
A. LH
B. FSH
C. Estrogen
D. Progesterone
Answer: B
36- Premature menopause starts before...?
A. 30
B. 35
smle ,2016

419
C. 40
D. 45
Answer: C
premature ovarian failure; before age 40
Source : Toronto Notes
37- Which of the following is equally effective to laparoscopy in a patient with unruptured
small ectopic pregnancy?
Methotrexate
Answer: A
goals of treatment: conservative (preserve tube if possible), maintain hemodynamic stability
surgical = laparoscopy
linear salpingostomy an option if tube salvageable
salpingectomy if tube damaged or ectopic is ipsilateral recurrence
15% risk of persistent trophoblast; must monitor -hCG titres weekly until they reach nondetectable levels
consider Rhogam if Rh negative may require laparotomy if patient is unstable, extensive
abdominal surgical history, etc.
medical = methotrexate
use 50 mg/m2 body surface area; given in a single IM dose this is 1/5 to 1/6 chemotherapy
dose, therefore minimal side effects (reversible hepatic dysfunction, diarrhea, gastritis, dermatitis)
follow -hCG levels weekly until -hCG is non-detectable
plateau or rising levels suggest persisting trophoblastic tissue (requires further treatment)
82-95% success rate, but up to 25% will require a second dose
tubal patency following methotrexate treatment approaches 80%
Source: Toronto Notes
38- Megaloblastic anemia in pregnant.
give folate
Answer: A
Prevention: 0.4-1 mg folic acid PO daily for 1-3 mo preconceptually and throughout T1, or 5 mg
folic acid per day with past history of oNTD, DM, or antiepileptic medication use
Source : Toronto Notes
Reference:http://www.ncbi.nlm.nih.gov/pubmed/8612357
39- Which of the following viruses can cross the placenta?
A. rubella
B. mumps
C. HBV

Answer: A ?
*HBV: depends if the disease is active or not.
Diseases that can cross the placenta:
All TORCH; Toxoplasmosis Others: e.g. syphilis Rubella
smle ,2016

420
, CMV
, HSV
HIV, Chicken pox, CMV, Erythema Infectiosum (Fifth Disease), Hepatitis B.
Reference: Toronto Notes
Agree to this answer and revised
-rubella can cross placenta, TORCH can cross
-HBV cannot:
http://www.who.int/csr/disease/hepatitis/whocdscsrlyo20022/en/index1.html
40- A pregnant lady had a child with 3500 grams with the use of forceps, presented to you 20
days postpartum with whitish vaginal discharge but with no itching or cervical tenderness. On
examination cervix is pink. Microscopic examination reveals epithelial cells with leukocytes.
What would you do for your patient?
A.
B.
C.
D.
E.

Dipstick urinalysis
Pelvic ultrasound
Reassure
Metronidazole
Culture discharge

Answer: C
Reference: http://www.ncbi.nlm.nih.gov/books/NBK288/
Agree
in order to answer this q you have to know the types of vaginal discharge and the scenario of
each one:
-we reassure because normal in microscope and no itching so most probably it normal secretion.
-you have to differentiate between bacterial vaginosis, candidiasis, trichomonad
source: master the board
41- Female patient came with signs and symptoms of PCOS. Lab results: FSH= 1.5, LH= 10.
What is the most likely Diagnosis?
PCOS
Answer: A
Agree
in order to answer this q you have to know some basic information about PCOS:
1-symptoms:
-amenorrhea or irregular menses, hirsutism & obesity, Acne, DM type2
2-diagnostic tests:
-pelvic ultrasound:bilateral enlarge ovaries with multiple cysts
-LH to FSH ratio more than 3:1
Source :master the board
42- Female came to infertility clinic because she cannot conceive. She has a history of three
elective abortion and D&C in the past. She refused to be examined. What is the most likely
diagnosis?
smle ,2016

421
A. Sheehan Syndrome
B. Asherman Syndrome

Answer: B
Agree
-sheehan syndrome after postpartum hemorrhage present as inability to breastfeed.
-The buzz word for Asherman is multiple D & C
source:
Master the board
43- Which of the following is an absolute contraindication for breastfeeding?
active HIV
Answer: A
Contraindicated if mother:
is receiving chemotherapy or radioactive compounds:
has HIV/AIDS, active untreated TB, herpes in breast region:
is using >0.5 g/kg/d alcohol or illicit drugs
is taking medications known to cross to breast milk
Reference: Toronto Notes
Agree
you have to know the list of contra indication:
agree and revised from master the board
44- (long scenario) Pregnant lady in 1st trimester (12 weeks) on iron trial, complaining of fatigue and shortness of breath. CBC show: Hb = low , MCV = 70 , hematocrit = normal , reticulocyte = 10% . What is the most likely diagnosis?
a.
Physiological.
b.
Iron deficiency.
c.
Thalassemia.
Answer: B
Agree
the key to answer dos q is know the types of anemia:
-iron deficiency anemia in pregnancy caused by increased hepcidin level which inhibit iron
transport. (so it is most common cause of anemia)
-Thalassemia can present with low Hg and MCV you will suspect it if the anemia does not improve with iron.
source:
Master the board

smle ,2016

422
45- 28 weeks pregnant (nuli), presents with
generalized fatigue, BP 162/95, 3+ protein in
urine. What is your next step?
1. Mgso4
2. Labetalol
1. MethylDopa
Answer: A
Agree
In order to answer this q you have to know
some information about the preeclampsia:
1-chronic hypertension: is blood pressure
>140/90 before patient become pregnant.
here you can treat with Labetalol or Methyldop or nifedipine
2-gestational hypertension: is blood pressure >140/90 start after 20 week gestation + no proteinuria + no edema. here you can treat
with Labetalol or Methyldopa or nifedipine.
3-preclampsia: see the table and the figure.(they are from master the board)

We recommend administration of antenatal corticosteroids for all pregnant women at 24 to 33


weeks
Betamethasone is given if < 34 weeks who are at increased risk of preterm delivery within the
next seven days.
Reference: Master the Board
46- 45 years old gravida 4 para 3, week 8 pregnant. Last pregnancy she had a down syndrome baby so shes asking for check out ,regarding Down syndrome. What are the complications that you are going to tell her when you take consent?
a.
rupture of amniotic sac
b.
unintended miscarriage
Answer: B
smle ,2016

423
Chorionic villus sampling (10 - 12 wk):1-2% risk of spontaneous abortion. Reference: Toronto
Notes
Agree
-Down syndrome increased with increase maternal age
-there is some investigation you can do at every specific gestational age some time it differ
slightly according to your reference.
Chronic villous sampling( pregnancy loss rate 0.7%), Amniocentesis (preganancy loss rate 0.5%),
percutaneous blood sampling (preganancy loss rate 2-5%)
source: master the board
47- A patient presented to the ER with severe RLQ pain and positive B-HCG. What is the Diagnosis?
a.
ruptured ectopic pregnancy.
Answer: A
Agree and revised
source: matter the board step2 ck

48- 63 years old female. Pap smear showed atypical squamous cells of undetermined significance (ASCUS). You gave her local estrogen and after one week pap smear results still
showing ASCUS. What will be your next step?
Answer: colposcopy + biopsy
Repeat the Pap test in one year. If this test is normal, the woman can return to regular screening. If an abnormality is found, then a colposcopy should be done.
Reference:
http://www.uptodate.com/contents/follow-up-of-low-grade-abnormal-pap-tests-beyond-thebasics
http://www.mayoclinic.org/tests-procedures/pap-smear/basics/results/prc-20013038
49- What antibiotic is safe in pregnancy?
a.
Ciprofloxacin
b.
Amoxicillin
c.
Chloramphenicol
Answer: B
Some of the antibiotics that may be prescribed safely during pregnancy include: Amoxicillin,
Ampicillin, Clindamycin, Erythromycin, Penicillin, Gentamicin, Ampicillin-Sulbactam, Cefoxitin,
Cefotetan and Cefazolin
C- Chloramphenicol has been assigned pregnancy category C by the FDA
A-This drug should not be used during pregnancy unless the benefit outweighs the risk
to both fetus and mother.
References:http://www.webmd.com/baby/tc/medicines-during-pregnancy-overview
50- What is the most common cause of secondary amenorrhea?
smle ,2016

424
a.
Pregnancy
Answer: A * amenorrhea is pregnancy until proven otherwise.
Reference : Medscape
51- Mild Preeclampsia patient (34 weeks of gestation). What will you do next?
a.
Immediate C/S (culture and sensitive)
b.
Observe BP
Answer: BPatients with preeclampsia without severe features are often induced after 37
weeks' gestation. Before this, the immature fetus is treated with expectant management with
corticosteroids to accelerate lung maturity in preparation for early delivery.
Reference : Medscape
52- A 10 weeks pregnant lady, known case DM and HTN. Her BP 160/95. How will you manage this case?
a.
Observation
b.
Termination of pregnancy
c.
ACEI
Answer:A or what written below
Methyldopa, nifedipine or labetalol. Reference: Master the boards.
The available data are insufficient to rule out unrecognised adverse effects of early and prolonged use of -blockers in pregnancy. Methyldopa is a drug of first choice for control of mild to
moderate hypertension in pregnancy. Oral hydralazine, a direct vasodilator, is effective as monotherapy or as add-on therapy to methyldopa in the long term management of chronic hypertension in pregnancy. Reference:http://www.medscape.com/viewarticle/406535_6
53- Multipara, 38 weeks pregnant. Cervical os is 7 cm with cord prolapse?
a.
CS
Answer: A
Administer oxygen to the woman 4-6 L/minute.
If the fetus is viable, place the mother in the knee-chest position (patient facing the bed, chest
level to bed, knees tucked under chest, pelvis and buttocks elevated) or head-down tilt in the
left lateral position and apply upward pressure against the presenting part to lift the fetus away
from the prolapsed cord. This can be done manually (gloved hand/two fingers pushing upwards
against the presenting part or once the presenting part is above the pelvic brim, using continuous suprapubic pressure in an upwards direction) or by filling the urinary bladder.
Manual replacement of the prolapsed cord above the presenting part is not currently recommended. Avoid handling the cord outside the vagina, as this induces vasospasm.
Proceed to emergency caesarean section as soon as possible. Reference:http://patient.info/doctor/prolapsed-cord
54- Old woman, atrophic vaginitis, low mood, osteoporosis, .etc. what is your management?
a.
Estrogen
b.
SSRI
smle ,2016

425
Answer: A
local estrogen replacement (ideal): Premarin cream, VagiFem tablets, or Estring
oral or transdermal hormone replacement therapy (if treatment for systemic symptoms
is desired)
good hygiene
Reference: Toronto Notes , http://patient.info/health/menopause-and-hormone-replacement-therapy-hrt
55- Pregnant women (30 Something but definitely less than 38) with BP baseline 80 now present with 140 or 160 (Not sure), LL edema. +2 proteinuria, no change in LFT (not sure). Admitted to the hospital. What will you give her?
a.
Betamethasone
b.
Labetalol
c.
MgSO4
Answer: B
Intravenous (IV) labetalol and hydralazine have long been considered first-line medications for
the management of acute-onset, severe hypertension in pregnant women and women in the
postpartum period. Available evidence suggests that oral nifedipine also may be considered as
a first-line therapy.
Reference :
http://emedicine.medscape.com/article/261435-overview and contain why not chose the
other answer .
56- A female patient presented with green vaginal discharge and pruritus?
a.
Trichomonas
Answer: A
Refer to the table at the end of OB/GYN section.
Trichomoniasis
infection is the most common nonviral STD in the world. Many patients (20-50%) are asymptomatic. If discharge is present, it is usually copious and frothy and can be white, gray, yellow, or
green
57- What is the treatment of choice for Trichomoniasis?
A. Fluconazole
B. Metronidazole

Answer: B
Check the table at the end of OB/GYN section. Metronidazole the drug of choice for trichomoniasis , they give single dose 500mg twice daily for 7 days
Reference ; Toronto note , uptodate ,first aid usmle step 2 CK, web-med.com, master of the
board step2 , e-medicine .medscape.com
58- What is the drug of choice for eclamptic seizure?
a.
Phenytoin.
b. Diazepam.
smle ,2016

426
c.
Magnesium Sulfate.
Answer: C
Although the definitive treatment is delivery, and the seizure should be controlled with magnesium sulfate.
Reference: Toronto Notes, usmle , uptodate , master of the board.
59- Pregnant woman in 3rd trimester, with vaginal infection (discharge), after delivery the
baby got eye infection (conjunctivitis and discharge). What is the most likely cause?
a.
chlamydia
b.
gonorrhea
Answer: B
Gonococcal conjunctivitis tends to occur 2-7 days after birth but can present later.
Chlamydial conjunctivitis usually has a later onset than gonococcal conjunctivitis; the incubation
period is 5-14 days.
Reference: http://emedicine.medscape.com/article/1192190-clinical
Answer: A ,, Reference : Toronto note
60-Pregnant at 5 weeks of gestation. Cervical incompetence was diagnosed. what will you
do?
a.
cerclage now
b.
cerclage at 12-13 weeks
Answer: B
Usually at the end of the 1rst trimester and removed in the third trimester.
Emerging evidence indicates that progesterone suppositories are superior to cerclage in preventing preterm labour late in pregnancy. Reference: Toronto Notes.
61- A female patient cant get pregnant for 3 years. Recently she developed breast milk. What
is the most likely diagnosis?
a.
Hyperprolactinemia.
Answer: A
Reference : master of the board 3

62- A pregnant lady presented with flank pain. On examination there was tenderness. Labs
showed leukocytosis and positive nitrate, what is the management?
a.
Admission to treat pyelonephritis
b.
Drink plenty of fluids
c.
Start antibiotics
Answer: A
Reference: Kaplan Obstetrics and Gynecology USMLE step 2.
Master of the board e-medicine.medscape.com,
63- A pregnant lady with a positive OGTT, what is your action?
a.
Repeat the test
smle ,2016

427
b.
Check HgA1c
c.
Start insulin
d.
Do a random blood glucose
Answer: C
Reference: Master the Boards. Toronto note
64- Pregnant G1P0 who has a history of travelling 1 year ago, came for check up. Result shows
HIV +ve. What is the action in this case ?
a.
Acyclovir for the mother during 1 week.
b.
(something) given to the baby after delivery.
c.
(something) given to the mother and baby after delivery.
d.
Acyclovir is contraindicated.
Answer: zidovudine for mother and baby
In an HIV-infected pregnant woman who has never been exposed to antiretroviral medication,
HAART should be started as soon as possible, including during the first trimester. Combination
antiretroviral therapy should be offered in all cases. As zidovudine (ZDV) is the only agent specifically shown to reduce perinatal transmission, it should be used whenever possible as part of
the highly active antiretroviral therapy (HAART) regimen.
All HIV-exposed infants should receive zidovudine.
Reference: http://emedicine.medscape.com/article/1385488-overview#showall
This q I check master of the board and they wrote the therapy is antiretroviral triple therapy include ZDV not ZDV alone because is mono-therapy
65- Female complaining of irregular menstruation for 6 months, history reveal normal babies
with normal deliveries but she did D&C after the second delivery for retained part of placenta, investigations (I cant remember), what is your diagnosis ?
a.
Ashermans syndrome.
b.
Polycystic ovary.
Answer: A
Intrauterine adhesion (or intrauterine synechiae) is a condition in which scar tissue develops
within the uterine cavity. Intrauterine adhesion accompanied by symptoms (eg, infertility,
amenorrhea or hypomenorrhea) is also referred to as Asherman syndrome. Its primarily
caused by curettage for pregnancy complications.
Reference: Uptodate e-medicine.medscape.com
Curettage after delivery or abortion may result in endometrial injury and subsequent development of intrauterine adhesions, termed Asherman syndrome. The development of uterine synechiae may also be associated with prior endometrial ablation procedures. Intrauterine adhesions may make future diagnostic curettage more difficult and increase the risk of uterine perforation. Previous procedures such as endometrial ablation may also increase the risk of cervical stenosis.
66- Postmenopausal women complaining of itchy vulva and erythema of the labia majora and
sometimes bleed. On examination there is a pea shaped mass. What is the diagnosis?
a.
Bartholin gland cyst.
b.
Bartholin gland carcinoma.
smle ,2016

428
c.
Bartholin abscess.
Answer: B
It is generally recommended that women over age 40 with a Bartholin cyst or abscess undergo
drainage and biopsy of the gland to exclude the possibility of an underlying carcinoma.
https://quizlet.com/53890288/obgyn-neoplasia-flash-cards/
67- What is Adenomyosis?
a.
Presence of endometrial tissue and gland in Uterine Ligament.
b.
Presence of endometrial tissue and gland in Uterine Muscle.
c.
Presence of endometrial tissue and gland in Cervix.
d.
Presence of endometrial tissue and gland out Uterus.
Answer: B
It occurs when endometrial tissue, which normally lines the uterus, exists within and grows into
the muscular wall of the uterus.
68- Woman pregnant 42 Gestation was given gel for induction of labor. Later she was found
to have 4 cm dilated cervix and 70% effaced. However her contractions lasted 2 minutes and
fetal HR dropped from 140 to 80. What to do?
a. Give Oxygen
b. Immediate c-section
c. Give some medication
Answer: B
Reference: Toronto Notes. OB47
69- What is the most common sign and symptom in placental abruption ?
a. Vaginal bleeding
b. Uterine tenderness
c. Uterine contractions
d. Fetal distress
Answer: A
Placental abruption is mainly a clinical diagnosis with all the above findings. the most common
symptom is dark red vaginal bleeding with pain during the third trimester of pregnancy (80%)
and abdominal or uterine tenderness (70%).
Bleeding may occur at various times in pregnancy:
Bleeding in the first trimester of pregnancy is quite common and may be due to the following:
miscarriage (pregnancy loss) ectopic pregnancy (pregnancy in the fallopian tube)
Bleeding in late pregnancy (after about 20 weeks) may be due to the following: placenta previa
or placental abruption.
Reference: AlQassim Booklet. Q84
70-A patient did a PAP smear and the result showed high grade intraepithelial cells. what will
you do next?
a.
Cone biopsy
b.
Total hysterectomy
smle ,2016

429
c.
Colposcopy
Answer: C ( Refer to the chart at the end of OB/GYN section.\
71- (long scenario) female patient with bacterial vaginosis. What is the most appropriate
treatment?
a.
Ceftriaxone.
b.
Clindamycin.
c.
Ampicillin.
d.
Fluconazole.
Answer: B
Refer to the table at the end of OB/GYN section
72- 30 year old lady having whitish vaginal discharge, odorless and labial erythema. What is
the most likely cause ?
a.
Candida infection.
Answer: A
Candida vulvovaginitis: Erythematous, excoriated vulva/vagina with thick white discharge without odor.
73- 34 year old lady pregnant, complaining of amenorrhea, bleeding, and abdominal pain. Bhcg done showed levels of 1600, she was given methotrexate. One week later she still has abdominal pain despite analgesia. B-hCG done showed 6000 units. What is the best management?
a.
Continue methotrexate.
b.
Exploratory laparoscopy.
c.
Salpingectomy
d.
Salpingostomy
Answer: B
74- 15 years old girl, presented with pain during menses only. What is the nerve and the muscle involved?
Answer: ?
Dysmenorrhea is due to prolonged uterine contractions and decreased blood flow to the myometrium.
Nerve: uterine nerve. Reference: Medscape
75- Sexually abused child vaginally, the hymen tear will be in which position?
a.
2 o'clock
b.
4 o'clock
c.
6 o'clock
Answer: C ????
Blunt penetrating trauma to the vaginal orifice produces a characteristic pattern of injury;
bruising, lacerations and/or abrasions are typically seen between the 4 and 8 oclock positions of the hymen.
Reference (1) : WHO -Child sexual abuse
smle ,2016

430
http://www.who.int/violence_injury_prevention/resources/publications/en/guidelines_chap7.pdf

Reference(2)https://books.google.com.sa/books?id=zbjiCQAAQBAJ&pg=PA448&lpg=PA448&dq=vaginally,+the+hymen+tear+position+o%27clock&source=bl&ots=TbuUb8hgfl&sig=lJx9tJhAto-xqSdRcKe4x9xWijI&hl=ar&sa=X&ved=0ahUKEwiFiMaKqMbNAhXJtBoKHYskAq4Q6AEIQzAE#v=onepage&q=vaginally%2C%20the%20hymen%20tear%20position%20o'clock&f=false
76- What is the best time to estimate the chorionicity of the twins on ultrasound?
Answer: 10-13 weeks.
Assessment of chorionicity: Ultrasonography is an effective prenatal tool for determining amnionicity and chorionicity. The optimal time for performing the ultrasound examination is in the
first trimester after 7 weeks (sensitivity 98 percent), with lower but acceptable accuracy in the
early second trimester.
Reference(1): http://www.uptodate.com/contents/twin-pregnancy-prenatal-issues
The first trimester is generally considered to be the ideal time to confirm or establish accurate
gestational age dating, and it is statistically superior to second trimester dating.
Reference(2) : http://sogc.org/wp-content/uploads/2013/01/gui260CPG1106E.pdf
77- A 19 years old white female. Weight: 52 kg & Height: 145 cm with no history of twins in
the family, got spontaneously pregnant by twins. What is the risk factor in this case?
a.
Age
b.
Race
c.
Weight
Answer: B

smle ,2016

431
Dizygotic twins are the most common. Identifiable risk factors include IVF, newly discontinued
OCP, race (e.g. certain African regions), increased maternal age, geography, family history, or
ovulation induction.
References: Kaplan Lecture Note; Ob/Gyn and Toronto Notes
78- The most common cause of postpartum hemorrhage(PPH) is..? Same Q in Page 78 Q43
a.
Uterine atony
Answer: A
Uterine atony is the most common cause of PPH.Reference: Toronto Notes.
The most common cause is poor contraction of the uterusfollowing childbirth.
79- 20 years old female, presented with amenorrhea, short stature and webbed neck. Which
hormone will be affected?
a.
Decreased estrogen
Answer: A
Reference:Uptodate.
80- A mother delivered her first baby with cleft lip and palate. What is the percentage of recurrence for her next pregnancy?
a.
1%
b.
4%
c.
15%
d.
20%
Answer: B
Once parents have a child with a cleft, the risk that the next child ... only one in his/her family
with a cleft has a 2-5% chance that his or her child
Reference:
http://cleftline.org/docs/Booklets/GEN-01.pd f
81-Pregnant lady with a history of 2 SVD and 1 CS.How will you manage?
A. Natural vaginal delivery trial

B. Admit at 38 weeks for CS


Answer: A
82- A pregnant lady 34 weeks gestation presents with headache, epigastric pain and blurred
vision with a BP of 163/89. What is the best course of management?
a.
Stabilize the general condition, magnesium sulphate, .
b.
Deliver immediately
c.
Give magnesium sulphate, stabilize the general condition.
Answer: A, obs/gyne consultant said (stabilize then deliver).
ABC > MgSO4 + hydralazine + Delivery
83- Pregnant female is HIV positive. What is the most likely mode of transmission to the
baby?
smle ,2016

432
a.
Through the placental
b.
Through the blood cord
c.
By breast feeding
d.
Through hand contamination of mother
Answer: A
An HIV-positive mother can transmit HIV to her baby in three ways:
1. During pregnancy
2. During vaginal childbirth
3. Through breastfeeding
https://www.aids.gov/hiv-aids-basics/prevention/reduce-your-risk/pregnancy-and-childbirth/
84- What is the best place to take a cervical sample for Pap smear?
a.
Endocervix Cancer,
b.
Exocervix
c.
Transformation
d.
Vaginal vault
Answer: C
The transformation zone is the site of origin for most cervical neoplasia and should be the focus
of cytology specimen,collection.
References:
1. http://www.cytopathology.org/specimen-collection-adequacy-requisition/
https://books.google.com.sa/books?id=0flWgd3OJLEC&pg=PA11&lpg=PA11&dq=-#v=onepage&q&f=false
2. Gynaecology by Ten Teachers, 19th Edition
3. Hacker &Moores Essentials of Obstetrics and Gynecology 5th
85- What to do after a Pap smear show atypical changes ?
a.
Hysterectomy
b.
Guided colposcopy biopsy
c.
Excisional biopsy
Answer: B
Refer to the chart at the end of OB/GYN section.
1. http://www.mayoclinic.org/tests-procedures/pap-smear/basics/results/prc-20013038
2. Gynaecology by Ten Teachers, 19th Edition
3. Hacker &Moores Essentials of Obstetrics and Gynecology 5th
86- Uterine Myoma was found incidentally by US. What will you tell the mother?
a.
Regress normally
b.
May cause abortion
Answer: ? I think the first choice is not completed !
Almost 90 percent of women with fibroids detected in the first trimester will have regression in
total fibroid volume when re-evaluated three to six months postpartum, but 10 percent will
have an increase in volume. Regression may be less in women who use progestin-only contraception.

smle ,2016

433
Reference:
1. http://www.uptodate.com/contents/pregnancy-in-women-with-uterine-leiomyomasfibroids?source=outline_link&view=text&anchor=H26#H26
2. http://www.ncbi.nlm.nih.gov/pmc/articles/PMC3136622/
However, it's possible that fibroids could cause infertility or pregnancy loss. Submucosal fibroids may prevent implantation and growth of an embryo. In such cases, doctors often recommend removing these fibroids before attempting pregnancy or if you've had multiple miscarriages.
Reference:
1. http://www.mayoclinic.org/diseases-conditions/uterinefibroids/basics/complications/con-20037901
-Similar Q was found in AlQassim Booklet (Q509) and UQU (Q64):
87-Pregnant women has fibroid with of the following is True?
a.
Presented with severe anemia .
b.
Likely to regress after delivery .
c.
Surgery immediately.
d.
Presented with antepartum hemorrhage.
Answer: B
Fibroids may also be the result of hormones. Reproductive hormones like estrogen and progesterone can stimulate cell growth, causing fibroids to form. During pregnancy, your influx
of hormones may cause your fibroids to grow in size. After pregnancy and during menopause most fibroids begin to shrink, due to a lack of hormones.
88- A female tried to get pregnant for one year. She is healthy and her husband is known to
be healthy as well. What to do 1st?
a.
Hysterosalpingography
b.
Serum prolactin level of the woman
c.
Semen analysis
d.
Serum progesterone
Answer: C
since the woman is healthy we should investigate man, most common factor for them is semen
abnormalities, therefore, semen analysis and culture.
Reference:
1. http://www.cdc.gov/reproductivehealth/infertility/

smle ,2016

434

89- What are the steps in staging cervical cancer?


a.
Colposcopy, cystoscopy, hysteroscopy
b.
Colposcopy, Colonoscopy, Hysteroscopy
c.
Laparoscopy
Answer: A
Reference:
1. http://emedicine.medscape.com/article/253513-workup
90- A pregnant during labor. Her cervical opening is 6 cm. Which stage?
Answer: ?

Reference:
1. Obstetrics by Ten Teachers, 19E Page: 192

smle ,2016

435
91- Pregnant lady, everything was normal except hemoglobin was low. What is the next step?
a.
Iron
b.
Nothing
c.
Folate
d.
B12
Answer: A
Reference:
1. Step-Up To Obstetrics & Gynecology
2. http://www.ncbi.nlm.nih.gov/pmc/articles/PMC1447059/
Iron supplementation is almost universally recommended during pregnancy to correct or prevent iron deficiency.
Iron deficiency anemia accounts for 75-95% of the cases of anemia in pregnant women.
whilefolate deficiency is much less common than iron deficiency. A woman who is pregnant often has insufficient iron stores to meet the demands of pregnancy. Encourage pregnant women
to supplement their diet with 60 mg of elemental iron daily. The clinical consequences of iron
deficiency anemia include preterm delivery, perinatal mortality, and postpartum depression.
Fetal and neonatal consequences include low birth weight and poor mental and psychomotor
performance. Medscape
92- What is the best investigation to establish ectopic pregnancy?
a.
HCG
b.
Laparoscopy
c.
Pelvic U/S
Answer: B
-HCG: 85% of ectopic pregnancies demonstrate abnormal -HCG doubling
U/S: is only definitive if fetal cardiac activity is detected in the tube or uterus
Laparoscopy: for definitive diagnosis/ invasive
Reference:
2. Toronto Notes + Master the boards
3. Step-Up To Obstetrics & Gynecology
4. Hacker &Moores Essentials of Obstetrics and Gynecology 5th

93- 40 y/o woman G3P1, history of 2 months pregnancy. Upon examination her uterus is
large for gestational age. HCG: very high, U/S: no fetus or heart sounds. She was diagnosed to
have cancer which is sensitive to chemotherapy and easily treated. What does she have? (She
had molar or ectopic pregnancy before)
a.
Endometriosis
b.
Gestational Trophoblastic Disease.
c.
Ovarian CA
Answer: B
Hacker &Moores Essentials of Obstetrics and Gynecology 5th

smle ,2016

436
94- Pregnant lady known case of DM 1 presented with persistent hyperglycemia even after
adjustment of her insulin dose. What is the most likely cause?
a.
Maternal hyperglycemia
b.
Maternal hypoglycemia
c.
Fetal Hyperglycemia
d.
Fetal Hypoglycemia
Answer: ?
Similar question in AlQassim Booklet but they were asking about the complication (Answer:
Fetal Hypoglycemia
95- 40 weeks of gestation primigravida presents with hypoxia, drowsiness and agitation for 6
hours, LL edema (Long scenario with blood tests). What does she have?
a.
Amniotic Emboli
b.
PE
Answer: B
Patients with preeclampsia with severe features display end-organ effects and may complain of
the following:

Headache
Visual disturbances: Blurred, scintillating scotomata
Altered mental status
Blindness: May be cortical [3] or retinal
Dyspnea
Edema: Sudden increase in edema or facial edema
Epigastric or right upper quadrant abdominal pain
Weakness or malaise: May be evidence of hemolytic anemia
Clonus: May indicate an increased risk of convulsions

Signs and symptoms of amniotic fluid embolism might include:


Sudden shortness of breath
Excess fluid in the lungs (pulmonary edema)
Sudden low blood pressure
Sudden failure of the heart to effectively pump blood (cardiovascular collapse)
Life-threatening problems with blood clotting (disseminated intravascular coagulopathy)
Altered mental status, such as anxiety
Chills
Rapid heart rate or disturbances in the rhythm of the heart rate
Fetal distress, such as a slow heart rate

smle ,2016

437
Seizures
Coma
Sudden fetal heart rate abnormalities
Bleeding from the uterus, incision or intravenous (IV) sites
So amniotic embolism does not cause lower limp edema
96- 35 weeks of gestation, come mention she doesnt feel the baby movement. Fetus is dead.
She is hypoxic with decreased DLCO (75% Normal 80%) Blood test: Low Hct, prolonged PT and
PTT. What does she have?
a.
Amniotic embolism
b.
DIC
c.
ITP
Answer: A ?
Amniotic fluid embolism causes DIC and ARDS together.
97- Women 34 weeks pregnant, however fundal height is 28cm. What is the most likely cause
of IUGR?
a.
GDM
b.
Oligohydramnios
c.
Polyhydramnios
Answer: B
http://emedicine.medscape.com/article/261226-overview#a2
most fetuses of diabetic mothers exhibit growth acceleration, growth restriction occurs with
significant frequency in pregnancies in women with preexisting type 1 diabetes
so most probably the cause in this case is B
98- Treatment of community acquired pneumonia in pregnancy?
Answer: ?
For pregnant women:
Community acquired pneumonia and no features of severe disease: antipneumococcal
beta-lactam (ceftriaxone, cefotaxime, ampicillin-sulbactam) plus azithromycin
Allergic reactions to cephalosporins: clindamycin plus aztreonam, unless they have severe
pneumonia.
severe pneumonia and past reactions to cephalosporins: vancomycin plus azithromycin plus
aztreonam.
Reference:http://www.uptodate.com/contents/treatment-of-respiratory-infections-in-pregnant-women
99- Women with itchy, whitish vaginal discharge. KOH shows pseudohyphae. What is the
treatment?
a.
Ointment Miconazole
smle ,2016

438
Answer: A
Refer to the table at the end of OB/GYN section
100- Women developed itchy rash with whitish vaginal discharge after nitrofurantoin course
for UTI. How will you treat this patient?
a.
Miconazole
Answer: A
Refer to the table at the end of OB/GYN section.
101- Anovulatory female. What will you give to induce ovulation?
a.
Clomiphene
b.
Danzo
c.
Pulsatile push of LH
Answer: A
infertility
medical induction of ovulation: clomiphene citrate, human menopausal gonadotropins
(HMG [Pergonal]), LHRH, recombinant FSH, and metformin
Metformin may be used alone or in conjunction with clomiphene citrate for ovulation
induction
ovarian drilling (perforate the stroma), wedge resection of the ovary
bromocriptine (if hyperprolactinemia)
REFERENCE :
TORONTO NOTE
102- Calculate day of delivery.
Answer:?
EDC using Naegles Rule: 1st day of LMP + 7 d 3 mo e.g. LMP = 1 Apr 2013, EDC = 8 Jan 2014
(modify if cycle >28 d by adding number of d >28) Reference: Toronto Notes.
103- Calculate day of delivery.
Answer:?
EDC using Naegles Rule: 1st day of LMP + 7 d 3 mo e.g. LMP = 1 Apr 2013, EDC = 8 Jan 2014
(modify if cycle >28 d by adding number of d >28) Reference: Toronto Notes.
104- Woman with bilateral ovarian abscess what dx
Answer:
Pelvic inflammatory disease
The major complication of pelvic inflammatory disease are tuboovarian abscess,chronic pelvic
pain, infertility and ectopic pregnancy
105- Vaginal infection lead to neonatal conjunctivitis
Answer:
The most common bacteria that can cause serious eye damage are gonorrhea and chlamydia.
These can be passed from mother to child during birth.
smle ,2016

439
Time of onset:
Neisseria gonorrhoeae: Delivery of the baby until 5 days post-birth (Early onset)
Chlamydia trachomatis: 5 days post-birth to 2 weeks (Late onset - C.trachomatis has longer incubation period)
Wiki, Medline pluse
106-female with clear presentation of UTI. history of URTI. urine analysis showed nitrate :
+ Esterase : + what is the organism:
A.
Klebsiella pneumoniae
B.
E.coli
C.
Pseudomonas
Answer: E. coli
nitrate test is commonly used in diagnosing urinary tract infections (UTI). A positive nitrite test
indicates that the cause of the UTI is a gram negative organism, most commonly Escherichia
coli
A leukocyte esterase test (LE test) is a urine test for the presence of white blood cells and
other abnormalities associated with infection.
White blood cells in the urine usually indicate a urinary tract infection.also used to screen for
gonorrhea and for amniotic fluid infections.
The combination of the LE test with the urinary nitrite test provides an excellent screen for establishing the presence of a urinary tract infection (UTI).
107-A girl with bilateral ovarian abscess with fever ?
A.
immediate laparotomy
B.
laparoscopic
C.
trans us drain
D.
antibiotic
Answer:
hospitalization, IV antibiotics, if not improved after 48 hr
first-line therapy typically uses a second generation cephalosporin with anti-anaerobic activity
and it is still inconclusive whether additional anti-anaerobic coverage is needed above and beyond the second-generation cephalosporin, cefoxitin or cefotetan, plus doxycycline , proceed
to trans US drain.
up to date...
108- Trichomonas vaginosis read about treatment?
Answer: oral metronidazole for both the patient and her sexual partner .
109- Diagnosis of trichomonas vaginosis?
A. yellow discharge
B. white discharge
C. clue cell
Answer: yellow-green, and frothy
clue cells will be in bacterial vaginosis not in trichomonas

smle ,2016

440
110 -Yellow secretion pv with pmn >10 ?
Answer: trichomonas

111- Self breast examination decrease breast cancer by years ?


A. 1 year
B.
2year
C.
3 year
D. 4year
Answer:
recommend CBE be performed at least every 3 years starting between ages 20 and 39 and annually starting at age 40. The ACOG,[14] ACR,[28]and AMA[29] recommend starting CBE at age 40
and annually thereafter.
Ref: Medscape
112- Adenomyosis treatment?
A. hysterectomy
B.
ocp
C.
gonadotropin analogue
Answer: hysterectomy is the definitive surgical treatment
GnRH agonist , NSAIDs and OCPs may be used for bleeding and pain
113- Pt has a 3 years infertility and have breast milk the lab show
Hight wbc and hight prolactine what visual field will be
affected:
A.
B.
C.
D.

Upper outer
Di ....
???
???

Answer:
if its a pituitary adenoma it will present with bitemporal hemianopia.loss in peripheral visual
field due to the pressure on optic chiasm
114- Cervical insufficiency: the canal is less than
A. 10 mm

smle ,2016

441
B. 20
C. 30
D. 40 mm

Answer: B
We make a diagnosis of cervical insufficiency in women with one or two prior second-trimester pregnancy losses or preterm births and cervical length <25 mm on TVU examination or
advanced cervical changes on physical examination before 24 weeks of gestation. Risk factors
for cervical insufficiency support the diagnosis.
Reference: http://www.uptodate.com/contents/cervical-insufficiency?source=search_result&search=cervical+insufficiency&selectedTitle=1%7E36
115- while giving birth she lost sensation in the medial thigh: what nerve?
A- Pudendal,
B-obturator
Answer: B
-Lithotomy positioning during delivery or in gynecologic/urologic procedures also has been associated with compressive femoral neuropathy.
The sensory branch of the femoral nerve, the saphenous nerve, innervates skin of the medial
thigh and the anterior and medial aspects of the calf.
-The cutaneous branch of the obturator nerve supplies the skin of the middle part of the medial thigh.
Ref :http://emedicine.medscape.com/article/1141793-overview#a7
116- Pregnant early, low appetite nausea fatigue. Blood: low Hb high mvc mcvc high tibc:
what's the reason
Answer: macrocytic anemia( high MCV, High MCHC ) due to vitamin b12 or folate deficiency .
while High TIBC is due to pregnancy .
117- Woman with IUD and came with vaginal pain and discharge (what organism)
Answer:
Actinomyces infections in association with IUD use have been reported.
and PID ( Actinmuces ) is the most common infection associated with IUD ( resource : First aid
Obs&Gyne )
118- Sever symptoms of preclampsia
A.
Abdominal pain
B.
high urea
C.
high blood pressure
Answer:
A. abdominal pain due to hepatic sub capsular swelling from edema which may develop to
hematoma or Hepatic rupture even its rare but can occur .(resource : Lippincott
Obs&gyne recall )
the other options can occur in mild preeclampsia too.

smle ,2016

442

Severe preeclampsia

Blood pressure: 160 mm Hg or higher systolic or 110 mm Hg or higher diastolic on two occasions at least six hours apart in a woman on bed rest
Proteinuria: 5 g or more of protein in a 24-hour urine collection or 3+ or greater on urine
dipstick testing of two random urine samples collected at least four hours apart
Other features: oliguria (less than 500 mL of urine in 24 hours), cerebral or visual disturbances, pulmonary edema or cyanosis, epigastric or right upper quadrant pain, impaired liver
function, thrombocytopenia, intrauterine growth restriction

119- Pt in 30th week G2P2 came to usual checkup. The previous pregnancy was emergent
c/s by examination everything is normal and there is low transverse section in the abdomen
with double suture of uterus or something like that what is the plan for her
A. Cc/s
B. natural vaginal delivery.

Answer Shes candidate of TOLAC ( trial of Labor after cesarean) because she underwent one
low-transverse CS and everything is normal with no other uterine scar or previous rupture
Resource : first aid for obs&Gyne
120-DVT prophylaxis for pregnant woman ?
Answer: heparin or LMWH Resource : first aid for obs&Gyne

121- Pregnant in 38th week, by examination the fetus in breech position, the cervix is closed.
What is the next step?
C- Deliver her by c/s
Answer: Criteria for Vaginal Breech Delivery
Frank or complete breech, GA >36 wk
EFW 2,500-3,800 g based on clinical and U/S assessment (5.58.5 lb)
Fetal head flexed
Continuous fetal monitoring
2 experienced obstetricians,assistant, and anesthetist present
Ability to perform emergency C/S within 30 min if required
Ref: Toronto Note 2016
smle ,2016

443

122- Risk factor of recurrent UTI in females:


A- (Whipping from back to front)
Answer: Women get UTIs more often than men primarily due to anatomy. In women, the opening of the urethra is closer to the external genital area and anus, so bacteria are more likely to
enter the urethra, and the urethra is shorter so bacteria are more likely to move up it.
Ref: https://www.uhs.umich.edu/uti
UTI Risk Factors:
Stasis and obstruction
Residual urine due to impaired urine flow e.g. PUVs, reflux, medication, BPH, urethral stricture, cystocele, neurogenic bladder
Foreign body
introduce pathogen or act as nidus of infection e.g. catheter, instrumentation
Decreased resistance to organisms
DM, malignancy, immunosuppression, spermicide use, estrogen depletion, antimicrobial use
Other factors:
Trauma, anatomic abnormalities, female, sexual activity, fecal incontinence
Ref: Toronto Note 2016
123- Tubal ligation then c/o vaginal spotting after 6 wk of amenorrhea
Answer:
Delayed complications of laparoscopic tubal ligation include the following:
- Failure
- Filshie clip complications
- Regret
- Ectopic pregnancy
- Menstrual changes
- Hysterectomy
- Sexual function
Ref: http://emedicine.medscape.com/article/1848429-overview#a6
124- Pregnant lady with lobar pneumonia, what is the type of immune the baby will have?
Answer: No choices but the answer is passive immunity
ADDED QUESTION: ( need to be more checked )
125- Typical case of PCOS what is skin manifestation associated with?
Answer:
A Acanthosis nigricans
http://www.ncbi.nlm.nih.gov/m/pubmed/17645376/
126. G8P7 in operation room she tell you that she had after all previous pregnancies severe postpartum hemorrhage what you will do you do to pt:
A. give her crystalloid I.V during C/S or labour.

smle ,2016

444
B. active third stage.

Answer: B
Routine oxytocin administration in third stage of labor can reduce the risk of PPH by > 40%
(Toronto notes)
127. 40 year old lady early pregnant, what is useful for her:
A. urine dip steak.
B. .blood group and rh factor
C. .ultrasonography

Answer: 3
8-12 weeks GA> blood group and Rh
(Toronto notes)
128. What is the best drug given to prevent postoperative thromboembolism?
A.
B.
C.
D.

LW heparin
Uf heparin
Warfarin
eno.,,

Answer: 2
Unfractionated heparin (UFH) may be preferred if the patient is likely to have immediate surgery because of its shorter half-life and reversibility with protamine compared with LMWH.
(Medscape)
129. Lady with metromenorrhagia, from 6 month ago and abdominal pain interfere with her
activity , what is the best drug ?
A. hysterectomy
B. OCCP
C. estrogen analogous

Answer: I dont know


-NSAID used for relief of mild to moderate pain. Inhibits inflammatory reactions and pain by decreasing activity of cyclooxygenase, which is responsible for prostaglandin synthesis.
Acute bleeding(stabilize and Iv estrogen or d&c)
Chronic bleeding
- (anatomical or organic problem>>Iv estrogen or d&c if no response after 24 hrs.
-(ocps -long progestin -NSAID)
http://emedicine.medscape.com/article/255540-medication#7
Management of dysfunctional uterine bleeding:
(acute, severe DUB)
Replace fluid losses, consider admission
a) estrogen (Premarin) with dimenhydrinate or anti-fibrinolytic (e.g. Cyklokapron)
b) any OCP with minimum 50 g estradiol with dimenhydrinate
*after (a) or (b), maintain patient on monophasic OCP for next several months or consider alternative medical treatment
*Clomiphene citrate: consider in patients who are anovulatory and who wish to get pregnant
*Surgical: endometrial ablation; consider pre treatment with danazol or GnRH agonists
smle ,2016

445
If finished childbearing
*Repeat procedure may be required if symptom reoccur especially if <40 yr hysterectomy: definitive treatment
130. What is the best ttt of endometriosis?
Answer:depends on certainty of the diagnosis, severity of symptoms, extent of disease, desire
for future fertility, and impact to GI/GU systems (e.g. intestinal obstruction)
medical
NSAIDs (e.g. naproxen sodium Anaprox)
-pseudopregnancy:
1-cyclic/continuous estrogen-progestin (OCP)
2-medroxyprogesterone (Depo-Provera)
3-dienogest (Natazia)
pseudomenopause
1-2nd line: only short-term (<6 mo) due to osteoporotic potential with prolonged use,
unless combined with add-back therapy (e.g. estrogen/progesterone or SERM); if long-term use
required, add-back estrogen+progesterone
2-danazol (Danocrine): weak androgen
3-leuprolide (Lupron): GnRH agonist (suppresses pituitary)
can use 12 mo with add-back progestin or estrogen
surgical
conservative laparoscopy using laser, electrocautery laparotomy
ablation/resection of implants, lysis of adhesions, ovarian cystectomy of endometriomas
definitive: bilateral salpingo-oophorectomy hysterectomy
Ref: torento note
131- 30 years old female has 1 child ,Want to delay pregnancy 3 years later, she didn't want
OCCP nor intravaginal Device Doctor advice her for transdermal patch ,
What is the best advice to tell the patient about the patch ?
A.
B.
C.
D.

decrease compliance
increase blood clot
less effective than OCCP (same effect )
less skin complication(skin irritation )

Answer: B
http://www.mayoclinic.org/tests-procedures/ortho-evra/basics/risks/prc-20013014
132. Patient complains of abdominal pain , Missed period ,Ultrasound show douglas pouch
fluid and dark blood , dx ?
A. reputed ovarian cyst
B. ruptured ectopic pregnancy

Answer: B
http://radiopaedia.org/articles/ectopic-pregnancy

smle ,2016

446
133. Patient present at 10 week with painless bleeding not part of conception cervix closed
what the diagnosis
Threatened abortion
Answer please see attached table

Ref: Toronto Note 2014 page 851


134. when do you do US for screening of the fetus?
A. early 2end trimester,
B. late 2end,
C. early 3rd and late 3red

Answer : please see attached table

smle ,2016

447
135. A female postpartum with upper lateral quadrant mass, redness, tenderness,with +ve
lymph nodes:
Answer: Mastitis
The diagnosis of mastitis is made clinically. Lactational mastitis typically presents as a hard, red,
tender, swollen area of one breast associated with fever >38.3 C in a nursing mother. Other
systemic complaints may variably include myalgia, chills, malaise, and flu-like symptoms. In the
early stages of breast infection the presentation can be subtle with few clinical signs, while patients with advanced infection may present with a large area of breast swelling with overlying
skin changes (eg, erythema). Reactive lymphadenopathy can also cause axillary pain and swelling. Septic shock rarely occurs.
In a lactating woman, severe engorgement can be distinguished from mastitis because engorgement is bilateral with generalized involvement [ 1 ].
Laboratory tests are not needed for the diagnosis of mastitis. If the white blood cell is markedly
elevated with a marked left shift, streptococci, particularly Group A streptococci, is more likely
to be present. Culture of the breast milk is useful when the infection is severe, hospital acquired, or unresponsive to appropriate antibiotics [ 5,6 ]. Blood cultures are of little value unless the patient appears septic.
Imaging is useful if lactational mastitis does not respond to supportive care and antibiotics. Ultrasound is the most effective method of differentiating mastitis from a breast abscess and also
allows for an abscess to be drained with ultrasound guidance
Ref: UpToDate
https://yhdp.vn/uptodate/contents/mobipreview.htm?0/28/454
136. pt complains of scanty pubic hair and primary amenorrhea & secondary sexual character
& develop breast with bilateral growing swelling what diagnosis ?
ANDROGEN INSUFFICIENCY ??
Complete androgen insensitivity syndrome is due to mutations that cause severe impairment
of androgen receptor function.
The typical presentation is a phenotypic woman seen for primary amenorrhea and found to
have little or no axillary and pubic hair, testes, no uterus, a 46,XY karyotype, and serum testosterone concentrations in the adult male range. Affected women have unambiguously female
external genitalia. The labia and clitoris are normal or slightly underdeveloped, and the vagina is
either absent or short and blind-ending.
Testes may be located in the abdomen, the inguinal canals, or the labia majora and have the
histologic appearance of undescended testes with a normal or increased number of Leydig cells
and no spermatogenesis. The urogenital tract is characterized by absence or near absence of
mllerian structures; rarely, uterine remnants are present [ 2 ].
Epididymides and vasa deferentia are usually absent, but in one study, the epididymis and vas
deferens were identified histologically adjacent to the testes in 10 of 33 affected children (average age six years) [ 3 ]. Tumors may develop in the cryptorchid testes
smle ,2016

448
Growth pattern follows that of normal girls, but final height is closer to that of normal boys
[ 5 ]. Breast development is that of a normal woman, and overall body habitus is female ( picture 1 ). Axillary and pubic hair is markedly decreased or absent. These women are taller (average height 172 cm) and heavier than normal women [ 6,7 ], suggesting that the Y chromosome
has somatic effects not mediated by the androgen receptor. Psychologic development is feminine, including typical maternal instincts [ 8,9 ]. Although many patients report satisfactory outcome after vaginoplasty, including normal libido and ability to achieve orgasm [ 10 ], women in
whom the vagina remains small may experience sexual problems [ 11 ].
The incidence of CAIS may be as high as 1 in 20,000. In one study, it was the third most frequent
cause of primary amenorrhea (after gonadal dysgenesis and congenital absence of the vagina)
Ref: UpToDate
https://yhdp.vn/uptodate/contents/mobipreview.htm?22/5/22616

smle ,2016

449

Ref: Toronto Note 2014 page 507

137. pt known cervical carcinoma & her baby 8 years what to do for her baby ?
A. .GIVE HER CONTRACEPTION
B. .NOT related to family history

Answer : ?
Explanation :
Cervical cancer may run in some families. If the mother or sister had cervical cancer, the
chances of developing the disease are 2 to 3 times higher than if no one in the family had it.
Some researchers suspect that some instances of this familial tendency are caused by an inherited condition that makes some women less able to fight off HPV infection than others
-Two vaccines against HPV are licensed in most countries.
-vaccines prevent over 95% of HPV infections caused by HPV types 16 and 18
-The WHO recommended target group for vaccination is 913 year old girls who have not yet
become sexually active.
Ref : http://m.cancer.org/cancer/cervicalcancer/detailedguide/cervical-cancer-risk-factors +
WHO
138. Most common complication in hysterectomy?
A. Bladder injury
B. Ureteral injury

Answer: Ureteral injury

smle ,2016

450
The ureter is the organ most at risk during hysterectomy procedures because it lies only one
centimeter lateral to the cervix. The second most likely place of injury is at the level of in fundibulopelvic ligaments at the time of oophorectomy. It should be identified visually by the characteristic peristalsis, and its course traced down towards the bladder .
Ref : UpToDate
https://yhdp.vn/uptodate/contents/mobipreview.htm?14/22/14698
139. Pap smear the way of doing it?
From external surface of the cervix (ectocervix) and the cervical canal (endocervix)
There are two methods for preparing a specimen for cervical cytology: the conventional Pap
smear and the liquid-based, thin layer preparation
For both methods, cells are obtained from the external surface of the cervix (ectocervix) and
the cervical canal (endocervix) to evaluate the transformation zone (squamocolumnar junction), the area at greatest risk for neoplasia.
Ref: UpToDate
https://yhdp.vn/uptodate/contents/mobipreview.htm?34/32/35337?source=see_link
140. pregnant in labor cervical opening 6 cm Which stage
Answer:
Stage 1 active phase
Labor stages :
First Stage of Labour
latent phase
uterine contractions typically infrequent and irregular
slow cervical dilatation (usually to 3-4 cm) and effacement
active phase
rapid cervical dilatation to full dilatation (nulliparous ~1.2 cm/h, multiparous ~1.5 cm/h)
phase of maximum slope on cervical dilatation curve (see Figure 10, OB43)
painful, regular contractions q2-3 min, lasting 45-60 s
contractions strongest at fundus, weakest at lower segment
Second Stage of Labour
from full dilatation to delivery of the baby
mother feels a desire to bear down and push with each contraction
women may choose a comfortable position that enhances pushing efforts and delivery
.upright (semi-sitting, squatting) and LLDP are supported in the literature
progress measured by descent
Third Stage of Labour
separation and expulsion of the placenta
can last up to 30 min before intervention indicated
start oxytocin IV drip or give 10 U IM after delivery of anterior shoulder in anticipation of
placental delivery, otherwise give after delivery of placenta
routine oxytocin administration in third stage of labour can reduce the risk of PPH by >40%
smle ,2016

451
Fourth Stage of Labour
first postpartum hour
monitor vital signs and bleeding
repair lacerations
ensure uterus is contracted (palpate uterus and monitor uterine bleeding)
inspect placenta for completeness and umbilical cord for presence of 2 arteries and 1 vein
3rd and 4th stages of labour most dangerous to the mother (i.e. hemorrhage)
Ref :Toronto Note 2016 Page 869
141. epileptic breastfeeding mother on phenobarbital
A. stop breastfeeding
B. continue breastfeeding

Answer:B
Continue Breast Feeding
Explanation : the American Academy of Neurology and the American Academy of Pediatrics advise that women with epilepsy taking AEDs can breastfeed. If mothers receiving ethosuximide,
phenobarbital or primidone choose to breastfeed, they should exercise caution and closely
monitor the infant for sedation, lethargy and any significant clinical findings.
Ref : Medscape
142. complicated labor switch to c-section when to give antibiotics
A. before c-section
B. after
C. during

Answer:A
a single intravenous dose of a narrow spectrum antibiotic should be administered preoperatively to all women undergoing cesarean delivery
http://www.uptodate.com/contents/cesarean-delivery-preoperative-issues#H9
recent evidence supports the use of pre-incision, broad-spectrum antibiotics which result in less
maternal morbidity with no disadvantage to the neonate.http://www.ncbi.nlm.nih.gov/pmc/articles/PMC3059069/
143. Hormonal changes at menopause in estrogen, LH, FSH.
Answer:
Follicle-stimulating hormone (FSH) levels are higher than luteinizing hormone (LH) levels, and
both rise to even higher values than those seen in the surge during the menstrual cycle. FSH is
the diagnostic marker for ovarian failure. Estrogen levels begin to fall.(Toronto notes GY34).
144. A pregnant in 32 weeks of gestation, she is in true labor, what to do:
a. Call neonatologist, give corticosteroids, strict bed rest
b. Call neonatologist, give corticosteroids, give fluids
c. Call neonatologist, give antibiotics, bed rest.
Answer:B

smle ,2016

452
A is not true coz she is in true labor

145. Best way to diagnose bacterial vaginosis:


A. Gram stain
B. Dark field microscopy
C. PCR
D.culture
Answer:A
Gram's stain Gram's stain of vaginal discharge is the gold standard for diagnosis of BV
http://www.uptodate.com/contents/bacterial-vaginosis#H5
BV is a sexually associated condition. Diagnostic tests include real-time clinical/microbiological
diagnosis, and the current gold standard, the standardized evaluation of morphotypes on Gram
stain analysis.http://www.ncbi.nlm.nih.gov/pmc/articles/PMC2095014/
146. pregnant women in 1st trimester never got chickenpox her antibody titer is zero what is
the best management ?
A-avoid exposure
B-acyclovir 3
C-Varicella vaccine
Answer:A
http://www.cdc.gov/vaccines/pubs/preg-guide.htm
http://www.uptodate.com/contents/vaccination-during-pregnancy-beyond-the-basics#H3

smle ,2016

453
Its contraindicated to take varicella vaccine during pregnancy if pt. got contact there is varicella
zoster immune globulin (VariZIG) that can help to reduce the risk of becoming infected with
chickenpox.
non immune pregnant woman can take the vaccine one month prior to being pregnant or after
delivery. (Toronto notes OB30 )
All non-immune pregnant women should be informed of the risk of varicella infection to themselves and their fetuses. They should be instructed to seek medical help following any contact
with a person who may have been contagious. (II-3B) 5. In the case of a possible exposure to
varicella in a pregnant woman with unknown immune status, serum testing should be performed. If the serum results are negative or unavailable within 96 hours from exposure, varicella zoster immunoglobulin should be administered.http://www.ncbi.nlm.nih.gov/pubmed/22385673

147- lactating women with positive HBVs what to do


A. continue breastfeeding
B. stop breastfeeding

Answer:A
These data support the recommendation of the American Academy of Pediatrics that HBV infection not be considered a contraindication to breastfeeding of infants who receive the HBIG
and HBV vaccine as advised.
Medscape
Is it safe for a mother infected with hepatitis B virus (HBV) to breastfeed her infant immediately
after birth?
Yes. Even before the availability of hepatitis B vaccine, HBV transmission through breastfeeding
was not reported. All infants born to HBV-infected mothers should receive hepatitis B immune
globulin
https://www.cdc.gov/breastfeeding/disease/hepatitis.htm
148-. tamoxifen for breast cancer has metrorrhagia, US showed thick endometrium what to
do next ?
A. endometrial biopsy
B. CA 125 -

Answer: A
The risk of developing endometrial cancer from tamoxifen is low (less than 1% per year).
Women taking tamoxifen must balance this risk against the benefits of this drug in treating and
preventing breast cancer. This is an issue women should discuss with their doctors. If you are
taking tamoxifen, you should have yearly gynecologic exams and should be sure to report any
abnormal bleeding, as this could be a sign of endometrial cancer
An endometrial biopsy is the most commonly performed test for endometrial cancer.
(American cancer society)
In asymptomatic women using tamoxifen, screening for endometrial cancer with routine transvaginal ultrasonography, endometrial biopsy.
smle ,2016

454
ACOG
149- mother in labor you did vaginal examination, touched orbit ridge, nasal and
chin, What is the presentation?
Same Q in page 49 Q61
A. 1-cephalic
B. 2-brow
C. 3-face

Answer: 3
In a face presentation, the fetal head and neck are hyperextended, causing the occiput to come
in contact with the upper back of the fetus while lying in a longitudinal axis. The presenting portion of the fetus is the fetal face between the orbital ridges and the chin.( medscape)
The diagnosis of face presentation is made by vaginal examination in the late first or the second
stage of labor [5]. Palpation of the orbital ridge and orbits, saddle of the nose, mouth, and chin
is diagnostic of face presentation. (uptodate)
150. with heavy menses each 2 wk what to do?
A.endometrial bx
Answer: Investigations
CBC, serum ferritin
-hCG
TSH, free T4
coagulation profile (especially in adolescents): rule out von Willebrands disease
prolactin if amenorrheic
FSH, LH
serum androgens (especially free testosterone)
day 21 (luteal phase) progesterone to confirm ovulation
Pap test
pelvic U/S: detect polyps, fibroids; measure endometrial thickness (postmenopausal)
SHG: very sensitive for intrauterine pathology (polyps, submucous fibroids)
HSG
endometrial biopsy: consider biopsy in women >40 yr
must do endometrial biopsy in all women presenting with postmenopausal bleeding to
exclude endometrial cancer
D&C: not for treatment; diagnosis only (usually with hysteroscopy)
Ref: (toronto notes)

smle ,2016

455
151. the safe vaccine in pregnancy
A.varicella
B. influenza
Answer: B
Inactivated influenza vaccine
Ideally, all women who are pregnant or might be pregnant during the influenza season should
receive the inactivated influenza vaccine as soon as it becomes available and before onset of
influenza activity in the community, regardless of their stage of pregnancy [42]. Vaccination after onset of influenza activity can still be beneficial as long as influenza viruses are circulating.
Inactivated influenza vaccine is administered by intramuscular injection into the deltoid muscle.
(uptodate)
http://www.mayoclinic.org/healthy-lifestyle/pregnancy-week-by-week/expert-answers/vaccines-during-pregnancy/faq-20057799
152. which OCP cause hyper k :
A. levo..
B. nor..
Answer:
Yasmin (ethinyl estradiol+drospirenone progestin) and Yaz causes hyperkalemia (rare side effect, but contraindicated in renal and adrenal insufficiency)
Ref: (Toronto notes)
153.Female in 12 wk gestation develop UTI treated then at 27 wk again had uti what you will
do ?
a. you will treat if asymptomatic
Answer: A
Any asymptomatic UTI in pregnant women should be treated.
Management of asymptomatic bacteriuria in pregnant women includes antibiotic therapy tailored to culture results and follow-up cultures to confirm sterilization of the urine. For those
women with persistent or recurrent bacteriuria, prophylactic or suppressive antibiotics may be
warranted in addition to retreatment. (uptodate)
154.Pregnant woman with significant edema in his hand and foot what will you do ? ( bp :#
160 /
a. give him diuretic
b. low diet salt
c. labtolol
b. observation in hospital
Answer: C
If a pregnant woman's blood pressure is sustained greater than 160 mm Hg systolic and/or 110
mm Hg diastolic at any time, lowering the blood pressure quickly with rapid-acting agents is indicated for maternal safety.

smle ,2016

456
Labetalol has a more rapid onset of action, may be given orally or parenterally, and is generally
preferred as a first-line agent.
(Medscape)
physicians begin antihypertensive treatment when systolic pressure is 160 mmHg or diastolic
pressure is 105 to 110 mmHg .
We suggest labetalol or hydralazine as first-line agents for acute therapy of severe hypertension.
Labetalol We recommend intravenous labetalol for first-line therapy because it is effective,
has a rapid onset of action, and a good safety profile. (uptodate)
155. pregnant with repeated infection wbc 100000, treated and come agin with same symptom what?
Answer:
http://emedicine.medscape.com/article/235054-overview
156.smoker pregnant women the effect to baby
A. macrosomia
B. low birth weight
C. transient tachypnea
Answer:B
The negative impact of cigarette smoking on fetal health is well established. Cigarette smoking
has been associated with numerous adverse outcomes, including spontaneous pregnancy loss,
placental abruption, preterm premature rupture of membranes (PPROM), placenta previa, preterm labor and delivery, low birth weight (LBW), and ectopic pregnancy. . While the pathophysiology is not completely understood, as discussed above, several possible mechanisms related to
impaired gas exchange, direct toxicity, and sympathetic activation have been proposed.( UpToDate)
Cigarette smoking during pregnancy is the most important modifiable risk factor associated
with adverse pregnancy outcomes [1]. In 2002 in the United States, 5 to 8 percent of preterm
deliveries, 13 to 19 percent of term infants with growth restriction, 5 to 7 percent of pretermrelated deaths, and 23 to 34 percent of sudden infant death syndrome (SIDS) deaths were attributable to prenatal smoking.( Uptodate)
http://www.babycenter.com/0_how-smoking-during-pregnancy-affects-you-and-yourbaby_1405720.bc
157.Pregnant woman in labor room , when she delivered her baby, sudden onset of bleeding
from vagina , the baby is not infected , after 2 hours mother onset bleeding from
A. DIC
B. deficiency in factor llx
Answer:? A
PPH is described as primary or secondary: Primary PPH occurs in the first 24 hours after delivery
(also called early PPH) and secondary PPH occurs 24 hours to 12 weeks after delivery (also
called late or delayed PPH). Coagulopathy is both a cause and result of PPH since persistent

smle ,2016

457
heavy bleeding, irrespective of the cause, leads to consumption of clotting factors and hemodilution of remaining clotting factors. UpToDate
Hemophilia A (factor VIII deficiency) is the most common X-linked genetic disease and the second most common factor deficiency after von Willebrand disease (vWD) (Females usually are
asymptomatic carriers). Medscape
158- positive culture of budding yeast in urine what is the management ?
A-Fluconazole
B-caspofungin
-Answer: A
increasing are the numbers of fungal UTI, particularly those caused by Candidaspp, and, to a
lesser extent, by Aspergillusspp and Cryptococcus neoformans. Candiduria is a condition most
often found in elderly, hospitalized, or immunocompromised patients. Candida albicans is the
most common species isolated, accounting for more than half of all fungal infection cases. The
mainstay of antibiotic treatment for candiduria is the azolic compounds, mainly fluconazole 200
mg orally daily for 2 weeks
159. pregnant BP: 170/120 management?
A. Mg sulphate and deliver
B. Mg sulphate and wait till 34w :/
C. Call anesthesia and deliver

Answer: B
When hypertension is first identified during a woman's pregnancy and she is at less than 20
weeks' gestation, blood pressure elevations usually represent chronic hypertension. In contrast,
new onset of elevated blood pressure readings after 20 weeks' gestation mandates the consideration and exclusion of preeclampsia.
If a pregnant woman's blood pressure is sustained greater than 160 mm Hg systolic and/or 110
mm Hg diastolic at any time, lowering the blood pressure quickly with rapid-acting agents is indicated for maternal safety. Anticonvulsant therapy may be undertaken in the setting of severe
preeclampsia (primary prophylaxis) or in the setting of eclamptic seizures (secondary prophylaxis). The most effective agent is IV magnesium sulfate; phenytoin is an alternative, although
less effective, therapy.
Women with suspected, mild, or diagnosed preeclampsia remote from term or labile blood
pressures due to chronic hypertension and/or gestational hypertension should be hospitalized
for close observation, bed rest, and frequent fetal monitoring.
When preeclampsia develops remote from term (ie, < 34-36 weeks' gestation), attempts are often made to prolong the pregnancy to allow for further fetal growth and maturation. Medscape
160. w pregnant cervix closed, baby HR 120 after one hour of induction baby's HR 80 and
contractions last 2 mins. Management?
A.
B.
C.
D.

CS
Oxygen
Oxytocin
Observe

smle ,2016

458
Answer: A
http://www.uptodate.com/contents/c-section-cesarean-delivery-beyond-the-basics#H9
Because of weak contractions + fetal distress
161.Prolong labor. She might have post-partum hemorrhage. How to asses?
A.
B.
C.
D.

Visual blood loss


Hematocrit count
Pulse ??
Hemoglobin

Answer We make the diagnosis of PPH in postpartum women with bleeding that is greater
than expected and causes symptoms (eg, pallor, lightheadedness, weakness, palpitations, diaphoresis, restlessness, confusion, air hunger, syncope) and/or results in signs of hypovolemia
(eg, hypotension, tachycardia, oliguria, oxygen saturation <95 percent) (table 1).
Diagnosis may be delayed in symptomatic women without heavy vaginal bleeding who are
bleeding internally, such as intra-abdominal bleeding related to a cesarean delivery or a broad
ligament or vaginal hematoma due to a sulcus laceration.
162.Best way to avoid transmission of tetanus in pregnant lady to her baby?
A.
B.
C.

Newborn tetanus toxoid


Neonate anti tetanus
Giver the mother early tetanus toxoid

Answer 3
http://www.uptodate.com/contents/tetanus?source=machineLearning&search=neonatal+tetanus&selectedTitle=1~4&sectionRank=3&anchor=H1917252#H1917252
163. pregnant lady miss pregnant symptom since 1week and started complain of
spot bleeding the most valuable investigation in this condition is:
A. hcg-alpha
B. feto-us ...

answer 2
http://www.uptodate.com/contents/overview-of-the-etiology-and-evaluation-of-vaginal-bleeding-in-pregnant-women?source=search_result&search=pregnancy+bleeding&selectedTitle=1~150#H3
164.increase in frequency of menstruation
A. 1.polymenorrhea
B. 2-hyper
C. 3-metro..

answer A
165- 3 questions about contraception.
Answer:
http://emedicine.medscape.com/article/258507-overview
http://www.uptodate.com/contents/contraceptive-counseling-and-selection

smle ,2016

459
166. Pregnant 40 g.a did not follow up,, examination and ultrasound reflect breech presentation,,, in progressive labor pain.. Cervical full dilation and full effacement, intact membrane..
Engagement zero,, what to do ?
A- Amniotomy
B- CS
Answer: B
http://emedicine.medscape.com/article/262159-overview
http://www.uptodate.com/contents/delivery-of-the-fetus-in-breech-presentation?source=search_result&search=breech+delivery&selectedTitle=1~150
167. Pregnant lady with control DM there is hx of IUFD now she is in 32 wk management:
A- Induce labor in 36 wk
B- CS at 38
C- Observe
Answer: B. CS at 38 wks ??
A cesarean delivery (C-section) may be recommended for obstetric indications such as severe
preeclampsia with an unfavorable cervix, estimated fetal weight >4500 grams, history of a Csection, or fetal distress.
Reference:
http://www.endotext.org/
http://www.medscape.com/viewarticle/712289_3
168. Safest antibiotic in breastfeeding?
Answer: 1. Cephalosporins (i.e. Keflex or cephalexin)
2. Erythromycin.
3. Trimethoprim-sulfamethoxazole (Bactrim, Septra) is compatible with breast-feeding, but its
use should be avoided when nursing infants are younger than two months because of its potential for causing increased bilirubin levels.3
4. Metronidazole (Flagyl): cessation of breastfeeding for 12 to 24 hours is recommended. Topical preparations of metronidazole (MetroGel-Vaginal) is safe
5. Fluconazole (Diflucan) is commonly prescribed for yeast infections of the nipple in breastfeeding mothers.
https://www.drugs.com/drug-safety-breastfeeding.html
169. Pt with Vaginal discharge no other symptoms and normal investigation?
Answer:
- clear, white, flocculent, odourless discharge; pH 3.8-4.2
- smear contains epithelial cells, Lactobacilli
- increases with increased estrogen states: pregnancy, OCP, mid-cycle, PCOS, or
premenarchal
- if increased in perimenopausal/postmenopausal woman, consider investigation for
other effects of excess estrogen (e.g. endometrial cancer)
Ref: Toronto Notes 2016

smle ,2016

460
Non infective Physiological During the reproductive years the fluctuating levels of oestrogen
and progesterone throughout the menstrual cycle affect the quality and quantity of cervical
mucus which is perceived by women as a change in their vaginal discharge. Initially, when oestrogen is low, the mucus is thick and sticky. As oestrogen levels rise, the mucus gets progressively clearer, wetter and more stretchy. After ovulation, there is an increase in the thickness
and stickiness of the mucus once more.
170. Acute Salpingitis case?
Answer:
http://emedicine.medscape.com/article/275463-overview#a2
There were also missing questions about the following:
Pap test chart

ASCUS = abnormal squamous cells of unknown significance; LSIL = low grade squamous intraepithelial lesion; HSIL = high grade squamous intraepithelial lesion; ASC-H = abnormal squamous
cells cannot rule out HSIL; AGUS = atypical glandular cells of unknown significance.
Infectious vulvovaginitis

smle ,2016

461

smle ,2016

462

Prenatal Screening and Diagnostic Tests


*CVS chorionic villus sampling, FTS first trimester screen, IPS integrated prenatal screen, PAPPa pregnancy-associated plasma protein a, MSAFP maternal serum a-fetoprotein, OGCT oral glucose challenge test, GBS Group B Streptococcus.

Endometriosis (Presentation, investigations, treatment)


Added Qs 7th update
171- Female 3 children , doesn't want to get pregnant anymore ,has a hx of endometriosis in
ovary, She removed it, now she has another one in the right ovary ..how to manage ?
a.
Hysterectomy bilateral salbingo oophrectomy ,
b.
aspiration of the mass ..?
Answer: A
Definitive: bilateral salpingo-oophorectomy +/- hysterectomy
Reference (Toronto notes GY14)
172- Patient with pelvic inflammatory disease,didn't respond to abx after 3 days examination
revealed fluctuating mass, how to manage?
a.
Laparoscopy .
Answer:
A case of tubo-ovarian abscess?

smle ,2016

463

173-Pregnant lady ,had an outbreak asking for all vaccination can be given what you will give
?
a.
Influenza
b.
MMR ,
c.
rubella ,
d.
varicella
Answer: A
174- Patient in 3rd trimester have high blood blood glucose despite close observation What
is the suspect cause?
A.
neonate hyperglycemia
B.
neonate hypoglycemia
C.
mother hyper
D.
mother hypoglycemia
Answer: C
175- Patient have obesity , hirsutism , HTN , insulin resistance What is the dx? In options
there's no pcos
A.
kallman syndrome
B.
kleinfilter syndrome
I don't know the answer but I think there is another name of pcos
Answer: stein leventhal syndrome
Polycystic ovarian syndrome also called chronic ovarian androgenism, hyperandrogenic anovulation (HA), or SteinLeventhal syndrome
176- Lady with metromenorrhagia, from 6 month ago .. And abdominal pain interfere with
her activity , what is the best drug ?
A.
hysterectomy
B.
Occp
C.
"" i think "" estrogen analogous
Answer:
Management of dysfunctional uterine bleeding:
(acute, severe DUB)
replace fluid losses, consider admission
a) estrogen (Premarin) with dimenhydrinate or anti-fibrinolytic (e.g. Cyklokapron)
b) any OCP with minimum 50 g estradiol with dimenhydrinate

smle ,2016

464
*after (a) or (b), maintain patient on monophasic OCP for next several months or consider alternative medical treatment
*clomiphene citrate: consider in patients who are anovulatory and who wish to get pregnant
*surgical: endometrial ablation; consider pre treatment with danazol or GnRH agonists
if finished childbearing
*repeat procedure may be required if symptom reoccur especially if <40 yr hysterectomy:
definitive treatment
177-Patient complain of abdominal pain .. Missed period .. Ultrasound show dougles pouch
fluid and dark blood , dx ?
A. reputed ovarian cyst
B. ruptured ectopic pregnancy
Answer : B
178-Lady with epithelial cell on UA, what's the cause?
A. Vulvar contamination
B. cervix lesion
Answer: A
http://www.mayoclinic.org/tests-procedures/urinalysis/basics/results/prc-20020390
Epithelial cells cells that line your hollow organs and form your skin in your urine may
be a sign of a tumor. But more often, they indicate that the urine sample was contaminated
during the test, and a new sample is needed.
http://emedicine.medscape.com/article/2074001-overview#a2
Generally 15-20 squamous epithelial cells/hpf or more indicates that the urinary specimen is
contaminated
179- Case of typical presentation of syphilis
Answer: from Toronto notes

smle ,2016

465

12-Snowstorm appearance in pregnant what's the Dx?


Answer:Complete mole(GTD)
Toronto notes

13-Postmenopausal lady taking tamoxifen, which of the following u will carefully assess?
a.vaginal bleeding
Answer:A
-Risk factors for endometrial cancer, Box 41-1 (chronic Tamoxifen use)
-Etiology of postmenopausal bleeding, Table 41-1 (exogenous estrogen 30%)
Reference: Hacker and Moores, page 429, 5th edition

smle ,2016

466
http://www.uptodate.com/contents/medications-for-the-prevention-of-breast-cancer-beyondthe-basicshttp://www.uptodate.com/contents/medications-for-the-prevention-of-breast-cancer-beyond-the-basics
http://www.uptodate.com/contents/medications-for-the-prevention-of-breast-cancer-beyondthe-basics
14-Pregnant in labor, on PV you felt the orbital margin and the nose, what is the presentation
of the fetus?
A. Mento anterior < the most common(60%)
B. Mento posterior(25%)
C. Lateral mento transvers(15%)
D. Medial mento transvers
Answer:A
Reference: Hacker and Moores, 5th edition
Face presentations are classified according to the position of the chin (mentum):
Left Mento-Anterior (LMA), Left Mento-Posterior (LMP), Left Mento-Transverse (LMT); Right
Mento-Anterior (RMA), Right Mento-Posterior (RMP), Right Mento-Transverse (RMT).
A mento-posterior face presentation:

15- A 39 weeks pregnant female. Came with BP 160/95 , no proteinuria. The previous visits
the BP was normal. What is your diagnosis?
A. gestational hypertension
B. chronic HTN
C. preeclampsia
Answer: A, because no proteinuria
Gestational hypertension Definition:
sBP>140 or dBP>90 developing after 20th wk GA in a woman known to be normotensive before pregnancy
(Toronto notes)
16-A G3P0 female with 3 recurrent abortions, on investigating the last abortus you found an
aneuploidy 45X. What are the chances of having this abnormality in her next pregnancies?
A. 30%
B. 40%
C. 50%
D. 60%
Answer: A

smle ,2016

467
The most common cause of spontaneous abortionis a significant genetic abnormality of the
conceptus. In spontaneous first-trimester abortions, about two thirds of fetuses have significant
chromosomalanomalies, with about half of these being autosomaltrisomies and most of the remainder being triploid,Tetraploid, or 45 X monosomies
Fortunately, most ofthese are not inherited from either mother or fatherand are single nonrecurring events.
Reference: Hacker and Moores, page 76, 5th edition
17-A female had an IUD inserted 2 years ago, now she's complaining of lower abdominal pain
and vaginal discharge which was foul smelling few days ago. On examination you found a
right 9-adnexal mass. A gram stain of the cervical discharge showed a gram positive beading
bacillus, what is the most likely causative organism?
a.
Perfringens
b.
Bacteroidesfragilis
Answer: none of the above mentioned answers
Repeated Q
Gram-positive branching filaments of Actinomyces species.
18-G8P7 in operation room she tell you , she had after all her previous pregnancy a severe
postpartum hemorrhage what you will do to prevent bleeding ?
A. give her crystalloid I.V during c.s or labour
B. ACTIVE third stage
answer:B
Routine oxytocin administration in third stage of labor can reduce the risk of PPH by > 40%
19-Early pregnancy , what is useful for her?
A. urine dip steak
B. blood group and Rh factor
C. ultrasonography
answer: C Repeated
20-Pregnant woman in third trimester she as in airline about 18 hours and developed a sudden chest pain , with dyspnea, what is the cause ?
A. pulmonary hypertension
B. pulmonary embolism
C. myocardial ischemia
answer: B
SOB and chest pain symptoms of PE
21-pregnant woman in labour room , when she delivered her baby, sudden onset of bleeding
from vagina , the baby is not infected , after 2 hours mother onset bleeding from mouth and
nose , what is the cause ? A. Dic
smle ,2016

468
B. deficiency in factor llX
answer:repeated
22- Pregnant lady everything normal except hemoglobin low, next step ?
A. Iron Nothing
B. Folate B12
answer :A
23- Pregnant with repeated infection wbc 100000, treated and come again with same symptom what to do ? Treat her even she is asymptomatic ?
answer :treat even asymptomatic
Uptodate
24- Patient present at 10 week with painless bleeding not part of conception cervix closed
what is ?
answer : Threatened abortion
25-Female in 12 week gestation develop UTI treated then at 27 week again had UTI what you
will do ? E. coli >100000 ?
a- treat if asymptomatic
answer : treat even if asymptomatic
26-pt with Hx of lower transverse incision and double uterine fold suture and she is at 37 wk
and doing fine?
A-CS
B-SVD
answer :B ,studies suggested that rupture less likely if double so SVD safe unless there is permanent cause such CPD)
27- 42 weeks preganant with heavy menses each 2 wk what to do
answer
Repeated
28-Young patient with 4 weeks amenorrhea , presented with abdominal pain and severe vaginal bleeding, the bleeding from ?
A-Ovary
B-Fallopian tube
C-Cervix
answer:B,(webmed)
29-Complicated labor switch to c-section when to give antibiotics ?
A. before c-section
B. after
C. during
Answer: A
smle ,2016

469
Single dose prophylactic antibiotic should be used (e.g. Cefazolin 1-2g)
30-Pregnant women at 14 weeks with splenomegaly, low platelet what is diagnosis?
Answer:?
*In normal pregnancy there will be mild asymptomatic thrombocytopenia >70.000
Which will return to normal within 2-12 weeks postpartum!
31- Women during delivery developed dyspnea and hypotension seizure, diagnosis:
Answer: amniotic fluid embolism
32- Pregnant Women with hypotension and low platelet what is type of anesthesia you prefer ?
A. Epidural
B. Pudendal block
C. SpinaL
D. General
Answer: D
33-Follicular cell of ovary what it is orign embryologically???
Answer:coelomic epithelium ,hacker and moores page 30 table 3-1 5th edition.
34-Patient with BPH+HTN what TTT, what is the best management ?
Answer: alpha blocker.
35- MMR vaccine in breastfeeding : A-harm to baby
B-Safe to baby
C-Delayed feeding 72 hours
D- Live attended vaccine
Answer D
36-female was dx with ovarian cancer , she haven't ever used OCP , what will you tell her
daughters ?
A- OCP can protect you from ovarian cancer
B- Bilateral oophorectomy is recommended for you
Answer A
37-in mastectomy what you will cut ?
Answer: Pectorals major muscle
38-female pregnant with twins , in her 34 week ( she had some complication I forget it ) the
doctor said that she needs emergency CS cuz the presentation of twin A might result in fetal
complications , what is the presentation of twin?
A-twin Cephalic- breech
B.Transverse-cephalic
C.Breech-cephalic
D.Cephalic-cephalic
smle ,2016

470
Answer C
39-pregnant lady miss pregnant symptom since 1 week and started complain of spot bleeding
the most valuable investigation in this condition is :
A. hcg
B. alpha feto
C. us
Answer: C?
40- 3 questions on contraception
41-You performed a pudendal nerve block on a woman in labor, which of the following structures will be fully sensitive and not blocked by the anesthesias?
A. Perineal body
B. Urogenital diaphragm
C. Rectum
Answer:C
42-The sensory and motor innervation of the perineum is derived from the pudendal nerve,
which is composed of the anterior primary divisions of the second, third, and fourth sacral
nerves. The pudendal nerves 3 branches include the following:
A. Dorsal nerve of clitoris, which innervates the clitoris
B. Perineal branch, which innervates the muscles of the perineum, the skin of the labia majora
and labia minora, and the vestibule
C. Inferior hemorrhoidal nerve, which innervates the external anal sphincter and the perianal
skin
http://emedicine.medscape.com/article/83078-overview#a1http://emedicine.medscape.com/article/83078-overview - a1
The anal canal also has differing nervous innervations above and below the line. Above the
pectinate line, the nerve supply is visceral, coming from the inferior hypogastric plexus. As is it
visceral, this part of the anal canal is only sensitive to stretch. Below the pectinate line, the
nerve supply is somatic, receiving its supply from the inferior rectal nerves (branches of the pudendal). As it is somatically innervated, it is sensitive to pain, temperature, and touch.
http://fitsweb.uchc.edu/student/selectives/Luzietti/Painful_anus_anorectal_anatomy.htm
42-What you should look for in a case of gonorrhea?
A. Chlamydia trachomatis
Answer: A
The uterine cervix is the most common site of mucosal infection with N. gonorrhoeae in
women.
Symptomatic infection typically manifests as vaginal pruritus and/or a mucopurulent discharge. Some women may complain of intermenstrual bleeding or menorrhagia. Pain is

smle ,2016

471
atypical in the absence of upper tract infection. On examination, the cervix may appear normal or show signs of frank discharge. The cervical mucosa is often friable.Importantly, these
symptoms and signs of gonococcal cervical infection, when present, are indistinguishable
from those observed with acute cervicitis of other causes.
Other sexually transmitted pathogens, including Chlamydia trachomatis, Trichomonas
vaginalis, and Mycoplasma genitalium, as well as herpes simplex virus (HSV) and syphilis,
can cause similar symptoms caused by N. gonorrhoeae.
Use of nucleic acid amplification testing (NAAT) is recommended as the optimal method for
the diagnosis of genital and extragenital infections caused by N. gonorrhoeae.
N. gonorrhoeae not only causes similar clinical syndromes as C. trachomatis but also coexists
in a significant proportion of patients with chlamydial infection. Thus, any testing for N. gonorrhoeae should also prompt testing for C. trachomatis
43-pregnant in 40weeks suddenly become drowsy seizure what is the cause :
A- PE
B- amniotic Emboli.
Answer: B
A woman in the late stages of labor becomes acutely dyspneic with hypotension; she may experience seizures quickly followed by cardiac arrest. Massive DIC-associated hemorrhage follows
and then death. Most patients die within an hour of onset.
Currently no definitive diagnostic test exists. The United States and United Kingdom AFE registries recommend the following 4 criteria, all of which must be present to make the diagnosis of
AFE[1, 13, 14] :
1.
2.
3.
4.

Acute hypotension or cardiac arrest


Acute hypoxia
Coagulopathy or severe hemorrhage in the absence of other explanations
All of these occurring during labor, cesarean delivery, dilation and evacuation, or within
30 minutes postpartum with no other explanation of findings
http://emedicine.medscape.com/article/253068-overview#a5
44-Questions about the contraindications of external cephalic version
Answer:
Barring contraindications, ECV is recommended by several national organizations for all women
with an uncomplicated singleton fetus in breech presentation at term to improve their chances
of having a cephalic vaginal birth.
Contraindications to ECV exist either when the procedure may put the fetus in jeopardy or
when the procedure is very unlikely to succeed
Clearly, if cesarean delivery is indicated for reasons other than breech presentation, ECV is contraindicated. Placenta previa or abruptio placentae, nonreassuring fetal status, intrauterine
growth restriction in association with abnormal umbilical artery Doppler index, isoimmunization, severe preeclampsia, recent vaginal bleeding, and significant fetal or uterine anomalies
are also contraindications for ECV.Other contraindications to ECV include ruptured membranes,
fetus with a hyperextended head, and multiple gestations, although ECV may be considered for
smle ,2016

472
a second twin after delivery of the first.Relative contraindications include maternal obesity,
small for gestational age fetus (less than 10%), and oligohydramnios because they make successful ECV less likely.[24] Previous uterine scar from cesarean delivery or myomectomy may
also be a relative contraindication for ECV.
http://emedicine.medscape.com/article/1848353-overview
45-about Endometriosis best management ? And treatment ..
Answer:
Clinical manifestations of endometriosis fall into three general categories: pelvic pain, infertility,
and pelvic mass. The goal of therapy is to relieve these symptoms. There is no high quality evidence that one medical therapy is superior to another for managing pelvic pain due to endometriosis, or that any type of medical treatment will affect future fertility. Therefore, treatment
decisions are individualized, taking into account the severity of symptoms, the extent and location of disease, whether there is a desire for pregnancy, the age of the patient, medication side
effects, surgical complication rates, and cost.
Treatment options include:
Expectant management
Analgesia
Hormonal medical therapy
Estrogen-progestin oral contraceptives, cyclic or continuous
Gonadotropin-releasing hormone (GnRH) agonists
Progestins, given by an oral, parenteral, or intrauterine route
Danazolhttp://www.uptodate.com/contents/danazol-drug-information?source=see_link
Aromatase inhibitors
Surgical intervention, which may be conservative (retain uterus and ovarian tissue) or definitive (removal of the uterus and possibly the ovaries)
Combination therapy in which medical therapy is given before and/or after surgery
http://www.uptodate.com/contents/endometriosis-treatment-of-pelvic-pain

46-Pregnant in the third trimester with history of recurrent herpes simplex, she is in labour,
during exam no lesions what to do?
A. CS
B. IV acyclovir
C. Do specular exam before
Answer: B
Although treatment during primary infection lessens morbidity, it does not eradicate latent
virus, which can subsequently reactivate. Clinical recurrences are common and can be
treated episodically or prevented with continual antiviral suppression.

smle ,2016

473
Recurrent genital infection refers to the occurrence of genital HSV infection in a patient seropositive for the HSV type recovered from the lesion. The risk of neonatal transmission at
delivery is much lower than in patients with primary or first episode genital infection.
Transmission of herpes simplex virus (HSV) to neonates usually occurs during labor and delivery as a result of direct contact with virus shed from infected sites (vulva, vagina, cervix,
perianal area). Importantly, viral shedding can occur when maternal symptoms and lesions
are absent
Suppressive antiviral therapy is suggested at 36 weeks of gestation through delivery for
women with a history of recurrent genital herpes to reduce the risk of lesions at the time of
delivery
Suppressive therapy reduces the risk of clinical recurrence of HSV and asymptomatic viral
shedding at delivery, and thus the need for cesarean delivery. However, the clinical impact
on neonatal HSV is unknown.
http://www.uptodate.com/contents/genital-herpes-simplex-virus-infection-and-pregnancy?source=see_link
47-about pregnant lady with twins on 24 weeks and you discover one fetal death what to do?
A. Delivery + dexamethasone
B. wait until 34 weeks
C. wait until 37 weeks
Answer : B, wait until 34 weeks
48-Pregnant women has GGT diagnostic what is your action ?
A. do HgA1c
B. start insulin
C. do Random blood Glucose
Answer:?
49-about old female with recurrent fracture ..
A. Estrogen Def
Answer:?
50-about the side effect of OCP ?
A. breast cancer
B. cervical cancer
C. DVT
Answer: C, DVT
Reference: Step-Up To Obstetrics & Gynecology.pdf

smle ,2016

474

51target
HA1C
in
type 1 DM ?
Answer:< 6.5
REFERENCE: http://care.diabetesjournals.org/content/38/Supplement_1/S77

52-Female with no sexual hx presented with amenorrhea and thyroid disease has been excluded what first investigation will you do ?
A. Pregnancy test
B. TSH
C. Prolactin
Answer:
First rule out pregnancy, then hyperprolactinemia.
REFERENCE: http://emedicine.medscape.com/article/252928-workup
http://emedicine.medscape.com/article/252928-workup
53-Questions about cervical cancer (staging, biopsy)

smle ,2016

475

53-Group B strep positive pregnancy and she is in her 24 weeks , when to give prophylactic
antibiotic ?
Answer:during labour
REFERENCE:
Hacker
&Moores Essentials of
Obstetrics
and Gynecology 5th edition
54-Primary
amenorrhea , what is the next investigation ?
Answer:
http://emedicine.medscape.com/article/252928-workup

smle ,2016

476

55You
are
inserting
catheter in male, you are afraid to injure:
Answer:
A- membranous urethra
B- prostatic urethra
answer: A ( narrowest part)
REFERENCE:
http://emedicine.medscape.com/article/451797-overview
56-pregnant women in her 1st trimester never got chickenpox her antibody titer is zero what
is the best management ?
A-avoid exposure
B-acyclovir
C-Varicella vaccine
Answer: A
REFERENCE: http://emedicine.medscape.com/article/451797-overview
Pregnant women who have not had chicken pox, or who are known to be seronegative for
chicken pox, should avoid contact with persons who have chicken pox or shingles and should
promptly inform their clinician of potential exposure.
57-what cross the placenta?
A-rubella
B-mumps
C-HBV
Answer: A
REFERENCE:www.glowm.com/section_view/heading/Infection%20in%20Maternal.../item/173

smle ,2016

477

58-pregnant in labour with 6 cm dilation and 1+ effacement and spontaneous rupture of


membrane, which analgesic to give?
1- Pudendal
2- Cervical
3- General
Answer: anesthesia section
58-pregnant lady no symptoms UTI, no frequency no urgency no dysuria, Positive urine culture? Diagnosis?
Answer: Asymptomatic bacteriuria
http://emedicine.medscape.com/article/2059290-overview#a4
http://emedicine.medscape.com/article/2059290-overview - a4
59-positive culture of budding yeast in urine what is the management ?
A-Fluconazole
B-caspofungin
answer: A
Candidiasis in Medscape:http://emedicine.medscape.com/article/2059290-overview#a4
60-tamoxifen for breast cancer has metrohagia, US showed thick endometrium what to do
next ?
A-endometrial biopsy
B-CA 125
answer:
numerous endometrial abnormalities have been reported with tamoxifen use. These have
included both endometrial adenocarcinoma and uterine sarcoma. ( drug.com )
NCIs PDQ cancer information summary and ACS agree that for women at increased risk for
endometrial cancer due to a history of receiving either estrogen therapy or tamoxifen therapy, there is no indication that routine screening would improve early detection or survival
rates. As with women at average risk, these women generally present with symptoms at an
early stage when the prognosis is good. ( Medscape )
To date there have been no published studies evaluating the effect of endometrial cancer
screening modalities on mortality among women taking tamoxifen for the treatment or prevention of breast cancer. ( SOGC Clinical Practice Guideline )
61- mother in labor you did vaginal examination, touched orbit ridge, nasal and chin What is
the presentation?
A-cephalic
B-brow
C-face
answer: C
62-isotretinoin most feared complication ?birth defect?
Answer:
smle ,2016

478
Isotretinoin is a teratogen highly likely to cause birth defects if taken by women during pregnancy or even a short time before conception. A few of the more common birth defects this
drug can cause are hearing and visual impairment, missing or malformed earlobes, facial dysmorphism, and mental retardation. Isotretinoin is classified as FDA Pregnancy Category X and
ADEC Category X, and use is contraindicated in pregnancy
(Wikipedia)
63-Married female Patient with white vaginal discharge, odorless, Wet test was negative,
KOH test showed Psudohyphae, what is the diagnosis?
Vaginal candidiasis.
Answer:Vaginal candidiasis

64- early pregnant , what is useful for her?


A. urine dip steak
B. blood group and rh factor
C. ultrasonography
answer: C ?
65-female pregnant in her 24 week , came to you for her first prenatal visit , lab are provided
all were normal except that the Hgb is slightly low ( I think it was 10) , what will you do ?
A.Nothing
B.Folate
C.Vit B
D.Iron
Answer D
Iron deficiency is the most common cause of anemia in pregnancy.
66-Pregnant with HIV on medication, used to have 400 copies and now 200 copies on labor?
A.Spontaneous Vaginal delivery
B.Forceps delivery
C.CS
Answer: A

smle ,2016

479

67-Q about OCP side effects on the liver ?


a-hepatocellular carcinoma
b-sinusoidal diletation
c-hepatic adenoma
Answer:C
Hepatocellular adenomas occur mostly in women of childbearing age and are strongly associated with the use of oral contraceptive pills (OCPs) and other estrogens( medscape)
68- Women with negative pregnancy test have vaginal bleeding, Hgb 9 mangement?
A.Blood transfusion
B.Progesterone
C.Conjugated Estrogen
Answer: b
Its DUB the drug of choice progesterone
( Wikipedia)
69-Contraindicated antibiotic in pregnancy?
A.Nitrofurantoin
B.Erythromycin
C.Tetracycline
answer: C
Tetracycline use should be avoided in pregnant or lactating women ( Wikipedia)
*Antibiotics contraindicated in pregnancyMCAT
M Metronidazole
C Chloramphenicol

smle ,2016

480
A Aminoglycoside
T Tetracyclines

70-Pregnant lady with positive nitrite and leukocyte esterase and E.coli?
A. Penicillin
B. Advise her to drink a lot of fluid.
answer: A
uncomplicated UTI
first line: amoxicillin (250-500 mg PO q8h x 7 d)
alternatives: nitrofurantoin (100 mg PO bid x 7 d)
Ref. Toronto notes
71- Lady with metromenorrhagia, from 6 month ago, and abdominal pain that interferes with
her activity, what is the best drug?
A.OCPs
B.Estrogen analogs
C.Hysterectomy
Answer: A
Medical treatment of endometriosis:
-NSAIDs (e.g. naproxen sodium Anaprox)
Pseudo pregnancy-1st line:
-cyclic/continuous estrogen-progestin (OCP)
-IM-medroxyprogesterone (Depo-Provera)or oral dienogest (Natazia)
-Mirena IUS
(Pseudomenopause- 2nd line:
-Only short-term (<6 mo) due to osteoporotic potential with prolonged use,unless combined
with add-back therapy (e.g. estrogen/progesterone or SERM);
-Iflong-term use required, add-back estrogen+progesterone
-Danazol (Danocrine): weak androgen
Side effects: weight gain, fluid retention, acne, hirsutism, voice change
-Leuprolide (Lupron): GnRH agonist (suppresses pituitary)
Side effects: hot flashes, vaginal dryness, reduced libido. Can use 12 mo with add-back progestin or estrogen
Surgical
Conservative laparoscopy using laser, electrocautery laparotomy
Ablation/resection of implants, lysis of adhesions, ovarian cystectomy of endometriomas
Definitive: bilateral Salpingo-oophorectomy hysterectomy
follow-up with medical treatment for pain control not shown to impact on preservation of fertility best time to become pregnant is immediately after conservative surgery
Reference:Toronto notes

smle ,2016

481
72-Pregnant 40 GA did not follow up, examination and ultrasound reflect breech presentation, in progressive labor pain. Cervical full dilation and full effacement, intact membrane. Engagement zero, what to do?
A. Amniotomy
B. CS
Answer: B
The standard of care now in most practices is to deliver all breeches by cesarean birth
+ the patient did not follow up
Reference: Hacker and Moores page-167, 5th edition
73-Missed period 2 months, high b-HCG, examination shows 16 weeks GA. US shows fetus
small for date? Diagnosis:
A.Choriocarcinoma
B.Hydatiform mole
C.Placental site trophoblastic tumor
Answer: B is the only possible answer since it might be a partial mole, which often presents
with a coexistent fetus
Repeated Q
Reference: Hacker and Moores, page 435, 5th edition
74- Patient complains of abdominal pain. Missed period. Ultrasound show Douglas pouch
fluid and dark blood, dx?
A.Ruptured ectopic pregnancy
B.Reputed ovarian cyst
Answer:A
The classic triad of ectopic pregnancy: prior missed menses, vaginal bleeding, abdominal pain
Acutely ruptured ectopic pregnancy: intraperitoneal hemorrhage, severe abdominal pain
Reference: Hacker and Moores, page 291, 5th edition
75-Patient with Invasive Cervical Cancer, you want to stage her cancer, which of the following
tests you should perform?
A. Proctoscopy, colonoscopy, hysteroscopy
B. Proctoscopy, cystoscopy, hysteroscopy
C. Proctoscopy, gastroscopy, hysteroscopy
D. Proctoscopy, laparoscopy, hysteroscopy
Answer: B. Proctoscopy, cystoscopy, hysteroscopy
Repeated
Reference: Hacker and Moores, page 407, 5th edition
76-Single Female came to your clinic one day after condom rupture during vaginal intercourse, she is worried about becoming pregnant. What you will do?
A.Pregnancy test
B.Wait and arrange for appointment after one week
smle ,2016

482
C.Give post-coital contraception
D.Give progesterone only contraception
Answer: C
EMERGENCY CONTRACEPTION,
Hormonal EC (Yuzpe or Plan B, usually 2 doses taken 12 h apart) or post-coital IUD insertion
Hormonal EC is effective if taken within 72 h of unprotected intercourse (reduces chanceof
pregnancy by 75-85%), most effective if taken within 24 h, does not affect an established pregnancy
Post-coital IUDs inserted within 5 d of unprotected intercourse are significantly more effective
than hormonal EC (reduces chance of pregnancy by ~99%)
*Yuzpe method = 98% (within 24 h), decreases by 30% at 72 h
*Plan B levonorgestrel only= 98% (within 24 h), decreases by 70% at 72 h
Reference: First Aid for the OBGYN clerkship, page 201, 3rdedition
77-35 year old female P3+0 with amenorrhea for 6 months, thinning and dryness of vaginal
mucosa, she underwent D & C 3 years ago due to retained placental tissue following one of
her deliveries. On examination: normal cervical canal, normal uterus with non-palpable ovaries. Her hormonal profile is: (numbers was given with the normal range) FSH: high, LH: high
TSH: normal Estradiol: low, What is the diagnosis?
A. Ashermans syndrome.
B. Turner syndrome.
C. Premature Ovarian failure.
Answer: C. Ovarian failure
Although she has a history of D&C which might let us think of Ashermans, these patients tend
to have normal ovulatory cycles with cyclical premenstrual symptoms,
All the other symptoms are symptoms relating to amenorrhea with hypo-estrogenism
Reference: Hacker and Moores, page 359, 5th edition
78-Postpartum patient with bilateral breast engorgement and tenderness, what is the management?
A. Hot compression and continue breastfeed
B. Cold compression and stop breastfeed
C. Oral Dicloxacillin and continue breastfeed
D. Oral Dicloxacillin and pump the milk into bag and discard it
Answer: C. Oral Dicloxacillin and continue breastfeed
79-best prenatal screening in the first trimester for Down syndrome
A. Chorionic villous biopsy
B. Amniocentesis.

smle ,2016

483
Answer: acording to hacker its incresed nuchal translucency ,elevated Bhcg and low plasma
protien A. page 81,5th edition.
80-postmenopausal women has hot flushes what altered enzyme is the reason behind her
symptoms ?
A. TSH
B. estrogen
C. Progenstron
Answer: B
81-postmenopausal lady came with vaginal spotting , on examination there was cystic nodule
in her labia majora , what is the dx ?
A. Bartholin cyst
B. Bartholin carcinoma
Answer:
Repeated
82-female pt obese with regular menstrual cycle , on PE/ she had acne , other examination is
normal , what investigation will you order ?
A. TSH
B. ACTH
Answer:
83-clear scenario abou PCOS and they gave the other name of it (repeated)
when a couple should seek help regarding infertility ?
A. 12 months
B. 6 months
Answer:

84-pregnant lady come to you with splenomegaly and platelet = 50 and uterus on the level of
symphysis pubic , what is Dx :
smle ,2016

484
A- idiopathic thrombocytopenic purpura
B- gestational thrombocytopenia
C- thromboembolic disease
Answer:
85- pregnant lady with hypotension , what type of anesthesia you will given :
a- pudendal
b- epidural
c- general
Answer: anesthesia section
86- pregnant lady with vaginal discharge and + KOH , what is Dx :
A-bacterial vaginisis
answer: A
87- pregnant lady with vaginal discharge caused by n. Gonorrhea , which of the following is
associated with this case :
a- chlamydia
b- HSV
answer:A
88- pregnant lady with whitish vaginal discharge :
a- trichomonas
b- bacterial vaginosis
answer: question is not complete , could be normal vaginal discharge

89- vaginal infection which organism can causes :


a-HSV 1
b- HSV 2
answer: b,HSV 2
90- female with 3 months menses loss what is best action :
a- reassurance
answer:?
91- female last delivery with forceps what is the complication :
a- primary postpartum hemorrhage
b- secondary postpartum hemorrhage
c- uterine inversion
answer:A, primary PPH which occurs within the first 24 PP
92-female with ovarian mass, hysterectomy was done, specimen showed thecal cell tumor,
what other things you would find in the specimen?
smle ,2016

485
a. Moles in the uterus
b.Endometrial hyperplasia
c.Others
answer:B
93-pregnent lady miss pregnant symptom since 1 week and started complain of spot bleeding
the most valuable investigation in this condition is :
A)HCG
B)alpha feto
C)US
answer:C
94-increase in frequency of menses:
A) Metrorrhagia
B) hypermenorrhea
C) metrorrhagia
D)Polymenorrhea
answer:D
hypermenorrhea : abnormally heavy or prolonged menstruation; can be a symptom of uterine
tumors and can lead to anemia if prolonged
Metrorrhagia: is uterine bleeding at irregular intervals, particularly between the expected menstrual periodshttps://en.wikipedia.org/wiki/Menstrual_period
Polymenorrhea is the medical term for cycles with intervals of 21 days or fewer.
Oligomenorrhea is the medical term for infrequent, often light menstrual periods (intervals exceeding 35 days).
menometrorrhagia (meno = prolonged, metro = uterine, rrhagia = excessive flow/discharge)
may be diagnosed. Causes may be due to abnormal blood clotting, disruption of normal hormonal regulation of periods or disorders of the endometrial lining of the uterushttps://en.wikipedia.org/wiki/Uterus
https://en.wikipedia.org/wiki/Uterus
https://en.wikipedia.org/wiki/Uterus
https://en.wikipedia.org/wiki/Uterus
95-post menpose women pallor vagina and weak what is the Treatment :
A)estrogen
B)proges
C)diet
answer:A
http://www.medscape.org/viewarticle/556471
http://www.mayoclinic.org/diseases-conditions/vaginal-atrophy/diagnosis-treatment/treatment/txc-20200195

smle ,2016

486
therapeutic standard for moderate to severe vaginal atrophy is estrogen therapy, administered
either vaginally at a low dose or systemically. There has been a relative lack of randomized controlled trials performed to date, but they have shown that low-dose, local vaginal estrogen delivery is effective and well tolerated for treating vaginal atrophy.
96-during pv exam you found cervical mass 1012mm what you will do :
A) reassure ..
B) biobsy..
C)test for human pappiloma virus
answer:C
https://my.clevelandclinic.org/health/diseases_conditions/hic_Understanding_HPV
https://my.clevelandclinic.org/health/diseases_conditions/hic_Genital_Warts/hic_Understanding_HPV
97-Type of fibroid that causes abortion :
answer: submucosal fibroid
https://my.clevelandclinic.org/health/diseases_conditions/hic_Understanding_HPVhttp://www.fibroidsecondopinion.com/fibroids-and-pregnancy/
Fibroids that bulge into the uterine cavity (submucous) or are within the cavity (intracavitary)
may sometimes cause miscarriages. The fertilized egg comes down the fallopian tube and takes
hold in the lining of the uterus. If a submucosal fibroid happens to be nearby, it can thin out the
lining and decreases the blood supply to the developing embryo. The fibroid may also cause
some inflammation in the lining directly above it. The fetus cannot develop properly, and miscarriage may result.Fibroids that bulge into the uterine cavity (submucous) or are within the
cavity (intracavitary) may sometimes cause miscarriages.
98-A girl 15 years old never had menses. Examination breast bed ,fine hair >n After 1 year
come e increase height >6cm And gain kilograms:
A) Primary amenorrhea
B) amenorrhea Precocious puberty
C) Constitutional
answer:https://my.clevelandclinic.org/health/diseases_conditions/hic_Understanding_HPV
http://emedicine.medscape.com/article/252928-overview
99-Postmenopausal with osteoporosis and high Alkaline phosphatase ,What will you give her
:
answer:Bisphosphonate
DISCUSSION: Bisphosphonate treatment lowered ALP levels, and this decrease was strongly correlated with a decrease in BAP. Among blood test data, the decrease in BAP had the strongest
correlation with the ALP decrease.
CONCLUSION: For treatment of osteoporosis, ALP is an acceptable alternative to BAP. Elevated
ALP in postmenopausal women is mainly caused by high bone turnover.
https://my.clevelandclinic.org/health/diseases_conditions/hic_Understanding_HPV
http://www.ncbi.nlm.nih.gov/m/pubmed/25534961/
http://www.ncbi.nlm.nih.gov/m/pubmed/25534961/
100- pt have obesity , hirsutism , HTN, insulin resistance What is the dx? In options there's
no PCOS?
smle ,2016

487
A. kallmann syndrome
B. klinefelter syndrome
I don't know the answer but I think there is another name of pcos
Repeated
101- female with ductal carcinoma. Doctor want treat her, what is the gene responsible for
that cancer? (No BRCA1 in options)
A. tp53
I don't remember the remaining
answer:http://www.cancer.gov/types/breast/hp/breast-ovarian-genetics-pdq

102- what is the antithyroid used in pregnancy?


propylthiouracil
answer:PTU
REFERENCE: www.uptodate.com/contents/antithyroid-drugs-beyond-the-basics
103-17 year old deliver her baby in the home with help of her friend , what type of perineal
muscle tear ?
A. pubococcygeus
B. ischiocavernosus
answer:Repeted
104- 42-year-old patient with PCOS, nulligravida, she never took any medication to regulate
her period, endometrial biopsy showed endometrial hyperplasia, what is the cause?
A. Old age.
B. Unopposed estrogen.
Answer: B,unopposed estrogen.

smle ,2016

488
105- Old female with endometrial biopsy showing: high grade hyperplasia with atypia. What
is the best management?
A. Trial of OCP.
B. Total abdominal hysterectomy.
C. Cauterization.
Answer: B
106- Pregnant female in 24 weeks gestation, known case of chronic DM type 2 and chronic
HTN, fundal height is 25 cm, which one of the following is a complication of her pregnancy?
a. Preeclampsia.
b. Shoulder dystocia.
c. Large infant for gestational age.
Answer: A
Hypertension/preeclampsia (especially if pre-existing nephropathy/ proteinuria): insulin resistance is implicated in etiology of hypertension
107- On examination: her cervix is dilated by 3 cm and effaced by 70% and fetal presenting
part at 0 station. After 6 hours or so, her cervix is dilated by 7 cm and effaced by 80%, but she
is complaining of tenderness when palpated the uterus, her temperature is 38. what is the
best management?
A. Give intrapartum antibiotics.
B. Emergency CS.
C. Wait for spontaneous delivery.
Answer:A ,reference :uptodate

smle ,2016

489
108- Pregnant in 3rd trimester with pain and bleeding, CTG showed late deceleration. Uterus
is distended. What is the diagnosis?
Placenta previa.
Vasa previa.
Abruptio placenta.
Answer:
Causes of late deceleration:fetal hypoxia and acidemia, maternal hypotension or uterine hypertonus (Toronto notes OB35)
Additional Questions ( 8th update)
1- Patient with secondary dysmenorrhea + infertility?
A.Endometriosis.
B.Leiomyoma.
Answer: a
The most common cause of secondary dysmenorrhea is endometriosis. The main clinical features are dysmenorrhea, chronic pelvic pain and infertility.
Reference : www.uptodate.com/contents/antithyroid-drugs-beyond-the-basics
2- Pt with postpartum depression what is Tx:
A.triptaline
B.psychotherapy
answer : B
Sertraline, paroxetine & nortriptyline are the safest & most effective in PPD. Psychological
treatments for PPD are often the treatment of choice for women, as they are effective for the
treatment of depressive symptoms and do not involve the risks of exposure to medications.
Reference : www.uptodate.com/contents/antithyroid-drugs-beyond-the-basics
3- Mother breastfeeding,needs MMR vaccine
A -Give the vaccine
B -Give and stop breastfeeding for 72 hours
C -MMR is harmful to the baby
answer : A
Breast feeding does not interfere with the response to MMR vaccine, and your baby will not be
affected by the vaccine through your breast milk.
Reference : www.uptodate.com/contents/antithyroid-drugs-beyond-the-basics
4- parent have twin male and female, they told you that female get puberty characteristics
and concerned about the male who isn't have puberty features. What they learn regarding
male puberty compared to female:
A.female gets puberty 1-2 years earlier than male
B.female same as male.
answer : A

smle ,2016

490
5- female pt present with laughing and coughing passing out urine O/E there is bulging in labia majora ( stress incontinence )
A. Cystocele
B. Urethrocele
Answer: A

smle ,2016

491

6- You performed a pudendal nerve block on a woman in labor, which of the following structures will be fully sensitive and not blocked by the anesthesias?
smle ,2016

492
A. Perineal body
B. Urogenital diaphragm
C. Rectum
Answer: C
7- pt with polyhydramnios what at risks?
A. kidney
B. esophagus
Answer: B
8- old lady postmenopausal with osteoarthritis and risk for osteoporosis , what you will do :
A. calcium , TSH , dihydroxy vit D
B. bisphosphonate ,vit D ,calcium
c .DEXA scan
Answer: B
9- contraindication of external cephalic version?
answer: Bicornuate uterus
10- pt dont want to get pregnant for years. what will you give her?
A. estrogen
B. androgen
C. estrogen and androgen
Answer: ?
Methods of long acting reversible contraception:
Available LARC methods include IUDs and the subdermal implant:
1- Hormonal intrauterine device (Mirena - also known as IUC or IUS)
2- Nonhormonal intrauterine device with copper (US -ParaGard)
3- Subdermal contraceptive implant (US -Nexplanon/Implanon/Implanon NXT; internationally Norplant/Jadelle)
4- Some shorter-acting methods are sometimes considered LARC:
- Depot medroxyprogesterone acetate injection (DMPA; US - - Depo Provera shot)
- Combined injectable contraceptive
11- Pregnant lady with hypertension. You're mostly concerned about:
A. IUGR.*
Answer:12- Patient with amenorrhea and discharge from her breast with high prolactin level what to
do next:
A - check estrogen level
B - exclude pituitary lesion
C - TSH level
answer: B

smle ,2016

493
Studies should be performed to test for hypothyroidism and renal insufficiency. Magnetic resonance imaging (MRI) of the head should be performed in a patient with any degree of hyperprolactinemia to look for a mass lesion in the hypothalamic-pituitary region, except if the
patient is taking a medication known to cause hyperprolactinemia.
Ref: UpToDate
https://yhdp.vn/uptodate/contents/mobipreview.htm?18/51/19254?source=see_link
https://yhdp.vn/uptodate/contents/mobipreview.htm?18/51/19254?source=see_link
13- Pt pelvic inflammatory disease with salpingitis , On ceftriaxone and no improvement ,
What is the cause?!
A. N gonorrhea
B. Chlamydia
C. Adenovirus
D. Herps
Answer: B
The organisms most commonly isolated in cases of acute PID are Chlamydia trachomatis and
Neisseria gonorrhoeae, should be targeted for treatment;. Ceftriaxone has better coverage
against N. gonorrhoeae.
Ref : UpToDate https://yhdp.vn/uptodate/contents/mobipreview.htm?34/63/35832
https://yhdp.vn/uptodate/contents/mobipreview.htm?34/63/35832
14- Pregnant lady week 12 discovered to have small fibroids, what should she expect?
A-Asymptomatic
B-Degenerates
Answer : A
Most fibroids are asymptomatic. However, severe localized abdominal pain can occur if a fibroid
undergoes so-called red degeneration, torsion (seen most commonly with a pedunculated subserosal fibroid). Pain is the most common complication of fibroids in pregnancy, and is seen
most often in women with large fibroids (> 5 cm) during the second and third trimesters of pregnancy.
Some studies have suggested that small and large fibroids ( 6 cm) have different growth patterns in the second trimester (small fibroids grow whereas large fibroids remain unchanged or
decrease in size), but all decrease in size in the third trimester.
fibroids that did increase in volume, the growth was limited almost exclusively to the first trimester, especially the first 10 weeks of gestation, with very little if any growth in the second and
third trimesters.
Ref :https://yhdp.vn/uptodate/contents/mobipreview.htm?34/63/35832
http://www.ncbi.nlm.nih.gov/pmc/articles/PMC2876319/
15- Pregnant lady presenting lower limb swelling, HTN and mild proteinuria, what will you
give her?
a-Methyldopa
b-ACEI
c-ARB
Answer: A
major goal of treatment of hypertension is to minimize the risk of cardiovascular or cerebrovascular events. Untreated mild hypertension over the course of a pregnancy is unlikely to affect
smle ,2016

494
this risk. On the other hand, untreated severe hypertension, even over the short-term, may be
associated with adverse outcomes.
Mild preeclampsia:
< 37 weeks, expectant Rx in the hospital with no need for antihypertensive or MgSo4 + close
monitoring for both fetus and mother.
> 37 weeks, prompt delivery is the choice
Severe preeclampsia:
Antihypertensive; start treatment with either methyldopa or labetalol . A long-acting calcium
channel blocker (eg, nifedipine ) can be added as either second- or third-line treatment
MgSo4
Prompt delivery regardless the gestational age
For women with preeclampsia between 23 and 34 weeks of gestation, we recommend a course
of antenatal glucocorticoids ( betamethasone)
Ref : UpToDate
https://yhdp.vn/uptodate/contents/mobipreview.htm?13/7/13434#H22
https://yhdp.vn/uptodate/contents/mobipreview.htm?21/23/21882#H6
https://yhdp.vn/uptodate/contents/mobipreview.htm?21/23/21882 - H6
16- Q about the polycystic ovarian Investigation ?
Answer
Polycystic Ovarian Syndrome Workup . Exclude all other disorders that can result in menstrual
irregularity and hyperandrogenism, including adrenal or ovarian tumors, thyroid dysfunction,
congenital adrenal hyperplasia, hyperprolactinemia, acromegaly, and Cushing syndrome.[3, 4, 5]
Baseline screening laboratory studies for women suspected of having PCOS include the following:

Thyroid function tests(eg, TSH, free thyroxine)

Serum prolactin level

Total and free testosterone levels

Free androgen index

Serum hCG level

Cosyntropin stimulation test

Serum 17-hydroxyprogesterone (17-OHPG) level

Urinary free cortisol (UFC) and creatinine levels

Low-dose dexamethasone suppression test

Serum insulinlike growth(IGF)1 level


Other tests used in the evaluation of PCOS include the following:

Androstenedione level

FSH and LH levels

GnRH stimulation testing

Glucose level

Insulin level

Lipid panel
Imaging tests
smle ,2016

495
The following imaging studies may be used in the evaluation of PCOS:

Ovarian ultrasonography, preferably using transvaginal approach

Pelvic CT scan or MRI to visualize the adrenals and ovaries


Procedures
An ovarian biopsy may be performed for histologic confirmation of PCOS; however, ultrasonographic diagnosis of PCOS has generally superseded histopathologic diagnosis. An endometrial biopsy may be obtained to evaluate for endometrial disease, such as malignancy.
Ref : medscape,https://yhdp.vn/uptodate/contents/mobipreview.htm?21/23/21882#H6

17- Delivery can feel nose chin mouth what kind of presentation
a-Face presentation
b-Cephalic presentation
c-Breech presentation
d-Brow presentation
Answer: a
In face presentation the fetal neck is sharply deflexed, allowing the occiput to touch the back
and the face (from forehead to chin) to present in the birth canal
Ref : UpToDate, https://yhdp.vn/uptodate/contents/mobipreview.htm?37/27/38326
18- Best time to check chorionicity and amnionicity of twins?
a-Early 2nd trimester
b-Late 2nd trimester
c-Early 3rd trimester
d-Late 3rd trimester
Answer : A
Ultrasonography is an effective prenatal tool for determining amnionicity and chorionicity. The
optimal time for performing the ultrasound examination is in the first or early second trimester
Ref : UpToDate
https://yhdp.vn/uptodate/contents/mobipreview.htm?39/46/40682#H7
https://yhdp.vn/uptodate/contents/mobipreview.htm?39/46/40682 - H7
19- ovarian cancer tumor marker
Answer: CA125
Measurement of the serum concentration of the CA 125 glycoprotein antigen is the most
widely studied biochemical method of screening for ovarian cancer.

smle ,2016

496

Ref : UpToDate + Toronto Note 2014 page 533


https://yhdp.vn/uptodate/contents/mobipreview.htm?10/14/10474
https://yhdp.vn/uptodate/contents/mobipreview.htm?10/14/10474
20- Sign of fetal distress?
a-Blood loss
b-Early decelerations
c-Late decelerations*
Or various decelerations
Answer ; C
Generally it is preferable to describe specific signs in lieu of declaring fetal distress that include:
1. Decreased movement felt by the mother
2. Meconium in the amniotic fluid ("meconium stained fluid")
3. Non-reassuring patterns seen on cardiotocography:
increased or decreased fetal heart rate (tachycardia and bradycardia), especially during and
after a contraction
decreased variability in the fetal heart rate
late decelerations
Biochemical signs, assessed by collecting a small sample of baby's blood from a scalp prick
through the open cervix in labor
fetal metabolic acidosis
elevated fetal blood lactate levels (from fetal scalp blood testing) indicating the baby has
a lactic acidosis
https://en.wikipedia.org/wiki/Lactic_acidosis
smle ,2016

497
21- Pap smear in old postmenopausal lady, showed ASCUS, was given estrogen cream, FU
PAP showed ASCUS again. Whats your next step?
a-Colposcopy
Answer : A
There are two options for women with an ASC-US Pap test who are ages 21 or older:
Repeat the Pap test in one year. If this test is normal, the woman can return to regular screening. If an abnormality is found, then a colposcopy should be done.
or
Test for human papillomavirus (HPV) infection. for women age 21 years or older with a cervical cytology finding of ASC-US is to have reflex human papillomavirus (HPV) testing (ie, collecting a specimen for HPV testing when the cytology sample is collected), but performing the HPV
test only if the cytologic results are ASC-US. If testing for high risk strains of HPV testing is positive, women should undergo colposcopy
Postmenopausal women Postmenopausal women are managed the same way as premenopausal women [ 4 ]. Cellular changes (eg, nuclear enlargement) associated with postmenopausal vaginal atrophy may be mistaken for ASC. In the past, estrogen administration followed by
repeat cytology was suggested to help distinguish atypical atrophic epithelium, which matured
into normal squamous epithelium with estrogen therapy, from true intraepithelial neoplasia,
which is not affected by estrogen [ 31 ]. However, this is no longer recommended.
Ref: UpToDate, https://yhdp.vn/uptodate/contents/mobipreview.htm?33/6/33895
22- Mother G2 p 1 with Rh + and father - the last baby + and what is % the baby will have - :
a-50%
b-25%
c-100%
d-0%"
Answer: A
Rhesus antigen (Rh-D) is inherited in an autosomal dominant pattern
In order to have a rhesus negative child, neither parent must be homozygous for the D allele.
Some Examples:

23- 34 yo Female after examination with Pap smear you found ASCUS what is your next step:
A - Cone biopsy
smle ,2016

498
B - colposcopy
C - repeat later
D - do HPV test
answer: D
(Toronto notes 2016 GY45)
24- I don't remember the scenario exactly,Female while giving birth full dilatation and effacement the child heartbeat decrease from baseline what is the best analgesic:
A - pudendal
B - paracervical
C - general
D - narcotic anesthesia
answer: C no source
She had to go for CS , epidural anesthesia is not possible if the cervix is dilated more than 5 cm.
(confirmed by student get full mark in Oby/Gyne )
25- Had history of HPV when she was young, you did Pap test and found nothing what to do
now:
A - do nothing
B - repeat every 5 years
C - repeat annually
answer: C no sure
26- typical of polycystic ovarian syndrome diagnosis? hacker page 364.
27- mother close to delivery developed respiratory symptoms + fetal distress :
A- Amniotic fluid embolism
ANSWER: A, not close enough to meet AFE. refer to Toronto notes 2016 OB42.
28- At delivery, baby developed distress , what type of anesthesia given to mother :
A. General anesthesia ?
B. Narcotic analgesia ?
ANSWER : B, no source
29- Long scenario of pt with symptoms of dysfunctional uterine bleeding; you diagnosed her
what medication you'll prescribe:
A- OCP
Answer: A ,Toronto notes 2016 GY12.
30- Nulliparous Patient came to ER with heavy bleeding 18 month history of heavy bleeding
and trying to conceive for 1 year,Vitals given ,Hgb= 10,Pregnancy test -ve; what to give:
A- blood transfusion
B - Progesterone
C - Estrogen

smle ,2016

499
Answer: B, progesterone makes endo thin whereas erstrogen makes endo thick and fragile,
which means more bleed. So the answer is progesterone. http://www.cemcor.ubc.ca/resources/very-heavy-menstrual-flowhttp://www.cemcor.ubc.ca/resources/very-heavy-menstrual-flow
*The right answer is progesterone the medication called duphastone
Or because she wants to get pregnant
Clomiphene if there's:

31- same weight for 6 months with amenorrhea :


a-Eating disorder
answer?Not clear
32- mother after delivery sees snakes crawling into her baby bed :
Postpartum psychosis
Answer: A no other choices but the choice goes with Qs
33- Pregnant with placenta abruption what's suspected complication :
a-Fetal distress
b-Other
Answer:
For the baby
Born at low birthweight
Preterm delivery
Asphaxia fetal distress
Fetal death & stillbirth.
Complications:
fetal complications: perinatal mortality 25-60%, prematurity, intrauterine hypoxia
maternal complications: <1% maternal mortality, DIC (in 20% of abruptions), acute renal
failure, anemia, hemorrhagic shock, pituitary necrosis (Sheehan syndrome), amniotic uid embolus
(Tornto notes 2016 OB15)

smle ,2016

500
34- Patient with grey vaginal discharge, erythema of skin:
A. candida
B. BV
answer: B, Because it's grey and itchy + bad smell
Bacterial vaginosis is asymptomatic in up to 50% of women. If a discharge is present, it is typically thin, homogeneous, malodorous, and grayish white or yellowish white in color. Vaginal
pain or vulvar irritation is uncommon. Pruritus may occur.
35- Patient with severe painful vesicles on genital area:
a- syphilis
b- HSV
answer: B
Symptoms of genital herpes
Local symptoms include pain, itching, dysuria, vaginal and urethral discharge, and tender lymphadenopathy.Clinical features in women: Herpetic vesicles appear on the external genitalia,
labia majora, labia minora, vaginal vestibule, and introitus. In moist areas, the vesicles rupture,
leaving exquisitely tender ulcers.
36- Epithelial cell source in female sample:
A. chlamydia urethritis
B. vulva contamination
C. renal stones
answer: B
37- Female pt diagnosed with IBD on ceftriaxone with no benefit wt is the organism
Answer: Chlamydia
38- 2 years female C/O atrophic vaginitis , weakness of pelvic floor muscle , urine incontinence What is the most appropriate management for her?
a-Kegel exercise.
b-Surgical
Answer:Kegel Exercise
39- 24 year old mother presented with her child who is diagnosed with Down syndrome CLINICALLY, she's asking about the risk of down in her next child, what is the best investigation:
A. karyotype this child
B. karyotype the mother and child
C. do US next pregnancy
D. do amniocentesis next pregnancy
answer: C

smle ,2016

501
We have to screen first then diagnose. US is a screening test while amniocentesis is a diagnostic
test.

Algorithm for down syndrome screening (reference : aafp )


40- lady post-delivery had numbness and paresthesia in medial aspect of her thigh immediately after delivery then relieved after few days. What is the possible nerve that get compression by delivery ?
a-obturator
b-internal pudenal
Answer:obturator N
The main complaints in obturator nerve entrapment include difficulty with ambulation and the
development of an unstable leg. In an anterior branch entrapment, symptoms can consist of exercise-related pain or groin pain. The patient may describe a deep ache in the region of the adductor origin at the pubic bone that increases with exercise; the pain may radiate down the medial aspect of the thigh toward the knee. An athletes ability to jump may be reduced. The
weakness in these patients usually worsens with exercise.
41- Gynecologist see atypical invasive cell on colposcopy , Otherwise the patient is fine. what
is the initial management for her?
a-Clinical staging
b- conization
c- and..
smle ,2016

502
d- surgical
Answer: B ? not sure

42- Pregnant with hyperthyroidism.treatment ?


A. Levothyroxine
B. Propylthiouracil
Answer: B
Reference : uptodate

smle ,2016

503
43- pregnant lady 11 weeks GA , come to weird about infectious disease outbreak in the
school of her child that may affect her pregnancy . What is the appropriate vaccine at that
time?
a-rubella
b-varicella
c-influenza
Answer: c
Varicella & rubella vaccines are contraindicated during pregnancy.
http://www.cdc.gov/vaccines/pubs/preg-guide.html
44- Seven weeks pregnant lady c/o vaginal bleeding with tissue. Her cervix was open and you
can see some product of conception. Her fundal height is equal to 7 to 8 weeks.
A.Threatened abortion
B.Incomplete abortion
C.Missed abortion
D.Molar pregnancy
answer : B
45- A patient with blood only noticed when she wipes with toilet paper, where is the bleeding?
A. urethra
B. vulva
C. uterine body
D. uterine cervix
answer:D
http://www.womens-health-advice.com/cervical-cancer/symptoms.htmlhttp://www.womenshealth-advice.com/cervical-cancer/symptoms.html
46- When does a pregnant patient do GDM ?
A. 12 weeks
B. 16 weeks
C. 20 weeks
D. 28 weeks
Answer: D, 24-28 weeks for GDM screening
http://care.diabetesjournals.org/content/26/suppl_1/s103
47- Definition of postpartum hemorrhage:
A.
More than 500 ml post SVD
B.
Less than 500 ml post CS
C.
More than 500 ml post CS
D.
Less than 500 ml post SVD
answer: A

smle ,2016

504
48- Which part of the female urethra is more susceptible to be damaged (in an intervention I
can't remember what)
answer:?
49- A patient whos 36 weeks pregnant started having contractions lasting 30 secs. CTG was
done and was good. What to do?
A- Give tocolytics
answer:?
50. women had yellow watery foul smelling vaginal discharge
A. bacterial vaginosis
B. trichominus vaginalis
C. atrophic vaginitis
Answer: trichominus vaginalis. see table in Q115 Gyne section.
51.nullipara with diabetes gestational diet. Normal contraction. During labor she full
extension and one nurse push the baby from fundus and the other nurse push the
above symphysis pubis with no labor then the doctor did episiotomy. What response
for delayed labor?
A. Full extension
B. Pushing the fundus
C. Pushing symphysis pubic
D. Episiotomy
Answer: B
52.which drug is safe for pregnant women
A- cimetidine
B- cefoxizime
Answer: A
http://www.medscape.com/viewarticle/515100_5
http://www.medscape.com/viewarticle/515100_5
53.pregnant woman with chronic HTN and uncontrolled DM she is on week 10 of gestation
what is the best action?
A- bed rest
B- ACE inhibitor
C- termination of pregnancy
Answer: bed rest?
ACEI is contra indicated during pregnancy
54.contraindication to IU
A-Active PID
B-Coagulation abnormality
Answer: A
reference: Toronto
http://emedicine.medscape.com/article/1998022-overview
smle ,2016

505
http://emedicine.medscape.com/article/1998022-overview
55.pt in labour and has pre-eclampsia. MgSO4 and hydralazine were given. Respiratory rate
was 12. What do you give?
A- Narcan
B- Naloxone
C- Atropine
Answer:?
She has magnesium sulphate toxicity > respiratory depression so Calcium gluconate
Is the antidote for magnesium sulphate
56. 1ry dysmenorrhea, what is the first line of management:
A- NSAIDs
B- Acetaminophen
Answer: A
reference: uptodate
57.If a Pap smear shows HPV, the cytology will show what:
Answer: see the chart in OB/Gyne section

58. fibroid was found in a healthy asymptomatic 52 year old woman, it was 5x6cm, what will
you do?
A- Follow up every two months with ultrasound and CBC
B- Follow up regularly
C- Immediate myomectomy
D- Immediate hysterectomy
Answer: B
conservative ttt if: minimal or asymptomatic, <6-8 cm or stable in size, not submucosal, currently pregnant. toronto
we perform annual pelvic exams and, in patients with anemia or menorrhagia, check a complete blood count. Uptodate
59.vulvar carcinoma will present as a unifocal, ulcerative and lesion in :
A- Labia Majora
B- Clitoris
C- Mons pubis
D- Perineum
Answer: A
60. multipara pregnant. Medically free.. she is term.. effacement 90% ,4 cm,regular contractions, spontaneous rupture of membrane, suddenly baby is stress from 140 to 80 beats.which
type of anesthesia would you use:
A- General anesthesia
B- Narcotics
smle ,2016

506
C- Pudendal n
D- upper utrian
Answer:
61. 42 years old female complaining of amenorrhea, night sweat and flushing for the last 6
months. What is the most likely diagnosis?
A- Hypothyroid
B- Hypoprolactinemia
C- Congenital adrenal Hyperplasia
D- Pheochromocytoma
Answer: A?
62.Women after CS have persistent hypotension , What is best management
a.Normal saline
b.Iv dopamine
Answer: ?
63. wiff test positive what to do to confirm the diagnosis ( bacterial vaginosis)
A- serology
B- gram stain
C- fungal culture
D- PH
Answer B
Demonstration of clue cells on a saline smear is the most specific criterion for diagnosing BV.
Clue cells are vaginal epithelial cells that have bacteria adherent to their surfaces. Medcsape.
64. Female had vaginitis coming with fever , rash?
Toxic shock syndrome
65. Pregnant with Hypothyroidism.. treatment?
A) Levothyroxine
B)Propylthiouracil
Answer: A
66. 12 week pregnant w high blood pressure:
Answer: pre-existing hypertension
67.What changes will occur during pregnancy
A) Increase tidal volume
B) Increase total lung capacity
C) Increase residual capacity
Answer: a
respiratory changes during pregnancy: No Change in VC and FEV1. Decrease TLC, FRC and RF,
reference: toronto.

smle ,2016

507

9th update
Our link Qs up to 28th of December
1-pregnant works under sun, developed patches over the forehead and cheeks A.cholasma faciei (melisma)
Answer: A
Melasma is thought to be the stimulation of melanocytes (cells in the epidermal layer of skin
that produce a pigment called melanin) by the female sex hormones estrogen and progesterone to produce more melanin pigments when the skin is exposed to sun.
2- Causes of fetal distress:
Answer:
1- Cord prolapse
2-placenta abruption
3-anemia
4-oligo/polyhydramnios
5-DM/PET
6-IUGR
7-post term
3-Endometriosis typical case presentation and asking about the diagnosis
Answer:
Gold standard is laparoscopy
The definitive diagnosis is generally made by thecharacteristic gross and histologic findings obtainedat laparoscopy or laparotomy.
Reference: Hacker and Moores, page 302, 5th edition
4-30 yo female presenting with hirsutism, acne, and irregular menstruation. What is the most
probable finding?
Answer: High androgen
PCOS
(high androgen, insulin resistance, acanthosis nigricans)
To diagnose PCOS
1-US 2- lab (high: testosterone, androgen, insulin, Low:progesterone, increase ratio LH/FSH >
2:1)
Clinically, the most common signs of PCOS are hirsutism (90%), menstrual irregularity (90%),
and infertility (75%).
The increased LH level promotes androgen secretionfrom ovarian theca cells, leading to elevatedlevels of ovarian-derived androstenedione and testosterone.
Reference: Hacker and Moores, page 364, 5th edition

smle ,2016

508
5-Patient complaint progressive wrist pain since 2 months and increased in the past 1month ,
gave a history of cesarean delivery on exam there is numbness and normal range of motion.
What is the Rx?
a-Thumb cast
b-Whole arm cast
c-Surgical decompression
Answer:
Symptoms usually resolve over period of week after delivery
We recommend nocturnal wrist splint, surgical decompression rarely done
http://www.uptodate.com/contents/carpal-tunnel-syndrome-treatment-and-prognosis?source=outline_link&view=text&anchor=H956780404#H956780404
6-Mother after ROM came to ER the pt give history of herpes infection 2 years back OE doctor
see inactive H.simplex what will do:
A- CS
B-instrumental delivery
C-sterile speculum examination
D-give acyclovir
(The presence of active genital herpes is an important concern in the presence of ruptured
membranes, but in this case it is not)
Answer: C
7-Pregnant is in her 38 weeks gestation with a blood pressure of 140 over 90. No proteinuria
and
completely asymptomatic what will you do:
a-Immediate delivery
b-Antihypertensives
c-Observation (frequently)
Answer:C
This case consider as gestational hypertention( bp<150/90 and no proteinuria)
Mx: close BP monitoring
8-A pregnant lady in her first trimester did not have any vaccination for rubella what to do?
Answer:dont give the vaccine.
Remember thatMMR, varicella, and HPV vaccinesare contraindicated during pregnancy.
Reference: Hacker and Moores, page 10, 5th edition
So if the Pt is not immunized give MMR vaccine after delivery

9-Pregnant lady with history of 2 SVD and 1 CS, how will you manage?
Answer: A
A. Natural vaginal delivery trial
B. Admit at 38 weeks for CS

smle ,2016

509
A trial of labor may be offered if one or two previousLTCDs were performed, the uterine incision did notextend into the cervix or uterine upper segment, andthere is no history of prior
uterine rupture. Adequate
maternal pelvic dimensions should be noted by clinicalexamination
Reference: Hacker and Moores, page 226, 5th edition
10-Best treatment for premenstrual dysphoric syndrome:
A. OCP
B. Fluxitine
C. Bromocriptin
Answer : B
SSRI like ( fluxitine) first line treatment of PMDD
Fluoxetine taken at dosages of 20 to 60 mgper day during the luteal phase of the cycle provides
significant symptom improvement in 50% to 60% of patients.
Reference: Hacker and Moores, page 388, 5th edition
11-Non hormonal treatment for postmenopausal symptoms
A. Black cohosh
B. Paroxetine
C. Phytoestrogen
Answer: B
http://www.uptodate.com/contents/nonhormonal-treatments-for-menopausal-symptoms-beyond-the-basics?source=outline_link&view=text&anchor=H1#H1
Paroxetine can treat women with moderate to severe VMS associated with menopause
http://www.medscape.com/viewarticle/838691
http://www.medscape.com/viewarticle/838691
12-Pregnant in labor cervical opening 6 cm. Which stage?
Answer:active stage
First Stage: latent>> 3-4cm dilation
Active>> from 4cm to 10cm
13-Pregnant with vaginal bleeding she delivered a normal baby but she is bleeding from nose
, gum and IV line?
A-factor v liden
B-DIC
C-protine s,c def
Answer: B
14-HCV mom can she breastfeed?
A-continue
b-stop
Answer: A

smle ,2016

510
There is no documented evidence that breastfeeding spreads HCV. Therefore, having HCV-infection is not a contraindication to breastfeed. HCV is transmitted by infected blood, not by human breast milk. There are no current data to suggest that HCV is transmitted by human breast
milk.
Similar info in: Hacker and Moores, page 211, 5th edition
15-Prevent congenital heart disease in pregnancy ?
A-rubellavaccine
B-amniocentesis
Answer: A
To decrease risk of congenital heart disease:
1-Viral Infections Women who contract rubella (German measles) during the first three
months of pregnancy have an increased risk of having a baby with a heart defect,women should
avoid becoming pregnant for one month after receiving the MMR vaccine. If a woman is not
vaccinated, she should talk to her health care provider about any possible risks.
2-Take 400 micrograms of folic acid supplement a day during the first trimester (first 12 weeks)
of your pregnancy
3-Avoid drinking alcohol or taking drugs.
4-If you have diabetes, make sure it's controlled.
16-Mother came after one of ROM at home when to administer the antibiotic:
a-before prepare of operation
b-during CS
c-after one day
Answer: A
Ampicillin or erythromycin significantly prolongsthe interval to delivery in patients with PPROM.
Theneonates delivered from patients receiving prophylaxisalso have less morbidity.
Reference: Hacker and Moores, page 152, 5th edition
http://www.uptodate.com/contents/preterm-premature-prelabor-rupture-of-membranes?source=outline_link&view=text&anchor=H879788#H879788
http://www.uptodate.com/contents/preterm-premature-prelabor-rupture-of-membranes?source=outline_link&view=text&anchor=H879788 - H879788
17 -lady in 37 week came to ER with continuous uterine contraction, Cervix is dilated 7 cm,OE
baby is breach mood of delivery?
a-CS
b-assesst delivary by forceps
c-vacum
Answer: A
18-how to assess progress in labour:
a-severity of uterine contraction
b-descending of presenting part
c-molding of head
smle ,2016

511
d- duration of contraction
Answer :B
Vaginal examinations have become a routine intervention in labour as a means of assessing labour progress( affecment , dilatation,desnding of baby)
19:-girl with bilateral overian abscess with fever , what to do?
A. immediate laprotomy
B. laproscopic
C. trans US drain
D. antibiotic
Answer: ?
http://www.uptodate.com/contents/management-and-complications-of-tuboovarian-abscess?source=outline_link&view=text&anchor=H26297647#H26297647
http://www.uptodate.com/contents/management-and-complications-of-tuboovarian-abscess?source=outline_link&view=text&anchor=H26297647 - H26297647
20- adenomyosis treatment?
a-hystroectomy
b-ocp
c-gonadotropin analogue
Answer: a
21 -10 week pregnant with DM Nephropathy and HTN. BP is high (162/141) and +3 protein in
urine. What to do?
A-ACEI
B-Bed rest
C-Termination of pregnancy
B-observation
Answer ??:
http://www.uptodate.com/contents/pregnancy-in-women-with-diabetic-kidney-disease?source=outline_link&view=text&anchor=H11#H11
http://www.uptodate.com/contents/pregnancy-in-women-with-diabetic-kidney-disease?source=outline_link&view=text&anchor=H11 - H11
22-16 year old female no menstrual cycle yet, all other features are present. What is the diagnosis?
a-Mullerian agenesis.
b-Ovarian agenesis.
Answer :A
Mullerian agenesis: to development of mullerian duct ( no vagina, uterus, cervix) but present
with primary amenorrhea and secondary sexual chch
23-26 year old female with three months history of bilateral clear fluid coming out of her
breasts, it was obvious on physical examination. Normal menstrual cycle. What investigation
you would like to do?
smle ,2016

512
a-Neuroimaging.
b-Mammogram.
c-Prolactin assay.
d-ACTH.
Answer c :
24-The most accurate diagnostic investigation For ectopic pregnancy:
Culdocentesis
A-PelvicU/S
B-Endometrialbiopsy
C-Serial B-HCG
D-Laparoscopy
Answer :D-laproscopy
25- A 14 years female,with 6month history o f lower mid abdomina lpain ,the pain is colicky
radiate to the back and upper thigh, begin with onset of manse and last for 2-4 days, , physical examination of abdomen and pelvis normal, normal secondary sex development, what is
the most likely diagnosis?
A-Primary dysmenorrhea
B-Secondary dysmenorrhea.
Answer a
26- Female patient known to have Bicornuate uterus present in labor , give History of kicking
in lower abdomen and on Examination there is round object in fundus on auscultation the
heart positive in the umbilicus of his mother , what is the most likely presentation ?
A-Face
B-Vertex
C-Breach
Answer :c
27- Young female with whitish grey vaginal discharge KOH test? Smell fish like, what is thediagnosis?
A-Gonorrhea
B-Bacterial Vaginosis
C-TraichomanousVaginalis
Answer: B
28-Lady with menses every 15 days
A-Menometrorrhea
B-Polymenorhea
C-Hypermenorhea
Answer: B

smle ,2016

513
29- Common cause of pregnant lady mortality :
A-Syphilis
B-Toxoplasmosis
C-CMV
Answer: C
30-. Premenstrual syndrome:
A-More in the first half of menses
B-More in the 2nd half of menses
Answer b :
31-Pregnant with past history of depression on Paroxitine for long time. She is asking the physician if she can use this medication or not while she is pregnant. What you have to tell her ?
A. It is not safe because the risk of cardiac congenital malformation
B. It is not safe ...
C. It is safe ...
Answer a ,not sure
http://www.uptodate.com/contents/risks-of-antidepressants-during-pregnancy-selective-serotonin-reuptake-inhibitors-ssris?source=outline_link&view=text&anchor=H530686176#H530686176
32-: Pregnant lady, primgravida, medically free, BP reading today 158/89 and 2 weeks ago
160/96. What is the diagnosis?
a) Liable Hypertension
b) Gestational hypertension
Answer:
33- How to assess progress in labor?
A-Severity of contractions.
B-Descending of the presenting part.
C-Molding of the head.
(Something regarding the duration of the contractions).
Answer :b
34-Definition of leiomyoma?
answer :
Leiomyoma: A benign tumor of smooth muscle, the type of muscle that is found in the heart
and uterus. A leiomyoma of the uterus is commonly called a fibroid.
35-24 years old G1P0 , she has gestational diabetes which is controled by diet only, and no
other medical problems. She is in the 2nd stage of labor which last more than 2 hours, normal
uterine contractions, baby's head comes down with eatch contraction and go back when
uterus is relaxed, the mother's hip is maximally flexed, one nurse is apllying suprapubic pressure, other nurse applying fundus pressure, the doctor decided to do episiotomy and deliver
the posterior shoulder.

smle ,2016

514
Which of the following will cause delay in delivery?
A) suprapubec pressure
B ) fundus pressure
C) hip flexion
D) delivery of posterior shoulder.
answer:b
36-G8P7 in 36 weeks of gestation, with past medical history of postpartum hemorrhage in
each previous delivery that required blood transfusion.
What should be done regarding this delivery?
A) deliver patient in OR
B) perform CS at 38 weeks
C) give patient IV fluid before delivery to compensate for any hemorrhage that may happen
D) perform active management of 3rd stage golabor
answer:D
Third stage of labor: from the delivery of fetus till the delivery of placenta
Active management of the third stage: (1) Uterotonic medication administered within one minute after delivery of baby after ruling out presence of another fetus; (2) controlled umbilical
cord traction and counter traction to support the uterus until separation and delivery of the
placenta; (3) uterine massage after delivery of the placenta
The best preventive strategy is active management of the third stage of labor
Refrences:
http://www.aafp.org/afp/2007/0315/p875.html
37_What is the most common cause for postpartum hemorrhage?
A) uterine atony
B )multiparity
C)multiple gestation
D)macrosomia
answer:a
Reference : toronto notes
38-Pregnant pt came with high bp was given magnisum sulfate, which of the following is sign
of low maginsum in the body ? ( sizure wasn't included)
Answer :
Symptoms of magnesium deficiency include hyperexcitability, muscular symptoms (cramps,
tremor, fasciculations, spasms, tetany, weakness), fatigue, loss of appetite, apathy, confusion,
insomnia, irritability, poor memory, and reduced ability to learn. Moderate to severe magnesium deficiency can cause tingling or numbness, heart changes, rapid heartbeat, continued
muscle contractions, nausea, vomiting, personality changes, delirium, hallucinations, low calcium levels, low serum potassium levels, retention of sodium, low circulating levels of parathyroid hormone (PTH),and potentially death from heart failure.Magnesium plays an important
role in carbohydrate metabolism and its deficiency may worsen insulin resistance, a condition
that often precedes diabetes, or may be a consequence of insulin resistance.
smle ,2016

515

39- Long case


Pregnant lady -almost at labour I think-with breech presentation . Face flexed, the lichoer is
fair the baby found to be small and the pelvis of mother has ? Somthing I forgot but sure they
didn't mention the type of pelvis
Which of the following will prevent you from trying ECV ?
A)small baby
B)fair lichor
C)pelvic
D)flexed face of baby
Answer :
Absolute contraindication to external cephalic version :
1- if cesarean delivery is indicated for reasons other than breech presentation
2- Placenta previa or abruptio placentaehttp://emedicine.medscape.com/article/252810-overview
3- nonreassuring fetal status,
4- intrauterine growth restriction in association with abnormal umbilical artery Doppler index
5-isoimmunization
6-severe preeclampsiahttp://emedicine.medscape.com/article/1476919-overview
7-recent vaginal bleeding
8-significant fetal or uterine anomalies
9-ruptured membranes,
10- fetus with a hyperextended head,
11- multiple gestations
Relative contraindications:
Relative contraindications include maternal obesity, small for gestational age fetus (less than
10%), and oligohydramnios because they make successful ECV less likely.[24] Previous uterine
scar from cesarean delivery or myomectomy may also be a relative contraindication for ECV.
40-2 weeks infant came for routine check up the doctor exam the baby and he looks well , but
when the doctor ask the mother about her baby she told somthing else she said the baby is
not well he is confused and he has evil power or somthing like that,What does the mother
have?
Answer:Post partum psychosis
Postpartum psychosis has a dramatic onset, emerging as early as the first 48-72 hours after delivery. In most women, symptoms develop within the first 2 postpartum weeks
The mother may have delusional beliefs that relate to the infant (eg, the baby is defective or
dying, the infant is Satan or God), or she may have auditory hallucinations that instruct her to
harm herself or her infant.

smle ,2016

516
Postpartum depression develops most frequently in the first 4 months following delivery but
can occur anytime in the first year and it interferes with the mother's ability to care for herself
or her child
postpartum blues: Symptoms peak on the fourth or fifth day after delivery and last for several
days, but they are generally time-limited and spontaneously remit within the first 2 postpartum
weeks
refrence :http://reference.medscape.com/article/271662-overview#a6
41-which medication decrease effect of OCP?
A-anti epileptic
B-anticoagulant
Answer: a
http://www.ncbi.nlm.nih.gov/pubmed/11945109
42- baby born full term,enlarge labia the cause is :
A)estrogen
B)hcg .
c)proges.
Answer :?
43-when amniotic fluied less than 400 it is :
A- oligohydro
B- poly
answer:
At 12 weeks' gestation, the average volume is 60 ml.2 By 16 weeks, when genetic amniocentesis is often performed, the mean volume is 175 ml.2, 15 From 20 weeks on, there is greater
variance of amniotic fluid volume. Based on numerous studies using dye or para-aminohippurate dilution, radioactive isotopes, and actual collection of amniotic fluid at amniotomy, it has
been determined that amniotic fluid volume increases steadily throughout pregnancy to a maximum of 4001200 ml at 3438 weeks; however, wide variation does exist
44- women develop gestational diabets and doctor give her insulin after delivery she is at risk
to have :
A- Dm type 1
B -dm type 2
answer: b
The mother is at risk of developing type 2 diabetes in the next 20 years
reference : toronto notes
45 -pregnent lady in third trimester suddenly she developed LL swilling from hip to toes
best investigation is :
Answer:dopplex , because it is above the knee

smle ,2016

517

46--the most effective way to prevent cardiac anomaly in pregnancy is ?


A- smoking cessation
B-genetic screen
answer:?
47-which vaccination cant give to pregnant woman?
Answer: all live vaccines
48-14 ys girl menarche at age of 12 she suffering from sever dysmenorrhea with normal
amount what is the appropriate action :
A/ NSAID
B/ acetaminophen
C/ OCP
D/ progesterone
Answer:a
Most patients with primary dysmenorrhea show subjective improvement with NSAID treatment
primary dysmenorrhea: menstrual pain in the absence of organic disease , begins hours before
bleeding onset and persist hours or days (48-72 hours)
References:toronto notes
http://www.aafp.org/afp/1999/0801/p489.htmlhttp://www.aafp.org/afp/1999/0801/p489.ht
ml
49-lady in labour of breach presentation cervix fully dilated membrane i think rupture but no
preceding in labor for i think 2hs what will you do?
A/ continuo with vaginal delivery
B/ cs
Answer:B
50-pt G1P0 27weeks came sick with high blood pressure was admitted in ICU for observation.
The Dr. prescribed magnesium sulphate what is the indication of such drug
A/ prevent the seizer
B/ control of high BP
C/ some thing for renal management
Answer:A
risk of seizure is highest in the 1st 24 hours post partum -continue MgSO4 for 12-24 hours after
delivery
References: Toronto notes
51- pt in routine check up during pregnancy discover high Bp in 3 time 160/... You will start :
A/ methyl dopa
B/ atenolol
C/ labetalol ( not sure if it was within choices)

smle ,2016

518
Answer:a
prefered antihypertensive agents during pregnancy :
Labetalol , Nifedipine, a-methyldopa
ACEI , ARB, atenolol , prazosin are all contraindicated during pregnancy

52- case was obvious bacterial vaginosis ttt is?


A/ metronidazol
B/ flucanazol
C/ ceftraixaon
D/ ciprofloxacin
Answer:A
reference : toronto notes
53- pregnant lady fall from stairs presented to ER with severe abdominal pain and back pain
,abdomen was tender and distended ,there was vaginal discharge black like blood with fetal
distress?
A-abruptio placente
B-uterain rupture
Answer: abruptio placente
54- pregnant with chest infection ?
A. trimethoprim/sulphamethaxazol
B. augmentin
C. ceftraioxon
Answer:?
55- pt has history of gonoreheai ,came with complain of infertility hystroscopy done with result of normal uterus but dye cant be seen freely from tubes ( tubal block) what is the best
way for lady to conceive ?
A.in vitro fertilization and embryo transplant
B.induction of ovulation
C.clomophen
Answer:A
56- 25 ys old female came to ER with sever Rt lower abdominal pain she has history of aminorrhea for 2 month what is the diagnosis ?
A. rupture tubal pregnancy.
B. acute appendicitis
C. diverticulitis
Answer:A
57-Lady came with history of infertility, she has BMI of 30( and other features of PCO) which
of the following will help her to conceive?
smle ,2016

519
A) wt reduction
B) in utero fertilization
C) Clomophine
Answer: C
58-pregnant woman diagnosed with gestational diabetes what is the ttt should be started?
A) Insulin
B) Metformin
C) Sulphanylurea
Answer: A
59- Female pregnant lady with hyperthyroidism but not symptomatic what is the management?
A) MMI theantithyroid
B) PPT the antithyroid
C) b blocker
Answer:?
60- female patient shes 30 yers old she did pap smear yearly for 9 years ,, all normal, this
time pap smear showed low grade . What is your management?
A) remove the lision by electrical something
B) repeat after 1 year
C) colposcopy
Answer:?
61-65 years old female patient at night she wake up and want to urinate but she urinate before arrive to bath ?
A-urgency
B-overflow
C-or...or
answer:A
62-18 y.o girl presented to the clinic complained of amenorrhea for almost 2 consecutive
months. She denied the pelvic examination. What is your next step ?
A. TRH, TSH, T4, T3
B. Brain MRI
C. BHcg Urine Test
Answer :C, not sure
63- Best way to diagnose Adenomyosis ?
A. Histology section of hysterectomy
B. Endometrial Biopsy
C. Pelvic MRI
answer:A
smle ,2016

520

64-pregnant female in her 34 week , cervix is affect 80% and 1 cm dilated , fetal position is +
1 , what type of Anastasia will give ?
A.Pedundale nerve block
B.GA
C.Narcotic
D.Epidural
Answer :C
65- when a couple should seek help regarding infertility ?
A.12 months
answer: A
66-menopause lady came with vaginal spotting , on examination there was cystic nodule ,cervix examination showed tumor what to do?
A. cone biopsy
B. Directed biopsy
C. Pap smear answer :-?
67-primary amenorrhea , what is the next investigation?
http://www.uptodate.com/contents/evaluation-and-management-of-primary-amenorrhea
68-Q: pregnant lady with hypotension, what type of anesthesia you will give her?
A.pedundal.
B.epidural.
C.general.
Answer: C
69-: Breastfeeding mother known history of seizure on phenytoin, Ask about breastfeeding?
A.Reassurance.
B.Feeding after 8 hours.
Answer: 1
https://www.drugs.com/pregnancy/phenytoin.html
70/ Young lady came to clinic complained of not being pregnant for 2 years. She has dysmenorrhea. What is your diagnosis ?
A.Endometriosis
B.Endometritis
C.Leiomyoma
Answer: A
71-Diabetic female c/o itchy vaginal discharge ?
Answer: Candidiasis
For reading :
smle ,2016

521

72-36 yr old female use condom as contraceptive. she complain of nausea & amenorrhea.
what is?
first investigation to do is beta HCG
73- pregnant lady in her 8 week of gestation came and complain that she loss pregnancy sensation & there is vaginal spotting. what to do to establish the diagnosis ?
A.mother serum A P
B.trans vaginal US
C.serum b HCG
Answer:?
74- which of the following is contraindicated for assistant delivery by forceps
breach presentation?
A.face presentation
B.cephalopelvic disproportion
Answer:?
75- pregnant lady in her 41 wk of gestation admitted for delivery induction. after oxytocin
was given she start having contraction and there is 4 cm dilation & 60 % effacement. after
one hour there is 8cm dilatation of the cervix & 80% effacement. baby pulse is 120-140, also
there is acceleration & variability. what is the correct action to do?
A.expectant delivery
smle ,2016

522
B.stop oxytocin
C.go immediately for CS
Answer:?
76- which of the following drug safe during pregnancy?
A.erythromycin
B.cephalosporin
C.warfarin surgery
answer:?
77-HIV moter 34 week GA on antiviral medication , viral load is low what do you do ?
Continoue with antiviral medcatin and elective CS
78-pregnant 8w present missed symptom of pregnency and bleed no pain to approve your DX
you do?
A.Vaginal us
B.Bhcg
Answer: A we suspect abortion
79- pregency 36w present with liquid to support dx ?
A.chemical ..
B.spculum examin
C.vaginal examin
answer: A

80- pregnenat have flu symptom what to give ?


A.oslamavir
B.ozanamivr
C.actomab
answer :A
Oseltamivir is preferred for treatment of pregnant women. Pregnant women are recommended
to receive the same antiviral dosing as nonpregnant persons http://www.cdc.gov/flu/professionals/antivirals/antiviral-dosage.htm
It is best to start antiviral medications within the first 48 hours of developing symptoms, but antivirals can also be used after this time period. A 75-mg capsule of oseltamivir (Tamiflu) twice
per day for 5 days is the recommended first choice antiviral. https://medlineplus.gov/ency/article/007443.htm
https://www.nlm.nih.gov/medlineplus/ency/article/007443.htm
81- ob mass mobile not related to cyclic pain ?
A.fibroadeoma
B.fibrocystic change
C.intraducal papailoma
answer : A
smle ,2016

523

82- og clinic came with odor vaginal discarge ( only information) Dx ?


A.trichomanoais
B.gonnoreah
C.chlamydia
answer: A
Bloody or brown:Irregular menstrual cycles, or less often, cervical orendometrial cancer
Abnormal vaginal bleeding:pelvic pain
Cloudy or yellow: Gonorrhea
Bleeding between periods:urinary incontinence, pelvic pain
Frothy, yellow or greenish with a bad smell: Trichomoniasis
Thick, white, cheesy: Yeast infection
White, gray, or yellow with fishy odor:Bacterial vaginosis
83-pregnancy take iron come complain of weak and fatigue lab HB low and MCV low what
Dx?
A.iron deficency
B.hypothyroid
answer: may we have to think about DD of low MCV :
thalassemia , anemia of chronic illness , lead poising .
The normal physiological change of an increase in plasma volume causes haemodilution in a
pregnant woman. Although the red cell mass increases, plasma volume increases disproportionately, resulting in a lowering of the hemoglobin (Hb) to approximately 11.5 g/dL.
The National Institute for Health and Clinical Excellence (NICE) advises that women should be
offered screening for anemia at booking and at 28 weeks of gestation.
[2] Anemia is defined as an Hb level <11.0 g/dL at booking; haemodilution will result in further
drops during pregnancy and subsequent reduction in oxygen-carrying capacity. In the second
and third trimesters the diagnostic level for anaemia is an Hb level of <10.5 g/dL. Postpartum
the diagnostic level is 10.0 g/dL.
[3]Iron-deficiency anemia accounts for 85% of all cases of anemia that are identified and is
characterized by low mean cell volume (MCV). It is usually caused by nutritional deficiency or
low iron stores resulting from previous pregnancy or previous heavy menstrual blood loss.
Management
Routine iron replacement is not recommended in the UK.[3]
Women with known haemoglobinopathy should have serum ferritin checked and be offered
oral supplements if their ferritin level is low
Women with unknown haemoglobinopathy status with a normocytic or microcytic anaemia,
should start a trial of oral iron and haemoglobinopathy screening should be offered.
Non-anaemic women at increased risk of iron deficiency should have a serum ferritin checked
early in pregnancy and be offered oral supplements if ferritin is low.
Women with established iron deficiency anaemia should be given 100-200 mg elemental
iron daily. They should be advised on correct administration to optimise absorption. Supplementation should continue for at least three months and should aim to restore iron stores.

smle ,2016

524
Cochrane comments that although iron therapy restores indices to normal, data on outcomes are scarce and gastrointestinal side-effects are common.[5]
Referral to secondary care should be considered if there are significant symptoms and/or severe anaemia (Hb<70 g/dL) or late gestation (>34 weeks) or if there is failure to respond to a
trial of oral iron.
84- pap smear 2 sample was atypia:?
A.biopsy endometrrial
B.colposcopy
answer : B
85-pregnancy has HX of previous DVT what give ?
A.heparin
B.enoxaparin (anticoagulation)
answer:?
86- lactate female take phenytone ?
A.stop
B.continuo
C.take after 8 h
answer: B
EPILEPSY
Although anticonvulsants are excreted into breast milk, most mothers who require the use of
these drugs can safely breast-feed their infants.12,13 Determination of maternal serum drug
levels may be a useful adjunct to clinical monitoring of the infant when evaluating the drug exposure of the infant.
Phenytoin (Dilantin) and carbamazepine (Tegretol) are compatible with breast-feeding.6,8,10,12Although the AAP considers valproic acid and its derivatives (valproic sodium and
divalproex sodium) to be compatible with breast-feeding, some experts recommend against
their use during breast-feeding because of the potential for fatal hepatotoxicity in children
younger than two years.6,10,12
During breast-feeding, anticonvulsants other than phenobarbital and primidone (Mysoline) are
preferred because the slow rate of barbiturate metabolism by the infant may cause sedation.6,10,12 Infant serum levels may be helpful in monitoring toxicity.
http://www.aafp.org/afp/2001/0701/p119.html
87- mother pregnancy get pneuonua devple IGG what type of of imminty baby devope ?
A.active natural
B.artifical
C.passive natural
D.-----artifical
Answer: C
Passive immunity

smle ,2016

525
Passive immunity is the transfer of active immunity, in the form of readymade antibodies, from
one individual to another. Passive immunity can occur naturally, when maternal antibodies are
transferred to the fetus through the placenta, and can also be induced artificially, when high
levels of human (or horse) antibodies specific for a pathogen or toxinare transferred to non-immune individuals.
Passive immunization is used when there is a high risk of infection and insufficient time for the
body to develop its own immune response, or to reduce the symptoms of ongoing or immunosuppressive diseases.Passive immunity provides immediate protection, but the body does not
develop memory, therefore the patient is at risk of being infected by the same pathogen later.
Naturally acquired passive immunity.
Maternal passive immunity is a type of naturally acquired passive immunity, and refers to antibody-mediated immunity conveyed to a fetus by its mother during pregnancy. Maternal antibodies (MatAb) are passed through the placenta to the fetus by an FcRn receptor on placental
cells. This occurs around the third month of gestation. IgG is the only antibody isotype that can
pass through the placenta. Passive immunity is also provided through the transfer of IgA antibodies found in breast milk that are transferred to the gut of the infant, protecting against bacterial infections, until the newborn can synthesize its own antibodies.
Artificially acquired passive immunity
Artificially acquired passive immunity is a short-term immunization induced by the transfer of
antibodies, which can be administered in several forms; as human or animal blood plasma, as
pooled human immunoglobulin for intravenous (IVIG) or intramuscular (IG) use, and in the form
of monoclonal antibodies (MAb). Passive transfer is used prophylactically in the case of immunodeficiency diseases, such as hypogammaglobulinemia. It is also used in the treatment of several types of acute infection, and to treat poisoning. Immunity derived from passive immunization lasts for only a short period of time, and there is also a potential risk for hypersensitivity
reactions, and serum sickness, especially from gamma globulin of non-human origin.
The artificial induction of passive immunity has been used for over a century to treat infectious
disease, and prior to the advent of antibiotics, was often the only specific treatment for certain
infections. Immunoglobulin therapy continued to be a first line therapy in the treatment of severe respiratory diseases until the 1930s, even after sulfonamide lot antibiotics were introduced
Active immunity
The time course of an immune response. Due to the formation of immunological memory, reinfection at later time points leads to a rapid increase in antibody production and effector T cell
activity. These later infections can be mild or even unapparent.
When B cells and T cells are activated by a pathogen, memory B-cells and T- cells develop, and
the primary immune response results. Throughout the lifetime of an animal these memory cells
will remember each specific pathogen encountered, and are able to mount a strong secondary response, if the pathogen is detected again. The primary and secondary responses were
first described in 1921 by English immunologist Alexander Glenny although the mechanism involved was not discovered until later.This type of immunity is both active and adaptive because
the body's immune system prepares itself for future challenges. Active immunity often involves
both the cell-mediated and humoral aspects of immunity as well as input from the innate im-

smle ,2016

526
mune system. The innate system is present from birth and protects an individual from pathogens regardless of experiences, where as adaptive immunity arises only after an infection or immunization and hence is "acquired" during life.
Naturally acquired active immunity
For more details on this topic, see Immune system.
Naturally acquired active immunity occurs when a person is exposed to a live pathogen, and develops a primaryimmune response, which leads to immunological memory. This type of immunity is natural because it is not induced by deliberate exposure. Many disorders of immune system function can affect the formation of active immunity such as immunodeficiency (both acquired and congenital forms) and immunosuppression.
Artificially acquired active immunity
Main articles: artificial induction of immunity and vaccination
Artificially acquired active immunity can be induced by a vaccine, a substance that contains antigen. A vaccine stimulates a primary response against the antigen without causing symptoms
of the disease. The term vaccination was coined by Richard Dunning, a colleague of Edward Jenner, and adapted by Louis Pasteur for his pioneering work in vaccination. The method Pasteur
used entailed treating the infectious agents for those diseases so they lost the ability to cause
serious disease. Pasteur adopted the name vaccine as a generic term in honor of Jenner's discovery, which Pasteur's work built upon.
Poster from before the 1979 eradication of smallpox, promoting vaccination.
In 1807, the Bavarians became the first group to require that their military recruits be vaccinated against smallpox, as the spread of smallpox was linked to combat. Subsequently the
practice of vaccination would increase with the spread of war
There are four types of traditional vaccines:
A.Inactivated vaccines are composed of micro-organisms that have been killed with chemicals
and/or heat and are no longer infectious. Examples are vaccines against flu, cholera, plague,
and hepatitis A. Most vaccines of this type are likely to require booster shots.
B.Live, attenuated vaccines are composed of micro-organisms that have been cultivated under
conditions which disable their ability to induce disease. These responses are more durable and
do not generally require booster shots. Examples include yellow fever, measles,rubella, and
mumps.
C.Toxoids are inactivated toxic compounds from micro-organisms in cases where these (rather
than the micro-organism itself) cause illness, used prior to an encounter with the toxin of the
micro-organism. Examples of toxoid-based vaccines include tetanus and diphtheria.
D.Subunit vaccines are composed of small fragments of disease causing organisms. A characteristic example is the subunit vaccine againstHepatitis B virus.
Most vaccines are given by hypodermic or intramuscular injection as they are not absorbed reliably through the gut. Live attenuated polio and some typhoid and cholera vaccines are given
orally in order to produce immunity based in the bowel.
88- Breastfeeding mother treated with HCV by interferon more than one year what the risk of
breastfeeding on infant ?
A.Nipple crickeld
B.Mother with anemia
smle ,2016

527
C.Infant complain of oral candidacies
D.Not follow up of infant immunization
answer:?
89) what is the adenomyosis ?
Adenomyosis (ad-uh-no-my-O-sis) occurs when endometrial tissue, which normally lines the
uterus, exists within and grows into the muscular wall of the uterus
90- treatment of endometriosis ?
91)Triad of ascites pleural effusion and ovarian mass. What is the most likely tumor?
A.
Sex cord stromal tumors
B.
B - epithelial tumors
C.
germ cell tumors
Answer: A
Meigs syndrome is defined as the triad of benign ovarian tumor its benign sex cord-stromal tumour
92- 24 married for 9 month with regular heavy menses and pain , on examination
there is a nodule in cervix and tenderness, what is the cause?
A.
fibroid
B.
endometriosis
C.
cervical cancer
D.
vaginal cancer
Answer: A
http://www.fibroidsecondopinion.com/fibroid-symptoms/ http://www.fibroidsecondopinion.com/fibroid-symptoms/
93-pregnant women complaining of UTI at 12 week then treated, now complaining of dysuria,
She take () medication for 4 days ,, what u will do?
A.
give her small dose Abx till delivery
B.
change drug
C.
treat even asymptomatic
Answer: C
94-80 years women had yellow watery foul smelling vaginal discharge?
A.
bacterial vaginosis
B.
trichomonas vaginalis
C.
atrophic vaginitis
answer: B
95-Case of PROM at 32 weeks what to do ?
A.
Sterile speculum exam
B.
vaginal exam
C.
chemical investigation of liquor
smle ,2016

528
Answer: C if the presentation of this pt gush of fluid or leakage
96-what is papanicolaou smear ?
( Choices: how many sample and how many area ?)
answer:
http://www.uptodate.com/contents/screening-for- cervical-cancer?source=search_result&amp;search=pap+smear&amp;selectedTitle=1~139#H3213283
97- Minimal investigation for Monitor pre-eclampsia ?
to diagnose preeclampsia, you have to have high blood pressure and one or more of the following
complications after the 20th week of pregnancy:
Protein in your urine (proteinuria)
A low platelet count
Impaired liver function
Signs of kidney trouble other than protein in the urine
Fluid in the lungs (pulmonary edema) New-onset headaches Visual disturbances
Previously, preeclampsia was only diagnosed if a pregnant woman had high blood pressure and
protein in her urine. However, experts now know that it possible to have preeclampsia, yet
never have protein in the urine.
A blood pressure reading in excess of 140/90 mm Hg is abnormal in pregnancy. However, a single high blood pressure reading doesn mean you have preeclampsia. If you have one reading in
the abnormal range or a reading that&#39;s substantially higher than your usual blood pressure your doctor will closely observe your numbers. Having a second abnormal blood pressure reading four hours after the first may
confirm your doctor suspicion of preeclampsia. Your doctor may have you come in for additional blood pressure readings and blood and urine tests.
Tests that may be needed
If your doctor suspects preeclampsia, you may need certain tests, including:
Blood tests. These can determine how well your liver and kidneys are functioning and whether
your blood has a normal number of platelets the cells that help blood clot.
Urine analysis. A single urine sample that measures the ratio of protein to creatinine a
chemical That always present in the urine may be used to make the diagnosis. Urine samples taken over 24 hours can quantify how much protein is being lost in the urine, an indication
of the severity of preeclampsia.
Fetal ultrasound. Your doctor may also recommend close monitoring of your baby growth, typically through ultrasound. The images of your baby created during the ultrasound exam allow
your doctor to estimate fetal weight and the amount of fluid in the uterus (amniotic fluid).
Non stress test or biophysical profile. A nonstress test is a simple procedure that checks how
your Baby heart rate reacts when your baby moves. A biophysical profile combines an ultrasound with a nonstress test to provide more information about your baby breathing,
tone,movement and the volume of amniotic fluid in your uterus (((mayo clinic)))

smle ,2016

529
* female with severe pain during her period and heavy bleeding on examination nodules in
uterosacral ligament:
98- Endometriosis.?
Answer: A
Signs and symptoms of Endometriosis: Cyclic pelvic pain, abnormal heavy bleeding and nodular
uterus or adnexal masses.
Diagnosis: laparoscopy (dark brown clusters of lesions called Endometrioma Chocolate Cyst)
Treatment: NSAIDs, OCPs, Danazol androgen derivative, leuprolide acetate leupron both
are used to decrease FSH &amp; LH.
source: Master the boards: USMLE STEP 2 CK
99- female delivered her baby 4 months ago breastfeeding needs contraception and concerned about not having her period?
A.
Reassure and counsel about contraception
B.
Order prolactin level
Answer: B
Women who breastfeed have a delay in resumption of ovulation postpartum. This is believed to
be due to prolactin-induced inhibition of pulsatile gonadotropin-releasing hormone release
from the hypothalamus.
source: http://www.uptodate.com/contents/overview-of- postpartum-care
100-Pregnant, developed edema from inguinal to ankle what to give her?
A.
Heparin
B.
Warfarin
Answer: A
(Uptodate)
Pregnancy and the puerperium are well-established risk factors for deep vein thrombosis (DVT)
and pulmonary embolism (PE), which are collectively referred to as venous thromboembolic
disease (VTE).
Initial management of suspected VTE during pregnancy depends on the degree of clinical suspicion, whether anticoagulation is contraindicated, and whether PE, DVT, or both are suspected.
For pregnant women, we recommend adjusted dose subcutaneous low molecular weight heparin (SC LMWH), rather than adjusted dose intravenous unfractionated heparin (IV UFH) (Grade
1B) or vitamin K antagonists (Grade 1A). We recommend against the use of oral direct thrombin
inhibitors (eg, dabigatran) or anti-Xa inhibitors (eg, rivaroxaban, apixaban) in pregnant women
(Grade 1C).
We suggest that anticoagulant therapy continue at least six weeks postpartum (Grade 2C). We
suggest a total duration of anticoagulant therapy of at least three to six months for women
whose only risk factors for VTE were transient (eg, pregnancy) (Grade 2C). Patients with persistent risk factors for VTE may require longer therapy.
Thrombolytic therapy should be reserved for pregnant or postpartum patients with lifethreatening acute PE (ie, persistent and severe hypotension due to the PE))
smle ,2016

530

101)Multiparous with cervical dysplasia, has chlamydia and HSV 2, what is the cause of her
dysplasia ?
A- Chlamydia
B- HSV
C-HPV
Answer :C
(Uptodate)
Human papillomavirus (HPV) is the major etiologic agent of cervical precancer and cancer. The
association between HPV and cervical neoplasia is so strong that most other behavioral, sexual,
and socioeconomic covariables have been found to be dependent upon HPV infection and do
not hold up as independent risk factors.
HPV infection is necessary but not sufficient to develop cervical neoplasia. The two major factors associated with development of high-grade CIN and cervical cancer are the subtype of HPV
and persistent infection. Environmental factors (eg, cigarette smoking) and immunologic influences also appear to play a role.
Low-oncogenic-risk HPV subtypes, such as HPV 6 and 11, do not integrate into the host genome and only cause low-grade lesions (eg, low-grade SIL and CIN 1) and benign genital warts
High-oncogenic-risk HPV subtypes, such as 16 and 18, are strongly associated with high-grade
lesions, persistence, and progression to invasive cancer, but also cause low-grade lesions.
The primary approach to prevention of CIN and cervical cancer is HPV vaccination. Although
HPV is a sexually transmitted infection, condoms are only partially protective. For women with
CIN, appropriate monitoring and treatment are used as secondary prevention of cervical cancer.
102) AIDS patient 34 w pregnant her CD count dropped to 200 what will u do?
A.
Book for CS
B.
CS when spontaneous labor
C.
Vaginal delivery
Answer: I think A
http://www.uptodate.com/contents/hiv-and-pregnancy-beyond-the-basicshttp://www.uptodate.com/contents/hiv-and-pregnancy-beyond-the-basics
https://aidsinfo.nih.gov/contentfiles/hivandpregnancy_fs_en.pdfhttps://aidsinfo.nih.gov/contentfiles/hivandpregnancy_fs_en.pdf
103) Pregnant lady, fall from stairs, and started to have vaginal bleeding, Diagnosis?
A- Placenta abruption
104)Lactating women the doctor prescribed phenytoin for seizures regarding breast feeding
she should ?
A.
1 stop breast feeding
B.
2 feed after 8 hours
Answer 1

smle ,2016

531
Phenytoin may cause harm to an unborn baby, but having a seizure during pregnancy could
harm both mother and baby. Tell your doctor right away if you become pregnant while taking
this medicine.
If you become pregnant while taking phenytoin, your name may be listed on a pregnancy registry. This is to track the outcome of the pregnancy and to evaluate any effects of phenytoin on
the baby.
Phenytoin can make birth control pills less effective. Ask your doctor about using non hormonal
birth control (condom, diaphragm with spermicide) to prevent pregnancy while taking this
medicine.
Phenytoin can pass into breast milk and may harm a nursing baby. You should not breast-feed
while you are using this medicine.
http://www.drugs.com/phenytoin.html
105) A pt diagnosed with n. Gonorrhea what other infection you should look for ?
answer : chlamydia
(master the board)
106.Female had vaginitis coming with fever , rash ?
Toxic shock syndrome
answer: A
source: http://emedicine.medscape.com/article/169177-clinicalhttp://emedicine.medscape.com/article/169177-clinical
http://emedicine.medscape.com/article/169177-clinical
107) Pt diagnosed with ovarian germ cell theca , what other finding?
a.
chronic salpingitis
b.
endometrial hyperplasia
answer : B
hyperestrogenic findings including: hyperplastic endometrium and abnormal uterine bleeding,
breast tenderness, postmenopausal bleeding, menstrual abnormalities, and in children sexual
precocity
http://www.uptodate.com/contents/sex-cord-stromal-tumors-of-the-ovary-granulosa-stromalcell-tumors?source=machineLearning&search=granulosa+stromal+cell+tumor&selectedTitle=1~10&sectionRank=1&anchor=H10#H20
108)- pathophysiology of PCOS ?
A.
Increased insensitivity of androgen
B.
Increased androgen activity
Answer: I think B
http://www.medscape.com/viewarticle/438597_1http://www.medscape.com/viewarticle/438597_1
http://www.medscape.com/viewarticle/438597_1
109) Pregnant lady 24 weeks GA, thyroid function test as the following ?:
TBG High TSH Normal TOTAL T4 high Free T4 low
A)
Pregnancy
smle ,2016

532
B)
Oral contraceptives use
C)
Compensated euthyroid
D)
Hyperthyroidism
Answer: A
110)How to obtain a pap smear ?
A.
3 specimens from internal canal
B.
Two specimens from different sites
C.
One specimen from cervical os
Answer: i think its B
http://www.cytopathology.org/specimen-collection-adequacy-requisition/http://www.cytopathology.org/specimen-collection-adequacy-requisition/
http://www.cytopathology.org/specimen-collection-adequacy-requisition/
111) pregnant type DM1 class f w/ nephrotic complication and control HTN what is likely
complication?
A.
A.preeclampsia
B.
B.Stillbirth
C.
C.Shoulder dystocia
D.
D.large for GA
Answer: a ?
112) Lady comes to you at 20 days postpartum complaining of yellowish odorless vaginal discharge and the cervix is pink to red color, Her pregnancy was normal with no complication;
What you will do ?
A.
Reassurance
B.
Do culture
Answer: A
113)-The most accurate diagnostic investigation For ectopic pregnancy ?
A-Culdocentesis
c.
Pelvic U/S
d.
Endometrial biopsy
e.
Serial B-HCG
f.
Laparoscopy
Answer: E
Laparoscopy remains the criterion standard for diagnosis; however, its routine use on all patients suspected of ectopic pregnancy, However initial diagnosis of ectopic pregnancy is a clinical diagnosis made based upon serial serum human chorionic gonadotropin (hCG) testing and
transvaginal ultrasound (TVUS)
114)18 y/o married missed her period for two months, came with rt sided abd pain wts
thedx:
A.
ruptured ectopic
B.
. Similar question to previous Q but what's the test to order:
smle ,2016

533
C.

urine hcg

115)a woman who had spontaneous rupture of membranes came to the hospital stating that
the fluid that came out was clear. O/E her temp. 38.4 c and there's Pain score was 8 out of 10.
On palpation of uterus when not in contraction, there's tenderness. How to manage?
A.
give antipyretic
B.
give antibiotics while in labor
C.
don't do anesthesia
D.
do immediate CS
ANSWER B ?? NOT SURE
116)-vginal bleeding week 10 fundus 15cm closed os what is the diagnosis?
A.
_threatened abortion.
-117) Pregnant 10wks has bleeding and fetus delivered , os is opened and still some remnants
? What to do ?
With missed, incomplete, or inevitable abortion present before 13 weeks' gestation, the standard therapy has been suction D&C (medscape).
Women with an incomplete, inevitable, or missed abortion can be managed surgically, with
medication, or expectantly. All three management approaches are effective, but treatment is
completed more quickly with surgical management and involves fewer medical visits. The
choice of method is typically based upon patient preference
118)- pregnant lady in the week 41 of gestation, effacement is 50%, 2 cm dilated for the past
2 weeks, now the effacement is 60%, dilated 3 cm, the fetal condition is good based of CTG
findings, what is your next step in her management?
A.
give oxitocin and amniotomy.
B.
give . And amniotomy.
C.
give epidural then CS.
Answer from Toronto note Obstetrics , give oxitocin and amniotomy .conservative(not sure)
119)a mother with HCV , and she's breastfeeding her child?
A.
stop breastfeeding (WRONG)
B.
check HCV in breast milk
120)Endometriosis definition ?
The presence of tissue that normally grows inside the uterus (womb) in an abnormal anatomical location. Endometriosis is very common and may not produce symptoms, or it may lead to
painful menstruation. It has also been associated with infertility. Endometriosis occurs most
commonly within the Fallopian tubes and on the outside of the tubes and ovaries, the outer
surface of the uterus and intestines, and anywhere on the surface of the pelvic cavity. It can
also be found, less often, on the surface of the liver, in old surgery scars or, very rarely, in the
lung or brain.

smle ,2016

534
Endometriosis occurs in the reproductive years. The average age at diagnosis is 25-30.Endometriosis may be suspected by during a physical examination; it is confirmed by surgery, usually
laparoscopy; available treatments include medication for pain, hormone therapy, and surgery
http://www.medicinenet.com/script/main/art.asp?articlekey=3240http://www.medicinenet.com/script/main/art.asp?articlekey=3240
http://www.medicinenet.com/script/main/art.asp?articlekey=3240
http://www.medicinenet.com/script/main/art.asp?articlekey=3240
121) female with cervical proliferation, +ve herpes simplex virus, +ve
Chlamydia, which factor contribute to the risk?
A.
human papilloma virus.
B.
herpes simplex type2.
C.
Chlamydia infection.
Answer : A- human papilloma virus.
122)A 14 years female, with 6 month history of lower mid abdominal pain , the pain is colicky
radiate to the back and upper thigh, begin with onset of manse and last for 2-4 days, , physical examination of abdomen and pelvis normal, normal secondary sex development, what is
the most likely diagnosis?
a)
Primary dysmenorrhea
b)
Secondary dysmenorrhea.
answer: A
Primary dysmenorrhea refers to the presence of recurrent, crampy, lower abdominal pain that
occurs during menses in the absence of demonstrable disease that could account for these
symptoms.
Secondary dysmenorrhea has the same clinical features, but occurs in women with a disorder
that could account for their symptoms, such as endometriosis, adenomyosis, or uterine fibroids
.
123)-Women post intercourse bleeding ?
A.
Uterine cervix
B.
Uterine body
C.
Valve
D.
Vagina
ANSWER A
124) - postmenopausal lady taking tamoxofin, which of the following u will carefully assess?
A.
vaginal bleeding
Answer A
because it may increase the risk of uterine malignancy
Tamoxifen may increase the risk of the following, particularly in women over age 50 years:
Cancer of the uterus (endometrial cancer and sarcoma).
Blood clots within deep veins (deep vein thrombosis), usually in the legs, which can travel to the
lungs (pulmonary embolism).
Reference:

smle ,2016

535
http://www.uptodate.com/contents/tamoxifen-drug-information?source=outline_link&view=text&anchor=F224618#F224618http://www.uptodate.com/contents/tamoxifen-drug-information?source=outline_link&view=text&anchor=F224618 - F224618
http://www.uptodate.com/contents/tamoxifen-drug-information?source=outline_link&view=text&anchor=F224618 - F224618
125) snowstorm appearance in pregnant what's the Dx?
Answer: seen in complete hydatidiform mole
126)-Female patient known to have Bicornuate uterus present in labor , give History of kicking in lower abdomen and on Examination there is round object in fundus on auscultation the
heart positive in the umbilicus of his mother , what is the most likely presentation ?
a)
Face
b)
Vertex
c)
Breach
answer: C
127)Can use for pregnant women "?
A.
Paracetamol
B.
Aspirin
C.
ibuprofen
ANSWER A
128)Lady with menses every 15 days?
a)
Menometrorrhea
b)
Polymenorhea
c)
Hypermenorhea
answer: B
Polymenorrhea is the medical term for cycles with intervals of 21 days or fewer
Menometrorrhagia is a condition in which prolonged or excessive uterine bleeding occurs irregularly and more frequently than normal. It is thus a combination of metrorrhagia and menorrhagia.
Hypermenorrhea, also known as menorrhagia, is a disruption in the normal menstrual flow of
girls and women.
129)-a 46 year old woman G2P2 expressed that she want to get pregnant again, but she had
aaenorrhea since 7 months now. What will consider before you can tell her wither she can or
can't get pregnant?
A.
LH and FSH Level
B.
estrogen level
C.
prolactin level
ANSWER A
-130) Pregnant lady at 32 wks comes with regular uterine contraction, Fetal head at -2 What
is your action?
smle ,2016

536
A.
bed rest
B.
Give her steroid
(I think it is bed rest) *ahmedaraffa
-131)Picture looks like his with long history , I think it is vesicoureteral reflux ?
but im not .sure , read about it
132)pregnant in 37 week , BP 140/90 no proteinuria
- pregnant g3p2 in labor , cervical dilatation 3cm ,, effacement 100%
membrane rupture , after 3 hr still 3 cm ?
A.
c/s
B.
oxytocin
C.
waiting
133) pregnant women , if not allergic , by which abx you treat UTI ?
A.
A - ampicillin
B.
B - Sulfametha
C.
C - Nitrofurantoin
D.
D - I think ciprofloxacin
Answer: C
134)- Best medication for gestational diabetes mellitus is ?
A.
insulin
135) pregnant female present with bleeding and abdominal contractions started at night has
hx of mild hypertension what the dx?
B.
placental abruption
(why not preeclamsia)?? **ahmed araffa
-136)pregnant women with no prenatal hx present with regular uterinecontractions every 5
min , cervical dilation 10 cm on examination baby i
breech and neck is extended what to do?
A - CS
B - Vaginal delivery
C - partial Breech extraction.
answer: C
REFERENCE: Hacker &Moores Essentials of Obstetrics and Gynecology 5th
(pt in labor , fully dilated cervix with breach position .( I think breachextraction).**ahmedaraffa
-137)pregnant at 10 weeks GA, came RLQ pain (no other symptoms), vitals was normal except for tachycardia, CBC was normal, what is the Dx:?
A.
**- rupturedappendicit
smle ,2016

537
B.
**- ruptured tubal pregnancy
C.
-pertusis case
\ couldnt find answer
-138)increase risk of dysmenorrhea?
A.
1-copper releasing hormone
B.
2-levonorgestrel releasing hormone
C.
3-magnesium
D.
4-nifedipine
answer: http://www.webmd.com/sex/birth-control/intrauterine-device-iud-for-birth-control
1.

5 to 6 cases about vaginal discharge and treatment

2.

2 case PCO

3.

1 case ectopic

4.

1 abortion

5.

1 androgen insensitivity

6.

2 to 3 cases about milestone

139) what prevent fracture in post menopause ?


A-daily vit D supplements
B-weight bering exercise
decrease obesity
answer: B
http://www.uptodate.com/contents/osteoporosis-prevention-and-treatment-beyond-the-basics
Read about ???
- type of pregnancy ?
-- Read sexual infections !! May 6 Q
-- endometriosis .. How diagnosis ?Laproscopy or biopsy !!!

smle ,2016

538
REFERENCE: Toronto Notes 2016 PDF (Essential Med Notes 2016).pdf
-- pregnant lady developed edema and hypertension :
MGSO4
-- pregnant with DVT previously , what to do :
1.
Aspirin
2.
Enoxapirin++
3.
Heparin
No anticoagulant
-- mother after delivery sees snakes crawling into her baby bed :
**Postpartum psychosis
-- same weight for 6 months , amenorrhea :
**Eating disorder ?
-- typical of polycystic ovarian syndrome diagnosis .

-- mother close to delivery developed respiratory symptoms + fetal distress :


**Amniotic fluid embolism
-- Fibroid during pregnancy does it Degenerate or stays asymptomatic Or
what ?
**Red Degeneration
-- DM pregnant with Hx of fetal death before delivery now she is 32 weekspregnant with a
new baby what to do?
1.
Deliver her immediately
2.
Wait until 36 weeks
3.
Weekly biophysical profile or fetal heart rate testing can be combined withMaternal kick
counts in the third trimester.
4.
For patients who haveexperienced earlier loss, frequent ultrasound is reassuring
-- DM pregnant her oral glucose tolerance test came positive what to do next
A/Repeat OGTT
smle ,2016

539
B/HbA1C
Answer: B
Toronto Notes 2016 PDF (Essential Med Notes 2016)
-- Rupture of membrane during pregnancy when to give antibiotics?
A- pre op
B- before incision

The American College of Obstetricians and Gynecologists (ACOG)


recommends a seven-day course of intravenous ampicillin and
erythromycinfollowed by oral amoxicillin and erythromycin if watchful
waiting is attempted before 34 weeks.[2] Amoxicillin-clavulanic acid
increases the risk of fetal bowel death (necrotizing enterocolitis) and should
be avoided in pregnancy
-- female with Tubo ovarian abscess what is the treatment ?
Treatment is different if the TOA is discovered before it ruptures and can betreated with IV antibiotics. During this treatment, IV antibiotics are usuallyreplaced with oral antibiotics on an

outpatient basis
-Vulvar cancer cause and treatment ?
Surgery is a mainstay of therapydepending on anatomical staging and is usually reserved for .cancers thathave not spread beyond the vulva -Radiation therapy may be used in moread-

smle ,2016

540
vanced vulvar cancer cases when disease has spread to the lymph nodesand/or pelvis Chemotherapy is not usually used as primary treatment butmay be used in advanced cases with spread to the bones, liver or lungs. Itmay also be given at a lower dose together with radiation .
W
h
e
n
t
h
e
l
e
s
i
o
n
i
n
v
o
l
v
e
s
t
h
e
v
a
g
i
n
a
l
v
a
u
l
t
,
s

Elevated in menopause lady?


A.
Progesterone
B.
LH
Answer: B
Toronto Notes 2016 PDF (Essential Med Notes 2016).pdf
- Mass out of vagina with coughing and defecation?
Ureterovaginal prolapse or rectovaginal prolapse
Reference: Hacker &Moores Essentials of Obstetrics and Gynecology 5th.pdf
146) What is the mean age of menopause in normal women ?
A.
48.4
B.
51.4
C.
53.4
D.
55.4
Answer : B- 51.4
(Ref. dr. Tameem lecture slides)
Toronto Notes 2016 PDF (Essential Med Notes 2016).pdf

147) What is the chemoprophylaxis for Vibrio Cholera in pandemic area ?


A- Penicillin
B- Cefalexin
C- Tetracyclin
D- Erythromycin
Answer :c
http://emedicine.medscape.com/article/962643-treatment#d1
148) 62 years old came with vaginal bleeding. What is the most common
Benign cause of bleeding in this age ?
A- Cervical erosion
B- Cervical polyps
C- Atrophic vaginitis
D- Endometrial Hyperplasia
Answer : C
Toronto Notes 2016 PDF (Essential Med Notes 2016).pdf
149) pregnant patient with anemia, MCV high , what will you give her ?

smle ,2016

541
A.
Iron
B.
Folate
C.
something
D.
something
Answer : B- Folate
Toronto Notes 2016 PDF (Essential Med Notes 2016).pdf
150) Primigravida, Diet control GDM, on prolonged second stage of labor. She did full flexion
of her hip. the head of the baby descent during contraction
and going up during relaxation, One nurse applied pressure on fundus, while another nurse
applied pressure on the supra pubic area. What is the cause of her delayed labor ?
A.
Full flexion of the hip.
B.
Apply pressure on the fundus
C.
Apply pressure on the suprapubic area
D.
something irrelevant
COULDNT FIND ANSWER IN THE RESOURCES
151)Grey Virginal discharge what can u see in microscpy ?
( there is no clue cell or hyphee in the anwer )
a.
Intre epithelium
b.
Flagellated protozoan
Answer: b >Trichomonas vaginalis

Toronto Notes 2016 PDF (Essential Med Notes 2016).pdf


152)Pregnant present with bloody discharge in 10 week and fundus hight is 16 what is cos ?
a. Ruptue of cyssti
b. Ectopic pregnancy
????
153)Most common affect symptom in premenstrual dysphoric disorder?
1.
*Irritability
2.
**Mood swings
3.
**Depression
4.
**Anxiety
ALL CORRECT
Toronto Notes 2016 PDF (Essential Med Notes 2016).pdf
154) HELLP syndrome is?
A. Hypertension, elevated liver enzyme, low platelet
B. Hemolysis, elevated liver enzyme, low platelet
Answer is B
Toronto Notes 2016 PDF (Essential Med Notes 2016).pdf
155)Premenstrual syndrome ?
smle ,2016

542
a)
More in the first half of menses
b)
More in the 2nd half of menses
answer: B
www.nlm.nih.gov Home Medical Encyclopedia
156)40 year-old female, completed her family. She has endometrioma. Presented complaining of mild dysmenorrhea and severe pain during intercourse. What is the most appropriate
management?
A.
Removal of the cyst and ablation of the endometriosis lesions
B.
TAH + BSO
answer : B
Toronto Notes 2016 PDF (Essential Med Notes 2016).pdf
157)G3P2+0. Her first visit was on the 20th week of gestation. She has history of two premature deliveries. Her cervical length was 30 mm. what is your appropriate management?
A.
Strict bed rest
B.
Terminate her pregnancy
C.
Immediate cerclage
D.
Inject her with progesterone
answer: D or C
Several studies have indicated that the likelihood of preterm delivery increases with decreasing
cervical length. A cervical length of 2530 mm before 32 weeks gestation seems to increase the
risk of preterm delivery. If examination and ultrasound show that you have an abnormally short
cervix, and youre less than 24 weeks pregnant, your practitioner may recommend cerclage, a
procedure in which she stitches a band of strong thread around your cervix to reinforce it and
help hold it closed. However, theres a lot of controversy about whether cerclage should be
used in this situation."
https://healthymother.wordpress.com/2008/07/16/what-is-cervical-shortening/
158)Pt with odorless vaginal disch, grey-white. Spores on wet mount.?
A.
-Candida
B.
-Othet opts
Answer: candida
Toronto Notes 2016 PDF (Essential Med Notes 2016).pdf
.159) Pt with foul vaginal discharge, greenish color.Microscopy flagellate organisms. What's
the treatment: (trach vaginitis)?
A.
-Oral metronidazole
Toronto Notes 2016 PDF (Essential Med Notes 2016).pdf
. 160)Pregnant lady 39 weeks presented with high blood pressure for the first time.No proteinuria or seizures, wts her dx:?
-Gestational hypertension
Hacker &Moores Essentials of Obstetrics and Gynecology 5th.pdf
smle ,2016

543
161). When to swap for GBS in pregnant ladies:?
B.
-25 wks
C.
-30 wks
D.
-35 wks
E.
-40 wks
Answer: 35 wks
CDC has recommended routine screening for vaginal strep B for all pregnant women. This
screening is performed between the 35th and 37th week of pregnancy
Toronto Notes 2016 PDF (Essential Med Notes 2016).pdf
162) 62 years old came with vaginal bleeding. What is the most common Benign cause of
bleeding in this age ?
A.
Cervical erosion
B.
Cervical polyps
C.
Atrophic vaginitis
D.
Endometrial Hyperplasia
Answer: C
Toronto Notes 2016 PDF (Essential Med Notes 2016).pdf
163) pregnant patient with anemia, MCV high , what will you give her ?
A.
Iron
B.
Folate
Answer: C
Toronto Notes 2016 PDF (Essential Med Notes 2016).pdf
164) Primi gravida, Diet control GDM, on prolonged second stage of labor. she did full flexion
of her hip. the head of the baby descent during contraction and going up during relaxation,
One nurse applied pressure on fundus, while another nurse applied pressure on the supra pubic area. What is the cause of her delayed labor ?
A.
Full flexion of the hip.
B.
Apply pressure on the fundus
C.
Apply pressure on the suprapubic area
D.
something irrelevant
165) Case about infertility , what's the initial evaluation?
A.
-Temperature chart
B.
-Semen analysis
C.
-Refer to reproductive clinic
Answer: B
166)Scenario,,, female c/o amenorrhea , with normal breast development & normal pubic
hair . O/E no uterus & cervix .Dx?!
A.
-Mulleirn duct
B.
-Gonadal dysgenesis
smle ,2016

544

Mllerian agenesis (including absence of the uterus, cervix and/or vagina) is the etiology in
15% of cases of primary amenorrhoa.
gonadal dysgenesis has no 2' sexual characters.
167)Scenario,,, about female have bright red spots what's the source of this blood ?!
A.
-Uterine
B.
-Cervix
C.
-Vulva
Uptodate:
heavy bleeding: uterus
staining, spotting, light bleeding: genital tract
brown: uterus, cervix, upper vagina
red: genital tract
postcoital: cervical
168)Pt e hirsutism , obese x-ray shows cystic ovary , she wants to conceive , best Tx?!
- Clomiphene citrate
169).25 y/o c/o lower abd. Cramp , 6 wks from the last normal period . She has vaginal bleeding but no passage of tissue. Dx?!
- Ectopic pregnancy
complete abortion: bleeding + complete passage of tissue
incomplete abortion: extremely heavy bleeding, cramps, passage of tissue noticed
missed: no bleeding
threatened: vaginal bleeding +- cramps
inevitable: Increasing bleeding and cramps rupture of membranes
ectopic pregnancy: 4Ts and 1S Temperature >100.4oF (20%) Tenderness: abdominal (90%)
rebound (45%) Tenderness on bimanual examination, cervical motion tenderness Tissue: palpable adnexal mass (50%) (half have contralateral mass due to lutein cyst) Signs of pregnancy (e.g.
Chadwicks sign, Hegars sign)
cramping suggests: abortion
170).Female e osteoprosis , best Tx?!
a.
Vitamin D , Ca++ & bisphosphonates
Toronto: osteoporosis
1,000-1,500 mg calcium OD, 800-1,000 IU vitamin D, weight-bearing exercise, smoking cessation
bisphosphonates (e.g. alendronate)
selective estrogen receptor modifiers (SERMs): raloxifene (Evista) mimics estrogen effects
on bone, avoids estrogen-like action on breast and uterine cancer; does not help hot flashes
HRT: second-line treatment (unless for vasomotor instability as well)

smle ,2016

545
171)Female c/o malaise rash all over the body sparing the face:?
a.
syphillis
secondary syphilis (can resolve spontaneously) 2-6 mo after initial infection nonspecific
symptoms: malaise, anorexia, headache, diffuse lymphadenopathy generalized maculopapular
rash: palms, soles, trunk, limbs condylomata lata: anogenital, broad-based fleshy gray lesions
serological tests usually positive except the face, this sx after unprotected sex. Dx?!
20.Clear Hx about OCD and ask about dx
172)Female e jaundice . Her husband is HBV +ve , they did for her HBS antigen -v- e HAV -ve
HCV -ve . What to do next?!!
a.
HBC antigen
173).Female e DM well controlled , she wants to get pregnant, to avoid th-e complication dm
control- should be ?
A.
Started before pregnancy
B.
1st trimester
C.
2nd trimester
Answer: A
174).Pregnant lady in the 8 wks gestation came e hx of bleeding for the last 12 hrs + abd pain
, she passed tissue. O/E os is opened uterus is 7-8 wks in size, Dx?!
A.
-Incomplete abortion
B.
-Threatened abortion
C.
-Molar pregnancy
Answer: A
.175)Female present e oligomeorrhea " she had 3 periods in the last year" she had acne stirusim Body wt 60 , pv normal,dx?!
A.
PCOS
176)Scenario about PCOS - they asked what invx u will screen for?!
a.
Glucose tolerance+ lipid profile
177).Pt e hx of amenorrhea for 6 wks presented e abd pain . O/E there's fluid in douglas of
pouch , Dx?!
B.
Ectopic pregnancy
178)Pregnant women is Rh +ve and her baby is R-h -ve .what -will happen to -the mother?!
C.
No reaction
D.
Mild hemolysis
E.
Hydrops fetalis
F.
It's due to RH - mother and RH + baby..

smle ,2016

546
Rh disease is also called erythroblastosis fetalis during pregnancy. In the newborn, the resulting condition is called hemolytic disease of the newborn (HDN).
Some of the more common complications of Rh disease for the fetus and newborn baby include
the following:
Anemia (in some cases, the anemia is severe with enlargement of the liver and spleen)
Jaundice--yellowing of the skin, eyes, and mucous membranes.
Severe anemia with enlargement of the liver and spleen
Hydrops fetalis--this occurs as the fetal organs are unable to handle the anemia. The heart begins to fail and large amounts of fluid build up in the fetal tissues and organs. A fetus with hydrops fetalis is at great risk of being stillborn.
.
179)Scenario about 1ry dy-smenorr-hea .what's the 1st linesigns and symptomsf ttt?!
A.
NSAIDs
B.
OCPs
180)Trauma p-t with sx of sho-ck, what u will do 1st?!
A.
Ringer lactate
B.
Blood transfusion
Answer: A
.181)Breech presentation at 34 wks,,?!
A.
ECV at 36 wks
182).Female after delivery started to develop pelvic pain , fever, & vaginal discharge -,There's
test -ment-ioned in the Q . Dx?!
B.
PID
C.
bacterial vaginosis
D.
vaginal yeast
>20% clue cells = squamous epithelial cells dottedwithcoccobacilli(Gardnerella) Paucity of
WBC PaucityofLactobacilli Positivewhifftest:fishyodorwithadditionof KOH to slide (due to
formation of amines)
183) premenstraul 40 years c/o heavy bleeding & intercyclic bleeding , not pregnant not using
oc- . & not sexual active from a year, dx?!
A.
-Anovulatory cycle
a.
endometrial biopsy
endometrial biopsy: consider biopsy in women >40 yr must do endometrial biopsy in all
women presenting with postmenopausal bleeding to exclude endometial cancer
184)Female present e defemenization " breast atrophy & deepi- ng of voice" they found to
have ovarian cancer , dx?!
B.
Sertoli leyding cell
smle ,2016

547

.185)43 y/ e irregular menses 3 months back & 1-2 days spotting , what next to do next ?
186)female abdominal pain examination tender nodular retroverted uterus what investigation?
A.
laparoscopy
B.
hysteroscopy
C.
hysterosalpengiogram
answer : A
187)25 y o woman came to the clinic with her 6 weeks old baby, complaining of irritability ,
weight loss, and inability to sleep?
A.
post Partum thyroditis *
B.
hyperthyroidism
C.
hashimoto thyroditis
ANSWER A
188)bacterial vaginosis swap what to do on it?
A.
gram stain
-189)women with mastitis ?
A- stop breast feeding
B- clean nipple with alcohol
C- surgical drainage
ANSWER MAY BE MISSED OPTION CONTINUE BREAST FEEDING
190)How can stimulate breast feeding secretion ?
A- Breast feeding
B- Increase fluid intake
C- Increase caloric intake
ANSWER A
191)Dew drops on rose petals vaginal lesions, dx:?
A- herpes simplex
B- syphillis
C_ chanchroid lesion
D- herpangia
Answer: A
Herpes Simplex viruses generally cause mucocutaneous infection, that is, cells of the skin and
the mucous membranes are infected. This can manifest as cold sores on the lips, or as genital
sores. The typical rash has been described as "dew drops on a rose petal", it consists of vesicles
(blisters) that are initially clear and then crust over, typically with yellowish exudate. These vesicles are generally painful, and further, the area of skin and/or mucosa and the subcutaneous
tissues in the region where the rash will appear commonly becomes sensitive and even swollen
smle ,2016

548
before eruption of the vesicles. Tissue swelling may increase as the rash blossoms, and then,
generally over a course of a week to 2 weeks, resolves completely - leaving no scarring.
http://en.citizendium.org/wiki/Herpes
192)What is most stimulus factor for milk secretion ?
Answer: Suckling ?
193)-pregnant female with UTI which drug if safest during all trimesters:?
A-ampicillin
B-nitrofurantoin Other choices contraindicated
ANSWER A
194)Famale can not pregnant and try for 3 mount and she normal regular menstral cycle and
hisbund normal what to do ? It case of infertility)
1- try more
2- semen analysis
3 - genetic study
195) Lady with lower abdominal pain, Vaginal examination fornices tenderness , suprapubic
tenderness, purulent vaginal discharge?
A- Acute cervitis
B- Acute salpingitis
C- Acute appendicitis
Answer : B
Pelvic inflammatory disease (PID) refers to acute and subclinical infection of the upper genital
tract in women, involving any or all of the uterus, fallopian tubes, and ovaries
sx : Lower abdominal pain is the cardinal presenting symptom in women with PID. The abdominal pain is usually bilateral and rarely of more than two weeks' duration
On physical examination, most women with PID have abdominal tenderness on palpation,
greatest in the lower quadrants, which may or may not be symmetrical. Rebound tenderness,
fever, and decreased bowel sounds are usually limited to women with more severe PID.
Acute cervical motion, uterine, and adnexal tenderness on bimanual pelvic examination are the
defining characteristic of acute symptomatic PID [7,8]. Purulent endocervical discharge and/or
vaginal discharge is also common. uptodate
196)32 GA , uterine contraction , high PB 160/110, epigastric pain , Next:
A- Urine dipstick analysis
B- Give Tocolytic
answer: A
197) Adenomyosis definitive diagnosis?
A- Endometrial sampling.
B- Hysterectomy with ...
Answer: B

smle ,2016

549
A definitive diagnosis of adenomyosis can only be made from histological examination of a hysterectomy specimen. Uptodate
198) During delivery the cord is before the fetal head, management?
A- C/S.
B- Vacuum.
C- Forceps.
Answer : A
The gold Standard obstetrical management of cord prolapse in the setting of a viable pregnancy
typically involves immediate delivery by the quickest and safest route possible This usually requires cesarean section, especially if the woman is in early labor and to avoid fetal compromise
or death from compression of the cord. However, vaginal delivery may be a reasonable option
in select cases when delivery is imminent.
Refrrence : Uptodate
199)- During delivery something happened C/S was required, what type of anesthesia?
A- Pudendal.
B- General.
answer: epidural and spinal anesthesia, B/c in General anesthesia the drugs are given to the
mother will affect the infant.
Ref.: http://www.uptodate.com/contents/c-section-cesarean-delivery-beyond-the-basicshttp://www.uptodate.com/contents/c-section-cesarean-delivery-beyond-the-basics

https://www.arupconsult.com/Algorithms/Anemia.pdfhttps://www.arupconsult.com/Algorithms/Anemia.pdf

200)- History of PID, presented with secondary infertility, what to do?


A- Colposcopy
Answer : Laparoscopy
201)- youre a gynecologist in clinic, a lady come to you with profuse vaginal discharge, diagnosis?
202)- Pregnant lady with nausea and vomiting and abdominal pain, whats your first priority
in management?
A- IV fluids
B- Pain management
C- IV Antibiotics
Answer : A
-203) Order to know what therapy you are going to give? Ki67 or her2
B.
Ki67 = Anastrozole
smle ,2016

550
C.

Her2 = Trastuzumab or Tamoxifen

204)- Old lady did hysterectomy and bilateral oophorectomy histology showed Ovarian gem
cell theca something What other findings ?
A- chronic salpingitis
B- endometrial hyperplasia
C- uterine navus
F- cervical something
Answer: B
Ref: http://emedicine.medscape.com/article/254489-overview#a6http://emedicine.medscape.com/article/254489-overview - a6
http://emedicine.medscape.com/article/254489-overview - a6
http://emedicine.medscape.com/article/254489-overview - a6
205) A patient whos 36 weeks pregnant started having contractions lasting 30 secs. CTG was
done and was good. What to do?
A.
Give tocolytics
206) Lady ,12 month trying to conceive, regular menstruation , her husband > normal semen
analysis & temperature is normal What is the cause?
206)- Polycystic ovaries syndrome fertility what is the cause?
A- Endometrial.
B- Ovarian.
Answer; B
207)- Pregnant on iron therapy has fatigue and SOB , Hb is low , MCV is low ,retics=10%, What
is the dx?
A) IDA
B) Thalassemia
Answer: A, due to iron treatment, reactional elevation in retics count
208)- 40 y/o Pt was normal cycle, now heavy and bleeds intermittently, wt to do to dx:?
A- pap
B- colposcopy
Answer : A
AUB investigations ; in addition to lab tests, you have to screen for cervical cancer,& possibility
of cervicitis by gonorrhea or chlamydia. For pt younger than 45 years old with unopposed prolonged estrogen exposure ( e.g. Obesity ) or with persistent AUB despite medical management ,
u have to take an endometrial biopsy.
Frequent, heavy or prolonged AUB in women > 45 years necessitate endometrial biopsy.
http://www.uptodate.com/contents/approach-to-abnormal-uterine-bleeding-in-nonpregnantreproductive-age-women?source=outline_link&view=text&an-

smle ,2016

551
chor=H16090927#H16090927http://www.uptodate.com/contents/approach-to-abnormal-uterine-bleeding-in-nonpregnant-reproductive-age-women?source=outline_link&view=text&anchor=H16090927 - H16090927
http://www.uptodate.com/contents/approach-to-abnormal-uterine-bleeding-in-nonpregnantreproductive-age-women?source=outline_link&view=text&anchor=H16090927 - H16090927
http://www.uptodate.com/contents/approach-to-abnormal-uterine-bleeding-in-nonpregnantreproductive-age-women?source=outline_link&view=text&anchor=H16090927 - H16090927
209)- Previously trying to get pregnant, now 4 w amenorrhea, breast tenderness..etc scenario
of pregnant came to u what will u order to test?
A- Progesterone
B- TSH
Answer: Beta hCG
210)- Pregnant with asthma scenario came with SOB, what to do?
A- Chest X-ray
B- Spirometry
A- CT
Answer : B
211)- Pregnant, early with closed os, no adnexal masses, came with mild spotting.. (scenario
clearly denying ectopic)
A- Implantation bleeding
B- D & C
Answer : A
212)- Multiparous, 3 hours 0 station, child vertex, 5 cm dilated, 2 contractions in 10 min, monitor is normal, what to do?
A- Observation
B- Oxytocin
Answer; B?
213)- post hysterectomy lady, doing fine, urine output is ., temperature . , urinary catheter
removed, what will make the doctor not discharge this patient ?
A- inadequate urine output
B- fever
Answer ; A ( by urologist resident )
214)- Lady with lower abdominal pain, Vaginal examination; fornices tenderness ,suprapubic
tenderness, purulent vaginal discharge?
A.
Acute cervicitis
B.
Acute salpingitis
C.
Acute appendicitis
Answer ; B

smle ,2016

552
215)-Lady with cyclic abdominal pain , heavy bleeding, not in contraceptive ,
trying to conceive , what is the investigation ?:
B.
Laparoscopy
-216) 36 GA , uterine contraction , high PB 160/110, epigastric pain , Next: ?
A.
Urine dipstick analysis
B.
Give Tocolytic
Answer : A
High blood pressure with epigastric pain may indicate early eclampsia. Urine dipstick analysis is
appropriate to detect the proteinurea & the need for delivery.
-217) Pregnant w hx of DVT, how to manage?
A.
Heparin
B.
Enoxaparin
C.
Warfarin
D.
No anticoagulant
Answer : B
218)- Adenomyosis definitive diagnosis?
A.
- Endometrial sampling.
B.
- Hysterectomy with ?
Answer : B
219)-pregnant in 3 trimister she compline if swelling in lower limb what will you do ?
1Venogram <bed rest <heparin
2Dpler ,bed rest >heparin
3Clinical , bed rest < warfarin

220)-pregnant relative contraindication of( methyl) post partum hemorage ?


1Dm
2Htn
221)- Breast abscess, gram +ve organism what will u do?
A- Coagulase
B- Oxidase
Answer : A
222)- Painless genital ulcer + lymph nodes enlargement ?
A.
syphilis
B.
2n syphilis
Answer : A
223)- Assessment of delivery by?
smle ,2016

553
A.
-Number of contractions.
B.
-Force of contractions.
C.
-Fetal station.
Answer : C?
evaluation of status of labor, including a description of uterine activity, cervical dilation and effacement, and fetal station and presentation, unless vaginal exam deferred;
evaluation of fetal status, including interpretation of auscultation or electronic fetal monitoring
strips,
224) Post cautery now complain if post coital bleeding. What is the source of bleeding?
A.vulva.
B.vagina.
C.uterine cervix.
D. Uterine body
225) Women deliver baby (dawn syndrome) and she want to know about future pregnancy ?
C.
Aminocenthesis in next pregnancy
D.
Keryotype of infant
E.
Keryotype infant and mother
F.
U/s in next pregnancy
226)16 months post partum present with progressive loss of hearing in RT ear , and now in LT
,
conductive H , dehiscent semilunar canal :?
A.
glue ear,
B.
otosclerosis
C.
Tympanosclerosis
D.
meniere disease
227) Post partum one week only of tearing low mood then normal ?
A.
maternity blues,
B.
post partum depression,
C.
post partum psychosis)

228)Pregnant in 2nd trimester hx of tiredness in first now she is ok all labs normal ex hemoglobin level 10 so management :
iron
folic acid
none
229)Primary amenorrhoea normal breast spared axially and pubic hair ?
A.
Turner
smle ,2016

554
B.

androgen insensitivity syndrome

ADDED Q??
1.
physiologicl changes at term pregnancy ?
A.
increase residulal volume
B.
increase tidal volume
Answer: B

2.
Screening of cervical cancer :?\
pap smear
pt with dysparunia and sever dysmenorrhea on examination post fornix nodule , What
is the managment
1) danazol
Answer:
3.

4.
female came to the primary health (long senario).
How many months after puberty need to complete the length of her spine?
A. 6
B.12
C.36
D. 24
Answer : i think D
5.
G1 P0 came in labor with 6 cm dilation...
She asked for anasthesia .. pudandal nevere blocked . What sensation will be intact?
A. Prenial body
B. Ana sphincter
C. Anterior vestibule
D. Vulva
Answer : C
6.

Pregnant lady (7th week) presented with RLQ pain, febrile (38.5) with tachycardia and
hypotention. labs: normal CBC (no leukocytosis) UA: Normal Diagnosis?

smle ,2016

555
A- Ruptured appendex
B- Ruptured Ectopic Pregnancy*
C- Ruptured Ovarian Cyst
Answer: B
7.

Management and procedures done fro ueterine mass

8.

Pt did pap smear three times and was negative and then had menstrual abnormalities .
she has one kid. Did pap smear and found to be positive of a tumor , whts the next step

9.
2*3 cm ampulla ectopic pregnancy, patient is hemodynamically stable. management?
A.
Laproscopy
B.
Medical* (by Methotrexate, revise the criteria)
C.
Laprotomy
D.
Observe
Answer: B
10. Pregnant taking sulfunurea what the congenital defect:
A.
Teeth
B.
Renal
Answer:
11. 34weeks gestation with decreasre fetal movements u will do?
A.
Non stress test
B.
Biophysical profile
C.
Stress test
Answer: A
12. which ocp will cause hypertension?
Estradiol levonorgesterel
Estradiol progesterone
Estradiol depressing....
Answer:
1St c/s & 2nd forceps delivery now pregnant with 34weeks gestation & 50%effaced cervix
how you will deliver her
c/s
Induction with protein
Induction with syntocinon
Answer:

smle ,2016

556
when you can diagnose the dichroic twins by ultra sound
Early 2nd trimester
Late 2nd trimester
3Rd trimester
Answer: A
which legament prevent utraine prolapse?
raound legament
utrosacral legament
answer : b
vaginal discharge with offensive oder , what is the cause ?
a)gonococcus
b)trachomenatus
c)chlamidia
answer:
gyne, pt has hx of PID came with adnexal mass what next step
13. Average age of menopause?
51
53 years
Answer: A
median age of 51.4 years in normal women
Ref: UpToDate
https://yhdp.vn/uptodate/contents/mobipreview.htm?43/45/44761https://yhdp.vn/uptodate/contents/mobipreview.htm?43/45/44761
https://yhdp.vn/uptodate/contents/mobipreview.htm?43/45/44761
Vaginal discharge Treatment?
Metrondozole cream 4times 7days
Metrondozole tablet
Clindamycine tablet
Clindamycine cream
Answer: depend on other Signs/Symptoms and Discharge color in history..

smle ,2016

557

Transmession of hiv?
Through breast feeding
Answer:
HIV infection is usually acquired through sexual intercourse, exposure to contaminated blood,
or perinatal transmission.
Mother-to-child HIV-1 transmission occurs in utero, peripartum, and postnatally via breastfeeding; without interventions the risk of perinatal HIV-1 transmission is 20 to 45 percent
Ref: UpToDate
https://yhdp.vn/uptodate/contents/UTD.htm?13/2/13353?source=see_link&anchor=H4#H4https://yhdp.vn/uptodate/contents/UTD.htm?13/2/13353?source=see_link&anchor=H4 - H4
https://yhdp.vn/uptodate/contents/mobipreview.htm?28/23/29050https://yhdp.vn/uptodate/contents/mobipreview.htm?28/23/29050
https://yhdp.vn/uptodate/contents/mobipreview.htm?28/23/29050
Adenomyosis treatment?
Ablation
Hysterectomy
OCP
Answer: B
Adenomyosis synonym: endometriosis interna (uterine wall may be diffusely involved)
The only guaranteed treatment for adenomyosis is total hysterectomy.
Ref: UpToDate
https://yhdp.vn/uptodate/contents/mobipreview.htm?26/35/27191#H9https://yhdp.vn/uptodate/contents/mobipreview.htm?26/35/27191 - H9
smle ,2016

558
https://yhdp.vn/uptodate/contents/mobipreview.htm?26/35/27191 - H9
Pregnant lady, assymptomatic, UA: 50,000 CFU bacteria WBC:2 Diagnosis?
Cystitis
Pyelonephritis
Assymptomatic Bacterurea*
Answer: ??
The term asymptomatic bacteriuria refers to the presence of a positive urine culture in an
asymptomatic person.
Asymptomatic bacteriuria in women is guidelines as two consecutive clean-catch voided urine
specimens with isolation of the same organism in quantitative counts of 100,000 CFU/mL
Cystitis is a symptomatic infection of the bladder that can occur alone or can be complicated by
ascending infection and pyelonephritis. Acute cystitis in the pregnant woman is generally considered to be complicated.
A urine culture should be performed in pregnant women with symptoms of acute cystitis. coliform colony counts in voided urine as low as 100 CFU/mL have been noted to reflect bladder
infection
Acute pyelonephritis is suggested by flank pain, nausea/vomiting, fever (>38C), and/or costovertebral angle tenderness and may occur in the presence or absence of cystitis symptoms.
Ref: UpToDate
https://yhdp.vn/uptodate/contents/mobipreview.htm?42/40/43657?source=see_linkhttps://yhdp.vn/uptodate/contents/mobipreview.htm?42/40/43657?source=see_link
https://yhdp.vn/uptodate/contents/mobipreview.htm?42/40/43657?source=see_link
old lady with dry vaginal &itching what you will give?
oesterogen cream
Answer: A
Treatment of Menopause
goal is for individual symptom management
.vasomotor instability
.HRT (first line), SSRIs, venlafaxine, gabapentin, propranolol, clonidine
.acupuncture
.vaginal atrophy
.local estrogen: cream (PremarinR), vaginal suppository (VagiFemR), ring (EstringR)
.lubricants (ReplensR)
.urogenital health
.lifestyle changes (weight loss, bladder re-training), local estrogen replacement, surgery
.osteoporosis
.1000-1500 mg calcium OD, 800-1000 IU vitamin D, weight-bearing exercise, quit smoking
.bisphosphonates (e.g. alendronate)
.selective estrogen receptor modifiers (SERMs): raloxifene (EvistaR) mimics estrogen effects
on bone, avoids estrogen-like action on breast and uterine cancer; does not help hot flashes
smle ,2016

559
.HRT: second-line treatment (unless for vasomotor instability as well)
.decreased libido
.vaginal lubrication, counseling, androgen replacement (testosterone cream or the oral form
AndriolR)
. cardiovascular disease
.management of cardiovascular risk factors
.mood and memory
.antidepressants (first line), HRT (augments effect)
.alternative choices (not evidence-based, safety not established)
.black cohosh, phytoestrogens, St. Johns wort, gingko biloba, valerian, evening primrose oil,
ginseng, Don Quai
Ref: Toroonto Note 2014
amenorrhea otherwise normle .. What to do ?most appreciate test to confirme the diagnosis?
Cardioabdomnal scan
Hormonal test
Bone scan
Onset of mesturation
Answer: B
Diagnostic approach to amenorrhea
-hCG, hormonal workup (TSH, prolactin, FSH, LH, androgens, estradiol)
progesterone challenge to assess estrogen status
medroxyprogesterone acetate (ProveraR) 10 mg PO OD for 10-14 d
any uterine bleed within 2-7 d after completion of ProveraR is considered to be a positive
test/withdrawal bleed
withdrawal bleed suggests presence of adequate estrogen to thicken the endometrium; thus
withdrawal of progresterone results in bleeding
if no bleeding occurs, there may be inadequate estrogen (hypoestrogenism) or excessive androgens
karyotype: indicated if premature ovarian failure or absent puberty
U/S to confirm normal anatomy, identify PCOS
Ref: Toronto Note 2014
increase risk of mortility in pregnant at 35 week
.Pheochromocytoma
.heart faulire
Answer:
???
Pt with PCO was on progestrone and now is off it, at risk of wt:
endometrial ca
cervical ca

smle ,2016

560
Answer: B
Cervical cancer There appears to be an increased risk for developing cervical cancer among
women who have taken OCs
Endometrial cancer The use of oral contraceptive pills decreases the risk of endometrial
cancer
Ref: UpToDate
https://yhdp.vn/uptodate/contents/UTD.htm?38/7/39034?source=see_link
https://yhdp.vn/uptodate/contents/UTD.htm?38/7/39034?source=see_link
https://yhdp.vn/uptodate/contents/UTD.htm?38/7/39034?source=see_link
18 y/o married missed her period for two months, came with rt sided abd pain wts the dx:
ruptured ectopic
Answer: A
Clinical manifestations typically appear six to eight weeks after the last normal menstrual period, but can occur later
The classic symptoms of ectopic pregnancy are:
Abdominal pain
Amenorrhea
Vaginal bleeding
These symptoms can occur in both ruptured and unruptured cases.
Ref: UpToDate
https://yhdp.vn/uptodate/contents/mobipreview.htm?1/9/1178https://yhdp.vn/uptodate/contents/mobipreview.htm?1/9/1178
https://yhdp.vn/uptodate/contents/mobipreview.htm?1/9/1178
40 y/o Pt was normal cycle, now heavy and bleeds intermittently, wt to do to dx:
pap
colposcopy
Answer: A
Papanicolaou (Pap) smear test results for cervical cytology should be current.
Cervical specimens should be obtained if the patient is at risk for an infection.
Ref: medscape
http://emedicine.medscape.com/article/255540-workuphttp://emedicine.medscape.com/article/255540-workup
http://emedicine.medscape.com/article/255540-workup
http://emedicine.medscape.com/article/255540-workup
24 yo bright red bleeding with breast tenderness bad mood neg BHCG OE no abnormality
what to do?
Treatment of Premenstrual Syndrome
goal: symptom relief
smle ,2016

561
psychological support
diet/supplements
. avoid sodium, simple sugars, caffeine and alcohol
. calcium (1200-1600 mg/d), magnesium (400-800 mg/d), vitamin E (400IU/d), vitamin B6
medications
.NSAIDs for discomfort, pain
. spironolactone for fluid retention: used during luteal phase
.SSRI antidepressants: used during luteal phase x 14 d or continuously
.OCP: primarily beneficial for physical/somatic symptoms
.danazol: an androgen that inhibits the pituitary-ovarian axis
.GnRH agonists if PMS is severe and unresponsive to treatment
mind/body approaches
. regular aerobic exercise
. cognitive behavioural therapy
. relaxation, light therapy biofeedback and guided imagery
herbal remedies (variable evidence)
. evening primrose oil, black cohosh, St. Johns wort, kava, ginkgo, agnus castus fruit extract
BSO if symptoms severe
Ref: Toronto Note2014

6 cm dilatation given epidural anesthesia the pain came back and gave her venylphantanile??
Baby stated in distress given fluid without improvement. Ephedrine?? Is it antidote?
Ephedrine Antidote
For hypertension, 5 mg phentolamine mesylate diluted in saline may be administered slowly
intravenously, or 100 mg may be given orally. Convulsions may be controlled by diazepam or
paraldehyde. Cool applications and dexamethasone 1 mg/kg, administered slowly intravenously, may control pyrexia.
Ref: http://www.rxlist.com/ephedrine-drug/overdosage-contraindications.htmhttp://www.rxlist.com/ephedrine-drug/overdosage-contraindications.htm
http://www.rxlist.com/ephedrine-drug/overdosage-contraindications.htm
Characteristic for premenstrual syndrome. Which phase or behavioral or symptomatic abnormality.
Diagnostic Criteria for Premenstrual Syndrome
at least one affective and one somatic symptom during the 5 d before menses in each of the
three prior menstrual cycles
. affective: depression, angry outbursts, irritability, anxiety, confusion, social withdrawal
. somatic: breast tenderness, abdominal bloating, headache, swelling of extremities
symptoms relieved within 4 d of onset of menses
symptoms present in the absence of any pharmacologic therapy, drug or alcohol use

smle ,2016

562
symptoms occur reproducibly during 2 cycles of prospective recording
patient suffers from identifiable dysfunction in social or economic performance
Ref: Toronto Note 2014
young lady around 27 (not sure) symptomatic ( do not remember ) US shows bilateral ovarian
cysts next in assessment ?
CA 125
Histopathology (not sure)
Answer:??
best to confirm menopause?
FSH
LSH
Estrogen
Progesterone
Answer: A ,based on old studies
We diagnose menopause as 12 months of amenorrhea in the absence of other biological or
physiological causes. A high serum FSH is not required to make the diagnosis.>>>>>UpToDate
https://yhdp.vn/uptodate/contents/mobipreview.htm?43/45/44761#H93667232
https://yhdp.vn/uptodate/contents/mobipreview.htm?43/45/44761 - H93667232
Action of cytotoxic?
IL6
IL10
TNF gamma
Answer:not clear
lactating women with mastitis?
Continue breastfeeding
Draining Antibiotic
Answer: A(Toronto notes 2016 OB48)
Pap smear method is?
3 samples from endocervix
smle ,2016

563

Answer:not enough choices

after dilvery of baby , placenta not yet deliverd, after 30 min bleeding >800 ml Type of postpartum hemrr?
1ry
2ry
Answer: A(Toronto notes 2016 OB45)
primary
secondary
27 Gastion pregant with monoamniotic twin , one of them died?
Give steroid and deliver
Wait to 34 then deliver
Wait 37 then deliver
Wait until SVD
Answer: a or b
80% effecment , 4 cm dilated cervix on i.v oxytocin, she is stable on ctg variable acceleration ?
Stop oxytocin
smle ,2016

564
Give terbutaline
Change mother position
Expectant delivery
Answer:D (not sure and no source)
pregnent lady came to the clininc with at 10 week with 2 hr bleeding , examination revealed
close Os , the fundus is palpapul ? Cm above symphusis pupic . what is the cause :(not complete q)
Rupture cyst
Inferiorly located placenta
Ectopic pregnancy
Answer:early bleeding, goes more with Ectopic and cyst.
for pap smear
three spicemin from endocervix
-2 spicemn from two different areas
one spicemin from endocervix
one spicemn from cervical os
Answer: Anot sure

giving birth, cervix 6 cm dilated, fetus is left occiptoposterior , signs of molding can be felt
what is the stage
first
second
third

smle ,2016

565
Answer: A(first stage of labor)

40 years old female came for pap smear , everything normal in imaging , history and examination what to tell her
pap smear is not indicated
indicated annually
every 5 years
3 normal and then no indication
Answer: Dnot sure
obese female with uncontrolled DM, presented with menorrhagia how to investigate
MRI
Endometrial biopsy
Answer: B not sure
pregnant lady with DVT what is the best investigation to diagnose?
-d-dimer
-dupplex to calf muscle
-CT-angio
Answer: B(Toronto notes 2016 OB31)
case of 40 weak Gestation 5cm cervical dilation rupture membrane ,good CTG contraction is
good 3 hours what is your action?
-prostaglandin E1(cervical cream)
oxytocin
smle ,2016

566
Answer: Baccording to bishop score, this is favorable so oxytocin. Not sure
49 -marker for ovarian cancer
-ca125
Answer: A
Ovarian Tumour Markers
Epithelial cell CA-125 Stromal
Granulosa cell inhibin
Sertoli-Leydig androgens Germ cell
Dysgerminoma LDH
Yolk sac AFP
Choriocarcinoma -hCG
Immature Teratoma none
Embryonal cell AFP + -hCG
(Tornto notes 2016 GY41)
50- 20 years old lady, pregnant, exposed to rubella virus since 3 days, never was vaccinated
against rubella mumps or measles, what's the best thing to do?
Give IG
Vaccine
-Do nothing
-Terminate the pregnancy
Answer: C no source
51- most common cause of post partum hemorrhage?
-Uterine atony
Answer: A(tornton notes 2016 OB45)
52- site of fibroid cause abortion?
-submucosal
-intramural
serosal
smle ,2016

567

Answer: A
Leiomyomata/Fibroids
Benign smooth muscle tumour of the uterus (most common gynecological tumour)
(Tornoto notes 2016 GY15)
53) 3 months pregnant woman scenario of Bacterial vaginosis what to give?
-oral metronidazole
-ceftriaxon
-cream
54-15 year old girl came with her mother complain that there is no menses , there is breast
bud and pubic hair (normal 2ndary sexual characteristics ) :
-primary amenorrhea (my answer although it should be 16yo)
-secondary amenorrhea
55)-PRESNTESTION OF MALIGNANT VULVAR LESIONS ?
Perineum
labia major
a clittores
labia minor
Answer \ LABIA MAJOR torento note 2015
56)- to perform instrumental delivery you should first exclude
cephalopelvic disproportion
placental rupture
breech presentation
uterine rupture
Answer: a
http://www.slideshare.net/cdhnmj/instrumental-delivery
57)- after delivery of baby , placenta not yet delivered, after 30 min bleeding >800 ml Type of
PPH?
1ry
2ry
Answer: primary
58)- 80% effecment , 4 cm dilated cervix on i.v oxytocin, she is stable on ctg variable acceleration ?
Stop oxytocin
smle ,2016

568
Give terbutaline
Change mother position
Expectant delivery *
59)-70 years with dysfunction uterine bleeding , rx??
OCP
hystrectomy *
the answer is 2 ?
60)- Women treated in the past from pelvic Inflammatory disease... Now in US have bilateral
ovary cyst.. During surgery dark blood come from ovaries.. Dx ?
ANSWER : Chocolate cysts (Ovarian endometriosis)Chocolate cysts are affecting women during
their reproductive period and may cause chronic pelvic pain associated with menstrual periods
(menstrual cramps, endometriosis).
the chocolate cyst is the cyst of the ovary with intracavitary hemorrhage and formation of a hematoma containing old brown blood.
http://lakecharlesobgyn.com/Complete/246-Endometriosis-ChocolateCysts.aspxhttp://lakecharlesobgyn.com/Complete/246-Endometriosis-Chocolate-Cysts.aspx
61-Case of pregnant complaining of sever pain and bleeding , US done showed fibroid and viable fetus :
hysterectomy
termination of pregnancy
analgesic
62)- Patient 39 week in labor , reactive CTG , on examination you feel orbital margin nose and
chine How will you manage her?
Delevire her in operating room
Emergency c/s
Oxytocin

63)- 25 years old female was diagnosed with pelvic inflammatory disease 3 years ago which
was completely treated for it, presented now with 3 month trying to convince but didn't success Investigation were Norma and semen analysis was normal also Her BMI is 35 LH and FSH
were low How will manage her ?
Induction ovulation and IVF
Induction ovulation and normal conceive
Advice here to reduse Her BMI to 23 and trying to get pregnant
64)- Pregnant 8 week of gestation presented with sever abdominal pain followed by heavy
bleeding Examination reveals tens abdomen
smle ,2016

569
What is most likely diagnosis:
Threatened abortion
Ectopic pregnancy*
C . Ovarian failure
answer: 2?
65)Q-Poly cystic ovary What investigation help you in reaching diagnosis:
FSH /LH*
Ct of thica cells of ovary
answer: 1
67)-36 yr old female use condom as contraceptive. she complain of nausea & amenorrhea.
what is first investigation to do?
beta HCG
68)- pregnant lady in her 8 week of gestation came and complain that she loss pregnancy sensation & there is vaginal spotting. what to do to establish the diagnosis
1. mother serum A P
2. trans vaginal US
3. serum b HCG
69)- which of the following use in judgment on progression of delivery
1.frequency of the contraction
2. strong of contraction
3. descent of the baby

70)-which of the following is contraindicated for assistant delivery by forceps ?


1. breach presentation
2. face presentation
3. cephalopelvic disproportion
Answer: 3
reference: http://emedicine.medscape.com/article/1848372-overview#a5http://emedicine.medscape.com/article/1848372-overview - a5

71)- pregnant lady in her 41 wk of gestation admitted for delivery induction. after oxytocin
was given she start having contraction and there is 4 cm dilation & 60 % effacement. after
one hour there is 8cm dilatation of the cervix & 80% effacement. baby pulse is 120-140, also
there is acceleration & variability. what is the correct action to do
1. expectant delivery

smle ,2016

570
2. stop oxytocin
3. go immediately for CS
Answer: 1
72)-which of the following drug safe during pregnancy
1. erythromycin
2. cephalosporin
3. warfarin surgery
Answer: 1
http://www.mayoclinic.org/healthy-lifestyle/pregnancy-week-by-week/expert-answers/antibiotics-and-pregnancy/faq-20058542http://www.mayoclinic.org/healthy-lifestyle/pregnancyweek-by-week/expert-answers/antibiotics-and-pregnancy/faq-20058542
http://www.mayoclinic.org/healthy-lifestyle/pregnancy-week-by-week/expert-answers/antibiotics-and-pregnancy/faq-20058542
73)HIV moter 34 week GA on antiviral medication, viral load is low what do you do?
Continose with antiviral medication and elective CS
Med: direct HX what DX: acute pericarditis
74)- pregnant 8w present miss symptom of pregnency and bleed no pain to approve your DX:
Vaginal us
Bhcg
Answer: 1 we suspect abortion
75- pregnancy 36w present with liquid to support dx:
chemical ....
spculum examin
vaginal examin
answer: 1
76)- pregnency have flu symptom give:
oslamavir
ozanamivr
actomab
answer :1
Oseltamivir is preferred for treatment of pregnant women (Rasmussen, 2009;
2011). Pregnant women are recommended to receive the same antiviral dosingas nonpregnant
persons http://www.cdc.gov/flu/professionals/antivirals/antiviral-dosage.htm.
It is best to start antiviral medications within the first 48 hours of developing symptoms, but
antivirals can also be used after this time period. A 75-mg capsule of oseltamivir (Tamiflu) twice
per day for 5 days is the recommended first choice antiviral. https://www.nlm.nih.gov/medlineplus/ency/article/007443.htmhttps://www.nlm.nih.gov/medlineplus/ency/article/007443.htm
https://www.nlm.nih.gov/medlineplus/ency/article/007443.htm
smle ,2016

571
76)- ob mass mobile not related to cyclic pain:
fibroadeoma
fibrocystic change
intraducal papailoma
answer: 1
77)- pap smear 2 sample was atypia:
biopsy endometrrial
colposcopy
answer: 2
76)-pregnancy has HX of previous DVT what give:
heparin
enoxaparin (anticoagulation)
answer:2
77)- Breastfeeding mother treated with HCV by interferon more than one year what the risk
of breastfeeding on infant?
Nipple crackled
Mother with anemia
Infant complain of oral candidacies
Not follow up of infant immunization
Answer: 1
https://www.cdc.gov/breastfeeding/disease/hepatitis.htmhttps://www.cdc.gov/breastfeeding/disease/hepatitis.htm
https://www.cdc.gov/breastfeeding/disease/hepatitis.htm
78)- Triad of ascites pleural effusion and ovarian mass. What is the most likely tumor?
Sex cord stromal tumors
B - epithelial tumors
germ cell tumors
Answer: 1
Meigs syndrome is defined as the triad of benign ovarian tumor its benign sex cord-stromal tumour
79)- 24 married for 9 months with regular heavy menses and pain, on examination there is a
nodule in cervix and tenderness, what is the cause?
fibroid
endometriosis
cervical cancer
vaginal cancer

smle ,2016

572
Answer: 1 http://www.fibroidsecondopinion.com/fibroid-symptoms/http://www.fibroidsecondopinion.com/fibroid-symptoms/
http://www.fibroidsecondopinion.com/fibroid-symptoms/
80)- pregnant women complaining of UTI at 12 weeks then treated, now complaining of dysuria, shetakes () medication for 4 days, what u will do:
give her small dose Abx till delivery
change drug
treat even asymptomatic
Answer: 3
81)-80 years women had yellow watery foul smelling vaginal discharge:
bacterial vaginosis
trichomonas vaginalis
atrophic vaginitis
answer: 2
.
82)female with Tubo ovarian abscess what is the treatment?
Treatment is different if the TOA is discovered before it ruptures and can be
treated with IV antibiotics. During this treatment, IV antibiotics are usually
replaced with oral antibiotics on an outpatient basis
depending on anatomical staging and is usually reserved for
-. cancers thathave not spread beyond the vulva -Radiation therapy may be used in more
advanced vulvar cancer cases when disease has spread to the lymph nodes
and/or pelvis Chemotherapy is not usually used as primary treatment but
may be used in advanced cases with -spread to the bones, liver or lungs. It
may also be given at a lower dose together with radiation. therapy
83)- Elevated in menopause lady?
Progesterone or LH
84)- Mass out of vagina with coughing and defecation?
Ureterovaginal prolapse or rectovaginal prolapse
85)- Pregnant, early with closed os, no adnexal masses, came with mild spotting.. (scenario
clearly denying ectopic)
1- Implantation bleeding
2- D & C
Answer : 1
86)- post hysterectomy lady, doing fine, urine output is ., temperature . , urinary catheter
removed, what will make the doctor not discharge this patient ?
1- inadequate urine output
2- fever
smle ,2016

573
Answer ; 1 ( by urologist resident )
87)- 36 GA , uterine contraction , high PB 160/110, epigastric pain , Next:
1 . Urine dipstick analysis
2 . Give Tocolytic
Answer : 1
High blood pressure with epigastric pain may indicate early eclampsia. Urine dipstick analysis is
appropriate to detect the proteinurea & the need for delivery.

88)-pregnant in 3 trimister she compline if swelling in lower limb what will you do ?
1Venogram <bed rest <heparin
2Dpler ,bed rest >heparin
3Clinical , bed rest < warfarin
89)-pregnant relative contraindication of( methyl) post partum hemorage ?
1Dm
2Htn
90)- Breast abscess, gram +ve organism what will u do?
1- Coagulase
2- Oxidase
Answer : 1
91) - Painless genital ulcer + lymph nodes enlargement
1- syphilis
2- 2n syphilis
Answer : 1
27- staph saprophyticus vaginal infection, whats a risk factor for it:
A. septicides in condoms
B. douching habits
Answer: A
sorce: http://cid.oxfordjournals.org/content/40/6/896.fullhttp://cid.oxfordjournals.org/content/40/6/896.full
postpartum patient present with passing of stool through vagina :
a) vesicuvaginal fistula
b) rectovaginal fistula
Answer:B
pregnant lady had placenta apriva to know type by&gt;&gt;
Q-48 Y Wom en with fibroid 5 or 6 cm asymptomatic :
Hysterectomy
smle ,2016

574
Myomectomy
Regular follow up yearly
Follow up and CBC every 2 month
Q- G3P2+0. Her first visit was on the 20th week of gestation. She has history of two premature deliveries. Her cervical length was 30 mm. what is your appropriate management?
Strict bed rest
Terminate her pregnancy
Immediate cerclage
Inject her with progesterone
answer: 4 or 3
Several studies have indicated that the likelihood of preterm delivery increases with decreasing
cervical length. A cervical length of 2530 mm before 32 weeks gestation seems to increase the
risk of preterm delivery. If examination and ultrasound show that you have an abnormally short
cervix, and youre less than 24 weeks pregnant, your practitioner may recommend cerclage, a
procedure in which she stitches a band of strong thread around your cervix to reinforce it and
help hold it closed. However, theres a lot of controversy about whether cerclage should be
used in this situation."

Methotrexate Overdose, treatment?


A. Folic Acid*
B. Folonic Acid
C. Cobalamine
Answer:
Most common cause of Leukuria ?
answer: a-estrogen imbalance
Leukorrhea is a thick, whitish or yellowish vaginal discharge.There are many causes of leukorrhea, the usual one being estrogen imbalance. The amount of discharge may increase due to
vaginal infection or STDs, and also it may disappear and reappear from time to time, this discharge can keep occurring for years in which case it becomes more yellow and foul-smelling; it
is usually a non-pathological symptom secondary to inflammatory conditions of vagina or cervix.[3]
ptn with Resistant salpingitis what is the organism ? Gonorrhea , chlamydia ,strepto,e.coli (i
think)
Fractured pelvis, injury to gonadal artery, what organ affected? (not sure of the Q)
ovary?
Answer: the pelvis forms one major ring and two smaller rings of bone that support and protect
the bladder, intestines and rectum.
Organ pelvic protect (Bladder, lower colon, lymph nodes, uterus, and vagina)
-typical case of PCO. Dx ?
smle ,2016

575
SteinLeventhal syndrome(other name)

Drug that interfere with OCP ?


Antiepileptic ? Not sure
patient with Resistant salpingitis what is the organism ?
Gonorrhea
chlamydia
Strepto
e.coli
Answer:
case about an Rh ve women married an Rh +ve man, their first child was Rh +ve like the father, now the wife is pregnant for the second time,
not sure about the question, or what they were asking about, may about the frequency of
monitoring, and the titers.
Lady on cervical exam you visualize mass 00*00 cm what to do ?
Cone biopsy
excision
Reassure
Take sample for histopathology (or smth like this )
staph saprophyticus vaginal infection, whats a risk factor for it:
A. septicides in condoms
B. douching habits
Answer: A
sorce: http://cid.oxfordjournals.org/content/40/6/896.fullhttp://cid.oxfordjournals.org/content/40/6/896.full
postpartum patient present with passing of stool through vagina :
a) vesicuvaginal fistula
b) rectovaginal fistula
Answer:B
Signs and symptoms of a rectovaginal fistula may include:
Passage of gas, stool or pus from your vagina.
Foul-smelling vaginal discharge.
Recurrent vaginal or urinary tract infections.
Irritation or pain in the vulva, vagina and the area between your vagina and anus (perineum)
Pain during sexual intercourse.

smle ,2016

576

pregnant lady had placenta apriva to know type by&gt;&gt;


Q-48 Y Women with fibroid 5 or 6 cm asymptomatic :
Hysterectomy
Myomectomy
Regular follow up yearly
Follow up and CBC every 2 month

http://www.uptodate.com/contents/overview-of-treatment-of-uterine-leiomyomas-fibroids?source=see_link
Q- G3P2+0. Her first visit was on the 20th week of gestation. She has history of two premature deliveries. Her cervical length was 30 mm. what is your appropriate management?
Strict bed rest
Terminate her pregnancy
Immediate cerclage
Inject her with progesterone
answer: 4 or 3
Several studies have indicated that the likelihood of preterm delivery increases with decreasing
cervical length. A cervical length of 2530 mm before 32 weeks gestation seems to increase the
risk of preterm delivery. If examination and ultrasound show that you have an abnormally short
cervix, and youre less than 24 weeks pregnant, your practitioner may recommend cerclage, a
procedure in which she stitches a band of strong thread around your cervix to reinforce it and
help hold it closed. However, theres a lot of controversy about whether cerclage should be
used in this situation."
Methotrexate Overdose, treatment?
A. Folic Acid*
B. Folonic Acid
C. Cobalamine
Answer:: B
http://www.mdpoison.com/media/SOP/mdpoisoncom/ToxTidbits/2009/January%202009%20toxtidbits.pdfhttp://www.mdpoison.com/media/SOP/mdpoisoncom/ToxTidbits/2009/January 2009 toxtidbits.pdf
http://www.mdpoison.com/media/SOP/mdpoisoncom/ToxTidbits/2009/January 2009 toxtidbits.pdf
Most common cause of Leukuria ?
answer: a-estrogen imbalance
Leukorrhea is a thick, whitish or yellowish vaginal discharge.There are many causes of leukorrhea, the usual one being estrogen imbalance. The amount of discharge may increase due to

smle ,2016

577
vaginal infection or STDs, and also it may disappear and reappear from time to time, this discharge can keep occurring for years in which case it becomes more yellow and foul-smelling; it
is usually a non-pathological symptom secondary to inflammatory conditions of vagina or cervix.[3]
ptn with Resistant salpingitis what is the organism ? Gonorrhea , chlamydia ,strepto,e.coli (i
think)
Fractured pelvis, injury to gonadal artery, what organ affected? (not sure of the Q)
ovary?
Answer: the pelvis forms one major ring and two smaller rings of bone that support and protect
the bladder, intestines and rectum.
Organ pelvic protect (Bladder, lower colon, lymph nodes, uterus, and vagina)
-typical case of PCO. Dx ?
SteinLeventhal syndrome(other name)

Drug that interfere with OCP ?


Antiepileptic ? Not sure
Medications that May Interact with the Oral Contraceptive (OC):
MEDICATION

SIGNIFICANCE

SEIZURE MEDICATIONS: Phenytoin (Dilantin) Felbamate (Felbatol) Carbamazepine (Tegretol) Barbiturates (Pentobarbital,
Seco-barbital, Primidone, Butabarbital, Butalbital)

Lamictal

Sodium valproate (Depakene)

7 more rows, 2 more columns


patient with Resistant salpingitis what is the organism ?
Gonorrhea
chlamydia
Strepto
e.coli
smle ,2016

578
Answer:
case about an Rh ve women married an Rh +ve man, their first child was Rh +ve like the father, now the wife is pregnant for the second time,
not sure about the question, or what they were asking about, may about the frequency of
monitoring, and the titers.
Lady on cervical exam you visualize mass 00*00 cm what to do ?
Cone biopsy
excision
Reassure
Take sample for histopathology (or smth like this )

Surgery
smle ,2016

579

Q1/ Patient had injury to the ulnar nerve?


a.
Complete claw hand
b.
Partial claw
Answer: B
Effects of the ulnar nerve injury: ulnar claw hand (Partial claw hand) and hollowing of skin in the first
web space on dorsal aspect of hand.
Complete claw hand results from combined lesions of the median and ulnar

Reference: Textbook of Anatomy - Upper Limb and Thorax


Q2/Pronator teres syndrome, which nerve is entrapped
a.
Ulnar
b.
Radial
c.
Median
Answer: C
Reference: https://www.youtube.com/watch?v=ZqhO1dzqTtY

smle ,2016

580
Pronator teres syndrome is a compression neuropathy of the median nerve at the elbow. It is rare compared to compression at the wrist (carpal tunnel syndrome) Reference: Wikipedia
Reference:
i.
https://www.youtube.com/watch?v=ZqhO1dzqTtY ,
ii.

http://radiopaedia.org/articles/pronator-teres-

syndrome-2

Q3/Basal skull fracture with loss of sensation under the eye. Which of the following nerves is affected?
a.
frontal
b.
trochlear
c.
infraorbital
d.
supraorbital
Answer: C

Q4/ 16 years old female complaining of abdominal pain for 2 days. It started in the periumbilical
area then it radiated to the right lower quadrant. It is associated with anorexia. On examination,
right lower mass was felt. What is the management?
a.
surgery immediately
b.
conservative
*Antibiotics werent mentioned in the choices.
Answer: B

smle ,2016

581
Appendectomy remains the only curative treatment of appendicitis, but management of patients with an
appendiceal mass can usually be divided into the following 3 treatment categories:
Patients with a phlegmon or a small abscess: After intravenous (IV) antibiotic therapy, an interval appendectomy can be performed 4-6 weeks later.
Patients with a larger well-defined abscess: After percutaneous drainage with IV antibiotics is performed,
the patient can be discharged with the catheter in place. Interval appendectomy can be performed after the
fistula is closed.
Patients with a multicompartmental abscess: These patients require early surgical drainage.
Reference: Medscape.
If an appendix mass is present and the condition of the patient is satisfactory, the standard treatment is the conservative OchsnerSherren regimen. This strategy is based on the premise that the
inflammatory process is already localised and that inadvertent surgery is difficult and may be dangerous. It may be impossible to find the appendix and, occasionally, a faecal fistula may form. For
these reasons, it is wise to observe a nonoperative programme but to be prepared to operate should
clinical deterioration occur.
Reference: Bailey & Loves Short Practice of Surgery 26th
Q5/The most common cause of breast bloody discharge:
a.
ductal papilloma
Answer: A
Reference: Bailey & Loves Short Practice of Surgery 26th
Q6/A mass at the middle of the Esophagus:
a.
Adenocarcinoma
b.
Squamous
Answer: B
We have two types of Esophageal CA:
Adenocarcinoma usually at the GE junction, due to chronic reflux that cause dysplasia.
Squamous cell carcinoma in most of the esophagus
Reference: Surgical Recall, 6th edition
Q7/Which of these supplements protect against colorectal cancer?
a.
folic acid
b.
vit E
Answer: A
Reference:http://www.hopkinscoloncancercenter.org/CMS/CMS_Page.aspx?CurrentUDV=59&CMS_Page_ID=1293D614-71B1-4A5A-8CFD-7BF8760295FA
Q8/ Median nerve injury leads to:
a.
Complete claw hand
b.
Ape hand
c.
Partial claw
d.
Ulnar Claw
Answer: B
Reference: Textbook of Anatomy - Upper Limb and Thorax
Q9/ Cancer of the lower lip. What are the lymph nodes that you will examine first?
submental, submandibular.
Submental, Buccal.

smle ,2016

582
Buccal, parapharyngeal.
Answer: A
The lymphatic drainage of both the upper and lower lips is primarily to the submandibular group of
lymph nodes. To a lesser extent, drainage may go to submental intraparotid, or internal jugular lymph
nodes.
Reference: http://www.ncbi.nlm.nih.gov/pubmed/8460042
http://www.cancer.gov/types/head-and-neck/hp/lip-mouth-treatment-pdq#link/_364_toc
Q10/A patient with achalasia. Repeated attempt of balloon expansion relapses again. What is the
management?
stent insertion
myotomy
nasogastric tube
Answer: B
Surgery was considered after 3 trials of balloon dilatation within a period shorter than 12 months
http://www.ncbi.nlm.nih.gov/pmc/articles/PMC3862145/
Q11/Patient with chronic osteomyelitis of her leg and there is sinus draining. She has cancer of skin
near the sinus which has has eosinophilic inclusions. What is the type of cancer?
Squamous cell carcinoma.
Basal cell carcinoma.
Answer: A
De novo squamous cell carcinoma emerge in the setting of long-standing ulcers, burn scars, or osteomyelitis. Reference: Journal of Skin Cancer.
Q12/A patient was not able to move her arm above shoulder after breast CA surgery that
involved axillary dissection. What is the nerve injured?
long thoracic nerve
rotator cuff
supraspinatus
brachial plexopathy
Answer: A
We have four nerves must the surgeon be aware of during an axillary dissection:
1. Long thoracic nerve travels from superiorly to inferiorly along the chest wall at the medial aspect of the
axilla and innervates the serratus anterior muscle. Injury to this nerve causes a winged scapula in which
the medial and inferior angle of the scapula abduct away from the chest wall with arm extension.
2. Thoracodorsal nerve courses along the posterior border of the axilla from superiorly to inferiorly on the
subscapularis muscle and innervates the latissimus dorsi. Injury to this nerve causes weakness in arm abduction and external rotation.
3. Medial pectoral nerve travels from the posterior aspect of the pectoralis minor muscle around the lateral border of the pectoralis minor to the posterior aspect of the pectoralis major muscle. It innervates the
lateral third of the pectoralis major; injury to this nerve results in atrophy of the lateral pectoralis major
muscle.
4. Lateral pectoral nerve
Intercostal brachial sensory nerves: travel laterally in the axilla from the second intercostal space to the
medial upper arm. Transection causes numbness in the posterior and medial surfaces of the upper arm.
Reference: Surgical Recall, 6th edition

smle ,2016

583
Q13/Patient underwent orthopedic surgery. 2 days after the surgery he started to complain of swelling, erythema and tenderness at the site of surgery. Which organism can cause the wound infection?
a. Clostridium perfringens
b. Other choices included different subtypes of clostridium
C. perfringens is the only Clostridium species can cause wound infection (Gas gangrene; A medical emergency)
Answer: A
Q14/A 32 years-old alcoholic male patient is brought to the emergency department with the history
of vomiting large amount of bright red blood. Physical examination revealed splenomegaly and ascites. Which of the following is the most likely source of bleeding?
a.
Duodenal ulcer.
b.
Proton pump.
c.
Esophageal varices.
d.
Gastric cancer.
Answer: C
Q15/A patient with indirect inguinal hernia. What artery runs medial to it?
inferior epigastric artery
Answer: A
Q16/What is the commonest nerve injured after thyroidectomy
a.
recurrent laryngeal nerve
Answer: A
Q17/Patient came with abdominal swelling and weak abdominal wall. Whats the diagnosis?
direct inguinal hernia
Answer: A
Q18/ Winging scapula:
a.
long thoracic nerve injury
Answer: A
Q19/DVT in pregnant. What will you do?
Compression ultrasound
Answer: A
NB. CUS is the best initial investigation in pregnancy
Reference: Medscape
Q20/Obturator nerve injury. Which muscle will get full paralysis?
Adductor Magnus
Adductor longus
Answer: B (Adductor longus)
NB. Adductor magnus has nerve supply from 2 nerves.
NB. The hamstring part of Adductor Magnus innervate by (Tibial branch of sciatic nerve) .
Reference: Clinical Anatomy
Q21/23 years old woman with cyclic bilateral nodularity in her breast since 6 months. On examination there is tender 3 cm mobile subareolar mass on her right breast. What will you do next?

smle ,2016

584
FNA with cytology
Mammogra
Biopsy
Follow up for next cycle
Observation
Reference: First Aid USMLE
NB. There is shortage information in this Q, some consultants opinion was D.

Q22/Long thoracic nerve damage with winging of scapula, this nerve arises from which part of the
brachial plexus?
Upper trunk.
Posterior cord.
Medial cord.
Root
Answer: D Root: C5, C6, C7
Reference: Toronto Notes 2015
Q23/Pectoralis major muscle was removed during mastectomy. Which action will be lost?
Abduction
Adduction
Extension
Internal rotation or flexion
Answer: B
adduction and medial rotation
Reference: Clinical Anatomy

smle ,2016

585
Q24/A patient presented complaining of dark stool and vomiting blood. He is stable
now. What is your investigation?
Upper GI endoscope.
Answer: A
Mostly from upper GIT bleeding
Reference: Master The Boards 3rd Edition
Q25/In which position will you apply a splint in patients with carpal tunnel syndrome.
Dorsiflexion.
Plantarflexion
Extension
Abduction
Answer: A
The ideal position of the wrist is close to neutral with 2 + 9 of dorsiflexion and 2 + 6 ulnar deviation to prevent prolonged flexion or extension that prevents any raise in the pressure.
Reference: Medscape

Q26/Which one of these layers is involved in chagas and hirschsprung disease?


a.
muscularis externa
b.
mucosa
c.
submucosa
Answer: A
Hirschsprung disease (congenital megacolon) and Chagas disease have different etiologies, but both
inhibit intestinal motility by affecting the myenteric (Auerbach's) plexus located between the layers
of the muscularis externa.
Reference: Davidsons's essentials of medicine.

Q27/ Child was crying since 4 hours ago. He has fever and vomiting. On examination there was a
mass on the RUQ. What is your diagnosis?
Appendicitis
Gastroenteritis
Pancreatitis
Intussusception
Answer: D
Historical Facts Vomiting, abdominal pain and bloody, mucoid stools (that look like 'currant jelly') are
classic, although it is rare that all three present in the same patient. The symptoms are episodic, in time
with the peristaltic waves, and in between the child is left exhausted, sometimes to the point of coma.
Pertinent Physical Examination Findings The child is intermittently fussy and typically has guaiac-positive stool (bloody). An allantoid-shaped mass is sometimes palpable in the right upper quadrant (RUQ).
Reference: MedScape.

Q28/ Treatment of hirschsprung disease


a.
Surgical repair

smle ,2016

586
Answer: A (Resection of the affected segment)
Reference: Davidsons's essentials of medicine.

Q29/ 40 years old woman with no pain but you noticed jaundice. She has high direct bilirubin
and high ALT what is your diagnosis?
Gilberts disease
Biliary stone
Answer : ?
- Gilberts disease classified as a prehepatic (associated with high indirect bilirubin) so we exclude this
answer.
- Other DDx: pancreatic head CA, Cholangiocarcinoma and primary biliary cirrhosis
o A 3-fold or more increase in ALT strongly suggests pancreatitis.
o Intrahepatic obstruction: Alanine aminotransferase (ALT) is predominantly found in the liver, and
most elevations are due to intrahepatic disease. Although less specific to the liver, the AST level is
also elevated in cases of intrahepatic cholestasis. ALT and AST levels are usually elevated to the
same degree in patients with viral hepatitis and those with drug-induced liver damage. In association
with alcoholic liver disease, cirrhosis, and metastatic lesions to the liver, the AST level is elevated
more frequently than the ALT level. In general, AST levels are usually higher than ALT levels.
Answer: its most probably is a hepatic cause like Viral hepatitis, liver cirrhosis, ascending cholangitis

Q30/ You did rectal examination to someone and found there is a mass in front of anterior rectal
wall which was diagnosed as adenoma. What is the location of this neoplasm?
Anterior
Posterior
Lateral
Median
Answer: B?
The prostate is divided into lobes:
The anterior lobe is the portion of the gland that lies in front of the urethra. It contains no glandular
tissue but is made up completely of fibromuscular tissue.
The median or middle lobe is situated between the two ejaculatory ducts and the urethra.
The lateral lobes make up the main mass of the prostate. They are divided into a right and left lobe and
are separated by the prostatic urethra.
The posterior lobe is the medial part of the lateral lobes and can be palpated through the rectum during
digital rectal exam (DRE).
Reference: http://www.cdc.gov/cancer/npcr/pdf/abstracting/prostate.pdf
Reference: baily and love

smle ,2016

587

Q31/ A patient with diaphragmatic hernia and you want to cut the phrenic nerve. Where can you
do that?
Anterior to scalenus anterior
Anterior to scalenus medius
posterior to scalenus anterior
posterior to scalenus medius
Answer: A

Q32/ 24 years old girl came to you with a painless mobile breast mas that does not change with
menstrual cycle. It started increasing since 3 months. Now its 3 cm in size. What is the most likely
cause?
Fibroadenoma
Fat tissue
Cyst
Answer: A
Fibrocystic: changes with menstrual cycles (hormones), lumps in both breasts
Fibroadenomas. These are the most common benign tumors. They are solid, round, rubbery lumps that
move freely. Theyre usually painless.

smle ,2016

588
Simple cysts: Simple cysts are fluid-filled sacs that usually happen in both breasts. There can be one or
many. They can vary in size. Tenderness and size often change with menstrual cycle.
Intraductal papillomas: These are small, wart-like growths in the lining of the mammary duct near the
nipple. They usually affect women who are 45 to 50. They can cause bleeding from the nipple.
Q33/ They want to stop screening for breast cancer for women under 47 to decrease unnecessary
anxiety to the public. You didnt agree because there is a gene that cause cancer in young women.
What is the gene?
APC
BRCA2
Answer: B
BRCA1 or BRCA2 mutation.
Q34/ What will tell you tell a young lady in regard to breast ca?
a.
Self exam is obsolete now
b.
Self exam as well as mammography are important
c.
Only mammography
d.
CT scan
Answer: B (Were not sure about the guidelines they are following)
Q35/ A patient with thenar atrophy but with no numbness. What is the nerve affected?
Axillary
Ulnar
Median (Anterior interosseous nerve)
Radial
Answer: C
Thenar atrophy is an eroding of muscle tissue, which can impair control over the thumb and leave the
hand disfigured. The problem can be a complication of several different conditions and disorders, including carpal tunnel syndrome, acromegaly, or direct trauma to the wrist or thumb.
36/ 12 years old male with undescended testes. The surgeon excised it. What is the most likely histology?
Malignant transformation
Normal histology
Answer: B
6 % of testicular tumors develop in patients with a history of cryptorchidism (undescended testes).
Reference: Surgical Recall, 6th edition
Q37/ 45 years old female presented with neck swelling and anxiety. On examination the swelling is
moving with swallowing and lateral to midline. What is the most likely diagnosis?
Thyroglossal cyst
Branchial cyst
Thyroid nodule
Answer: C
Q38/ Which of the following suggest ductal papilloma on breast examination?
Blood stain on nipple
Hemorrhagic discharge

smle ,2016

589
Serous discharge
Pus from nipple
Answer: A
Q39/ A patient came to your clinic with inflammation, redness and swelling around the nail. You
gave 1 week course of augmentin but of no use. What should be done? (Picture was also attached)
give augmentin for one more week
incision and drainage
warm soaks
splint
Answer: B
Q40/ What is the best study to determine cystic breast lesion?
Answer: US is the best to evaluate cystic lesions
Q41/ You need to put cannula in the great saphenous vein. Where can you find its
branch?
in front of medial malleolus
in front of lateral malleolus
behind medial malleolus
behind lateral malleolus
Answer: A
Q42/ A patient presented with hard mass on the outer upper area of the breast. which lymph node
you have to examine?
posterior axillary
anterior axillary
lateral axillary
medial axillary
Answer: B
Anterior (pectoral) group: from the lateral quadrants of the breast.
Q43/ What is the most common site of Meckels diverticulum?
lower ileum
Answer: A
Most (75%) Meckel diverticula are found within 100 cm (two feet) of the ileocaecal valve
44- What is the most common site of breast cancer?
superior and lateral
Answer: A
Approximately one half of breast cancers develop in the upper outer quadrants.
Reference: Surgical Recall, 6th edition
Q45/ Case of burn (Know how to calculate the BSA and how to determine the degree of
burn).
Parkland Formula: Ringers lactate 4 cc/kg/%BSA burned; give half in first 8 h, half in next
16. Maintenance fluids are also required if patient cannot tolerate PO hydration

smle ,2016

590

Reference: Toronto Notes


Q46/ Patient with adrenal mass on examination. What is the next step before you do FNA?
renin/aldosterone ratio
dexamethasone stress test
free metanephrines
Answer: C
Pheochromocytoma should always be suspected to avoid the risk of lethal hypertensive crises, especially
during biopsy or surgery. Plasma-free metanephrines provide the best test for excluding or confirming
pheochromocytoma.
Reference: http://www.medscape.com/viewarticle/442383_3

Q47/ A patient presented with lymph node enlargement on the horizontal line of the inguinal ligament. What is the structure that you must examine?
anal canal
gluteal folds
anterior medial leg
anterior medial thigh
Answer: A
Superficial inguinal lymph nodes: Anal canal (below pectinate line), skin below umbilicus (except popliteal territory), scrotum. Reference: FA USMLE step1

Q48/ Patient complaining of pain in the 2nd digit. Which tendon is affected?
Answer: ? The question is not clear yet!

smle ,2016

591

Q49/ Patient with abdominal pain and fever. Lab showed high amylase. What is the diagnosis?
Acute pancreatitis
Answer: A
Symptoms of acute pancreatitis include the following:

Abdominal pain (cardinal symptom): Characteristically dull, boring, and steady; usually sudden in
onset and gradually becoming more severe until reaching a constant ache; most often located in the upper abdomen and may radiate directly through to the back

Nausea and vomiting, sometimes with anorexia

Diarrhea
Patients may have a history of the following:

Recent operative or other invasive procedures

Family history of hypertriglyceridemia

Previous biliary colic and binge alcohol consumption (major causes of acute pancreatitis)
The following physical findings may be noted, varying with the severity of the disease:

Fever (76%) and tachycardia (65%); hypotension


Abdominal tenderness, muscular guarding (68%), and distention (65%); diminished or absent
bowel sounds

Jaundice (28%)

Dyspnea (10%); tachypnea; basilar rales, especially in the left lung

In severe cases, hemodynamic instability (10%) and hematemesis or melena (5%); pale, diaphoretic, and listless appearance

Occasionally, extremity muscular spasm secondary to hypocalcemia


The following uncommon physical findings are associated with severe necrotizing pancreatitis:

Cullen sign (bluish discoloration around the umbilicus resulting from hemoperitoneum)
Grey-Turner sign (reddish-brown discoloration along the flanks resulting from retroperitoneal
blood dissecting along tissue planes); more commonly, patients may have a ruddy erythema in the
flanks secondary to extravasated pancreatic exudate

Erythematous skin nodules, usually no larger than 1 cm and typically located on extensor skin surfaces; polyarthritis
http://emedicine.medscape.com/article/181364-overview
Q50/ Patient had Upper GI bleeding. What is the diagnosis?
Zollinger ellison syndrome
Answer: A

smle ,2016

592
Read about Upper GI bleeding.
http://emedicine.medscape.com/article/187857-overview

Q51/ A patient presented with RLQ pain, guarding, tenderness and positive obturator sign. What is
the most likely diagnosis?
Appendicitis.
Answer: A
Q52/ Cancer associated with Smoking?
Bladder
Colorectal
Answer: A
http://www.cancer.gov/about-cancer/causes-prevention/risk/tobacco/cessation-fact-sheet
Q53/ A patient presented with splenic injury after abdominal trauma. The surgeon decided to
embolize the splenic artery. Which of the following will be compromised?
Stomach fundus
Splenic flexure
Answer: A
Reference: https://en.wikipedia.org/wiki/Short_gastric_arteries
Q54/ A patient with a stabbed wound to the Gluteus. Examination: The patient tilt to the unaffected
side while walking. Which nerve is affected?
Femoral N
Obturator N
Superior Gluteal N
Inferior Gluteal N
Peroneal N
Answer: C
Superior gluteal nerve is a nerve that originates in the pelvis and supplies the gluteus medius, the gluteus
minimus (abductor muscles), and the tensor fasciae latae muscles. (Trendelenburg gait)

Q55/ Pelvic fracture with blood coming out from urethral meatus. What will be your initial step?
Answer: ?
Rectal and pelvic examinations are of most importance during the initial evaluation to rule out the presence of an open fracture. Blood in the vaginal vault or in the rectum should raise the level of suspicion for

smle ,2016

593
an open injury. Palpable bony spicules within the rectum or vagina may be present indicating an open injury. A high-riding prostate may also be detected on rectal examination, indicating the presence of a periurethral or periprosthetic hematoma occurring secondary to genitourinary injury >> Pelvic stabilization
for patients with pelvic fractures who are hemodynamically unstable. Reference: http://www.sportssurgerynewyork.com/articles/diagnosis-management-pelvic-fractures.pdf
In any male patient with suggestive symptoms or signs urethral injury, the diagnosis is confirmed by retrograde urethrography. This procedure should always precede catheterization.Urethral catheterization in a
male with an undetected significant urethral injury may potentiate urethral disruption (eg, convert a partial disruption to a complete disruption). Female patients require prompt cystoscopy.
Q56/ loss of sensation in medial part of thigh nerve affected.
Answer: ?
Obturator nerve supply Medial thigh, also Anterior cutaneous branches of the femoral nerve supply Anteromedial thigh

Q57/ 40 years old male patient work on the computer a lot. After that get tingles around lateral aspect of the hand. What is the most accurate test?
Tinels sign
Phalen Test
Durkan's carpal test
Compression
Answer: C
Durkan: sensitivity: 89% and specificity: 96%. It is considered by some to be a better provocative test
than tinel and phalen tests. Reference:https://books.google.com.sa/books?id=D4mQCgAAQBAJ&pg=PA110&dq=tinel+phalen+compression&hl=en&sa=X&redir_esc=y#v=onepage&q&f=false
Q58/ A patient had appendectomy. The artery that supplies the appendix is a branch from which of
the following?
Superior epigastric Artery
Inferior epigastric Artery
Superior Mesenteric Artery
Inferior Mesenteric Artery
Answer: C

smle ,2016

594
The appendicular artery (appendiceal artery) is a terminal branch of the ileocolic artery which is a branch
of Superior mesenteric artery. Reference: Wikipedia.

http://teachmeanatomy.info/abdomen/vasculature/arteries/superior-mesenteric/

Q59/ A patient had prostate CA and it was removed. Now he presented with metastasis.
What is the most probable location?
Scalp
Brain
Bone
Lung
Answer: C
Two triangle Rule that love bone. Thyroid, Breast. Kidneys and Prostate
Reference:
http://www.cancer.org/cancer/prostatecancer/detailedguide/prostate-cancertreating-treating-pain
Q60/ A patient received clindamycin before surgery. 3rd day post-op patient developed watery diarrhea. What test will you order?
Stool ova and parasite
EIA for C.difficile
Viral
Answer: B
Reference : EIA for detecting toxins A and B: This test is used in most laboratories (moderate sensitivity, 79-80%; excellent specificity, 98%)

smle ,2016

595
http://www.uptodate.com/contents/clostridium-difficile-infection-in-adults-clinical-manifestationsand-diagnosis?source=see_link
Q61/ Patient came with acute urinary retention. Urologist did catheterization for the patient. He
had BPH. Surgery was done and they injured the prostatic plexus. Whats the complication?
Stress incontinence
Urinary incontinence
Erectile Dysfunction
Answer: C
Erectile Dysfunction then Retrograde ejaculation. (Answered by a urologist).
https://en.wikipedia.org/wiki/Prostatic_plexus_(nervous)
Q62/ urge and stress incontinence treatment.

Reference: Toronto Notes.


Q63/ Patient underwent orthopedic surgery. 2 days after the surgery he started to complain of
swelling, erythema and tenderness at the site of surgery. Which organism can cause the wound infection?
a.
Clostridium perfringens
b.
Other choices included different subtypes of clostridium
Answer: A
C. perfringens is the only Clostridium species can cause wound infection (Gas gangrene; A medical emergency)
Q64/ Elderly patient is having bilateral Hydronephrosis:
BPH
Answer: A
Reference: PubMed

Q65/ Old patient with COPD, DM and now hes diagnosed to have BPH, what medication will you
use?
a.
B-Blocker
b.
Terazosin
Answer: B

smle ,2016

596
Terazosin is an alpha blocker

Q66/ A man presented with pain and mass in the flank. What is the diagnosis?
Answer: ?
DD of flank pain and mass:
Renal abscesses: patient have pain because inflammation and edema produce stretch of renal capsule.
Sometimes, a flank mass may be palpable.
Polycystic kidney disease.
Renal vein thrombosis.
Renal cell carcinoma (pain, mass and hematuria)
Reference: Medscape + RightDiagnosis
Q67/ (long scenario) lactating women 10 days after delivery complaining of fever and rigors. On examination: tender left breast and nodules in upper outer area (investigations result included). What
is the most likely diagnosis?
a.
Postpartum sepsis.
b.
Breast abscess.
c.
Inflammatory breast cancer.
Answer: B
Postpartum fever is defined as a temperature greater than 38.0C on any 2 of the first 10 days following
delivery exclusive of the first 24 hours. (which is not clearly met here).
Reference: http://emedicine.medscape.com/article/796892-overview#showall
Q68/ Patient have blunt in right 4 intercostal space. Which of the following is most likely affect affected?
a.
Upper lobe of lung
b.
Lower lobe of lung
c.
Horizontal
Answer: C
The horizontal fissure arise from the right oblique fissure and follow the fourth intercostal space from the
sternum until it meets the oblique fissure as it crosses right 5th rib. Reference: http://radiopaedia.org/articles/horizontal-fissure
Q69/ (long scenario) man with solid thyroid nodule, what is the most appropriate thing to do?
a.
Incisional biopsy.
b.
Excisional biopsy.
c.
Fine needle aspiration.
Answer: C
Fine-needle aspiration biopsy is used for definitive diagnosis. FNAB has attracted much attention in the
adult population in the evaluation of thyroid nodules.
Reference: http://emedicine.medscape.com/article/924550-workup#c5
rosses right 5th rib. Reference: http://radiopaedia.org/articles/horizontal-fissure
Q70/ Patient have appendectomy what is the most common infection will be come after operation?
a. fragilis
b.
Staph aureus
c.
Shigella
d.
Pseudomonas
Answer: A

smle ,2016

597
Enterococcus faecalis, Escherichia coli and B. fragilis are common pathogens in Surgical Site Infections
after clean contaminated surgery.
Reference:
https://books.google.com.sa/books?id=n9Y58-950vYC&pg=PA543&lpg=PA543&dq#v=onepage&q&f=false
http://www.cdc.gov/hicpac/SSI/table7-8-9-10-SSI.html
Q71/ What is the common presentation of perforated duodenal ulcer?
Answer: ?
The most characteristic symptom is the suddenness of the onset of epigastric pain. The pain rapidly becomes generalized although occasionally it moves to the right lower quadrant.
Reference: http://www.ncbi.nlm.nih.gov/books/NBK6926
Epigastric pain is the most common symptom of both gastric and duodenal ulcers. It is characterized by a
gnawing or burning sensation and occurs after mealsclassically, shortly after meals with gastric ulcer
and 2-3 hours afterward with duodenal ulcer. Food or antacids relieve the pain of duodenal ulcers but provide minimal relief of gastric ulcer pain.
Duodenal ulcer pain often awakens the patient at night.
Reference: http://emedicine.medscape.com/article/181753-clinical

Q72/ In appendectomy. Which artery you have to pay attention to during the procedure?
Answer: ?
Gonadal vessels
Iliac artery/vein
One must be cognizant of anatomical structures in the right lower quadrant, in order to avoid injuries during an appendectomy. These include the right ureter, gonadal vessels, iliac artery/vein,psoas muscle, ileum and cecum.
Reference: http://www.eaes-eur.org/getmedia/7994a09c-344d-4c5e-81d5-4b9a75b8304b/Grantcharov_Appendectomy.pdf

Q73/ During a cholecystectomy, there was an injury to the cystic artery. The surgeon applied pressure on the free margin of the lesser omentum. What is the name of the artery compressed in this
maneuver?
The right gastric artery
Coeliac
Hepatic
Splenic
Gastro-epiploic/duodenal
Answer: C

smle ,2016

598

Reference: Farquharson's Textbook of Operative


General Surgery, 10th Edition and Wikipedia.
Q74/ 17 year old complaining of abdominal pain that shifts to the right iliac fossa. what is the most
likely diagnosis?
Answer: ?Appendicitis
Features of the abdominal pain are as follows:
Typically begins as periumbilical or epigastric pain, then migrates to the RLQ
Patients usually lie down, flex their hips, and draw their knees up to reduce movements and to avoid
worsening their pain Reference: http://emedicine.medscape.com/article/773895-overview
Q75/A female patient who has high risk for breast cancer. Which gene is responsible for that?
Answer: ?
BRCA1
The family history characteristics that suggest increased risk of cancer are summarized as follows:
Two or more relatives with breast or ovarian cancer
Breast cancer occurring in an affected relative younger than 50 years
Relatives with both breast cancer and ovarian cancer
One or more relatives with two cancers (breast and ovarian cancer or 2 independent breast cancers)
Male relatives with breast cancer
BRCA1 and BRCA2 mutations
Ataxia telangiectasia heterozygotes (quadrupled risk)
Ashkenazi Jewish descent (doubled risk)
Reference: http://emedicine.medscape.com/article/1947145-overview
Q75/ 17 years old boy with acute appendicitis, appendectomy was done. If you take it under microscope, which of the following will be found ?
a.
Neutrophils in muscularis propria.
b.
Mucus filled lumen.
c.
Neoplastic tumor at the tip.
Answer: A
Microscopy demonstrates neutrophil infiltrate of the mucosal and muscularis layers extending into the lumen.
Reference: http://emedicine.medscape.com/article/773895-workup#c20

smle ,2016

599
Q76/ 7 years old boy brought by his parents, he has nausea, severe vomiting for 20 minutes and
now semi comatosed. The parents mentioned that he has same episode two weeks ago for 5 minutes
without deterioration in consciousness. On examination there is right testicular mass that does not
transilluminate with light. What is the best action to do ?
a.
Radiology.
b.
ESR.
c.
Surgical exploration.
Answer: C
The most likely diagnosis is strangulated hernia
Once the diagnosis of a hernia is made, surgical repair (a herniotomy) will be performed.
Reference: http://www.rch.org.au/kidsinfo/fact_sheets/Inguinal_hernia/

Q77/ Child was diagnosed to have meckel's diverticulae. What is the best diagnostic test?
Answer: ?
Meckels scan (technetium-99m pertechnetate scanning)
Meckel's scan has a sensitivity of 85 to 97 percent in pediatric patients
Reference: Uptodate

smle ,2016

600

Q78/ What is the cause of skin dimpling in breast cancer?


Cooper ligament
Lactiferous duct
Answer: A
Q79/ A patient presented with urethral injury (already established) with blood coming out from the
meatus, What will you do for him?
Insert suprapubic catheter.
Answer: ?

http://emedicine.medscape.com/article/451797-trea tment
This is the safest approach because it establishes urinary drainage and does not require either urethral manipulation

Q80/ A patient presented with ischemia of the gluteus area. Whats the origin of gluteal artery?
internal iliac artery
Answer: A
Q81/ What area could be affected if there was occlusion of the internal iliac artery?

smle ,2016

601
Answer: ?

Q82/ Patient presents with blunt trauma that affect the inferior mesenteric artery, Which one of the
following branches NOT affected?
Answer: ?
Inferior mesenteric artery has 3 branches:
- Left colic artery
- Sigmoidal arteries
- Superior rectal artery
Q83/ One cell type testicular mass, what is the diagnosis?
Seminoma
Answer: ?

smle ,2016

602

Most common urologic diagnosis in men <50 yr. Prevalence 2-12%

* KEEPS Klebsiella sp.


E. coli (90%), other Gram-negatives Enterococci
Proteus mirabilis, Pseudomonas
S. saprophyticus

Reference: Toronto Notes


Q84/ Prevent of hemorrhoid:
a. high fiber diets
Answer: a
The best way to prevent hemorrhoids is to keep your stools soft, so they pass easily. To prevent hemorrhoids and reduce symptoms of hemorrhoids, follow these tips:
1.
2.
3.
4.
5.

Eat high-fiber foods. Eat more fruits, vegetables and whole grains. ...
Drink plenty of fluids. ...
Consider fiber supplements. ...
Don't strain. ...
Go as soon as you feel the urge. ...

smle ,2016

603
6.
Exercise. ...
7.
Avoid long periods of sitting.
Ref : http://www.mayoclinic.org/diseases-conditions/hemorrhoids/basics/prevention/con-20029852
Q85/30 years old female dancer with breast mass that disappeared by aspiration ?
??
Q86/ which thyroid cancer have the best prognosis?
Answer: Papillary cancer
Q87/ Difficulty in initiation of urination which type of incontinence pt have
Urge incontinence
Overflow incontinence
Stress incontinence
Answer: non of the answers make sense due to the following
Stress Incontinence:
With this type, urine leaks due to weakened pelvic floor muscles and tissues. It can happen when pressure
on your bladder increases -- such as when you exercise, laugh, sneeze, or cough.
Urge Incontinence:
This is also called overactive bladder (OAB). With this type, you have an urgent need to go to the bathroom and may not get there in time.
Overflow Incontinence:
If you can't empty your bladder, you may have overflow incontinence. This means you may dribble urine.
If there was no choice for non of the above the choose overflow incontinence .
Q88/ mass in the neck in cervical area, the best initial test?
Answer: CT
Ref:
http://www.uptodate.com/contents/evaluation-of-a-neck-mass?source=outline_link&view=text&anchor=H13#H13
Q89/ kidney tumor oncogene ?
ps53
cant remember
Answer:
-MET proto oncogene responsible for hereditary papillary renal cell cancer
Ref : pubmed
http://www.ncbi.nlm.nih.gov/pmc/articles/PMC2929006/
-Cont. alteration of p3 chromosome result in alteration of either tumor suppressor gene ( VHL , TSC ) or
oncogene ( MET )
Ref : medscape
http://emedicine.medscape.com/article/281340-overview#a2
Q90/ The narrowest part of male urethra is?
A- membranous urethra.
B- penile urethra.
Answer is A
Answer : membranous urethra (1 cm long): passes through the urogenital diaphragm, surrounded by
sphincter urethrae the shortest and narrowest portion.
Reference:
http://radiopaedia.org/articles/male-urethra

smle ,2016

604

Q91/ indirect hernia related to spermatic cord ?


anterio medial
posterio medial
something another something
Answer: Indirect inguinal hernia lies within the cord, Some websites mention that the hernia is located
anterio-medial to the cord.
Answer:Complete disappearance of the breast mass after aspiration of non bloody fluid , indicate simple
cyst
References : pubmed & medscape
http://www.medscape.org/viewarticle/718139
http://www.ncbi.nlm.nih.gov/pmc/articles/PMC2855917/#!po=15.7407
Q92/ gene mutation in breast cancer?
Answer: BRCA1 & BRCA 2
Genetic Factors
Although 20-30% of women with breast cancer have at least one relative with a history of breast cancer,
only 5-10% of women with breast cancer have an identifiable hereditary predisposition. BRCA1 and
BRCA2 mutations are responsible for 3-8% of all cases of breast cancer and 15-20% of familial cases.
Rare mutations are seen in the PTEN, TP53, MLH1, MLH2, and STK11 genes
http://emedicine.medscape.com/article/1945957-overview#a5
Ref: pubmed
Q93/30 years pt with cyclic breast pain and tenderness ..pain is relieved after the cycle..
By examination there is periaerular nodular mass 3cm ..what is invx?
A.FNA
B.mammogram followed by US ( newly added )
C.u/s
D.follow her to next period CT scan ( newly added )
Answer: U/S
The first line of investigation is imaging not cytology nor tissue biopsy, So that excludes FNA.
Because the pt is young (30 yr old) U/S is more sensitive. Because of the tissue density.
Ref: Baily & Love and toronto notes
Q94/ Ulcerative colitis skin lesions :
Most frequent skin lesions associated with IBD inculde erythema nodosum and pyoderma gangrenosum
Source: uptodate
Q95/Which type of renal stones is associated with recurrent infections
Answer: struvite stones
Struvite stones account for 15% of renal calculi. They are associated with chronic urinary tract infection
(UTI) with gram-negative rods capable of splitting urea into ammonium, which combines with phosphate
and magnesium
http://emedicine.medscape.com/article/437096-overview#a5
Q96/ ttt of uncomplicated cystitis
Answer: The first-choice agents for treatment of uncomplicated acute cystitis in women include nitrofurantoin monohydrate/macrocrystals, trimethoprim-sulfamethoxazole (TMP-SMX), or fosfomycin.
Ref: Medscape + uptodate

smle ,2016

605

Q97/ scrotal pain with dysuria ttt


Answer: the diagnosis most likely epididymi-orchitis so , the ttt
Conservative management (analgesia and IV AB) unless the complications occur(abscess)
And scrotal support
pubmed
Q98/ sign of hyperthyroidism wth nodule in lateral to mid line
Answer: First rule any thyroid nodule not showing or changing hormone activity and signs it is most
likely malignant nodule
Hyperthyroidism signs and symptoms :
Common symptoms of thyrotoxicosis include the following:
Nervousness
Anxiety
Increased perspiration
Heat intolerance
Hyperactivity
Palpitations
reference : medscape
SKIN The skin is warm (and may rarely be erythematous) in hyperthyroidism due to increased blood
flow; it is also smooth because of a decrease in the keratin layer. Other changes include:
Sweating, which increases due to increased calorigenesis; this is often associated with heat intolerance
Onycholysis (loosening of the nails from the nail bed, Plummer's nails) and softening of the nails
Hyperpigmentation, which can occur in severe cases; it appears to be mediated by accelerated cortisol metabolism, leading to increased corticotropin (ACTH) secretion
Pruritus and hives, which are occasional findings, primarily in patients with Graves' hyperthyroidism
Vitiligo and alopecia areata, which can occur in association with autoimmune disorders
Thinning of the hair
EYES Stare and lid lag occur in all patients with hyperthyroidism.
CARDIOVASCULAR Patients with hyperthyroidism have an increase in cardiac output, due both to
increased peripheral oxygen needs and increased cardiac contractility. Heart rate is increased, pulse pressure is widened, and peripheral vascular resistance is decreased. High- or normal-output congestive heart
failure can occur in patients with severe hyperthyroidism. Atrial fibrillation occurs in 10 to 20 percent of
patients with hyperthyroidism
GASTROINTESTINAL

Vomiting and abdominal pain, rarely

Dysphagia due to goiter

smle ,2016

Abnormalities in liver function tests, particularly high serum alkaline phosphatase concentrations and, rarely, cholestasis

606
Reference ; up to date
http://www.uptodate.com/contents/overview-of-the-clinical-manifestations-of-hyperthyroidism-inadults?source=search_result&search=signs+of+hyperthyroidism&selectedTitle=2~150
Q99/gastrectomy which one will be defecient
Lipase
Pepsin
Trypsin
Answer: pepsin
Pepsin is secreted (in an inactive or zymogen form as pepsinogen) by the chief cells of the stomach. Pepsinogen is activated by contact with stomach acid. Although protein digestion usually begins with the actions of hydrochloric acid and pepsin, pancreatic enzymes complete the job as the food passes into the
small intestine
http://rfwdata.net/USMLEQBANK/block17questions.html#q1
Q100/painful axillary lump with tender and erythematous with black head papule and large
pores? repeated
A.Local antibiotic
B.Oral antibiotic
C.Surgery
Answer: C
It is most probably epidermoid cyst , if not infected it will resolve spontaneously but recurrence in high ,
if fluctuating I and D is indicated .
Reference ; up to date
http://www.uptodate.com/contents/overview-of-benign-lesions-of-the-skin?source=outline_link&view=text&anchor=H1101420445#H1101420445
Q101/old man complaining of inability to start urine. after voiding = rectal exam revel distended full bladder ? repeated
A.Stress
B.overflow
C.reflex
Answer: B
overflow have full bladder , not A because there no past prostate surgery .
Incomplete emptying (overflow incontinence) Incomplete emptying is a less common cause of incontinence that is due to impaired detrusor contractility and/orbladder outlet obstruction. A common presentation is nocturnal enuresis (bedwetting) due to pelvic floor relaxation that occurs at night, combined with a
very full bladder.
Reference: up to date
http://www.uptodate.com/contents/urinary-incontinence-in-men?source=search_result&search=overflow+incontinence&selectedTitle=3~31
Q102/Post appendectomy .. penicillin resistance? What antibiotic use?
Answer: Broad-spectrum gram-negative and anaerobic coverage is indicated .Penicillin-allergic patients
should avoid beta-lactamase type antibiotics and cephalosporins. Carbapenems are a good option in
these patients.
Referance: http://emedicine.medscape.com/article/773895-treatment#d10
NON PERFORATED APPENDICITIS :

smle ,2016

607
Postoperative management With both the open and laparoscopic approaches, most patients are discharged within 24 to 48 hours of surgery. Patients may be started on a clear liquid diet post-operatively
and advanced to regular diet as tolerated. Antibiotics are not required postoperatively in nonperforated
appendicitis.
PERFORATED APPENDICITIS :
Postoperative management Postoperatively, these patients often have an ileus, and diet should only be
advanced as the clinical situation warrants. Patients may be discharged once they tolerate a regular diet,
usually in five to seven days.
Referance: http://www.uptodate.com/contents/management-of-acute-appendicitis-in-adults
Q103/most ommon intracranial tumor in adult ?
A.Hemangiomblastoma
B.Ependymoma
C.Shwanoma
Answer: B
Reference: up to date
Glioblastoma multiforme (GBM) , also known as gliomas is the most common primary brain tumor in
adults. Ependymoma is a type of glioma.
http://braintumor.org/brain-tumor-information/understanding-brain-tumors/tumor-types/
Ependymomas account for less than 10 percent of tumors arising in the central nervous system (CNS) and
25 percent of primary tumors originating in the spinal cord.
(http://www.uptodate.com/contents/ependymoma?source=see_link)
Q104/during laparoscopic surgery of inguinal hernia you find artery superficial going upward ?
A.Inferior epigastric artery.
Answer: Inferior Epigastric vessels
Great care must be exercised as the dissection approaches the iliac vessels. In addition, obturator vessels
often cross the dissection planes and may need to be clipped and divided.
The inferior epigastric vessels are identified, and dissection lateral to the vessels leads to the space of
Bogros, the cord structures, and indirect hernias (see the video below). The proper plane of dissection is
between the transversalis fascia and the peritoneum. This is identified by retracting the inferior epigastric
vessels upward against the rectus muscle. A plane containing areolar tissue is identified, and this plane is
dissected toward the pelvic sidewall
Referance : medscape
http://emedicine.medscape.com/article/1534321-overview
Q105/MVA diagnosed with fracture of base of skull and injury to nerve pass through jugular foramen
Answer:cranial nerve 9th , 10th , 11th
Vernet syndrome or jugular foramen syndrome is involvement of the IX, X, and XI cranial nerves with
basal skull fracture. Patients present with difficulty in phonation and aspiration and ipsilateral motor paralysis of the vocal cord, soft palate (curtain sign), superior pharyngeal constrictor, sternocleidomastoid,
and trapezius.
Referance : http://emedicine.medscape.com/article/248108-overview#showall
Q106/ a patient has a trauma to his neck and he cant abduct his shoulder, what is the nerve that
was injured?
A.Axillary
B.long thoracic

smle ,2016

608
Answer: B ??
Abduction is carried out by the deltoid(Axillary) and the supraspinatus in the first 90 degrees. From 90180 degrees it is the trapezius and the serratus anterior(long thoracic).
Referance: https://en.wikipedia.org/wiki/Shoulder_joint
Q107/a patient weighs 70 kg had a circumferential 3rd degree burn in his trunck, how much lactate
ringer you will give him in the first 8 hours?
A.12.5 L
B.10.5
C.8.5
D.6.5
Answer: D
Resuscitation fluids - 3-4 mL Ringer lactate X weight (kg) X %TBSA burned (second-degree and third
degree); half administered over the first 8 hours (from time of injury), remaining half administered over
the next 16 hours.
Referance: http://reference.medscape.com/article/934173-treatment
Q108/patient post lap choly develope sob cough respiratory distress What's dx
A.PE
B.Stroke
C.Pneumonia
D. Atelectasis
Answer:D
Atelectasis is a common pulmonary complication in patients following thoracic and upper abdominal procedures. General anesthesia and surgical manipulation lead to atelectasis by causing diaphragmatic dysfunction and diminished surfactant activity.
PE is one of the correct options if the surgery is prolonged like in joint replacements (so unlikely after lap
chole) and usually present 5-7 days after surgery, unlike atelectasis which develop one day after surgery.
Reference:
http://emedicine.medscape.com/article/296468-overview.
Q109/Pt with carpal tunnel what is the diagnostic test:
A.mri
B.ct
C.x-ray
D.nerve conduction
Answer: D
Electrophysiological tests (nerve conductive study). Electrical testing of median nerve function is often
done to help confirm the diagnosis.
Reference: http://orthoinfo.aaos.org/topic.cfm?topic=a00005
Q110/multiple myeloma in the spine
(Histopath report)
Answer:
Dx of multiple myeloma requires the following:
* bone marrow aspirate or biopsy showing that at least 10% of the cells are plasma cells or the presence
of plasma cell tumor ( plasmacyte ) plus at least one of the following
- evidence of the damage to the body by plasma cells ; bone damage , kidney failure , anemia or high calcium .

smle ,2016

609
- defection of one of the following ; > 60% plasma cells in bone marrow, free light chain ration of 100 or
more, MRI showing involvement of more than one lesion in the bone or bone marrow
Reference: http://www.uptodate.com/contents/multiple-myeloma-symptoms-diagnosis-and-staging-beyond-the-basics?source=outline_link&view=text&anchor=H12#H12
Answer:
CBC
to determine if the patient has anemia, thrombocytopenia, or leukopenia. The CBC and differential may
show pancytopenia. The reticulocyte count is typically low. Peripheral blood smears may show rouleau
formation.The erythrocyte sedimentation rate (ESR) is typically increased. Coagulation studies may yield
abnormal results.
Histologic Findings
Plasma cells are 2-3 times larger than typical lymphocytes; they have eccentric nuclei that are smooth
(round or oval) in contour with clumped chromatin and have a perinuclear halo or pale zone The cytoplasm is basophilic.
Many MM cells have characteristic, but not diagnostic, cytoplasmic inclusions, usually containing immunoglobulin. The variants include Mott cells, Russell bodies, grape cells, and morula cells. Bone marrow
examination reveals plasma cell infiltration, often in sheets or clumps . This infiltration is different from
the lymphoplasmacytic infiltration observed in patients with Waldenstrom macroglobulinemia. Analysis
of bone biopsy specimens may reveal plasmacytic, mixed cellular, or plasmablastic histologic findings.
Reference http://emedicine.medscape.com/article/204369-workup#c17
Q111/honeycoomb cystitis which organism
Answer:
I think they mean Honeymoon cystitis*
Honeymoon cystitis (or "honeymoon disease") is cystitis caused by sexual activity.
The microbial spectrum of uncomplicated cystitis and pyelonephritis in women consists mainly of Escherichia coli (75 to 95 percent),
Reference: http://www.uptodate.com/contents/acute-uncomplicated-cystitis-and-pyelonephritis-inwomen?source=search_result&search=cystitis&selectedTitle=1~80
but similar question in USMLE mentioned "gram +ve " in this case Staphylococcus saprophyticus is common when acquiring a first or new sexual partner
Reference First Aid Q&A for the USMLE Step 2 CK, Second Edition
Q112/Rx uncomplicated 5 yrs cystitis
A. IM cefttiaxon
B. IV copra
C. Oral.
Answer: C
Children with cystitis usually do not require special medical care other than appropriate antibiotic therapy. A 4-day course of an oral antibiotic agent is recommended for the treatment of cystitis. Antibiotic
agents used include Sulfamethoxazole and trimethoprim (SMZ-TMP), Amoxicillin and clavulanic acid,
Cephalexin, Cefixime, Cefpodoxime, Nitrofurantoin.
Reference: http://emedicine.medscape.com/article/969643-treatment#d12
Q113/Injury to jugular foramen which structure will not be affected all of option are muscle :
A.Strenochleinomastoid
B.Sphenioplatine and
C.2 more option

smle ,2016

610
Answer:
sternocleidomastoid is least muscle affected during jagular foramen injury ,
Jugular
foramen
muscle
passing:
sternocleidomastoid
sphenopalatine
Digastric
muscle
-If one of these choose SCM , if non of these choose it
Reference: by Neurosurgery consultant
A.Strenochlinomastoid
The accessory nerve is a cranial nerve that controls the sternocleidomastoid pass threw JF
B.Sphenioplatine
Sphenopalatine artery, ganglion and nerves not related to JF
C.2 more option
NOTES:
1- NO MUSCLE called ' Sphenioplatine
2- Cranial nerves IX, X, and XI and the internal jugular vein pass through the jugular foramen.
Reference http://emedicine.medscape.com/article/1298684-overview#a 8
Q114/Injury to temporal area and superficial temporal artery bleed which layer of the skull where
vessels are ?
peri cranial
epi cranial aponeurosis
Answer: B
Reference: http://emedicine.medscape.com/article/881374-overview#a9
Q115/ surgeon take graft from rectus muscle which artery should be dissected
A.superior epigastric
B.inferior epigastric
C.superficial epigastric
Answer: B
Reference: http://www.ncbi.nlm.nih.gov/pmc/articles/PMC3365455/
Other Reference http://www.atlasofpelvicsurgery.com/10MalignantDisease/14TransverseRectusAbdominisMyocutaneousFlapandVerticalRectusAbdominisMyocutaneousFlap/cha10sec14.html
Q116/what is the most common type of gallstone
A.cholesterol screening
B.bile pigmented
Answer: A
Reference: http://www.mayoclinic.org/diseases-conditions/gallstones/basics/causes/con-20020461
Q117/bladder ca surgical removed and the area of cancer now clear and no invasion of the muscle
A. follow up with cystoscopy and biopsy
B. intravesical chemo..
Answer: B intravesical chemotherapy
Reference : uptodate + toronto notes
http://www.uptodate.com/contents/overview-of-the-initial-approach-and-management-of-urothelial-bladder-cancer?source=outline_link&view=text&anchor=H3#H3
Q118/lap-cholecystectomy injured cystic artery, surgeon put pressure with the free margin of lesser
omentum,which artery is compressed?
A.hepatic

smle ,2016

611
B.coeliac
C.splenic
D.gastro-epiploic/duodenal ?
Answer: hepatic
Q119/ What is the drug that can make a cholecystitis pain worse?
Acetaminophen
Morphine
Meperidine
Answer; B
Reference: http://www.meb.uni-bonn.de/dtc/primsurg/docbook/html/x3982.html
Q120/Tumor in rt testis, Hcg high, most likely?
A.Seminoma
B. Lydig cell
D.Teratoma
Answer: A
Reference: http://www.cancer.net/research-and-advocacy/asco-care-and-treatment-recommendationspatients/tumor-markers-testicular-cancer-and-extragonadal-germ-cell-tumors-teenage-boys-and-men
Q121/Ligament in inguinal canal?
A.Broad
B.Round
Answer: b
Q122/Ascites Bilateral ovarian mass, what is the most likely tumor?
Answer:

Reference: uptodate
Q123/Single thyroid nodule iodine sensitive. Best management?
A.Lobectomy
B.Radio iodine therapy
C.Anti thyroid drug
Answer:

smle ,2016

612

Reference :american family physician


Q124/Perianal itching. Most likely?
A.Perianal abscess
B.Hemorrhoid
Answer:B Hemorrhoids, which cause painful swelling of blood vessels in the anal area, can cause itching.
References:
http://www.emedicinehealth.com/script/main/mobileart-emh.asp?articlekey=58920&page=2
http://www.emedicinehealth.com/script/main/mobileart-emh.asp?articlekey=58920&page=2
Q125/Pt indwelling catheter having stone most common ?
A.Ca oxalate
B.Ca phosphate
C.Uric acid
Answer
Struvite stones account for 15% of renal calculi. They are associated with chronic urinary tract infection
(UTI) with gram-negative rods capable of splitting urea into ammonium, which combines with phosphate
and magnesium
Reference: http://emedicine.medscape.com/article/437096-overview#a5
Q126/Pt co of infected wound ulcer ,intact pulse ,no improve by AB & pt known uncontrolled DM2
Amputation
A.AB
B.surgical debridement
Answer: B

smle ,2016

613
A good clinical response for mild to moderate infections can be expected in 80%90% of appropriately
treated patients [10, 50] and, for deeper or more extensive infections, in 50%60% [64, 86]. When infection involves deep soft-tissue structures or bone, more thorough debridement is usually needed. Bone resections or partial amputations are required in about two-thirds of this patient group. Most of these amputations can be foot sparing, and long-term control of infection is achieved in >80% of cases. Infection recurs in 20%30% of patients, many of whom have underlying osteomyelitis. Factors that predict healing
include the absence of exposed bone, a palpable popliteal pulse, toe pressure of >45 mm Hg or an ankle
pressure of >80 mm Hg, and a peripheral WBC count of <12,000/mm3 [19]. The presence of edema or
atherosclerotic cardiovascular disease increases the likelihood of amputation. Amputation may be more
often required for patients with combined soft-tissue and bone infection than for patients with either type
of infection alone [86]. Patients who have had one infection are at substantial risk of having another
within a few years; thus, educating them about prevention techniques and about prompt consultation
when foot problems occur is critical.
Reference: http://cid.oxfordjournals.org/content/39/Supplement_2/S104.full
Um-alqura 3rd ed. :
Q127/

Q128/

Q129/female patient with high prolactin What to exclude


A.thyroid disease
B.pituitary tumors
Answer:B
most common cause of female hyperprolactinemia is pitutary tumor (microprolactinoma )
Reference : up to date
http://www.uptodate.com/contents/causes-of-hyperprolactinemia?source=outline_link&view=text&anchor=H1#H1
Q130/patient with BPH+HTN what TTT?
Answer: alpha blocker
Reference : up to date
http://www.uptodate.com/contents/image?imageKey=NEPH%2F63628&topicKey=NEPH%2F3869&source=see_link
Q131/case head trauma on parietal lobe subdural hematoma which artery is injured ?
A.superficial temporal .

smle ,2016

614
B.mid cerebral
D.Rt.cerebral .......
Answer: b
The cortical branches of the MCA supply the lateral surface of the hemisphere, except for the medial part
of the frontal and the parietal lobe (anterior cerebral artery), and the inferior part of the temporal lobe
(posterior cerebral artery)
http://www.radiologyassistant.nl/en/p484b8328cb6b2/brain-ischemia-vascular-territories.html
Q132/patient presented with appendicitis appendectomy done and send to pathology what you expect to see :
A.neutrophil in wall .
B.distended lumen with mucus ...........
C.renal colic was easy
D.splenctomy what vaccin he should took .
Answer: A
Neutrophils extend into and through the wall of the appendix in a case of acute appendicitis. Clinically,
the patient often presents with right lower quadrant abdominal pain. Rebound tenderness of the right
lower quadrant is often noted on physical examination, as well as positive obturator or psoas sign. An elevated WBC count is usually present.
http://library.med.utah.edu/WebPath/GIHTML/GI058.html
Q133/Trauma to shoulder PT cant raise hand the injury is in:
A.Posterior cord of biracial plexus
B.Medial cord
C.Lateral cord
Answer: A
The axillary nerve is a branch of the posterior cord of the brachial plexus
http://emedicine.medscape.com/article/1877731-overview#a2
Q134/gene mutation of suprarenal tumor?
The Li-Fraumeni syndrome is caused by inherited mutations that inactivate the p53 tumor suppressor
gene. This syndrome causes few cases of adrenal cancer in adults (1 of every 20), but is often the cause of
adrenal cancer in children. In fact, about 8 of every 10 cases of adrenal cancer in children are caused by
Li-Fraumeni syndrome. Many other adrenal cancers have also been found to have abnormal p53 genes
that were acquired after birth (not inherited).
http://www.cancer.org/cancer/adrenalcorticalcancer/detailedguide/adrenal-cortical-cancer-what-causes
Mutations in the VHL gene cause von Hippel-Lindau syndrome.
http://ghr.nlm.nih.gov/condition/von-hippel-lindau-syndrome
many different syndromes
Q135/Patient with hematuria diagnosed bladder cancer did resection what is the next step ?
a-BCG intrbladder
b-Other choice
Answer: mitomycin-C
Within the first 24 hours, a single intravesical instillation of mitomycin-C (40 mg in 20 mL of saline) has
been shown to reduce the frequency of tumor recurrence.
Reference: http://emedicine.medscape.com/article/1951622-overview#a4

smle ,2016

615
Q136/ Patient diagnose with renal cell carcinoma which gene do u think have mutation ?
A-PKD
B-PKHD-1
C-VHL
Answer: C
Referance : http://emedicine.medscape.com/article/281340-overview#a3
Q137/Case Symptoms of cholecystitis what best initial modality ?
a-U/S
Answer: A
Sonography is the preferred initial imaging test for the diagnosis of acute cholecystitis, and scintigraphy is
the preferred alternative.
CT is a secondary imaging test that can identify extrabiliary disorders and complications of acute cholecystitis, such as gangrene, gas formation, and perforation.
Reference:
http://emedicine.medscape.com/article/171886-workup#c7
Q138/Eldary Flank pain hematuria and mass what best imaging?
Answer: CT with contrast.
the pt. present with classical triad of renal tumor (hematuria, flank pain and flank mass)
References:
http://emedicine.medscape.com/article/281340-clinical
Contrast-enhanced CT scanning has become the imaging procedure of choice for diagnosis and staging of
renal cell cancer and has virtually replaced excretory urography and renal ultrasonography. http://emedicine.medscape.com/article/281340-workup

smle ,2016

616
Q139/pt noticed lump for three months the mass
freely mobile no discharge not related to menstrual?
a-cystic
b-fibroadenoma
c-ductal papilloma
d-something
Answer:b
Reference: http://www.mayoclinic.org/diseases-conditions/fibroadenoma/basics/definition/con-20032223
Added Qs 7th update
Q140/anterior abdominal stab wound omuntam come through the wound ?
A.Fast
B.Ct
C.Exploratory laparotomy
Answer: C
Exploratory laparotomy is the answer , conservative treatment is sometimes used in selective patients
Reference: http://www.ncbi.nlm.nih.gov/pmc/articles/PMC4379793/
Q141/old patient presented with congested neck veins bilateral upper limb swelling tumor in the
right upper lung which part it's compressing ?
A.superior mediastinal
B.anterior mediastinal
C.posterior mediastinal
Answer: A
This patient has superior vena cava (SVC) syndrome due to lung mass compression
The SVC is located in the anterior right superior mediastinum
Reference: http://www.ncbi.nlm.nih.gov/pmc/articles/PMC4093359/
Q142/best investigation of ovarian cancer:
a-U/s
b-CA125
c-?
d-?
Answer: Uptodate : -the only way to diagnose ovarian cancer is with exploratory operation.
-Intra operative biopsy is not usually done , biopsy may spread cancer cells.
-Ca125 is for monitoring ( pre and post operative ) - it is not diagnostic and not specific.
-Imaging like u/s is recommended initially , but can not be diagnostic .
References:http://www.uptodate.com/contents/ovarian-cancer-diagnosis-and-staging-beyond-the-basics?source=outline_link&view=text&anchor=H4#H4
For screening :Check the NICE guide lines
http://www.nice.org.uk/guidance/cg122/resources/ovarian-cancer-recognition-and-initial-management35109446543557
Q143/Patient history of trauma and right femur fracture with blood loss which is accepted change ?
a-Decrease
coronary blood flow
b-Decrease
venous capacity
Answer: The cardiovascular system initially responds to hypovolemic shock by increasing the heart rate,

smle ,2016

617
increasing myocardial contractility, and constricting peripheral blood vessels. This response occurs secondary to an increased release of norepinephrine and decreased baseline vagal tone (regulated by the baroreceptors in the carotid arch, aortic arch, left atrium, and pulmonary vessels).
Q144/Patient did IVP shows filling defect then do us shows hypoecnecnty ?
a-Uric acid stone
b-Papillary
Answer: ??
Q145/Pt with high aptt what mechanism of action of that drug?
a-" unfractinated heparin"
b-Antithrombin
Answer: heparin act indirectly by binding to antithrombin rather than binding directly to coagulation factors.
Reference: http://www.uptodate.com/contents/therapeutic-use-of-unfractionated-heparin-and-low-molecular-weight-heparin?source=outline_link&view=text&anchor=H3#H3
Q146/bronchial cancer mets to sypathatic plexus what the sign ?
a-ptosis
b-dilated pupil
Answer : the answer is horners syndrome (ptosis ,anhydrosis,miosis)
-it results from an interruption of the sympathetic nerve supply to the eye and is characterized by the classic triad of miosis (ie, constricted pupil), partial ptosis, and loss of hemifacial sweating (ie, anhidrosis).
-Horner syndrome can be congenital, acquired, or purely hereditary (autosomal dominant). The interruption of the sympathetic fibers may occur centrally (ie, between the hypothalamus and the fibers point of
exit from the spinal cord [C8 to T2]) or peripherally (ie, in cervical sympathetic chain, at the superior cervical ganglion, or along the carotid artery).
Reference: http://emedicine.medscape.com/article/1220091-overview#a3
Q147/pt noticed lump for three mos the mass freely mobile no discharge not related to menstrual?
a-cystic
b-fibroadenoma
c-ductal papilloma
d-something
Answer:b
Reference: http://www.mayoclinic.org/diseases-conditions/fibroadenoma/basics/definition/con-20032223
Q148/read about lower limb dermatom20-knee dermatom?
a-L1-L2
b-L3-L4
c-L4-L5
d-L5-S1
Answer : b

smle ,2016

618

Q149/ 3 years old boy with UTI,, what youll do before starting treatment
-US
-culture urethra !!
-p.s. no assenting urethrogram within the answers
Answer : (the answer should be : send urine culture)
-Urine culture should be done beside urine analysis , but do not delay antibiotic.
- renal ultrasonography should be considered for any child with a first febrile UTI in whom good followup cannot be ensured.
- blood cultures are not recommended .
Source :
http://emedicine.medscape.com/article/969643-workup
Toronto note :
U/s in recommended for all febrile "infants" with UTI
uptodate : US is done if :
child with recurrent UTI at any age
children younger than 2 yrs with a first febrile UTI
children who dont respond as expected to appropreate ttt .
children of any age with with a UTI who have family history of renal or urologic disease , poor growth or
HTN.
Q150/ Diabetic patient went for surgery; he was given insulin and dextrose. Then developed neurological symptoms (Low Na). What is the mechanism?
-Water overload.
-SIADH.
Answer: incomplete Q the two pictures explain the possible answers

smle ,2016

619
Reference: pics and toronto note
Q152/Most common neurosurgical tumor?
Astrocytoma.
Medullablastoma.
Answer:

Most common malignant tumor in adult is Gliomas (principally astrocytoma) account for approximately 30 percent .
http://www.cancerresearchuk.org/about-cancer/type/brain-tumour/about/types-of-primary-brain-tumours
and Medullablastoma in children .
http://www.hopkinsmedicine.org/neurology_neurosurgery/centers_clinics/pediatric_neurosurgery/conditions/pediatric_brain_tumors.html

Most common primary tumor in the brain is meningioma http://www.brainsciencefoundation.org/bUnderstandbPrimaryBrainTumors/Meningioma/tabid/186/Default.aspx

in adults, metastatic tumors to the brain are actually more common than primary brain tumors.
http://www.cancer.org/cancer/braincnstumorsinadults/detailedguide/brain-and-spinal-cord-tumors-inadults-types-of-brain-tumors
Gliomas (principally astrocytoma) account for approximately 30 percent and meningioma 35 to 40 per-

cent of symptomatic primary brain tumors.


Reference : pic and uptodate

Q153/ 60 y.o smoker with lung cancer. When the tumor compresses the sympathetic innervation, it
will lead to ?
anhidrosis
Ptosis
Hydrosis
Myadrasis
Answer : both 1 and 2 are correct
- Pancoast tumors leads to compression of sympathetic ganglion causing Horner's syndrome.
- Horner's syndrome presents with : miosis(constriction of the pupils), anhidrosis (lack of sweating), ptosis (drooping of the eyelid) and enophthalmos (sunken eyeball).
Q154/ 47 y/o female, her only child was born when she was 35 y, now she develop epithelial ovarian
cancer, no BARCA1 mutation, what will you tell her daughter about her risk to develop cancer?
1.Family history has no risk
2.OCP has protective effect
3.Barrier contraceptive has protective effect

smle ,2016

620
Answer: 2
protective factors for ovarian, fallopian tube:
Oral contraceptives
Tubal ligation
Breastfeeding
Risk-reducing salpingo-oophorectomy
Reference : uptodate
Q155/55 y/o male present with erectile dysfunction, he mention that only worse with stress, medically free, not on any medication, what is the best action?
1.Refer to endocrine
2.refer to urology
3.advice relaxation strategies
Answer :
In Om alqora : 3
This article is discussing the management of erectile dysfunction in general:
http://www.journalofandrologicalsciences.eu/media/04_Gentile.pdf
I didn"t find a different answer but this article is discussing erectile dysfunction from all prospective
Q156/ Pheochromocytoma, associated with which neoplasm?
Insulinoma
Thyroid cancer
answer: 2
Pheochromocytoma is associated with Medullary thyroid carcinoma , hyperparathyrodism .
Pheochromocytoma is a type of MEN IIA ( multiple endocrine neoplasia )
Reference: Step-up medicine P 185 .

Q157/ man did vasectomy, change his mind want to reproduce again, they found antisperm antibodies what is the cause ?
antigen release or something like that?
cross reactivity with viral infection
inappropriate response of MHC II to antigen presenting cell
answer: 2
I searched for answer but I didnt find something reliable I found this it may help http://www.newyorkfertilityservices.com/causes-of-infertility/anti-sperm-antibodies/

Q158/ Case Symptoms of cholecystitis what best intial modelity ?


1.U/S
answer: 1
Ultrasound (US) is the preferred initial modality in the investigation of right upper quadrant pain.
Q159/ 60 years old male presented with left flank pain, weight loss and hematurea, O/E palpable
firm mass in the lumbar area, BP 150/90
What is the best Initial investigation?
1.ct scan
2.mri

smle ,2016

621
3.ultrasound
4.radionuclide scan
answer: 1

Refrenc : Kaplan USMLE Step 2 CK Surgery Lecture Notes, (2014)


http://radiopaedia.org/articles/renal-cell-carcinoma-1
the Washington manual of Surgery: urologic surgery page 827

Q160/ Chronic alcoholic wake up at 3 am with abdominal pain after a little while the pain generalized all over the abdomen and is severely exacerbated with movement:
1.Peptic ulcer perforation
2.Choycystitis
3. DiverticulaTitis
4. Pancreatitis
answer : Pancreatitis
-Alcohol do NOT cause ulcers. They delay the healing of ulcers
-Alcohol is a major cause of acute and chronic pancreatitis
Q161/ In thyroidectomy which nerve injury lead to horseness of voice?
1.Recurrent laryngeal
2.Right vagus
Answer :1
recurrent laryngeal nerve: Unilateral damage results in a hoarse voice. Bilateral damage presents as laryngeal obstruction after surgery and can be a surgical emergency: an emergency tracheostomy may be
needed.
Q162/ Ureteral stone what is the rdiological study to detect it?
1.Abd.ultrasoubd
2.Ct with contrast
3.Ct without contrast
(There was No xray in options)
answer: 3 by radiology Consultant
Medscape : Acute renal colic with resultant flank pain is a common and sometimes complex clinical
problem. Whereas noncontrast abdominopelvic computed tomography (CT) scans have become the imaging modality of choice
Q163/ most reliable screening for prostate cancer is:
1.PR examination

smle ,2016

622
answer : Prostate specific antigen (PSA)
DRE is less effective compared to PSA
Reference:
http://www.uptodate.com/contents/prostate-cancer-screening-beyond-the-basics
Q164/ patient lost sensation at the level of umbilicus after MVA . the lesion will be at the level of
1.T4
2.T10
3.T11
4.T 12
answer : T10

Q165/ man came with loose stools , history of loose stools before ,, Its watery like with mucous not
containing blood ( forget the other details ) what is you diagnosis:
1. IBS
2. chrons disease
3. ulcerative colitis
Answer : 1
Because IBS ( irritable bowel syndrome ) characterized by change in stool frequency which we have in
this Q ( loose stool then watery like ) .
Ulcerative colitis (US) : bloody stool
Crohn's disease : diarrhea that may or may not be bloody .
So we need more details in this Q .
Reference: Usmle step1 P355-356 (2012)
Q166/pt came with history for alternative bowle habits some time loose and other day constipation
for weeks your diagnosis :
1.IBS
2.crohns disease
3.ulcerative colitis
answer is: 1
Reference:http://www.mayoclinic.org/diseases-conditions/irritable-bowel-syndrome/basics/tests-diagnosis/con-20024578

smle ,2016

623

Q167/ Patient came with history of upper abdominal pain the investigation show high amylase your
diagnosis will be:
1. acute pancreatitis.
2.gastric ulcer.
3.acute cholecystitis.
answer is: 1
Reference: http://www.healthline.com/health/amylase-blood
Q168/which of the following has strong association for colorectal cancer:
1.familial polyposis
2.ulcerative colitis
3.chrons disease
answer is:1
Reference: https://ghr.nlm.nih.gov/condition/familial-adenomatous-polyposis
Q169/ pt came with Lt flank pain radiated to groin no fever no change in urine color or amount no
sign other than tenderness this pain most likely:
1. uretric stone
2. renal colic
3. diverticulitis
answer ; 2
The site of obstruction determines the location of pain. Upper ureteral or renal pelvic lesions lead to flank
pain or tenderness, whereas lower ureteral obstruction causes pain that may radiate to the ipsilateral testicle or labia.
Costovertebral angle tenderness on the affected side is common.
reference ; emedicine
170/patent came from RTA , in preparing of setting of intubation patient became desating more
and ambobag not maintaing saturation which should be done ?
1. more jaw thrust
2. more head tilt
3. precede immediate for intubation
answer ; if the patient's oxygenation cannot be maintained, immediate rescue by (usually) cricothyrotomy
is necessary.
reference : uptodate
http://www.uptodate.com/contents/the-failed-airway-in-adults?source=outline_link&view=text&anchor=H351136572#H351136572
171/When you are going to intubate unconscious male there were difficult ventilation with bagmask what to do:
1. procced to intubation
2. apply cricoid pressure
3. head tilt
answer ;
mask seal Mask seal requires reasonably normal anatomy, absence of facial hair, lack of interfering substances, such as excessive vomitus or bleeding, and the ability to apply pressure to the face with the mask.

smle ,2016

624
reference : uptodate
http://www.uptodate.com/contents/the-difficult-airway-in-adults?source=outline_link&view=text&anchor=H4#H4
172/ Pt with past hx of hodgikon lymphoma .. But cured completely .. Presented with back pain ...
Examination and evaluation show paraspinus Edema and fluid collection -ve burecella titer and
tuberclin test ,, what the cause ?
1.burecellosis
2.breast cancer
3.recurrent hodgikon lymphoma
answer: 3
http://www.ncbi.nlm.nih.gov/pmc/articles/PMC3892516/
Q173/ 18-year-old healthy male was playing baseball and suddenly he felt abdominal pain. On examination he has para-umbilical mass. His vital signs are: BP 100/76, RR 30, HR 100, O2 sat is
95% with 100% oxygen mask. What is your next step in the management?
1.Abdominal US.
2.CT scan.
3.Erect Chest x-ray.
4.Reassure and send home.
Answer : 3
Intestinal obstruction due to hernia ,Erect chest xray looking for air fluid level.
Source : Toronto note
Q174/ Patient is having infertility came to infertility clinic. He is complaining of testicular pain. On
examination the testicle is swelled like a bag of worms. What is the diagnosis?
1.Varicocele.
2.Spermatocele.
3.Hydrocele.
4.Testicular cancer.
Answer: Varicocele
Palpating a varicocele can be likened to feeling a bag of worms https://en.wikipedia.org/wiki/Varicocele
Q175/Patient is having unilateral testicular pain and fever. What is the diagnosis?
1.Epidedmo-orchitis.
2.Prostatitis.
3.Testicular torsion.
Answer: 1. Epidedmo-orchitis
Acute epididymo-orchitis is a clinical syndrome consisting of pain, swelling and inflammation of the epididymis, with or without inflammation of the testes. The most common route of infection is local extension and is mainly due to infections spreading from the urethra (sexually transmitted infections (STIs)) or
from the bladder.[1] Orchitis (infection limited to the testis) is much less common. Chronic epididymitis
refers to epididymal pain and inflammation (usually without scrotal swelling) that lasts for more than six
months.
Q176/ Patient after RTA had spleen injury, he underwent surgical exploration and splenectomy.
Which of the following should be given?
1. Some kind of antibiotic.
2. Meningiococcal vaccine.
Answer: Meningiococcal vaccine

smle ,2016

625
Reference: uptodate
http://www.uptodate.com/contents/prevention-of-sepsis-in-the-asplenic-patient?source=outline_link&view=text&anchor=H16#H16
Q177/Rx of patient with lung ca stage IIIb came with sudden lower back pain?
1.MRI only
2.MRI with steroid
3.radiotherapy
Answer: MRI only
Reference: uptodate
http://www.uptodate.com/contents/overview-of-the-initial-evaluation-diagnosis-and-staging-of-patientswith-suspected-lung-cancer?source=outline_link&view=text&anchor=H57205858#H57205858
Q178/case of burn, how much fluids you will give in the 1st 8 hours?
Answer: of fluids
Parkland formula: 4 ml * kg * TBSA%
Reference: toronto notes

Q179/best screening method for prostate cancer?


1.digital rectal examination
2.cytology for prostate cancer
3.specific prostate antigen
answer: 3
Reference : http://www.uptodate.com/contents/prostate-cancer-screening-beyond-the-basics

Q180/Theoretically which of the following cancer will prevented by vaccination?


1.ALL
2.CML

smle ,2016

626
3.adult T cell leukemia
4.mycosis fungoides?
Answer: 3
Reference : http://www.intechopen.com/books/t-cell-leukemia-characteristics-treatment-and-prevention/prevention-of-human-t-cell-lymphotropic-virus-infection-and-adult-t-cell-leukemia
Q181- old man complaining of in ability to start urine after voiding = rectal exam revel distended
full bladder ?
1. Stress
2.overflow
3. reflex
answer: 2
Q182 - surgeon take graft from rectus muscle which artery should be dissected
A.
.superior epigastric
B.
inferior epigastric
C.
.superficial epigastric
answer : superior epigastric .. reviewed by my consultant
Q183-pt presented to you with Rt scrotal swelling , he said that it is increasing in size day after day ,
on examination , it is not transilluminate , what will you do ?
Reassure him and get surgical opinion
Answer:
not translucent could be Hemorrhage hydrocele further us or Need to evacuate Depends on other Q's But
most probable A Since no trauma hydrocele.
Source :baily and love ,

Q184-Pt complain of lower back pain after lifting heavy objects !


answer:
The vast majority of people with low back pain improve within 4 to 6 weeks without treatment or with
simple measures that can be performed at home.
Patients who are treated for acute back pain with bed rest have more pain and slower recovery than ambulatory patients.
Therapy should focus on temporary symptomatic relief, to maximize patient comfort. We suggest a trial
of short-term treatment with either an NSAID or acetaminophen .
source:up to date http://www.uptodate.com/contents/treatment-of-acute-low-back-pain?source=outline_link&view=text&anchor=H25#H25
Q185-which indicate how long open wound ..in inflammatory process ?
1.coverd by eshar
2.covered by epithelium
3.neovascularization
Answer:2
sorce : up to date
http://www.uptodate.com/contents/wound-healing-and-risk-factors-for-non-healing?source=outline_link&view=text&anchor=H1070222553#H1070222553
Q186-RTA, splenic injury type 1, Rx?

smle ,2016

627
1.Laparotomy
2.Diagnostic Laparoscopy with splenectomy
3.Conservative
answer: 3
Hemodynamically stable patients with low-grade (I to III) blunt or penetrating [32] splenic injuries without any evidence for other intra-abdominal injuries, active contrast extravasation, or a blush on CT, may
be initially observed safely .
Hemodynamically unstable Based upon ATLS principles, the hemodynamically unstable trauma patient
with a positive FAST scan or DPA/DPL requires emergent abdominal exploration to determine the source
of intraperitoneal hemorrhage .
sorce : up to date + first aid
http://www.uptodate.com/contents/management-of-splenic-injury-in-the-adult-traumapatient?source=outline_link&view=text&anchor=H9#H9
Q187-Patient has deep ulcer with erythema on the sole of the foot, he is a known case of uncontrolled DM. what is the treatment?
1.Give oral antibiotics
2.Admit the patient for debridement and deep culture
3.Perform superficial culture
Answer: 2
Adequate debridement, proper local wound care (debridement and dressings), redistribution of pressure
on the ulcer by mechanical off-loading, and control of infection and ischemia (when present) are important components of treatment for all ulcers, regardless of stage and depth.
For most patients with diabetic foot ulcers, we suggest surgical (sharp) debridement rather than another
method .
up to date
http://www.uptodate.com/contents/management-of-diabetic-foot-ulcers?source=outline_link&view=text&anchor=H28#H28
Q188-abdominal radiological investigation showed the inner and outer surface of intestine , what
the pt has .
1.Peroration
2.Obstruction
answer:2?
Proximal bowel dilation with distal bowel collapse Small bowel obstruction can be diagnosed if the
more proximal small bowel is dilated more than 2.5 cm (outer wall to outer wall) and the more distal
small bowel is not dilated .
up to date
http://www.uptodate.com/contents/epidemiology-clinical-features-and-diagnosis-of-mechanical-smallbowel-obstruction-in-adults?source=outline_link&view=text&anchor=H2972181#H2972181
Q189- If there was inferior mesenteric artery thrombosis. Which artery will not be affected!
1.descending colon *IMA
2.sigmoid *IMA
3.splenic*IMA
4.cecum *SMA

Answer: D cecum

smle ,2016

628
Branches of IMA :
left colic artery Splenic
two-to-four sigmoid arteries
superior rectal artery (terminal branch)

Source : http://radiopaedia.org/articles/inferior-mesenteric-artery

Q190- Bilateral breast cancer with :


1.paget disease *uni
2.lobular carcinoma * bilatral
3.mucinus carcinoma *uni
4.ductal carcinoma*uni
answer: 2 .lobular ca
Q191- Case of RLQ pain and mass , what is Tx :
1.conservative .
2.surgery .
3.antibiotics .
answer: 3. antibiotics. Inflamed Appendicular mass is either a phlegmon or an abscess , both are treated
non surgically by antibiotics ( in some abscess cases by US guided drainage ).
Q192- which renal stone associated with infection and alkaline urine :
1.calcium oxalate
2.uric acid
3.struvite
4.cystine
answer: 3 .struvite
source:First aid

smle ,2016

629

Q193- patient MVA and come with fracture of femur , tibia and fibula what is your action :
1.refer to orthopedic
answer: supportive then refer
http://www.ncbi.nlm.nih.gov/pmc/articles/PMC3524792/
Q194- patient did gastrectomy and need to take vit B12 for life ,, which cells are responsible :
1.goblet cells
2.chief cells
answer: parietal cells
https://en.m.wikipedia.org/wiki/Vitamin_B12
Q195-What is the best drug given to prevent postoperative thromboembolism?
1.LW heparine
2.Uf heparine
3.Warfarin
4.enoxaparin
Answer: 4 .enoxaparin (answered by consultant )
most to use now enoxaparin unless there is contraindication (Renal impairment results in decreased elimination of enoxaparin and increased anticoagulant response or hx of heparin induced thrombocytopenia .
https://www.clevelandclinicmeded.com/medicalpubs/pharmacy/janfeb2003/enoxaparin.htm
Q196- Long scenario about pt doing surgery, The key point in Q the pt 1st day develop fever +
sob?
1.atelectasis
Answer: 1

smle ,2016

630

Q197-long scenario I don't remember exactly but there is upper limb hypertension and low or absent lower limb pulse What is Dx?
1.Coarctation of aorta
Answer: 1
Keys to diagnosis Coarctation of aorta include blood pressure (BP) discrepancies between the upper and
lower extremities and reduced or absent lower extremity pulses to palpation.
Reference: http://emedicine.medscape.com/article/895502-clinical#b4
Q198-Pt e CAD angio done for him Lift coronary 90% stenosis Right 40% What is the management :
1. Left coronary bypass
2. Bypass all vessel Stent
Answer: 1
Reference: http://emedicine.medscape.com/article/164682-overview#a2
Q199- pt with melanoma above eyebrow which lymph node will examine ?
1. Cervical ..
2. Parotid lymph nodes
Answer: Parotid lymph nodes
Melanoma spread to lymph nodes follows the lymphatic drainage (SLN). And the lymphatuc drainage of
the face is as follows:
-Anterior part of the face and forhead => Submandibular LN
- Lateral part of the face => Parotid LN
- Cebtral part of the lower lip and chin => Submental LN
So (Submandibular LN) would be more correct than (Parotid).. there may be missed choices !!

Q200-Generalized cervical lymphadenopathy + mild tenderness + low grade fever. What's the most
likely diagnosis:
1. lymphocytic lymphoma
2.hodgkin's lymphoma
Answer: difference between Hodgkin's and non-Hodgkin's lymphoma by examining the cancer cells under a microscope. Hodgkin's lymphoma associated with ReedSternberg cell.
References:
http://emedicine.medscape.com/article/201886-overview

smle ,2016

631
http://emedicine.medscape.com/article/203399-overview
Hodgkin's lymphoma is charactrized by enlargement of single group of lymph nodes (generalised lymphadenopathy is rare) and is less common than NHL
Lymphocytic lymphoma is a NHL charactrized by generalised lymphadenopathy and is more common
than HL
Q201-smoker with hilar mass what suspect
1.lymph node
2.squamous cell ca
3.adenocarcinoma
answer : squamous cell ca
It is impossible to defrentiate between them radiologicaly but squamous cell ca usually caused by heavy
smoking
Reference:http://radiopaedia.org/articles/squamous-cell-carcinoma-of-the-lung
Q202-what is the most common type of gall stone
1.Cholesterolbile pigmented
answer : cholesterol stone
Reference: http://www.mayoclinic.org/diseases-conditions/gallstones/basics/causes/con-20020461
Q203- fecal incontinence which level is affected
1.above c2
2.below c2
3.above t12
4.bolow t12
Answer: below t12
Pudendal nerve (s2-4) is responsible for inervation to pelvic floor muscles and external anal sphincter
Reference: http://www.nafc.org/spinal-cord/
Q204-Case Symptoms of cholecystitis what best intial modelity ?
Answer :U/S
Reference: Recall of surgery

Q205-Women with dysuria routine urine analysis show epithelial cell diagnosis :
1. Cerviacl erosion
2. Clymdia urethritis
3. Vaginal conta
4. renal Stone
Answer: 3
Renal tubular cells are 1.5 to 3 times larger than white cells and are further distinguished by a round,
large, centrally-located nucleus.
Reference : up to date
http://www.uptodate.com/contents/urinalysis-in-the-diagnosis-of-kidney-disease?source=outline_link&view=text&anchor=H19#H19
Q206-testicular lymph node drainage ? ( I am not sure is it testicular or scrotum )
1.Left aortic lymph node
2.Superficial inguinal lymph node

smle ,2016

632
3.Deep inguinal lymph node
Answer: 1
Testis: to para-aortic lymph nodes
Lymphatic drainage of the testis follows the testicular vessels (in the spermatic cord) to the right and left
lumbar (caval/aortic) and preaortic lymph nodes at the second lumbar level.
scrotal: to superficial inguinal lymph nodes.
Reference: http://teachmeanatomy.info/pelvis/the-male-reproductive-system/testes-epididymis/
Q207-Which of the following protect against some cancers?
1. Fibrate
Reference : up to date
http://www.uptodate.com/contents/cancer-prevention?source=outline_link&view=text&anchor=H11#H11
Q208-which indcate how long open wound ..in inflammatory process ?
1.coverd by eshar
2.covered by epithilium
3.neovasculrization
Answer: 2
Reference : up to date
http://www.uptodate.com/contents/wound-healing-and-risk-factors-for-non-healing?source=outline_link&view=text&anchor=H1070222553#H1070222553
Q209-Renal artery stenosis , Angie was done , what is next ?
Since the renal artery stenosis is already detected by the angiography, there is no role for CT scan further
this point. The next step is to differentiate between atherosclerosis & the fibromuscular dysplasia since
the management of them differ.
There is similar question with ( renal artery biopsy ) was one of the choices. I think it is the proper next
step.
Q210-Patient came for routine checkup CXR shows a unilateral calcified nodule on the upper zone
of his hung:
1.SCC
2.Hamartoma
Answer:2.hamartoma
Reference: http://emedicine.medscape.com/article/356271-overview
Q211-Something about post facial surgery, patient developed some neurological symptom, which
branch is effected options were branches of the main nerves?
Answer: Sensory or Motor?? Q is messing imp info
Q212-Patient underwent supradrelenectomy, you are worried not to injure which organ:
1.nferior vena cava
2.kidney
3.appendix
Answer: 2
http://m.endocrinediseases.org/adrenal/surgery_complications.shtml

smle ,2016

633
Q213-Old patient developed mass in the lung (CXR was attached)
1.SCC
2.adenocarcinoma
Answer: it depend on CXR and Hx
If it is arise in the peripheral, areas of the lungs. They also have a tendency to spread to the lymph nodes
and beyond. multiple sites in the lungs and spreads along the preexisting alveolar walls. It may also look
like pneumonia on a chest X-ray. and is very common in non-smoking women and in the Asian population. adenocarcinoma
more common than adenocarcinomas arise in the central chest area in the bronchi. This type of lung cancer most often stays within the lung, spreads to lymph nodes, and grows quite large, forming a cavity.
SCC
http://www.webmd.com/lung-cancer/guide/lung-cancer-types
Q214- infant periumblical hernia , what you will do:
1.put plastic in mid abdomen
2.reassurance
3.do hernia repair before start school
answer: 2? Umbilical hernias: Most umbilical hernias do not require surgical repair until approximately
age 5 years. For that reason, managed by simple observation
http://emedicine.medscape.com/article/932680-treatment
Q215- 16 years old female . Fever and Chronic diarrhea for 10 months Post meal para umbilical
pain Sometimes blood mixed with stool
1.crohn
2.chronic pancreatitis
Answer: 1Crohn's disease Symptoms include watery diarrhea,abdominal pain,fever and weight loss.
http://www.webmd.com/ibd-crohns-disease/crohns-disease/crohns-disease
Additional Questions ( 8th update)
Q216- Pt with swelling in the thyroid , She is euthyroid what will you do ?
1.Thyroid lobectomy
2.FNA
3.Biopsy
4.Excisional biopsy
answer :2
If the serum TSH concentration is normal or elevated, and the nodule meets criteria for sampling, then
FNA biopsy is indicated.
http://www.uptodate.com/contents/diagnostic-approach-to-and-treatment-of-thyroid-nodules?source=outline_link&view=text&anchor=H11#H11
Q217-Hiatal hernia .. during surgery nerve position
position of nerve
Answer : ?
Q218- Pt was skiting on the stair , then he was falling with a saddle trauma, he developed penile,
scrotal & lower abdominal swelling with hematuria , which organ was affected ?
Bladder
Urethra

smle ,2016

634
Testes
Prostate
Answer :
Q219- 48 yrs women, without any family history of Breast CA had Normal mammogram when to
repeat it?
1. 2 yrs
2. 3 yrs
3. 5 yrs
answer : ?
annual screening for some women annual screening for some women (eg, premenopausal)
http://www.uptodate.com/contents/screening-for-breast-cancer-strategies-and-recommendations?source=outline_link&view=text&anchor=H79986#H79986
Q220- Patient with breast lump painless increased in size >3 cm I don't remember the details:
1. fibroadenoma
2.cancer
3.fibrocystic changes
Answer: ?
Q221-Testicular cancer with high AFP and BHCG
1.Seminoma ( high BHCG )
2.Embryonal ( high AFP )
3.Teratoma
answer :
Elevations in the serum concentration of AFP or beta-hCG cannot be attributed to teratomatous elements
http://www.uptodate.com/contents/anatomy-and-pathology-of-testicular-tumors?source=outline_link&view=text&anchor=H11#H11
Q222- patient presented with cervical mass not in the midline and move with swallowing, what is
diagnosis:
1.thyroid nodule
2.thyroglossal cyst
3.brachial plexus
4.cervical lymph nodes"
answer : 1
Assessment of the mass with swallowing is important as movement from swallowing suggests a lesion in
the thyroid gland or a thy- roglossal cyst . The latter also elevates with tongue protrusion .
http://www.ajol.info/index.php/cme/article/download/43974/27491
Q223- patient presented with cervical swelling associated with weight loss, night sweating and fever
but no respiratory or neurological symptoms, he was diagnosed as non hodgkin's lymphoma. On
CT scan, there are paraaortic and inguinal lymph nodes enlargement what is the stage:
1.IB
2.IIB
3.IIIBS
4.IIIIBD "
answer :3

smle ,2016

635
http://www.uptodate.com/contents/image?imageKey=HEME%2F97479&topicKey=HEME%2F4696&source=see_link
Q224- Patient with appendicitis but it presented as right upper quadrant pain. Cause?
1.Subhepatic appendix .
Answer: 1
http://www.sciencedirect.com/science/article/pii/S0378603X12000538
Q225- pt present with Right subcostal area pain ... and you do surgery and you find inflamed appendix?
Answer: sub-hepatic appendix
http://www.sciencedirect.com/science/article/pii/S0378603X12000538
Q226- male, found small stone, at level lumbar 3-4?
1.major calyces
2.minor calysis
3.ureter
4.renal cortex"
Answer: 3
Q227- Forceps forgotten inside patient after surgery, what will you do?
Answer : tell the patient what you found and referred him to OR
Q228- Patient with osteoid sarcoma. Then he started having pelvic pain. Cartilage and bone tumor
in pelvic bone on imaging. Cause?
1.Chondreoblastic Osteoma
Answer: Osteochondroma or Enchondroma ?
http://www.uptodate.com/contents/chondrosarcoma?source=outline_link&view=text&anchor=H2#H2
Q229- Staging of lung cancer, tracheal nodes were involved?
Answer : if lung cancer reash trachea it T4
N1 Metastasis in ipsilateral peribronchial and/or ipsilateral hilar lymph nodes and intrapulmonary nodes,
including involvement by direct extension
http://www.uptodate.com/contents/image?imageKey=ONC%2F80099&topicKey=NEPH%2F3840&source=see_link

smle ,2016

636

Q230- intra abdominal bleeding can lead to:


1.Hypovolemia
2. dehydration
answer: hypovolemia
Rapid volume repletion is indicated in patients with severe hypovolemia (ie, decreased peripheral perfusion, as indicated by delayed capillary refill and cool mottled extremities) or hypovolemic shock (ie, severe hypovolemia plus hypotension)
http://www.uptodate.com/contents/treatment-of-severe-hypovolemia-or-hypovolemic-shock-inadults?source=outline_link&view=text&anchor=H1#H1
Q231-baby bout by his mother with abdominal distention, bad smell vomiting, history of delayed
passege of muconime after 3 days after using enema rectal examination showed empty rectum
(typical history of hirschsprung's disease with pic of abdominal x ray showing multiple air-fluid
levels). and the doctors start to decompress his stomach. what is the definitive ttt ?
1.Leveling Colostomy
2. total colectomy
3. antibiotic
answer: abdominoperineal pull-through in two or three stages, in which patients initially underwent a
diverting colostomy

smle ,2016

637
http://www.uptodate.com/contents/congenital-aganglionic-megacolon-hirschsprung-disease?source=outline_link&view=text&anchor=H13#H13

Q232-pt with abscess in his forehead. where can you find a palpable LN ?
1. mastoid
2. superficial parotid
Answer: anterior auricular as forehead is the frontal scalp
http://www.uptodate.com/contents/image?imageKey=PEDS%2F71073&topicKey=PEDS%2F2867&source=see_link
http://www.bartleby.com/107/177.html
The lymphatic vessels of the scalp are divisible into (a) those of the frontal region, which terminate in the
anterior auricular and parotid glands; (b) those of the temporoparietal region, which end in the parotid and
posterior auricular glands; and (c) those of the occipital region, which terminate partly in the occipital
glands and partly in a trunk which runs down along the posterior border of the Sternocleidomastoideus to
end in the inferior deep cervical glands.
Q233- Orchitis case, pain in scrotum with urine +ve culture, whats the treatment?
1.IV cectriaxone+Doxycycline
2.IV amoxicillin-clavulanate
Answer:
Acute epididymitis most likely caused by enteric organisms (eg, men with a negative gram stain or nucleic acid amplification test for N. gonorrhoeae or men with epididymitis after urinary-tract instrumentation procedures) may be treated with fluoroquinolones alone .
Acute epididymitis most likely caused by sexually transmitted chlamydia and gonorrhea should be treated
with ceftriaxone plus doxycycline /azithromycin .
http://www.uptodate.com/contents/evaluation-of-the-acute-scrotum-in-adults?source=outline_link&view=text&anchor=H2012423711#H12
http://emedicine.medscape.com/article/436154-treatment#d12
Q234-pt came with symptoms of increase IC pressure (confusion , nausea , vomiting), before doing
Head CT what is the cranial nerve examination will support the DX.?

smle ,2016

638
1.optic
2.oculomotor
3.Facial
4.vestibulococlar
Answer: 1
papilledema is often used broadly to denote a swollen optic nerve head, the term papilledema should be
reserved for optic disc swelling that is due to raised intracranial pressure.
http://www.uptodate.com/contents/overview-and-differential-diagnosis-of-papilledema?source=outline_link&view=text&anchor=H6#H6
Q235- about pt with breast cancer on something and having bleeding on rotein check 2x3 cm then
became 3x6cm what suspect : type of cancer? "
Answer:??
Q236-Case scenario about patient with breast lump in upper outer tender and increase within days
from her period asking about the diagnosis:
Answer:
could be FBC (nonproliferative breast lesions) but the Q is missing some data
Referance: First Aid, USMLE Step 2 CK page 263 + 264
Q237-Body surface area rule of?
1.9s*
2.8
3.7
Answer: 1

Q238- Burn tanning bed swelling red blisters?


1.Grade 1
2.Grade 2*
3.Grade 3
Answer: 2

smle ,2016

639

Q239-Pt with fracture humerus what the nerve injury :?


1.Axillary
2.Median
3.Ulnar
4.Radial"
Answer: 1 & 4
Mid-shaft fractures may damage the radia nerve
Surgical neck fracture may damage axillary nerve

smle ,2016

640

Q240- patient with BPH, what will you give?


Answer: ?

Q241-Lesion that is volcano-like on a 70 year old farmers hand?

smle ,2016

641
1.Basal cell carcinoma
2.Squamous cell carcinoma
3.Melanoma
4.Leishmania
Answer: 2
Squamous cell carcinoma associated with sun exposure and usually present on the hand
Keratoacanthoma(spectrum of squamous cell carcinoma) with typical volcano appearance
Source: http://www.aafp.org/afp/1998/0915/p873.html
Q242-Theoretically if there could be a vaccine that could prevent an oncogenic disease, it would be?
1.AML
2.ALL
3.Adult t-cell lymphoma/leukemia
4.Mycosis fungoides
Answer :3 and 4
http://clincancerres.aacrjournals.org/content/12/12/3814
http://www.ncbi.nlm.nih.gov/pmc/articles/PMC1070436/
Q243-thyroid disease associated with papillary cancer ?
1.Hashimoto ?
2.Riedel thyroiditis ?
Answer: 1 (not sure, all what I found were studies )
two major forms of hashimoto thyroiditis : goitrous and atrophic ,atrophic variant is associated with thyroid lymphoma
( non hodgkin lymphoma)
http://www.cancerresearchuk.org/about-cancer/type/thyroid-cancer/about/thyroid-cancer-risksand-causes

Q244- Which of the following is a tumor marker for prostate cancer?


1.CEA
2.alpha fetoprotein
3.acid phosphatase
!!(PSA is not included)
Answer: 3

http://www.ncbi.nlm.nih.gov/pubmed/2431533
note: other markers are human glandular kallikrein 2 (hK2), urokinase plasminogen activator (uPA) and
its receptor (uPAR), transforming growth factor-beta 1 (TGF-1); interleukin-6 (IL-6) and its receptor
(IL-6R) and circulating tumor cells of epithelial origin (CELLSEARCH)
http://www.ncbi.nlm.nih.gov/pubmed/21604943
Q245- A patient was stabbed in his abdomen, o/e hes vitally stable, and some of the mesentery is
out. what will u do
1.exploratory laparotomy
2. wound exploration
3.observe
Answer: 1
http://www.trauma.org/archive/abdo/penetrating.html

smle ,2016

642

Q246- A patient was injured and now has scrotal pain, whats the most likely diagnosis?
1.Testicular torsion
Answer : scrotal hematoma
http://emedicine.medscape.com/article/441272-overview
Q247- women has lichen sclerosis which cancer she will get:
1.squamous cell carcinoma
2.Adenocarcinoma carcinoma
3.Adenosquamous carcinoma
answer : 1
http://www.uptodate.com/contents/lichen-sclerosus-beyond-the-basics
**take care while answering this Q because Lichen sclerosis is associated with vulval cancer, but is not a
cause , Many women with vulval cancer have lichen sclerosis at the time of diagnosis and it is estimated
that there is a low risk of cancer developing in a women with lichen sclerosis (around 35 per cent).
Source: Gynecology with ten teacher ,10th addition
Q248- Male after prostectomy through venous plexus the metastasis will go to:
1.Skull
2.Vertebral body
3.Lung
Answer : 2
http://www.aafp.org/afp/2002/0501/p1834.html
http://www.hindawi.com/journals/ijso/2011/107969/
Q249- pt with rt kidney 14 Cm and left kidney 7 cm .. Arteriography: renal artery stenosis
1.what to do
2.CT angio
3.Ct abdomen
4.biopsy"
Answer: ?
Q250- What the most common site to metastasis of previous case
Answer:
Q251-female pt present with laughing and coughing passing out urine O/E there is swelling in labia
majora ( stress incontinence )
Cystocele
Urethrocele
Answer: cystocele
Q252- after appendectomy he has lung abscess which antibiotic you will choose
( read about it)
Answer: ?
Q253-case of painful nodules in groin area and pain relieved after punching and discharge came
out of it?
Hidradenitis suppurativa
Frunculs

smle ,2016

643
Answer: ?
Q254- Thoracentesis in mid axillary line
4th intercostal space
5th intercostal space
Intercostal space 8
Between 7
Answer: 2
Q255-CT in trauma patient is done to detect what:?
Retroperitoneal bleeding
Intraperitoneal bleeding
It is most specific for trauma
Answer: 1
source: http://www.ncbi.nlm.nih.gov/pmc/articles/PMC3216157/
Q256- Auer rods indicate?
Acute myeloid leukemia
Answer: 1
source: http://www.pathologystudent.com/?p=4868
Q257- Parafollicular cell secrete which type of tumor ?
Follicular
Medullary
Answer: 2
Q258- hereditary chronic pancreatitis is?
autosomal dominant
polygene ?
Answer: 1
source: http://www.ncbi.nlm.nih.gov/pmc/articles/PMC177456/
Q259- "Microsatellite instability?
colorectal cancer
Answer: 1
Q260- Question about bladder cancer after they removed what you will do next
intravesical medication
Observe and serial assessment
And 2 another choices regarding treatment
Answer:? Depend on stage
Q261- Lump in throut barium enima and endoscopy normal?
Squamous cancer
Gerd
Peptic ulcer
Pharyngeal globus
Answer: ?
Q262- surgery is contraindicated for which lung CA?

smle ,2016

644
SCC
SCL
Adenocarcinoma
Answer: 2

smle ,2016

645

smle ,2016

646

9th update
Our link Qs up to 28th of December

Q263-Stone was found at L4 level where is it impacted ?


ureter*,
renal pelvis,
small calyces,
large calques)
Answer: A
WIKIPEDIA
Q264-x of ulcerative colitis when will do colonoscopy?
1y
6m
8y after dx
barem enma 1 y
Answer:C
First aid
Q265-To make sure stone in ureter Ix??
Ct
x ray
ct with iv or oral contrast
Answer:
Nonenhance ct scan
Medscape
Q266-Resected end of small bowel and beginning of large colon >> decfency in :
Fa
b12

smle ,2016

647
uc-..)
Answer :B
Crohns disease most commonly affects the end of the small intestine (the ileum) and the
beginning of the colon, but it can affect any part of the GI tract from the mouth to the
anus. When Crohns disease affects the small intestine, areas of diseased bowel may alternate with areas of normal bowel. The areas of active disease may narrow, forming
strictures, which can block the passage of digested food. And it's treated either with Strictureplasty or small bowel resection
In small bowel resection : a segment of the small intestine is removed and the two ends of healthy intestine are joined together (anastomosis). Small bowel resection may also be required if a hole develops in
the wall of the small intestine.
possible complication of bowel resection is a condition called short bowel syndrome. The small intestine
serves the essential function of absorbing nutrients from digested food into the bloodstream, where they
travel to nourish the body. If too much of the small intestine is removed, nutritional deficiencies may occur.
http://www.ccfa.org/resources/surgery-for-crohns-uc.html

http://www.ncbi.nlm.nih.gov/pmc/articles/PMC111082/#!po=15.6250

smle ,2016

648
Q267-female came for breast screening, her mother and sis had breast ca at 45-48 year. What will u
do now ?
brca1 gene screen
Answer:?
BRCA 1 And mamogram
Women at higher risk of breast cancer
Factors that greatly increase breast cancer risk include:
- A BRCA1 or BRCA2 gene mutation
- A strong family history of breast cancer, such as a mother and/or sister diagnosed
at a younger age

smle ,2016

649

Reference : http://ww5.komen.org/BreastCancer/BreastCancerScreeningForWomenAtHigherRisk.html
Read Also this : http://www.hopkinsmedicine.org/breast_center/breast_cancers_other_conditions/family_history_breast_cancer.html
Q268- 5 y/o male present with erectile dysfunction, he mentions that only worse with stress, medically free, not on any medication, what is the best action?
Refer to endocrine
refer to urology
advice relaxation strategies
Answer : C
emotional issues such as performance anxiety might be predominant and benefit most
from counseling. Whenever possible, interviewing and involving the partner will be useful.
Reference : http://m.cfp.ca/content/56/9/898.full#T1

smle ,2016

650
Q269-man done vasectomy, change his mind want to reproduce again, they found antisperm antibodies what is the cause ?
antigen release or something like that?
cross reactivity with viral infection
inappropriate response of MHC II to antigen presenting cell
Answer:A
Antigen relasing
Urology resisdance
The development of ASA in the male depends on the release of sequestered antigens on germ cells following the disruption of the blood-testis barrier
Source : http://www.ncbi.nlm.nih.gov/pmc/articles/PMC4342441/
Q270-ureteral stone what is the radiological study to detect it?
Abd.ultrasoubd
Ct with contrast
Ct without contrast
There was No xray in options
Answer c
Baily and love
Q271-pt came with Lt flank pain radiates to groin no fever no change in urine color or amount no
sign other than tenderness this pain most likely :
ureteric stone
renal colic
diverticulitis
Answer:B
ureteral coloic with chang urine color
https://www.auanet.org/education/kidney-stones.cfm
Q272-main difference between chron's , ulcerative and IBS should be known
Source : http://www.healthline.com/health/crohns-disease/crohns-ibd-uc-difference#Overview1
http://emedicine.medscape.com/article/179037-overview#a1
http://www.crohns.org.uk/about_ibd/differences-between-crohns-disease-and-ulcerative-colitis
Q273-And half years old un cercumcised boy , toilet Trained , came with his parent with UTI. What
is the best method to obtain urine culture:
Cathere
Clean cath
Suprapubic
Answer: clean cash by pediatric doctor
Source : http://emedicine.medscape.com/article/969643-workup#c1

smle ,2016

651

Q274- swilling test :


torsion test
cancer test
orcico epidermitis
Answer: 3
References :

smle ,2016

652

smle ,2016

653

smle ,2016

654

Q275-old patient in 60s of age smoker presented with history of sore throat hoarseness of voices
for 3 month with enlarged cervical lymph nodes : on examination the was large mass in the throat (
the history was highly suggestive of larynx Ca ) what is the most probably your action :
saline water gargles
analgesia and antibiotic
surgery and radiotherapy
Answer:C
The main treatment of laryngeal ca is radiotherapy or surgery .. They may also have chemotherapy
Reference : http://www.cancerresearchuk.org/about-cancer/type/larynx-cancer/treatment/types/treatmentoptions-for-laryngeal-cancer
Q276-Case of RLQ pain and mass, what is the treatment:
conservative.
surgery.
antibiotics.
Answer:C
Reference : http://www.ncbi.nlm.nih.gov/pmc/articles/PMC3017977/#!po=24.2188
Q277-pt did routine X Ray , they found 2*3 cm nodules that has calcified center , it was in the upper left side of lung he is asymptomatic , what is the next step ?
Follow up and repeat x Ray
Biopsy the lesion
Answer:A
Kaplan
Q278 -pt presented with dysphasia , he has hoarseness and excessive salivation and feels hbat there
is a lump in his throat , what is the dx ?
Achalisia
Diffuse esophageal spasm
Cricopharengeal dysfunction
Answer: 3 Cricopharengeal dysfunction
other answer is esophegeal cancer
Q279-pt had trauma, presented with tachycardia , bradychpnea and hypotension , what is the first
thing you will do ?
Needle thoracotomy
Answer:A (tension pneumothorax )
Q280- pt presented to you with Rt scrotal swelling , he said that it is increasing in size day after
day , on examination , it is not transluminate , what will you do?
Reassure him and get surgical opinion
Answer:A
Q281- in mastectomy what you will cut ?
Answer: Pectorals major muscle
Q282-abdominal radiological investigation showed the inner and outer surface of intestine , what
the pt has ?
Perfoaration

smle ,2016

655
Obstruction
Answer: double wall sign of pneumoperitoneum : perforation = rigler's sign {visualisation of air
in both sides of the bowel wall}
Q283-Pt with low intestinal obstruction most common S/S associated
absolute obstruction
diarrhea
vomiting
Answer: absolute obstruction
The four cardinal symptoms of bowel obstruction are pain, vomiting, obstipation/absolute constipation,
and distention. Obstipation, change in bowel habits, complete constipation, and abdominal distention are
the predominant symptoms in LBO. Vomiting occurs late in the course of the disease. On the other hand,
pain, vomiting, and distention are commonly seen in SBO
http://www.ncbi.nlm.nih.gov/books/NBK6873/
Q284- self-breast examination decrease breast cancer by years:
one year.
two year.
three year.
four year.
Answer:
prof in Breast surgery answer: SBE is not a sensitive tool so you put the least efficacy 1 year
Breast self-examination BSE
1- https://qap.sdsu.edu/screening/breastcancer/facts.html
2- Intensive instruction in BSE did not reduce mortality from breast cancer. Programs to encourage BSE
in the absence of mammography would be unlikely to reduce mortality from breast cancer. Women who
choose to practice BSE should be informed that its efficacy is unproven and that it may increase their
chances of having a benign breast biopsy. http://jnci.oxfordjournals.org/content/94/19/1445.short
Q285-Stab wound lateral to sternocleidomastoid. Patient unable to do abduction of the shoulder
and elevation over his head . What is the nerve injured:
Answer: long thoracic nerve
When the serratus anterior contracts, upward rotation, abduction, and weak elevation of the scapula occurs, allowing the arm to be raised above the head. The long thoracic nerve innervates the serratus anterior
Q286- patient with high ICP (I think it was due to a brain tumor), which of the following nerves is
most likely to be affected?
trochlear,
cranial,
optic,
facial
Answer:
Abducent nerve
https://en.wikipedia.org/wiki/Idiopathic_intracranial_hypertension
Q287-high fat and low fiber diet increases the risk of which of the following cancers?
Answer: colorectal cancer, https://www.nlm.nih.gov/medlineplus/ency/article/000262.htm

smle ,2016

656

Q288- Colon Cancer stage B2 means??? I don't know the answer


mets to LN
No LN involvment
Answer :B
http://www.cancerresearchuk.org/about-cancer/type/bowel-cancer/treatment/dukes-stages-of-bowel-cancer
https://en.wikipedia.org/wiki/Colon_cancer_staging

Q289 -most common parotid gland malignancy?


ANSWER: mucoepedermoid carcinoma
Medscape
Q290- boy was playing with skate board fall on stair raling " " on examination there was
bleeding from urethra . Which part of urethra most probably injured:
penile urethra
prostate urethra
ANSWER:A
Q291-first day after surgery, pt with distress and
Atlelectasis.
Answer :A
Atelectasis in first hr of surgery
van
Vein lateral
Artery middle
Nerve medial

smle ,2016

657

Read about:
Fertility Q
Case of cystitis.
Case of prostate cancer
Case about testicular torsion
case about renal colic
clear scenario about lateral epicondylitis
main difference between ( crohn's , ulcerative and IBS )
Dumping syndrome post surgery
Hernia (especially anatomical structures)
Carpal tunnel syndrome
Renal artery stenosis
Gallbladder stone
Prostatitis
big artery and there branches with their supply.
*dermatomes and nerve supply of upper and lower limb.
*acute cystitis
*testicular torsion
* Margin of neck Quadrants

12 update
Q292- Case of cystitis. http://www.mayoclinic.org/diseases-conditions/cystitis/basics/definition/con20024076
Q293-Case of prostate cancer http://www.mayoclinic.org/diseases-conditions/prostate-cancer/basics/definition/con-20029597
Q294-Case about testicular torsion http://www.mayoclinic.org/diseases-conditions/testicular-torsion/basics/definition/con-20033130

Q295-gunshot with wound bowel perforation , What antibiotics you should give?
Answer: from medscape:its dirty traumatic wound , Antibiotics should cover aerobic and anaerobic organisms: Metronidazole is typically used in combination with an aminoglycoside(Gentamicin) to provide

smle ,2016

658
broad gram-negative and anaerobic coverage. OR Metronidazole +Cefoxitin (a second-generation cephalosporin) OR Metronidazole + Cefoperazone ( a third-generation cephalosporin).
Q296-patient with BPH best investigation:
Cystoscopy
Annual renal function
Annual prostate antigen
Beta-blocker
answer : 3
source: http://emedicine.medscape.com/article/437359-workup
and Toronto notes 2014 FM17
Q297- 40 years old female with 3 months mass 2.5 cm firm, mammogram and
US normal what to do ?
follow up after two cycles
obtain biopsy
Answer: 2
Source: answered by GS resident.
Q298- pregnant women with UTI what is the best drug:
nitrofurantoin
answer: 1
References
1- http://emedicine.medscape.com/article/452604-treatment
2- Master The Boards CK.

Q299. Pt saying that she had a protruded jaw and she wants a cosmetic surgery, when you examine
her her jaw is normal with no protrusion and you say that she doesn't need the surgery She went
already to 2 doctors who also refused to do the surgery. She is done blepharoplasty & other cosmetic surgery before.What does she have?
Body dysmorphic disorder
answer: Body dysmorphic disorder

smle ,2016

659

Reference
1- Master The Boards CK
Q300. Pt with swelling in the thyroid , She is euthyroid what will you do ?
Thyroid lobectomy
FNA Biopsy
Excisional biopsy
answer : 2
Reference
1- Master The Boards CK

Q301-Temporal arteritis, confirmatory dx?


Biopsy temporal muscle
Biopsy temporal artery
Answer 2

smle ,2016

660
Superficial temporal artery biopsy (TAB) is the criterion standard for making a diagnosis of temporal arteritis. TAB should be obtained almost without exception in patients in whom GCA is suspected clinically

Reference
1- Master The Boards CK
Q302- Patient came to u her mother and her sister diagnosed with breast cancer, came for counseling, what to do now?
BRCA
MRI
PET scan
answer: US and mamogram if present + genatic counciing

Reference
1- Master The Boards CK

smle ,2016

661
Q303- what is the first sign of compartment syndrome of the leg?
mottled skin
numbness in the first web space
pulselessness
Answer: PAIN out of proportion

Reference
1- Master The Boards CK
Q304- PDA which nerve affected during surgery?
Lt Vagus N (this is the answer!)
Phrenic N
Superficial N of the heart
PDA operation ..... recurrent laryngeal >>MISSED OPTION

Reference : https://www.openanesthesia.org/pda_repair_recurrent_laryng_n_injury/
Q305- Patient underwent right radical mastectomy. Which of the following will be affected post operatively?

smle ,2016

662
Right arm adduction
Answer:1 - pec. Major action
Radical mastectomy is a surgical procedure in which the breast, underlying chest muscle (including pectoralis major and pectoralis minor), and lymph nodes of the axilla are removed as a treatment for breast
cancer.
http://emedicine.medscape.com/article/1830105-overview
Q306- Which of the following types has the highest risk for developing strangulated
hernia?
Direct inguinal
Indirect inguinal
Femoral
Answer: 3
*Femoral hernias are associated with a higher risk of developing complications than inguinal hernias. In
one study, the rates of strangulation were 22 and 45 percent at 3 and 21 months, respectively, for femoral
hernias, compared with 2.8 and 4.5 percent for inguinal hernias .
http://www.uptodate.com/contents/overview-of-treatment-for-inguinal-and-femoral-hernia-in-adults
Q307- Pt with high intestinal obstruction, what will be the symptom ?
Absolute constipation
Diarrhea
Change of habit
Answer : should be vomiting , if it's not in options it maybe B
http://emedicine.medscape.com/article/774140-clinical
Q308- 64 ys male with medial enlarged prostate, all labs normal, what to do?
http://emedicine.medscape.com/article/437359-treatment
Q309- Pt with RF has Mitral regur , valve was 0.7 cm and other findings in aortic valve ? What is
the proper management ?
Total valve replacement
valvoplasty
balloon
Answer: http://www.uptodate.com/contents/mitral-regurgitation-beyond-the-basics
http://www.echopedia.org/wiki/Classification_of_valve_stenosis_and_regurgitation
Q310- Pt reported that he was diagnosed with pancreatic cancer and received chemo and did not
improve and wants to do invx , his medical report was free (he was lying) , and when he confronted
by reports , he run ? Dx :
most probably malingering

smle ,2016

663

Q311- Q about seminoma of testis


(Germ cell tumor )Malignant; painless, homogenous testicular enlargement; most common testicular tumor, most common in 3rd decade, never in infancy. Large cells in lobules with watery cytoplasm and
fried egg appearance. placental ALP. Radiosensitive. Late metastasis, excellent prognosis.
first aid for usmle step 1
Q312- female pt, k/c of sickle cell, experiencing acute cholecystitis attacks from time to time, on US
there is 7 gall bladder stones, one of them is 2 cm large, now she is asymptomatic, what is your
plan?
cholecystectomy.
Q313-Antidot of digoxin over dose ? ( all answer one word name of medication )
Digibind
ANSWER: 1
Q314- female pt, k/c of crohns disease, presented to the ER with "symptoms of bowel obstruction
due to adhesions including constipation, vomiting " what is the diagnosis?
bowel obstruction.
Q315-best imaging for cyst in the breast
1-US
2-mammogram
Answer: US
Q316- male pt presented with testicular swelling, his abdomen was distended, hyperactive bowel,
gave a hx of constipation and vomiting, what you will do?
consult general surgeon.
consult urologist.
Answer: 1- consult general surgeon.( intestinal obstruction )

smle ,2016

664
Q317- years old lady, found a lump in the upper outer area of her right breast, it was large, firm
with irregular border, not tender, no palpable lymph node, what should you do?
FNA.
lumpectomy and radiation.
mastectomy.
Answer: 2 - lumpectomy and radiation.
Q318-pic of bowel obstruction with baby 2 week what is the treatment ?
primary surgery
leavler surgery
ANSWER: 1
Q319-mid inguinal point ...artery posterior to it ?
external iliac
Femoral
Superior epigastric
Femoris
ANSWER: 2 ??
Q320- 46 y. o. Woman with urine incontinance that makes her wake up at night but she urinates
herself before going to the bathroom, what type of incontinance is this?
urge
overflow
stress
ANSWER 1
Q321- man with hyperthyroidism, presented with thyroid nodule showing high radioactive uptake,
how would you manage him?
antithyroid medication.
radioactive iodine.
thyroidectomy.
Answer : 3, from medscape: Hot nodule (high radioactive uptake) - A hot toxic nodule may require medical therapy before surgical removal; the patient should receive suppressive doses of antithyroid medications often require antithyroid medications before surgery.
Q322-Pt with starry sky pattern on biopsy > burkhits
Wts the mutation:
C-myc gen
abl2
Bcr- abl
answer: 1
Q323- read about how to differentiate bw carpal tunnel syndrome and thoracic outlet obstruction

smle ,2016

665
Answer: http://www.uptodate.com/contents/carpal-tunnel-syndrome-clinical-manifestations-and-diagnosis?source=search_result&search=carpal+tunnel+syndrome&selectedTitle=2~130
http://www.uptodate.com/contents/overview-of-thoracic-outlet-syndromes?source=search_result&search=thoracic+outlet+obstruction&selectedTitle=1~150
Q324-Pt complaining of elbow pain that is radiating down and tenderness with dorsiflexion of
wrist, Dx?
Carpal tunnel syndrome
Q325- Pt with groin abscess after aspirate there was multiple cells, Cause
Immunodeficiency - C5
Q326- Pt diagnosed with papillary renal carcinoma? TTT
Q327- (70-60- 50-90 ) generalized body scaling ? Surface area affected
Q328- Cause of Delayed arterial radiofemoral pulse:
Answer: Coarctation of aorta
Q329- married pt male has hx of unprotected sex , days later he had purulent
discharge with gram negative intracellular diplococci ?
non gonococcal urethritis
Gonococcal urethritis
Answer: B
Q330 - young female noticed a mass 1 week after her menstrual cycle and
persist for three cycles , the mass is rubbery , mobile what&#39;s dx?
fibroadenoma
Fibrocystic change
Duct ectasia
Answer: A
Q331- read about embryology problem about cleft palate ?
Q332- Elder bedridden has an ulcer 4*5 reaching fascia and muscle , what is the
stage?
Stage 1
Stage 2
Stage 3
Stage 4
Answer: 4
Q333- hematuria, CH 16 &gt;&gt; poly cyctic kidney dis

smle ,2016

666

Q334-40 yrs present with 1.5 cm lung nodule what is the most useful to do?
transthoracic biopsy
thoraco biopsy
evalute by multible xray
look at prior x rays++
Q335-we do ct in blunt abd trauma why??
**To evaluate retropertonial hemmorage ++
Q336-pt had hx of infection 2 week later present with hematuria&gt?
post strep
Q337-pt has thyroid nodule we did tsh the result eutheroid&gt;&gt:
FNA
Q338-scrennig prostate ca &gt;&gt;
DRE (my ans)
PSA Two methods used: One is the digital rectal examination (DRE).
The other is the prostate-specific antigen (PSA) blood test
Q339-hyperthrodism pt present with thyroid nodule high radio iodine tx??
exision
radio iodine ablasion
Canti thyroid medication(my ans)
Q340-Pt underwent surgery 6 days later started to complain of infection what is
the source ?
staff hand
medical devices and instrument
gauze pressure
patient visitor
answer: 1
refrence: surgery consultant

smle ,2016

667

Q341-Pt complain of scrotal swelling on examination u saw a bag of worms


&gt;&gt?
varicocele
A patient with a varicocele is usually asymptomatic and often seeks an evaluation for infertility after
failed attempts at conception. He may also report scrotal pain or heaviness. Careful physical examination
remains the primary method of varicocele detection. An obvious varicocele is often described as feeling
like a bag of worms.

Refertence: http://emedicine.medscape.com/article/438591-overview#a10
Q342-cancer pt c/o bone pain if dr suspect mets what is the best radiological test?
bone scan(my ans)
MRI
positron imesion tomography
Answer: A
{refNuclearMedicine}99m

Tc bone scintigraphy is an effective method for screening the

whole body for bone metastases.

[60, 61]

] m Tc diphosphonatesmost commonly

ylene diphosphonate (MDP)is the most frequently used isotope.

Tc meth-

Tc planar bone scintiscans

help in detecting metastatic bone deposits by the increased osteoblastic activity they induce;
this finding is considered to be an indirect marker of tumor.

http://emedicine.medscape.com/article/387840-overview#a5

Q343- mamogram dectect breast ca before self examination by how many years?
A. 1
B. 2
C. 3
D. 4
Answer: B
Mammography is sensitive to microcalcifications that develop in breast tumors with sensitivity at less than 100
m. Mammography often detects a lesion before it is palpable by clinical breast examination and, on average,
1 to 2 years before noted by breast self-examination.

smle ,2016

668

Reference: http://emedicine.medscape.com/article/1947145-workup#c8
Q344- drug indused hyperurecemia :
Answer: Hydrochlorothiazide
Hydrochlorothiazide and other thiazide diuretics may cause metabolic disturbances especially at high
doses. They may provoke hyperglycaemia and glycosuria in diabetic and other susceptible patients. They
may cause hyperuricaemia and precipitate attacks of gout in some patients. Thiazide diuretics may be associated with electrolyte imbalances including hypochloraemic alkalosis, hyponatraemia, and hypokalaemia.

Reference: https://www.medicinescomplete.com/mc/martindale/current/2333-v.htm

Q345- Kid with bilateral abscess of inguinal nodes what causes this condition ?
Answer: Chronic granulmoatous disease
Abscesses characteristically heal slowly and leave prominent scars. Skin manifestations in older patients include healed scars of old lesions in the cervical or inguinal areas or scars secondary to multiple surgical procedures performed to drain abscesses.

Rference: http://emedicine.medscape.com/article/1116022-clinical#b4
Q346- NHL staging (Ann arbor)Patient with B symptoms and lymph nodes from
the neck to the para aortic with spleen involvement?
A. Stage 1
B. 2
C. 3
D. 4

Answer: 4

Reference: http://emedicine.medscape.com/article/2007081-overview
Q347- Urge incontinance Concept of treatment ?
A. Medical
B. Surgical
C. Or both+ lifestyle modification (fluid restriction, avoidance of caffeine(

smle ,2016

669
D. bladder retraining
Answer: C
Urge incontinence - Changes in diet, behavioral modification, pelvic-floor exercises, and/or
medications and new forms of surgical intervention
Reference : http://emedicine.medscape.com/article/452289-treatment

Q348- urge incontinence how to diagnose ?


Answer: Patients with urinary incontinence should undergo a basic evaluation that includes a history,
physical examination, and urinalysis. Additional information from a patient's voiding diary, cotton-swab
test, cough stress test, measurement of postvoid residual (PVR) urine volume, cystoscopy, and urodynamic studies may be needed in selected patients.
Reference: http://emedicine.medscape.com/article/452289-workup
Q349- Thyroid nodule Rx in the right side only?
Answer: All thyroid nodules that are found to contain a thyroid cancer, or that are highly suspicious of
containing a cancer, should be removed surgically by an experienced
thyroid surgeon. Most thyroid cancers are curable and rarely cause life-threatening problems. Thyroid
nodules that are benign by FNA or too small to biopsy should still be watched closely with ultrasound examination every 6 to 12 months and annual physical examination by your doctor. Surgery may still be
recommended even for a nodule that is benign by FNA if it continues to grow, or develops worrisome
features on ultrasound over the course of follow up.
Reference: www.thyroid.org/wp-content/uploads/.../Nodules_brochure.pdf
Q350- Peptic ulcer disease Rx the kid is on PPI and metronidazole
What to add?
Clarithromycin

Reference: http://emedicine.medscape.com/article/181753-medication
Q351- Lady with a mass in left upper quadrant of the breast, aspiration was
yellow fluid without masses, Dx?
Phyllodem, normal tissue variant, ...&quot;no simple cyst&quot;
Q352- child with blue dot in testis and painful mass in inguinal area?
testicular appendix torsion
The presence of a paratesticular nodule at the superior aspect of the testicle, with its characteristic bluedot appearance, is pathognomonic for this condition. A blue-dot sign is present in only 21% of cases.
Reference: http://emedicine.medscape.com/article/778170-clinical#b4
Q353- First degree spleen injury, ttt?
Immediate laparatome.
conservative in ward.
conservative in ICU
laparoscopy
answer: ?

smle ,2016

670
The trend in management of splenic injury continues to favor nonoperative or conservative management.
This varies from institution to institution but usually includes patients with stable hemodynamic signs,
stable hemoglobin levels over 12-48 hours, minimal transfusion requirements (2 U or less), CT scan injury scale grade of 1 or 2 without a blush, and patients younger than 55 years.
Reference: http://emedicine.medscape.com/article/432823-treatment
Q354- (Case of Intussusception) Child came with colicky abdominal pain,
vomiting, bloody stool. US showed doughnut sign. What is the most
important step in management of this case ?
urgent surgery referral
NGT decompression
IV Fluid resuscitation
Barium Enema
Answer : C- IV Fluid resuscitation (Ref. Master the board)
Q355- Which of the following is associated with Burkitts Lymphoma ?
EBV
HIV
cocxacki
HBV
Answer : A- EBV
Q356- trauma on Rt side of abdomin then abdominal swelling and heamatouria(I
think)
Wilms Tumor
Q357- Painless abdominal swelling ?
Wilms Tumor
refrence: http://emedicine.medscape.com/article/989398-overview
Q358- Famale married pt came with all symptoms and signs of typical
appendicitis( periumbilical pain but still didn&#39;t shift) + sever pain on rectal
exam + low grade fever?
Ovarian torsion
Ectopic pregnancy
Appndicitis
Colycisitis
answer is 3
Q359- 41 years old pt with recurrent episodes of acute cholecystitis U/S showed
small stones, you will give her: ?
ezitamib
fibramait

smle ,2016

671
Answer : ezetimibe

Q360- Pt with small lung ca and undergoing chemo , developed


I don&#39;t remember, But lab values : low blood Na, And low urine osmolality
The treatment: ?
Desmopressin
nothing
Answer : 2
NB. I think there is mistake in the Q and he was mean high urine osmolality not low . I think it's
about SIADH which caused by SSC
Q361- most reliable screening for prostate cancer is:
PR examination
Answer
The American Cancer Society (ACS) recommends that men have a chance to make an informed decision
with their health care provider about whether to be screened for prostate cancer. The decision should be
made after getting information about the uncertainties, risks, and potential benefits of prostate cancer
screening. Men should not be screened unless they have received this information. The discussion about
screening should take place at:
Age 50 for men who are at average risk of prostate cancer and are expected to live at least 10 more years.
Age 45 for men at high risk of developing prostate cancer. This includes African Americans and men who
have a first-degree relative (father, brother, or son) diagnosed with prostate cancer at an early age
(younger than age 65).
Age 40 for men at even higher risk (those with more than one first-degree relative who had prostate cancer at an early age).
After this discussion, those men who want to be screened should be tested with the prostate-specific antigen (PSA) blood test. The digital rectal exam (DRE) may also be done as a part of screening.
If, after this discussion, a man is unable to decide if testing is right for him, the screening decision can be
made by the health care provider, who should take into account the patients general health preferences
and values.
Assuming no prostate cancer is found as a result of screening, the time between future screenings depends
on the results of the PSA blood test:
Men who choose to be tested who have a PSA of less than 2.5 ng/mL may only need to be retested every
2 years.
Screening should be done yearly for men whose PSA level is 2.5 ng/mL or higher.
Q362- 50 yrs man diabetic, well controlled had colon cancer surgery ( coloectomy) they kept him on
insulin and dextrose, after surgery by 2 days he became irritable, in shock, his electrolyte ( Na 129 )
( K 3.2 ) urine and serum osmolality Normal, what's the Dx:
fluid overload
addison disease
SIADH
SIADH: hyponatremia, low serum osmolality, Normal K+
Addison: hyponatremia, hyperkalemia*

smle ,2016

672
Fluid overload:
Answer : 1
Reference : Master the Boards
Q363-18 y o boy came to ER with painful groin after he was playing football , Diagnosis?
Ruptured hydrocele
testicular torsion*
ANSWER 2
Q364-most common cause of death in flame burns?
trauma
smoke inhalation
ANSWER 2
http://emedicine.medscape.com/article/771194-overview
Q365-Rx of patient with lung ca stage IIIb came with sudden lower back pain?
MRI only
MRI with steroid
radiotherapy*
Answer: RADIOTHERAPY
http://www.uptodate.com/contents/non-small-cell-lung-cancer-treatment-stage-iv-cancer-beyond-the-basics?source=outline_link&view=text&anchor=H10#H10
Q366- Enlarged medial superficial inguinal lymph nodes. What would you like to examine?
Anal area*
Gluteal area
Medial thigh
Leg
Answer: 1
http://aibolita.com/sundries/10912-enlarged-superficial-inguinal-lymph-nodes.html

Q367-Patient with lesion above the left eye brows , first lymph node to be examined is :
Parotid*
Mental
Submandibular
Answer :1
Clinical Anatomy (A Problem Solving Approach), Second Edition By Neeta V Kulkarni

Q368-A man runs for a long distance then develops pain in the thigh with loss of hair diagnosis :
DVT
Acute leg Ischemia
chronic leg ischemia*

smle ,2016

673
Answer : 3
Toronto Notes 2016 PDF (Essential Med Notes 2016)
Q369-A patient was found to have a bilateral hypo-echoic cysts; by ultrasound. What is the next
step?
MRI
CT
Biopsy
Answer: CT tumor marker
http://emedicine.medscape.com/article/255865-workup#c1

Q370- baby with decreased air in the lung, abdomen looks scaphoid, what is the diagnosis?
diaphragmatic hernia.*
dextocardia.
Answer:1
Toronto Notes 2016 PDF (Essential Med Notes 2016)
Q371-Stap wonde in the abdomen with bacteroid fragilis what is the best antibiotic to use
ampicillin
ANSWER: 1
http://emedicine.medscape.com/article/233339-treatment
Q372- A scenario clearly describing a case of pelvic abscess. How will you drain it?
Laparoscopy
Laparotomy
Colpotomy
Medications?
answer : 1 or 2
Procedures used for drainage of the abscess include:
Ultrasound-guided aspiration and drainage: usually the abscess would be rectally drained in men, and in
females it would be drained vaginally.[5][6]
CT-guided aspiration and drainage. Percutaneous drainage often uses a trans-gluteal approach.[7]
Endoscopic ultrasound-guided drainage (EUS-guided drainage). Evidence supporting this as an effective,
minimally invasive option is growing.[8][9]
Laparotomy or laparoscopy with drainage of abscess may be required in some cases.
http://patient.info/doctor/pelvic-abscesses
Q373-most accurate test for acute cholysctitis?
A. Ultrasonography 90-95% sensitive for cholecystitis and is 78-80% specific.
B. CT & MRI The sensitivity and specificity greater than 95%

smle ,2016

674
C. Hepatobiliary Scintigraphy sensitivities and specificities 90-100% and 85-95%

Answer : C
http://emedicine.medscape.com/article/171886-workup#c1
Q374-Patient with stab wound what to do next ?
exploratory laparotomy and FAST
http://emedicine.medscape.com/article/2036859-overview#a1
Q375- patient with right hypochondrial pain and US showed echogenic shadow ?
Gallstones (Cholelithiasis)
Answer : Gallstones (Cholelithiasis) ,stone in the gallbladder neck with typical acoustic shadow.
Although it originally referred to ultrasonographic findings of echogenic, nonshadowing, microscopic
material within the gallbladder, the term biliary sludge currently indicates a precipitate of microcrystals
occurring in bile with high mucous content. Sludge may contain microliths. Milk of calcium bile, a calcium carbonate precipitate opaque on plain radiographs, may coexist with cholelithiasis.Ref: http://emedicine.medscape.com/article/366246-overview#a1
Q376- Case scenario about patient with breast lump in upper outer tender and increase within days
from her period asking about the diagnosis ?
fibrocystic changes
http://www.webmd.com/breast-cancer/benign-breast-lumps
Q377- Urology pt had papillary cancer removed biopsy shows removal of all tumor till muscle
layer, What next?
Follow up with cystoscopy
given something medication i think chemo and other two options
Answer:
For patients at low risk of recurrence following TURBT, we recommend an immediate, single postoperative dose of chemotherapy. This is considered sufficient in itself without the need for additional therapy,
and Bacillus Calmette-Guerin (BCG) is never given in this setting. The most extensive data for intravesical chemotherapy are with mitomycin. Repeat urine cytology (particularly for high grade cases) and cystoscopy are generally advised at three to six month intervals, depending on the number of tumor recurrences, for the first four years and annually thereafter in the absence of tumor recurrence.
Ref. Uptodate
Q378- A patient undergone for a major surgery, a days later, his surgical wound was infected, what
is the most likely source of infection:
The patient visitor at visiting time.
The staff hands during examination and dressing.
The pressure dressing gauze.
The dressing tools and devices

smle ,2016

675
Q379-12 years old football player presented to the er with nausea and vomiting and tender scrotal
swelling , what is the management ?
urology referal
immediate surgery
Q380-test for carpal tunnel syndrome:( smle q bank )
Compression test
tinnel
phenel
durkan carpal test
answer : 4
Q381-Pt with small lung ca and undergoing chemo , developed ... I don't remember
But lab values: low blood Na And low urine osmolalityThe treatment:
Desmopressin*
Nothing
Q382- image of knee joint what DX ;
osteoarthritis
Q383- antier femur what ligment ;
?ACL ( )
Q384-lacrmal test :
ACL ( )
Q385-direct HX what DX:
acute appenditis
Q386-direct HX reged bogg by examination what DX:
acute prostatis
Q387-hx pf pt has swallon testis and bag of worm :
varicele
Q388- pt has high uptake radi scan what give :
Antithyrid hormone
radioactive idondine
answer: may 1 coz suspect gravis but not give HX of eye symptom
Higher-than-normal uptake may be due to an overactive thyroid gland caused by:
Graves disease
An enlarged thyroid gland that contains nodules producing too much thyroid hormone (toxic nodular
goiter)

smle ,2016

676
A single thyroid nodule that is producing too much thyroid hormone
These conditions often result in normal uptake, but the uptake is concentrated into a few (hot) areas while
the rest of the thyroid gland does not take up any iodine (cold areas).
Lower-than-normal uptake may be due to:
Factitious hyperthyroidism
Iodine overload
Subacute thyroiditis
Silent (or painless) thyroiditis
Amiodarone
https://www.nlm.nih.gov/medlineplus/ency/article/003689.htm
Q389-papillary tumor non invasive 2cm ( more details ) next step ;
intravesical bbg
radical
Answer: not know
#from tronto treatment of bladder ca:
*superficial (non-muscle invasive) disease: Tis, Ta, T1:
low-grade disease :single dose mitomycin c within 24 h of resection reduces recurrence rates
high-grade TURBT intravesical chemo/immuno-therapy (e.g. BCG, mitomycin C) to decrease recurrence rate
maintenance with intravesical chemotherapy with BCG for 3 cycles every 3 mo, may be continued for 23 yr
*invasive disease: T2a, T2b, T3:
radical cystectomy + pelvic lymphadenectomy with urinary diversion (e.g. ileal conduit, Indiana pouch,
ileal neobladder) or TURBT + chemo-radiation (bladder sparing) for small tumors with non-obstructed
ureters
neo-adjuvant chemotherapy prior to cystectomy may also be done
use of adjuvant chemotherapy after definitive local treatment is controversial
*advanced/metastatic disease: T4a, T4b, N+, M+:
initial combination of systemic chemotherapy irradiation surgery
Q390- Pt MVA present with open fracture , scenario of Wound infection what is the organism :
colostridum perfungens.
Q391- Hx of stress incontinance , management:
kegal exercise
Q392- What is pain killer used in cholecystitis:
Q393-pancreatitis chronic vs. Acute
enuresis

smle ,2016

677

Q394-surgery is contraindicated for which lung CA?


SCC
SCL
Adenocarcinoma
Answer: 2 ( small - oat - cell )
Q395-Elderly male, presenting with fever (38.5), dysurea, exam was normal except DRE which
showed tender enlarged prostate, no penile secretions, no flank pain, hemodynamically stable, normal level of conciousness. Labs: UA showed 20 WBC +ve citrate only .. Diagnosis?
Pyelonephritis
Acute Prostitis*
Cystitis
Gonnorhea
Q396-patient came with paicture of appendicitis, mass in RIF, what is next step?
Reassurance
surgery

Q397-Women with history 2 times of right hypochondreum pain she took analgesic for it . Presented now to Er with acute right hypocondreum pain . CT showed
Normal gallblader with small stone not obstructing the blader. Us showed small stone .3 cm what is
the managment?
...
simvastatin
ursodoxycolic acid
chenodoxicolic acid
No surgery or iv antibiotic in choices
Answer : 3
Q398-patient with back pain and signs of retention , what to do first?
psa
urodynamic
Answer : 1
Q399-Complain of lump like sensation.. No dysphagea..All ex normle investigation normle barum
and endoscopy ..what is the passible dignosis?
Pharangial dieverticuli!
urin retntion and bilateral hydronphrosis ?
Ureathral Meatal ..
Bladder cancer
Prostatic cancer
pelivic tumor

smle ,2016

678

Q400-Olf Pt has diffcult to urinate snd back pain..High prostatic enzyme..Most likely dignosis ?
BPH
Prostatic cancer
Q401-Xray showed opacity at L2 or L4. Stone at what level:
uretric
minor calyces
major calyces
renal pelvis
Answer:
Q402-Solitary thyroid nodule ..what is the difintive dignostic test?
FNA
Exsional
Insional
Q403-3 questions were about about milestone
1st q the mentioned :Tricycle
2nd q : smile Pull to sit
3rd q: Languadge development
Q404- pt male 50s i think with chronic epigastric abd pain , last wk lost 7 kg +ve stool occult blood
Chronic pancreatitis
Chronic cholecystitis
Chronic gastritis
Answer: i think due to significant wt loss should think about malignencey .. but the case is not clear .
Q405- young pt, with neck mass and itching, Dx?
Hodgkin lymohoma
Lyme disease
Answer: 1
https://en.wikipedia.org/wiki/Hodgkin's_lymphoma
Q406- in Splenectomy what organ might be affected
Q407- women with mastitis
stop breast feeding
clean nipple with alcohol
surgical drainage
Answer: question and choices are not complete how ever
Nursing should be continued with Analgesia and warm compressors. if not improved after 24 hours, start
abx like augmentin, if abscess developed ; surgical drainage

smle ,2016

679

Q408- pt with RLQ pain and swelling , loss weight , colonoscopy done showed mass in RLQ , Dx :
Appendix tumor
Cecum tumor
answer: 2
Q409- in open cholecystectomy, name of incision:
Kochar ( subcostal)
Q410- pt have parotitis, pain with eating that radiate to the ear , with nerve transmit pain with eating?
8
9
10
7*
Parotitis refers to inflammation of the parotid gland, usually as a result of an infection. The parotid gland
is enclosed in a tough fibrous capsule. This limits swelling of the gland, producing pain.The pain produced can be referred to the external ear. This is because theauriculotemporal nerve provides sensory innervation to the parotid gland and the external ear. it is branch of 5 Mandibular !! but not in options !!
http://teachmeanatomy.info/head/organs/salivary-glands/parotid/
but if not present Facial nerve the most likely answer
Q411- NERVE between heads of pronator teres muscle?
Median nerve
Q412- pressure in RT EYE =22 LEFT=40 Cup to disc ratio in RIGHT EYE W1??
glucoma ??
Q413-Retinal vessels displaced old age come with headache ?
Acute angle glaucoma
Diabetic retinopathy
Macular degeneration
No open angle glaucoma in choices
answer: 1
http://www.allaboutvision.com/conditions/narrow-angle-glaucoma.htm
Q414-surgeon did suprarenal something what he will hit anteriorly to it?
kidney
tail of pancreas(my answer)
Right adrenal gland
medial: right crus of the diaphragm, right inferior phrenic nerve
lateral: right lobe (bare area) of the liver
anterior: inferior vena cava (IVC)
posterior: right kidney 3, 6, 7

smle ,2016

680
Left adrenal gland
medial: left crus of the diaphragm, left inferior phrenic nerve
anteromedial: diaphragm
anterior: lesser sac, stomach, splenic artery, pancreas
posterior: left kidney 3, 6, 7
Q415-male patient complain of dysphagia for solid and liquid ,radiograph shows dilated esophagus
with tapering at end , what is ur dxx?
spasm
cancer
achalasia +(rat tail , birds peak)*
answer: 3
Q416-young male stabbed in his back cant (abduct or adduct ) his right thigh what is the affected
muscle ?
magnus
longus
answer: gluteus medius or gluteus minimus
gluteus medius and minimus are the most powerful hip abductors, assisted by piriformis and tensor fasciae latae
adductor longus, adductor brevis, adductor magnus, and gracilis are the powerful hip adductors, assisted
by pectineus.
Q417-dorsalis pedis artery :
answer: lateral to extensor hallucis longus
The dorsalis pedis artery is the distal extension of the anterior tibial artery in the foot. It is located just lateral to the extensor hallucis longus tendon and is best palpated proximal to the first metatarsal-cuneiform
joint.
https://en.wikipedia.org/wiki/Dorsalis_pedis_artery
Q418- patient with foot drop ?
deep peroneal nerve
common peroneal nerve (fibular)
answer: 2
Q419-muscle which make the knee flextion from upright position is?
answer: Flexion of the leg Muscles: sartorius, gracilis, hamstrings (semitendinosus, semimembranosus,
biceps femoris), gastrocnemius, plantaris
Q420-Sites of Displaceed appendix
??

smle ,2016

681
Q421-Drug that interfer with OCP ?
answer: Antiepileptic
http://www.medicinenet.com/oral_contraceptives/page4.htm
http://www.fpnotebook.com/gyn/pharm/OrlCntrcptvDrgIntrctn.htm
Q422-Patient had splenic trauma and splenectomy planned, if the surgeon ligate the splenic artery
high up what structure would be affected ?
answer: Stomach fundus
short gastric artery arise from splenic artery at the hilum of spleen .. supply the fundus
Q423-mucsle that dorsiflex the foot ?
answer: Tibialis anterior
http://www.dummies.com/how-to/content/muscles-that-move-the-ankle-and-foot.html
Q424-Internal iliac artery injury
The internal iliac artery is the smaller terminal branch of the common iliac artery. It supplies the pelvic
walls, pelvic viscera, external genitalia, the perineum, buttock and medial part of the thigh.
Gross anatomy
Origin
The common iliac artery bifurcates into the internal iliac artery and external iliac artery at the level of the
pelvic brim anterior to the sacroiliac joint.
Course
The internal iliac artery courses posteromedially towards the greater sciatic foramen. It is approximately 4
cm in length.
At the superior margin of the greater sciatic foramen it divides into an anterior division and posterior division. The anterior division continues down to the ischial spine anterior to piriformis giving off visceral
and parietal branches. The posterior division only gives rise to parietal branches.
Relations
anteriorly: ureter, ovary, uterine tube
posteriorly: internal iliac vein, lumbosacral trunk, sacroiliac joint
medially: peritoneum
laterally: external iliac vein, obturator nerve 3
Branches
Anterior division
umbilical artery (only patent in the fetus)
superior vesical artery (branch of the umbilical artery)
obturator artery (25% will branch off the inferior epigastric artery)
vaginal artery

smle ,2016

682
inferior vesical artery
uterine artery
middle rectal artery
internal pudendal artery (supplies the external genitalia)
inferior gluteal artery
The obturator, internal pudendal and inferior gluteal arteries are parietal branches, whereas the other arteries in the above list are visceral arteries (i.e. umbilical, superior and inferior vesical, vaginal, uterine and
middle rectal artery).
The nine branches of the anterior division of the internal iliac artery may be more easily remembered in
these divisions:
"three urinary": umbilical artery, superior vesical artery, inferior vesical artery
"three visceral": uterine artery, vaginal artery, middle rectal artery
"three parietal": obturator artery, internal pudendal artery, inferior gluteal artery
Posterior division
iliolumbar artery
lateral sacral arteries
superior and inferior branches
superior gluteal artery
Mnemonics to remember branches of internal iliac artery include:
I Love Sex or PILLS G (posterior division)
SOI VU MR PIG (anterior division)
I Love Going Places In My Very Own Underwear! (anterior and posterior divisions)
I <3 U SUMO SIL (anterior and posterior divisions)
Q425-Testicular lymphatic drainage

smle ,2016

683

Q426-Brain ventricles anatomy


http://teachmeanatomy.info/neuro/vessels/ventricles/
http://emedicine.medscape.com/article/1923254-overview
Q427-Diabetic pt developed foot ulcer, diabetes role for this non healing ?
Inhibit phagocytosis ... Cant remember other options
Q428 -56 mal patient came with Pterygium , what you will tell him ?
Pre malignant lesion
Affect his vision
Answer : 2
fibro vascular triangular encroachment of epithelial tissue onto the cornea, usually nasally may induce
astigmatism, decrease vision / torento note 2015
Q429-After fracture which u must do for evaluate bone density ?
X ray
Dexa
ANSWER: 2
http://www.radiologyinfo.org/en/info.cfm?pg=dexa
Q430-Obterator nerves injured which muscle affected. ?
ANSWER: adductur muscle(longus magnus brevis gracilis

smle ,2016

684

Q431-Flat foot i think want the type of ligament??


ANSWER: posterior tibial tendon
Dysfunction of the tibialis posterior tendon - a common and important cause
http://patient.info/doctor/pes-planus-flat-feet
Q432-Ptn with Post cataract surgery he came with Pain eye , yellow red reflux , muscle intact >>
what i 1DDx??
Enophalamitis
cellulitis
Answer: 1. Enophalamitis
Q433-Baby 18 month old with Delay speech other exam normal
hearing
development
answer: 1
Q434-What is the most sensitive and specific to diagone and localize bowl obs ?
Ct
Q435-When u operate for aneurism
more 5.5 diabetic thoracic aorta
more 5.5 abdominal atherosclerosis
answer: 2
Q436-Obterator nerves injured which muscle affected. ?
adductur muscle(longus magnus brevis gracilis
Answer: 1
Q437-Which of the following lung cancer contraindication of doing surgery ?
Small cell carcinoma
squamous cell carcinoma
adenocarcinoma
fibroma related to something (Ican not remember)
answer : Small cell carcinoma
Q438- If the surgeon going to do appendectomy ,So he have to ligate the arterial branch of which
the following major artery ?
Inferior mesenteric
Superior mesenteric
Superior epigastric
Inferior epigastric
answer : Superior epigastric

smle ,2016

685

Q439- What is the most common part of urethra could be injured during catheterization ?
Membranous urethra
Prostatic urethra
Penile urethra
Answer :Membranous urethra
Q440-Sickle cell patient with recurrent RUQ pain with elevated pancreatic exam ,During U/S there
was multiple gall stone (I think 7 gall stones), The largest one was2.5 cm when you did ERCP there
was no stone in billiary tree , what is your management ?
Antibiotic
Cholecytectomy
answer :2
Q441-patient after radical mastectomy can't raise her hand above the shoulder :
Long thoracic nerve
Answer :
Q442-upper outer quadrant breast lump , first metastasis to :
Anterior lymph node
posterior LN
Lateral LN
Answer :
Q443-40 years old women with biliary colick , what to give medically :
statin
urodoxyuric acid
Answer :
Q444- lumb in throut barium enima and endoscopy normal:
Squamous cancer
Gerd
Peptic ulcer
Pharungeal globus??
answer :
Q445-Prostatic venous metastasis?
Vertebrae
Skull
Lung
Brain

smle ,2016

686
answer :
Q446-which of these pt perotneal lavage is indicated ?
severe head trauma ,
hypotensive pt with abd distention
answer :
Q447-Chronic alcoholic wake up at 3 am with abdominal pain after a little while the pain generalized all over the abdomen and is severely exacerbated with movement:
Peptic ulcer perforation
Choycystitis
DiverticulaTitis
Pancreatiti
answer :
Q448-question about bladder cancer after they temoved what you will do next:
intravesical medication
observe and serial assesment
And 2 another choices regarding treatment
Q449-Child complain of unilateral scrotal swelling , does not transilluminate , what is your plan ?
Discharge the patient
Give Antibiotic
Do laparotomy
U/S and think about surgery*
answer :
Q450-a three weeks old boy presented with scrotal asymmetry. In examination both testicles were
palpable in the scrotum. (thats it, with no more information). What are you going to do next in examination:
trans-illumination test
let the patient to cry/cough to see if the is any bulging
answer :
Q451-after appendectomy a patient got abdominal infection by enterococcus faecium. He is allergic
to penicillin what are you going to give him:
ceftriaxone
vancomycin
cefotaxim
TMP-SMX or metronidazole (not sure which one of them was mentioned)

smle ,2016

687
answer :
Q452-sickle cell patient with hip pain ?
answer: A vascular necrosis
Q453-patient with foot pain ?
answer: Planter fasciits
Q454-Most comman Stian blood nipple discharge?
Answer:
Q455-patient got skin incision by a knife later he develops progressive pain, reddness and swelling,What is the diagnosis?
1. cellulitis
2. necrotizing fascitis
answer:
Q456-Older patient lifting heavy object with mass not reaching the scrotum?
Answer: Direct inguinal hernia
Q457-Case of female with hx of gallstones in the gallbladder she had 2. Attack which relived her
attack by analgisea,In the ct report,Low density 0.3 cm calcuali in the gallbladder What you will
perscribe
1. Imitadiben ( liped lowering agent )
2. Simvastarin
3. Cholydoxcoic
4. Urodoxcholic
"I asked radiologest consultant he said it is impossible to know the stone type from the ct"
answer:
Q458-case typically appendicitis
1. obstraction in appendecial wall
Answer: 1
Q459-Pt came to you with history of ingestion of 2 safety pin what is your action? "They didn specify where"
1. Surgery
2. Follow up with x-ray
Answer: Follow up with x-ray
Explanation: Objects with sharp edges or points present a special problem because of the possibility for
erosion or perforation. These include pins, needles, tacks, razor blades, pieces of glass, or open safety
pins. Children who have swallowed such objects should be vigilantly observed(Medscape)

smle ,2016

688
Q460-Pt. after tan bath presented with pain and swelling in chest and back :
1. 1st degree burn
2. 2nd degree
3. 3rd degree.
answer: 1st degree burn
Q461-Pt undergo laparoscopic hernioplasty then complains of respiratory depression.
Answer: Atelectasis
Q462-23 female with diffuse abdominal pain , vaginal exam normal with lower abdominal tenderness:
1. PID
2. cholecystitis
3. Appendicitis
Answer: PID
Q463-Pt with abdominal pain, nausea, vomiting, wt loss. On examination: palpable mass. Whats ur
action ?
1. Ppi
2. Follow up
3. Urgent referral
4. Ultrasound
Answer: Urgent referral
Q464-premature presented with abdominal distention and air on x-ray ?
1. Necrotizing enterocolitis
Answer: A
Q465-RLQ pain 9 out of 10 , mass felt tender on examination , US mass 6cm:
1. ectopic pregnancy
2. cystic rupture
3. appendicitis
4. cystic torsion
ANSWER:D
Q466Low intestinal obstruction , the 1st sx?!
1. Constipation?!!
2. Flatuase
ANSWER: 2
Q467-adult with breast mass the become large with menses came to you asking for dx Fna shows
yellowish secretion and it disapear after it what is dx:
1. Glactocele
2. Ductasia

smle ,2016

689
3. Normal variant
4. Nanovulatory
Answer:1 & 2 OUT CUZ first gives milk 2nd option gives blood stained
Q468-pt present with pleuritic chest pain, examination shows decrease air entry in the right lower
lung, attached chest x-ray showing radiolucent right costophrenic angle, what is next ?
1. Ventilation perfusion scan
2. Needle decompression
3. Chest tube
ANSWER:3
case with leg pain while walking, loss of hair in leg,nail changes , what is the cause ?
answer: peripheral vascular dx
Q469-patient with +ve Cullen sign and Grey-Turner sign what is the diagnosis ?
answer : acute pancreatits
Q470-patient with thyroid nodule and tortious dilated artery that compress the external laryngeal
nerve, which of the following will be affected
1. tone of the voice
2. lateral adduction of vocal cord
3. abduction of vocal cord
4. sensation above the vocal cord
answer :1
Q471-Heamorids are
1. Viens
2. Arteries
Answer: 1
Q472-40 years old women with biliary colick , what to give medically :
Answer: urodoxyuric acid
Q473-Right lower abdominal pain , nausea , guarding in examination :
Answer: appendecitis
Q474-male with right lower hypochondrium pain , in surgical exploration they found inflamed appendix , what is the congenital malformation responsible for this :
1.reverse rotation
2.non rotation
3.behind liver ..
Answer: non rotation
Q475-trauma with forearm streak long scenario , how to manage :

smle ,2016

690
Answer: Oral antibiotic If patient developed infection
Medscape:
3.1% rate of osteomyelitis in forearm fractures; both instances occurred in patients with massive crush
injuries Superficial infections respond well to appropriate antibiotics. The general principles of surgical
debridement and copious irrigation are key in treating deep infections
Q475-left side pain from lion to groin Dx:
1.Renal ston
2.Acute app
3.Acute pancrititis
4.Dudenal ulcer
Answer: renal stone
Reference : Medscape
The location and characteristics of pain in nephrolithiasis include the following:
Stones obstructing ureteropelvic junction: Mild to severe deep flank pain without radiation to the groin;
irritative voiding symptoms (eg, frequency, dysuria); suprapubic pain, urinary frequency/urgency, dysuria, stranguria, bowel symptoms

Stones within ureter: Abrupt, severe, colicky pain in the flank and ipsilateral lower abdomen;
radiation to testicles or vulvar area; intense nausea with or without vomiting
Upper ureteral stones: Radiate to flank or lumbar areas
Midureteral calculi: Radiate anteriorly and caudally
Distal ureteral stones: Radiate into groin or testicle (men) or labia majora (women)
Stones passed into bladder: Mostly asymptomatic; rarely, positional urinary retention
Q476-pt perform surgry to stamach that rapped her around the esophagus which nerve will be injured:
Answer: Vagal
Reference : medscape
Course of the vagus nerve : Vagus nerve branches in the abdomen The gastric branches (rami gastrici)
supply the stomach. The right vagus forms the posterior gastric plexus and the left forms the anterior gastric plexus. The branches lie on the posteroinferior and the anterosuperior surfaces, respectively.
Q477-child e elective oral surgery ,all lab with in normal on examination there is murmur 2/6 best
hear in left sternum or ( suprasternum ) NOT SURE , this murmur appear in sitting disappear in
supine . otherwise normal : what the appropriate inital step :
1.give antibiotics before surgery
2.postbone surgery
3.reassurance
4.some thing
Answer: reassurance
Refernce: Medscape
innocent murmurs and functional systolic ejection murmurs are mid- to late-systolic, medium- to high-

smle ,2016

691
pitched ejection murmurs audible at the left lower sternal border or the left second ICS, depending on
origin from the LV or RV outflow tract. These murmurs are usually blowing in character and grade I or
II/VI in intensity. They can vary in intensity with the bodys positioning and always end before the closure of the semilunar valves / AHA : Innocent murmurs do not require medication, do not create cardiac
symptoms, and do not mean that there is a heart problem or heart disease.

Q478-case of burn in ant.trunk & whole lower & upper limbs how much fluids we will give ?
Answer : we need to know his wight to answer .

Q479-case of truma pt unconscious e abdomen distension what intial ttt :


1.CT
2.iv fluid
3.fast
Answer: IV fluid
http://emedicine.medscape.com/article/434707-overview#a1
Q480-case of female underwent vaginal hystrectomy what the artery most be effected :
1.overian artery
2.femoral artery
Answer: overian artery
Q481-in patient post mastoctomy they do for her reconstrtion from the rectus muscle what is the
vesles my be injeried or effected
1.superior epegastric artery
2.inferior epigastric artery
Answer: superior epegastric artery

smle ,2016

692
http://emedicine.medscape.com/article/880615-overview#a11
Q482-Baby with vomiting and abdominal distention Examination gush of stool History of decrease
feeding and constipation, history of barium enema What next?
1.Colectomy
2.Hydrostatic reduction
3.Leveling colostomy
Answer: Leveling colostomy
http://emedicine.medscape.com/article/930411-treatment
Q483-abdominal solid mass ( renal i guess but not sure ) confirmed by
1. CT
2. MRI
3. US
Answer: US for detect renal mass , CT for perforation ((Step-Up to Medicine, 4E,p300))
Q484-pic of bowel obstruction with baby 2 week what is the treatment ?
1.primary surgery
2.leavler surgery
Answer: 1. 1ry surgery
http://emedicine.medscape.com/article/930411-treatment
Q485-most significant risk factor for OP?
Answer: Advanced age
http://emedicine.medscape.com/article/330598-overview#a4
Q486-mass moving with tongue protrusion?
Answer: Thyroglossal cyst
Q486-pt with thyroid goiter ,,,compress on external laryngeal nerve ,, what is the action affected:
1.tension of vocal cords
2.abuduction of vocal cords
3.adduction of vocal cords
4.Loss of sensation superior to vocal cords
Answer: tension of vocal cords
http://emedicine.medscape.com/article/1923100-overview#a3
Q487-VMA pt hemodynmaic collaps what is warning sing can presnent when he collaps ?
1.hypotion
2.rised jvp
3.deviation of trachea
4.resistance of ventlitor
Answer: hypotion.

smle ,2016

693

Q488-Senario,best site to insert the needle for liver biopsy:


1.Midaxillary at 6th intercostal space
2.Midaxillary at 7th intercostal space
3.Midaxillary at 10th
Answer: Midaxillary at 7th intercostal space
http://emedicine.medscape.com/article/149684-technique
Q489-Lady with a mass in left upper quadrant of the breast, aspiration was yellow fluid without
masses, Dx?
1.Phyllodem
2.normal tissue variant
"no simple cyst"
Answer:
http://www.mayoclinic.org/diseases-conditions/breast-cysts/basics/tests-diagnosis/con-20032264
490-do self brest ex every ?
1.month
2.3 month
3.6 month
Answer: month
http://www.cancer.org/cancer/breastcancer/moreinformation/breastcancerearlydetection/breast-cancer-earlydetection-acs-recs

491-women with mastitis.


1.stop breast feeding
2.clean nipple with alcohol
3.surgical drainage
ANSWER: MAY BE MISSED OPTION CONTINUE BREAST FEEDING
http://www.webmd.com/parenting/baby/tc/mastitis-while-breast-feeding-topic-overview
uro
Q492- Child after trauma in perineum region showed urine extravasation
Penile urethra
Prostate urethra
Ureter
Bladder
Answer: bulbar urethra is most commonly injured in perineum blunt trauma but since it is not in the answers, prostatic urethra will be the answer which is associated with pelvic fracture, Medscape
Q493-Patient has been treated four times by sulpha drug and there were lesions in the glans penis
and scrotum each time he was using that drugs ,what is the best explanation of this lesion ?
Vesicle
Patchy red nodule

smle ,2016

694
Redness of the ulcer with blister
Answer :
Fixed drug eruption
http://www.aafp.org/afp/2006/0101/p133.html
Q494-Baby in cystourethogram distended bladder and urethra with hydonephrosis distendid abdomin (Bully syndrom i think)
1. Uretrocysto reflex junction
2. Posterior urethral valve )the answer)
3. Polycyctic kidney
Answer:
Voiding cystourethrogram (VCUG) is the best imaging technique for the diagnosis of posterior urethral
valves. The diagnosis is best made during the micturition phase in a lateral or oblique views, such that the
posterior urethra can be imaged adequately 1,4. Findings include 1:
dilatation and elongation of the posterior urethra (equivalent of the ultrasonographickeyhole sign)
linear radiolucent band corresponding to the valve (only occasionally seen)
vesicoureteral reflux (VUR): seen in 50% of patients 1
bladder trabeculation /diverticula

Q495-Pt male with urine incontenence , go to the bathroom many times with urine retention O/e the
bladder was balapble , what is the incontenence type ?
1. Urge
2. Stress
3. Mixed
4. Overflow
answer:4
Types of urinary incontinence include:

Stress incontinence. Urine leaks when you exert pressure on your bladder by coughing, sneez-

ing, laughing, exercising or lifting something heavy.

Urge incontinence. You have a sudden, intense urge to urinate followed by an involuntary loss

of urine. You may need to urinate often, including throughout the night. Urge incontinence may be
caused by a minor condition, such as infection, or a more severe condition such as neurologic disorder
or diabetes.

Overflow incontinence. You experience frequent or constant dribbling of urine due to a blad-

der that doesn't empty completely.

Functional incontinence. A physical or mental impairment keeps you from making it to the

toilet in time. For example, if you have severe arthritis, you may not be able to unbutton your pants
quickly enough.

Mixed incontinence. You experience more than one type of urinary incontinence.

smle ,2016

695
Q496-old age pt c/o increase urination at night and weak urine stream lab result show PSA 1 , what
is your diagnosis ?
1. prostitis
2.
BPH
3. prostatic cancer
answer:2&3
http://www.hopkinsmedicine.org/healthlibrary/conditions/kidney_and_urinary_system_disorders/benign_prostatic_hyperplasia_bph_85,P01470/
Q497-old age pt c/o back pain and difficulty in urination lab results show PSA=80 and high
ALP,what is your diagnosis ?
1. prostatic cancer
2.
BPH
answer :1
http://www.urologyhealth.org/urologic-conditions/prostate-cancer
Q498-Injury to internal iliac in MVA what is affected structure.
1. bladder
2. ovaries
Answer: 1
Q499-A 70-year-old male with history of dysuria, frequency and urgency DRE revealed tenderness
but no masses, Temp high What is the most likely diagnosis?
1. Cystitis
2. Acute prostatitis
3. Rectal abscess
Answer: 2
https://en.wikipedia.org/wiki/Acute_prostatitis
Q500-Case of BPH , while prostatectomy, the doctor injured prostatic nerve , this patient will have
:
1.Erection inability
2.Urine incontinence
3.Stress incontinence
4.Stool incontinence
answer: 1
Oncology
Q501- child presented with something in the eye with lab result of increase of WBC what is
1. leukemia
2. neuroblastoma
Answer: It could be leukemia with eye infiltration but the answer could be in the 2 missing choices.
In neuroblastoma, there could be anemia or thrombocytopenia but not leukocytotsis.

smle ,2016

696
Q502-HIV patient ... (symptoms of intestinal obstruction )did intestinal resection,They found tumor white in color nearly encircling the wall,What is the tumor?
1. hodgkin
2. non hodgkin
3. Adenocarcinoma
4. plasmacytoma
Answer: B
REFERENCE : http://hivinsite.ucsf.edu/InSite?page=kb-04-01-11#S9X
http://library.med.utah.edu/WebPath/TUTORIAL/AIDS/AIDS088.html
Q503-Radio sensitive tumor:
Amswer: Seminomas << Yolk sac Embryonl carcinoma
Q504-Pt with metastatic breast ca presented with SOB , distended neck vain ,apex beat not palpable Distal heart sounds ? BP 70/... The best step to confirm the Dx is?
1.IV fursmide
2.IV fluid & urgent echo
Answer : convensional SV Cavography .
Inisial >> doplex US
Q505-Lung cancer 3cm Rt mediastinal LN (paratracheal) stage:
1.T1N1M
2.T1N2M<< Note: (N1=ipsilatral peribronchial N2=ipsilateral mediastinal 3.N3=contralateral mediastinal
Answer: T1N2M
Q506-Becks triad: hypotension, muffled heart sound, raised JVP ~> pericardiocentesis
Q507-Tension pneumothorax: needle decompression
pt with bladder cancer post surgery what to inject?
Answer: BCG
Q508-abdominal solid mass ( renal i guess but not sure ) confirmed by?
1. CT
2. MRI
3. US
ANSWER : A
Q509-Alcoholism , RUQ pain , jaundice , spleenomegly , ascites , caput medusa:
1.

PHT

2. hepatitis
3. pudd chiarri S
ANSWER : A

smle ,2016

697
Q510-Testicular tumor sensitive to radiotherapy?
1. Embryonal
2. Yolk sac
3. Seminoma
Answer: 3
Q511-H. 3.5 cm lung tumor with paratrachal lymph node?
1. T1 N 0
2. T2 N1 ( not sure N2 ?)
3. T3 N2
answer:
Q512-Old patient with back pain and difficulty in urination (Invistigation liver is normal,High
AlP,High PSA What is the diagnosis ?
1. Prostaitis
2. Prostate cancer
3. bPH
Answer: prostate ca with bone metastsis most likely
Q513-Marker of prostate ?
1. Alpha fetoprotine
2. Cea
3. Prostate acid phosphate
"No psa in the choices"
answer: Prostatic acid phosphatase (PAP), also prostatic specific acid phosphatase (PSAP), is anenzyme
produced by theprostate. It may be found in increased amounts in men who have prostate cancer or other
diseases.
Q514-Best prognostic indicator for breast cancer?
1. Estrogen receptor
2. Stage and grade
3. Involvement of axillary LN
Answer:
Q515-Lung Tumor, size: 3.5 cm, enlarged, ipsilateral bronchopulmonaryLN, what is the TNM
stage?
1. T1 N0 M0
2. T2 N1 M0
3. T3 N2 M0
4. T4 N3 M0
ANSWER: B
Q516-Question about thyroid cancer .

smle ,2016

698
"I remember that i choose hurthle cell adenoma but not sure if its the correct answer or not hurthle cell
adenoma":
http://emedicine.medscape.com/article/279462
Q517-upper eyelid cancer which LN mets
1. parotid
2. deep cervical
Answer:
source: http://www.eyeplastics.com/eyelid-anatomy-eyelid-blepharoplasty-or-eyelift-surgery.html
The lymphatic drainage of the eyelid is rather extensive. The majority of the upper eyelid and the outer
half of the lower eyelid drain into the pre-auricular lymph nodes, while a small part of the middle of the
upper eyelid and the inner half of the lower eyelid drains into the submandibular lymph nodes.
http://www.cancer.ca/en/cancer-information/cancer-type/eye/eye-cancer/eyelid-tumours/?region=on
Note: The regional lymph nodes include those around the ear (preauricular nodes), lower
jaw (submandibular nodes) and neck (cervical nodes).

Q518-Most specific predictor of local recurrence in breast cancer.


1. size.
2. ln mets. lymph node number
3. estrogen recep
4. progest resept
Answer:2
The most significant prognostic indicator for patients with early-stage breast cancer is the presence or absence of axillary lymph node involvement.
http://theoncologist.alphamedpress.org/content/9/6/606.full
Q519-Old woman with bone cancer originated from pelvic bone !!
1. Chondroblastoma
2. Sarcoma
3. ewing sarcoma
Answer: 1
Chondrosarcoma occurs most often in the pelvis
http://www.cancer.gov/types/bone/bone-fact-sheet
http://www.orthobullets.com/pathology/8023/chondrosarcoma
Q520-Old man smoker and alcohol drinker developed oral ulcer that bleeds when
Touching?
1. squamous cell carcinoma
2. aphthous ulcer
Answer: 1
http://www.cancer.gov/types/head-and-neck/patient/lip-mouth-treatment-pdq
https://www.merckmanuals.com/professional/ear,-nose,-and-throat-disorders/tumors-of-the-head-andneck/oral-squamous-cell-carcinoma

smle ,2016

699
http://www.ahns.info/resources/education/patient_education/oralcavity/
Q521-can't recall the senior very well but it was symptoms and sign of pancreatic disease that
caused thrombophlebitis ?
1.acute pancreatitis
2.pancreatic tumor
Answer: pancreatic tumor
Thrombophlebitis migrans can be a sign of malignancies such as pancreatic carcinoma (Trousseau sign of
malignancy)
https://en.wikipedia.org/wiki/Thrombophlebitis
Trousseau syndrome is also referred to as migratory superficial thrombophlebitis, carcinogenic thrombophlebitis, and tumour-associated thromboembolism.
Pancreatic cancer appears to be associated with the highest risk of Trousseau syndrome, but other tumours, particularly adenocarcinomas (cancers that develop in the lining or inner surface of an organ) can
also cause the syndrome. Lung cancers are commonly reported.
http://www.dermnetnz.org/systemic/trousseau.html
Q522-femoral tumors contain tumors and calcification
1. osteosarcoma
2. ewing
answer: 1
Calcifications or mineralization within a bone lesion may be an important clue in the differential diagnosis.
There are two kinds of mineralization: a chondroid matrix in cartilaginous tumors like enchondromas and
chondrosarcomsa and an osteoid matrix in osseus tumors like osteoid osteomas and osteosarcomas.
http://www.radiologyassistant.nl/en/p494e15cbf0d8d/bone-tumor-systematic-approach-and-differentialdiagnosis.html
Q523-Vulvar carcinoma commonest present
1.labia majora mass.
2.cltoris mass
Answer: 1
On physical examination, the vulvar lesion is usually raised and may be fleshy, ulcerated, leukoplakic, or
warty in appearance. Most squamous cell carcinomas are unifocal and occur on the labia majora
source: http://www.aafp.org/afp/2002/1001/p1269.html
Q524-biopsy of lung found some material in the macrophage ( i cant remebre the material )
Q525-(PCP )pneocyteic jnera
1.CMV
2.Bacterial
3.HF
Answer: HF
Q526-case of hyperthyroism clear but with brady cardia

smle ,2016

700
1. hypo thyrsism
2.hyprethyerim
3.goiter
4.neoplasm
Answer:
Q527-vit c difeciency which affected in stages of wound healing?
Answer: Collagen synthesis
Vitamin C is essential to the formation of new connective tissue in a healing wound. The important component in healing is collagen, which is comprised of the amino acids lysine, proline, and glycine. Collagen forms the structure of the connective tissue that becomes the framework around which the new tissue
is rebuilt. The enzymes critical to forming collagen cannot function without their co-factor, which is vitamin C.
http://www.surgerysupplements.com/the-role-of-vitamin-c-in-wound-healing/
Q528-25 y o woman came to the clinic with her 6 weeks old baby, complaining of irritability ,
weight loss, and inability to sleep?
1.post Partum thyroditis *
2.hyperthyroidism
3.hashimoto thyroditis
Answer: post partum thyroiditis
The thyrotoxic phase occurs 1-4 months after delivery of a child, lasts for 1-3 months and is associated
with symptoms including anxiety, insomnia, palpitations (fast heart rate), fatigue, weight loss, and irritability
http://www.thyroid.org/wp-content/uploads/patients/brochures/Postpartum_Thyroiditis_brochure.pdf
Q529-case of renal artery stenosis (investigation)
Answer:

Serum creatinine to estimate GFR.

Serum potassium: hypokalaemia or low-to-normal potassium may suggest activation of the


renin-angiotensin-aldosterone system.

Urinalysis and sediment evaluation (to exclude glomerular disease): RAS, in the absence of coexistent diabetic nephropathy or hypertensive nephrosclerosis, is typically non-proteinuric without abnormalities in the urinary sediment.
ultrasonography offers a safe, non-invasive assessment, its sensitivity and specificity are low, and its use
provides only indirect evidence of the presence of stenosis. Other non-invasive techniques (i.e., CT angiography or MR angiography) have a risk associated with the use of contrast media (radiocontrast
nephropathy and nephrogenic systemic fibrosis, respectively). Conventional angiography, despite its procedural risk (e.g., atheroemboli, bleeding) and the risk of radiocontrast nephropathy, has the advantage of
being able to determine the clinical significance of the lesions by measurement of the pressure gradient
across a stenotic lesion, and the possibility of concurrently performing endovascular therapy.
It is recommended to start with a non-invasive imaging test in patients with a high clinical probability of
RAS.

smle ,2016

701
If the results of non-invasive tests are inconclusive and the clinical suspicion for RAS is high, invasive
testing is recommended.

Duplex ultrasound (sensitivity 84% to 98%, specificity 62% to 99%). Can identify discrepancy
in kidney size, velocity of renal blood flow, and resistive index. Ultrasound diagnostic criteria for
significant renal artery stenosis are:
o Renal artery to aorta peak systolic velocity ratio (renal-aortic ratio) >3.5
o Peak systolic velocity >200 cm/sec with evidence of post-stenotic turbulence
o End-diastolic velocity >150 cm/sec (>80% renal artery stenosis) when present with a peak
systolic velocity of >200 cm/sec
o Renal resistive index >0.8 (sometimes used to predict response of blood pressure or kidney
function for revascularisation).

Gadolinium-enhanced magnetic resonance angiography (sensitivity 90% to 100%, specificity


76% to 94%).

CT angiography (sensitivity 59% to 96%, specificity 82% to 99%).


Invasive testing
Conventional angiography:

The most sensitive and specific test for assessing anatomical narrowing of the renal artery.

Also allows for therapeutic intervention at the same time.

Requires arterial catheterisation.


http://bestpractice.bmj.com/best-practice/monograph/435/diagnosis/step-by-step.html
Q530- s.s of hyperthyroid wh present ?
retraction led
prietabial myxedema
Answer: retraction led
http://www.uptodate.com/contents/hyperthyroidism-overactive-thyroid-beyond-the-basics
Q531-How can stimulate breast feeding secretion ?
1.Breast feeding
2.Increase fluid intake
3.Increase caloric intake
Answer: breast feeding
During the first few weeks, the more a baby suckles and stimulates the nipple, the more prolactin is produced, and the more milk is produced. This effect is particularly important at the time when lactation is
becoming established.
http://www.ncbi.nlm.nih.gov/books/NBK148970/
Q532- case with shock baroreceptors is activated and his is having tachycardia which of the following will be decreased?
a.
Heart rate
b.
Venous capita
c.
Coronary blood flow
d.
Something irrelevant
Answer: b
Reference: http://www.datasci.com/solutions/cardiovascular/baroreflex

smle ,2016

702
When a person has a sudden drop in blood pressure, for example standing up, the decreased blood
pressure is sensed by the baroreceptor as a decrease in tension therefore will decrease in the firing
of impulses. This causes the vasomotor center to uninhibit sympathetic activity in the heart and
blood vessels and decrease vagal tone (parasympathetic influence on the cardiac SA node) causing an increase in heart rate. The baroreflex responds to acute changes in blood pressure.

Q533-patient complaining of hematuria with WBC in urine and the culure is negative. Cystoscopy,
revealed submucosal hemorrhage what is the cause:
a.
Cystolithiasis
b.
interstiatial cystitis
Answer: b
Reference: http://www.aafp.org/afp/2001/1001/p1199.html
http://www.merckmanuals.com/professional/genitourinary-disorders/voiding-disorders/interstitial-cystitis
Q534-A 30-year-old male presented with severe headache, never had like this headache before, photophobia, neck rigidity. Intracranial hemorrhage was suspected.Where is the most likely site of this
hemorrhage?
a.
Intracerebral
b.
Subarachnoid
c.
Epidural
Answer: b
Reference: http://www.merckmanuals.com/professional/neurologic-disorders/stroke-cva/subarachnoid-hemorrhage-sah
Q535-A 40-year-old male, 2 years history of difficulty swallowing and lump sensation in throat, excessive salivation, intermittent hoarseness, weight loss:
1. Achalasia
2. Scleroderma
3. Diffuse esophageal spasm
4. Cricopharyngeal dysfunction
Answer: 4
Reference: http://emedicine.medscape.com/article/836966-overview#a10
Q536-Which Ligament when you do epidural anaesthesia?
Answer: the Q is not complete ?
Reference: http://www.usmle-forums.com/usmle-step-1-mnemonics/1943-mnemonic-lumbarpuncture-layers.html
Mnemonic of lumber puncture layers
SLOW FAHAD SEE IT`S LUMBER EPIDURAL DISSECTION.......
1- Skin
2- Facia and SC fat
3- Surpaspinous ligament
3- Interspinous ligament
5- Ligamentum flavum
6- Epidural space and fat (epidural anesthesia needle stops here)
7- Dura

smle ,2016

703

Q537-Lipid lowering agent can case sever muscle weakness?


Answer: Statins
Reference: http://www.surgerysupplements.com/the-role-of-vitamin-c-in-wound-healing/
Q538-Best muscle relaxing agent?
Answer: ?
Reference: http://www.spine-health.com/treatment/pain-medication/muscle-relaxants
Q539-4 weeks old infant, mother happy he never cries. on examination: Jaundice + Umbilical hernia + Distended Abdomen + Coarse face features + Bulging frontal fontanel Diagnosis?
1. Congenetal Hypthyroidism*
2. Gilbert's Syndrome
3. Cerebral Palsy
4. Rickets
Answer: a
Reference: http://emedicine.medscape.com/article/919758-overview
Q540-Characteristic sings of nephritis syndrome?
1. HYPERTENSION
2. Hyperlipidemia
3. Hypoalbuminia
4. Edema
Answer: 3 & 4
Reference: http://www.uptodate.com/contents/the-nephrotic-syndrome-beyond-the-basics
Q541-women with abd pain they perform laparatomy then a darck blood when they open (i do not
remmeber if ther is by latral adenxial mass or not)
What is the Dx:
Answer: Choclet hemorhagic cyst
Reference: http://radiopaedia.org/articles/haemorrhagic-ovarian-cyst
Q542-ORAL hyperglycemic drugs case hypoglycemia
Q543-CT in truma important for :

A- to detect diaphragmatic injury
B- retroperitoneal injury
Answer: b
Reference: http://www.ncbi.nlm.nih.gov/pmc/articles/PMC3216157/pdf/mjms-17-2-029.pdf
Q544-Patient with ulcer on the nose. Diagnosis?
1.Squamous carcinoma.
2. Basal cell carcinoma.
Answer: B

smle ,2016

704
The most characteristic symptom is the suddenness of the onset of epigastric pain. The pain rapidly becomes
generalized although occasionally it moves to the right lower quadrant. hyperresonant percussion on liver
>> new information
Reference: http://www.cancer.org/acs/groups/cid/documents/webcontent/003139-pdf.pdf

Q545- prostatic ca w/ back pain investigation?


back sincitograph
back CT
Answer: he has mets so it should be CT so 2
Toronto notes
Reference: http://emedicine.medscape.com/article/1967731-workup
Q546-case of patient female 50 YO travelled for 12 hours, at the end of the flight she could
not wear her shoes , what investigation should you do ?
Note: This is most probably edema due to DVT
A. Pelvic CT
B. Abdominal US
C. Compression US
Answer: C
Reference: uptodate
http://www.uptodate.com/contents/deep-vein-thrombosis-dvt-beyond-the-basics
Q547-case scenario of intussusception what is the gold stander investigation :
A. abdominal CT
B. CXR- barium study
C. abdominal X-ray
Answer: non of the above, ultrasonography is the most sensitive and specific followed by abdominal X-ray
Reference: Medscape
http://emedicine.medscape.com/article/930708-workup#c8
Q548-18 boy play basketball he came with abdominal pain without any injury in match
physical exam was tenderness in paraumbilical area what to do next:
A. Chest xray
B. abdominal CT

smle ,2016

705
C. 24 hr recheck
D. kidney US
Answer: B .. this could be spleen injury * not sure
Q549-carpal tunnel syndrome. pt. can't work and write. which muscles affected?
A.
thenar eminence muscle
B.
palmar interossei muscle
C. dorsal interossei muscle
Answer: A (median nerve)
Reference: orthopedics consultant

A.
B.
C.
D.

Q550- A patient with unilateral parotid swelling, hes post - cholecystectomy. Saliva was
cloudy ( I think) Culture from parotid saliva was negative
Sarcoid granulo
Bacterial
Cancer
Sjogren syndrome
Answer: B
Postsurgical acute suppurative parotitis is a bacterial gland infection that occurs from a few days
up to some weeks after abdominal surgical procedures
in elderly pt with poor dental hygiene & have been intubated. S.aureus is the org. surgical drainage & abcs is the Rx
Reference: http://www.ncbi.nlm.nih.gov/pubmed/18949350
Q551-DUKE criteria:
Reference: Kumar & Clark's

smle ,2016

706

Q552-Yollow pt all normle only high indirect biliurbin


and alkain phsphatase? What the diagonsis
Answer: Extrahepatic obstruction
http://emedicine.medscape.com/article/187001-workup
Q553-Renal or small cell cancer stage III with bone
pain what is the immediate action?
A. MRI only
B. Radiothyrpy
C. Iv steriod and MRI
D. No imediate action
Answer:A
Maybe change to stage IV
Similar question in another exam with different choices:
Q554-Acute prostatitis presents with dysuria with:
Perineal pain

smle ,2016

707
Tender prostate on examination
The diagnostic yield of urine culture is greatly increased with prostate massage.
Treat in the same way as you would for pyelonephriti:
-Ceftriaxone, ertapenem
-Ampicillin and gentamicin until culture results are known
-Ciprofloxacin (oral for outpatient)
Long-term therapy with TMP/SMZ for 6 to 8 weeks is used for chronic prostatitis.
- Master the board

Q : non small cell lung cancer has 4 risk factors which are stage of the disease , condition of the patient
and :
Answer: male sex
Q555-Most common site of metastasis:
A .Bone
B .Kidney
C.Breast
Answer: A
The most common sites of cancer metastasis are, in alphabetical order, the bone, liver, and lung.
http://www.cancer.gov/about-cancer/what-is-cancer/metastatic-fact-sheet#q3
Q556-Young patient presented with ptosis miosis depressed orbit and in the other eye there's something he also has a neck mass what is the most likely diagnosis:
Hodgkin's lymphoma
Ewing's sarcoma
Wilms tumor
answer: B
https://books.google.com.sa/books?id=q0oKjWH2-agC&pg=PA863&lpg=PA863&dq=ewing%27s+sarcoma+ptosis&source=bl&ots=jFabbLuLj&sig=K7kq2CfbG1CNnGUjlop6wfCy9XQ&hl=ar&sa=X&ved=0ahUKEwjR0MLMr9PNAhVEuBoKHddLBrAQ6AEIQDAF#v=onepage&q=ewing's%20sarcoma%20ptosis&f=false
if said side effect of ttt it will be: c
http://internationalscholarsjournals.org/download.php?id=936489544622946824.pdf&type=application/pdf&op=1
Q557-Child presented with burn in the upper right extremity with bluster what is the degree of the
burn
2nd degree more than 15 %
2nd degree less than 15 %
3rd degree more than 15 %
3rd degree less than 15 %
Answer: 2nd degree less than 15 %

smle ,2016

708

Q558Sparing and dissecting which artery during flap of rectus muscle


Inferior epigastric
Superior epigastric*
Answer: B
http://www.microsurgeon.org/rectus

smle ,2016

709

Orthopedics

smle ,2016

710

1- Military soldier with flat foot. Which of the following will be sustained?
Flexor retinaculum
Extensor retinaculum
Spring ligament
Achilles tendon
Answer: C
Medial arch (navicular) injury: Injuries to the navicular bone and related structures jeopardize the structural integrity of the medial foot arch and thus can lead to severe disability and chronic pain. Therefore,
navicular injury is considered high risk, particularly for runners. Such injuries include navicular stress fractures, tendinopathy of the posterior tibialis tendon insertion onto the navicular, traumatic separation of
an accessory navicular, and partial or complete tears of the attachment of the plantar calcaneonavicular
(spring) ligament. Unless the clinician has experience managing injuries of the medial arch, consultation
with a foot and ankle surgeon should be obtained for navicular injuries, including fractures or separation
of an accessory navicular, as well as for complete or partial ruptures of the posterior tibialis tendon or the
spring ligament, as such injuries often compromise the integrity of the longitudinal arch.
Reference: http://www.uptodate.com/contents/evaluation-and-diagnosis-of-common-causes-of-footpain-in-adults#H13378296
2- Injury to the surgical neck of humerus cause injury to :
Radial A
Median
Ulnar
Brachial plexus
Answer: D
- This type is proximal humerus injury
- A, B and C go with distal and humeral shaft injury. Assessment: Perform a neurological examination,
particularly examining the axillary nerve Assess and brachial plexus injury through distal neurological examination.
3- A patient was diagnosed with scoliosis, based on the cobb angle, when to do an orthopedic referral?
> 5 degrees
> 10 degrees
> 15 degrees
> 20 degrees
Answer: B
Reference: Toronto notes.
4- Patient with septic arthritis the labs showing resistance to the antibiotic what u will do ?
stop antibiotic
start vancomycin
add vancomycin
gentamicin
Answer is B
http://emedicine.medscape.com/article/236299-medication#2

smle ,2016

711
5- High density bone in dexa With scheduled total hip replacement
osteoporosis
osteomalacia
osteoarthritis
paget disease
Answer: c. C osteoarthritis
Bone density is actually HIGHER rather than LOWER in osteoarthritis. Low bone density is the telltale
sign of osteoporosis, a skeletal disorder characterized by weakened bones due to excessive loss of bone
mass.
Osteoarthritis, on the other hand, is characterized by increased bone density and bony growths (osteophytes) in conjunction with articular cartilage degeneration
Reference:
http://www.orthop.washington.edu/?q=patient-care/articles/arthritis/osteoarthritis.html
Artefactual causes of raised BMDno true increase in bone mass
OA
DISH
AS
Vertebral fractures
Vascular calcification
Thalassaemia major
Abdominal abscesses
Gallstone
Renal calculi
Gluteal silicon implants
Gauchers disease
Intestinal barium
Surgical metalwork
Laminectomy
Vertebroplasty and kyphoplasty
http://www.ncbi.nlm.nih.gov/pmc/articles/PMC3651616/
6- Pt with long scenario had trauma to knee..the knee displaced to anterior ..which structure is
injured?
anterior cruciate ligaments
posterior cruciate ligaments
medial meniscus
lateral meniscus
Answer : a
Pivot shift test: The pivot shift test is performed by extending an ACL-deficient knee, which results in a
small amount of anterior translation of the tibia in relation to the femur. During flexion, the translation
reduces, resulting in the "shifting or pivoting" of the tibia into its proper alignment on the femur. It is
performed with the leg extended and the foot in internal rotation, and a valgus stress is applied to the
tibia.
Ref :http://emedicine.medscape.com/article/89442-overview
7- Osteoporosis, how to prevent spinal compression fx ?
Daily vit D
Aerobic exercise

smle ,2016

712
Weight bearing
Walking
Answer: A
''Physical therapy is recommended for gait and core strengthening when the patient can tolerate this
level of activity'' http://www.uptodate.com/contents/osteoporotic-thoracolumbar-vertebral-compression-fractures-clinical-manifestations-and-treatment?source=search_result&search=osteoporotic+vertebral+compression+fracture&selectedTitle=1~150#H1565673
8- Man make RTA he was conscious , oriented , alert , but his extremity cold, type of shock ?
irreversible
neuro
cardio
hemorrhagic
Answer: Cold extremity most likely Hemorrhagic
Reference : Toronto note
9- Minutes of X Ray radiation?
25
15
5
0
Answer: 15 ? Q is not clear but the following may help ,
AIRPORT PASSENGER SCREENING There has been increasing concern regarding radiation exposure from
whole-body transmission or backscatter radiograph scanners at airport security checkpoints that have
been deployed in some airports across the United States. Reports calculate the dose emitted from
backscatter radiograph scanners as being equivalent to approximately three to nine minutes of radiation
received from the environment through normal daily living [79,80]. Put another way, among 100 million
annual passengers, only six cancers may be attributed to airport radiograph screening over the lifetime of
these individuals. Current evidence thus suggests that there is negligible individual risk from airport passenger screening [81]. Millimeter-wave scanning is a similar technology without the use of ionizing radiation which may become a comparable technology and preferable alternative to radiograph airport passenger screeners
Reference: Uptodate
10- Pt. Presented with deformity in the hand, x ray showed fracture in the radius, they put picture of x
ray and the hand. How you will fix it ?
Close reduction ... below elbow
Closed reduction. ... above elbow
Open reduction ... below elbow
Open reduction above elbow
Answer: B

smle ,2016

713

Reference: http://www.uptodate.com.ezp.uod.edu.sa/contents/distal-radius-fractures-inadults?source=search_result&search=fracture+of+radius&selectedTitle=3~150
11- Football player came with + anterior drawer test, where is the injury:
ACL
PCL
Medial collateral
Lateral collateral
Answer is: A
Reference: http://physicaltherapyweb.com/anterior-drawer-test-orthopedic-examination-knee/
12- picture was provided for 4 ys old boy presented with distal ulnar and redial bone fracture the skin
was slightly injured by the bone what is the management1??
closed reduction with cast below elbow
closed reduction with cast above the elbow
open reduction with plaid
Aggressive debridement and irrigation and Fracture stabilization
Answer:D
Reference: http://www.uptodate.com.ezp.uod.edu.sa/contents/distal-forearm-fractures-in-childreninitial-management?source=search_result&search=fracture+of+radius+treatment&selectedTitle=3~150
13- What is the most accurate test for carpal tunnel syndrome:
Tinel
Compression test
Durkan's carpal test
Phalen's test
Answer: B
Durkans carpal test = Compression test
http://www.orthobullets.com/hand/6018/carpal-tunnel-syndrome
14- Pt after RTA , no abduction and lateral rotation of the arm.. . What is the origin of the affected
nerve ?
Medial plexus

smle ,2016

714
Lateral plexus
Lower plexus
Root
Answer: Question not clear, the above scenario shows injury to axillary nerve (C5-C6) from the posterior
cord. D?
15- Old patient with back pain. Spine DEXA results is provided below:
Cervical 0.05
Thoracic <-1.5
Lumbar <-2.8
Sacral <-1.3
What is the diagnosis?
Osteopenia
Severe osteopenia
Osteoporosis
Established osteoporosis
Answer: D (Osteoporosis itself is asymptomatic)
up to date http://www.uptodate.com/contents/image?imageKey=ENDO%2F53999&topicKey=ENDO%2F2056&source=see_link
16- Tibial fracture, next step?
CT
MRI
Angiography
X-RAY
Answer: D
The standard protocol is to obtain anteroposterior and lateral radiographs of the injured leg. Reference:PubMed
N.B: Tibial plateau fracture CT is Mandatory. (To check the knee joint)
17- Mother and father were holding their son from his hand and they elevate them over the ground,
then the child complain of pain in his elbow he was flexing his arm with pronation he didn't let anybody
touch it .What was the injured ligament?
quadrant ligament
annular ligament
sheet between radius and ulnar bone
something else..
Answer: B
http://orthoinfo.aaos.org/topic.cfm?topic=A00717
18- hyperextension injury developed pain in distal phalanx, tender palms, can't flex distal phalanx :
Rupture superficialis tendon
Rupture Profundus tendon,
Extraarticular fracture of distal phalanx
Intra Articular fracture of middle phalanx
ANSWER: B
NOTE: The flexor digitorum profundus flexes the distal interphalangeal joint; the flexor digitorum superficialis flexes the proximal interphalangeal joint

smle ,2016

715

19- Postmenopausal with fractures and osteoporosis, what is the best drug?
Calcitonin
Calcitriol
Alendronate
Estrogen
Answer : C (bisphosphonates is management for post-menopausal osteoporosis)
20- carpal tunnel syndrome, how do you splint the hand?
Dorsiflexion
Plantarflexion
Extension
Abduction
Answer: A (Relieve pressure)
21- Pt can't do abduction at hip joint, site of injury?
gluteus maximus
inferior gluteal artery
head of femur
can't remember
Answer : A
22- Football player was examined by a doctor, on supine position with partially flexed left knee, the
doctor pushes the leg toward his body, which structure he was examining?
ACL
PCL
Lateral collateral ligament
Medial collateral ligament
Answer : B
23- 10 years old child complaining of pain in tibial tubercle he has just had growth spurt. what is the
cause of the pain:
Osgood schlatter disease
legg-perthes-canal disease
stress fracture
tibial splint
Answer: A
24- patient has left leg trauma, you did for him plaster of paris cast , after 5 hours come to ER complain of leg pain. Which of following support your diagnosis :
Decrease pulse in L foot
Decreased sensation in L foot??
Loss of movement
Pain out of proportion in L foot
Answer: D
pain out of proportion to clinical situation is usually first and the most sensitive symptom of compartment syndrome " (Paralysis and pulse loss are late findings)
http://www.orthobullets.com/trauma/1001/leg-compartment-syndrome

smle ,2016

716

25- Golfer with symptoms of golfer's elbow + Difficulty in flexing the wrist. What is your next step?
Refer to orthopedician
Refrain golf
Do an x-ray
Suggest surgery
Answer: B
26- Football player had injury from the lateral side of his knee, and found some laxity on examination
to medial knee, what is the injury:
medial meniscus
medial collateral ligament
lateral meniscus
lateral collateral ligament
Answer: B
27- Pt with trauma, femoral/hip fracture. Lt leg short and laterally rotated. What muscle is responsible
for lateral rotation:
gluteus maximus
rectus femoris
gracilis
adductor magnus
Answer: A
The lateral rotators are: the superior gemellus, inferior gemellus, obturator externus, obturator internus, quadratus femoris, gluteus maximus and the piriformis.
28- 37 years old presented with back pain. On examination there was tenderness when palpating
paraspinal muscles, neurovascular exam was normal. What is the treatment?
physiotherapy
biofeedback
cortisone
surgery
Answer: A
29- Girl fall on stretched out hand on xray open fracture of radius and ulna what is the treatment
internal fixation
external fixation
Drainage and fixation (did not mention type of fixation in this choice)
antibiotic
Answer: C
30- Man was hit by a car cleared by trauma team vitaly stable with pic showing open fracture of tibia
and fibula the other pic there is bleeding in the shin how to manage
external fixation
internal fixation
drainage with fixation (did not mention the type of fixation in this choice )
IV antibiotic
Answer B

smle ,2016

717

31- What is the commonest nerve injury with fracture humerus? (It is not mentioned whether it is
fracture neck or shaft)
radial
ulnar
median
axillary
Answer: D
32- Football player on ex. Supine position + left knee flexed Doctor pull his left leg toward his body
(doctor body ).. What examine?
ACL
PCL
Collateral fibular
Collateral tibia
Answer: A
33- Injury to the lateral side of knee, positive valgus stress test, swelling on the medial side of the
knee, injured structure is?
Medial collateral ligament
Lateral collateral ligament
Medial meniscus
Lateral meniscus
Answer: A
34- non athlete heard a click at the left leg calf when he forcefully plantarflex his left foot while the
leg is fully extended.. On examination, he can't plantar flex his foot against resistance & can't walk by
toe-foot walking of the affected site.. What is the injured structure ?
Calcaneal ligament
Quadriceps ligament
Plantar ligament
Femoris rectus
Answer: A (calcaneal tendon = achilles tendon)
35- Lachman test positive indicate:
ACL
PCL
Medial meniscus
Lateral meniscus
Answer: A
37- Young boy was playing sports with his friends, then presented to hospital with left shoulder pain,
the arm is adduction with internal rotation. This boy has dislocation in which area?
Inferior
Subglenoid Anterior
Subacromial Anterior
Supracondylar Posterior
Answer: B

smle ,2016

718
Reference: http://shoulderdislocation.net/anatomy/dislocated-anatomy
38- most specific sign of osteoporosis ?
Increase alp??? fracture
Increase calcium
Calcification
Decreased bone matrix
Answer: A
39- pt fall on his elbow what will you see in lateral x ray: ( no other info)
radial line anterior to acetabulum
some line bisect with acetabulum
anterior fat
posterior fat
Answer: D (NOT CLEAR)
40- fracture in the humerus and ulna , the ptn cant extend his arm , wrist and fingers what the nerve
affected ?
Radius nerve in the spiral groove
Median nerve in supepichondile region
Median nerve in .
Ulnar nerve in the medial epicondyle
Answer: A
41- footballer player came to doctor with pain in the knee cause the other player hit him from the side
of the knee , valgus test is positive ?
Fibula collateral
Tibial collateral
ACL
PCL
Answer: A (MCL)
42- Women work on computer for long time as typer she developed pain and numbness over the hand
Pain at rt hand palmar arch test showing insufficient blood flow which could be injured. What the artery occluded?
Anterior interosseous
Posterior interosseous
Ulnar
Radial
Answer: C (Not sure)
43- case of carpal tunnel and difficulty in movement of fingers, what muscle is affected ?
Medial lumbrical
Dorsal interosseus
Palmar interosseus
Thenar
Answer: D (Lateral lumbricals are also affected)

smle ,2016

719
44- GCS Bone sac showed osteolytic femur lesion and circumstances skull(something) , headache Labs
ca Normal only they wrote acid phosphatase is high ?
Osteoporosis
Osteomalacia
Paget
Hyperthyroidism
Answer: C
45- Occipital headache and neck pain can't do lateral flexion weakness of deltoid bilaterally on x-ray
osteophyte
neck strain
cervical spondylitis
brachial plexus neuropathy
Answer: C (Spondylosis not litis)
46- Female pt , injured her left index, so now she unable to flex the distal part of the finger associated
with swelling, the structure injured is?
Fracture to the distal phalanx bone
Fracture to the proximal phalanx bone
Torn digitorum profundus
Torn digitorum superficialis
Answer: C
47- A patient presented with Trendelenburg gait. The hip is slightly to the right . Which structure is responsible?
gluteus medius
gluteus Maximus muscle
inferior gluteal nerve
obturator nerve
Answer : A
http://www.uptodate.com/contents/overview-of-hip-pain-in-childhood
48- Pt obese came with pain if rising arm cant rise above shoulder Numbness in ulnar distribution
ulnar artery thrombophlebitis
thoracic outlet syndrome
carpal tunnel syndrome
Answer: B
49- Adult with mid-clavicular fracture. X-ray showed overlapped bone fragments. How will you
treat?
internal fixation
external fixation
splint
Answer: A
http://www.orthobullets.com/trauma/1011/clavicle-fractures
50- Blood supply of post. Leg compartment:
tibial

smle ,2016

720
Common fibular
Superficial fibular
Answer: posterior tibial artery
Explanation:The posterior tibial artery is the larger of the terminal branches of the popliteal artery. It
descends deep to soleus, where it can be exposed by splitting gastrocnemius and soleus in the midline,
then becomes superficial in the lower third of the leg and passes behind the medial malleolus between
the tendons of flexor digitorum longus and flexor hallucis longus. posterior compartment has two parts (
superficial and deep )
Superficial contains : Gastrocnemius , soleus, plantaris. Deep contains : popliteus, flexor hallucis longus,
flexor digitorum longus and tibialis posterior
Reference: Review of orthopedics by Miller + Gray's anatomy
51- A child with avascular necrosis of the head of femur. What should be done?
surgical correction
keep immobile for 6 weeks
keep externally rotated
Answer: A
Reference: http://emedicine.medscape.com/article/86568-treatment
52- What is the type of fracture that has several segments in the diaphysis?
compound
impacted
comminuted
Answer: C
Reference: The pathophysiology of fractures Textbook.
53- Trauma to the knee ABI ( ankle branch index) was low next step?
Angio Doppler
CT
MRI
Answer: A
54- A patient with Tibial fracture with abnormal ABI. What will you do
next?
MRI
CT
Angiography
Answer: C
Tibial fracture with ABI/PVR abnormal, Needs vascular team involvement and urgent vascular imaging
(CT angiography)
Reference: Tibial Plateau Fractures Initial Management Guidelines. See: http://www.medicine.virginia.edu/clinical/departments/orthopaedics/orthopaedic-education/residency%20programs/treatment-protocols/Tibial_Plateau_Fractures_2013_Initial_Management_Guidelines.pdf
55- Old lady complaining of right femur pain when she goes upstairs and downstairs. Examination of
the right hip: flexion, extension and rotation are normal but abduction caused pain. What is the best
investigation to do ?

smle ,2016

721
Radiological assessment.
Blood testing.
Rheumatoid factor.
Answer: A
(Pain while abduction of rt hip) Most probably to be posterior hip dislocation because all movements
are intact (flexion internal rotation...) except abduction so we need radiological views to confirm the diagnosis
Reference: Netters concise orthopedic anatomy
56- Football player he can't raise his big toe. what muscle affected:
Deep muscle under foot
Superficial muscle under foot
Tibia
Answer: A
(Pain while raising up the big toe) Deep muscles under foot responsible for dorsiflexion of big toe most
probably the diagnosis will be turf toe, which is common in football players (it sprains to the ligaments
around the big toe)
Reference: Millers review of orthopedic
57- Child coming with fracture of the forearm (x-ray picture show open fracture of radius & ulna),
what is the treatment?
Closed reduction
Open reduction
Surgical intervention
Answer: C
58- picture of tibial fracture proximal peripheral perfusion normal and normal sensation the best next
to investigate?
angiography
MRI
doppler us
Answer: B (To observe knee joint)
59- What is the most specific sign for osteoporosis ?
bone density
subchondral cyst
compression fracture
Answer: C
Fractures are the most dangerous aspect of osteoporosis. Debilitating acute and chronic pain in the elderly is often attributed to fractures from osteoporosis and can lead to further disability and early mortality.[14] These fractures may also be asymptomatic. The most common osteoporotic fractures are of the
wrist, spine, shoulder and hip. The symptoms of a vertebral collapse ("compression fracture") are sudden back pain, Reference: https://en.wikipedia.org/wiki/Osteoporosis
60- What type of radial nerve injury?
neuropraxia
neurotmesis

smle ,2016

722
axonotmesis
Answer: A
i think depend on scenario , if Q mean closed fracture most likely the answer will be neuropraxia
Ref: http://www.orthobullets.com/trauma/1015/proximal-humerus-fractures
to know what is diffrence btw these type look the link https://jammyjasmine.wordpress.com/2015/04/06/dont-panic-a-laypersons-guide-to-nerves-and-nerve-damage/
Radial nerve injury association with humeral fracture :
proximal humeral fracture :
Nerve injury
axillary nerve injury (up to 58%)
increased risk with anterolateral acromial approach
axillary nerve is found 7 cm distal to the tip of the acromion
suprascapular nerve (up to 48%)
humeral shaft
Radial nerve palsy
Incidence seen in 8-15% of closed fractures
increased incidence distal one-third fractures
neuropraxia most common injury in closed fractures and neurotmesis in open fractures
Holstein-Lewis fracture
a spiral fracture of the distal one-third of the humeral shaft commonly associated with neuropraxia of
the radial nerve (22% incidence)
distal humeral :
ulnar nerve
resides in cubital tunnel in a subcutaneous position below the medial condyle
radial nerve
resides in spiral groove 15 cm proximal to distal humeral articular surface
between brachioradialis and brachialis proximal to elbow divides into PIN and superficial radial nerve at
level of radial head
61- Patient complaint progressive wrist pain since 2 months and increased in the past 1 month , give
history of cesarean delivery on exam there numbness and normal range of motion what is the Rx?
Thumb cast
Whole are cast
Surgical decompression
answer: C
it is pregnancy induced carpal tunnel syndrome
62- which nerve is affected in case of fibular fracture?
common peroneal n
deep peroneal
superficial peroneal
Answer: A
Reference: https://www.nlm.nih.gov/medlineplus/ency/article/000791.htm
63- trendelenburg gait features:
internal rotation with adduction
internal rotation with ab
external rotation with abduction

smle ,2016

723
Answer: A
Since the trendelenburg gait develops as a result of weakness in Abductor muscle, the affected side tend
to be in adduced position with internal rotation, based on the pictures below

64- young man close the door on his nail color become blue under nail what will you do ?
evacuate hematoma
remove nail
reassure it well resolved by itself
Answer: A
http://www.orthobullets.com/hand/6109/nail-bed-injury
65- young adult playing golf's came with Hx of and tenderness at medial epicondyle first evaluate :
x-ray
CT scan
reassure
Answer: C
Radiographs usually unremarkable except for MRI
66- Physician in the clinic tell the child to bend forward and hang his both hands freely. This test is
used in detection of ?
Rectal Prolapse
Sexual Abuse
Scoliosis
Answer : C
Reference: http://www.physio-pedia.com/Adam's_forward_bend_test

smle ,2016

724

67- Scoliosis what radiological method will you order:


Xray
CT scan
MRI
Answer : A
"PA spine radiographs Radiographs are required to confirm the diagnosis of scoliosis, evaluate the etiology (congenital, neuromuscular, idiopathic), determine the curve pattern and measure the magnitude
(Cobb angle), and to evaluate skeletal maturity (to determine the risk for progression).
MRI of the spine Magnetic resonance imaging (MRI) of the spine may be indicated in patients with
scoliosis and clinical or plain radiographic findings suggestive of intraspinal pathology (tumor, dysraphism, infection)"
Reference.uptodate http://www.uptodate.com/contents/adolescent-idiopathic-scoliosis-clinical-features-evaluation-and-diagnosis?source=machineLearning&search=scoliosis&selectedTitle=1~150&sectionRank=2&anchor=H15#H15
68- Patient presented with distal thigh pain erythema and warmth seems inflamed but the knee is
completely normal there's limited range of motion with tenderness which of the following would be
helpful in establishing the diagnosis:
C-reactive protein (or ESR)
X ray
Blood culture
Answer : B this is a case most probably osteomyelitis (The others are prognostic not diagnostic)
http://www.orthobullets.com/trauma/1057/osteomyelitis--adult
69- which part of bone is firstly affected in hematogenous osteomyelitis?
epiphysis.
metaphysis.
diaphysis.
Answer is: B
Reference: http://www.uptodate.com/contents/overview-of-osteomyelitis-in-adults?source=outline_link&view=text&anchor=H1#H1
70- women complaining of left hand tingling mainly at thumb and index on exam there was mild atrophy of thinner muscle Tinel's test was positive which nerve may be affected??
radial nerve
median nerve
musculocutaneous nerve
Answer: B
Reference: http://www.medicinenet.com/script/main/mobileart.asp?articlekey=16687
71- Pt with 1st metatarsal joint pain, redness and erythema,High temperatures? What is the cause?
staph aureus
Monosodium urate crystal
Ca pyrophosphate crystal
Answer: B (Gout).
Reference: PubMed

smle ,2016

725
72- picture of tibial fracture proximal peripheral perfusion normal and normal sensation, the best next
to investigate?
angiography
MRI
doppler us
Answer: B
CT scans and MRI are reserved for more complex injuries. MRI has particular value in higher tibial fractures that may extend into the knee joint or are suspected of involving the tibial plateau. MRI also helps
delineate associated meniscal or ligamentous injuries.
73- Pts with active osteoarthritis with knee swelling on examination; there is effusion but no pain not
hot with crepitus during movement. What is the best next?
Bilateral knee x Ray and ibuprofen
Aspirations of fluid
Bone something
Answer:A
74- Pain in the snuff box would most likely be due to fracture of?
Scaphoid
Head of hamate fracture
Colles fracture
Answer : A
75- old lady postmenopausal with osteoarthritis and risk for osteoporosis , what you will do:
calcium , TSH , dihydroxy vit D
bisphosphonate , vit D , calcium
DEXA scan
Answer: B
76- i got 2 pelvic X-rays for children with similar choices
Metaphyseal displaced
Avascular necrosis of head of femur
Fracture head of femur
Answer: b? (Legg-Calve-Perthes disease?)
77- Elderly having knee pain for years and he is on NSAID when needed.. now he developed Left foot
edema .. what is the best investigation ?
Echo
CBC
Doppler
Answer: C
78- Patient presented with Right foot drop.Which nerve is affected:
Common peroneal
Deep peroneal
Tibial

smle ,2016

726
Answer: A
79- fracture of the tibia , displaced transverse, managment?
close redaction
open reduction with internal fixation
abx
Answer: B
80- Patient with low back pain, and normal neuro exam, MRI showed mild spinal stenosis.. what is the
Rx:
surgery.
physiotherapy.
biofeedback.
Answer: B
81- RTA, vitally stable, had pain in all of his 4 extremities, which type of shock might be developed?
Neuro
Cardiac
Hypovolemic
D.
Reversible
Answer: A ?
82- Pt after do femur surgery after 6 day developed fever and inflammation at site of surgery, got
wound infection at site of operation , what is the source of that infection ?
cause:
Surgeon not hand wash
Equipment not clear
vistors
Answer: ?
83- osteochondroma develop mass or pain in pelvic histo cartilage DX:
Ewdge
Chondroblastoma disease
Chondrosarcoma
Answer : C ( chondrosarcoma cz their HX of osteochondroma )
Reference: Tronto
84- MVA with shock and fracture femur, what will be increased?
ALP
CK
ALT
Answer A
85- Humerus fracture, how will he present?
Waiters tip hand
Wrist drop
Claw hand
Answer: B due to injury to the radial nerve (Mid shaft fracture)

smle ,2016

727

86- Child was playing with his toy, as he was walking around he tripped and twist his leg, since then he
does not want to walk and want to be carried around. What is wrong with him?
Fracture of the tibia
Fracture of the femur
Soft tissue swelling of the ankle
Answer: C because of the mechanism of injury
87- old man with multiple fractures during the last 2 years, he is k/c of osteoporosis. What should you
give him at this stage?
Alendronate.
vitamin D.
estrogen.
Answer: A .
88- Best x ray to show fracture rib
AP
PA
Oblique
Answer: C
45 oblique view on expiration is recommended for radiographic imaging of patients with clinical signs of
fracture, e.g. evaluation of lower rib fractures, while 45 oblique view during fast breathing is recommended for suspected upper rib fractures.
http://www.ncbi.nlm.nih.gov/pmc/articles/PMC3529706/
89- X ray Pic of fibula fracture which was multiple,,, + other pic for the leg of the pt and gave hx and
asked what u will do also,,
Discharge home
Debridement internal fixation
External fixation
Answer: Question not clear
90- Child with FOOSH. He has pain on affected arm and shoulder. X-ray shows midclavicular fracture
with segments overriding. What's the management:
sling
surgery
closed reduction
Answer: B
91- Pt with leg pain for a while. X-ray shows periosteal elevation and onion skin. What's the management?
IV antibiotic
Warm comp, rest and elevation
Steroids
Answer: A
Multilayered periosteal reaction, also known as a lamellated or onion skin periosteal reaction, demonstrates multiple concentric parallel layers of new bone adjacent to the cortex, reminiscent of the layers
on an onion. The layers are thought to be the result of periods of variable growth.

smle ,2016

728
It has been associated with:
Osteosarcoma
acute osteomyelitis*
Ewing sarcoma
Langerhans cell histiocytosis (LCH)
92- 8 y/o girl came with her father on wheel chair with pain in her knee on examination limited flexion and extension they didn't mention anything about knee effusion. Bp 105/58 hr 101 temp 39.6 RR:
27 how to diagnose
RF
ASO
arthrocentesis
Answer: B
93- MRI pic to the shoulder , with scenario of pt has trauma to his shoulder and unable to rise his hand
above his head X ray ( small white spot inside the upper part of the joint) :
rotator cuff injury
In....... Syndrome
acromion bone subluxation
Answer: A
94- case scenario about child fall on his toys and twisted his ankle, they mention that he couldn't walk
and want to be held by his parents:
Soft tissue swelling of the ankle
spiral fracture of tibia
dislocation of fibula
Answer: A
95- Osteoporosis optimal treatment
Ca,
vit d,
bisphosphonate*
Answer: C
96- Carpenter 80yrs.cant raise shoulder. X Ray showing osteopenia. Diagnosis
Glenohumeral arthritis
Acromial arthritis
Bursitis
Answer: B?
97- patient falled on outstretched hand presented 4 weeks later with inability to elevate and externally rotate on examination the same: ( they provided mri pic )
Acromioclavicular junction
Shoulder Dislocation
Rotator Cuff muscle tear
Answer: C
98- Muscle responsible of unlocking the knee:

smle ,2016

729
Gastrocnemius
fibularis
Populates
Answer: c
https://www.youtube.com/watch?v=jcoDfCqTeCU
Reference:
https://books.google.com.sa/books?id=0h7FTCb02eUC&pg=PA412&lpg=PA412&dq=popliteus+muscle+unlock&source=bl&ots=M0SwF6rNME&sig=-g9JvcwfhSLnDy9vJMRC8micow&hl=ar&sa=X&ved=0ahUKEwj08_2Py8zJAhUD_HIKHYq2DEI4KBDoAQgcMAE#v=onepage&q=popliteus%20muscle%20unlock&f=false
99- acute knee swelling with balloted patella, what to do next
ESR
CBC
Arthrocentesis
Answer: C
100- fracture of distal ulna and radius with a pic whats the immediate management
debridement , irrigation and fixation
close reduction and cast above the elbow
close reduction and cast below the elbow
Answer: B
101- 60 years old man with a fractured thoracic vertebrae. T Score = -2.6. What is the diagnosis?
Osteoporosis
Established osteoporosis
Osteopenia
Answer: B
Dexa scan interpretation T-score Diagnosis:
>1.0
Normal
<1.0, >2.5
Osteopenia
<2.5
Osteoporosis
<2.5 plus fragility fractures
Severe/ Established osteoporosis
stiffness with pain in shoulders and osteopenia :
bursitis
Arthritis
adhesive capsulitis
Answer : C
102- flat shoulder 1/3 middle arm sensation lost :
Ape palsy
Complete clow
Partial clow
Answer: (Either wrist drop or erbs palsy)
103- Pts came with scoliosis next step
CT Scan

smle ,2016

730
x ray
A damage test
Answer:B
104- Elderly patient with osteoporosis , most appropriate advice to lower risk of compression fracture?
avoid obesity
aerobic exercise
vitamin D
Answer: C
105- Pt with history of fall on his Right leg in sever pain the pain is decease with passive stretch of his
leg which of the following muscles is most likely affected?
Gastrocnemius
Popliteus
Soleus
Answer: ?
106- pathological rib fracture , spine T score -2.6 :
osteomalacia
osteoporosis
established osteoporosis
Answer: C
107- NERVE between heads of pronator teres muscle?
Median nerve,
Answer: A
Reference: Grays Anatomy
108- Epileptic patient had an episode of seizure presented to ER with arm in adduction and internal
rotation what do you expect him to have:
Anterior dislocation
Posterior dislocation
Inferior dislocation
Answer: B
Reference: http://emedicine.medscape.com/article/93323-clinical#b4
109- x-ray for displaced tibial fracture with another pic of skin sinus with discharge, received IV antibiotic and tetanus toxoid , what is ur next management :
discharge on oral antibiotic
external fixation
surgical debridement with internal nail fixation
Answer: C
110- Surgical neck of the humerus fracture. Which of the following nerves has high risk of
injury?
radial

smle ,2016

731
median
axillary
Answer: C
The surgical neck of the humerus is a constriction below the tubercles of the greater tubercle and lesser
tubercle. It is much more frequently fractured than the anatomical neck of the humerus. A fracture in
this area is most likely to cause damage to the axillary nerve and posterior circumflex humeral artery.
Reference: Wikipedia.
111- A 13 years old boy with right hip pain for 3 months x ray shows degenerative collapse of femoral
neck Wbc is 17 Esr normal ALP is normal
Perthe diseases
Hip arthritis
Tumor
Answer: A
112- knee aspiration in young boy showing labs result: wbc > 75, >25 neutrophils .. Typical scenario
how you gonna treat
oral antibiotic
IV antibiotic
surgical drainage with IV antibiotic
Answer: B
113- Hyperextension of DIP will cause:
2 choice Rupture tendon muscle ,flexor digitorum profundus
Intra articular fracture of PIJ join
Extra articular fracture of DIJ
Answer: A (Jersey finger)
114- Old lady presented with back pain increased when walking downhill but relieved when walking uphill with normal neurovascular exam. What is the diagnosis?
Spinal canal stenosis
Vascular claudication
Answer: A

smle ,2016

732
115- child sitting always in w position what the change in bone:
metatarsus adductus.
femoral anteversion (femoral torsion)
Answer: B
femoral anteversion (femoral torsion)
In Toeing : metatarsus Adductus, internal tibial torsion, femoral anteversion
Out toeing: femoral retroversion
For more info : http://www.wheelessonline.com/ortho/femoral_anteversion
Reference: VMA course

116- Tx of perthes d (avascular necrosis)


surgery
without any treatment
Answer: (Depends on the age and the classification)
Until recently most children with Perthes disease were treated with a plaster cast or brace, or surgery.
However, it is now known that at least half of cases heal well without any treatment, particularly children aged five and under, and milder cases
The aim of treatment is to promote the healing process and to ensure that the femoral head remains
well seated in the hip socket as it heals and remodels. Treatments advised can depend on the age of the
child and the severity of the condition. Treatments may include observation, bed rest and crutches, a
plaster cast or special leg brace, or surgery\
http://www.perthes.org.uk/what-is-perthes-disease/
117- pt had posterior hip dislocation?
internal rotation ..adduction
external rotation ..abduction.
Answer: A
http://slideplayer.com/slide/4311100/

118- child 9 years with metaphyseal femur or tibial avascular necrosis.. what do you want to do
?
surgery.

smle ,2016

733
avoid weight bearing for 6 months or weeks
Answer: A
Ref: http://www.ncbi.nlm.nih.gov/pubmed/3047257
119- Fraction of head of tibia with drop feet which nerve?
Deep peroneal
Common peroneal
Answer: B
120- Best exercise for osteoporosis?
Low resistance exercise and conditioning
Low resistance and highly repetitive weight bearing
Answer: B
Increase weight bearing and muscle strength exercise
Reference: Medscape
http://emedicine.medscape.com/article/330598-overview#showall
121- Girl with high heel break the heel of Right foot with inversion of RT foot what's the injured tendon?
Calculo Tibialis
anterior tibialis
Answer: Talofibular (anterior talofibular ligament,)
http://www.iaaf.org/download/download?filename=5b2291a7-258c-4635-bbc08657eec73740.pdf&urlslug=Chapter%2010%3A%20Specific%20injuries%20by%20anatomic%20site
122- Retire farmer recently he developed pain in the left arm, which with time progress until he can't
sleep on his left side, by examination found to have severe decrease in motion, what he has ?
osteoporosis of the bone
spondylitis (arthritis that affects the spine only)
Answer: both answers are wrong, the case scenario goes more with Bursitis as it mostly affect the shoulder joint, old age and occupation ( any repetitive motion or pressure on particular bursa
Reference: http://www.mayoclinic.org/diseases-conditions/bursitis/basics/risk-factors/con-20015102
123- Case of young boy with falling in outstretched hand MRI image attached
shoulder dislocation
rotator tear
http://www.orthobullets.com/sports/3043/rotator-cuff-tears
http://www.orthobullets.com/sports/3051/posterior-instability-and-posterior-dislocation
124- Old female with lytic bone lesion , high ALP what's the tx ?
Bisphosphonate
selective estrogen
Answer:A
http://www.uptodate.com/contents/overview-of-the-management-of-osteoporosis-in-postmenopausal-women?source=outline_link&view=text&anchor=H40#H40
125- Elderly patient with low back pain, what is the drug cause relaxation of muscle ?
diazepam

smle ,2016

734
clorcarpine
Answer: A (one of diazepam uses is to relieve muscle spasm) http://www.webmd.com/drugs/2/drug6306/diazepam-oral/details
126- Old patient with inability to abduct and externally rotate the shoulder. What's the abnormality?
Acromio something
Rotator cuff injury
Answer: B
Reference: Toronto Notes OR 13
127- 12 y/o Obese can't bear weight with left hip external rotation, x-ray provided?
Slipped capital femoral epiphysis
Fracture of femoral neck
Answer: A
femoral neck Fracture presentation :
Symptoms
impacted and stress fractures
slight pain in the groin or pain referred along the medial side of the thigh and knee
displaced fractures
pain in the entire hip region
Physical exam
impacted and stress fractures
no obvious clinical deformity
minor discomfort with active or passive hip range of motion, muscle spasms at extremes of motion
pain with percussion over greater trochanter
displaced fractures
leg in external rotation and abduction, with shortening
http://www.orthobullets.com/trauma/1037/femoral-neck-fractures
Slipped capital femoral epiphysis presentation :
Symptoms
groin and thigh pain
most common presentation
knee pain
can frequently present as knee pain (15-23%)
Motion
patients prefer to sit in a chair with affected leg crossed over the other
Duration
symptoms are usually present for weeks to several months before diagnosis is made
Physical exam
abnormal gait
coxalgic, externally rotated gait or Trendelenburg gait
decreased hip motion
obligatory external rotation during passive flexion of hip
loss of hip internal rotation, abduction, and flexion
abnormal leg alignment
externally rotated foot progression angle
Weakness
thigh atrophy

smle ,2016

735
http://www.orthobullets.com/pediatrics/4040/slipped-capital-femoral-epiphysis
Peroneal nerve injury - Often the result of direct contusion in proximal fibula Fx and has a variable prognosis'' http://www.orthopaedicsone.com/display/Main/Proximal+fibula+fractures

128- which of the following is non traumatic fracture in osteoporosis ?


Vertebral fracture
statin induced myopathy in old lady
Answer: A
Vertebral fracture is the most common clinical manifestation of osteoporosis.
http://www.uptodate.com/contents/clinical-manifestations-diagnosis-and-evaluation-of-osteoporosisin-postmenopausal-women?source=outline_link&view=text&anchor=H3#H3
129- Old male not known to any medical illness presented with RT knee swelling, no fever or tenderness, what investigation will you order?
Arthrocentesis
Knee MRI
Answer: Always order first X-ray for fracture, arthrocentesis is diagnostic and therapeutic.
Reference: http://www.aafp.org/afp/2000/0415/p2391.html
130- pt can't do dorsiflexion & eversion :
common peroneal N
Deep perineal N
Answer: A (Common peroneal injury is commoner than deep peroneal, depends on the cause and the
site of injury, for example, injury at the ankle level is deep peroneal)
http://neuromuscular.wustl.edu/nanatomy/cp.htm
131- bilateral shoulder and hip stiffness and pain what is Dx :
Polymyalgia rheumatica
OA
Answer: A
132- loss of adduction of fingers caused by injury to:
ulnar
median
Answer: A
133- Patient with numbness of index finger when he scissor .... What is Dx:
OA
ducyptus ????
Answer: (nerve entrapment syndrome)
134- limping child, painful left hip? ( x ray given )
Slipped capital femoral epiphysis
Perthes disease.
Answer: B (Q not complete)
135- Artery supplying gluteal region :

smle ,2016

736
Internal pudendal artery.
Internal iliac artery.
Answer: B
136- What is the type of elbow joint:
hinge
cartilage
Answer : A
137- Patient complaining of hip pain after long periods of using the hip it keeps him awake at night
and have prolonged hours of stiffness in the morning:
osteoporosis
osteoarthritis
Answer: B
138- Patient lost elevation of shoulder and external rotation what is the diagnosis:
rotator cuff
impingement
Answer: A
139- SCA with hip pain?!
Avascular necrosis
Osteoarthritis
Answer: A
140- Shoulder dislocation, was put into place then loss of sensation over lateral arm occurred. Whats
the nerve affected?
Axillary
Radial
Answer: A
141- Female with leg pain when she walk 300 m. Relieved by rest..
claudication
DVT
Answer: A (Q not complete)

smle ,2016

737

142- a clear case of septic arthritis - high WBC high ESR


aspiration and antibiotic
ASO
Answer: A
143- Falls onto an outstretched hand lead to:
clavicle fracture
Colle's fracture
Answer: B
144- Which one of these is a feature of osteoporosis:
decreased bone mass
Decreased mineralization of bone
Answer: A
145- girl with high heel break the heel of RT foot- 13 with inversion of RT foot what the tendon injured?
calculo tibialis
anterior tibialis
Answer: Anterior talofibular ligament.
146- femoral neck # his leg was rotated laterally which muscle responsible
Rectus femoris
Gluteus maximus
Answer: B
147- a question about the foot tendons... I cannot remember it..
Spring ligament
Achilles tendon

smle ,2016

738
Answer : not clear Q
148- boy was playing football with barefoot and was injured in his sole , and Dr found that it is only
superficial plantar injury which structure may be affected:
posterior tibialis tendon
adductor hallucis longus
Answer: flexor digitorum longus
149- old male not known to any medical illness presented -54 with RT knee swelling , no fever or tenderness , what investigation will you order ?
Anthrocentesis
Knee MRI
Answer: x ray
150- Painful limiting fixation of femur at 30 degree with slightly internal rotation and maybe shortening not sure
Perths
long name means something separate from something on top of femur head *
Answer: options of answers are missing but internal rotation could be caused by fracture of neck of femur (like perthes) or posterior dislocation
151- pt medical free came with knee swelling no tenderness or redness next step :
MRI
xray
Answer : B
152- x ray showed narrow joint space and osteophytes:
Rheumatoid arthritis
Osteoarthritis
Answer : B
Reference: Toronto notes
153- Postmenopausal women with bone metastasis, BMD T score -3. Came with vertebral fractures.
What is your appropriate management?
Estrogen
Bisphosphonate
Answer: B
154- Pain anterior to the heel Worse in the morning and better along with the day? Diagnosis?
Plantar fasciitis
Calcaneal heel spur
Answer: B? (Calcaneal heel spur pain is worse in the morning)
plantar fasciitis occurs, the pain is typically sharp[9] and usually unilateral (70% of cases).[7] Heel pain
worsens by bearing weight on the heel after long periods of rest.[10] Individuals with plantar fasciitis
often report their symptoms are most intense during their first steps after getting out of bed or after
prolonged periods of sitting.[2] Improvement of symptoms is usually seen with continued walking
If you have foot pain at night, you may have a different problem, such as arthritis, or a nerve problem
such as tarsal tunnel syndrome.

smle ,2016

739
Major symptoms consist of pain in the region surrounding the spur, which typically increases in intensity
after prolonged periods of rest. Patients may report heel pain to be more severe when waking up in the
morning. Patients may not be able to bear weight on the afflicted heel comfortably. Running, walking, or
lifting heavy weight may exacerbate the issue.
155- Man with osteoarthritis, initial management:
intra articular steroid
exercise to increase the strength of thigh muscle
opioid
Answer: B
Reference: http://emedicine.medscape.com/article/330487-treatment
156- function of ACL:
medial rotation of tibia in relation to femur
prevents anterior (forward) movement of the tibia off of the femur
Answer: B (it prevent A so if A was Prevent medial rotation of tibia in relation to femur it would be
correct).
157- Old age with R hip pain when palpate the joint with normal adduction flexion and abduction
best next management is
NSAID
Physiotherapy
Answer: A
158- Old lady with osteoporosis best prevention is?
weight bearing exercise
daily Vit. D supplements
Answer: A
Reference: http://emedicine.medscape.com/article/330598-treatment
159- Gradual neck pain and loss of side-to-side movement, neck x ray showed osteophytes and narrowed joint space, dx?
Cervical spondylosis
Muscle stiffness
Answer : A
160- Patient fell on outstretched arm what did it cause, complain of failure of external rotation? X-ray
was attached ( there wasn't colles fracture in the options )
rotator cuff tear
impingement
Answer: A
161- a woman with neck pain after the pain there is tingling and shock like waves and weakness ,what
is diagnosis
whiplash injury
brachial plexus injury
Answer: A, scenario is not complete (there should be acceleration deacceleration injury)

smle ,2016

740
162- young male, complain of pain in elbow and shoulder since (1 w or 1 month , not sure ) he can't in
full extend his elbow , also mention he squash player what is diagnosis :
olecranitis
fracture
Answer:? Tennis elbow is injury to tendons, not fracture, plus the pain extends to the wrist not shoulder.
Fracture is well localized pain, decreasing range of motion. Could be post-traumatic elbow arthritis.??
163- child with hx of comminuted clavicular # due to MVA , ttt :
arm sling
close reductions
Answer : ORIF
164- kwon case of sca Complaining from right shoulder pain X ray show head humerus necrosis What's
long term therapy
hydroxyurea
penicillin
Answer: long term treatment i think folic
http://www.houstonmethodist.org/orthopedics/where-does-it-hurt/shoulder/osteonecrosis-humeralhead/
165- Pt with bilateral deltoid muscle weakness and other symptom ?
Brachial plexus neuropathy
Neck spasm
Answer: Cervical spondylopathy
166- Pt c/o mild neck pain then sudden he c/o electrical pain on left arm and weakness and loss of
tendon reflexes... Diagnosis ?
cervical disc prolapse
polymyalgia rheumatica
Answer: A
167- 5 yrs old pt c/o severe avascular necrosis Treatment?
surgery
weight bearing for 6 months
Answer: A (most likely due to high success rate)
Reference: http://emedicine.medscape.com/article/333364-treatment#d7
168- Unilateral knee pain, tibial laxity on ant position more on R side than in L. structure injured is?
ACL
Meniscus
Answer: A? (not clear question)
169- Child with painless limping.
Perthes disease
Answer: ?
Differential diagnosis of painless limping in children:
Developmental dysplasia of the hip
Neuromuscular disease (Cerebral palsy and muscular dystrophy)

smle ,2016

741
Lower limb length discrepancy
Perthes disease: Typically present with painless limp, but may be associated with groin or anterior thigh
pain (It become painful as it progresses)
170- What is the most commonly injured Carpal bone?
Scaphoid.
Answer: A
Important tips to remember:
Scaphoid (palpated in anatomic snuff box) is the most commonly fractured carpal bone and is prone to
avascular necrosis owing to retrograde blood supply.
Dislocation of lunate may cause acute carpal tunnel syndrome.
A fall on an outstretched hand that damages the hook of the hamate can cause ulnar nerve injury.
Reference: FA USMLE step 1
171- Knee Examination with positive lachman test indicates injury of:
Anterior cruciate ligament
Answer: A
The anterior drawer test also can be used to assess ACL
Reference: 3rd Edition UQU > Orthopedics > Q 62.
172- A male patient running 20 km complaining of upper leg pain.
stress fracture
Answer: A
173- Mid humerus shaft fracture:
radial nerve injury (loss of wrist extension)
Answer: A
Reference: http://www.orthobullets.com/trauma/1016/humeral-shaft-fractures
Important tips to remember:
Fracture of humerus at:
Surgical neck: axillary nerve and posterior humeral circumflex artery.
Midshaft: radial nerve (loss of wrist extension) and profunda brachii artery.
Supracondylar region: median Nerve and brachial artery.
Medial epicondyle: ulnar nerve.

174- Athlete presented with severe painful plantar flexion which prevents him from raising the foot.
Which ligament is affected?
Plantar fascia (Plantar fasciitis)
Answer: A

smle ,2016

742
175- septic arthritis organism?
Staph aureus
Answer: A
Staphylococcus aureus is the cause of the vast majority of cases of acute bacterial arthritis in adults and
in children older than 2 years.
Neisseria gonorrhoeae is the most common pathogen (75% of cases) among younger sexually active individuals.
Reference: http://emedicine.medscape.com/article/236299-overview
176- constrictor with repeated hand use:
Lateral epicondylitis
Answer: A
Lateral epicondylitis (tennis elbow): Repetitive extension (backhand shots) or idiopathic pain - near lateral epicondyle.
Medial epicondylitis (golfers elbow): Repetitive flexion (forehand shots) or idiopathic - pain near medial
epicondyle.
177- Women worn high shoes And then broken her feet everted laterally which ligament injured
deltoid ligament
Answer: A, " deltoid ligament injury occurs with pronation (eversion) trauma leading to forced external
rotation and abduction of ankle "
If there is lateral ligament choose it because deltoid very hard to be injured
http://www.orthobullets.com/foot-and-ankle/7005/ankle-ligaments
178- Female patient with vertebral compression fracture. Which on of the following will prevent osteoporosis?
Vit D supplement
Answer: A
179- A patient with bone pain and pathological fracture. X Ray: lytic lesion. Labs: high ALP. What
is the most likely diagnosis?
Paget's disease
Answer: A
Pagets disease: Common, localized disorder of bone remodeling caused by increase in both osteoblastic
and osteoclastic activity. Usually asymptomatic, but may present with aching bone or joint pain, headaches, skull deformities, fractures, or nerve entrapment (leads to loss of hearing in 3040% of cases).
The investigations show increase in serum alkaline phosphatase and lytic bone lesion in x-ray with normal calcium PTH and phosphate levels.
Reference: FA USMLE step1 and step2 CK
180- Question about old female with recurrent fracture ..
Estrogen Def
Answer: Estrogen improves calcium absorption and reduces the amount of calcium lost in urine
so deficiency of estrogen will lead to osteoporosis and osteoporosis-related fracture are at high risk of
recurrent fractures.
181- Old pt with spinal stenosis, management?
Physical therapy?

smle ,2016

743
Answer: oral medications, physical therapy, and corticosteroid injections
Reference : Orthobullet
http://www.orthobullets.com/spine/2037/lumbar-spinal-stenosis
182- Patient presented with knee swelling and pain they did x ray and aspiration and found negative
birefringence needlelike what are you going to discharge the patient with?
Allopurinol
answer : A

183- Male pt complaine of weakness in flexion of both Rt knee and Right hip which muscle affected ?
Sartorius
answer: A
http://www.orthobullets.com/anatomy/10055/sartorius
184- Active osteoarthritis in knee pain best exercise:
Quadriceps Ms strengthen.
answer:"Quadriceps and Hamstring Ms lengthen+strengthen"
185- The most common cause of olecranon bursitis is ?
Repeated injury
Answer: A
Reference: http://www.nhs.uk/Conditions/Bursitis/Pages/Causes.aspx
186- The nerve which works on arm extension is?
Radial nerve.
Answer: A
The radial nerve is the "great extensor" of the arm: it innervates all the extensor muscles in the upper
and lower arm.
Reference: https://informatics.med.nyu.edu/modules/pub/neurosurgery/motor.html
187- Skull fracture?
C-spine
Answer :?
188- Women worn high shoes And then broken her feet everted laterally which ligament injured

smle ,2016

744
deltoid ligament
Answer: A
An eversion sprain is a tear of the deltoid ligaments, on the inside of the ankle. It is often called a medial
ankle sprain or a deltoid ligament sprain.
189- Patient presented with knee swelling and pain they did x ray and aspiration and found negative
birefringence needlelike what are you going to discharge the patient with:
Allopurinol
Answer : A
190- 7 years old presented with back pain. had a similar attach 1 year ago. What is the diagnosis?
Ankylosing spondylitis
Answer: Not A.
Spondylolisthesis.
191- Old with osteoporosis risk scenario, has spine compressive fracture, what to give her to protect?
Bisphosphonate, calcium, vit D Other
Answer: A
http://www.m.webmd.com/a-to-z-guides/spinal-compression-fractures-preventing
192- Pt stretch out of hand with fracture in medial epicondylar. , After reduction and cast, there is a
loss of movement in wrist and hand, Which nerve ?
Radial in anterior spiral groove
Answer: Ulnar nerve injury
Ulnar N passed post to medial epicondyle if injured >> claw hand and sensory lost over medial aspect of
the hand.
Radial N passed anteriorly to the lateral epicondyle within the cubital fossa of the forearm if injured >>
depend on the site ( if in the anterior spiral groove >> wrist drop )
http://teachmeanatomy.info/upper-limb/nerves/the-radial-nerve/#In_the_Radial_Groove
193- Pt with multiple trauma post MVA, vitally unstable. First step:
IVF
Answer: ? not A
Reference: http://emedicine.medscape.com/article/434707-overview#a1
194- young with pain in forearm worsening day by day , x ray shows "onion peel" best investigation?(i guess it's ewing sarcoma)
MRI
Answer: A.
http://www.orthobullets.com/pathology/8047/ewings-sarcoma
195- pt with long scenario have low pack pain and invest revealed fracture also have high temperature
and night sweating what dx?
TB
Answer: A
196- The cause of fracture in osteoporosis is due to?
Vit D deficiency

smle ,2016

745
Answer: Medscape:
Furthermore, in periods of rapid remodeling (eg, after menopause), bone is at an increased risk for fracture because the newly produced bone is less densely mineralized, the resorption sites are temporarily
unfilled, and the isomerization and maturation of collagen are impaired.
Fractures occur when bones fall under excess stress. Nearly all hip fractures are related to falls.[23] The
frequency and direction of falls can influence the likelihood and severity of fractures. The risk of falling
may be amplified by neuromuscular impairment due to vitamin D deficiency with secondary hyperparathyroidism or corticosteroids.
197- Mid humerus fracture, loss of sensation in dorsum of the hand and inability to support the hand
in upper position?
Radial n Injury*
Answer: A
198- investigation for bone density ?
Dexa scan
Answer: A
199- about complication of femur neck fracture:
Avascular necrosis
Answer: A
200- 60 years old female with vague Symptoms Labs : ALP high , DEXA scan showed mild osteoporosis
, what to give ?
Bisphosphonate
Answer: A
Reference: http://emedicine.medscape.com/article/330598-treatment#d8
202- 2 yrs old boy fall and twisted his leg by toys; spiral fracture in the right
Tibia
Answer: A
203- man with Low back pain and lytic lesion biopsy shows schistocytes and giant cells what is the diagnosis ?
Giant cell Tumor of bone
Answer: A
204- displaced tibial fracture with open wound &lt is 1 cm :
Closed reduction
Answer: A
205- pt with ankylosing spondylitis associated with ?
uveitis
Answer: A
206- A man arrived to ER after MVA the neck of the femur was fractured, Which of the following may
happen in sequence?

smle ,2016

746
Answer
Injury of sciatic nerve; blood supply to femoral headavascular necrosis
(AVN)
Reference: Surgical recall 6th edition
207- Patient came to the clinic with severe plantar flexion and heard a snap (No Achilles tendon in
the answers).
Answer
The usual mechanism of lateral ligament injury is inversion and plantarflexion. This may be accompanied
by an audible snap, crack or tear. Depending on the severity of the injury, the athlete may have been
able to continue activity immediately or have been forced to rest. Swelling usually appears soon after
the injury, although occasionally it may be delayed some hours.
208- Long scenario. What is the ligament that prevent overextension of hip?
Answer
Iliofemoral ligament: Y-shaped. It prevents hyperextension of the hip joint during standing by screwing
the femoral head into the acetabulum.
209- Function of anterior forearm muscle:
Answer
Superficial group: 5 muscles; flexor carpi radialis, Flexor carpi ulnaris, flexor digitorum superficialis, palmaris longus and pronator teres.
Deep group: 3 muscles; flexor digitorum profundus, flexor pollicis longus and pronator quadratus.
All responsible for flexion and pronation
Reference: Zuckerman handbook of fractures
210- (Long scenario) Patient with low back pain, loss of sensation, weakness and loss of reflexes, what
will do next?
Answer: ?
Urgent MRI
The most common indication for the use of imaging procedures (MRI or CT) is the clinical setting of Low
back pain complicated by radiating pain (radiculopathy, sciatica), as well as in cauda equina syndrome
(bilateral leg weakness, urinary retention, saddle anesthesia), neurogenic claudication and/or spinal stenosis. MRI of the lumbar spine has become the initial imaging modality of choice in complicated LBP, displacing myelography and CT in recent years.
See: http://www.guideline.gov/content.aspx?id=35145
211- girl with Right knee swelling Aspirations results show WBCs and what best management:
Discussion:
- if the WBC 15,000 to 200,000 the diagnosis is (Septic Arthritis)
- Empiric intravenous antibiotic treatment of septic arthritis should be based on the organism found in
the Gram stain of the synovial fluid, or on the suspicion of a pathogen from the patient's clinical presentation.

smle ,2016

747

References:
all about the knee : http://www.aafp.org/afp/2000/0415/p2391.html#abstract
management of septic arthritis according the gram stain
http://www.aafp.org/afp/2011/0915/p653.html#sec-3
212- carpal tunnel syndrome vs. thoracic outlet obstruction??
Carpal tunnel syndrome:
Entrapment of the median nerve at wrist caused by decrease space of the carpal tunnel leading to
parasthesia, pain, paralysis.
Causes: overuse of wrist flexors, associated with DM , thyroid dysfunction ,pregnant , middle aged
women.
Ref. Step 2 CK page 221
Thoracic outlet obstruction :
Thoracic outlet syndrome (TOS) refers to a constellation of signs and symptoms arising from compression of the upper extremity neurovascular bundle by various structures in the area just above the first
rib and behind the clavicle, typically within the confined space of the thoracic outlet. The term thoracic
outlet syndrome is not a specific diagnosis and the appropriate type of TOS, such as neurogenic (nTOS),
arterial (aTOS), or venous (vTOS) thoracic outlet syndrome should be used. (See 'Introduction' above.)

smle ,2016

748
Thoracic outlet syndromes are due to rib anomalies, muscular anomalies, or a result of injury. Cervical
ribs predispose the patient to TOS after hyperextension-flexion (whiplash) injury. The absence of a rib
anomaly makes the diagnosis of arterial thoracic outlet syndrome less likely. Many patients with nTOS
have a prior history of neck trauma or repetitive occupational physical stress. Similarly, vTOS is highly
associated with repetitive movements, particularly with repetitive overhead upper extremity movements. (See 'Pathogenesis' above.)
Reference: uptodate
http://www.uptodate.com/contents/overview-of-thoracic-outlet-syndromes?source=search_result&search=thoracic+outlet+syndrome&selectedTitle=1%7E36
213- AVN (Avascular Necrosis) or Osteonecrosis:
Commonest Presentation:
Pain (inguinal area then radiated to the buttocks and thigh).
Investigation:
Radionuclide bone scanning Technetium-99m bone scanning has been used for patients with suspected disease who have negative radiographs, unilateral symptoms, and no risk factors.
MRI without contrast (Gold standard) is far more sensitive than plain radiographs or bone scanning, with
reported sensitivity of up to 100% Focal lesions are well-demarcated and inhomogeneous on T1weighted images. The earliest finding is a single-density line (low-intensity signal) that represents the
separation of normal and ischemic bone.
Management:
Nonoperative management essentially, with analgesics and pharmacological agents,
Restricted patient weight bearing with the use of a cane or crutches has not been shown to affect the
natural history of the disease and is useful only in controlling symptoms, poor outcome (only 15% resolved with nonoperative intervention).
Joint-preserving procedures In early stage 0 to II lesions, in young active patients, core decompression is the most conservative surgical procedure that offers the best chance at preserving the femoral
head.
Joint replacement

Type of bone hunger.


Answer:
There are no types!

smle ,2016

749
Hungry bone refer to severe and prolonged hypocalcemia which occurs after parathyroidectomy or less
commonly thyroidectomy despite normal or even elevated level of parathyroid hormone ( PTH) . The fall
in serum calcium is primarily due to functional or relative hypoparathyroidism leading to increase calcium influx of Ca+ into bone in patient without end-stage renal disease .
Reference: Uptodate and toronto notes.
214- Boutonnire deformity (BD) ?
can manifest itself acutely after trauma, but most BDs are found weeks following the injury or as the result of progressive arthritis. The proximal interphalangeal (PIP) joint of the finger is flexed, and the distal
interphalangeal (DIP) joint is hyperextended.
Reference: Medscape
215- case of osteoarthritis. Dx?
Answer:
Symptoms of OA: joint pain or tenderness, Stiffness, loss of flexibility, Grating Sensation, & bone Spurs
Tests & Diagnosis: X-ray will show narrowing of the space between the bones in the joint
Reference: http://www.mayoclinic.org/diseases-conditions/osteoarthritis/basics/tests-diagnosis/con20014749
http://emedicine.medscape.com/article/330487-differential
216- girl with high heel break the heel of Right foot with inversion of RT foot what the tendon injured?
Answer:
Anterior talofibular ligament.
217- ptn wear high heel ..then her Right heel is broken ..which structure is injured?
Answer: Might be Achilles tendon??

218- Surgical neck fx of humerus what nerve


Answer:
Axillary nerve
http://www.orthobullets.com/trauma/1015/proximal-humerus-fractures
219- Mid shaft humerus fx? Wrist drop ..
Answer:
RADIAL NERVE

smle ,2016

750
220- Treatment of medial epicondylitis
Answer: conservative physiotherapy
Reference: miller
221- Pt does not complain of anything ,, has sudden knee swelling ? What is the best thing to do ?
Answer:
may be knee x-ray to exclude any injury if case related to sport injury, rule out broken or dislocated
bones
Occasionally, a swollen knee develops rapidly without any injury. The most common causes of this are:
Fluid removed during aspiration of a swollen knee due to gout
Infection: Infections increase in the amount of fluid produced in the joint resulting in a swollen knee. They
usually develop after surgery or a deep cut, but sometimes an infection in your body can spread to your
joint. It is very difficult for your body to fight an infection within a joint and sometimes surgery is required
Gout: High levels of uric acid (produced as part of the digestive process) cause sharp, needle like crystals
to form in your joints leading to inflammation and water on the knee. It is usually treated with medication
and proper diet. Find out more in the Gout Knee section
swollen knee that develops immediately after an injury, within minutes, is usually due to haemarthrosis,
where blood accumulates in the joint. Essentially what happens is that a structure inside the knee gets
damaged and starts to bleed, building up pressure in the joint. The swelling is normally profuse and the
knee balloons up. It will feel tense and very sore and is often accompanied by bruising, although that may
take longer to develop. There are three main causes of a swollen knee from a haemarthrosis:
Ligament tears are a common cause of knee swelling.
Ligament Tear: Where a ligament ruptures (tears completely). This is the most common cause and usually
involves the ACL (Anterior Cruciate Ligament)
Meniscus Tear: A tear in the outer rim of the cartilage lining the knee
Bone Fracture: A break in one of the knee bones
A swollen knee like this needs urgent medical attention. Visit the Knee Injuries section to find out more
about these common causes of knee swelling, including symptoms and treatment options.
Reference: http://www.knee-pain-explained.com/swollen-knee.html#sthash.NlwrJagJ.dpuf
222- twisted ankle, What is the most common affected ligament ?
Answer :
anterior talofibular ligament,and calcaneofibular ligament
If both in choices , choose talofibular
Reference :
https://www.aofas.org/footcaremd/conditions/ailments-of-the-ankle/Pages/Ankle-Sprain-.aspx
223- boy after running for hours, has pain in knee and mass on upper surface of tibia, Dx?
Answer: Mechanism of osgood schlatter disease
iliotibial band osgoodschlatter disease or syndrome (tibial tubercle apophyseal traction injury and
epiphysitis of the tibial tubercle) is an irritation of the patellar ligament at the tibial tuberosity. it is
characterized by painful lumps just below the knee and is most often seen in young adolescents. risk
factors include excess weight and overzealous conditioning (running and jumping). diagnosis is made
clinically treatment is conservative with rice (rest, ice, compression, and elevation), and if Required
acetaminophen.
http://www.orthobullets.com/sports/3029/osgood-schlatters-disease-tibial-tubercle-apophysitis

smle ,2016

751
224- Muscle that extends the knee?
Answer:
The main muscle for extension is the quadriceps femoris, which is the most important muscle in stabilizing the knee joint. The quadriceps is made up of the vastus medialis and lateralis, rectus femoris, and
vastus intermedius. The tensor fasciae latae is a weak extensor.
Reference: http://emedicine.medscape.com/article/1898986-overview#a2
225- Chronic Gout ?
answer: allopurinol

226- Pseudogout ?
answer: pyrophosphate crystal

227- Signs of osteoporosis on X-ray?


Answer :
http://emedicine.medscape.com/article/330598-workup#c9
228- case of tibial collateral ligament injury?

smle ,2016

752
answer:.
Reference: Uptodate
http://www.uptodate.com/contents/lateral-collateral-ligament-injury-and-related-posterolateral-corner-injuries-of-the-knee?source=search_result&search=tibial+collateral+ligament+injury&selectedTitle=4%7E150
229- tttx of de quervain syndrome
Answer:
itis a tenosynovitis of the sheath or tunnel that surrounds two tendons that control movement of the
thumb
Answer
forearm-based thumb spica splint with the interphalangeal joint free as well as a concurrent trial of nonsteroidal antiinflammatory drugs (NSAIDs) for pain relief, if persistante local glucocorticoid injection
For patients with persistent symptoms despite splinting and one or two glucocorticoid injections, surgical therapy may help relieve symptoms usually to start with conservative interventions, which include a
forearm-based thumb spica splint with the interphalangeal joint free along with short-term nonsteroidal
antiinflammatory drugs (NSAIDs). We suggest a local glucocorticoid injection for patients whose symptoms have not resolved with conservative management. Most patients recover with this intervention.
Patients who present with severe symptoms may benefit from a glucocorticoid injection at the initial
presentation. Surgical release is generally reserved for patients who have not improved with conservative therapy and one or two glucocorticoid injections .
up to date http://www.uptodate.com/contents/de-quervain-tendinopathy?source=outline_link&view=text&anchor=H660498#H660498
230- Case of tibial collateral ligament injury
Answer:
Mechanism of injury of lateral collateral ligament contact injury, such as a direct blow to the medial side
of the knee, or a noncontact injury, such as a hyperextension stress, may result in a varus force across
the knee injuring the LCL
Valgus stress testing of the MCL
Varus stress testing of the LCL
Injury severity
Grade I - Less than 5 cm laxity (partial tear)
Grade II - 5-10 cm laxity
Grade III - More than 10 cm laxity (complete tear)
medial collateral ligament (MCL) injury depends on the severity of the injury.[7, 14] Recommendations
for treatment include the following:
Grade I - Compression, elevation, and cryotherapy are recommended. Short-term use of crutches may
be indicated, with weight-bearingastolerated (WBAT) ambulation. Early ambulation is recommended.
Grade II - A short-hinged brace that blocks 20o of terminal extension but allows full flexion should be
used. The patient may ambulate, WBAT. Closed-chain exercises allow for strengthening of knee musculature without putting stress on the ligaments.
Grade III - The patient initially should be nonweight-bearing (NWB) on the affected lower extremity. A
hinged braced should be used, with gradual progression to full weight-bearing (FWB) over 4 weeks.
Grade III injuries may require 8-12 weeks to heal.
All MCL injuries should be treated with early range of motion (ROM) and strengthening of musculature
that stabilizes the knee joint. Conservative measures usually are adequate, but, if the patient fails to
progress with treatment, a meniscal or cruciate ligament tear is suggested.

smle ,2016

753
Lateral collateral ligament (LCL) injuries heal more slowly than do MCL injuries, due to the difference in
collagen density. Recommendations for the treatment of LCL injuries include the following:
Grades I and II - These injuries are treated according to a regimen similar to that for MCL injuries of the
same severity. A hinged brace is used for 4-6 weeks.
Grade III - Severe LCL injuries typically are treated surgically due to rotational instability, because they
usually involve the posterolateral corner of the knee. Patients may require bracing and physical therapy
for up to 3 months in order to prevent lateral instability.
medscape
http://emedicine.medscape.com/article/307959-treatment#d9
231- Case of closed Fx of ulnar and radius, what's the tttx??
Answer:
open reduction internal fixation is treatment of choice in fracture radius and ulna together .
From Prof. Mohammed Jalaat Alfaisal
http://emedicine.medscape.com/article/1239187-treatment#d10
232- low back pain treated with NSAID (master the board).
233- Pt with +ve sign of Finkelstein test what is your management?
+ve Finkelstein sign in case of De Quervain's tenosynovitis.
Answer: Most likely is thumb splint.
Reference: http://orthoinfo.aaos.org/topic.cfm?topic=a00007
234- Pt fall on outstretched hand , he can't flex the distal part of his second finger ? Muscle injury?
Answer: The flexor digitorum profundus acting on the proximal and distal joints, and the flexor digitorum superficialis acting on the proximal joints
Reference: Wikipedia
235- Muscle spasm after run
Answer : gastrocnemius muscle.
236- Pt does not complain of anything ,, has sudden knee swelling ? What is the best thing to do ?
Answer: aspiration
General measures to relieve knee pain and swelling should be tailored to the individual patient. These
may include advice for partial or non weight bearing, splints, cold packs, and prescription of simple analgesics and nonsteroidal anti-inflammatory drugs if not contraindicated. However, antibiotics should not
be started before cultures are obtained from appropriate diagnostic sampling. Similarly, intra articular
steroids should not be administered unless an appropriate diagnosis has been reached and contraindications have been ruled out.
reference: medscape
http://www.medscape.com/viewarticle/714757
237- Pt fall in outstretched hand , he can't flex the distal part of his second finger ?Muscle injury ?
Answer: flexor digitorum profundus(for distal part only)

smle ,2016

754

238- which part of body bone is forming the ankle?


Answer:
Ankle joint: the fibula, the tibia and the talus.
239- Examiner ask to stand on toes, What the Name of nerve is being examine?
Answer:
Toe Dorsiflexion by Deep perineal N
Toe Plantar flexion by Tibial N
So By Tibial in this case.
Reference: http://www.orthobullets.com/spine/2002/lower-extremity-spine-and-neuro-exam
240- What type of hip joint
answer: Ball and socket.
Reference: http://emedicine.medscape.com/article/1898964-overview
241- patient with loss of shoulder passive and active movement:
answer: adhesive capsulitis.
Reference: http://emedicine.medscape.com/article/1261598-overview#a5
242- Looks of dorsiflexion of foot which nerve injury:
Answer: Peroneal nerve.
243- soldier walks 1000miles developed pain on foot :
Answer: Spring ligament.
244- male with lumbar and femoral fractures. Osteoporosis on x ray. Alkaline phosphatase level abnormal but the rest is fine. Whats the most likely cause?
Answer : ??
245- X-ray knee + case of osteoarthritis?
246- Osgood schlatter ? Chondritis
Answer: inflammation of the patellar ligament at the tibial tuberosity.
247- child with radial and ulnar fracture and 1 cm laceration in the volar aspect of the hand, what is
most likely damaged nerve?
Answer: ulnar nerve.

smle ,2016

755

248- an elderly woman who has chronic back pain which increases while walking uphill and decreases
while walking down hill, what is the most likely diagnosis?
Answer: lumbar spine stenosis.
249- football player got trauma in his Left lateral knee.. later he developed Left medial knee edema
with tenderness. Which ligament is affected
Answer: Left medial collateral ligament.
250- In which condition bone age is greater than chronological age?
Answer: precocious puberty.
251- A lady wearing High heel then she felt and had ankle sprain, which (Tendon/muscle) affected?
the most common injury is ankle sprain
rather in supination most common or in pronation less common ....
there is no tendon rupture or tear in this injury ... there is ligaments injury and
are lateral ligaments complex ...
ATFL.
anterior talofibular lig
PTFL
posterior talofibular lig
CFL
calcaneofibular lig
ATFL is most common
Answer:
252- Read about slipped capital femoral epiphysis and DDH.
Reference: http://emedicine.medscape.com/article/91596-overview
http://emedicine.medscape.com/article/1248135-overview
253- Twisted ankle What is the most common ligament
Answer:
Inversion (lateral) ankle sprain
The most common type of ankle sprain occurs when the foot is inverted too much, affecting the lateral
side of the foot. When this type of ankle sprain happens, the outer, or lateral, ligaments are stretched
too much. The anterior talofibular ligament is one of the most commonly involved ligaments in this type
of sprain. Approximately 70-85% of ankle sprains are inversion injuries.
254- polymyalgia rheumatica:
Answer: Muscle tenderness.
Reference: http://emedicine.medscape.com/article/330815-overview#a5
255- carpal tunnel syndrome what muscle will be affected:
Answer: Thenar
Reference: http://orthoinfo.aaos.org/topic.cfm?topic=a00005
256- There were also missing questions about the following:
Osgood-Schlatter disease:

smle ,2016

756
Mechanism of injury: Overuse apophysitis of the tibial tubercle. Causes localized pain, especially with
quadriceps contraction, in active young boys.
Treatment: Decrease activity for 23 months or until asymptomatic. A neoprene brace may provide
symptomatic relief.
257- 60 years old female with distal phalangeal joint swelling and shoulder pain and knee pain:
258- shoulder displacement after 5 hours he go to the ER , close reduction done,, but there is a weakness in abduction & internal rotation ,, which nerve affected?
Answer : Axillary N
Mid shaft humerus > radial nerve injury > wrist drop
Surgical neck > axillary nerve
Medial epicondyle > ulnar nerve > claw hand
Supra condylar > median n > ape hand
259- case of closed Fx of ulnar and radius, what's the tttx??
The following are specific indications for operative treatment:
Fracture of both bones (ie, radius and ulna)
Fracture dislocations, Monteggia fracture dislocations, and Galeazzi fracture dislocations
Isolated radius fractures
Displaced ulnar shaft fractures
Delayed union or nonunion
Open fractures
Fractures associated with a compartment syndrome, irrespective of the extent of displacement
Multiple fractures in the same extremity, segmental fractures, and floating elbow
Pathologic fractures
Reference: http://emedicine.medscape.com/article/1239187-treatment
260- case of tibial collateral ligament injury
Answer
Medial collateral ligament (MCL) injuries of the knee are very common sports-related injuries and it is
the most commonly injured knee ligament. Injuries to the MCL occur in almost all sports and in all age
groups. Contact sports such as hockey, wrestling, rugby, football, and judo are responsible for the most
MCL injuries.
in PE:
Inspection and palpation of the knee(presence and location of point tenderness, localized soft tissue
swelling, deformity, or ecchymosis). A large joint effusion indicates an associated intra-articular injury.
-The integrity of the MCL is tested with a valgus stress, Testing should be performed in full extension and
at 30 of flexion.If the valgus stress test reveals laxity at 30 degrees of flexion, the superficial portion of
the MCL may be injured. Laxity at 0 degrees of flexion suggests injury to the deeper structures of the
MCL and a possible disruption of the anterior cruciate ligament (ACL), which acts as a secondary restraint to valgus stress.
Classification systems include the following:
American Medical Association Committee on the Medical Aspects of Sports (1966)
Grade 1 - 0-5 mm of opening
Grade 2 - 5-10 mm of opening
Grade 3 - Greater than 10 mm of opening
O'Donoghue classification

smle ,2016

757
Grade 1 - Few torn fibers, structurally intact
Grade 2 - Incomplete tear, no pathologic laxity
Grade 3 - Complete tear, pathologic laxity
reference:
http://www.uptodate.com/contents/medial-collateral-ligament-injury-of-the-knee#H11
http://emedicine.medscape.com/article/89890-clinical#b4
261- Description of anterior cruciate ligament tests?
Answer:
The knee is flexed at 2030 degrees with the patient supine. The examiner should place one hand behind the tibia and the other grasping the patient's thigh. It is important that the examiner's thumb be on
the tibial tuberosity.The tibia is pulled forward to assess the amount of anterior motion of the tibia in
comparison to the femur. An intact ACL should prevent forward translational movement ("firm endpoint")
Anterior Cruciate Ligament (ACL)
primary function Resists anterolateral displacement of the tibia on the femur
secondary function Resists varus displacement at 0 degrees of flexion
Anatomy
intrasynovial
origin
lateral femoral condyle
PL bundle originates posterior and distal to AM bundle (on femur)
insertion
broad and irregular
anterior and between the intercondylar eminences of the tibia
Composition
90% Type I collagen
10% Type III collagen
reference : orthobullets
262- Pain in the midline of the plantar foot with walking ?
263- case scenario young adult with avascular necrosis of head of humerus, what's the best treatment
?
Reference: http://emedicine.medscape.com/article/333364-treatment
264- Question about presentation of posterior hip dislocation?
Reference: http://emedicine.medscape.com/article/86930-overview
265- sensirion, valgus test positive which ligament will be affected ?
Answer: indicative of medial collateral ligament damage and may also indicate capsular or cruciate
ligament laxity.
Reference: https://en.wikipedia.org/wiki/Valgus_stress_test
266- X-ray pic of hip for a child ,, the qs mention hxx of trauma .. external rotation >> in the picture
there is fracture of neck of femur?
Answer: slipped capital femoral epiphysis.

smle ,2016

758
267- Electrodiagnostic testing, primarily with nerve conduction studies (NCS), sometimes supplemented with needle electromyography (EMG), is a standard part of the evaluation for CTS because it
has a high sensitivity and specificity for confirming the diagnosis
268- X ray Pic of fibula fracture which was multiple,,, + other pic for the leg of the pt and gave hx and
asked what u will do also
Answer:
269- accident loss lateral rotation of lower limb? Which muscle?
Reference: https://en.wikipedia.org/wiki/Lateral_rotator_group
270- read about when to do bone scintigraphy :
to determine whether the pain from the bone or from its surrounding tissue .
271- type of intervertebral disc joint?
Answer : secondary cartilaginous, gliding
272- case of cervical disc present with numbness ,shoulder pain and stiffness.........
Answer:
273- gower test, positive trendelenburg gait what next investigation?
274- 3 years old girl with typical history of elbow dislocation, which of the following ligament is affected?!
275- young with pain in forearm worsening day by day , x ray shows "onion peel" best investigation?(it's ewing sarcoma)
-MRI all bone tumors with MRI or US except Osteoid osteoma with CT
276- Sign in duchenne muscular dystrophy?
Answer: growers sign
Reference: http://emedicine.medscape.com/article/1173204-clinical
277- Pt numbness in Rt arm +feel electric shock dx ?
Answer: cervical disc prolapse or spondylitis
278- Construction worker presented with pain in the lateral epicondyle of the elbow
Answer: lateral epicondylitis
279- LeggCalvPerthes Disease:
Reference: http://emedicine.medscape.com/article/1248267-overview
280- Patellofemoral pain syndrome:
Reference: http://emedicine.medscape.com/article/308471-overview
281- Stress fracture ?

smle ,2016

759
282- 21-Old age with Rt hip pain when palpate the joint with normal adduction flexion and abduction
best next management is
A. NASID
B. Physiotherapy
Answer: A
283- Painful limibing fixation of femur at 30 degree with slightly internal rotation and maybe shortening not sure
-Perths
- Slipped Capital Femoral Epiphysis
long name means something separate from something on top of femure head
284- Patient lost elevation of shoulder and external rotation what is the diagnosis?
A. rotator cuff
B. impingement
Answer : A
285- Multiple fractures in child?
Richts ... I do not know the answer
286- after fracture which one will be elevated?
A. ck
B. Alp
Answer: B
287- Pt with hx of epilepsy came with shoulder pain, adduct or internally rotate the arm
What is the injury?
A. inferior dislocation
B. Subacromial posterior dislocation
Answer: B
288- Child with scoliosis when refer him to orthopedic D ?
a-5
b-10*
c-15
d-20
Answer: B
Reference : http://www.aafp.org/afp/2001/0701/p111.html
289- Cushing syndrome with fracture, because of :
Osteoporosis
Answer : A

smle ,2016

760

ENT
smle ,2016

761

1. Picture of tonsillitis, what is the best drug for treatment?


A.
Ceftriaxone.
B.
Acyclovir.
C.
Ampicillin.
D.
Doxycycline.
Answer: ?
The best is penicillin or amoxicillin; penicillin allergic patient are treated with Cephalexin if the reaction
only rash , if allergy is anaphylactic use clindamycin or a macrolides
Reference: Master the Board step 2 CK and this link
http://www.wikidoc.org/index.php/Tonsillitis_medical_therapy
2. 17 years old male presented with hearing loss in the left ear (picture of conductive hearing loss Audiogram). What is your diagnosis?
A.
Presbycusis
B.
Otosclerosis
C.
Interosseous
D.
Otitis media
Answer: B

smle ,2016

762
Otosclerosis is traditionally diagnosed by characteristic clinical findings, which include progressive conductive hearing loss, a normal tympanic membrane, and no evidence of middle ear inflammation.
3. How to examine child ear? Pull pinna?
A.
Inferior backwards
B.
Superior backward
C.
Inferior forward
D.
Superior forward
Answer A
B for adult.
Reference: http://www.meddean.luc.edu/lumen/meded/medicine/pulmonar/pd/pstep18.htm
4. 38 y.o pregnant female had progressive mixed type hearing loss, has mother had the same history
at 40, what is the diagnosis?
A.
Acoustic neuroma
B.
Presb
C.
Perforated TM
D.
Otosclerosis
Answer: D
Otosclerosis: fusion of stapes footplate to oval window so it can't vibrate. Its AD, more common in Female; progress during pregnancy ( hormone responsive)
Reference: Toronto note
5. Painful ear, fever with TM having hemorrhagic vesicles on PE. Whats the cause?
A.
Otitis externa
B.
Fungal
C.
Streptococcus pyogenes
D.
Pseudomonas
Answer : D
Myringitis is a form of acute otitis media in which vesicles develop on the tympanic membrane. Myringitis can develop with viral, bacterial (particularly Streptococcus pneumoniae), or mycoplasmal otitis
media.
Reference: www.merckmanuals.com
6. A patient with unilateral parotid swelling, hes postcholecystectomy. Saliva was cloudy ( I think)
Culture from parotid saliva was negative
A.
Sarcoid granulo
B.
Bacterial
C.
Cancer
D.
Sjogren syndrome
Answer: B
read about Post surgical Parotitis
Postoperative parotitis is a well known entity which can develop in patients who undergo major abdominal surgery.
Reference: http://www.ncbi.nlm.nih.gov/pmc/articles/PMC1410208/
7. How to assess hearing in adult:
A.
recording music

smle ,2016

763
B.
recording dog parking
C.
sound of watch
D.
paper flapping
Answer: C
Reference: AAFM
8. 6 years old female underwent tonsillectomy and started bleeding 24 hours after surgery. You examined the surgical site and found no abnormality. Her platelet count, bleeding time, platelet function, PT, aPTT and clotting time are all normal, what test confirms her diagnosis:
A.
Fibrinogen
B.
Plasminogen
C.
Clot lysis test
D.
vWF
Answer:C
9. most sensitive test for maxillary sinusitis?
A.
CT
B.
US
C.
Transillumination
D.
X-Ray
Answer: A
The criterion-standard imaging modality in the diagnosis of sinusitis is CT http://emedicine.medscape.com/article/384649-overview
10. Otalgia, fever , sore throat? what is the nerve that refers ear pain ?
A.
Glossopharyngeal
B.
Vagus
C.
Sphenopalatine
D.
Nasopalatine
Answer : A
if the scenario suggest the referred pain is most likely due to tonsillitis or pharyngitis the answer is (A)
11. pt with thyroid goiter ,,,compress on external laryngeal nerve ,, what is the action affected:
tension of vocal cords
abduction of vocal cords
adduction of vocal cords
Loss of sensation superior to vocal cords
Answer: A
12. child with ear secretion with defenses and move pinna what is DX?
Otitis media
Cholesteatoma
Otitis externa
foreign body
Answer: A
http://www.healthline.com/health/ear-infection-acute
13. What is the most sensitive part to the Linear Acceleration?

smle ,2016

764
Semicircular Canal.
Organ of corti.
Utricle.
Saccule.
Answer: C
The otolith organs sense gravity and linear acceleration such as from due to initiation of movement in a
straight line. Persons or animals without otolith organs are imbalanced.. A set of hair cells are coupled to
a mass of stones. When the stones accelerate, with respect to the hairs, they exert a shearing force on
the hairs. This force is detected by the hair cells and sent to the brain via branches of the vestibular
nerve. The utricle sends input to the brain via the superior division of the nerve, and the saccule, via the
inferior division. There is considerably more complexity to the organization of the utricle and saccule,
including different types of hair cells and detail to the sensory macule (patch of sensory cells) that we
have omitted.
The otolithic organs sense motion according to their orientation. The utricle is largely horizontal in the
head, and largely registers accelerations acting in the horizontal plane of the head (called the axial plane
by radiologists). The saccule is largely vertical, actually parasagittal, in the head, and registers accelerations in the vertical plane (called parasagittal or coronal plane).
otolithic organs= the saccule and utricle >> for linear acceleration
Reference: http://www.tchain.com/otoneurology/disorders/bppv/otoliths.html + medscape
14. 2-year old came with history of ear pain and reddens decrease tympanic membrane mobility you
diagnose otitis media start amoxicillin patient come to you after 3 week in examination there is fluid
behind tympanic membrane a limited mobility just in insufflation What is the next step in management ?
Waiting watchful
Decongestant
Continue antibiotic
Answer: A
Case of OM with effusion ttt by observation ( 90% resolve in 3 months) - Toronto note.
15. Elderly patient with sudden onset hearing loss, loss of balance, and loss of eye blinking, where is
the tumor?
Foramen Ovale
Foramen Secundum
Acoustic neuroma
Answer: C ; Acoustic neuroma or called Schwannoma
http://www.nidcd.nih.gov/health/hearing/pages/acoustic_neuroma.aspx
16. Neck infection can spread to mediastinal through
Carotid artery
Parapharyngeal space
Retropharyngeal space
Answer: C
17. Case fall then bruise and laceration over the nose, X-ray shows no fracture or displac , what to do ?
Ask for CT
Refer to ENT

smle ,2016

765
Nasal packing
Answer: ?? Could be B
18. 40 years old male, complaining of sudden dizziness attack for 30-40 min, associated with nausea,
vomiting, unilateral tinnitus and hearing loss.
Acoustic neuroma
Neuritis
Meniere disease
Answer: C
Menieres disease: a cause of recurrent vertigo with auditory symptoms more common among females.
Hx/PE: Presents with recurrent episodes of severe vertigo, hearing loss, tinnitus, or ear fullness, often
lasting hours to days. Nausea and vomiting are typical. Patients progressively lose low-frequency hearing
over years and may become deaf on the affected side.
Reference: 3rd Edition UQU > ENT > Q 104.
19. Man complaining of ear pain. On examination he feel pain when moving ear pinna and there is
erythema of auditory canal and normal tympanic membrane. What is the most likely diagnosis ?
Otitis Externa.
Otitis Media.
Squamous cell carcinoma.
Answer: A
20. PT have sleep apnea & on pacemaker not responding to CPAP (continuous positive airway pressure) what you do ?
Nose surgery
Chest surgery
Do C-Pap again
Answer: C
CPAP is the best initial management. If failed, bilevel positive airway pressure (BiPAP) device then an
oral appliance (OA) therapy
21. 50 year old Male Patient presented with history of unilateral nasal obstruction and epistaxis. Ct
scan done and shows soft tissue filling the whole nose in that side What is the cause of these symptoms?
nasal polyps
angiofibroma
malignant tumour
Answer: B
Reference: http://emedicine.medscape.com/article/872580-overview
22. 18 years old ear examination is normal and there is audiogram picture, what's the diagnosis?
presbycusis
Otosclerosis
otitis media
Answer: B
Since there is normally ear examination we exclude OM , and the age of the patient also excludes presbycusis so most likely to be Otosclerosis! Btw we need to see Audiogram result.

smle ,2016

766

23. Most common or specific sings for otitis media:


Pain
Hearing loss
Discharge
answer : A
most common: pain. most sensitive: immobility
reference: Master the Board , UQU
24. pt is continuously exposed to loud sounds over an extended period of time what will he have ?
conductive deafness
non-neuronal hearing loss
vomiting
Answer : noise induced SNHL ?
25. ENT patient with pinna pain..?
Otitis media
Otitis externa
Cholesteatoma
Answer : B
26. prevent symptoms of allergy to pollens and dust by using what:
Terbutaline
Montelukast
Ipratropium bromide
Answer: B
Reference: Toronto note
27. patient with an URTI with hoarseness and painful swallowing, where is the site of infection
larynx
bronchiole
trachea
Answer : A
28. pt in his work exposed to noise what type of hearing problem may has:
conductive hearing loss
non sensory hearing loss
sensory hearing loss
Answer : C
sensorineural hearing loss
29. Unilateral obstruction nose then foul smell,nasal discharge , erosion of vestibule , normal in exam
of nose
x ray head and chest
nasal exam in GA
antibiotic
Answer: this is a case of rhinosinusitis
Reference: http://www.uptodate.com/contents/acute-sinusitis-and-rhinosinusitis-in-adults-treatment

smle ,2016

767

30. Ear pain . Fever , Most common bacteria ?


S.pneumonia
H.influenzae
N.meningitidis
Answer: A
AOM
Reference: http://www.uptodate.com/contents/acute-otitis-media-in-adults-suppurative-and-serous?source=outline_link&view=text&anchor=H10#H10
31. Which of the following is indication for tonsillectomy:
Retro pharyngeal abscess
Obstructive apnea
Large asymptotic tonsils
answer: B
32. In case of epistaxis whats first thing to do?
Insert tampon
Let him lie on his side lateral
Pinch the fleshy part provide pressure
Answer: C
33. adolescent with epistaxis for 10 min with no Hx of trauma, on examination: there was posterior
oozing, Rx?
Conservative
posterior tampon,
constrictive spray
Answer: B?
34. child has an URTI then has ear pain , rinne test is negative but weber test show the sound is louder
in the affected area
Otitis media
Mastoditis
cholesteatoma
Answer: A
35. a question about superior laryngeal nerve if got injured what will be affected?
sensation above vocal cord
Tension of vocal abduction
Adduction
Answer: B
Reference: http://emedicine.medscape.com/article/1923100-overview
36. swelling between ear and base of jaw with inflammation,what is the complication of it ?
facial N paralysis
hearing loss
encephalitis
Answer: if the Q suggest Mumps the answer will be C

smle ,2016

768

37. Young patient with conductive hearing loss and semicircular canal dehiscence?
Glue ear
Otosclerosis
Tympanosclerosis
Answer :
38. Nerve supply to tensor tympani and stapedius :
Trigeminal and facial
Facial and auditory
Trigeminal and ....
Answer : A
39. Patient presenting with hearing loss , dizziness , tinnitus, vertigo what are you going to do to locate the lesion ?
Audiogram
MRI cerebellopontine
CT of temporal area
answer : C
40. patient presented with vertigo what is the cause:
ethambutol
streptomycin
Ionised
Answer : B
Reference:
http://www.drugs.com/sfx/streptomycin-side-effects.html
41. pt has congestion in tonsils and peritonsillar and c/o pain and can't open mouth ?
Quinsy
peritonsillar abscess
Tonsillitis
Answer: A and B
Quinsy also known as a peritonsillar abscess
42. case scenario dizziness vertigo progressive hearing loss what to see in CT?
Hemorrhage
Stroke
Normal CT
Answer: C
43. old patient with epistaxis when he went from outside to home in winter " cold to warm place"
what you gonna give him:
nasal steroid
send home
ephedrine nasal spray "not sure about this option"
Answer: C

smle ,2016

769
Hot and dry environments The effects of such environments can be mitigated by using humidifiers,
better thermostatic control, saline spray, and antibiotic ointment on the Kiesselbach area.
Reference: http://emedicine.medscape.com/article/863220-overview#a5
44. Child with painful ear and runny nose and mild cough what will u give him?
Antibiotic
Antihistamine and decongestant
Paracetamol
Answer: A
source: http://emedicine.medscape.com/article/994656-overview
45. most common cause of tinnitus is:
hypertension
chronic use of salicylate
sensorineural deafness
Answer: C
subjective tinnitus ( only heard by the Pt ):
common otologic cause is: Presbycusis. common drug cause: ASA
objective tinnitus ( can be heard by others ):
benign intracranial HTN
reference: toronto note and this link
http://www.webmd.com/a-to-z-guides/understanding-tinnitus-basics
46. the most common complication of mump?
Hearing loss
Encephalitis
Sterility
Answer : CNS involvement is the most common extrasalivary complication of mumps.
Reference: http://reference.medscape.com/article/966678-overview

47. man working in factory , loud noises , his father and grandfather worked there. Hearing loss both
of them. What to tell him ?
use cover protection for ear
familial hearing loss
Answer: A
48. 40 years old man presents with decreased hearing but his own sound feels louder than before to
him. What is diagnosis?
presbycusis
otitis media
Answer: B, but if there is osteosclerosis it will be more appropriate.
this a case of Otitis Media with Effusion
child with middle ear fluid and bulging in ear drum the RX?
tymbanosomy
grommet
Answer: surgery: myringotomy ventilation tubes adenoidectomy (if enlarged or on insertion of second set of tubes after first set falls out)

smle ,2016

770
ventilation tubes to equalize pressure and drain ear
Reference: Toronto note
49. Most sensitive test for maxillary sinus:
CT
X-ray
Answer: A
50. A 17 years old male complains of sore throat, cough, fever, ear pain. On examination you found
hemorrhagic vesicles over his tympanic membrane, what's the causative organism:
Pseudomonas
Strept. pyogenes
Answer:
This is called Myringitis
(particularly Streptococcus pneumoniae), or mycoplasma otitis media.
Reference:http://www.merckmanuals.com/home/ear,-nose,-and-throat-disorders/middle-ear-disorders/infectious-myringitis
51. Patient came to ER with inability to breath from one nostrils and on examination the nostril is
edematous and swollen What's the initial management?
Local decongestant
Steroid
Answer: B

smle ,2016

771

Refrencee: http://www.aafp.org/afp/2010/0615/p1440.html
52. pt with recurrent episodes of vertigo , she feels of ear fullness , SNHL and nausea , what is the dx?
Meniere's diseases
Benign Progressive vertigo
Answer: A
Refrencee: http://www.nidcd.nih.gov/health/balance/pages/meniere.aspx
53. scenario about adult male complaining of severe ear pain with discharge inflamed tympanic membrane ( missed some points) What is the Dx?
otitis externa
cholesteatoma
Answer: B
The majority (98%) of patients with cholesteatoma have ear discharge or hearing loss or both in the affected ear.
Reference: https://en.wikipedia.org/wiki/Cholesteatoma
O.E is not associated with tympanic membrane infection
54. case of serous otitis media with effusion , how to relieve symptoms ?
Myringotomy
Grommet insertion

smle ,2016

772
Answer: A
Reference: Toronto note
55. Runny nose and unilateral nasal obstruction for more than one year, on exam the inferior turbinates were swollen. Whats the most likely dx?
Allergic rhinitis
Sinusitis
Answer: A
Turbinates swollen : allergic rhinitis
56. pt has fronciosis in the left side of the nose, then he developed orbital edema. Which vien will
carry this effect ?
maxillary
ophthalmic
Answer: B
ophthalmic vein is the most accurate answer, if it not one of choices then ethmoidal , if both are not
present choose: Ophthalmic A.
57. Child complaining of painless unilateral hearing loss. On examination, tympanic membrane was
erythematous
serous otitis media
acute otitis media
Answer: A
reference: AAFM
History

Physical Findings

suggested cause of
CHL

sudden painless

cerumen

complete canal occlusion

sudden painful

narrow canal with debris

OE

normal canal with red, immobile TM

COM

gradual painless

Normal TM

Otosclerosis

Reddish- blue pulsatile mass behind intact TM

glomus tumor

perforated TM

cholesteatoma

case of otitis media what is complication :


encephalitis
hearing loss
answer: B

smle ,2016

773
Extracranial: hearing loss, speech delay, perforation, extension of suppurative process to adjacent
structures ( mastoiditis, petrositis, labyrinthitis ), cholesteatoma, facial nerve palsy, ossicular necrosis,
vestibular dysfunction.
Intracranial: meningitis, epidural and brain abscess, subdural empyema, lateral and cavernous sinus
thrombosis, carotid artery thrombosis
Year old man with chronic unilateral nasal obstruction , imaging shows fleshy mass
polyp
chronic forging body effect
answer : polyp
rhinorrhea, cough and conjunctivitis etiology ?
rhinovirus
adenovirus
answer : B
child with croup what is the best initial investigation to diagnosis5.
chest x-ray
pharyngeal swab
Answer:A
chest x ray .(croup is clinically diagnosis (parking cough and stridor) CXR in atypical presentation steeple sign from subglottic narrowing ) from toronto note
child with middle ear fluid and bulging in ear drum, what is the treatment:
tympanostomy.
grommet.
Answer:
this is a case of otitis media with effusion and I think the question was about TTT

Complications of mastoiditis include the following:


Orchitis
Encephalitis
Answer: B
Wts responsible for angular acceleration sensation:
semicircular canal
organ of corti
Answer: A
case about deafness and paralysis of tympanic muscle and other muscle i can not recall it
trigeminal

smle ,2016

774
facial
Answer:
the q not clear mean The tensor tympani is a muscle within the ear,Innervation of the tensor tympani is
from the tensor tympani nerve, a branch of the mandibular division of the trigeminal nerve
https://en.wikipedia.org/wiki/Tensor_tympani_muscle
if the q mean tympanic membrane so, the nerve is facial N.
Pharyngitis and tonsillitis pain from
Sphenopalatine nerve
Glossopharyngeal nerve
Sensory: Each of the three sections of the pharynx have a different innervation:

The nasopharynx is innervated by the maxillary nerve (CN V2).

The oropharynx by the glossopharyngeal nerve (CN IX).

The laryngopharynx by the vagus nerve (CN X).


Answer :MOST LIKELY (B)
20 year boy snoring ... he has enlarged tonsils treatment of obstructive sleep apnea .
Cpap.
reduced weight...
REFERENCE: OM ALQORA
IF there is tonsillectomy choose it IF not present CPAP will be appropriate since choices not complete
here
cavernous sinus thrombosis which vein drainage?
Maxillary
Ophthalmic
Answer:not clear question ,but thrombophlebitis in facial vein can easily cause cavernous venous thrombosis. Read about venous drainage of this sinus.
diabetic with dark color nasal discharge
cryptococcosis
zygomycosis
Answer: regarding dark colour discharge :Aspergillosis, blastomycosis and mucormycosis are
types of fungal infections that can cause sinus or lung infections. Mucormycosis is more common in diabetics .
Pt with history of pinna pain with discharge from the ear, best mx?
Oral feuroxim
Topical neomycin
Answer: b
Sources:http://emedicine.medscape.com/article/994550-overview
right peritonsillar abscess and fever
acute quincy ++
retropharyngeal abscess( complication)
Answer: the question is not clear .
Pt with submandibular mass that increase in size with eating Diagnosis ?

smle ,2016

775
Acute sialadenitis
Canal calculi
Answer: B (sialothiasis) and is painful swelling.
retired man used to work in factory with sensorineural hearing loss ?
Noise related
Answer: may be the question was about the cause,not complet question.
What is the most common site of malignancy in paranasal sinus?
Maxillary
Answer: A
Reference: 3rd Edition UQU > ENT > Q 92.
Child complaining of painless unilateral hearing loss. On examination, tympanic membrane was
opaque.
serous otitis media
Answer: A
http://www.cochlea.eu/en/pathology/conductive-hearing-loss very nice page .

Post tonsillectomy patient has loss of taste of the posterior 1/3 of tongue. What is the nerve injured?
Glossopharyngeal nerve injury
Answer: A

The Posterior third is innervated by: Sensory branches of the Glossopharyngeal Nerve
(CN IX), for sensation and taste. Motoric branches of the Hypoglossal Nerve(CN XII)
In presbycusis (SNH loss). Which structure in the inner ear gets affected?
Hair Cells.
Answer: A
http://reference.medscape.com/article/855989-overview to know more about presbycusis.
sore throat, palpable neck mass, hoarseness voice, hearing loss Ask about tx?
surgery and radiotherapy
(I thought about nasopharyngeal ca)
Answer: I dont know

from toronto note

Otitis media case + child with acute ear pain , redness and bulging of tympanic membrane, what's the
Dx
otitis media
Answer:

smle ,2016

776
Clinical Features:
triad of otalgia, fever (especially in younger children), and conductive hearing loss
rarely tinnitus, vertigo, and/or facial nerve paralysis
otorrhea if tympanic membrane perforation
infants/toddlers ear-tugging (this alone is not a good indicator of pathology) hearing
loss, balance disturbances (rare) irritable, poor sleeping vomiting and diarrhea anorexia
otoscopy of TM hyperemia bulging, pus may be seen behind TM loss of landmarks: handle and long process of malleus not visible
Ref:Toronto note

treatment of OM in children?
Amoxicillin
Answer: A
Antimicrobial agents for AOM 1st line treatment (no penicillin allergy) amoxicillin: 75 mg/kg/d to 90
mg/kg/d divided 3x/d
Ref: Toronto note
Antibiotics should be prescribed for bilateral or unilateral AOM in children aged at least 6 months with
severe signs or symptoms (moderate or severe otalgia or otalgia for 48 hours or longer or temperature
39C or higher) and for nonsevere, bilateral AOM in children aged 6-23 months
On the basis of joint decision-making with the parents, unilateral, nonsevere AOM in children aged 6 -23
months or nonsevere AOM in older children may be managed either with antibiotics or with close follow-up and withholding antibiotics unless the child worsens or does not improve within 48-72 hours of
symptom onset
Amoxicillin is the antibiotic of choice unless the child received it within 30 days, has concurrent purulent
conjunctivitis, or is allergic to penicillin; in these cases, clinicians should prescribe an antibiotic with additional -lactamase coverage
Clinicians should reevaluate a child whose symptoms have worsened or not responded to the initial antibiotic treatment within 48-72 hours and change treatment if indicated
In children with recurrent AOM, tympanostomy tubes, but not prophylactic antibiotics, may be indicated
to reduce the frequency of AOM episodes
Clinicians should recommend pneumococcal conjugate vaccine and annual influenza vaccine to all children according to updated schedules
Clinicians should encourage exclusive breastfeeding for 6 months or longer
http://emedicine.medscape.com/article/859316-treatment
PT complaining of aphonia & doing laryngoscope it was normal & ask him to coughing he is cough
what diagnosis?
Functional aphonia
Answer: A
https://sites.google.com/site/drtbalusotolaryngology/Home/laryngology/functional-aphonia

Most common cause of deafness in children is


recurrent Otitis media
Answer: A
http://www.asha.org/public/hearing/Hearing-Loss/ i could not find a sourse for most common cause
particularly.

smle ,2016

777

Otitis media pt. Not taking antibiotic probably then after week came back by tenderness and swelling
behind the ear,dx?
Acute mastoiditis
Answer: A

Long scenario ear pain discharge red edematous auditory canal


Otitis externa
answer : A

smle ,2016

778

Neck mass; pathology report: ki67 +ve What else you expect
BCl2 overexpression
Answer: burkitt lymphoma
Boy with unilateral nasal obstruction , foul smelling dx?
Foreign body
Answer: A ..typical question and typical case .
Child c/o recurrent nasal congestion, rhinorrhea, sneezing , tearing eyes, dx?
allergic rhinitis
Answer: A
parotid duct obstruction and pain during eating in ear what nerve responsible for this pain
Facial Nerve
32 years old female, presenting with decrease in hearing, her mother lost hearing in her 30s. Dx?
acoustic Neuroma
Answer: A?

smle ,2016

779
case of boy recently bought a cat developed allergy :
Allergic rhinitis
Answer: A
EBV pharyngitis what is treatment :
Iv acyclovir
Answer: A
http://emedicine.medscape.com/article/222040-overview
Case battery lodged at esophagus wat to do :
Urgent endoscopy
Answer:
Nasal drop vasoconstrictor?
Rebound phenomena
Answer: https://en.wikipedia.org/wiki/Rhinitis_medicamentosa
Hemorrhagic vesicles on tympanic membrane
Answer: Bullous Myringitis
source:
http://www.msdmanuals.com/professional/ear,-nose,-%20and-throat-%20disorders/middle-ear%20and-%20tympanic-membrane-%20disorders/myringitis
Innervation of stapedius ?
Answer: Facial
The stapedius emerges from a pinpoint foramen in the apex of the pyramidal eminence (a hollow, coneshaped prominence in the posterior wall of the tympanic cavity), and inserts into the neck of the stapes.
Innervation: The stapedius is innervated by the nerve to stapedius, a branch of the facial nerve.
chronic OM with green discharge oxidase +ve organism ?
Answer: pseudomonas
Child with red bulging tympanic membrane, ear pushed down and forward, what is the dx?
Answer : Mastoiditis - From Lecture note
Intracranial growth compressing the maxillary artery. Which one of the following is affected?
Answer:
The maxillary artery can be defined as one of the continuations of the external carotid artery, and distributes the blood flow to the upper (maxilla) and lower (mandible) jaw bones, deep facial areas, cerebral dura mater and the nasal cavity. Hence it is considered a blood vessel which supports both hard and
soft tissues in the maxillofacial region.
NB Know the anatomy and branches.
Old guy cant tolerate loud noises which nerve is affected?
Answer:
Facial nerve due to loss of protective muscle (stapedius)
A patient with chronic tonsillitis complaining of fever, chills, left throat pain and uvula deviation.
What is the diagnosis?

smle ,2016

780
Answer:
Peritonsillar abscess
60 year old male with unilateral parotid swelling tender but no facial nerve dysfunction?
Answer:
If there is fever along with the tenderness the answer would be bacterial parotitis , if no systematic Sx
usually there will be no tenderness where the answer would be Benign pleomorphic adenoma , facial
nerve involvement would rise the suspicion toward malignancy .
Reference: Medscape
Nasal obstruction + rhinorrhea + pale swelling
Answer: Allergic Rhinitis
What is the artery supply posterior inferior nasal septum ???
Answer: sphenopalatine artery
The most common cause of otitis media in all age group ?
Answer: streptococcus pneumonia, haemophilus influenzae, moraxella catarrhalis.
Reference: AAFM
Thick White plaques in throat associated with gingiva, dx?
Answer: Could be fungal infection ?
What causes mixed hearing loss ?
Answer: Causes of mixed hearing loss vary wildly. Typically, the sensorineural hearing loss is already present and the conductive hearing loss develops later and for an unrelated reason. Very rarely, a conductive hearing loss can cause a sensorineural hearing loss.http://www.coastalhearingcenters.com/mixedhearing-loss/
Child complaining of painless unilateral hearing loss. On examination, tympanic membrane was
opaque. serous otitis media what is the management !!
Answer :
this is a case of otitis media with effusion

smle ,2016

781

red erythematous epiglottis...causative organism


Answer: H.influenzae tybe b
Best treatment for allergic rhinitis.
Answer: ?
Prevention with avoidance of the precipitating allergen:
Close the windows and use air conditioning to avoid pollen.
Get rid of animals to which the patient is allergic.
Cover mattresses and pillows.
Use air purifiers and dust filters.
Intranasal corticosteroid sprays
Antihistamines: loratidine, clemastine, fexofenadine, brompheniramine
Intranasal anticholinergic medications: ipratropium
Desensitization to allergens that cannot be avoided
Ref: Master the boar
Case of shingle ,what is diagnosis
Patient had trauma then had vertigo, dizziness and unilateral decreased hearing . what is the area affected?
Auditory assessment sheet, otosclerosis?
Dx of peritonsillar abscess , hot potato voice
Child felt from 10 meter height with bleeding from his ears, where is the injury? FRACTURE BASE OF
THE SKULL?

smle ,2016

782

115-I think truma >> clear nasal discharge which affect


a) optic n
b) olfactory n
c) ophtham- ... )
Answer: B
Explanation: Clear nasal discharge is a hint of CSF due to fracture of the cribriform plate bone which contains nerve fibers of the olfactory nerve.

116-patient had a RTA on x-ray there was multiple forehead fractures , and there is nasal discharge , what nerve is affected ?
optic
olfactory
ophthalmic
Answer???
117-Pt came to ER with jaw fracture plan for reduction and fixation which of the following
muscle forcefully close the jaw ?!
Lateral pterygoid
Masster
Temporalis
Answer: The muscles that power the jaw movements during chewing are known as the
muscles of mastication or masticatory muscles and are functionally classified as;[1]
Jaw elevators: the masseter, temporalis, medial pterygoid, and superior belly of the lateral pterygoid;
Jaw depressors; the anterior digastrics, geniohyoid, mylohyoid and inferior belly of the lateral pterygoid.

https://en.wikipedia.org/wiki/Masticatory_force
118-Man fall down from stairs on his face with many fracture on his jow . Which muscle help
in mastication
masseter
temporalis
Answer: A
119-frontal bone fracture Or infection . Examine which Lymph Nods ?
Submental
submandibular

smle ,2016

783

Ophthalmology

smle ,2016

784

1- Painful vision loss?


a.Acute angle closure glaucoma
Answer: A
Reference: Lecture note on ophthalmology, 9th edition

2- Picture of snellen's test.What is the visual level of this patient?


Answer:

e.g. A patient can read until the 3rd line, what is the visual acuity? 20/70
3- A known case COPD and DM, diagnosed to have primary open angle glaucoma. What is the optimal
treatment for glaucoma?
a. Topical Timolol
b. Topical Carteolol
c. Systemic Acetazolamide
Answer: C
4- 24 years old female newly diagnosed type 2 DM, she is wearing glasses for 10 years, how frequent
she should follow with ophthalmologist?

smle ,2016

785
a. 6 months
b. 12 months
c. 5 years
d. 10 years
Answer: B
For type 1 diabetic: retinal screening annually beginning 5 years after onset of diabetes, general not before onset of puberty.
For type 2 diabetic: screening at the time of diagnosis then annually.
Reference: 3rd Edition UQU > Ophthalmology > Q 78.
5- Lacrimal gland tumor lead to proptosis in which direction?
a.Down\in
Answer: A
Maxillary sinus growth: Superior
Lacrimal gland tumor: Inferomedial
Frontal or ethmoidal sinus lesion: Inferolateral
Reference: Textbook of Ophthalmology
6- A patient with unilateral painful vision loss.What is the cause?
a-papilledema
b-toxic amblyopia
c-optic neuritis
Answer: C
Explained previously, see Q1
7- A patient with eye movement abnormalities and ptosis.What is the nerve is involved?
a-3rd
b-4th
c-6th
Answer: A
Oculomotor (3rd cranial nerve): Eye movement (SR, IR, MR, IO), pupillary constriction (sphincter pupillae: Edinger-Westphal nucleus, muscarinic receptors), accommodation, eyelid opening (levator palpebrae)
Loss of levator palpebrae function will cause ptosis
Trochlear (4th cranial nerve): Eye movement (SO)
Abducens (6th cranial nerve): Eye movement (LR)
Reference: FA USMLE step 1
8- Iris neovascularization caused by?
a-Non-proliferative diabetic retinopathy.
b-Central retinal vein occlusion
Answer: B
Abnormal iris blood vessels (neovascularization) may obstruct the angle (secondary glaucoma) and
cause the iris to adhere to the peripheral cornea, closing the angle (rubeosis iridis). This may accompany
proliferative diabetic retinopathy or central retinal vein occlusion due to the forward diffusion of vasoproliferative factors from the ischaemic retina.
Reference: Lecture note on ophthalmology, 9th edition

smle ,2016

786

9- What is the treatment of chronic ptosis?


Answer:
http://eyewiki.aao.org/Aponeurotic_ptosis#General_treatment
10- Patient came for annual check up & found to have cupping of disk.What is the diagnosis?
a-Retinal detachment
b-Diabetic retinopathy
c-Chronic open angle glaucoma
Answer: C
11- Ring lesion in eye?
a-Acanthamoeba infection
Answer: A
12- Picture of a patient with corneal ulcer.What is the treatment?

a-Contact lens
Answer:
The Contact lens is one of the causes of corneal ulcers.
The Question could be similar to the next (Q13)
13- Corneal epithelial defects (CED) (Another term of corneal abrasion or ulcer) due to fingernail injury.What is the treatment?
a-Double patch.
Answer: A

The answer confirmed by an intern who had the same Q in his exam and got the full mark in
ophthalmology section.
14- Corneal epithelial defects (CED), in addition to this presentation, what else this patient might
have?
a-Photophobia
b-Visual loss
Answer: A
15- Long term topical steroid drops can cause:
A- Glaucoma.
B- Posterior subcapsular cataract

smle ,2016

787
Answer: it will cause steroid induced glaucoma
16- A patient known case of COPD and glaucoma complaining recently of intermittent cough?
a-Pitoxolol
b-Timolol
Answer: B
Timolol is A Nonselective B blocker used to treat glaucoma by decrease secretion of aqueous humor.
Should be avoided in asthma/COPD patient due to increase risk of exacerbation. Reference: FA USMLE
step 1
17- How to administer eye drops and ointments?
a-One drop in the lower fornix
Answer: A
18- A patient came with drooling and lacrimation (cholinergic reaction). What to give?
a-Physostigmine
Answer:
NB. No atropine in the choices
*Physostigmine is NOT the right answer, it is an acetylcholinesterase inhibitor (It can be the cause of the
cholinergic reaction)
19- Trabeculectomy is an operation used for which of the following conditions?
a- Open angle glaucoma
b-Closed angle glaucoma
Answer: A
Trabeculectomy is effective for chronic angle-closure glaucoma. However, compared to primary openangle glaucoma, any aqueous-draining procedure in an eye with a shallow anterior chamber and a
chronic closed angle poses the risk of further shallowing the anterior chamber or precipitating malignant
glaucoma.
Reference: http://emedicine.medscape.com/article/1205154-treatment#showall
20- A patient with painful swelling (dacryocystitis). What is the best management?
a-Oral antibiotic
b-Drain surgically
Answer: B
In general, dacryocystitis is a surgical disease. Surgical success rates in the treatment of dacryocystitis
are approximately 95%. Acute cases are best treated surgically after the infection has subsided with adequate antibiotic therapy.
For acute dacryocystitis, an external dacryocystorhinostomy is preferred after several days of initiating
antibiotic therapy.
Rarely, dacryocystorhinostomy must be performed during the acute phase of the infection to facilitate
clearing of the infection. Reference: Medscape.
21- A patient has painless vision loss for 3 days?
a-Retrobulbar optic neuritis
b-Uveitis
Answer:

smle ,2016

788
Explained previously, see Q1. Reference: Lecture note on ophthalmology,
9th edition
Retrobulbar optic neuritis: Pain with eye movement in retrobulbar optic neuritis because rectus contraction pulls on the optic nerve sheat.
Uveitis: http://emedicine.medscape.com/article/798323-overview
22- A patient with follicular keratosis. What will you check?
a-Eyes
Answer: A
Because of Vit. A deficiency
Keratosis pilaris is another name of follicular keratosis
Reference: http://disorders.eyes.arizona.edu/category/clinical-features/keratosis-pilaris
23- A patient with increased ICP. What will you check?
a-Papilledema (optic nerve swelling)
Answer: A
24- Cavernous sinus aneurysm leading to diplopia & blurred vision, what nerve affected?
a-Trochlear
b-Abducens
c-Optic
d-Ophthalmic (V1)
Answer: B
Cavernous sinus syndromepresents with variable ophthalmoplegia, decrease corneal sensation,
Horner syndrome and occasional decreased maxillary sensation. 2 to pituitary tumor mass effect, carotid-cavernous fistula, or cavernous sinus thrombosis related to infection. CN VI is most susceptible to
injury. Reference: FA USMLE step 1
25- Eye drops that is contraindicated in acute closed angle glaucoma?
Answer: ?
1-agonists (Epinephrine) because of Mydriasis side effect.
Reference: FA USMLE step 1
26- A male patient presented with breast enlargement and decreased sexual desire. Labs: Hyperprolactinemia. What visual deficiency this patient might have?
a-Bitemporal hemianopsia
Answer: A
27- How to test visual acuity?
a-Snellen chart
Answer: A
28- A patient comes with signs and symptoms of glaucoma. What is treatment?
Answer:
IV acetazolamide, topical pilocarpine and B blocker. Reference: UQU

smle ,2016

789
29- Adult complaining of pain when moving the eye, fundoscopy revealed optic disc swelling. What
is the most likely diagnosis of this case?
a.Optic neuritis.
b.Central vein occlusion.
c.Central artery occlusion.
Answer: A
30- Father came with his 6 years old daughter to the clinic, she has esotropia. What you will do?
a.Glasses.
b.Surgery.
Answer: Q not completed (squint some time corrected by glasses and some time need surgery)
Nonsurgical treatments include patching, correction of full hyperopic refractive error (glasses)
then if not improved surgery.
31-A patient presented with pterygium. What is the possible complication?
a-Corneal scarring
b-Corneal perforation
c-Strabismus
Answer: A
32- 47 year old man with HTN & DM, past history of treatment of mycobacterial infection. He gave history of blurred and decreased vision. On examination there is a flame shape on the retina, cotton
wool spots and macular edema (Other investigations were included as well). What is the cause?
a.Retinal vein occlusion.
b.Ethambutol.
c.DM.
Answer: C
Diabetic retinopathy is common in patients with chronic poor glycemic control. Features include:
Microaneurysms: The earliest clinical sign of diabetic retinopathy
Dot and blot hemorrhages
Flame-shaped hemorrhages
Retinal edema and hard exudates
Cotton-wool spots
Venous loops and venous beading: Their occurrence is the most significant predictor of progression to
proliferative diabetic retinopathy (PDR).
Macular edema: Leading cause of visual impairment in patients with diabetes
Reference: http://emedicine.medscape.com/article/1225122-overview
Also flame shape hg on the retina it cased by hypertensive retinopathy
33- Diabetic patient with high cup to disc ratio. What is the cause?
Answer:
Patients with diabetes were previously thought to have a greater risk of developing primary chronic
glaucoma with loss of visual field. However, more recent papers suggest that diabetes is not a greater
risk factor, but simply that glaucoma was found more readily.

smle ,2016

790
Patients with PDR are at risk of developing secondary glaucoma, particularly neovascular (rubeotic)
glaucoma
Rubeosis iridis is the growth of new vessels on the iris in eyes with advanced retinal ischaemia. Rubeosis
neovascularization of iris ( NVI) may induce a severe form of intractable glaucoma with growth of new
vessels in the anterior chamber angle (NVA). If uncontrolled, NVA leads to closure of the aqueous fluid
drainage route in the anterior chamber angle of the eye by fibrovascular tissue
Reference:http://www.icoph.org/dynamic/attachments/taskforce_documents/2012-sci-267_diabetic_retinopathy_guidelines_december_2012.pdf
34 - pic Eyelid inflammation what most associated symptoms
a-secretion
b- vision loss

Answer: A
35 - pic Eye with entropion

Answer: Entropion is a malposition resulting in inversion of the eyelid margin.


36 - orbital cellulitis 2 times
Answer: Orbital cellulitis and preseptal cellulitis are the major infections of the ocular adnexal and orbital tissues. Orbital cellulitis is an infection of the soft tissues of the orbit posterior to the orbital septum, differentiating it from preseptal cellulitis, which is an infection of the soft tissue of the eyelids
and periocular region anterior to the orbital septum. Patients with orbital cellulitis frequently complain of fever, malaise, and a history of recent sinusitis or upper respiratory tract infection. Other
signs include ; Conjunctival chemosis, Decreased vision, Elevated intraocular pressure, Pain on eye
movement. The patient with orbital cellulitis should be promptly hospitalized for treatment, with hospitalization continuing until the patient is afebrile and has clearly improved clinically. Medical management is successful in many cases. Consider orbital surgery, with or without sinusotomy, in every
case of subperiosteal or intraorbital abscess formation.

smle ,2016

791

37 - What is the function of superior rectus muscle?


a-in down
b-out down
c-up in
d-up out
Answer: C
function of superior rectus muscle elevates , intorsion,and rotate medially the eye(Adduction)
Ref : https://quizlet.com/50874061/eyes-flash-cards/
38 - keratitis caused by parasites
Answer: Acanthamoeba
39 - For what test use distance eye
Answer: visual acuity
40 - patient complaint of red eye and decreased vision in left eye since 3 days, in examination there
are ciliary flush, not fully reactive pupil and floaters in anterior chamber. What is the diagnosis:
a-Uveitis
b-Glaucoma
c-Retinitis
d-Keratitis
Answer: A
41 - Watery discharge , congested eye preauricular lymph node enlargement, diagnosis?
Answer: viral conjunctivitis ( adeno? )
42 - Which of the following cause loss of color vision?
a- acute angle closure glaucoma
b- mature cataract.
c- severe iridocyclitis.
d- optic atrophy.
Answer : D
43 - Pic about periorbital swelling what other symptoms could the pt has?

a- vision
b-uveitis
Answer: decrease vision if they mean (eye cellulitis) in thi senario .

smle ,2016

792

44 - Red eye pic after pharyngitis and fever what is ur dx?


Answer: adenovirus conjunctivitis
45 - Patient had laser for refractive correction ..one of post op complications of this procedure is dryness of the eye..how to prevent this dryness?
a-blockage of lacrimal duct
b-blockage of lacrimal canal
c-blockage of punctua
d-blockage of lacrimal sac
Answer: c.
46 - Pt wake up with mucopurulent discharge and red eye,and thick mdre esh . what is d?
a-viral
b-bacterial
c-allergy
d-hypersensitive I think
Answer:B
47 - What is distichiasis
Answer: abnormal growth of lashes from the orifices of the meibomian glands on the posterior lamella of the tarsal plate
48 - pic of disc cup ask for diagnosis
Answer:

49 - pic of cornea after nail injury ask diagnosis


Answer: corneal abrasion?
50 - stage of hypertensive or increase iop cant remember ask about third stage optic change?
a-papilloedema
b-nipping of vein
c-something
d-another something
Answer:
Grade I : mild arteriolar narrowing
Grade II : arteriovenous crossing
Grade III : retinal hemorrhage, exudate, cotton wool
Grade IV : papilledema

smle ,2016

793

51 - Which layer remove excessive water from cornea


a- stroma
b- tear film
c-endothelium
d-epithelium
Answer : c
52 - 20 y.o had trauma to left eye, he has corneal ulcer, photophobia, and pain, how do you manage?
a- debridement with Burr and systemic antibiotics.
b- cotton swab debridement and local steroid.
c- irrigation, systemic antibiotics and cycloplegia.
d- local antibiotics, cycloplegia and referral.
Answer : D
53 - Bacterial keratitis occurs with contact lenses mc organism: (repeated)
a- S.aurea
b- pseudomonas aeruginosa
Answer: B
54 - Man got new glass , after one week he can not see well by the new glass , after examination
change the glass and referred to ophthalmologist.. What is the cause of change the old glass ?
a-Glucose in lens
b-Cataract
c-Glaucoma
Answer:A
55 - 2 q about the eye parasitic infection one is the diagnosis ( acanthamoeba ) and the other q what
other manifestation for the disease
Answer : skin manifestations : ulcers, nodules, or subcutaneous abscesses.
CNS : meningoencephalitis ;Mental status changes, Seizures, Hemiparesis, Fever, Headache, Meningismus, Visual disturbances, Ataxia, Nausea and vomiting, Hallucinations, Personality change, Photophobia,
Sleep disturbances.
Disseminated disease without CNS involvement may manifest as skin lesions, sinusitis, pneumonitis, or a
combination.
Reference : medscape
56 - Sudden eye swelling redness and pain, hazy cornea :
Answer: Acute angle closure glaucoma
Reference ; introduction to clinical emergency medicine text book. / uptodate
57 - long case of eye trauma then repair then developed most likely endophthalmitis done enucleation, found in the uvea lymphocyte and CD4 i think, what's is the cause? (Repeated)
a-cross reactivity
b-activation of lymphocytes
c-antigen release or something?
Answer: antigen release

smle ,2016

794

58 - Painless loss of vision in right eye with headache. First thing to do?
a- Prednisolone
b- CT
Answer Prednisolone
59 - commonest cause of iris neovascularization is :
a-DM
b-HTN
Answer A
60 - pic of glaucomatous cupping

Answer:
normally 0,2- 0,4 Physiological cupping =0.5
But could be pathological if there is abnormalities by the other investigation ot there is difference btw
two eyes
Pathological >0.5
61 - A young male complaining of redness and discomfort in his eyes during the exam time, his visual
acuity is 6/6 in both eyes unaided. Both the anterior and posterior chambers are normal. What is the
most likely diagnosis:
a-.Myopia
b-Anisometropia
c-Astigmatism
d-Hyperopia
Answer: D?
62 - Dilated RT pupil and constricted Lt while light on rt
a-Rt optic
b-Lt optic
c-Rt oculomotor
d-Lt oculomotor
answer : C
Afferent ---> optic nerve ----> when light shone on 1 eye both eyes are stimulated
Efferent ---> oculomotor nerve ---> constrict the ipsilateral pupil
63 - you did examination on elderly Hypertensive man you find increase in cupping he is not complaining of anything what is the cause of cupping ?
a-Acute angle glaucoma
b-HTN-dm

smle ,2016

795
c-Retinal detach .
Answer: B?
Acute angle glaucoma ----> painful
retinal detach ------> curtain like vision loss or floaters
64 - Case of chronic eye irritation and Watery secretion + eye Entropion
Answer: conjunctivitis
65 - Case of eye injury by fingernails Which stain is used ( blue eye stain)?
Answer:fluorescein stain
fingernails it is corneal abrasion
de-epithelized area stains with fluorescein
Ref: toronto note 2015
66 - Case of patient with progressive painless visual loss -8 6/18 is it
a-Physiological
b-Pathological
c-Curvature Index alized area sta
Answer: pathological
67 - Type of gonorrhea discharge in eye:
a-Purulent
b-Mucopurulent if chlamydia
Answer: A
68 - when to refer conjunctivitis to ophthalmologist :
a- photophobia
b- bilateral conjunctivitis
c- mucopurulent discharge
Answer: A
Ophthalmologist referral is indicated for any pt with conjunctivitis presented with one of the following Sx:
eye tenderness
Difficult seeing clearly
Difficult keeping the eyes open or sensitivity to light
Severe headache with nausea
Recent trauma to the eye
Use of contact lenses
Reference : uptodate
http://www.uptodate.com/contents/conjunctivitis-pinkeye-beyond-the basics?source=outline_link&view=text&anchor=H8#H8
69 - patient had eye pain and photophobia upon examination by slit lamp there was circumcorneal
congestion, keratic precipitates and posterior corneal opacity. what is the diagnosis:
a-anterior uveitis
b-acute keratitis
c-acute angle glaucoma"
answer : A

smle ,2016

796
ref: http://emedicine.medscape.com/article/798323-clinical#b4
70 - Complications of cataract surgery
a-Endophthalmitis
answer : A
71 - Long scenario Cherry red macula :
a-Central retinal artery occlusion
answer : A
72 - Long scenario about patient with eye problems, he have myopia, bilateral decreased vision,
glasses was advice, after month he became not satisfied, there is refractive error and astigmatism:
a- keratoconus
b- keratoglobus
c- keratectasia
d- kerato*****
answer: A
73 - pt wakes up from sleep with watery discharge from RT eye with redness. everything else is normal. what is the ttt?
a. topical steroid
b. topical Antihistamine
c. topical antibiotics
d. reassurance
answer: reassurance ? (viral) or (eye dryness)
74 - Conjunctivitis patient with bottom eyelash turned inwards?
a-Ectropion
b-Entropion
Answer : eyelashes turned inward: trichiasis
Lid margin turned inward: entropion
(Ref: http://emedicine.medscape.com/article/1212456-overview)
75 - diabetic female come to the Ophthalmologist with RT.eye pressure 44mmgh , left eye pressure 22
mmhg , examination of retina of rt eye show disc to rim ratio and nasal deviation of retinal vessels
.what is the most likely Dx.?
Answer= Glaucoma
76 - Picture of eyelid swelled there is pain and increased tearing .what is most likely sign ?
a- decrease vision
b- discharge
not correct Q
c- uveitis
d- scleritis "
Answer : eyelid swelling is not a specific sign for any of the choice
ref: http://www.webmd.boots.com/eye-health/guide/swollen-eyes
77 - Pt stik her eyes by her finger and came with pain burning and crying what the suspected symptom or complications :

smle ,2016

797
a-Blindness
b-Photophobia
c- 2 other
Answer : B
78 - symptom associated with corneal abrasion is: pain, blurred vision, tearing and
photophobia.
79 - complication include: bacterial keratitis, corneal ulcers, traumatic iritis, and recurrent erosion syndrome.
80 - AIDS , retinitis:
Answer: Cytomegalovirus.
reference: Toronto note
81 - URTI + conjunctivitis with hyperemia + watery discharge what other ocular symptom you expect:
epithelial nummular keratitis
Answer: A
82 - Patient with vesicles in forehead and supraorbital region for one day, wt will u do ?
a- Antiviral
b- Antiviral and refer to ophthalmologist.
c- Reassure
answer : B
83 - lateral movement of one eye of a child when you close the other eye
a-squint
b-nystagmus
Answer : A. (another name for strabismus)
84 - picture of Red eye.. conjunctivitis.. how to prevent transmission of infection?
a- Ocular blaster
b- Handwashing
c- Steroids ointment
Answer: B
85 - Pic of eye.. mother got tear in her eye by her daughter fingernail. . Treatment is:
a- Antibiotics drops
b- Antibiotics ointment
c- Steroids drops
d- Steroids ointment
Answer: B Antibiotic drops also, Antibiotics drops can be given
86 - treatment of orbital pseudotumor?
a- Radiation
b- Antibiotics
c- surgical excision

smle ,2016

798
d- Systemic steroid
Answer : D mainstay therapy of orbital pseudotumor is Systemic Steroid.
87 - Pt. Came complain of foreign body sensation in his eye after splash something on him.. after removing what should you give him?
a- Antibiotics oral
b- Antibiotics topical
c- Steroids oral
d- Steroids topical
Answer: B, Reference; UQU
88 - Child has red eye and tearing since birth .
a- Congenital glaucoma
b- herpes keratitis
c- bacterial conjunctivitis
answer : A

89 - Ectropion pic

answer: Ectropion is an abnormal eversion (outward turning) of the lid margin away from the globe.
90 - Pic of inflammation of upper eyelid what is di:
a-Say or tay
b- Chilotis
Answer: see pictures to match it with the description.
A chalazion is noninfectious obstruction of a meibomian gland causing extravasation of irritating lipid
material in the eyelid soft tissues with focal secondary granulomatous inflammation. Disorders that
cause abnormally thick meibomian gland secretions (eg, meibomian gland dysfunction, acne rosacea)
increase the risk of meibomian gland obstruction.
A hordeolum (stye) is an acute, localized swelling of the eyelid that may be external or internal and usually is a pyogenic (typically staphylococcal) infection or abscess. Most hordeola are external and result
from obstruction and infection of an eyelash follicle and adjacent glands of Zeis or Moll glands. Follicle
obstruction may be associated with blepharitis.
Reference : Mercks manual
91 - Patient presenting with red eye and ciliary flush, acute painful vision loss in his right eye, and
headache.His right eye had cloudy oedematous cornea, his left eye showed superior temporal rim

smle ,2016

799
cupping of the disc. Tonometer showed high IOP in the right eye, and normal IOP in left eye. Which of
the following is most likely explanation for his presentation
a-bilateral glaucoma.
b-uveitis in right eye and congenital anomaly in left eye
c-macular degeneration in right eye..
d- retinal detachment in right eye..
Answer: A
92 - Pt c/o watery discharge , irritation ( there is a picture ) ..
a-Entropion
b-Ectropion
c-Pterygium
In choices no glaucoma , conjunctivitis or uveitis
Answer:Incomplete
93 - Patient around 50 y with unilateral eye discoloration they give discretion of something in the
sclera I think ! What youll tell him ! I DONT KNOW
a-hes in high risk of visual loss
b-this is a manifestation of systemic disease
c- vitamin A deficiency
Answer : Incomplete , I think it is bitot spot and is associated with vit. A deficiency may be sclearal
nevious
94 - Neonate presented to the hospital with eye discharge ..the mother had a vaginal discharge before
delivery ,,, what is the organism
Answer : Gonorrhea
95 - Best to dx corneal abrasion ?
a- fluorescein eye test
b- Slit lamp test
c-Others
Answer: A
96 - Pic of eye with dendritic lesion Stained by fluorescein.. Management ?
a- Antiviral
b- Anti.... (Fungial)
c- Others
Answer: A
97 - picture of a patient with redness of the eye and tearing , what
is the most likely you will find ?
a- Papilla
b- Follicles
c- Papilloma
Answer: B
98 - Best distance between "Snellen Chart" & the patient ?

smle ,2016

800
a- 3 Meters
b- 4 Meters
c- 6 Meters * [NOT SURE 100%]
d- 9 Meters
Answer : C
99 - In a patient who is sensitive to light, your action is going to be:
Wear sunglasses to protect from sunlight
100 - What is the treatment for high myopia:
LASIK
Refractive surgery
Phakic Intraocular lens
Answer : C
101 - a patient is presenting with a history of excessive tearing, the picture is showing the eye of the patient.What is the
cause of tearing in this patient?
a.Entropion
Its an inward direction of the eyelids; causing excessive rubbing
of the eyelashes against the eye surface.
102 - Corneal ulcer is diagnosed by?
a.Slit-lamp biomicroscope with fluorescein staining
103 - The classes of medications that have the potential to induce acute angle closure are?
Certain group of medications are contraindicated in glaucoma patients as they might precipitate an
acute angle closure. The pathophysiology of drug-induced angle-closure glaucoma is usually increased
pupillary block (ie, increased iris-lens contact at the pupillary border) from pupillary dilation; which decreases the drainage of aqueous humor.
A- Topical anticholinergic.
B- Sympathomimetic dilating drops.
C- Tricyclic antidepressants.
D- Monoamine oxidase inhibitors.
E- Antihistamines.
F- Antiparkinsonian drugs.
G- Antipsychotic medications.
104 - Upon examining a patient you discover that he has a Follicular Keratosis. What do you expect
this patient to have?
Vitamin A deficiency
105 - You are seeing a baby in the clinic coming with crossed eyes . You notice that his left eye is deviated inwards with the right eye being normal, with no diplopia. What is your diagnosis?
strabismus
Answer:
An eye that is turned inwards is called an esotropia.
An eye that is turned outwards is called an exotropia.

smle ,2016

801
An eye that is turned upwards is called a hypertropia.
An eye that is turned downwards is called a hypotropia.
106 - A male patient sustained a blunt eye injury from a tennis ball. There was bleeding in the anterior
chamber on examining the injured eye.
Which of the following that you must exclude first ?
Presence of Foreign body
Increased Intraocular pressure
Answer : b
107 - Young with sudden unilateral painless eye blindness what the DX ?
Answer: review the pic in old Q
108 - Injury of vagus nerve cause which of eye symptoms?
One of the choices was ptosis
109 - On fundoscopic examination of a young child, you find a
white or cream colored mass associated with increased vascularization. What is your diagnosis?
Retinoblastoma

nodular

110 - A patient is complaining of ocular pain and itching after 4


regimen of anti-TB medications. Which drug cause this
manifestation?
A- Ethambutol

months

There were also missing questions about the following:


Orbital cellulitis.

Proliferative diabetic retinopathy.

smle ,2016

802

111 - Child opaque lens with signs of inflammation?


Cataract
Neuroblastoma
Retinoblastoma
Answer : C
112 - A neonate appeared to have discharge from both eyes. Gram stain of the discharge revealed a
gram -ve diplococci. What is your management?
IV Ciprofloxacin
Local antibiotics
Ceftriaxone 2550 mg/kg IV/IM, single dose (max 125 mg)
Answer : C
113 - A patient is coming with unilateral red eye with discharge, photophobia and decreased vision.
What is your probable diagnosis?
Keratitis
114 - pt c/o red eye after nail injury or something (there is pic that whitish things deposit in anterior
chamber ) the Q is what other symptoms pt has?
Blurred vision
Photophobia
Answer : A
115 - Child is coming with fever, unilateral eye proptosis, eye movement were seen to be affected,
what is your probable diagnosis?
Retinoblastoma
Orbital cellulitis

smle ,2016

803
Answer : b
116 - A patient presented to the hospital with one day history of
right eye pain, photophobia and decreased vision. There was no
history of trauma. Slit lamp examination with fluorescein staining
showed the following.
What is your diagnosis?
Scleritis
Episcleritis
Herpes simplex Keratitis [Dendritic corneal ulcer]
Answer : C
117 - A patient with sickle cell disease comes with sudden painless loss of vision. A cloudy retina and
cherry red spot finding the macula What is your diagnosis ??
Retinal artery occlusion
Retinal vein occlusion
Retinal detachment
Answer : A
Typical funduscopic findings of a pale retina with a cherry red macula (ie, the cherry red spot) result
from obstruction of blood flow to the retina from the retinal artery, causing pallor, and continued supply
of blood to the choroid from the ciliary artery, resulting in a bright red coloration at the thinnest part of
the retina (ie, macula).
118 - Vertical and horizontal nystagmus is associated with?
Phencyclidine toxicity
Phencyclidine toxicity cause : both vertical , horizontal , rotatory nystagmus.
119 - You are seeing a child coming with his mother complaining of itchy eyes for one month duration.
The mother provided a history of asthma in the child.
What is your diagnosis?
Bacterial conjunctivitis
Viral conjunctivitis
Vernal [Atopic] conjunctivitis
Answer : C
120 - Which dye is used to examine the cornea?
Fluorescein dye
121 - At a daycare center 10 out of 50 had red eye in the 1st week. Another 30 developed same condition in the next 2nd week, what's the attack rate?
40%
60%
80%
Answer : C
Attack rate = Number of new cases in the population at risk / Number of persons at risk in the population
122 - A hemiplegic patient came with Upbeat nystagmus. Where is the site of the lesion?

smle ,2016

804
Pons
Medulla oblongata
Internal capsule
Midbrain
Answer : b
The most common causes of upbeat nystagmus are cerebellar degeneration and brainstem or cerebellar
stroke. Most strokes are infarctions, involving the medial medulla or paramedian pontine tegmentum.
Hemorrhages are not infrequent, occurring in the cerebellum, affecting the brachium conjunctivum and
vermis, pons, or medulla.
http://www.neuroophthalmology.ca/textbook/disorders-of-eye-movements/x-nystagmus
123 - An infant was discovered to have a hemangioma in the left eye. It needs to be resected in a way
that doesn't affect his vision. What will be your action?
immediately
2 weeks
3 months
6 months
Answer : B to avoid Amblyopia
Ref: UQU
124 - A patient who has been discovered to have a bruit over the eye. What do you suspect this patient to have ?
Cavernous sinus thrombosis
125 - A child has Esotropia of 25 degree, the initial management will be:
Orthoptic
Exercise prism
Medial muscle recession
Answer : C
In patients with acquired esotropia, surgery is indicated when the deviation is greater than 15 PD and
stable.
http://emedicine.medscape.com/article/1198784-treatment#d6
The surgeon has documented if the infant has a constant infantile esotropia exceeding 12 PD, surgical
realignment should be performed.
http://emedicine.medscape.com/article/1198876-treatment#d6
126 - A 50 years old patient coming with painless loss of vision in one eye, with headache and pain
when touching the hair on the same side. What is your next step?
Topical steroids
Oral steroids
Brain CT
Answer : b
The universally accepted treatment of giant cell arteritis (GCA) is high-dose corticosteroid therapy,The
major justification for the use of corticosteroids is the impending danger of blindness in untreated patients. Patients who present with visual symptoms have a 22-fold increased chance of visual improvement if therapy is started within the first day. Damage may be irreversible if treatment is delayed beyond 48 hours.
http://emedicine.medscape.com/article/332483-treatment

smle ,2016

805

127 - A patient is post surgery in lacrimal gland , what is the site to put tube ?
Lacrimal duct
Lacrimal gland
Puncta
Answer : A
128 - A 20 years old gentleman came with blocked nose and URT symptoms, followed by swelling of
the eyes and redness. What is your probable diagnosis?
Meningococcal conjunctivitis
Chlamydial conjunctivitis
Parainfluenza virus
Answer : C
129 - A patient came to ER with acute eye pain and headache, nausea and vomiting. The patient is a
known case of acute glaucoma?
IV acetazolamide + pilocarpine eye drops
Oral acetazolamide
Topical acetazolamide and topical pilocarpine
Answer : A
130 - Cover test to the right eye causes lateral movement in left eye:
Strabismus
Nystagmus
Answer : A
131 - Type of Glaucoma you will do Trabeculectomy for?
ACAG,
COAG,
CCAG,
Congenital
Answer : B
132 - A young girl coming with crossed eyes. What does she have?
Squint
133 - You examine the red reflex in an infant and you find leukocoria. What should you be concerned
about?
Retinoblastoma
134 - A patient comes and provides a history of severely relenting headache that is situated around
the eye on one side, with rhinorrhea and lacrimation. It is used to come in clusters and at certain time
of the year. What does this patient have?
Clusters headache
Refractive error
Glaucoma
Migraine
Answer : A

smle ,2016

806

135 - female young pt complaining of eye discomfort , photophobia for 3 days


not improved with patch
136 - You examine a patient in a primary care center coming with a history of eye pain and photophobia and you find a vesicular rash above the supraorbital ridge. What is your next step?
Refer the patient to an ophthalmology clinic.
[Herpes zoster of the ophthalmic division of trigeminal nerve.]
137 - What is a common cause of corneal abrasions?
Contact lenses
138 - How can you prevent Post LASIK dry eye ?
use Punctal plugs
A punctal plug (tear duct plug) is a small medical device that is inserted into the tear duct (puncta) of an
eye to block the duct. This prevents the drainage of liquid from the eye. They are used to treat dry eye.
Artificial tears are usually still required after punctal plug insertion.
http://www.ncbi.nlm.nih.gov/pubmed/22358586
139 - which of the following choices best describe the pathophysiology for retinal detachment?
Retinal detachment occurs when subretinal fluid accumulates between the neurosensory retina and the
retinal pigment epithelium.
http://eyewiki.aao.org/Retinal_Detachment#Pathophysiology
140 - Opaque lens in an elderly patient?
Cataract
141 - A parent coming with her child for she has been noticing drooping of her child's eyelid in the
right eye with right neck mass. What do you have to rule out?
Wilms Tumor
Neuroblastoma
Answer : b
Neuroblastoma is the most common occult malignancy to be associated with pediatric Horner syndrome
http://www.ncbi.nlm.nih.gov/pmc/articles/PMC3743544/
142 - A patient is coming with teary red eye that is sensitive to light. What do you suspect?
Keratitis
Uveitis
Others
Answer : b
143 - A patient is presenting to the ER with sudden onset of proptosis with eye pain and swelling. Eye
movements were limited and painful, and vision was normal. What is your diagnosis?
Orbital cellulitis
Cavernous sinus thrombosis
Answer : A

smle ,2016

807
144 - A patient presented with unilateral eye swelling with purple skin discoloration , what is the organism?
Staph.aureus
B hemolytic strep
Haemophilus influenzae
Answer : b ?
145 - A patient is coming with watery eye discharge. She has a recent history of URTI. Lymph nodes were palpable on on neck and
cervical areas. What is this condition called?
Nummular keratitis [cloudy dots on the cornea]
It is a common feature of adenoviral keratoconjunctivitis (an ocular
adenovirus infection), as well as approximately 1/3rd of cases of
Herpes Zoster Ophthalmicus infections. It represents the presence
of anterior stromal infiltrates.
https://en.wikipedia.org/wiki/Nummular_keratitis
146 - A painful mass is felt in medial side of the eye to the nose of a young patient. What is your management?
Oral antibiotic
Topical antibiotic
Topical corticosteroid
Surgical drainage
Answer : D
147 - When do you start the ophthalmic screening for type 1 DM?
5 years from the time of diagnosis
146 - A man works on the computer for long hours per day came with tearing of eyes.
What is the underlying mechanism for his condition?
Lack of blinking
Mast cell mediated reaction
Answer : A
Lack of blinking will result in a dry eye. Excessive tearing is a compensatory reaction in a dry eye.
147 - Lower visual field loss with flashes and other symptoms?
HTN
DM
Retinal detachment
Answer : C
148 - Post op patient complaining of severe pain on eye with ciliary congestion? What do you suspect?
Endophthalmitis
149 - A foreign body sensation after exposure to (something). The patient was managed safely. What
is your next step?
Oral antibiotics
Oral steroid
Local antibiotics

smle ,2016

808
Local antibiotics
Answer : b
150 - The most common presenting sign of retinoblastoma?
Leukocoria
Nystagmus
Strabismus
Answer : A
151 - Man with features of temporal arteritis , biopsy showed giant cells with skipping lesions. What is
your most concern in regards to this patient?
Loss of vision
152 - a patient sustained a blow trauma to the eye presented with subconjunctival hemorrhage and
weak upward gaze.What is your diagnosis?
Orbital base fracture
153 - About chlamydia conjunctivitis, about prevention?
By Water colorization
154 - A patient with tumor affect eye what is the cause ?
Ewing sarcoma
Neuroblastoma
Nephroblastoma
Answer : A or B not sure
155 - A patient with Diplopia and decrease in visual acuity, had a previous similar episode. Dx ?
MRI
CT
EMG
Answer : A
156 - a patient is presenting visual disturbance, nausea and vomiting, and abdominal pain, what is the
most fitting choice to this scenario?
Digoxin toxicity
Angle glaucoma
Answer : A
157 - pt with ptosis how to manage ?
158 - A 2 years old child is presenting with a neck mass. He was discovered to have ptosis, miosis, anhidrosis and heterochromia. What is your probable diagnosis? Features of Horners syndrome
Wilms
Neuroblastoma
Answer : b
159 - A patient had red eyes, tearing, change in pupil shape that didn't improve with patch:

smle ,2016

809
keratitis
glaucoma
Answer:
160 - A Retinal tear happens between which layers of the retina?
a retinal tear occurs between the neurosensory retina and the retinal pigment epithelium.
161 - Patient with unilateral bacterial conjunctivitis, what is the antibiotic ?
Topical erythromycin
162 - An elderly female came with right eye pain and vomiting, what do you suspect?
Hyperthyroid
Acute angle closure glaucoma
Hyphema
Answer: b
163 - A patient with controlled DM and HTN sees flashes and loss part of visual field. What should be
in the top of your differential?
Retinal detachment
Diabetic retinopathy
Hypertensive retinopathy
Answer: A
164 - baby with inward deviation of rt eye , most likely due to :
Answer: refraction error
165 - Eye picture red conjunctiva with white ring around cornea Vernal conjunctivitis <. Similar picture
found on Google
Chlamydia
Vernal conjunctivitis
Answer: vernal conjunctivitis
166 - old man after crash blurred vision,on o/c anterior chamber cloud or something ...what cause
these symptoms?
retinal attachment
cataract
Answer : b
167 - pic of eye with fluorescent , Dx?
bacterial keratitis
fungal keratitis
herpetic keratitis
Amoebic keratitis
Answer : C
168 - Retinoblastoma affect the vision MRI show intact optic N (something like that) Ttt?
chemotherapy
steroid

smle ,2016

810
removal of the eye
Answer : C
169 - In retinal detachment where the fluid collect ?!
- photoreceptors and retinal pigment epithelium
170 - Headache , pain in Rt eye ,Halos around light :
-Hyphema
-Digoxin intoxication
Answer: acute angle Glaucoma
171 - Young ,very rapid reduce vision:
Answer:Glaucoma
172 - what is the diagnosis of DM patient with history of sudden unilateral vision loss pupil afferent
affected -retinal haemorrhages and macular edema ?
retinal artery occlusion
retinal vein occlusion
Answer:
173 - 23 y girl come e one eye blindness+past hx 2 year back headache + diplopia ...ect best investigation?
MRI
CT
CBC
Answer: A
174 - For which of the following we can use trabeculectomy ?
Open angle glaucoma
closed angle glaucoma
answer : Open angle glaucoma
175 - If you are examining the right eye by using with light reflex and there was no any changing of pupil of the right eye but there was changing in pupil of the left eye(consensual) , so where is the lesion ?
A. Rt optic nerve
B. Lt optic nerve
C. Rt oculomotor nerve
D. Lt oculomotor nerve
answer : Rt oculomotor nerve
176 - During examination of the eye of uncontrolled diabetic patient ,what you may find ?
Central Retinal artery ischemia (I think)*
Glaucoma
Papilledema
Answer b?
177 - Patient admitted to hospital with headache , nausea and vomiting (signs of increased ICP ) what
you find in eye examination ?

smle ,2016

811
Papilledema*
Central retinal artery schema
Glaucoma
answer : corneal injury under fluorescent
178 - A patient who presented with ciliary flush, bilateral eye redness and pain. Examination revealed
keratic precipitate and presence of cells in the anterior chamber (classical scenario of uveitis) what is
the treatment:
systemic corticosteroid
cyclopentolate with topical steroids
Answer : A
both choices are right but bilateral uveitis usually associated with systemic dis
179 - patient presented with foreign body sensation in his eyes, itchiness and grittiness. (not sure if
mentioned watery discharge or no). he is taking (a drug) and antihistamine. Also he mentioned that
he sit in front the screen for six hours daily. What is the cause of his disease? ( the scenario was not
very clear whither it is allergic or just simple dryness)
mast cell degranulation and histamine release.
Corneal dryness
answer : b
he had hx of sitting in front of a screen for 6hr daily and that will cause dryness due to infrequent blinking
180 - pt c/o red eye and itching ,hx of trauma , what u will do ?
Give him topical AB
ing but ther is ntopical corticosteroid
Reassure
answer : A?
181 - pt c/o red eye after nail injury or something (there is pic that whitish things deposit in anterior
chamber ) the Q is what other symptoms pt has??
blurred vision
photophobia
answer : A These are AC cells
182 - pic of congested eye , history of URTI one week ago , eye has watery discharge not purulent ,
what will support ur Dx :
papilla
folliculi
papilloma
answer : b
Ref : http://emedicine.medscape.com/article/1191370-workup?src=refgatesrc1
183 - Eye Watery discharge no itching ?
Answer:
Watery viral conjunctivitis, allergic conjunctivitis, eye allergies, dry eyes, eye injury, dacryocystitis
Toronto:

smle ,2016

812
VIRAL CONJUNCTIVITIS
serous discharge, lid edema, follicles, may be associated with rhinorrhea
preauricular node often palpable and tender initially unilateral, often progresses to the other eye
mainly due to adenovirus
Dacryocystitis
pain, swelling, redness over lacrimal sac at medial canthus epiphora, crusting, fever digital pressure on the lacrimal sac may extrude pus through the punctum in the chronic form, epiphora may be
the only symptom
184 - pt with excessive tearing and they gave a picture of entropion and they asked about cause of
tearing ?
Answer: conjunctivitis?
185 - child with strabismus ttt?
Glasses
186 - 4 yo girl came with her father for bilateral esotropia what is management ?
- observe
-glasses
-botulism
Answer :glasses
187 - Patient with lesion above the left eye brows , first lymph node to be examined is :
a) Parotid
b) Mental
c) Submandibular
Answer: A
188 - Which one enhance apoptosis?
P53
PCL-2
Answer : A
189 - pic of eye ( there is proptosis and redness of upper eyelid on one eye ) what is diagnosis?
orbital cellulitis
Answer: chalazion
190 - Periorbital lesion: what to do
Answer:
They gave us a scenario of patient with Herpetic dendrites and a picture also.
They asked what is associated with this condition? [Exactly the same picture]

smle ,2016

813

Emergency
Medicine

smle ,2016

814

1- Patient after road traffic accident developed chest pain. On examination: no lung sound and hyperresonance on one side. What is the 1st thing to do?
a.
Needle decompression
Answer: A
Tension pneumothorax is suspected.
So Immediate attention to the ABCs (airway, breathing, circulation) while assessing vital signs and oxygen saturation is paramount, particularly in patients with thoracic trauma. Definitive Management includes Needle thoracostomy at 2nd intercostal space at Mid clavicular line followed by chest tube at 5th
intercostal space at anterior axillary line. Reference: Toronto notes.
2- Patient ingested 20 gram of acetaminophen 8 hours ago, what is your next step?
a.
Charcoal
b.
N acetylcysteine
c.
Gastric lavage
Answer: B
gastric lavage/emesis (if <2 h after ingestion)
oral activated charcoal (if the patient has a stable mental and clinical status and presents to the emergency department within 1 hour of ingestion)
N-acetylcysteine (NAC, Mucomyst) can be given PO or IV (most effective within 8-10 h of ingestion, but
should be given no matter when time of ingestion) > promotes hepatic glutathione regeneration.
Toronto Notes
http://emedicine.medscape.com/article/820200-treatment
3- Patient ate from a restaurant. 2 days after that started to complain of diarrhea, vomiting and urticaria.
a.
Food poisoning
b.
Food allergy
Answer: ?
Food poisoning: http://emedicine.medscape.com/article/175569-overview
Food Allergy: http://emedicine.medscape.com/article/135959-overview
4- Treatment of bee sting.
Answer: ABC management
Systemic reactions:
Epinephrine for respiratory symptoms/hypotension
AntihistaminesH1 (diphenhydramine) and H2 (cimetidine, ranitidine, or
famotidine) blockers
Steroids (prednisone, methylprednisolone, or dexamethasone)
Inhaled -agonist for wheezing/shortness of breath
For persistent hypotension: 0.9% NS IV fluid resuscitation or Vasopressor
Removal of remnants of stinger at site of envenomation by scraping, not squeezing
Local reactions: Cool compress Elevation - Topical antihistamine/topical steroidal cream as needed
Reference: 5-Minute Emergency Medicine Consult, Toronto Notes and Medscape.

smle ,2016

815
5- What is the antidote for organophosphate poisoning?
Atropine
Answer: A
Reference: Master the Boards, 5-Minute Emergency Medicine Consult
Organophosphate poisoning signs and symptoms in the mnemonic DUMBBELS: Diarrhea, Urination, Miosis, Bradycardia, Bronchorrhea, Emesis (NV), Lacrimation, Salivation/Sweating.
6- A patient comes to the emergency with sudden dypsnea. X ray picture was attached showing dark
area without vascular marking. What is the diagnosis?
Pneumonia
Pulmonary edema
Pneumothorax
Pulmonary embolism
Answer: C
7- A guy was standing at a bus stop then fell. No previous history and no cardiac anomalies. What is
the diagnosis?
Cardiac syncope
Fainting??!
Seizure
Vasovagal
Answer: D
Vasovagal cause fainting/ syncope. I dont think Fainting was included in the choices provided in the
exam.
8- A guy presents in emergency with decreased oxygen carrying capacity of blood cells. Suicidal attempt is suspected. what is the culprit?
Carbon monoxide
Cyanide
Answer: A
9- Patient came to the ER with status asthmaticus. What will you use to intubate him?
Propofol
Ketamine
Midazolam
Answer: B
Reference: http://www.ashp.org/doclibrary/policy/emergencycare/post-intubation-managementhandout.pdf
10- Trauma due to MVA. The patient has spinal injury with hypotension and bradycardia. What is the
type of shock?
Neurogenic
Answer: A

smle ,2016

816

11- Soldier tries to have an excuse from the military, presented to the ER with symptoms of tremors,
hypoglycemia. What is he using?
Factitious injection of insulin.
Metformin
Answer: A
12- Acute attack of cluster headache, what is the abortive treatment?
Oxygen 100%
Subcutaneous Sumatriptan
Answer: A
Both Oxygen 100% and Subcutaneous Sumatriptan are effective.
Reference: UpToDate.
Abortive Therapy for Cluster Headache: ergotamine, triptans (sumaptriptan, eletriptan, almotriptan,
zolmitriptan), 100% O2, prednisone, lithium. Prophylaxis: Verapamil
Abortive Therapy for Migraine Headache: ergotamine, triptans (sumaptriptan, eletriptan, almotriptan,
zolmitriptan). Prophylaxis (Given when there is 3 or more episodes per month): Propranolol (Best), CCB,
TCA, SSRI, Topiramate, Botulinum toxins injections. Reference: Master the Boards
13- Semiconscious polytrauma patient, has difficulty breathing on bag ventilator (Ambu bag), prepared for intubation. What is the next step?
Go directly and intubate
Cricoid pressure
O2 supplements
Jaw thrust
Answer: C
Rapid-Sequence Intubation*
1. Pre-oxygenate with 100% oxygen
2. Apply cricoid pressure
3. Induction: etomidate (0.3 mg/kg), propofol (0.52 mg/kg) or ketamine (2 mg/kg) IV

smle ,2016

817
push
4. Neuromuscular blockade: succinylcholine 1.5 mg/kg IV push
5. Wait 3045 sec
6. Intubate when optimal conditions achieved
Reference: 5-Minute Emergency Medicine Consult
14- A patient ate a wild mushroom. Which of the following will be inhibited?
RNA polymerase I
RNA polymerase II
RNA polymerase III
DNA Gyrase
Answer: B
Reference: Handbook of Mushroom Poisoning: Diagnosis and Treatment + Wikipedia
15- Contraindication for gastric lavage?
Answer: Initial resuscitation incomplete
Risk assessment indicates good outcome with supportive care and antidote therapy alone
Unprotected airway where there is a decreased level of consciousness or risk assessment indicates potential for these complications during the procedure
Small children
Corrosive ingestion
Hydrocarbon ingestion
Reference: http://lifeinthefastlane.com/ccc/gastric-lavage/
16- Post-traumatic amnesia, vitally stable but hes complaining of pain in all of his 4 extremities.
Which type of shock might be developed?
Neuro
Cardiac
Hypovolemic
Reversible
Answer: A
Reference: Toronto Notes
17- Person escaped from fire in his flat from 3rd floor and jumped through the window. What will be
the most life saving in this case?
a.
Maintain Airway.
b.
Call for help.
Answer: B
18- 4 members in the family developed nausea and diarrhea 8 hours after eating at a restaurant. Then
after 48 hours they improved. Stool test showed oxidase positive gram +ve bacilli. What is organism?
Shigella
Salmonella
Answer: ?
Shigella (gram ve)
Salmonella (gram ve)

smle ,2016

818
Bacillus cereus is a Gram-positive aerobic or facultatively anaerobic, motile, spore-forming, rod-shaped

bacterium that is widely distributed environmentally. B. cereus is associated mainly with food poisoning.

smle ,2016

819

19-What is the degree of shock in this patient?


Answer: ?

Reference: Toronto Notes


20- Aspirin toxicity resulting in which of the following?
Respiratory alkalosis followed by metabolic acidosis
Answer: A
Phase 1: Respiratory stimulation- hyperventilation and respiratory alkalosis with alkaluria
Phase 2: Paradoxical aciduria (pH <6) and respiratory alkalosis
Phase 3: Metabolic acidosis & hypokalaemia ( ongoing respiratory alkalosis)
Reference: http://www.rch.org.au/clinicalguide/guideline_index/Salicylates_Posioning/
21- Which of the following babies needs immediate action?
a.
Cough and wheezing.
b.
Shallow spinal skin.
c.
Absent lower pulse.
d.
Yellowish discoloration.
Answer : C Absent lower pulse.
22- 36 year old male having retrosternal chest pain that radiates to jaw and left arm for 20
minutes (vitals included). What is the first thing to give him?

smle ,2016

820
a.
b.
c.
d.
Answer: A

Aspirin.
Morphine.
Oxygen.
IV nitroglycerin.

23- Male patient presented to the ER after MVA, he has tachycardia, tachypnea, hypotension
and cardiac arrhythmia. What is your most correct next step is ?
a.
FAST to check abdominal hemorrhage
b.
Needle decompression
c.
Pericardiocentesis
Answer: A
Hypovolemic shock
24- You are working as an emergency doctor and you got a call that there was an accident at the nuclear plant and 2 men were exposed to nuclear radiation. They dont have any complaints right now.
What will you advise them?
Cover them up with dry thick blankets
Give them aspirin and ask them to keep their legs moving
Isolate them and advise them to stay calm
Give them antibiotics
Answer: C
In hospital casualty will be finally decontaminated and kept in a clean special ward. The decontamination room must be at the entry of the hospital and should be sealed off from other premises and should
have a separate ventilation system. References: http://www.apiindia.org/pdf/pg_med_2008/Chapter17.pdf
25-Young pt come to ER with palpitation, euphoria, visual hallucinations,
a. Amphetamin poisining
Answer: A
CLINICAL FEATURES Clinicians should consider the diagnosis of methamphetamine intoxication in any
diaphoretic patient with hypertension, tachycardia, severe agitation, and psychosis. Acutely intoxicated
patients may become extremely agitated and pose a danger to themselves, other patients, and medical
staff
Reference: uptodate

smle ,2016

821
26 - Unconscious pt after ingestion of overdose sleep pills, breathing was reflex response breathing ,
you give him 2 breathing by mask , then you check pulse it was rapid and weak , what is next step:
A. wait code blue team
B. put pt in recovery position
C. intubat and ventilation
D. do CPR 5 cycle 30:2
Answer: C
Airway protection Airway protection by endotracheal intubation should be performed early in the poisoned patient with depressed mental status, unless the cause is easily reversible (eg, opioid intoxication
or hypoglycemia), because of the high risk for aspiration and its associated complications, particularly
when gastric decontamination procedures need to be undertaken [40]. Tracheal intubation with mechanical ventilation is also indicated in the presence of severe acid-base disturbances or acute respiratory failure. Particularly when intubating a severely acidemic patient, it is important to prevent the development of a respiratory acidosis through inadequate minute ventilation. Occasionally, the management of high-grade physiologic stimulation may require sedation and/or paralysis with mechanical ventilation to limit the extent of complications such as hyperthermia, acidosis, and rhabdomyolysis. One rare
exception to this important principle of aggressive airway management is salicylate poisoning, in which
mechanical ventilation should be avoided unless absolutely necessary.
Reference: uptodate
27 - child girl obese try to suicide and eat alot of drug because of her friend and came to the ER?
A. referral immediate to the psychatry
B. treatmen for acute depression
C. something
D. another something
Answer: A
ensure patient safety: close observation, remove potentially dangerous objects from person and room
assess thoughts (ideation), means, action (preparatory, practice attempts), previous attempts
admit if there is evidence of intent and organized plan, access to lethal means, psychiatric disorder,
intoxication (suicidal ideation may resolve with few days of abstinence)
patient may require certification if unwilling to stay voluntarily
do not start long-term medications in the ED
psychiatry or Crisis Intervention Team consult
Reference: Toronto Notes
28 - after intubation still low in spo2 wt u will do
Answer:
Causes of hypoxia occurring soon after intubation: Think DOPES:
Displacement of the endotracheal tube (ETT)
Obstruction of the ETT
Patient especially pneumothorax; also: pulmonary embolism, pulmonary edema, collapse, bronchospasm

smle ,2016

822
Equipment ventilator problems
Stacked breaths a reminder about bronchospasm and ventilator settings.
First step in managing an intubated and ventilated patient who is hypoxic: Disconnect the ventilator and
administer high-flow 100% oxygen using a bag-valve-mask.
Reference: http://lifeinthefastlane.com/pulmonary-puzzle-012/
ADDED QUESTION: ( need to be more checked )
29.anterior abdominal stab wound omuntam come through the wound ?
A. Fast
B. CT
C. Exploratory laparotomy
Answer: C
Reference: http://www.ncbi.nlm.nih.gov/pmc/articles/PMC4379793/
30.patient came from RTA and the staff preparing him for intubation he devolved low ventilation on bivalve what to do ? repeated
A.Proceed to intubation
B. jaw thrust
C. More head tilt
D. Cricoid pressure
Answer: A
Indications for intubation: 1) inadequate oxygenation 2) inadequate ventilation 3) Anticipate development of inadequate oxygenation/ventilation 4) protect the airway.
References: http://www.up.ac.za/media/shared/Legacy/sitefiles/file/45/1335/17594/grandroundtopics/2014/difficultairwaymxinicumay2014jmatshe.ppt
31.Pt work outdoor in temp 42 c .. Tired and Complain of crampy abdominal
pain .. Lower limb pain and fever .. Otherwise normal ,, what to do ?
electrolytes and oral replacement fluid
warming
cooling
Answer: C
Heat Stroke - Rapid reduction of the core body temperature is the cornerstone of treatment because the
duration of hyperthermia is the primary determinant of outcome. Once heatstroke is suspected, cooling
must begin immediately and must be continued during the patient's resuscitation.
Reference: http://emedicine.medscape.com/article/166320-treatment
32.Patient is bleeding the baro receptor activated result in increased tachycardia and decrease of ?
HR
Ventricular rate
Coronary artery flow
Answer: A
Circulatory Reflex Initiated by the Baroreceptors. After the baroreceptor signals have entered the tractus solitarius of the medulla, secondary signals inhibit the vasoconstrictor center of the medulla and excite the vagal parasympathetic center. The net effects are (1) vasodilation of the veins and arterioles
throughout the peripheral circulatory system and (2) decreased heart rate and strength of heart contraction. Therefore, excitation of the baroreceptors by high pressure in the arteries reflexly causes the

smle ,2016

823
arterial pressure to decrease because of both a decrease in peripheral resistance and a decrease in cardiac output. Conversely, low pressure has opposite effects, reflexly causing the pressure to rise back toward normal.
Reference: Guyton and Hall Textbook of Medical Physiology 12th Ed
33.Atropine side effect ?
Dry mouth
Answer : A
Frequent effects include xerostomia (dry mouth), dry skin, blurred vision, cycloplegia, mydriasis, photophobia, anhidrosis, urinary hesitancy and retention, tachycardia, palpitation, xerophthalmia, and constipation, which may appear at therapeutic or subtherapeutic doses. Xerostomia is the dose-limiting effect
Reference: https://www.drugs.com/sfx/atropine-side-effects.html
34.Patient came to you with history of bee sting with light heeded and shortness of breath what is the
Rx ?
Oral histamine reassurance
Fluids and elevate the leg
Sc epinephrine , IV histamine
Answer : C
Acute Severe Systemic Reaction/ Anaphylaxis
ABCs, Epinephrine SC/IV, Antihistamines IV, Corticosteroids
Reference: 5-Minute Emergency Medicine Consult
35.Which type of insulin used in DKA ?
Regular ( short acting )
Answer: A
Only short-acting insulin is used for correction of hyperglycemia in DKA
36.man got a bee sting then his wife trying look for the epinephrine what it gonna inhibit?
leukotriene release from macrophages
cross reactivity with the cardiac..
inhibit immunocomplex formation
Answer: A

Reference: http://www.slideshare.net/nidafkhan1/adrenaline-noradrenaline
37.Pt conscious with multiple injuries. How do you maintain airway?
Mask
Oro pharyngeal airway

smle ,2016

824
Nasopharyngeal
Endotracheal Intubation
Answer : A mask
38.Depressed pt with HTN Brought by family to ER for drug overdose palpitation diaphoresis and ECG
shows arrhythmia. What is the possible drug ?
Ssri
Digoxin
Answer. B or if there is TCA in the choices
Patients have survived overdoses of each of the SSRIs that were many times their usually effective antidepressant doses without serious toxicity: no arrhythmias, no disturbance of blood pressure, no seizures, no coma, no respiratory depression. All of these adverse effects do occur with overdose of TCAs
as little as 5 times their therapeutic doses.
Digoxin toxicity may cause almost any dysrhythmia. Sinus bradycardia and AV conduction blocks are the
most common ECG changes in the pediatric population, while ventricular ectopy is more common in
adults. Nonparoxysmal atrial tachycardia with heart block and bidirectional ventricular tachycardia are
particularly characteristic of severe digitalis toxicity
Reference: http://www.emedexpert.com/compare/ssris-vs-tca.shtml , Medscap
39.Old pt with high cholesterol level measured before 4 months and he is on a diet. Came to ER complaining of chest pain. What will concern you?
Current symptoms
Cholesterol level before 4 m
Answer. A Current symptoms
40.Pt in ER MVA. First thing you do?
ABC
Answer :A abc
41.gunshot with wound bowel perforation. What antibiotics you should give
answer : IV antibiotics: Antibiotics with coverage against gram-negative and anaerobic
organisms: Ampicillin/sulbactam, Cefotetan, Cefoxitin, Piperacillin/tazobactam, Ticarcillin/clavulanate.
Reference: 5-Minute Emergency Medicine Consult
42.Case of RTA and they found difficulty in intubation what should they do :
just proceed to intubation whatever.
increase chit tilt .
press on cricoid
answer: c
Methods of emergency ventilation in a cannot-ventilate-cannot-intubate (CVCI) situation include use of
the laryngeal mask, combitube or transtracheal jet ventilation. Cricothyrotomy (not tracheostomy) is the
preferred method of surgical access to the airway in an emergency such as a CVCI problem. The ventilatory rate should be restricted to the minimum required to prevent life threatening hypoxia (4-6/min)
and a cricothyrotomy or tracheostomy undertaken without delay.
Reference: http://medind.nic.in/iad/t05/i4/iadt05i4p308.pdf
43. Patient has acute MI. which of the following enzymes will be elevated?

smle ,2016

825
Creatine kinase.
alanine aminotransferase.
Alkaline phosphatase.
Answer: A
44.Patient had bee sting on the hand yesterday, he is presenting to you with redness and itching on
the bite site. What is the treatment?
Oral steroid.
Antihistamine.
Oral Antibiotic.
Answer: B antihistamines
45.Patient had bee sting and presented to you immediately with shortness of breath and lightheadedness. What is the treatment?
SC epinephrine.
Answer: IM epinephrine
45. Patient with ischemic stroke presented after 6 hours. What is the treatment?
Tissue plasminogen activator.
Aspirin.
Clopedogril.
answer : B aspirin
Presentation within 3-4.5 h and no contraindication to thrombolysis 1st line tissue plasminogen activator
Presentation after 4.5 h or contraindication to thrombolysis 1st line aspirin
Reference: BMJ
46.Unconscious patient after RTA .. Ventilation with bag mask was difficult .. What to do ?
exaggerate jaw thrust
increase head tilt
coricoid pressure
intubation
answer : D Intubation
47.Man got bee sting ,, his wife gave him epinephrine .. What is the Immunologic action of epinephrine ?
Iterlukine..... Ect
Tryptanase .....ect
Answer A

smle ,2016

826

Reference: http://www.slideshare.net/nidafkhan1/adrenaline-noradrenaline
Additional Questions ( 8th update)
48.Young healthy guy comes to ER after light headedness nausea and heavy breathing 20 mins before
admission. What most appropriate thing to do?
a.Alcohol concentration.
b.CT brain.
c. Other choices unrelated.
Answer: B
Alcohol intoxication as a cause of altered mental status is a diagnosis of exclusion and should be considered only after ruling out more serious conditions such as head trauma, hypoxia, hypoglycemia, hypothermia, hepatic encephalopathy, and other metabolic and physiologic derangements. For head trauma
in particular, existing clinical decision rules such as the Canadian CT Head Rule and NEXUS criteria may
not have adequate sensitivity in intoxicated patients with minor head injury.
Uptodate: http://www.uptodate.com/contents/ethanol-intoxication-in-adults#H4
49. Elderly asthmatic patient comes to regular check up. His wife has osteoporosis. He's recurrent kidney stones.he's taking calcium and vitamin supplements. Labs show high ca, normal phosphate, PTH
high. Most diagnosis Is:
a- Sarcoidosis.
b- Paget's disease.
c- Vit D intoxication.
d- Hyperparathyroidism.**
Answer: D Hypepararthyroidism
50. Child with traumatic injury to elbow. X-ray will show?
a- Posterior fat pad.**
b- Anterior fat pad.
c-Others epicondylar injuries I forgot.
Answer: A
Supracondylar fracture over 60% of all paediatric elbow injuries
Posterior fat pad sign indicates effusion/injury: In children, it implies supracondylar fracture.
Anterior fat pad can found in normal xrays
Reference:5-Minute Emergency Medicine Consult, http://radiopaedia.org/articles/paediatric-elbowradiograph-an-approach
51.Long scenario .. Pt brought to ER with multiple injuries in head . Chest , abdomen , proximal upper
limb is amputated blood profuse , hypotensive , on o2 face mask 6L , what to do?
Tourniquet ,
clamp vessels ,
OR ,
mantain airway
Answer : D
Three goals exist in the emergency department treatment of the patient with hypovolemic shock as follows: (1) maximize oxygen delivery - completed by ensuring adequacy of ventilation, increasing oxygen
saturation of the blood, and restoring blood flow, (2) control further blood loss, and (3) fluid resuscitation.

smle ,2016

827
Refernce: Medscape.
52. asprin toxicity with ABG : show low CO2 and low HCO3 ?
with acidic PH ?. cause ?
resp. alkalosis followed by metabolic acidosis
metabolic acidosis followed by resp. alkalosis
metabolic acidosis
Answer: A
Respiratory alkalosis progressing to metabolic acidosis is the key for aspirin overdose diagnosis. Aspirin
interferes with oxidative phosphorylation increasing lactate levels. Reference: Master the board page:
604
53.bee sting treatment?
answer: For local reactions:
Provide supplemental oxygen
Diphenhydramine limits the size of the local reaction.
Clean the wound and remove the stinger if present.
Apply ice or cool packs.
Elevate the extremity to limit edema.
if generalized reactions developed; Treatment should include an initial intravenous (IV) bolus of 10-20
mL/kg isotonic crystalloids in addition to diphenhydramine and epinephrine.
Reference: http://emedicine.medscape.com/article/768764-treatment#d9
54. child ingested iron what is treatment:
Answer: Deferoxamine is the iron-chelating agent of choice. Deferoxamine binds absorbed iron, and the irondeferoxamine complex is excreted in the urine.
Reference: Medscape
55. in emergency department pt come with close head truma and loss of consciousness what is the
first thing to do:
intubation and hyperventalition
asses airway
pupalliry responce
glascow coma scale
Answer: B Assess Airway. Establishing and maintaining airway patency takes precedence over all other
treatment.
Reference: First Aid Step 2 page 466
56. An alcoholic comes to you with symptoms of alcohol withdrawal. Last drink he consumed was 2
days back. What drug will you give?
a. Naloxone
b. Diazepam
c. Nicotine
d. Disulfiram

smle ,2016

828
Answer : B Diazepam. Basic protocol for alcohol withdrawal treatment is diazepam 20mg PO every 2hrs
til regression of severe withdrawal symptoms + Thiamine 100 mg IM then 100 mg PO OD for 3 days in
addition to hydration.
Reference: Toronto notes.
57. Female came to ER .. agettated.. pupil is dilated.. she is toxic with what?
A. Organophosphate.
Answer: agitation and dilated pupils (mydriasis) are characteristic for anticholinergics toxicity e.g. atropine, TCA,
carbamazepine.
Reference: Toronto notes, Emergency medicine.

58. Young female stayed out in sun at 42 degree .. she came to ER later tired.. muscle cramp.. vitally
stable except T: 38 how to manage?
A. Normal sline
B. Cold pack
Answer: B
Heat Stroke - Rapid reduction of the core body temperature is the cornerstone of treatment because the
duration of hyperthermia is the primary determinant of outcome. Once heatstroke is suspected, cooling
must begin immediately and must be continued during the patient's resuscitation.
Reference: http://emedicine.medscape.com/article/166320-treatment
59- elderly patient had motor vehicle accident, there is problem with ventilation. what is next step:
exaggerated jaw thrust
more head tilt
intubate immediately
answer : C. Failure of ventilation is an indication for endotracheal intubation. Reference: Medscape.
60- Dog bite infections?

smle ,2016

829
Gram negative bacillus bacterua
virals
parasite
answer : A.
The most common pathogens in dog bites are Pasteurella spp. (both Pasteurella multocida and Pasteurella canis), Staphylococcus and Streptococcus spp., and the fastidious Gram-negative rod Capnocytophaga canimorsus (previously known as the CDC and Prevention Group Dysgonic Fermenter-2).
Treatment:is amoxicillin/clavulanate plus tetanus vaccination booster if more 5 years since last injection.
Reference: Master the board.
Reference: http://www.medscape.com/viewarticle/739023_4
61- patient presented to ER with history of drug overdose and coma for the last 8 hours on examination absent gag reflex best management is :
A.iv naloxone
B.gastric lavage
C.immediate endotracheal intubation
D. charcoal
Answer: C
Airway protection Airway protection by endotracheal intubation should be performed early in the poisoned patient with depressed mental status, unless the cause is easily reversible (eg, opioid intoxication
or hypoglycemia), because of the high risk for aspiration and its associated complications, particularly
when gastric decontamination procedures need to be undertaken
http://www.uptodate.com/contents/general-approach-to-drug-poisoning-in-adults
61- pt had trauma, presented with tachycardia , bradypnea and hypotension , what is the first thing
you will do ?
Needle thoracotomy
Answer: A
Tension pneumothorax is suspected.
So Immediate attention to the ABCs (airway, breathing, circulation) while assessing vital signs and oxygen saturation is paramount, particularly in patients with thoracic trauma. Definitive Management includes Needle thoracostomy at 2nd intercostal space at Mid clavicular line followed by chest tube at 5th
intercostal space at anterior axillary line. Reference: Toronto notes.
62 -case head trauma on parietal lobe subdural hematoma which artery is injured?
a-superficial temporal .
b- mid cerebral
c- Rt.cerebral
answer: C
Middle cerebral artery supplies parietal lobe. however, subdural hematomas are most commonly
caused by tearing of the bridging veins that drain from the surface of the brain to the dural sinuses. Arterial rupture can also result in Subdural Hematoma and most of them were caused by injuries to small
cortical arteries.
Reference: http://www.uptodate.com/contents/subdural-hematoma-in-adults-etiology-clinical-features-and-diagnosis#H2

smle ,2016

830
63- Patient is eating at a restaurant with his friend; he choked with a piece of meat, his friend who
was an emergency assistant performed Heimlich maneuver but without benefit. He then decided to
perform a tracheostomy, during tracheostomy which of the following structures will be cut?
Cricoid cartilage
Thyroid cartilage
Cricothyroid membrane
Answer: C.
64- patient MVA and come with fracture of femur , tibia and fibula what is your action :
refer to orthopedic
Answer: A
Parenteral analgesia should be administered when appropriate. pen fractures must be diagnosed and
treated appropriately (by orthopedician) Tetanus vaccination should be updated, and appropriate antibiotics should be given in a timely manner.
http://emedicine.medscape.com/article/826304-treatment#d10
65- young man close the door on his nail color become blue under nail what will u do ?
evacuate hematoma
remove nail
reassure
Answer: C
A painless and small subungual hematoma usually requires no treatment and its according to the nail
edges status. evacuation is indicated in the presence of a painful subungual hematoma with the nail
edges intact.
Reference: http://www.webmd.com/skin-problems-and-treatments/bleeding-under-nail?page=2 ,
http://emedicine.medscape.com/article/82926-overview#a2
66- pt involved in MVA the impact especially in forehead examination forehead laceration & fx ,discharge from nose clear positive glucose test ( I think mean CSF leak ) Which cranial n affected :
Optic
Olfactory
Ophthalmic
Oculomotor
Answer: D
Cerebrospinal fluid rhinorrhea in the acute phase after trauma has been reported in as many as 39% of
the patients with skull base fractures.[104] Patients present with a variety of symptoms depending on
the acuteness of the event. In the acute phase following the traumatic event, patients may present with
epistaxis, nasal discharge, periorbital ecchymosis, chemosis, oculomotor impairment, anosmia, motor
deficit, open-head injury with CSF leakage, loss of vision, cranial nerve deficits (most frequently, first
third and fifthseventh cranial nerve injuries), meningitis, and pneumocephalus Reference:
http://www.medscape.com/viewarticle/765460_3
67- What is the treatment of Aspirin toxicity?
Alkalinization of urine.
Gastric lavage.
Activated charcoal.
Answer: A. Alkalinization of urine. Reference: First Aid step 2 page 482

smle ,2016

831

68- Which of the following substance ingestion is a contraindication to gastric lavage?


Aspirin.
Benzodiazepine.
Some kind of bleach or cleaning products.
Maybe hydrocarbons.
Answer: C
Caustics ingestion (drain cleaners) is a contraindication to Gastric lavage.
Reference: Master the board page: 602

69- Common Antidotes:


70- child girl obese try to suicide and eat alot of drug because of her friend, came to ER:
immediate referral to psychiatry.
treatment of acute depression.
Answer: A
ensure patient safety: close observation, remove potentially dangerous objects from person and room
assess thoughts (ideation), means, action (preparatory, practice attempts), previous attempts
admit if there is evidence of intent and organized plan, access to lethal means, psychiatric disorder,
intoxication (suicidal ideation may resolve with few days of abstinence)
patient may require certification if unwilling to stay voluntarily
do not start long-term medications in the ED
psychiatry or Crisis Intervention Team consult
Reference: Toronto Notes
71. Pt with shock & Cherry red skin;

smle ,2016

832
Bacteremia
Septicemia
Carbon monoxidemia
Answer : c
9th update
Our link Qs up to 28th of December
72-Old man came to ER with SOB cough, fever, what is the next step of management.
a.Put him in negative pressure room
b.Xray
c-Antibiotics
answer: B
X- Ray ,cough and fever with
respiratory infection order
CXR to notes any interstitial
changes .
73-Glasgow coma scale
answer:

74-Pt dose not complain of


anything, has sudden knee swelling? What is the best thing to do?
answer:
If history of trauma, laboratory testing is unnecessary, X-ray
ESR, CRP, WCC: normal results may be useful in excluding inflammatory joint disease or septic arthritis.
Reference: http://patient.info/doctor/knees-that-swell
75-Child on picnic with family, presented to the ER with high suspicion of foreign body inhalation. The
most common location in the chest will be in?
a- Right Main Bronchus
b- Left Main Bronchus
c- Carina of Trachea
d- Inlet of Larynx
Answer: A
Right Main Bronchus 52% of cases
Reference: uptodate
76-Child swallowed battery, now it is lodged in esophagus Ttt ?
a-Endoscopic removal
b-Wait ..
Answer:A
Endoscopic removal
Reference: http://www.poison.org/battery/guideline
77-Man have 41 c temp with muscle spasm mangment?
a-core cooling

smle ,2016

833
b- syprying warm
c- warm iv
Answer: A
78-Healthy baby was in picnic with his family .. Sudenly he get SOB ..come to ER the chest oscultation
:RT wheezing in rt hemithorax, What you will find in the CXR (foreign body),?!
RT lower consolidation
Hyperinflation with midline shift.
RT Atlectasis
ANSWER: B
Children will more often display signs of air-trapping while adults will more often show atelectasis
80% of aspirated foreign bodies will be non-opaque on conventional radiography
Reference: http://learningradiology.com/archives2012/COW%20513-Aspirated%20FB/fbcorrect.html
79-Patient presented with mild burn, what to do?
a.Apply cold water (or ice).?
b. Apply butter.
Answer : A
*just cold water*

80-CPR in child according to American heart association in presence of 02 rescuer:


a.15 compression and 2 ventilation
b.30 compression and 2 ventilation
Answer : B
Reference: https://www.billingsclinic.com/app/files/public/1236/Child-BLS-Skills-Testing.pdf
81-case about someone stabbed in the 4th right intercostal space, what the structure behind it?
right horizontal fissure

smle ,2016

834
right apical lung
left something
left something
answer : A
1) Horizontal Fissure: 4th Intercostal Space OR inferior border of 4th Rib
2) Oblique Fissure: 6th Intercostal Space OR inferior border of the 6th Rib
Reference: https://quizlet.com/7143345/lungs-pleura-mediastinum-flash-cards/
82-basal skull fracture cavernous sinus affected>>not sure, which muscle is intact?
trapezius
Sternocliedmastoid
Answer: More often, oculomotor palsy occurs together with that of other ocular motor nerves contained in the
cavernous sinus in case of skull base fracture.
Reference: http://medind.nic.in/icf/t07/i2/icft07i2p89.pdf
83-man make RTA he was conscious, oriented, alert , but his extremity
type of shock ?
irreversable
neuro
cardio
hemorrhagic
answer: D hemorrhagic
84-Child ate a number of iron tablets presented with severe symptoms including constipation and
bloody stool nausea and vomiting and drowsiness how would you treat him:
IV deferoxamie.
Dialysis
Answer: A
Deferoxamine is the iron-chelating agent of choice. Deferoxamine binds absorbed iron, and the irondeferoxamine complex is excreted in the urine.
Reference: Medscape
85-What medication you can give to prevent haluin withdrawal symptom?
Methadone
Answer: http://emedicine.medscape.com/article/287790-treatment
86- Pt with Digoxin toxicity, what will order ?
lidocaine
Fb immunoglbulin
Answer: B
Decontaminate (activated charcoal) - Digoxin-specific Ab fragments
Reference: Toronto Notes

12th update

smle ,2016

835
87-ptn came to ER with multiple fracture no loss of conscious with Low bp , tachycardia , normal RR ,
O2 saturation = 95% ?
IV fluid
Save airway
Answer: B
All resuscitations should be performed using Advanced Trauma Life Support (ATLS) guidelines.For the
individual physician, assessment of the polytraumatized patient is performed using a stepwise longitudinal approach, in which the airway is handled first and no procedures are initiated until the airway is secured. Then, breathing and circulation are addressed
Reference: Medscape
88- asprin toxic:
Answer: alklination of urine.
89-Pt eat multiple drug what do (not give time)
A.
Gastric leavage
B.
Activite charcoal
Answer: B *in Acute situation*
Reference and for more information: http://www.uptodate.com/contents/gastrointestinal-decontamination-of-the-poisoned-patient?source=search_result&search=Gastrointestinal+decontamination+of+the+poisoned+patient&selectedTitle=1~150
90-A patient presented to ER with ingestion of multiple iron tablets. What is your next step?
A.
Induce emesis
B.
Ipecac syrup
C.
Gastric lavage
D.
Wait and monitor
Answer: D
if asymptomatic wait 6 h then discharge
91- female with right upper abdomen pain and fever no jaundice.. what is the
management :
Emergent surgery.
Iv fluid and antibiotics.
Discharge
Answer: B
Mild cholangitis may present with abdominal pain, jaundice, and fever. When transporting these patients to the hospital, place the patient on a monitor and insert an intravenous (IV) line.
Provide fluid resuscitation with IV crystalloid solution (eg, 0.9% normal saline).
Administer parenteral antibiotics empirically after blood cultures are drawn.
Reference: Medscape
92-MTX toxicity what to give:
A- folic acid
B- folnelic acid (something like that)
C- steroid
Answer: -

smle ,2016

836
leucovorin
Reference: http://reference.medscape.com/drug/fusilev-levoleucovorin-leucovorin-343736
93-RTA with hypovolemic shock signs , Hg low , what to give initially
A- rengar lactate
B- packed RBC
C- whole blood transfusion
Answer: A If there is no NS in options
94- patient presented to the ER, unable to talk, his face is blue, what is the next step in the management?
A- open mouth check for any foreign body.
answer: A
95-male paient came to ER with stap wound and hypotension what is your next step?
fresh frozen plasm
Iv ringer lactate
Packed RBC
Answer: B
2 large-bore IV lines with crystalloid infusion
If no response to 2 L of crystalloid, infuse 24 units packed red blood cells:
Reference: 5-Minute Emergency Medicine Consult,
96- pt with ECG showing bradycardia, what should you give?
A- atropine.
B- cardioversion.
answer: A
(check the ACLS Algorithm) .
97-long case about adult come to er complaing of diarrhea , have weekness in body and fatigue , suddenly he fall down , BP when he come was 120 / 80 Now it is 90 / 60 , What is the cause ?
A- Extracelular voluim loss
B- Intracellular fluid loss !?
C- Intracellular glocse loss
Answer: A
Reference: http://chemistry.elmhurst.edu/vchembook/255fluiddeficit.html

smle ,2016

837

98-RTA pt came to ER after 30 mins o/e chest


there's sound bilateral +
distended neck. Dx?
Hemorax
Pneumothorax
Cardiac tamponade
Answer: C
Reference:
http://www.emsworld.com/article/10324543/penetrating-chest-trauma
99-Why we use CT scan in trauma pt ?
a. Can detect retropretoinum hemorrhage
b. You can see the hemorrhage with contrast
Answer: A
CT scans of the abdomen and pelvis usually are performed together, using both IV and oral contrast. Use
this study to identify injuries to abdominal and pelvic organs and to identify bleeding in the retroperitoneum and pelvis
Reference: Medscape
100- boy play basket ball he came with abdominal pain without any injury in match physical exam was
tenderness in paramblicus what you next :
a. Chest xray
b .abdominal CT
c. 24 recheck
d. kidney US
Answer: C
Exercise Related Transient Abdominal Pain
1. Diagnosis based
primarily on hx Transient abdominal pain during exercise - relieved w/cessation of activity
Pain described as well localized, commonly in lumbar region of abdomen Usually on right side
2. Physical exam: Normal abdominal exam
Abnormal abdominal exam suggests different etiology of pain during exercise
3. Diagnostic testing :
None recommended unless physical exam or hx suggests etiology other than ETAP
Reference: https://mospace.umsystem.edu/xmlui/bitstream/handle/10355/13721/ExerciseRelatedTransientAbdominalPain.pdf?sequence=1
101-Pt came to ER , airway pt gasping ,, u gave 2 breath by mask , pulse is rapid & wee What to do?!
Waiting code team
Put pt in recovery position
Do CPR
intubation
Answer: C

smle ,2016

838
If the patient is unconscious, unresponsive, and is not breathing normally (occasional gasps are not normal) start CPR according to the resuscitation guidelines.
Reference: https://www.resus.org.uk/resuscitationguidelines/abcde-approach/
102-Dm pt e - DKA what to
give?
1 L of normal saline
Dextrose 50% followed by
insulin
Answer: A

Reference: Toronto notes


103-Dm pt e ABG values
show ph 7.2 - pt is hyperventilated. Why this happen?
answer: Hyperventilation causes the body to exhale and "get rid of" CO2 from the blood
104- Comatosed patient with respiratory depression and pinpointed pupil, drug abuse suspected what
is the cause ?
Cocaine
Opiates
answer: B
105- Witch of the following is contraindicated to do gastric lavage?
Drain cleaning solution
answer : A
106- Patient with multiple trauma and head of femur fracture witch of the following is elevated?
AP
Creatinie kinase
answer: A
alkaline phosphatase
Reference: http://www.ncbi.nlm.nih.gov/pubmed/627581
107- post RTA in ICU presnt with significant blood losse Hypotension Now in Multi Organ failure what
Is the most Organ causing other organ failure ?
heart
lung
kidney
liver
Answer:
108-Case of opioid toxicity which to do first:

smle ,2016

839
Iv naloxone.m
intubation
Answer: B
Airway protection by endotracheal intubation should be performed early in the poisoned patient with
depressed mental status, unless the cause is easily reversible (eg, opioid intoxication or hypoglycemia),
because of the high risk for aspiration and its associated complications, particularly when gastric decontamination procedures need to be undertaken
http://www.uptodate.com/contents/general-approach-to-drug-poisoning-in-adults
108-Typical case of organophosphorus poisoning dx:
Answer:
http://bestpractice.bmj.com/best-practice/monograph/852/diagnosis/step-by-step.html
109- Typical case of monoxide poisoning dx:
answer:
http://bestpractice.bmj.com/best-practice/monograph/432/diagnosis/step-by-step.html
110-Present to ER with severe headache and LP showed blood :
ruptured berry aneurysm,
epidural hematoma
Answer: A
Reference: Medscape * subarachnoid hemorrhage *
111-Case of carbon monoxide poisoning (signs very important)
Answer:
http://bestpractice.bmj.com/best-practice/monograph/432/diagnosis/step-by-step.html
112-Pt. Overdosed a medication and presented Comatose, dilated pupil, hyperreflexia, what is the
meds:
Erdophnium
SSRI
TCA
answer: B
113-Drug addicted .. Unconsious Came with no gag reflex, What would you do?
Intubation
Gastric lavage
give nalexone
answer: A
Airway protection by endotracheal intubation should be performed early in the poisoned patient with
depressed mental status, unless the cause is easily reversible (eg, opioid intoxication or hypoglycemia),
because of the high risk for aspiration and its associated complications, particularly when gastric decontamination procedures need to be undertaken
http://www.uptodate.com/contents/general-approach-to-drug-poisoning-in-adults
114-RTA pt came to ER after 30 mins o/e chest there's sound bilateral + distended neck, Dx?!
A- Hemothorax
B- Pneumothorax

smle ,2016

840
C- Cardiac tamponade
Answer: C
115-MI patient presented to ER after resuscitation he developed coma and then died, what postmortem change you will find:
A- interventricular hemorrhage
B- brown coloured area supplied by middle menengieal artey
Answer: O n completion of the staining the infracted myocardium will show up as pale pink colored area as
against the bright red colored normal myocardium. Even old infarcts and scars get delineated. In absence of infarction, both the surface of the slice will show uniform bright red coloration.
Reference:http://www.themedicalacademy.in/fxconsult1/userfiles/4_%20POST%20%20MORTEM%20%20STUDY%20%20OF%20%20HEART%20%20IN%20%20CASES%20%20OF%20%20SUDDEN%20%20CARDIAC%20%20DEATH%20%20USING%20%20TRIPHENYL%20%20TETRAZOLIUM%20%20CHLORIDE%20%20AND%20%20HAEMATOXYLIN%20%20&%20%20EOSIN%20%20STAIN.pdf
116-Road accedint presented with paralsis of 4 limb and extremity pink and hot what is the type of
shock?
Irreversable
Nurogenic
Hypotensive
Answer: B

117- 5 cycles of CPR - intubation - put pt. in recovery position? The Question is not clear

smle ,2016

841

118- hypertensive male presented to ER with


sever epistaxis fron inferioir posterior , what is
the most artery cause this bleeding :
Sphenopalatine
Greater palatine
Anterior ethmoidal
Answer: A

119-girl come to ER complaing of RT hypochondrial pain , vomiting , fever with similer attacks Before
On examination splenomegaly and Mild jaundice US show gallstones...your dx?
thalacemia
SCA
answer: Both are right , Reference: Introduction To Pathology For The Physical Therapist Assistant

120-Patient
came to
ER after
eating a
lot of
drug .. Came with eye dilatation What does he takes:
TCA.
Organophosphorus .
Answer:A
Reference: http://bestpractice.bmj.com/best-practice/monograph/342/diagnosis/step-by-step.html
121-A 6-year-old girl, brought by parents to ER with history of falling from height...
Not talking but crying, withdrawal from pain, open her eye only in response to Doctor talking,Calculate GCS.
9
10
11
12
Answer: B

smle ,2016

842

122-Male with abdominal stab wound, vital signs ( hypotension, tachycardia) ,What to give him first:
A. Ringer lactate
B. Fresh frozen plasma
C. Whole blood
D. Packed RBCs
Answer: A
123- 5 year old with blunt abdominal trauma, Upper GI series showed coil spin in 2nd 3rd duodenal
and high amylase, How to manage:
aparotomy and hematoma evacuation
gastroenterostomy
bowel rest
CT guided needle for hematoma extraction
Answer: C *Not sure*
124- Pneumothorax management ?
needle decompression
Answer: AClassical management of tension pneumothorax is emergent chest decompression with needle thoracostomy. A 14-16G intravenous cannula is inserted into the second rib space in the mid-clavicular line.
http://www.trauma.org/archive/thoracic/CHESTtension.html
125-Pts playing tennis, something bites him (birds I think) after few minutes he has respiratory distress and he was collapsed what is the immediate tratment for this patient?
Antihistamine
Epinephrine
Answer: B
Anaphylaxis
126-Case scenario about patient who had injury in his right knee, the doctor applied a valgus stress to
examine which ligament:
A. Anterior cruciate ligament
B. Posterior cruciate ligament
C. Fibular collateral
D. Tibial collateral
Answer: D
medial collateral (tibial collateral) ligament
127- ptn came to ER with multiple fracture no loss of conscious with Low bp , tachycardia , normal RR ,
O2 saturation = 95% ?
IV fluid
Save airway
answer: B
All resuscitations should be performed using Advanced Trauma Life Support (ATLS) guidelines.For the
individual physician, assessment of the polytraumatized patient is performed using a stepwise longitudinal approach, in which the airway is handled first and no procedures are initiated until the airway is secured. Then, breathing and circulation are addressed
Reference: Medscape

smle ,2016

843

128- An Elderly pt presented to ER due to decreased level of consciousness lethargy Pco2 20 mmhg,K
2,Ph 7.2 I can't remember the rest of the labs and choices
What does she have?
answer: high anion gap metabolic acidosis with primary respiratory alkalosis
we can't be sure about the anion gap without the rest of the lab result otherwise the answer is correct
129- Why CT is best in blunt trauma
is best in unstable patient
Better to detect on retero-peritoneum bleeding
answer: B
CT scans of the abdomen and pelvis usually are performed together, using both IV and oral contrast. Use
this study to identify injuries to abdominal and pelvic organs and to identify bleeding in the retroperitoneum and pelvis
Reference: Medscape
130-patient come to ER unconscious :
check pulse
examine pupils
IV fluid infusion
clear airway
Answer: D
131-MVA with weak heart sounds
and silent right side of the chest, distended neck veins, next?
Needle decompression
cardiac window
Pericardiocentesis
answer: A
Reference: http://www.emsworld.com/article/10324543/penetrating-chest-trauma
132-MVA victim presented to ER
with hypotension, given crystolloid his BP return to normal, and chest tube was inserted. what to do
next?
Re-examine the chest
ABG
Answer: A
http://www.cram.com/flashcards/intern-year-atls-practice-questions-2626175
133-The fastest way to clean child stomach with iron over dose?

smle ,2016

844
gastric lavage
serup something
enema something
other option .
Answer: Deferoxamine is the iron-chelating agent of choice. Deferoxamine binds absorbed iron, and the irondeferoxamine complex is excreted in the urine.
Reference: Medscape
134- a case of heat stroke?
warm
fluid
core
cooling whole body
electrolytes replacement
Answer : D
Heatstroke is a medical emergency and continues to be one of the leading causes of preventable death
in sports.[4] Rapid reduction of the core body temperature is the cornerstone of treatment because the
duration of hyperthermia is the primary
determinant of outcome.
Reference: medscape
135-The effectiveness of ventilation during CPR measured by:
a. Chest rise
b. Pulse oximetry
c. Pulse acceleration
Answer: A
136-Trauma Or depressed skull fracture what immediate thing to do (ER)
Intubate
O2 supplement*
IV fluid
Answer: A
Management
- ABCs
- ensure oxygen delivery to brain through intubation and prevent hypercarbia
- maintain BP (sBP >90)
- treat other injuries
early neurosurgical consultation for acute and subsequent patient management
Reference: Toronto notes.

smle ,2016

845

Family Medicine
smle ,2016

846

1- What is most common serious chronic infection found in expatriates coming to Saudi
Arabia?
Hepatitis A
Hepatitis B
Hepatitis C
HIV
Answer: B
Reference: http://applications.emro.who.int/emhj/v19/07/EMHJ_2013_19_7_664_670.pdf?ua=1
Reference: http://www.alliedacademies.org/articles/profile-of-viral-hepatits-in-saudi-arabia.pdf
http://www.ncbi.nlm.nih.gov/pubmed/24975313

smle ,2016

847

2- The most difficult method to prevent transmission:


a.
Person to person
b.
Vector
c.
Droplet
d.
Airborne
Answer: D
Reference: 3rd Edition UQU last touch > Family medicine and statistics > Q 3.
http://www.maine.gov/dhhs/mecdc/infectious-disease/epi/airborne/
http://www.medscape.com/viewarticle/741245_3
3- The chairman of the public health wants to reduce the incidence of stroke. He is reading
the literature but he is confused. What is the best intervention?
Cholesterol level test for all population
Anti Smoking campaign
Hypertension booth in the malls
Obesity booth in the mall
Answer: C
4- An adult( 18 years old he did not receive varicella vaccine ) came to you for varicella
vaccine. How will you give it to him?
one dose now and one after 2 weeks.
one dose now and one after 3 months
one dose now and one after 6 weeks
just give one dose now
Answer: C
For adult; two doses of varicella vaccine at least 4 weeks apart (the 2nd dose can be given after
4 weeks or more)
For children; two doses of varicella vaccine at least 3 months apart
http://www.immunize.org/askexperts/experts_var.asp
5.pt take one dose of varicella vaccine and after one year present to your clinic..
give dou The varicella vaccine in adult take 2 dose with 4-8 w in between .
you must catch up vaccination with a second dose for all adolescent and adult who may have
missed a second dose .
6- Least harmful vaccine in immunocompromised patient.
BCG
pneumococcal vaccine
measles
Mumps
Answer: B
Pneumococcal vaccine (both conjugate and polysaccharide) classified as a subunit vaccine,
which is like inactivated vaccine can be given to immunocompromised patients.

smle ,2016

848
BCG, measles and mumps are a live attenuated vaccine which should not be given to immunocompromised patient.
http://www.phac-aspc.gc.ca/publicat/cig-gci/p03-07-eng.php
MMR (measles, mumps and rubella) vaccine is the only exception of live attenuated vaccine
type can be administered to asymptomatic HIV/AIDS patient (not all immunocompromised
patients)
ble dose
give 2nd dose
start over
antibody test
Answer: give 2nd dose
B?
8- Which of the following vaccines is less harmful to immune-compromised pt?
Hepatitis recombinant B vaccine
BCG
salk polio
sabin polio
Answer: A
https://www.cdc.gov/mmwr/preview/mmwrhtml/00023141.htm
9- HIV patient scenario but no dx (no cd4 count), which vaccine he should not receive?
Hep B
Varicella
DTaP
Answer: B
HIV is a defect in cell-mediated immunity
http://www.who.int/vaccine_safety/committee/topics/varicella/Jul_2013/en/
Previous answer: If CD4 > 200, MMR and varicella are only live vaccine that should be
given to HIV pt
Don't give OPV to HIV pt or their contact
10- There is a new outbreak of TB. What will you do?
Give Ethambutol chemoprophylaxis.
Give Rifampicin chemoprophylaxis.
Bacillus Calmette-Gurin vaccine
Other 2 irrelevant choices.
Answer: B
Rifampicin: Prophylaxis for those exposed to people with N. meningitidis or HiB meningitis.
Reference: Toronto Notes
11- What would you advise someone who is travelling to (?) to do most importantly, as precaution from Travelers diarrhea?
Eat fruits that you can peel
Eat washed fruits and vegetables

smle ,2016

849
Prophylactic antibiotics
Drink iced water.
Answer: A
Reference:
https://wwwnc.cdc.gov/travel/yellowbook/2016/the-pre-travel-consultation/food-water-precautions
12- Ttt of traveler's diarrhea?
Answer: ciprofloxacin
Both as empiric therapy or for treatment of a specific bacterial pathogen, first-line antibiotics include fluoroquinolones, such as ciprofloxacin or levofloxacin.
https://wwwnc.cdc.gov/travel/yellowbook/2016/the-pre-travel-consultation/travelers-diarrhea
12- What is the most common problem faced by community medicine in PHCC.
HTN
Coryza
UTI
Answer: B, from FM consultant (OM al Qura, family medicine)
13- What questionnaire to ask about alcohol intake?
Answer:
http://www.niaaa.nih.gov/research/guidelines-and-resources/recommended-alcohol-questions
14- An example of secondary prevention is:
a.
Detection of asymptomatic diabetic patient
b.
Coronary bypass graft
c.
Measles vaccination
d.
Rubella vaccination
Answer: A
https://www.iwh.on.ca/wrmb/primary-secondary-and-tertiary-prevention
Primordial prevention: consists of actions to minimize future hazards to health and hence inhibit
the establishment factors (environmental, economic, social, behavioural, cultural) known to increase the risk of disease, e.g. improving sanitation. It addresses broad health determinants rather
than preventing personal exposure to risk factors, which is the goal of primary prevention.
Primary prevention: Action to protect against disease as immunization and to promote health as
healthy lifestyle.
Secondary prevention (screening): Identifying & detecting a disease in the earliest stage before
symptoms appears.
Tertiary prevention: Improves the quality of life of people with various diseases by limiting the
complications.
Reference: 3rd Edition UQU > Family medicine and statistics > Q 98.
15- An example for secondary prevention.
A) Vaccine

smle ,2016

850
B) Screening for HTN
C) Coronary bypass graft
Answer: B
https://www.iwh.on.ca/wrmb/primary-secondary-and-tertiary-prevention
16- Healthy young adult with high cholesterol level. When will you follow him up again for
dyslipidemia?
6 months
12 months
24 months
36 months
Answer: A
The benefits of lifestyle modifications usually become evident within 6 to 12 months. However,
the success of lipid lowering with lifestyle modification varies widely, and healthcare providers
sometimes elect to begin drug therapy before this time period is over. http://www.uptodate.com/contents/high-cholesterol-treatment-options-beyond-the-basics?source=see_link ,
Fasting lipid levels should be obtained 6-8 weeks following the initiation or alteration of therapy.
For patients at goal, lipid profiles should be performed annually.
Reference: https://ghcscw.com/SiteCollectionDocuments/Clinical_Practice_Guidelines/8_CPG_Dyslipidemia.pdf.
Less than two Cardiac Risk Factors with elevated lipids: Repeat lipid panel in 3-6 months
Reference:http://www.fpnotebook.com/cv/Lipid/Hyprchlstrlm.htm
17- A patient is concerned about microwave radiation and its risk of cancer on their children. They are asking for your advice, what will you tell them?
Microwave cause cancer but not in children
Microwave don't cause cancer
Answer: B
If you use a microwave oven in the correct way there is no known harmful effect on humans.
http://www.cancerresearchuk.org/about-cancer/cancers-in-general/cancer-questions/radiationmicrowaves-and-cancer
18- Vegetarian patient having angular cheilitis:
Vit B6 deficiency
Answer: A
Causes of angular chelitis: Vit B2, Vit B5, Vit B12, Vit B3, Vit B6 deficiencies, zinc and iron
deficiency,
Vitamins deficiency among vegetarians: Vit B12, D, B6,
B12 deficiency is a common problem among people who follow a vegan diet.
Reference: Webmed + Wikipedia + Medscape.
19- What is the organism found in undercooked meat?
Entameba histolytica
Answer: A
Organisms that can be found in raw meat:
smle ,2016

851
Beaf: E. coli O157:H7, Salmonella, Shigella, Staphylococcus aureus and Listeria monocytogenes
Poultry: Salmonella and Campylobacter
Shellfish: Vibrio gastroenteritis, Salmonellas, Plesiomonas shigelloides, Staphylococcus and Bacillus cereus

Reference: Toronto Notes.


20- Best types of carbohydrate in DM.
Monosaccharide
Disaccharide
Polysaccharide
Answer: C
Simple carbs have only one or two sugars, so they are digested quickly, making blood glucose
rise rapidly to a high peak, which is what diabetics need to avoid. Examples of simple carbs include the sugars found in fruits and milk, the added sugars in processed foods, and table sugar.
Complex carbs contain three or more sugars, so these take longer to digest and thus they cause a
less rapid rise in blood glucose and a lower peak. Examples of complex carbs include the fibers
in spinach, watercress, buckwheat, barley, wild or brown rice, beans, and some fruits.
21- Children who are living in a poor country with poor hygiene will have a high risk to
hepatitis:
a.
A
b.
D
c.
E
Answer: A
Hepatitis A is usually spread when the Hepatitis A virus is taken in by mouth from contact with
objects, food, or drinks contaminated by the feces (or stool) of an infected person. Most common
type of hepat
http://www.cdc.gov/hepatitis/hav/afaq.htm#overview
22- A school did a screening test for their students and they found that there is a good number of obese students(BMI) . The school doctor wants to know more about these students
before educating their parents. What should you provide him with?
smle ,2016

852
a.
HDL\LDL
b.
Girth measurement
c.
Dietary habits
Answer: C?
Reference: Oxford General practice, 4th edition, obesity (P:174)
23- Medical director discovered cretinism in 90% of children in his village, when he analyzed the water he found that it is deficient in iodine. The director wants to prevent and
manage cretinism. What he is going to do initially?
a.
Iodine supplementation
b.
Thyroxin supplement (Levothyroxine).
c.
TSH and T4 in 2 weeks.
Answer: B
The mainstay in the treatment of congenital hypothyroidism is early diagnosis and thyroid hormone replacement.
Reference: Medscape.
24- What is the mineral found in water prevents teeth cavities?
Folic acid
Iron
Fluoride
Answer: C
25- The targeted glycated hemoglobin in a patient with type 1 DM should be:
a.
0.065
b.
0.08
c.
0.095
d.
0.11
Answer: A
Therapy in most individuals with type 1 or type 2 diabetes should be targeted to achieve a
HbA1c 7.0% in order to reduce the risk of microvascular and if implemented early in the
course of disease, macrovascular complications.
More intensive glucose control, HbA1c <6.5%, may be targeted in patients with a shorter duration of diabetes with no evidence of significant CVD and longer life expectancy, to further reduce risk of nephropathy and retinopathy, provided this does not result in a significant increase
in hypoglycemia.
A HbA1c target <8.5% may be more appropriate in type 1 and type 2 patients with limited life
expectancy, higher level of functional dependency, a history of recurrent severe hypoglycemia,
multiple co-morbidities, extensive CAD, and a failure to attain established glucose targets despite treatment intensification. Reference: Toronto notes.
26- (long scenario) old man with DM and HTN, came for routine checkup. ( CBC, FBS,
Lipid profile .. etc.). What is the targeted glycosylated hemoglobin?
a.
0.065
b.
0.070
Answer: B

smle ,2016

853
See the explanation of previous question
27- 32-year-old athlete man, who has a family history of HTN and DM, came for check
up. He has good musculature and doing weight-lifting exercises. His blood glucose level
within normal range also his lipid profile except for total serum cholesterol 210 mg/dl.
What is your action?
a.
Start statin.
b.
Change diet.
c.
Check up in next few months.
d.
Reassure him.
Answer: B
Borderline high cholesterol treated with diet modification.
28- Young female with height of 167 cm and weight of 153 kg. In which class of BMI ?
a.
Overweight.
b.
Obesity class I.
c.
Obesity class II.
d.
Obesity class III.
Answer: D

Easy way to calculate BMI: Wt. 153 kg, Ht. 167 cm 1.67 meter.
Step 1: Wt./Ht. (in meters) = 153/1.67 = 91.6
Step 2: Result of step one divided again by Ht. (in meters) = 91.6/1.67 = 54.8
Answer is 54 Which is extreme obese.
29- 28-year-old girl came for checkup. She asks about when to do breast self examination?
a.
Not advised.
b.
Advise every 4 months.
c.
Advise every 12 months.
d.
Advise 3 years.
Answer: A
Reference: Oxford General practice, Edition 4 (P:686)

smle ,2016

854
30- Saudi Arabia has a good screening for military job. what the infection he man can't
take the job?
a.
Hepatitis A virus
b.
Hepatitis B virus
c.
Hepatitis C virus
d.
HIV
Answer: D?
Reference: http://www.beforejoiningthemilitary.com/military-medical-requirements-and-disqualifying-conditions/
31- What is the vaccine type of Hepatitis B?
Answer: ?
Subunit
32- What is the vaccine type of Pneumococcal vaccine?
Answer: ?
.Active vaccine
Live attenuated vaccine
Conjugated
Inactive vaccine
Both conjugate and polysaccharide are classified as a subunit vaccine (which is like an inactivated vaccine)
Refer to the table at the end of FM section.

33- Patient G2P0 had two abortions, came to you in clinic asking to give her any vaccine to
help her to get a baby, HCG is positive. Which vaccine will you give her?
Varicella
Rubella
Mumps
Influenza
Answer: D
Varicella, Rubella and Mumps vaccines are all contraindicated in pregnancy. Reference: Uptodate.
34- Pregnant woman came to the clinic asking about tetanus risks and prevention. There is
a history of contact with patients who have tetanus. On examination: Normal and the measurement of uterine level from symphysis is 12 cm. What is the action to prevent baby from
tetanus?
a.
Introduce Acyclovir to the mother at 18 weeks.
b.
Tetanus immunoglobulin and vaccine to the baby after delivery.
c.
Give Acyclovir to mother and baby after delivery.
d.
Give tetanus vaccine to the mother.
Answer: D
Reference: http://www.cdc.gov/vaccines/pubs/preg-guide.htm#tdap

smle ,2016

855
35 - best way of prevention :
Screening program
Genetic counseling
Increase individual health awareness
ENVIRONMENT MODIFICATION
Answer: Increase individual health awareness ( or behavioral modification of the person)
36- which of the following will potentially will decrease the disease
screening
environmental modification
improve the personal health
education
Answer : ??
37- What is the best measure in health care ?
improve personal something
screening
individual education
behavioral modification of the person( newly added)
Answer:2 statistical questions ..one about sensitivity and spectifity Another one about mortality rate
38- THE MOST R.F OF STROKE?
DM
HTN
SMOKING
.....
Answer: B
39- THE MOST """REVERSIBLE"""" CAUSE OF STROKE IS ?
OBESITY
DM
HTN
HYPERLIPIEMIA
Answer:C
40- patient with normal glucose level and HBA1c, he is 42 male that smokes for 20 years,
when do you do these tests again?
.after 3 months
6 months
12 months
36 months
Answer : D
41- Pt came with HTN and get controled by medication after 1 week she came with dry
cough which medication is prescribed ?
Perindoprill (ACE)
smle ,2016

856
Answer: A
42- increase of which of the following prevalence cause reactivation of TB in developed
countries?
DM
HIV
Answer: B
43-72 old patient start to have memory loss gradually since 2 yrs back ..but he is capable
of doing his daily activity dressing himself but lately he start to forget the burner on..
and his personality changed from kind and caring father to agg. And irritablewhat u will
do
Do cost effective Ix
Refer to geriatric
TCA trial
Give him Risperidone ( antipsychotic ) e) Arrange to transfer him to caring facility true for severe case
Answer:.
B (UQU sle ) q151 p447 4th edition
44- Which type of hepatitis have available vaccine
A. HBV
Answer: A
45-In KSA, there is strict rules for who are working in restaurant in order to prevent
which type of hepatitis:
A
B
D
E"
answer :A
46- Frequency of Blood sugar test screening in healthy man?
Answer: 3 years
47- Virus that can turn into a new virus and cause a pandemic?
Influenza
Rhinovirus
Parainfluenza
RSV
Answer: a
48-17.Most common infection that is found in expatriates before they start working?
HIV
Hep B
Hep C
smle ,2016

857
Answer : B
49-the community department want to decrease the icidende on Storke what is the appropriate action?
education the population about the Hypertension in mall
check for HYPERLIPIDEMIA in mall
check for BMI for people in malls.
Answer: education the population about the Hypertension
50-risk for osteoporosis :
Answer: Age( Aging process)
51-viral gastroenteritis prevented by which vaccine :
A-Rota vaccine
Answer:
rota vaccine prevent GI and given at 2 , 4 month
We recommend universal immunization of infants against rotavirus (Grade 1A).
Uptodate.com
52-vaccination
Rubella vaccination
Answer:Qs isnt complete, but it sounds that its about contraindicated Vaccines during pregnancy
Intrapartumimmunization :
Administration is dependent on the risk of infection vs. risk of immunization complications
safe: tetanus toxoid, diphtheria, influenza, hepatitis B, pertussis
avoid live vaccines (risk of placental and fetal infection): polio, measles/mumps/rubella,
varicella
contraindicated: oral typhoid
Torronto Notes. 2016
53- 29 yrs old female , came for her annual checkup , her father was dx with dyslipidemia
one year ago and she is anxious about she will have the same thing , he lab were all normal
except for high triglycerides , what will you give her ?
Answer: diat and excersisefor 3 month
In general, lifestyle modifications (eg, smoking cessation, diet, exercise, weight reduction) are
initiated before any pharmacologic therapy in the treatment of primary and secondary
dyslipidemia, particularly in patients who are asymptomatic
Medscape.com
53- effective Way for detected severity of pain with people non communicated
face ..
A- number
B- verbal
Answer: A , number

smle ,2016

858

54- If mammogram free when repeat it (


A-2 y
B-3 y
C-4 y
D-5 y
Answer: B , Every 3 Yrs
Oxford Handbok of General Practice 4th Ed. Chapter 20 page 694-695
56- What is the best sentence you ask him to know about his satisfaction about the asthma
?
Incomplete ! No choices given
57-Vaccine of hep.a missed second dose what to do ?
Answer: Children who have not received the 2nd dose by age 2 years should be vaccinated as
soon as feasible.
http://www.immunize.org/askexperts/experts_hepa.asp
58- patient with low back pain and normal neurological examination , MRI showed mild
spinal stenosis .. What is Tx :
a- surgery
b- physiotherapy
c- biofeedback
answer: B
Non-surgical treatments For mild to moderate pain, more conservative treatment methods can
include medications, physical therapy, and steroid injections.
http://www.back.com/back-pain/conditions/lumbar-spinal-stenosis/
Nonsurgical treatments used for lumbar spinal stenosis (LSS) have included physical therapy,
analgesic and anti-inflammatory medications, and epidural steroid injections
Uptodate
59- splenectomy what vaccin he should took
Answer: pcv +menengococcal vaccine + hib
Spleen is an organ that remove damaged RBCs and also it's also protect the body against bloodstream infection by removing the bacteria from the blood .( is important to fight infection..)
So, splenectomy or unfunctional spleen can cause sepsis
Most common organism which cause sepsis is streptococcus pneumoniae , less commonly neisseriameningitidis or Hib
smle ,2016

859
So ,
A-pneumococcal vaccine is recommended and it has 2 types :
1- PPSV23
2- PCV13
B- Hib vaccine
C- meningococcal vaccine _ meningococcal conjugated vaccine
You can read more on
http://www.uptodate.com/contents/preventing-severe-infection-after-splenectomy-beyond-thebasics?source=outline_link&view=text&anchor=H1#H1
60- mumps in child what is the most common complication ?
orchitis
meningitis
Answer: B commonest
Complications :
common: aseptic meningitis, orchitis/oophoritis
less common: encephalitis, pancreatitis, thyroiditis, myocarditis, arthritis, GN, ocular complications, hearing impairment
Source: Toronto note in (infectious diseases)
The most common complication of mumps in children is meningitis, sometimes associated with
encephalitis, and in young adults orchitis.
WHO
http://www.who.int/biologicals/areas/vaccines/mmr/mumps/en/
61-enteric fever best diagnosed in the first week of presentation by ?
A -blood culture
B -stool culture
Answer:-A
A blood culture during the first week of the fever can show S. typhi bacteria .urine and stool in
second week .
https://www.nlm.nih.gov/medlineplus/ency/article/001332.htm
Blood cultures are positive in 40 to 80 percent of patients, depending upon the series and culture techniques used. Blood cultures may require several days of incubation. The diagnosis can
also be made by culture of stool, urine, rose spots, or duodenal contents (via string capsule)
.Stool culture is positive in up to 30 to 40 percent of cases, but is often negative by the time that
systemic symptoms bring patients to medical attention .
Uptodate.
62 - women came for check up, her husband is+ve of HBsAg
put all investigation like HBcAb, HBsAg for women are -ve which marker should be detect
if +ve or -ve in this women :
HB Igm
HBeAg
Answer A )HBigM will be elevated first
IGM Anti HB indicate an acute infection within <6 months

smle ,2016

860
63- 2 years child came with Hx of fall down on x-ray # in radial what will do for this child:
referral to pediatrics
" to ortho
admission
splint for hand
ORTHO Qs
Answer: b!!
forearm fracture is evaluated according to age , deformity of the hand , stability of the fracture
the younger the child more likely that a simple closed reduction and casting
so i think orthopedic the one who can evaluated better
64-the most effective nonpharmacological treatment for hypertension ??
a-Low sodium diet
b-Decrease Wight
Answer a:A
non-pharmacological (lifestyle measures) as well as pharmacological means. Lifestyle changes
should be the initial approach to hypertension management and include dietary interventions (reducing salt, increasing potassium, alcohol avoidance, and multifactorial diet control), weight reduction, tobacco cessation, physical exercise, and stress management
http://www.ncbi.nlm.nih.gov/pmc/articles/PMC3028941/
DASH diet + Low Sodium Diet is most effective measures for high BP
65-Doctor is asking the patient to perform Physical examination?
Getting informed consent.
Show respect to the patient
Answer : explain what you will do and take permission
66-patients with a specific disease in a thousand. 2.5 days are disability and 1.5 absent
workers calculate the incidence of disability annually
answer : I didn't find the correct answer
67-A question about the definition of specificity (it was written in a very complex english so
understand the definition very well and read the Q with focus)
answer : the quality or condition of being specific also called true negative
68-Which of the following vaccination is appropriate for asplenic patients:
Pneumococcal vaccine
answer: Pneumococcal vaccine, HiB, Meningococcal those 3
http://www.cdc.gov/vaccines/adults/rec-vac/health-conditions/asplenia.html
If you do not have a spleen or your spleen does not work well, talk with your doctor about:
Influenza vaccine(http://www.cdc.gov/flu/protect/vaccine/ ) each year to protect against seasonal
flu
Tdap vaccine(http://www.cdc.gov/pertussis/vaccines.html) to protect against whooping cough
and tetanus
Hib vaccine(http://www.cdc.gov/vaccines/vpd-vac/hib/) to protect against Haemophilus influenzae type b (Hib) if you were not previously vaccinated with the vaccine
smle ,2016

861
Pneumococcal vaccines(http://www.cdc.gov/pneumococcal/vaccination.html) (both types) to
protect against pneumonia and other pneumococcal disease
Meningococcal vaccines(http://www.cdc.gov/meningococcal/vaccine-info.html) (both types) to
protect against meningitis and other meningococcal disease
Zoster vaccine(http://www.cdc.gov/shingles/vaccination.html) to protect against shingles if you
are 60 years and older
HPV vaccine(http://www.cdc.gov/hpv/vaccine.html) series to protect against human papillomavirus if you are a man up to age 21 or woman up to age 26
MMR vaccine(http://www.cdc.gov/measles/vaccination.html ) to protect against measles,
mumps, and rubella if you were born in 1957 or after and have not gotten this vaccine or have
immunity to these diseases
Varicella vaccine(http://www.cdc.gov/chickenpox/vaccination.html ) to protect against chickenpox if you were born in 1980 or after and have not gotten two doses of this vaccine or have immunity to this disease

69-case about addict person and ask which of the following questions is include in criteria
of CAGE questionnaire ?.
answer :
asks the following questions:
Have you ever felt you needed to Cut down on your drinking?
Have people Annoyed you by criticizing your drinking?
Have you ever felt Guilty about drinking?
Have you ever felt you needed a drink first thing in the morning (Eye-opener) to steady your
nerves or to get rid of a hangover?
https://en.m.wikipedia.org/wiki/CAGE_questionnaire
70- Female pt did mammogram which was negative, the doctor told her that it will repeated every 2 years, she insist to do it regularly, how many years the cancer develop before detection by mammogram
1
2
4
7
Answer : 2
71- patient can't take BCG vaccine Because he deficiency in
IL
gama INF
answer: B
72-22 female healthy with regular cycle, never had sex. History of bilateral breast pain 3
days before her cycle ,, no family history of cancers. She said she wants to get pregnant
within 2 years !!! What will you screen her for ,,,,, I DONT KNOW !!!
A) US breast
B) PaP Smear
C) human papilloma virus

smle ,2016

862
D) Gram stain for (streptococcus I think)
Answer: ??
73-the most effective way to prevent cardiac anomaly in pregnancy is ? #family
A- smoking cessation
B-genetic screen
or.....or.....
answer : most important diabetic control because it's associated with septal hypertrophy
74-What is the developmental Screening tool that relies on the parents information only?
Answer: Milestone
75- Patients with a specific disease in a thousand. 2.5 days are disability and 1.5 absent
workers calculate the incidence of disability annually
Answer: ??? I didn't find the correct answer
There were also missing questions about the following:
Vaccination types
Different types of research sampling

smle ,2016

863

76-Old lady with osteoporosis best prevention is :


- weight bearing exercise*
- daily Vit. D supplements
77-a boy dreamed of a bad dream and he woke up crying and feared but he does not remember any of the dream which stage of sleep he was on:
smle ,2016

864
1
2
3
4
78-Male patient K/C asthma on inhalor medication . Has oral white plaques difficult to remove . What medication causing these SE ?
- Ipratropium
- betamethasone
- sulbutamol
cromolyn
Answer: B
http://www.uptodate.com/contents/asthma-treatment-in-adolescents-and-adults-beyond-the-basics
79-Male was in swiming pool . 2 days later Has vomiting ,headache ,fever , No neck stiffness What is the possible bacterial ?
- S.pneumoniea
- H.influenzea
- N.meningitides
- Naeglria
Answer: D
http://www.cdc.gov/parasites/naegleria/
80-most common cause of cough in adult
Post nail drip
81-a case of T score of -3.5 what is the diagnosis:
osteopenia
osteoporosis
82-Diabetic pt on metformin and gliclazide not control what to add :
acarbos pioglotazon
83-How to treat water from entameba histoletica
boiling
clor
Answer : A
84-What is the prevention method for food poisoning
prolong heating and re cooking
antibiotic
prolong mastication ()
No peeled fuit in choices

smle ,2016

865
85-Man his wife diagnosed with osteoporosis, he has history of recurrent renal stone, labs
showed cant remember?!! what is the diagnosis ?
86- Parant came for consuling Hx of pt known to has HTN and now newly dignose of DM
And the medication given ? What was the medication for hypertension given in last appointment
a.ACEI...
87- duplex is
What to do?
angiograph
Venograph
duplex US
..MRI?
88-Osteoprosis
T spin -2.5
Z spin -2.0
T limb. ..
Z limb ...
dignosis scan
A.Esablish osteoprosis
B.Osteprosis
C.Oste...
89-Know about HSV1 and HSV2? What is the different ?
90-Consultation .pt with hypercholesterolemia What best food to avoid ?
Organ meats
Egg white
91-pt for annual checkup Everything is normle normle blood glucose What should be the
next check up?
3month ,
6,
12 month ,
36 month
92-Prevention medicine Whan to do lipde profile ?
93-Pt with hx of dyslipedemia on statins lipid profile is normal which drug would you add
to the medications ?
They mentioned the other medications of dyslipdmia
94-Drug cause hypoglacemia?
And they mentioned Some types of antideibetics

smle ,2016

866
Answer:
95-Incretins mechanism of action ?
Answer:
Workplace ..most common infectionous diseases at workplace.

HAV ,HBV ,HCV, HIV
96-Another one about most sigle risk factor modifiable to prevent strokes:
- cessation of smoking
Answer:
97-girl with depression in first 2 days of menstruation what dx? Mood swings, depression?!
98- Pt diagnosed as htn and started meds came back with high glucose. Wt was he given:
- Thiazides
Answer:
99-Pt diabeteic for years and was just dx as htn, wt to give:
A- ACEI- pril drugs
Answer:A
100-Pt with high chilsetrol on treatment, has muscles aches wt was she given:
A - statins
Answer: A
101-60 y/o has only HTN the best drug for him?!!
A- Diuretics
B- CCB
ANSWER I think ACEI
102- Side effect of atropine:
Dry mouth.
103. diabetic on oral hypoglycemic with hypoglycemia :
rosiglitazone
metformin
glyburide is sulfonylurea **
104-Most risk of CAD :
A - 50ys,DM,high cholesterol**
B - 55,HTN,obesity
C -young age,DM,obesity .
105-female came to PMC with recurrent UTI &amp; family physician asking you what
changes in urine analysis will confirm improvement?

smle ,2016

867
All choices contain urea , ph &amp; somthing which one of them increase or decrease.
106-old female smoker most risk for osteoporosis ? Fx ?
-age
-smoking
-vitamin d deficiency
107-16 year old came with headache band like , behind eye throbbing . Stressful life
- migraine
tension
108-old living alone Anemia ( )
B12 deficiency
109-10/ Best time to do self breast examination ?
Daily
Weekly
Monthly
Annually
110-Best first investigation to apply in pandemic disease?
Establish the diagnosis
Know the population who are in risk
111-Case of SSRI or TCA overdose (I am not sure). and they asked about diagnosis:
112-no Hx of DM in the family , asthmatic pt now his BS is within normal range when to
check again:
ANSWER 12m
113- 10 or 12 yrs girl diabetic since 3 years Which of the following should be checked annually?
OphthalmoscopY (annual follow-up visits with an optometrist or ophthalmologist examination
through dilated pupils whether symptomatic or not (tornto notes endocrinology )
- Celiac test
- kidney ultrasound
Answer: A
114-Question about tt of alzheimer
Treatment ( step 2 CK ,P 266)
n Prevention of associated symptoms:
n Provide supportive therapy for the patient and family.
n Treat depression, agitation, sleep disorders, hallucinations, and delusions.
n Prevention of disease progression: Cholinesterase inhibitors (donepezil,
rivastigmine, galantamine, tacrine) are first-line therapy for mild to moderate
disease. Tacrine is associated with hepatotoxicity and is less often used.

smle ,2016

868
Memantine, an NMDA receptor antagonist, may slow decline in moderate
to severe disease.
115- another Question about Alzheimer drug that is hepatotoxic
Tacrine is associated with hepatotoxicity and is less often used ( step 2 CK ,P 266)
116-how poliovirus vaccine works ?
117- Live attenuated vaccines( measles, mumps, DPT..)
118-Pt e high level of cholesterol , what to avoid?!
Organ meat
Avocado
Chicken
White egg
ANSWER: A
119-70 y/o female brought to ur clinic by her daughter . The daughter said her mother's
memory deteriorated in the last 2 years . She can dress her self but e difficulty , she can
cook for herself but sometimes leave the oven on ,,,,,,,,,what's the management ?!
Answer Refer her to geriatric clinic
120- Black man e high BP , +ve Fx Hx of HTN . Best med to give?'
- B blocker
ACEI
CCB firs then thiazide (first line antihypertensives: thiazide/thiazide-like diuretic, ARB, longacting CCB, -blocker (if age <60) tornto notes
ANSWER C
121-Tonsillitis case:
amoxicillin / clavilonic http://emedicine.medscape.com/article/871977-treatment#d10
vancomycin
ciprofloxacillin
trimethoprime / sulpha
ANSWER A
122-DM+HTN what to give medication
ACEI
123-pt on metformin , lab values given with low PH, normal urine osmolarity, no ketones ;
DKA
Hyperismolar hyperglycemia
Lactic acidosis
ANSWER C
124-Smoking withdrawal symptoms peak at ?

smle ,2016

869
1-3days (48hours)*
4-7 days
2 weeks
answer:A
125-Traveller diarrhea treatment !
rehydration therapy and antibiotic Like : fluoroquinolone antibiotics are the drugs of choice. Trimethoprim-sulfamethoxazole and doxycycline are no longer recommended because of high levels of resistance to these agent
answer :
126-viral gastroenteritis prevented by which vaccine :
Rota vaccine
127-Best medication for chronic pain disease
Ibuprofen
acetaminophen
ther is another choise started with ci >>>
answer :
128-Best ms relaxant in sever back pain?
Diazepam
Metaloxone
) 45
check routine

Lipid profile
ECG
answer :
129-farmer had sand fly related infection , how to treat ?
myofestin
answer :
130-Student had a meningitis what should you do for other students?
Antibiotics prohlyctic
answer :
131-A scenario about a patient who deals with flowers and got pricked by rose thorn, presented with redness at the prick site. which of the following is the cause:
Sporothrix schenckii .
Answer : A
smle ,2016

870
Sporotrichosis (also known as "Rose gardener's disease"[1]) is a disease caused by the infection
of the fungus Sporothrix schenckii
Referen
ce: https://en.wikipedia.org/wiki/Sporotrichosis
132-case of gonorrhea, what are you going to give his close contacts:
rifampin chemoprophylaxis
isolate all contacts for 4 weeks
meningiococal vaccine
133-F/u of blood lipids and F/u of blood sugar
Answer :
http://www.e-mercy.com/images/cholesterol-metabolism/research/2/Guidelines-to-InterpretResults-from-Health-Screening.pdf
134- High triglyceride what to add to statins?
Answer:
HMG CoA reductase inhibitor medications (statins) are most effective in lowering LDL cholesterol, mildly effective in increasing HDL cholesterol, and mildly effective in lowering triglycerides. Examples include pravastatin (Pravachol), lovastatin (Mevacor), atorvastatin (Lipitor), rosuvastatin (Crestor), and simvastatin (Zocor). Fibric acid derivative medications such as
gemfibrozil (Lopid) and fenofibrate (Tricor) are most effective in lowering triglycerides, effective in increasing HDL, and minimally effective in lowering LDL levels. Nicotinic acid (Niacin), known by the names Niacin, Niaspan, or Slo-Niacin, is most effective in increasing HDL,
effective in lowering triglycerides, and mildly to moderately effective in lowering LDL levels
http://www.emedicinehealth.com/script/main/mobileart-emh.asp?articlekey=114550&amp;page=7
135-athletic come for check up all thing normal except Xanthelasma on achllis tendon
and cholesterol?
-LDL receptor
-Apo ll
-Apo c
Answer: if they asked about mutation the answer will be LDL receptor Associated with severe
hypercholesterolaemia and elevated LDL levels Patients with xanthelasma had higher levels of
cholesterol, LDL-C, and apo B, and lower levels of HDL2-C than control subjects. The prevalence of the apo E4/E3 phenotype was higher in cases than in controls (P &lt; 0.05). Patients
with xanthelasma had a higher prevalence of personal and familiar history of cardiovascular
disease and were more overweight than control subjects. A stepwise discriminant analysis disclosed an independent association of xanthelasma with lower HDL-C, HDL2-C, and HDL3-C
levels in men, and with higher total cholesterol and lower HDL2-C levels in women
136-Normally, thechild cansit without support and support his head, laughing and cooing
at the age of:
4 months
6 MONTHS
smle ,2016

871
8 months
16 months
ANSWER:B
REFERENCE:
Toronto Note 2015 Pediatrics Section
Nelson Textbook of Pediatrics, 19 Ed
Illustrated Textbook of Paediatrics, 4th Ed (Reference advised by SCHS)
EXPLANATION:
This question involves three aspects of the normal developmental milestones of children; 1)
gross motor, 2) speech and language, and 3) social behavior or skills.
Children can start cooing at the age of TWO months(ranging from two to four months in some
children, but most of them achieve it by two months).
They start to sit unsupported at the age of SIX months (ranging from six to nine months in some
children, but most of them achieve it by six months).
Finally, they start laughing at the age of THREE months (they start SMILINGat the age of
TWO months, and LAUGHING at THREE months).
So, the correct answer will be 6 MONTHS.
137- Most Important risk factor for cardiacdiseases is:
LDL
Total cholesterol
HYPERTENSION
Smoking
ANSWER: C
REFERENCE:
Toronto Notes 2015 Cardiology and Cardiac Surgery Section
138- Burn patient, weighing70 kg, had circumferential upper and lower left limbs + anterior trunk .How much fluid youll give in the first 8 hours?
4 liters
6 liters
8 liters
10 liters
ANSWER: B
REFERENCES:
Toronto Notes 2015 Plastic Surgery Section
EXPLANATION
In burn cases, fluid resuscitation is a priority. The amount of fluid that should be given to the
patient is determine by the size of the burn. For 2nd and 3rd degree burns, rule of 9 should be
followed (this is in adults and children > 10 years of age; in children < 10 years of age, LundBrowder Chart should be used).
Rule of 9 divides total body surface area (TBSA) into:
Anterior head and neck 4.5 % of TBSA
Posterior head and neck 4.5 % of TBSA
Anterior trunk 18 % of TBSA
smle ,2016

872
Posterior trunk 18 % of TBSA
Anterior upper limbs 9 % of TBSA (each is 4.5% alone)
Posterior upper limbs 9 % of TBSA (each is 4.5% alone)
Anterior lower limbs 18 % of TBSA (each is 9% alone)
Posterior lower limbs 18 % of TBSA (each is 9% alone)
Perineum 1 % of TBSA
In this Question, upper (9% because it is circumferential) and lower left (18% because it is
circumferential) and anterior trunk (18%) [9 + 18 + 18 = 45% TBSA].
Resuscitation should follow Parkland Formula. 4ml x TBSA (%) x body weight (kg); 50%
given in first eight hours; 50% given in next 16 hours.
during the first 24 hours. Half of the total fluid should be given during the first 8 hours, and the
remaining half during the following 16 hours.
For this Question (4 * 70 * 45 = 12600 ml (12.6 Liters) during the first 24 hours.
Thus, during the first 8 hours, (12.6 / 2 = 6.3 Liters).
Monitoring of resuscitation will be through monitoring urine output. In adults, it should be >0.5
cc/kg/h. For pediatrics, it should be >1.0 cc/kg/h.
139-Child is doing fine, his brother died while heading to work! What we should investigate for?
Hypertrophic cardiomyopathy
Familial hypertrophic cardiomyopathy is a heart condition characterized by thickening (hypertrophy) of the heart (cardiac) muscle. Thickening usually occurs in the interventricular septum,
which is the muscular wall that separates the lower left chamber of the heart (the left ventricle)
from the lower right chamber (the right ventricle). In some people, thickening of the interventricular septum impedes the flow of oxygen-rich blood from the heart, which may lead to an
abnormal heart sound during a heartbeat (heart murmur) and other signs and symptoms of the
condition. Other affected individuals do not have physical obstruction of blood flow, but the
pumping of blood is less efficient, which can also lead to symptoms of the condition. Cardiac
hypertrophy often begins in adolescence or young adulthood, although it can develop at any
time throughout life.
The symptoms of familial hypertrophic cardiomyopathy are variable, even within the same family. Many affected individuals have no symptoms. Other people with familial hypertrophic cardiomyopathy may experience chest pain; shortness of breath, especially with physical exertion;
a sensation of fluttering or pounding in the chest (palpitations); lightheadedness; dizziness; and
fainting.
While most people with familial hypertrophic cardiomyopathy are symptom-free or have only
mild symptoms, this condition can have serious consequences. It can cause abnormal heart
rhythms (arrhythmias) that may be life threatening. People with familial hypertrophic cardiomyopathy have an increased risk of sudden death, even if they have no other symptoms of the
condition. A small number of affected individuals develop potentially fatal heart failure, which
may require heart transplantation

smle ,2016

873
140- theoretically which of the following cancer will prevented by vaccination?
ALL
CML
adult T cell leukemia
myocois something
I THINK THE QUESTION IS WRONG!!
141-76old woman came for check up and said she hasdecrease calcium intake. She had a
pelvic fracture 10 years ago. What is the most important risk factor of osteoporosis for this
woman?
Gender
AGE
Calcium level
Previous fragility fracture
ANSWER: B
REFERENCE
Toronto Notes 2015 Endocrinology Section
Kumar 7th Ed (Reference advised by SCHS)
Medscape
EXPLANATION:
This lady has primary osteoporosistype 2 which is most common in elderly above 75. All the
options are considered risk factors for this ladys osteoporosis; however, THE AGE is the most
important risk factor for her.
Primary osteoporosis type 1 is more common post-menopausal women, which is due to decline
in estrogen levels.
142-Patient with muscle and joint pain. He takes anti-lipid medication; which medication
is most likely responsbilly for his complaint?
STATIN
Fibrate
Niacin
Bile acid sequestrants
ANSWER: A
REFERENCE
Toronto Notes 2015 Endocrinology Section
EXPLANATION:
Statins (HMG Co-A reductase inhibitor) are among the most common dyslipidemia medication
that are used. One of the most common side effects for is myalgia (muscle pain).
Fibrate can cause muscle pain as well; if COMBINED with statins.
143-Couples will get marry, and they are relatives. Which screening test should be done
before marriage?
G6PD qualitative test
HIV 1 & 2 antibodies screening
HB ELECTROPHORESIS (SICKLE CELL DISEASE)
Hepatitis B surface antigen

smle ,2016

874
ANSWER: C
REFERENCE
Toronto Notes 2015 Hematology Section
This question had no answers initially, only the explanation. So I wrote the answers, to match
the concept of the question. Thus, I couldnt find an appropriate Reference for this questions.
EXPLANATION:
Answer: usually choose the answer which contains a test for an autosomal recessive disease.
144- 45 years old male came for regular check-up.Apart from bronchial asthma, his BP is
125/80, HbA1C is 5.9. When is the next time he should check his blood glucose level?
After 1 month
After 3 months
After 6 months
After 12 months
ANSWER: B
REFERENCE:
Toronto Notes 2015 Endocrinology Section
Medscape
EXPLANATION:
The target for HbA1C in patients with DM-II is to be below 7% (pre-prandial). If the goal is
achieved, follow up for DM-II will be every THREE months.
However, if the goal is not achieved, a two-drug combination will be used, and follow up will
be in THREE months.

145- A child with DM type 1 was brought to school clinic unconscious. His last insulin dose
is unknown. What is the next best step to do?
IV DEXTROSE
RL
Insulin
Urgent referral to hospital
ANSWER: A
REFERENCE:
Medscape
Illustrated Textbook of Paediatrics, 4th Ed (Reference advised by SCHS)
EXPLANATION:
Short-term treatment of hypoglycemia consists of an intravenous (IV) bolus of dextrose 10%
2.5 mL/kg. The critical sample should be drawn before the glucose is administered. After the
bolus is administered, an IV infusion that matches normal hepatic glucose production (approximately 5-8 mg/kg/min in an infant and about 3-5 mg/kg/min in an older child) should be continued. This should be adjusted to maintain the plasma glucose level at more than 3 mmol/L.
Children with hyperinsulinemia may have much higher needs. Glucagon infusion at rates of
0.005-0.02 mg/kg/h should be used as a temporary treatment in children with hyperinsulinism
in whom adequate amounts of dextrose cannot be given. It can cause a rash and decreased appetite if used over the long term.

smle ,2016

875
146- What is the treatment of choice for respiratory syncytial virus?
SUPPORTIVE THERAPY
Antibiotic (amoxicillin)
Antiviral (ribavirin)
Beta 2 agonist (salbutamol)
ANSWER: A
REFERENCES:
Medscape
Toronto Notes 2015 Pediatric + Family Medicine Sections
http://www.mayoclinic.org/diseases-conditions/respiratory-syncytial-virus/basics/treatment/con-20022497
EXPLANATION:
Respiratory syncytial virus (RSV) is the most common cause of bronchiolitis in children. The
mainstay treatment of RSV is supportive therapy (for mild-moderate cases). This involves bed
rest, hydration, and analgesia. For severe cases, hospitalization is required.
The use of antibiotics in children has no rule. However, it can be used in elderly or in case of
pneumonia.
147-What is the best way to prevent transmission of brucellosis to humans?
Vaccination of domestic livestock
Prophylactic course of antibiotics (doxycycline)
Vaccination for humans
PASTEURIZATION OF MILK PRODUCTS
ANSWER: D
REFERENCES:
Kumar 7th Ed (Reference advised by SCHS)
UpToDate
REFERENCES:
Brucellosis (Malta fever) is a zoonosis and has a world-wide distribution. The highest incidence is
in the Mediterranean countries, the Middle East and the tropics; there are about 500 000 new cases
diagnosed per year.
Prevention and control involve careful attention to hygiene when handling infected animals
andvaccination with the eradication of infection in animals. No vaccine is available for use in
humans.
PASTEURIZATION OF MILK IS ALSO VERY IMPORTANT FOR THE PREVENTION
OF TRANSMISSION TO HUMAN.
Brucellosis is treated with a combination of doxycycline200 mg daily and rifampicin 600900 mg
daily for 6 weeks,but relapses occur. Alternatively, tetracycline can be combinedwith streptomycin, which is usually given for only the first 2 weeks of treatment.
148- Which vaccination can cause diarrhea?
DTaP
Varicella
ROTA VIRUS
MMR
REFERENCE:

smle ,2016

876
Toronto Notes 2015 Pediatrics Section
Medscape
EXPLANATION:
One of the most common side effects of rotavirus is diarrhea, which can be severe and leads to
hydration.

149- Calculation of BMI and what are the degrees of obesity?


The equation

REFERENCE:
Toronto Notes 2015 Family Medicine Section
150-What is type of punmoccocal vaccine PCV?
151-Most common cause HTN in adult ?
152- What is more present in Cow's milk than breast milk?
proteins
fat
carbs
calories
Answer:A
153- routine breast self exam advice:
daily
weekly
Monthly
yearly
Answer: C
Not Advice !!
154-contraindication for breastfeeding ?
HCV
HIV
herpes zoster
wart
smle ,2016

877
Answer: B and could be C if HZ was in the nipple .
155-(long scenario) man traveled for business, went to his flat to relax, second
day he noticed mold on his flat, he developed rash all over his body. We took
sample of mold and injected him, inflammation occurs after 30 minutes. What
does it indicate?
Immediate hypersensitivity.
delayed hypersensitivity
Answer: A
156-pt develop cough during exercise :: which medication want to give her before
exercise? B2 agonist
157- 22 y/o lady health present to check up she only complain of breast tenderness 3
days before menses /she never had sexual contact and wish to be pregnant in next 2 year
what is the best thing to do for screening?
brest US
HPV
pap smear
colposcopy
Answer: ??
158-Pt with moderate persistent bronchial asthma on beta agonist PRN and low
dose inhaled steroid Comes with uncontrolled BA ,What you will add to the steroid?
A. Long acting beta agonist
B. Theophylline
Answer: A
159-what is the treatment for female pt with uncontrolled asthma ( wake her up from
sleeping at night ) she is on short beta 2 agonist ?
long acting beta 2
steroid
theophylline
Answer: B Next step managment INHALED
160-HTN came for follow up a1c 6.9 when to follow up again
1 months
3 months
12 months
Answer:
161-Treat HTN and BPH??
atenolol
prazosin
162-How can prevent dust mite ?

smle ,2016

878

163- what is the cause to use every year influenza vaccine ?


A- resistance of antimicrobial
B- new antigen
C different type of transmission
drift .. So could be new antigen
164- Difference between types 1 and 2 of DM?
Cpr
Level of insulin
Insulin growth factor
Islet of b cell
Answer: D
The process responsible for type 1 diabetes is destruction of insulin-secreting pancreatic islet cells.
This is manifested by a mononuclear infiltrate and beta-cell lysis in the islets. The underlying
defect that causes type 2 diabetes is insulin resistance.
Ref: http://www.medscape.com/viewarticle/449804
165- 80 years old man with history of 3 fractures, he is on vitamin D and ca, what to add?
Answer: Bisphosphonates
Medical care includes the administration of adequate calcium, vitamin D, and anti-osteoporotic
medication such as bisphosphonates,
Ref: http://emedicine.medscape.com/article/330598-treatment
166- What is the effect of polio (IPV& OPV) on body?
All lead to the formation ag in the anterior horn
All lead to the formation of the ab in the serum which fight the virus
they all enter the intestinal mucosa where the entry of the virus is
they all lead to the formation of interferon gamma
Answer: B
The inactivated polio vaccine produces antibodies in the blood to all three types of poliovirus. In
the event of infection, these antibodies prevent the spread of the virus to the central nervous system
and protect against paralysis. OPV produces antibodies in the blood to all three types of poliovirus.
In the event of infection, these antibodies protect against paralysis by preventing the spread of wild
poliovirus to the nervous system.
Ref:
http://www.polioeradication.org/Polioandprevention/Thevaccines/Inactivatedpoliovaccine(IPV).aspx
http://www.polioeradication.org/Polioandprevention/Thevaccines/Oralpoliovaccine(OPV).aspx
167-What cause infection by food with soil contaminated?
Ascaris
Tinae
schistosoma
Answer: A: Ascaris lumbricoides
Ascariasis is the most common helminthic infection, with an estimated worldwide prevalence of
25% (0.8-1.22 billion people).[1] Usually asymptomatic, ascariasis is most prevalent in children

smle ,2016

879
of tropical and developing countries, where they are perpetuated by contamination of soil by human feces or use of untreated feces as fertilizer
Ref: http://emedicine.medscape.com/article/212510-overview
168-Parasite transmitted by meat?
Trichinosis
Trichinellosis, also called trichinosis, is a disease that people can get by eating raw or undercooked
meat from animals infected with the microscopic parasite Trichinella.
Ref: http://www.cdc.gov/parasites/trichinellosis/
169-A 43 y/o female undergone for a mammogram, and the result was negative, when do you
advise her to get tested again:
After 1 year.
After 3 year. - After 5 year.
Other choices, I cannot remember.

170- Most common cause of hepatitis HBA

smle ,2016

880

171- Most common infectious disease in pre employment ex HBV


172- Best sport for cardiovascular pt?
https://www.acsm.org/docs/current-comments/exercise-for-persons-with-cardiovascular-disease.pdf
173- Best sport for old DM?
http://www.cdc.gov/diabetes/managing/beactive.html
174- You advice pt for walking:
Walk 1.5 km 4 days/ week
Do brisk walking 30 min 5 days
Reduce sodium intake
Give potassium
Answer: B
Ref: http://www.heart.org/HEARTORG/HealthyLiving/PhysicalActivity/FitnessBasics/American-Heart-Association-Recommendations-for-Physical-Activity-in-Adults_UCM_307976_Article.jsp#.V2jeL9IrIy4
175- At which temperature does the blood bank preserve the blood?
37 C
4C
22 C
Answer: B
Ref: http://www.medscape.com/viewarticle/583145_3
Whole Blood 4 C
smle ,2016

881
Platelet 22 C
177- What most commonly cause itching?
Bile salt retention
Eczema
Pregnancy
Answer: B
Pruritus, or itch, is most commonly associated with a primary skin disorder such as xerosis, atopic
dermatitis, urticaria, psoriasis, arthropod assault, mastocytosis, dermatitis herpetiformis, or pemphigoid.
Ref: http://emedicine.medscape.com/article/1098029-overview
178- Which of the following enhance non-heme iron absorption?
Coffee
Tea
Milk
Vitamin C*
Answer: D
Vitamin C also plays an important role in immune function and improves the absorption of nonheme iron.
Ref: http://emedicine.medscape.com/article/2088649-overview#a4
179- which of the following is prescribed with iron supplement for better absorption?
A. Vit D
B. Vit E
C. Vit C
D. Calcium
Answer C http://www.ncbi.nlm.nih.gov/pubmed/2507689
http://www.ncbi.nlm.nih.gov/pubmed/6940487
180-a man who eat rice only , which vitamin will be deficient?
a)B1
b)B2
c)B6
d)B12
answer D ,, I couldnt find any explanation
181-Which in lipid profile is most important risk for coronary heart disease ?
LDL,
HDL,
Triglyceride ,
total cholesterol
answer A ,, I couldnt find any explanation
167-they want to stop screening for breast cancer for women under 47 to decrease unnecessary anxiety to the public , you didn't agree cus there is gene that cause cancer in young
women , what is the gene ?

smle ,2016

882
BRCA2
Answer:A
Explanation : breast cancer in younger women can be aggressive and less likely to respond to
treatment. Women who are diagnosed at a younger age also are more likely to have a mutated
BRCA1 or BRCA2 gene.

ETHICS
1- If you successfully treated your patient. Then the patient brought to you an expensive
watch . What would be your response?
Accept the gift and say thank you
Refuse the gift politely
ask him for more gifts.
Shout at him and say "I do not accept gifts"
Answer: B ,,, I couldnt find any explanation
2- (long scenario) lady complaining about work partner. She is agitated and mad. What is
your action?
a.
Instruction to deal with him.
b.
Empathy during session.
c.
Avoid him.
Answer: ? question is not clear.
3- You have old patient who can't understand you. What will you do to get more information?
you will take him to your clinic and ask him open ended question
Answer: ? question is not clear.
4- (long scenario) lady complaining about work partner, she is agitated and mad. What is
your action ?
a.
Instruction to deal with him.
b.
Empathy during session.
c.
Avoid him.
d.
!!!!
Answer:? question is not clear.

7- You have a patient and you took her permission to examine her. What are you doing?

Taking informed consent

smle ,2016

883

Being efficient in you job


Answer : no enough information
8- tell the pt information about his disease and side effect of medication is kind of
A- establish rabbot
B- information
Answer: B not sure.
9-Pt came to ur clinic and upsit b/c u have been late wt to do apologized
10-tell the pt information about his disease and side effect of medication is kind of:
establish rabbot
information
answer : B not sure.
11- Whats the best approach in hx?
Open ended Q
Answer: A
12- Patient entered the clinic, then the physician took the history from him, after that the
physician told the patient "Would you please let me examine you". This sentence is under
category of?
Taking Informed Consent
Respect the patient
Answer: A
similar Q in 7th update p. 50713- Doctor informed pt about his disease, prognosis ,medications , & side effect, what do
you call this form of talk?
Improve communication
Patient Doctor relationships
instructional communication?
Davo.. something
Educational care
Answer: C not sure
.U take hx from the pt & u want to examine him & u want to till him . What's this?!
-Informed consent
-Respect pt

smle ,2016

884

Community Medicine &


Research

smle ,2016

885

smle ,2016

886

smle ,2016

887

smle ,2016

888

http://www.ncbi.nlm.nih.gov/pmc/articles/PMC2998589/#!po=20.9677

smle ,2016

889

smle ,2016

890
1- Important equations:
Prevalence of Disease = People who have the disease / Total 100
Sensitivity = A/(A+C) 100
Specificity = D/(D+B) 100
Positive Predictive Value = A/(A+B) 100
Negative Predictive Value = D/(D+C) 100
Control event rate (CER) = c/c+d
Experimental event rate (EER) = a/a+b
(a) Relative Risk (RR) = EER/CER=(a/a+b)/(c/c+d)
(b) Relative Risk Reduction (RRR) = CER-EER/CER
(commonest reported measure of dichotomous treatment effect)
(c) Absolute Risk Reduction (ARR) = CER-EER
(d) Number Needed to Treat (NNT) = 1/ARR
A certain risk reduction may appear impressive but how many patients would you have
to treat before seeing a benefit? This concept is called "number need to treat" and is
one of the most intuitive statistics for clinical practice.
For example if:
Yes
No
Exposed
8(a)
992(b)
Not Exposed 10(c)
990(d)
The RR = (8/1000) / (10/1000) = 0.8 making the RRR = (1-0.8/1)=0.2 or 20%. Although this
sounds impressive, the absolute risk reduction is only 0.01-0.008=.002 or 0.2%. Thus the NNT is
1/0.002=500 patients. It is obvious that on an individual patient basis the pre-intervention risk or
probability is a major determinant of the degree of possible post-intervention benefit, yield, or risk
reduction.
2- Measure of Variability
- Range; the difference between highest and lowest score (for small data)
- Variance: the degree of spread within distribution (stable measure)
- Standard deviation: measure how the average score deviated away from the mean (most stable
measure)
3- Definition of epidemiology
The study of the distribution and determinants of health related events (including diseases and
application of this study to control of diseases and the others health problems) (Om al Qura, family
medicine )
4- Definition of endemic or epidemic disease
https://www.vocabulary.com/articles/chooseyourwords/endemic-epidemic/
Endemic refers to the constant presence and/or usual prevalence of a disease or infectious agent in
a population within a geographic area.
Hyperendemic refers to persistent, high levels of disease occurrence.
Occasionally, the amount of disease in a community rises above the expected level.
Epidemic refers to an increase, often sudden, in the number of cases of a disease above what is
normally expected in that population in that area.
http://www.cdc.gov/ophss/csels/dsepd/ss1978/lesson1/section11.html

smle ,2016

891

5- Definition of epidemic curve?


An epidemic curve gives a graphical display of the numbers of incident cases in an outbreak or
epidemic, plotted over time.
http://www.med.uottawa.ca/sim/data/Public_Health_Epidemic_Curves_e.htm
6- Definition of evidence based medicine (biosocial .)
Practice according available scientific evidence
Is about trying to improve the quality of the information on which decisions are based. It helps
practitioners to avoid information overload but, at the same time, to find and apply the most
useful information
Evidence-based medicine (EBM) is the integration of best research evidence with clinical expertise
and patient values.
http://ktclearinghouse.ca/cebm/intro/whatisebm
7- Cross sectional: observational study that assesses risk factor and outcome at snapshot in time
also called prevalence study
8- Best sentence describe case-control study: (Read about it )
A study that compares patients who have a disease or outcome of interest with patients who do not
have the disease or outcome, and looks back retrospectively to compare how frequently the exposure to a risk factor is present in each group to determine the relationship between the risk factor
and the disease
One of the most significant triumphs of the case-control study was the demonstration of the link
between tobacco smoking and lung cancer
observational study that assesses risk factor and outcome at snapshot in time also called prevalence study
9- Definition of case report research
Retrospective analysis of one (1), two (2), or three (3) clinical cases
http://viceprovost.tufts.edu/HSCIRB/case-reports/
10- What's mean by Standard Deviation?
The standard deviation is the measure of spread used most commonly with the arithmetic mean.
http://www.cdc.gov/ophss/csels/dsepd/ss1978/lesson2/section7.html
Measure that is used to quantify the amount of variation or dispersion of a set of data values. A
standard deviation close to 0 indicates that the data points tend to be very close to the mean (also
called the expected value) of the set, while a high standard deviation indicates that the data
points are spread out over a wider range of values.
Positive square root of variance
11- Why we use standard deviation?
For Measurement (UQU)
It is a measure that used to quantify the amount of variation or dispersion of a set of data values
12- Definition of specificity

smle ,2016

892
Specificity is the probability that an individual without the disease will test negative. It is the number of patients who have a negative test and do not have the disease (true negatives) divided by the
number of patients who do not have the disease. A test with high specificity will infrequently
identify patients as having a disease when they do not (ie, low false positive results). Uptodate
13- Definition of Positive predictive value & Negative predictive value
PPV: is the probability that subjects with a + screening test truly have the disease.
NPV: is the probability that subjects with a - screening test truly don't have the disease.
http://sphweb.bumc.bu.edu/otlt/MPH-Modules/EP/EP713_Screening/EP713_Screening5.html
14- What is the definition of Attributable risk?
A- Difference in rate of a condition between an exposed population and an unexposed population.
Answer: A [100% in research]
AR = the amount of disease incidence that can be attributed to a specific exposure
Difference in incidence of disease between exposed and non-exposed individuals
Incidence in non-exposed = background risk
Amount of risk that can be prevented
http://www.pitt.edu/~super7/30011-31001/30101-30111.ppt
It is a measure of the public health impact of a causative factor. The calculation of this measure
assumes that the occurrence of disease in the unexposed group represents the baseline or expected
risk for that disease.
http://www.cdc.gov/ophss/csels/dsepd/ss1978/lesson3/section6.html
15- Calculate relative risk:
a/ ( a+b ) / c ( c+d )
16- Research questions about:
Prevalence,
Most accurate test etc
Formula of relative risk,
Calculate and define the annual prevalence rate
Cohort study advantage
Definition of positive probability test
association ratio
relative rate or negative predictive value
Determine the range
17- Cohort study, calculate the Relative Risk.
Diseased

Healthy

Exposed

Not exposed

Relative Risk = (A / (A+B)) / (C / (C+D)).


smle ,2016

893

18- Case-control study, calculate the odds ratio.


Odds ratio: (A/B)/ (C/D) or A*D/ B*C

19- You conducted a study in which a group of epileptic patients using carbamazepine for 10
years. Now you compare them with their age equivalent healthy individuals. What is this
type of study called?
A- Case control study
B- Retrospective cohort study
C- Cross sectional
D- Prospective cohort study
Answer: D
Cohort study: compares a group with exposure (carbamazepine in this Q) to a group without
such exposure. Done in two ways; either Prospective (you give the exposure and follow the subjects for a specific time) or retrospective (the exposure occurred already in the past, you look
back in the history).
Case control study: compares a group with disease to a group without disease.
20- In a study they are selecting every 10th family in the city, what is the type of study
A- Systematic study
B- Stratified study
C- Non randomized study
Answer: A
Random sampling: preferred way of sampling, it is often difficult to do. It requires that a complete
list of every element in the population be obtained. Computer generated lists are often used with
random sampling.
Systematic sampling: every kth element is taken. This is similar to lining everyone up and numbering off "1,2,3,4; 1,2,3,4; etc". When done numbering, all people numbered 4 would be used.
Convenience sampling: readily available data is used. That is, the first people the surveyor run
into.
Cluster sampling: accomplished by dividing the population into groups called clusters - usually
geographically. The clusters are randomly selected, and each element in the selected clusters is
used.

smle ,2016

894
Stratified sampling: divides the population into groups called strata. For instance, the population
might be separated into males and females. A sample is taken from each of these strata using either
random, systematic, or convenience sampling.

21- Non response bias. What is this study?


A- Cross sectional.
B- Cohort.
C- Case-control
Answer: A
Non response bias (example of selection bias) occurs with cross sectional study.
22- Epidemiological study want to see the effect of smoking in lung cancer: 90% of lung
cancer patients are smokers while 30% of those who dont have the disease are smokers. The
specificity of smoking as a risk factor is:
A- 10%
B- 40%
C- 30%
D- 70%
E- 90%
Answer: D (100% in Research)
Explanation: Specificity = 70/ (30+70) = 70%
Lung Cancer

No Lung Cancer

Smoker

A (90)

B (30)

Non smoker

C (10)

D (70)

23- At a daycare center 10 out of 50 had red eye in the first week; another 30 developed the
same condition in the next 2 weeks. What is the attack rate (cumulative incidence)?
A- 40%
B- 60%
C- 80%
D- 20%
Answer: C
Attack Rate = cases due to a specific cause in a short period of time divided by the population at
risk, often associated with an epidemic situation such as food borne disease (cumulative incidence) Ref: 3rd Edition UQU
24- 80 persons found to have Typhoid Fever in a population of 20,000 during the last 4
years. If we measure it on a population of 100,000, what will be the incidence in one year?
A- 20
B- 100
smle ,2016

895
C- 150
D- 200
Answer: B
Incidence in a population of 20,000 for 1 year is 20/20,000.
Therefore, the incidence in a population of 100,000 for 1 year is 100/100,000
25- With table of sensitivity, specificity, positive and negative predictive values
Answer
Sensitivity: the probability that a diseased pt will have a positive test result
Specificity: the probability that a non-diseased pt will have a negative test result
See the table:

Source :UQU
26- In a cohort study on lubricant oil use and urinary bladder CA done over 20 years 10,000
exposed 10,000 non exposed 750 exposed got CA 150 non exposed got CA Then they asked
about the incidence in 1000 in one year?
A- 2.25
B- 45
C- .45
D- .225
Answer: A
In 20 y 20,000 p 900 Cases, so in one year 900/20=45 case for 20,000 population for 1000,
45/20=2.25
27- What is the incidence rate for 10000 population, 2000 old cases, and 1000 new cases?
12.5
1000/8000=.125 >.125*100=12.5

smle ,2016

896
28- Cohort study 2 groups: one of them exposed to patient with positive MERSA and the
other group is not exposed to these patients.
-ve MERSA

+ve MERSA

Total

Exposed

20

80

100

Non-exposed

394

400

414

86

500

What is the ratio of exposed to non-exposed?


A- 44:1
B- 55:1
C- 66:1
Answer: C
29- What is the risk of exposure to MERSA-virus ?
A-33
B-44
C-55
D-66
Answer: D
(80100)(5006)= 66
30- Question asking about the relative risk for exposure and none exposure to something.
The numbers are 80 out of 1000 in exposure. 6 out 5000 in non exposure.
A- 66:1 [100% in research]
31- Lung cancer affected 80 of 100 smokers, 6 of 500 non smokers, What is Relative risk
reduction?
A- 33:1
B- 55:1
C- 66:1
Answer: C
RR= [cases in exposed/total exposed]/[cases in non exposed/total non exposed]
=[80/100]/[6/500]= 66.6666
32- A study with some people exposed to MRSA patient and a control group who didnt calculate the risk or the ratio ,,, I DONT KNOW !!(( i think it was relative risk))
-ve MRSA +ve MRSA Total
Contact

smle ,2016

20

80

100

897

No Contact

494

500

514

86

600

Answer: relative risk ( a/(a+b) ) / (c/(c+d) ) 20/(20+80) / 494/(494+6) =0.809


33- It's found that patients on insulin something have a higher incidence of something compared to people who are taking another type of insulin Type one insulin event rate is 0.092
Type two insulin event rate is 0.022 Which one of the following is correct:
Peep is 0.07 Odd ratio something cases to harm is 8 Something cases to.... Is 14
Answer: ?
Control event rate (CER) = c/c+d
Experimental event rate (EER) = a/a+b
(a) Relative Risk (RR) = EER/CER=(a/a+b)/(c/c+d)
(b) Relative Risk Reduction (RRR) = CER-EER/CER
(commonest reported measure of dichotomous treatment effect)
(c) Absolute Risk Reduction (ARR) = CER-EER
(d) Number Needed to Treat (NNT) = 1/ARR
A certain risk reduction may appear impressive but how many patients would you have
to treat before seeing a benefit? This concept is called "number need to treat" and is
one of the most intuitive statistics for clinical practice.
For example if:
Yes
No
Exposed
8(a)
992(b)
Not Exposed 10(c)
990(d)
The RR = (8/1000) / (10/1000) = 0.8 making the RRR = (1-0.8/1)=0.2 or 20%. Although this
sounds impressive, the absolute risk reduction is only 0.01-0.008=.002 or 0.2%. Thus the NNT is
1/0.002=500 patients. It is obvious that on an individual patient basis the pre-intervention risk or
probability is a major determinant of the degree of possible post-intervention benefit, yield, or risk
reduction.
34- Study of 1000 with congenital heart ,20 have gestational diabetes And control group of
5000 no congenital, 80 have gestational diabetes What is odd ratio and RR
http://www.graphpad.com/guides/prism/6/statistics/index.htm?stat_interpreting_results_contingen.htm

Diabetes

Congenital
Non-ConHeart Disease genital
(cases)
(control)

Total

20

80

100

4920

5900

No Diabetes 980

smle ,2016

898

1000

5000

6000

Answer: OR= (20/980) / 80/4930) = 1.255


RR = (20/1000)/(80/5000) = 1.25
35- Study about GDM association with malformation. 2 groups. 5000 control group, 50
developed. 1000 diseased group 20 developed.(not sure about the numbers) The best for
risk factor association? (and calculate)
A- Relative risk =2
B- Odds ratio=2
C- RR= ?
D- OR =?
Answer: B (By community specialist)
OR used for case control study, OR = ( 20*4950 ) / ( 50*980 ) = 2
36- There is Q about cumulative incidence " I'm not sure about the Q but I'm sure about
the numbers" the new case if some disease for 2012 is 200 and the already existed disease
is 80 out of population 8,000,000" what is the CI for 100,000:
Answer: 3.5
New cases 200 Old cases 80 Total 280 So, (280/8000000)* 100000= 3.5
37- Case control study showing this information: Odd ratio .75, Control rate .05 whats correct:
A- Number needed to tx is 69
B- Number needed to harm 69
C- Relative risk is .12
Answer: C
38- Study compared effect of medication on baby whose mothers were/were not taking the
medication while pregnant and followed them till birth and after developing, type of the
study?
A- Cohort
B- Cross sectional
C- History cohort
Answer: A
39- A research about a certain disease, on 10,000 populations, in begging there were 2000 pt
but after 3 years of the study there were another 1000 pt, what is the incidence in one year?
Answer: 10%
40- What is the following of these is true, specificity ,sensitivity ,Positive predictive value and
Negative predictive value? 60 940 1000
The true answer specificity is 940/1000 (=94%) The other choices are wrong answer equation of
sensitivity, PPV, NPV.

smle ,2016

899
41- The number of URTI cases per 1000 population this year is 117 case. The average
number of disability from work is 2.5 days. What is prevalence of disability per 1000 ?
A- 292.5 [100% in research]
42- Incidence is 117 per 1000 per year. Median disability is 2.5 Median lost jobs is 1.6, what
is the prevalence of disability?
2900.5
43- What most determine accuracy of a study?
A- Specificity
B- Sensitivity
C- Positive predictive value
D- Negative predictive value
Answer: C
44- What type of study is used to comparing 2 groups of pregnant ladies on a specific diet,
outcome of pregnancy?
A- Cohort study
B- Cross sectional
C- Historical cohort
Answer: A , (its follow up study Cohort)
45- Epidemic investigation (study) what is the first step?
A- Identifying population at risk
B- Count the cases
C- Confirm Dx
Answer: A , (the most likely answer regarding the given information, otherwise check this
out : http://www.cdc.gov/ophss/csels/dsepd/ss1978/lesson6/section2.html
46- Risk of un-exposed population 2, risk of exposed population 3 , what is true ;
A- Number to be treated 20
B- Patient event risk 2
C- Number to be harm..
Answer : ARR = Risk1 - Risk 2 = 3-2 = 1 , NNT = 1/ARR = 1
47- You are reading a population study that states that 90% of lung cancer patient are smokers while 30% of lung cancer patient are non-smokers. What is the specificity of using smoking as a predictor of lung cancer?
A- 30
B- 70 + (umm alqura)

48- Bladder cancer Cohort study done, 10,000 exposed to risk factor, 750 develop disease,
10,000 not exposed, 150 develop disease. What is the incidence?
A- 0.2
B- 2

smle ,2016

900
C- 4
D- 16
Incidence among exposed = 750/10,000 = 0.075 , Incidence among nonexposed = 150/10,000
0.015 , Relative Risk = 0.075 / 0.015 = 5
49-There is a study to the effect on intensive insulin regimen in the reduction of neuropathy
in the patient, the results are as following:
Event in the regular insulin regimen:0.092
Event in the intensive insulin regimen:0.022
Which of the following is correct?
A- Patient expected event rate:0.07
B- Odd ratio is 4.2
C- Number needed to harm:7
D- Number needed to treat:12
50- Study we have 1000 DM 20 of them have congenital abnormal babies and 5000 control
of non DM have 50 congenital abnormal babies... What determine accuracy of the study?
A- OR 2.0
B- RR 2.0
C- OR 0.2
D- RR 0.2
51- 2 groups, diseased and non,, exposed to smoking and non ..
A- OR = 5
B- RR =5
C- OR= .5
D- RR = .5
Answer :D
52- Incidence of respiratory infection: 117 in 1000.
A- 2.5
B- 1.2
Answer : B
53- You have devised a new test to diagnose a disease and you want to check it against a gold
standard test. There were figures showing the number of those who actually have the disease
according
to the gold standard test and the number of those who were tested positive with your test.
Also figures showing the number of true negatives with your test. What is the specificity of
your new test?
A- 40%
B- 50%
smle ,2016

901
C- 60%
D- 80%
54- Exposed odds ratio of 3 and non exposed odds ratio 2. Which of the following is true?
A- Number needed to treat is ...
B- Number needed to harm is ...
C- Relative risk is ...
55- In a cohort study to study the association of some kind of lubricant and skin cancer, the
unexposed risk: 2, the exposed risk: 3, what can you calculate? Giving whats mentioned
above
56- Case control study between lung cancer and smoking, Non smoker 1, Low smoker 1.3,
Moderate 3, Heavy 5, What is the causation of the study?
57- Risk of infection among a population in a country in 2012, new cases 200, leave the country I think 12, died I think 20, total population 80,000,000, what is the risk in 100,000?
58- Study of relation of the bladder cancer and lubricating oil use, they take 10,000 male who
use the oil and 10,000 who don't, and follow them for 20 years. From the one who use the
lubrication oil, 750 developed cancer, and 150 who don't use the oil developed the cancer,
calculate the incidence per 1000 per one year
59- Patients with a specific disease in a thousand. 2.5 days are disability and 1.5 absent workers calculate the incidence of disability annually
60- 70% of the population with lung cancer are smokers. And 30% of the population without
lung cancer are smokers? What is the incidence and prevalence predicting the lung cancer
in relation to smoking ?

smle ,2016

902

Psychiatry

smle ,2016

903

1- Obese boy is teased by his friends tells you that he wants to take pills and never wake up. What's
your next step?
a.
Immediate psychiatry referral
Answer: A
2- What is the treatment of generalized anxiety disorder?
a.
Escitalopram
Answer: A
lifestyle: caffeine and EtOH avoidance, sleep hygiene
psychological: CBT including relaxation techniques, mindfulness
biological
SSRIs and SNRIs are 1st line (paroxetine, escitalopram, sertraline, venlfaxine XL)
2nd line: bupropion (caution due to stimulating effects ), buspirone (tid dosing)
add-on benzodiazepines (short term, low dose, regular schedule, long half-life, avoid prn)
Reference: Toronto Notes
3- Elderly patient his wife died recently. Hes been depressed and having difficulty sleeping since that
time. Which of the following is a short term medication that is suitable for him?
a.
Diazepam
Answer: A
Long acting benzodiazepines, with active metabolites should usually be avoided in the elderly e.g. diazepam, chlordiazepoxide, flurazepam, nitrazepam. When benzodiazepines are prescribed in older people,
short acting benzodiazepines with few active metabolites are preferable e.g. lorazepam 0.5mg bd/tid,
temazepam 10-20mg nocte, alprazolam 0.25mg bd/tid. Reference: http://www.svhf.ie/documents/BenzoGoodPracticeGuide.pdf
4- Why SSRI is the drug of choice for depression?
More tolerable and less side effects
Cheap
Answer: A
Reference: http://www.bpac.org.nz/BPJ/2012/december/docs/bpj_49_nzf_pages_34-35.pdf
5- A guy who is showing an erratic behavior lately. He is drinking a lot, spending too much money and
having involved in sexual activities a lot. What is your diagnosis?
drinking problem
mania
drug use
schizophrenia
Answer: B
Reference: Toronto Notes PS 10
6- An alcoholic comes to you with symptoms of alcohol withdrawal. Last drink he consumed was 2
days back. What drug will you give?
Naloxone
Diazepam
Nicotine

smle ,2016

904
Disulfiram
Answer: B
diazepam 10-20 mg IV/PO or lorazepam 2-4 mg IV/PO q1hr until calm
thiamine 100 mg IM/IV then 50-100 mg/d
magnesium sulfate 4 g IV over 1-2 h (if hypomagnesemic)
admit patients with delirium tremens (DT), or multiple seizures.
Reference: Toronto Notes.
7- What is the best factor for smoking cessation?
The patient desire.
Answer: A
8- 50 Female presented with depression and she was given amitriptyline 30 mg (TCA). Later she came
complaining of dizziness. What will you do? (Similar Q: A patient with depression on TCA came with
dizziness. How will you manage?)
Decrease dose to 10 mg
Switch to SSRI
Answer: B
9- A depressed patient on medication, but he had weight gain and erectile dysfunction. What is the
medication responsible for these side effects?
Sertraline.
Venlafaxine
TCA.
Answer: A
Sertraline is an antidepressant in a group of drugs called selective serotonin reuptake inhibitors (SSRIs)
and most common side effects is decreased sexual desire or ability, failure to discharge semen (in men)
and weight gain.
10- Old man, lives in a nursery home, missed his wife recently. Also he had heart failure and osteoarthritis recently which makes him more depressed. He is sad and having low mood. He doesnt eat that
lead to loss of 4 kgs of his weight in the last month. Sometimes he forgets. What is the most likely diagnosis?
Alzheimers disease
Depression
Answer: B
11- What is an alternative medication for severe depression?
triptyline
psychotherapy
electroconvulsive therapy
Answer A
(TCA), if you find RCT in the answers then choose it.
Reference: Toronto Notes
12- A patient moved to a new schoolWhat is the most likely diagnosis?
Adjustment syndrome
Answer: A

smle ,2016

905

13- Which of the following is considered good prognostic factor for schizophrenia.
No identified cause
Family history
Insidious onset
Affective symptoms
Answer: D
Good Prognostic Factors:
Acute onset, later age at onset, shorter duration of prodrome, female gender, good cognitive functioning, good premorbid functioning, no family history, presence of affective symptoms, absence of structural brain abnormalities, good response to drugs, good support system. Reference: Toronto Notes
14- A female patient says her newborn is evil and wont live long. What is your diagnosis?
Postpartum psychosis
Answer: A
Postpartum psychosis (or puerperal psychosis) It is a severe episode of mental illness which begins suddenly in the days or weeks after having a baby. Symptoms vary and can change rapidly. They can include
high mood (mania), depression, confusion, hallucinations and delusions. Postpartum psychosis is a psychiatric emergency, the patient needs hospitalization
15- Acute onset of disorientation, change level of conscious, decrease of concentration, tremor, he
mention that he saw monkey! He was well before. What's the diagnosis:
a) Parkinson dementia
b) Schizo
c) Delirium
d) Delusion disorder
Answer: C
16- Difference between delirium & dementia?
Impaired memory
B. state of consciousness
C. confusion
Answer: B

smle ,2016

906

Reference: Toronto Notes


deleruim, treatment
Answer:
- primary illness should be mainly focused
-nurse in well lit room
- make him comfortable
- less number of attendants
- less changes in staff
- lower doses of Risperidone 1-2 mg in two or three divided doses
- some sedative like zolpidem 10 mg at bed time
http://emedicine.medscape.com/article/288890-treatment
17- Clozapine is used in which disease in children
Bipolar
Depression
C. Substance abuse
D. Psychosis ( Or schizophrenia )
Answer: D
Reference: http://www.jaacap.com/article/S0890-8567%2809%2961687-7/abstract

smle ,2016

907

18- A patient presented with behavioral changes. His father died recently ( 3 days ). He is walking
around naked and saying that his father asked him to do that then returns to his normal state. What is
the most likely diagnosis?
a.
Brief psychotic disorder
b.
Schizophrenia
c.
Schizophreniform
d.
Schizoaffective
Answer: A
19- Patient turns to be erratic for 4 months, he said that people on TV know what hes thinking about
and they are talking about him. In the last 2 months he started to claim that he has special power that
no one has. What is the most likely diagnosis?
a.
Uni-polar...depression
b.
Bipolar...Mania
c.
Schizophrenia
Answer: C. or Schizophreniform
Reference: 3rd Edition UQU > Psychiatry > Q 30 and Toronto Notes
20- A man is behaving in a strange way after the death of his son. random people
on the road and ask them irrelevant questions. He wont listen when asked not to
do that. He had no such behavior before in life. Which of the following will be excluded from your differentials?
Brief psychotic disorder
Schizophrenia
Schizophreniform
Schizoaffective
Answer: A
21- A guy who is heroin addict. You want to start rehabilitation. What drug will you prescribe?
Diazepam
Methadone
Answer: B
Withdrawal: long-acting oral opioids (methadone, buprenorphine), -adrenergic agonists (clonidine)
Treatment of Chronic Abuse:
psychosocial treatment (e.g. Narcotics Anonymous) usually emphasize total abstinence
naltrexone or naloxone (opioid antagonists) may also be used to extinguish drug-seeking behavior
long-term treatment may include withdrawal maintenance treatment with methadone or buprenorphine
Reference: Toronto Notes.
22- Patient came to you for check up and tells you that he diagnosed two years ago with pancreatic
cancer and asking you for the medication but he lost the prescriptions, when you have looked in the
system there wasn't any data about him. When you face him with what you find, he runs away.
What is the diagnosis?
a.
Malingering.
b.
Somatization.
c.
Schizophrenia

smle ,2016

908
d. Drug addict.
Stigmatization is symptom like stories
Answer: D
23- Old patient with Alzheimer's dementia, became agitated and have hallucinations and delusions.
What is the appropriate drug in his case ?
a.
Haloperidol.
Answer: A
Haloperidol: is a typical antipsychotic medication.It is used in the treatment of schizophrenia, tics in
Tourette syndrome, nausea and vomiting, delirium, agitation, acute psychosis, and hallucinations .
24- Which one of the antipsychotic drugs is most likely to cause weight gain ?
a.
Ziprasidone.
b.
Olanzapine.
c.
Quetiapine.
d.
Aripiprazole.
Answer: B however ( B,C,D causes weight gain but more common in 40% is B )
25- Old man is walking on the street and asking random people the same question. When asked to
stop he doesnt stop. What does he have?
Loose of association
Delusion of control
Answer: DEMENTIA
26- A patient was brought by his family with 3 months history of delusions, hallucination, that are
gone by themselves now. No mood disorders. What is the diagnosis?
Brief Psychotic disorder
Schizophrenia
Schinziform disorder
Answer: C
27- Which of the following is a criteria of Somatization disorder?
intentionally symptoms
at least 2 GI symptoms must be present
more than 40 years old
loss of occupational / life function
Answer: B
8 physical symptoms that have no organic pathology including each of:
four pain symptoms related to at least four di erent sites or functions
two gastrointestinal symptoms, not including pain
one sexual symptom, not including pain
one pseudo-neurological symptom, not including pain (e.g. numbness, paresthesia)
onset before age 30; extends over a period of years.
Reference: Toronto Notes

smle ,2016

909
28- cases of obsessive compulsive disorder one of them was excessive washing of hand around 40
time , the other about wife complain that her husband is checking the door locked or not alot of times
10*pt with paresthesia , abdominal pain nausea Vomiting . For 2 years
A- somatization disorder
B- conversion
C- hypochondriasis
Answer: A
29- Pt think that he has cancer and did test to confirm he has no cancer and pt said it is significantly
affect his life:
Factitous disorder
OCD
hypochondriasis
Answer: C
30- about male pt he cover the TV because he thinks that he got instruction from the TV and the government Watching him
Mania
Depression
schizophrenia
Answer: C
31- 20s Female patient c/o Multiple GI symptoms such as Nausea, vomiting, diarrhoea, headache, fatigue, joint pain, urine retention. The all lab result is normal. These symptoms for 2 year unchanged:
A. Somatisation disorder
B. Conversion disorder
C. GAD
D. Dissociative disorder
Answer: A
Somatisation disorder:
The essential feature of a somatization disorder is several symptoms that not lead to any medical sense,
with a pattern of many physical complaints in persons younger than 30 years that occurs over several
years and results in unnecessary medical treatment and/or causes significant impairment in functioning.
All the following historical criteria are required for a diagnosis
*Four different pain sites (eg, head, abdomen, back, joints, extremities, chest, rectum) or painful functions (eg, menstruation, sexual intercourse, urination)
*Two gastrointestinal symptoms other than pain (eg, nausea, bloating, vomiting, or intolerance of several different foods)
*One sexual or reproductive symptom other than pain (eg, erectile or ejaculatory dysfunction, irregular
menses, excessive menstrual bleeding)
*One pseudoneurological symptom (eg, impaired balance, paralysis, aphonia, urinary retention)
Reference: Medscape http://emedicine.medscape.com/article/918628-overview#a2
32- The doctor asked the patient t: "do you think you are mentally ill?"
Its evaluating:
A. Insight

smle ,2016

910
B. Judgment
Answer: A
Judgment is an assessment of real life problem-solving skills
Testing: What should you do if you find a stamped (not canceled) and addressed envelope?
Insight is an understanding of their condition (including abnormal thoughts)
Testing: Tell me about your illness and if it is worse now? What medications are you taking?
33- Which one is known to cause restlessness, insomnia and drowsiness ?
A. SSRI
B. MAOI
C. Tetracycline antidepressants
D. Tricycle antidepressants
Answer: A
The trick of this question is recall all drugs side effects
Which all of this can cause restlessness and drowsiness but SSRI cause insomnia
Reference: http://www.drugwatch.com/ssri/
http://www.medicinenet.com/mao_inhibitors-oral/page2.htm#what_are_the_side_effects_of_MAOIs
http://www.mayoclinic.org/diseases-conditions/depression/in-depth/antidepressants/art-20046983
34- Middle age male presented in the ER with irritability, suspiciousness, overactive, and poor hygiene
what is the diagnosis:
A. Schizophrenia
B. Bipolar Disorder
C. Psychoses
D. Acute anxiety state
Answer: I'm not sure: C, We cannot judge by these symptoms ONLY its Schez or Bipolar
35- Cardiac patient with depression on regular medications, started to have convulsion then coma , He
took overdose of some pills which his relative does not know it, What is the cause of this seizure?
A.SSRI
B. Digioxin
C. quinine
Answer: C
36- 72 y male disoriented and hallucinating and disorganized thinking had aorto popletial graft and symptom fluctuates in the 2 days what the cause ?
A.multi infraction demntia
B.mania
C .demensia
D . delirium
Answer: D
37- case of poor hygiene in and hallucinating, what medication to give ?
Answer : -antipsychotic
antipsychotic medications used to treat hallucinations include haloperidol, olanzapine and risperidone.
http://www.news-medical.net/health/Hallucination-Treatments.aspx

smle ,2016

911
38- Pt loss of interest, sucidal attempt?
A.Major depression
B.Minor depression
Answer : A

39- Mania ?
A. Bipolar
B.affective disorder
C.Schizophrenia
Answer : A
It depend on the symptoms, the question is incompleted. To be more specific its Bipolar type I which is
include at least 1 episode of Manic episode (DSM-5 Page 126). Affective disorder is including depression,
bipolar disorder, and anxiety disorder.
40- case about addict person and ask which of the following quiestion is include in criteria of CAGE
questionnaire ?
Answer:
CAGE is a questionnaire indicated in alcoholism
Have you ever felt you needed to Cut down on your drinking?
Have people Annoyed you by criticizing your drinking?
Have you ever felt Guilty about drinking?
Have you ever felt you needed a drink first thing in the morning (Eye-opener) to steady your nerves or
to get rid of a hangover?
41- psychiatric patient said Tv he talk to me ....
Answer : delusion of reference
42- delusional disorder, what is it and what is its treatment:
Answer:
Delusional disorder is an illness characterized by at least 1 month of delusions but no other psychotic
symptoms
http://emedicine.medscape.com/article/292991-overview#a6
43- Pts talk about false belief and against culture . What dx;
a.ellosionb.delusion
c.schsoid
d.hallucinations
Answer: B
A delusion is a belief that is firmly held on inadequate grounds that is not affected by rational arguments
or evidence to contrary and that is not a conventional belief that a person might be expected to hold
given their educational, cultural and religious back ground
44- tttx of dementia?
A.edrophonium
B.revistagmine

smle ,2016

912
C.Neostigmine
Answer: B
Neostigmine is a cholinesterase inhibitor indicated in Symptomatic control of myasthenia gravis; antidote for nondepolarizing neuromuscular blocking agents after surgery; prevention and treatment of
postoperative distention and urinary retention edrophonium is indicated in myasthenia gravis and respiratory depression Rivastigmine is a cholinesterase inhibitor. It works by increasing the amount of a
certain substance (acetylcholine) in the brain, which may help reduce symptoms of dementia in patients
with Alzheimer disease.
http://www.rxlist.com/enlon-drug/indications-dosage.htm

45- Pt came with depression what is your assessment :


Answer: suicide
http://www.nimh.nih.gov/health/topics/depression/index.shtml

46- A 24 yo female athlete with body mass index (BMI) of 25 presented with c/o self-induced vomiting
after episodes of heavy eating. Physical examination reveals bilateral parotid enlargement and dental
erosion. Which of the following is the most likely diagnosis?
A. Anorexia Nervosa
B. Depression
C. Body dysmorphic disorder
D. Bulimia Nervosa
Answer: D
https://www.nationaleatingdisorders.org/bulimia-nervosa

47- Girl with BMI=16 says I'm fat.


Depression
Anorexia bulimia
Anorexia nervosa
Answer: C
48- Pt with manifestation of anorexia nervosa what's the lab asso e it?!!
- Increased liver enzyme
low: WBC, PLT, Na, K, phosphate, hypoglycemia, T3
normal: HGB, ESR, RFT
high: BUN, LFT, cholesterol
Hypokalemic hypochloremic metabolic alkalosis: Observed with vomiting
Acidosis: Observed in cases of laxative abuse
49- Whats the approach to a 40 yo lady with anxiety in work place because of conflicting with colleagues?
Educate about how to deal with ppl? Empathy?
Answer: Check if she is psychotic or not first.

smle ,2016

913
50- Guy treated with haloperidol comes with generalized rigidity with up rolling of eyes.
Acute dystonia?
Answer:Acute dystonia
Dystonic reactions are reversible extrapyramidal effects that can occur after administration of a neuroleptic drug. Symptoms may begin immediately or can be delayed hours to days. Although a wide variety
of medications can elicit symptoms, the typical antipsychotics are most often responsible.They reportedly arise from a drug-induced alteration of dopaminergic-cholinergic balance in the nigrostriatum (ie,
basal ganglia).
http://emedicine.medscape.com/article/814632-overview#a4
51- Poor schizophrenia:
- family history
- acute onset
- adolescence
- mood included in his history
answer: A
Factors Which May Indicate a Poor Prognosis
Earlier age of onset;
Being a male;
A higher number of negative symptoms;
A family history of schizophrenia;
A low level of functioning prior to onset;
Poor or no support system;
A history of substance abuse
52- A patient had weight gain, because she cant taste the food or smell. Examination is normal, she
was seen by neuro and psychiatry with no diagnosis. Whats her diagnosis?
A. Meningioma
B. Anurysm
C. Malingering
Answer : C
malingering as the intentional production of false or grossly exaggerated physical or psychological problems. Motivation for malingering is usually external (e.g., avoiding military duty or work, obtaining financial compensation, evading criminal prosecution, or obtaining drugs).
http://emedicine.medscape.com/article/293206-overview
53- Patient with irritably, has delusion and auditory hallucination, now he have flights of idea:
a- neurosis
b- psychosis
c- dissociative disorder
Answer: B
54- Man feeling depressed for 3 months, he is fighting with 2 of employee in job, 4 months ago he became the manager in his job:
A- Depression
B- adjustment
Answer: B

smle ,2016

914
Adjustment disorder is a stress-related, short-term, nonpsychotic disturbance. The discomfort, distress,
turmoil, and anguish to the patient are significant
55- Patient turns to be erratic for 4 months, he said that people on TV know what he's thinking about
and they are talking about him. In the last 2 months he started to claim that he has special power that
no one has. What is the most likely diagnosis?
A-Uni-polar...depression
B-Bipolar...Mania
C-Schizophrenia
Answer: Schizophreniform disorder
Schizophreniform disorder is characterized by the presence of the symptoms of schizophrenia, but it is
distinguished from that condition by its shorter duration, which is at least 1 month but less than 6
months.
http://emedicine.medscape.com/article/2008351-overview
56- Withdrawal symptoms of smoking cessation start after
Answer: 3-5 days.
https://quitsmokingcommunity.org/how-to-quit-smoking/nicotine-withdrawal-timeline-symptoms/
57- definition of insomnia?
Answer : difficulty in initiating sleep, persistent problems ( at least 3 days/ week or month )
insomnia
answer: According to guidelines from a physician group, insomnia is difficulty falling asleep or staying
asleep, even when a person has the chance to do so. People with insomnia can feel dissatisfied with
their sleep and usually experience one or more of the following: fatigue, low energy, difficulty concentrating, mood disturbances, and decreased performance in work or at school
58- Pt saying that she had a protruded jaw and she wants a cosmetic surgery, when you examine her
her jaw is normal with no protrusion and you say that she doesn't need the surgery
She went already to 2 doctors who also refused to do the surgery. She is done blepharoplasty & other
cosmetic surgery before. What does she have?
Answer : Body dysmorphic disorder
59- young patient came to u want a full investigations to roll out cancer.. he is very scared of cancer
that it can destroy his job and future.. no family hx of cancer .. what is his psyhcotic condition?
Answer: hypochondriasis

60- ADHD case what is the treatment?


Answer: behavioral, parents education and pharmacological
Medications:
stimulant such as: Amphetamine sulfate (Evekeo) Dextroamphetamine (Adderall and Adderall XR, Dexedrine, ProCentra, Zenzedi) Dexmethylphenidate (Focalin and Focalin XR) Lisdexamfetamine (Vyvanse)
Methylphenidate non stimulant such as: Atomoxetine , Clonidine hydrochloride, Guanfacine (Intuniv)
http://www.webmd.com/add-adhd/adhd-medical-treatment

smle ,2016

915

61- male disoriented and hallucinating and disorganized thinking had aorto popletial graft and symptom fluctuates in the 2 days what the cause ?
multi infraction demntia
mania
demensia
Delirium
Answer: D
62- Indication for CT brain for dementia, all true except:a) Younger than 60 years old
b) After head trauma
c) Progressive dementia over 3 years
Answer: C.
63- Maintenance bipolar drug
A- sodium valvex
B- lithium
C- olnzapin
answer : B
All atypical antipsychotic , lithium is the best
64- patient was constantly seeking physicians believing that he has oesophageal carcinoma (or another type of cancer forgot) all of his workup were negative and all doctors reassured him ?
A-Conversion disorder
B-somatization disorder
C-hypochondriasis
answer : C
65- Woman feels abandoned by everyone and been hospitalized for trying to commit suicide many
times.
What is the type of her personality disorder?
a- Borderline
b- Obsessive
answer : A
http://emedicine.medscape.com/article/913575-overview
66- alzheimer brain feature
Answer:?!
pathology involves neurofibrillary tangles,neuritic plaques with amyloid deposition, amyloid angiopathy,
and neuronal loss.
MRI or CT may show atrophy
ref; USMLE step 2 CK
67- Teenage girl had fight with her friend and now she didn't want to see or meet her, and if the friend
come place the girl get out of it ?! What is the diagnosis
A-avoidance
ANSWER: A

smle ,2016

916
http://emedicine.medscape.com/article/294307-overview#a4
68- pt has diarrhea , abd pain , gait abn but oter lab normal :(( smle made easy )
A.
somataization
B.
hypochondial
C.
concersion
answer :A
69- Patient who eats a lot what is appropriate approach:
A- cognitive behavioral therapy
b- interpersonal therapy
If these are choices: A
70- Haloperidol toxicity, uprolling of eyes and dyskinesia, what is this?
A- Tardive dyskinesia
B- Neuroleptic malignant syndrome
Answer: B
(Emedicine) Administration of neuroleptic medications can result in any of the consequences listed in
Physical; however, certain combinations of medications (eg, lithium + haloperidol, anticholinergics +
haloperidol), depot preparations (eg, fluphenazine and haloperidol), and stronger neuroleptics (eg,
haloperidol) are more likely to produce adverse effects, including neuroleptic malignant syndrome
(NMS)
71- Psychiatric patient swallowed 2 safety pins, found on duodenum what to do?
A- Immediate laparotomy
B- Admit and observe
C- Charcoal
Answer:B
72- secondary phenomena in panic attack not in presenting complaint?
Phobia
Chest pain
Epigastric pain
Palpitation
answer: C?
73- Male patient, who is otherwise healthy, has depression for 4 months. He retired 6 months ago.
Examination was unremarkable except for jaundice. Whats your diagnosis?
a) Major depressive disorder
b) Mood disorder due to medical illness
c) Adjstment disorder, depressed type
Answer: C

smle ,2016

917
74- A 34 yo man begins to sweat profusely every time he has to give a presentation @ the office. The
sweating causes such marked anxiety @ each presentation that he soon develops palpitation, dizziness, & difficult breathing. He finds that if he has several alcoholic drinks before the presentation, he
feels "calmer" & can complete the talk. What is your likely diagnosis?
A. Schizophrenia
B. Social phobia
C. Generalized anxiety disorder
D. Delirium
Answer: B
75- 70 year old man wife died due to chronic disease two months ago. He is feeling sad, fatigue and
sleep disturbances. Diagnosis?
A. Major depression
B. Minor depression
C. Bereavement
Answer: C
76- Attention Deficit Hyperactivity Disorder ( they give me the symptom not the diagnosis ) child
what is the management?
a)
Ecitalpram
b)
Atomoxetine
c)
Olanzapine
d)
Clonazepam
Answer: B
first time: methylphenidate, dextroamphetamine
Atomoxetine (Strattera) has become a second-line
77- Patient see in cars and people in his plate food, What is the dx?
Answer: visual hallucinations ( if there is no food in his plate )
If there's food then it's illusion!
78- patient in clinic suddenly he said he looks to left and said he saw his mother and no one can saw
her except me his mother died when he was child ?
A.audible hallucination
B.visual hallucination
C.delirium
Answer : B
79- Pt came with 3 hours hx of anxiety , diaphoretic , tachypneic , what is the cause ?
A- Sympathomemitcs
B- organophostate
C- anticholinergics
Answer A
80- A lady had progressive sadness over the past 2 years. She has hopelessness, insomnia, decreased
appetite, low self-esteem and suicidal ideation. Diagnosis?

smle ,2016

918
A.
Minor depression
B.
Depressive disorder
C.
Dysthymia
answer: C
81- which of SSRIs drugs suitable for young age and children?
A- fluoxetine.
Answer is A
http://www.webmd.com/depression/selective-serotonin-reuptake-inhibitors-ssris-for-childhood-andadolescent-depression
82- Pt e erectile dysfunction , what's tha med to avoid?!
SSRIs
83- Bipolar patient type 1, started on lithium 6 weeks ago, c/o excessive thirst
and urination. Serum lithium within the optimal therapeutic range. What's the
underlying cause of his symptoms?
A- Psychogenic polyuria
B- Central DI
C- Nephrogenic DI
Answer: C
because lithium is a common cause of NDI and even if it's in therapeutic rand we should suspected as
cause.
84- pt with hx of chronic schizophrenia, But he is not compliance Wt u will give him ?
IV Lorazepam + IV haloperidol
injection of haloperidol deconat
oral haloperidol
Answer: B
85- Stress induced cortisol release , and decrease the immunity, what type of therapy?
Psychoimmunology?!
86- Generalized anxiety disorder, doctor want to give medication that is very effective but has minimal abuse ability:
azatilopram?
87- amitriptan " TCA" side effect
a-weight gain
answer : A
88- antidepressants cause insomnia ...
TCA
89- SE of TCA .
Dry mouth
Blurred vision
Constipation

smle ,2016

919
Urinary retention
Drowsiness
Increased appetite leading to weight gain
Drop in blood pressure when moving from sitting to standing, which can cause lightheadedness
Increased sweating
Ref: mayoclinic
90- Effective half life of fluoxetine ?
18 hu
2 day
4 day
9 day
Answer:
Half life 4-6 days in chronic administration
1-3 day in Acute , Medscape
91- best management of trichotillomania?
No choices provided but Currently available evidence suggests that the first line of treatment for
trichotillomania is behavioral treatment and intervention. No medication has been approved for the
treatment of trichotillomania. Medscape.

92- Scenario about pt after the death of his father went out and do things , then he return normal but
he is confused about what happen to him in the prevoius days, Dx
-Schizophrenia
-Schizoaffective
-Brief psychotic disorder
Answer: C.
93- pt speaks rapidly , before he demonstrate his idea he goes to talk about other idea..
A- Loose of association
B- Preservation
C- Flight if idea
Answer C
94- a man walk in the street and and asks people and repeat even when he was asked to stop
A.preservation
b-flight of idea
c-poor insight
answer: A https://en.wikipedia.org/wiki/Perseveration
95- Q- female pt around 50 , complaining of palpitations weight loss insomnia , without reason
A - Generalized anxiety disorder
C - Panic disorder
Answer: If you find pheochromocytoma (choose it)
96- Treatment of OCD ;increase in sensitivity to serotonin

smle ,2016

920

97- Case of Adjustment disorder and asking about the best treatment
A) supportive psychotherapy
98- feeling Hopelessness might mean:
suicidal attempt
99- Patient with Premature ejaculation + libido + Erectile dysfunction he isthin and looks sad, he is
married for 26 years obese and annoying wife, he came for treatment:
a) Testosterone Injection every one week
b) Sublingual Nitroglycerin 6h before intercourse
c) SSRI
Answer: C.
100- A 72 yo man is brought to the ED due to altered mental status & combative behaviour. He was
admitted 2 weeks ago for hip fracture. Patient also has history of hypertension & recent UTIs. Though
normally "calm & loving" per her family's report, the patient has grown restless & combative with
staff & residents in past days. What is the most likely diagnosis?
A. Psychotic disorder resulting from a general medical condition
B. Delirium due to a general medical condition
C. Brief psychotic disorder
D. Depression
Answer: B
101- An elderly male told you that he's always occupied by the idea that his backyard is invaded by
aliens, although he knows that aliens don't exist and no one is invading his back yard. He's afraid that
he's "going insane". He has:
A. Delusions
B. Obsessions
C. Compulsions
D. Hallucinations
Answer: B
Obsession is the inability of a person to stop thinking about a particular topic or feeling a certain emotion without a high amount of anxiety
102- Posrpartum mother complain of anxity fear fatighe lasting few days then subside ??
A maternal blues
B postpartum depression
C postpartum psychosis
D anxity disorder
Answer: A.
103- pt come clinic the result of investigation is lung cancer the pt say with anxiety (it's a mistake)
Which of the following explain the reaction of pt:
A- denial
B- anger

smle ,2016

921
C- barogainer
Answer: A
104- A 54 yo man on an antidepressant medication presents to the ED with hypertensive crisis after
eating an aged cheese with his antidepressant medication. Which of the following antidepressant is
he on?
A. Amitriptyline
B. Fluoxetine
C. Phenelzine
D. Venalafaxine
Answer: C
105- Causes of eating disorder
exact cause of eating disorders is unknown, it is generally believed that a combination of biological, psychological,and/or environmental abnormalities
106- A man hears sounds from the refrigerator, he has:
Auditory hallucinations
107 -Pt elderly with symptoms of infections and delirium what to give:
haloperadol
108- p.t taking a medication , came to the ER suspecting she has
Overdose of her medication, her symptoms ( convulsion, dilated pupil, hyperreflexia and strabismus)
the medication is:
-TCA
-SSRI
-Ephedrine
-Hypervitaminosis
Answer: A
109- A 10 yo boy, Akeem, was diagnosed with nocturnal enuresis. After a thorough evaluation, his
doctor diagnosed him with 'depression' and which to prescribe a medication for him. Which of the following medications is the most appropriate for him?
A. Haloperidol
B. Imipramine
C. Venlafaxine
D. Fluoxetine
Answer: B
110- Stress cause increase in catecholamine and something what types of stress?
A) Psychoneural stress
Answer: A

smle ,2016

922

111- SE of bupropion
Answer: Nausea, vomiting, dry mouth, headache, constipation, increased sweating, joint aches, sore
throat, blurred vision, strange taste in the mouth, diarrhea, or dizziness may occur
Ref.: webmed
112- A pt cannot sleep, goes to work exhausted becuz she check gas, door and kids stuff for school
several times. What is this condition called in psychiatry?
Answer: OCD
113- Panic disorders treatment:
Answer : SSri 1st line and Benzodiazepines 2nd line

114- Diazepam used to treat which psychotics disorder in children


Answer: diazepam doesn't treat psychosis!

115- A 23 YO patient who sees her nose very big, but all doctors see its normal, already done two
plastic surgerys before for lips and eyelids. What is this condition called in psychiatry?
Answer: body dysmorphic disorder
116- A pt cannot sleep, goes to work exhausted becuz she check gas, door and kids stuff for school
several times. What is this condition called in psychiatry?
Answer: OCD
117- A male patient presented with depression. What is the first drug to be given in depression?
a. TCA
b. lithium
c. CBT
d. SSRI
Answer: D
118- Case of Adjustment disorder and asking about the best treatment.
A) supportive psychotherapy
Answer: A
119- patient with depression on anti depression medication , how long it will take to exerts its effect?
3-4 weeks
4-6weeks
Answer: 3-4weeks

smle ,2016

923
120- Patient afraid of diseases and germs, what dx?
A- Obsessive compulsive
B-Anxiety disorder
C-Specific phobia
Answer: C
http://www.apa.org/monitor/julaug05/fears.aspx
121- pt with fear of automobiles?
A- specific phobia.
B- GAD.
Answer A.

122- Pt came with his wife. she said that he walk around the house 8 times to make sure its
123- locked and wash his hand many times before eating. The pt said this not affecting his life. Past
hx: he likes to stay alone all the time What is the diagnosis ?
A. OCD
B. Generalized anxiety disorder
Answer: A
124- A 67 yo man presents to the ED with hypertension, diaphoresis, tachycardia, & tremor. He is agitated & confused stating that he can feel rats crawling all over him. Physical examination reveals that
he has ascities & a liver edge that is palpable 6 cm below the costal margin. What is the mainstay therapy for this man?
A. Benzodiazepine
B. Flumazenil
C. Glucagon
D. N-acetylcysteine
Answer: A
HINT: Delirium tremens
125- A 31 yo woman presented 3 weeks after her baby was born with feeling that her neighbours
were poisoning her water & planning to steal her 2 older children from school. She heard the neighbors talking through the walls of her house. She was feeling that there was no choice but to kill herself
& her children & made plans to drive them into a tree. What is the most likely diagnosis?
A. Postpartum depression
B. Postpartum blues
C. Postpartum psychosis
D. Generalised anxiety disorder
Answer: C.

smle ,2016

924
126- patient with history of 2 years depression, decreased appetite, low self- esteem
Major depression
Depression something
Dysthymia
Answer: C
source: Toronto notes 2014 PS11
127- Patient with fear of motor vehicle, increasing in intensity, what is the dx?
A- Specific phobia
128- 40 yrs old woman c/o thinking a lot about her children future. She is alert, anxious & cannot
sleep. She has poor appetite. Provisional diagnosis?
A. GAD
B. OCD
C. Schizophrenia
D. Phobic disorder
Answer: A
129- anxiety definition and gave me some disorders , which one included in this definition
a- obsessive compulsive
b- disociative
c-bipolar
Answer: A
130- A patient presented with history of panic attack of job meeting and she became anxious. What is
the most likely diagnosis?
A-specific anxiety disorder
B-panic attack
Answer:A
panic attacks must be associated with longer than 1 month of subsequent persistent worry about:
(1) having another attack or consequences of the attack, or
(2) significant maladaptive behavioral changes related to the attack.
To make the diagnosis of panic disorder, panic attacks cannot directly or physiologically result from substance use (intoxication or withdrawal), medical conditions, or another psychiatric disorder. Other symptoms or signs may include headache, cold hands, diarrhea, insomnia, fatigue, intrusive thoughts, and ruminations.
http://emedicine.medscape.com/article/287913-overview
131- Male patient is concerned about his sexual performance. During intercourse he develops palpitations & diaphoresis. Physical exam: normal. lab: within normal range. What is the diagnosis?
Answer: PANIC
132- A patient presented with history of washing his hand a lot. What is the most likely diagnosis?
a. obsessive compulsive disorder
Answer: A

smle ,2016

925

133- A 35 yo woman presents to her Psychiatrist with c/o inability to sleep properly and goes to work
exhausted because she checks cooking gas, doors, and Kids stuff for school several times at night.
What is the most likely diagnosis?
A. Obsessive-compulsive disorder
B. Generalized anxiety disorder
C. Hypochondriasis
D. Social phobia
Answer: A
134- Most common affect symptom in premenstrual dysphoric disorder:
Irritability
Mood swings
Depression
Anxiety
Answer: A
135- Girl feel irritable, palpitations, and nervous every time the ask her to speak in the class, What's
the management?
A. Small dose of lorazepam
B. Behavioural therapy
C. Relaxing techniques
Answer: B
Behavioral therapy include relaxation

136- long scenario when you read it you think it about psychiatry but I think it about premenstrual
syndrome What is the treatment
ssri
other psych medication
combined oral contraceptive
psych medication
Answer: A
137-Parents admitted to your clinc with their adolescent son ,they are seeking help of bad behaviors
of their son(I can not remember what was is )During assessmen of adolescent by( H.E.A.D.S.S) Home t
Environment E ducation Employment Activities Drugs Sexuality Suicide/Depression what is the best
one of communication to the adolescent ?
Group therapy
Individual therapy*
Communicate to the adolescent with presence of his parents
Answer : B

smle ,2016

926
http://www.bcchildrens.ca/Youth-Health-Clinic-site/Documents/headss20assessment20guide1.pdf
138- Management of somatization?
a) Multiple phone call
b) Multiple clinic appointments
c) Refer to pain clinic
d) Antidepressant
Answer: B.
139- pt continues to do something, if he try to stop he becomes irritable:
A- Obsession
B- Compulsion
Answer: B
140- Pt counts floor lines, he knows it is wrong, but he cant prohibit it:
A- compulsion
B- obsession
C- psychosis
Answer : A
141- Which of the following medications is associated with QT prolongation?
a) chlorpromazine
b) clozapine
c) haloperidol
d) ziprasidone
Answer: D
142- 52 y/o male complaining of impotence, he has anxiety and stressful things in his work?
give him sildenafil
relaxation exercise
refer him to urology
Answer : B
143- women brought her father had dementia of recent events what u will do ?
refer to geriatric
give him antipsycotic
measure IQ
Answer :A
144- 20 yo female came with her Dad with complain of tachycardia, abdominal pain & peripheral tingling after she failed her Math exam. Diagnosis?
A. Anxiety
B. Depression
C. Hyperventilation syndrome ?

smle ,2016

927
D. Oppositional defiant disorder Answer: C

Anesthesia

smle ,2016

928

1- Earliest sign of local anesthetic toxicity:


a.
Tongue and circumoral numbness
Answer: A
Reference: Toronoto Notes.
2- What is the fasting duration for non-breast milk?
a.
8 hours
b.
6 hours
c.
4 hours
d.
2 hours
Answer: B
8 hours (solid)
6 hours (formula)
4 hours (breast milk)
2 hours (clear liquid)
Reference: Toronto notes.
3- Which of these analgesia has x10 the strength of morphine?
a.
Fentanyl
b.
Tramado
c.
NSAID
Answer: A
Reference: Toronto Notes.
Extra notes:
Common Side Effects of Opioids: Nausea and vomiting, Constipation, Sedation, Pruritus, Abdominal pain, Urinary retention, Respiratory depression..
When prescribing opioids, consider: Breakthrough dose, Anti-emetics, Laxative.
4- What is the sign that indicate inadequate general anesthesia?
Dry skin.
Hypotension.
Bradycardia.
Dilated Pupils.
Answer: D
Reference: Toronto Notes
5- Pregnant on delivery she has hypotension and dyspnea. Which type of anesthesia will be
given?
a.
Pudendal n
b.
Local cervical
c.
General anesthesia
d.
epidural
smle ,2016

929
Answer: General anesthesia
Reference: UpToDate
6- What is the anesthetic drug that can be used as an analgesic in mild doses?
Midazolam
Thiopental
Answer: ?
ketamine (Answered by Anesthesiologist consultant)
Reference: http://www.ncbi.nlm.nih.gov/pubmed/25197290
7- Mallampati class 3 what you will see?
Soft palate and uvula
Soft palate and base of uvula
Soft palate,uvula and tonsillar pillars
Answer: B

8- During labor, the anesthesiologist inject analgesic drug at L3-L4, after 2 days the mother
still have pain in the site of injection. Which ligament is affected ?
a.
Anterior longitudinal ligament.
b.
Posterior longitudinal ligament.
c.
Ligamentum flavum.
d.
Interspinous ligament.
Answer: D
In this Q: we don`t know if the doctor used spinal or epidural but in general pain at site of injection = inflammation of supraspinous lig or interspinous lig. (Answered by an anesthesia consultant at KFMC)
Note: The associations between back pain and epidural analgesia are unclear.
- Local tenderness at the site of epidural or spinal placement are relatively common & usually
clears within several days to 3 weeks and may be related to superficial irritation of the skin or
periosteal irritation or damage.

smle ,2016

930
- Although short-term back pain is common, it does not appear to be related to the use of regional analgesia. Similarly, no causal relationship exists between the use of epidural analgesia
and the development of long-term postpartum backache.
9- How to prevent a high level of anesthesia?
Anti trendelenburg
Trendelenburg
Answer: A?
If the level of spinal anesthesia is not fixed, the Trendelenburg position can alter the level of
spinal anesthesia and cause a high level of spinal anesthesia in patients receiving hyperbaric
local anesthetic solutions. This can be minimized by raising the upper part of the body with a
pillow under the shoulders while keeping the lower part of the body elevated above heart
level. Reference: http://www.nysora.com/index.php?news=3424
- pt with local anesthesia cant remember how to prevent high level anesthesia?
anti trendlendburg
trendlenburg
Answer:a??
) position for cephele
Ant- tendrulge
Tendulergr
Lateral
Supain
11 - Station +1 80% effacement and 4 cm dilatation and RUpture of membrane with clear fluids, : what anesthesia to give?
A- VS
B-Pudendal,
C- paracervical,
D- GA
Answer: D
12 - Same of common about breast feeding
wt is indicative of poor anesthesia ??
Answer:
"awareness with recall" (AWR) refers to both intra-operative consciousness and explicit recall
of intra-operative events. Other terms, such as "intra-operative awareness during general anesthesia," "anesthesia awareness," or simply "awareness," are used as synonyms.
Anesthetic under dosing The most important contributing factor for AWR is under dosing of
anesthesia relative to a given patient's specific requirements.
This can occur for the following reasons [21-23]:
There is a mistake or failure in the delivery of anesthesia
The anesthetic technique results in inadequate anesthesia
smle ,2016

931
It is judged unsafe to administer sufficient anesthesia
A specific patient's needs are underappreciated
Ref :uptodate
13 - pregnant with late deceleration what type of anesthesia u will give
her
Answer:?
14- anesthesiologist put the needle lateral to spinous process, which ligament is perforated? 2 times
a.
interspinous
b.
ligamentum flavum
c.
Anterior ligament
D. posterior ligamint
Answer: B
The needle should be inserted 1 cm lateral to this point and directed toward the middle of
the interspace. The ligamentum flavum is usually the first resistance identified
http://www.nysora.com/index.php?news=3425
15- to increase hyperbaric in intrathecal analgesia put patient in :
A) lateral
B) supin
C)trundlberg
D)antitrendulberg
Answer: C
Hyperbaric solutions goes with gravity
Hypobaric solutions goes against gravity
So to increase hyperbaric solutions spread the patient is put in antitrendulenberg position
https://books.google.com.sa/books?id=rIFoAgAAQBAJ&pg=PA201&lpg=PA201&dq=intrathecal+analgesia+trendlburg+position&source=bl&ots=iR1E22mWdb&sig=24SxofkHANJms_YMLzR9c3ngS1A&hl=ar&sa=X&ved=0ahUKEwjE9ofG3rzNAhVGfRoKHfQ1CEcQ6AEIXzAJ#v=onepage&q=intrathecal%20analgesia%20trendlburg%20position&f=false
16 - Patient in dental clinic received local anesthesia and give numbness below eye , maxilla
and part of the nose most likely nerve blocked is :
Sphenopalatine
Infraorbital
answer: B
17- Best medication to give with analgesics?
Metoclopramide
Answer : i think its right because the analgesic drugs mimic peptic ulcer.
http://emedicine.medscape.com/article/171886-medication
smle ,2016

932

18 - Anesthesia Subdural Between L3 -L4 Which ligment will be porferated?


Interspanits
19- Pt in pain dificult to communicate with him how to asses his pain ?
1-face scale*
numerical scale
answer is A
20- increase effect of analgesia?
-metacloprmide
http://www.medscape.com/viewarticle/429668_3
21- Anesthesia provide sedation with sub anshthtic dose
Pentamin
Midzpam
KetameN
Answer : ketamine confirmed by intern took 100%in ansthesia
http://reference.medscape.com/drug/ketalar-ketamine-343099
22-Patient in dental clinic received local anesthesia and give numbness below eye , maxilla
and part of the nose most likely nerve blocked is :
A) Spheno-palatine
B) Infra-orbital
Answer: B
Explanation: Infra-orbital beneath the orbital rim, and supra-orbital for sensation above the orbital rim.

smle ,2016

933

Dermatology

smle ,2016

934

1- A beach guard who stays most of the time under the sun, presented to you complaining
of new papules over his nose and cheeks. What is the most likely diagnosis?
Melasma
Hamartoma
Freckling
Actinic keratosis
Answer: D
Premalignant lesions caused by sun exposure. Small, rough, erythematous or brownish papules or
plaques. Risk of squamous cell carcinoma is proportional to degree of epithelial dysplasia.
Reference: First Aid USMLE Step 1
2- A patient with a typical history of herpes zoster infection with dermatomal involvement?
HSV
Herpes Zoster
Bacterial vaginosis
Answer: B
3- Patient with psoriasis involving 15% of his body with nail involvement. What is the best
treatment for him?
Laser
Topical steroid
Topical vitamin D analogs
Methotrexate
Answer: D
Methotrexate can be used in severe psoriasis (involving more than 10% of body surface).
4- Picture of a woman's face with comedones and papules on the cheek. Which of the following drugs should be avoided?

Retinoic acid
Erythromycin
Tetracycline
Steroids
Answer: D

smle ,2016

935

5- Description of a man with red nodules and papules on the face involving mainly the cheeks
and nose. Few telangiectasias are also present. What treatment will you suggest?

Cold compresses
Oral doxycycline
Topical retinoic acid
Topical steroids
Answer: B
Its a case of rosacea.
6- 18 years old girl with vitiligo on the face and arm that is symmetrical started on medical
treatment 3 years ago. She wants to get married soon and wants the lesions go away. What
will you do for her?
Split thickness graft
Continue medical treatment
Melanocyte transfer
Stop medication and observe
Answer: C
7- A long scenario about a rash over the elbows, knees, and cheeks that is itchy, weeps and
crusts which got better with steroids. What is the diagnosis?
HSV
Staph
Fungal
Eczema
Answer: D
8- A patient complaining of itchiness that increases at night, superficial linear burrows
around 4th & 5th digits, inflammatory papules and nodules?
Atopic dermatitis
Allergic dermatitis
Scabies
Tinea
smle ,2016

936
Answer: C
9- Which of the following drugs is used for acne rosacea?
Erythromycin
Clindamycin
Cephalexin
Answer: A
For mild rosacea: We can use topical metronidazole or azelaic acid creams and gels.
For severe rosacea: doxycycline (first line therapy), tetracycline and erythromycin.

10- Rash in the dermatomal distribution of the trunk?


Answer: Shingles
11- Treatment of pyoderma gangrenosum?
Methotrexate.
Antibiotics.
Systemic Steroids.
Answer: C
Reference: http://emedicine.medscape.com/article/1123821-treatment
12- Condyloma lata is caused by?
Answer: Treponema pallidum
Wart-like lesions on the genitals .They are generally symptoms of the secondary phase of syphilis,
caused by the spirochete, Treponema pallidum.
13- Condyloma acuminata is caused by?
Answer: HPV 6 and11
14- 30 years-old man came with erythema of the nose and pain in the right eye, there is
erythema and nodules in left periorbital area and forehead. What is the most likely diagnosis?
Systemic lupus Erythematosus.
Herpes zoster.
Rosacea.
Measles.
Answer: B
15- Schoolboy brought by his mother, he have 2x2 hair loss in the temporal area, hair
around this area-clubbed hair. What is the most likely diagnosis?
Trichotillomania.
Alopecia areata.
Tinea infection.
Telogen effluvium
Answer: D

smle ,2016

937
16-4 years old child presented with area of 3x3 hair loss, on examination of the area there is
multiple pustule. What is the most likely diagnosis?
Aplasia cutis congenita.
Staphylococcal infection.
Trichotillomania.
Answer: B
(staph infection alone won't cause hair loss , its only cause pustules so in this case it's look like
infection on top of an original Dx which had caused the alopecia which is missing in this scenario
, if the child born with areas of alopecia the dx would be Aplasia cutis congenita which later could
be complicated by secondary infection )
Reference: http://dermnetnz.org/lesions/aplasia-cutis.html

17- A Picture of 1 month baby with rash and scale what is the most likely cause?

Answer: Seborrhoeic dermatitis


18-What is the main treatment for non inflammatory acne?
Azelaic acid
Isotretinoin
Differin cream
Answer: C
19- What is the treatment for moderate to severe acne vulgaris?
Isotretinoin
Tetracycline
Clindamycin
Answer: A
20-Patient with Positive HPV. What is associated with it ?
Hyperkeratosis
Parakeratosis
smle ,2016

938
Apoptosis
Answer: A
21 - Patient with single hypopigmented lesion on the forearm, with Ulnar nerve thickness,
what is the diagnosis?
Vitiligo
Amyloidosis
TB
Answer: I think it's associated more with leprosy because leprosy can presents with Hypopigmented or reddish skin lesion with Involvement of the peripheral nerves: Demonstrated by definite
thickening of the nerve with/ without loss of sensation and/or weakness of the muscles of the
hands, feet or eyes supplied. while vitiligo rarely to be associated with nerve thickness
Refrence : http://nlep.nic.in/pdf/manual3.pdf

22 - Single Red lesion on shoulder that keeps growing?


Strawberry Nevus
Pyoderma

gangrenosum

Answer : A
23 - Black to brown lesion on sole of the foot?
Answer: differential diagnosis includes benign nevus, a pigmented seborrhoeic wart, a squamous
cell papilloma, carcinoma or malignant melanoma. a capillary cavernous haemangioma and Kaposi's sarcoma. so the differential is so broad and we need more details.
24 - Diaper rash that is resistant to topical steroids and has satellite lesions on thighs (well
demarcated and red), topical antifungal?
Answer: Antifungal agents such as nystatin, clotrimazole, miconazole, ketoconazole, and
sertaconazole are effective topical therapies for diaper rash.
Reference : Uptodate
25- About baby with multiple plaques in his face, abdomen and feet what is the diagnosis?
Basal cell carcinoma
Squamous cell carcinoma
Answer: Most likely its eczema but im not sure
26- A patient with a typical history of headache, fever and then rash which type of herpes?
HS type 1
HS type
Varicella
Answer : C

smle ,2016

939
27- Patient with a localized patch of hair loss?
A.
Male
pattern
B.
Female
pattern
C. Something starting with T I dont know what is it but I chose it !!
Answer :
*Male pattern hair loss: receding hairline and/or hair loss on the top and front of the head.
*Female pattern hair loss: diffuse thinning of hair on the scalp
*Causes of localized patch of hair loss:
-Tinea capitis the most common cause of hair loss in children. patchy hair loss with some brokenoff hairs visible just above the surface of the scalp
-Alopecia areata appear literally overnight, or sometimes over a few days
-Trauma including trichotillomania
-Telogen effluvium
-Traction alopecia thinning from tight braids or ponytails
28- Patient came with lobulated nose, erythema over cheeks and (other features indicating
rosacea
type
4),
what
is
the
treatment?
A.
Doxycycline.
B.
Acyclovir.
Answer: A
4- First line of defense in the skin is?
Mucous membranes
Collagenous cell.
Keratinocytes.
Areolar connective tissue
Answer: C
29- First immunologic defense of skin?
A) Keratinocytes
B) Blast cells
C) Melanocytes
Answer: A
30- Scenario for patient with scabies what is the treatment of choice?
Answer:
1-treated overnight with 1-2 applications of 5% permethrin from neck down +their contacts should
be treated
2-oral ivermectin
3-symptoms treatment pruritus
Reference: FIRST AID USMLE STEP2 CK
31- A patient presented with macular papular rash and fever no vaccination before ?
A) Measles
B) Mumps
C) Rubella

smle ,2016

940
Answer: it could be A or C.
Although the distribution of the rubella rash is similar to that of rubeolameasles, the spread is
much more rapid, and the rash does not darken or coalesce
also Rubella has low grade fever
Measles has a high grade fever
Reference: Uptodate
Mumps: low-grade fever, malaise, headache, myalgias, and anorexia. These symptoms are generally followed within 48 hours by the development of parotitis
32-About patient on long uses of topical steroids, what will happen ?
A. Scaling
B.Atrophy
Answer: B
Side effects of local steroids includes: Skin thinning (atrophy)
Stretch marks (striae) in armpits or groin
Easy bruising (senile/solar purpura) and tearing of the skin.
Localised increased hair thickness and length (hypertrichosis)
Enlarged blood vessels (telangiectasia)
33- The best test for hypersensitivity type 1 ?
Answer: Subdermal skin test

34- Pregnant +ve hx of herpetic outbreak annually, on examination everything is normal.


What is the management?
A Reassurance
B Acyclovir
Answer: A ? not sure
35- How to decrease the keratosis?
A. Wound trauma
B. Use AB
C. Avoid sun exposure
Answer : C
36- Pt have nodulo-cystic acne how to management?
A. Clindamycin
B. Oral AB
C. Erythromycin
D. Isotretinoin
Answer : D

smle ,2016

941
37- A case of a patient diagnosed to have cutaneous leishmania or bghlabar/ bhagdad type which type of leishmania ? or what's the organism ?
Tropica
Donovani
Post kala-azar dermal leishmaniasis
Answer: A
Reference:
http://www.uptodate.com/contents/clinical-manifestations-and-diagnosis-of-cutaneous-leishmaniasis?source=search_result&search=leishmania&selectedTitle=1~93
38- Oropharyngeal maculopapular rash , Also rash in palm and foot ?
CMV
EBV
Coxsackievirus
Vaccinia virus
Answer: C
Reference:
http://www.uptodate.com/contents/hand-foot-and-mouth-disease-and-herpangina-an-overview
39- 2 year old complain of papule on the foot no itching pink pale not respond for antifungal?
Answer: Granuloma
Granuloma annulare is a common skin condition with raised, flesh-colored bumps that appear in
a ring. It may occur on any part of the body (though most commonly on the sides or backs of the
hands or feet). The bumps may be red at the beginning, but this disappears as the ring forms. There
is no itching or scaling.
Reference: http://www.uptodate.com/contents/granuloma-annulare

40- Isotretinoin most feared complication?


Answer: Birth defect.
Isotretinoin (roaccutane) is teratogenic it can cause embryopathy in 20 - 30 % and spontaneous
abortion is approximately 20 %
Reference: http://www.uptodate.com/contents/oral-isotretinoin-therapy-for-acne-vulgaris
41- Young female, loss of hair half of her head and the skin is normal?
Alopecia
Trichomania
Answer: No Enough details but it could be Trichomania
Reference: : http://emedicine.medscape.com/article/1071854-overview#a6
42- Patient diagnosed with shingles around thoracic dermatome, what will you give him?
A. Topical acyclovir and topical steroid
B. Oral steroid
C. Oral acyclovir

smle ,2016

942
Answer: C
43- Male with pustules and papules and telangiectasia , what is the diagnosis ?
Answer: Rosacea.
45- Question about description of scarlet fever rash?
Answer: The rash of scarlet fever is a diffuse erythema that blanches with pressure, with numerous small (1 to 2 mm) papular elevations, giving a "sandpaper" quality to the skin . It usually starts
in the groin and armpits and is accompanied by circumoral pallor and a strawberry tongue . Subsequently, the rash expands rapidly to cover the trunk, followed by the extremities, and, ultimately,
desquamates; the palms and soles are usually spared. The rash is most marked in the skin folds of
the inguinal, axillary, antecubital, and abdominal areas and about pressure points. It often exhibits
a linear petechial character in the antecubital fossae and axillary folds, known as Pastia's lines.
46- Patient present to the ER with erythema and peeling of the skin with fever and other
things?
A-Toxic epidermal syndrome
B-Neisseria meningitidis
C- Septicemia
Answer: A
47- Symmetrical hypo pigmentation with no scaling for 1 year and it's progressing with no
previous lesion?
Post inflammatory
Vitiligo
Leprosy
Answer: B
48- Scabies ,what's the organism ?
Answer: Sarcoptes scabiei
49- Purple,papule,polygonal rash on the flexors?
Answer: Lichen planus
50- Child with maculopapular rash on the face and nose ( not sure about vesicles ) White
spot inside cheeks, I think there is fever?
Mumps
Measles
HSV1
Varicella zoster virus
Answer: B
The answer depends on vesicle if it present it would be vsv if absent would be measles.
white spot inside mouth could be ( koplik spot ) in this case the answer is measles.
51- DM pt with has lesion papule on dorsum of right hand ?
Granuloma annulare

smle ,2016

943
Lichen plants
Fungal infection
Answer: A
52- Old patient with diffuse thinning of hair and without eyelash, What is the diagnosis ?
Answer: Alopecia areata
Reference: http://www.uptodate.com/contents/clinical-manifestations-and-diagnosis-of-alopecia-areata?source=see_link
53- What is most commonly cause itching ?
Bile salt retention
Eczema
Pregnancy
Answer: B
Reference:
http://www.uptodate.com/contents/pruritus-etiology-and-patient-evaluation?source=search_result&search=itching&selectedTitle=2~150#H690781
54- Female recently used eye cream and developed inflammation in the face with redness?
Contact dermatitis
Seborrheic dermatitis
Answer: A
Note: Seborrheic : condition affecting infant and cause yellow crusty greasy scaling.
Contact: due to contact to material of clothing soap lotion chemical detergent and medication.
55- Case of Contact dermatitis, what is the Treatment ?
Topical steroid
Systemic steroid
Systemic antibiotic
Fluconazole
Answer : A

56- Patient with nodulocystic acne with scar what is the treatment ?
Oral isotretinoin retinoids
Oral antibiotics
Topics antibiotics
Answer: A
57- Question about Angioedema ?
Answer : Angioedema is form of urticaria in which there is deeper swelling in the skin which
may take longer than 24 Hours to clear.
58- Rash that involved the palms and soles but spared the face?
smle ,2016

944
Chancroid
Syphilis
Herpes
Leishmaniasis
Answer : B
List of rashes involved Palm & soles :
Meningococcemia
Keratoderma
blenorrhagica
(Reiters
Typhus
Acral
lentiginous
Hand,
Foot,
and
Mouth
Mercury
poisoning
in
Bacterial
Tylosis
Rocky
Mountain
Spotted
Graft
Versus
Host
Disease
Kawasaki,
Measles,
or
Toxic
Shock
Steven
Johnson
Secondary and Congenital Syphilis with its characteristic Copper colored rash.

syndrome)
melanoma
Disease
children
endocarditis
Fever
rash.
Syndrome
syndrome

59- Itching in lower limbs ,otherwise normal ?


Tinea
Scabies
Answer:??
Maybe a sign of a serious internal disease not dermatological disease OR stress.
60- Vesicles highly suspected roundworms ?
Ascaris
Taenia saginata
Answer: ???
61- Alopecia, in a boy who performs poorly in school ?
Answer: Trichotillomania
It's a compulsive disorder resulting in Alopecia from repetitive hair manipulation by the patient`s
own hand.

62- Patient with hyperpigmented non pruritic papules in the dorsum of the hands not resolved with antifungal ?
Tinea corporis
Lichen planus
Answer: ?? Other choice!
Lichen planus : 6 P : pruritic , purple, polygonal, papules and plaques.

smle ,2016

945
63- Pic / pink patch under woods lamp it fluorescent a coral red color Dx ?
A. Erythrasma
B. Fungal infection
C. Vitiligo
Answer: A
Caused by G+ve bacteria Corynebacterium minutissimum.

64- Treatment of folliculitis ?


Answer:
scalp
folliculitis:
Topical antibiotics eg fusidic acid gel, clindamycin solution, erythromycin solution
Mild
topical
steroid
lotions
or
creams
Oral
antihistamines
Oral
antibiotics,
particularly
long-term
tetracycline
Oral isotretinoin long-term low dose treatment.
65Trichotillomania
treatment
Answer: I don't know Choices but the best answer is ( Cognitive behavioral therapy).

66Treatment
of
acne
rosacea?
Answer : oral AB ( Doxycycline) and topical Antibiotics, like metronidazole for mild to moderate.
67- Scrape of skin can be done in which of the following?
Answer: Scabies
Most effective drug for Rx is permethrin cream.
68- Picture of lichen planus
http://emedicine.medscape.com/article/1123213-overview
69- Patient is brought by his parent because of loss of hair. On examination: he had localized
patch of hair loss at temporal area, the end of hair looked broken and tapered. What is the
diagnosis?
Alopecia Areata.
Trichotillomania.
Answer: Trichotillomania
smle ,2016

946

70- Multiple myeloma in the spine?


Answer: Histopathology report
71- Adult male on multiple drug came with violaceous maculopapular lesion in the trunk
Diagnosis?
Erythema multiforme
Toxic epidermal necrolysis
Answer: Exanthematous ( maculopapular ) drug eruption

http://www.aafp.org/afp/2010/0315/p726.html
72- Patient c/o rash over elbows, knees, and cheeks?
Answer : Eczema , But If there's no cheek it will be psoriasis.
73- Treatment of moderate to severe acne vulgaris
Isotretinoin
Clindamycin
Tetracycline
Answer: A
74- Case of scabies very clear.
smle ,2016

947
Answer: May be as Diagnosis there's night itching.
Permethrin 5% lotion. Alternative drug therapy includes precipitated sulfur 6% in petrolatum,
lindane, benzyl benzoate, crotamiton, and ivermectin; a possible new option is albendazole. repeat
application in 7 days.
http://emedicine.medscape.com/article/1109204-medication
75- Patient dx with cutaneous leishmania what is the organism ?
Answer: Lishmenia aka aza
Kala azar is other name for a Visceral leishmaniasis!
76- Female patient obese with regular menstrual cycle , on PE/ she had acne , other examination is normal , what investigation will you order ?
TSH
ACTH
Answer: B for cushing!
77- Pyoderma with any disease?
Answer: immunodeficiency patient.
Commonly associated diseases include inflammatory bowel disease, either ulcerative colitis or
regional enteritis/Crohn disease, and a polyarthritis that is usually symmetrical and may be either
seronegative or seropositive. Hematologic diseases/disorders are other commonly associated conditions; these include leukemia or preleukemic states, predominantly myelocytic in nature or monoclonal gammopathies (primarily immunoglobulin A [IgA])
Refrence: http://emedicine.medscape.com/article/1123821-clinical
78- Keratitis caused by parasites?
Answer : Acanthamoeba keratitis
Note : Acanthamoeba keratitis, or AK, is a rare but serious infection of the eye that can cause
permanent vision loss or blindness. This infection is caused by a tiny ameba (single-celled living
organism) called Acanthamoeba. Acanthamoeba causes Acanthamoeba keratitis when it infects
the cornea, the clear dome that covers the colored part of the eye.
79- Infant suffer from groin rash , spare folds?
Answer: diaper rash dermatitis .
Other names: Napkin dermatitis.
80- White patches after sun exposure treatment?
Antibiotic
Antifungal ( selenium sulfide)
Answer : B
It's pityriasis versicolor ( tinea flava )

smle ,2016

948

81- Patient with acne come with comedones and presence of pustules what is the type of acne
?
Obstructive
Inflammatory
Infectious
Obtrusive
Answer : B
Papules and pustules represent inflammatory acne lesions.
2- Commonest site for Lichen planus is?
Mouth
Abdomen
Answer : A
82- Postcholecystectomy developed parotid enlarge ?
Bacterial sideritis
83- Angular cheilosis deficiency in ?
B12
B6
Answer: B
Angular cheilitis is a common inflammatory condition affecting the corners of the mouth or oral
commissure.
84- Patient with lower limb weakness and sensation also angular stomatitis ?
Vit B3 niacin
Vit B1 thiamin
Answer: Vit B1 thiamin Beri Beri
85- Non pruritic pink eruption of the right foot no scales no history of infection?
Answer : Granuloma annulare
86- Question about Hives ( urticaria ).
http://acaai.org/allergies/types/skin-allergies/hives-urticaria
87- Contact with moulds in a new apartment her son develop a Rash in his hand and resolve
completely, After a while he develop multiple rash?
Maculo papular
wheal (urticaria)
Plaque
Cup?
Answer: B

smle ,2016

949
88- Man live in desert present with skin lesion on forearm ,microscope show donovani what
is treatment?
Leishmaniasis
http://emedicine.medscape.com/article/220298-treatment#d11

89- Lady with spot of hair loss over the scalp with normal underlying skin, what's the Dx?
Answer: Alopecia areata
Alopecia areata is a recurrent non scarring type of hair loss that can affect any hair-bearing area
and can manifest in many different patterns.
The presence of smooth, slightly erythematous (peach color) or normal-colored alopecic patches
is characteristic.
Referance: http://emedicine.medscape.com/article/1069931-clinical#b4
90- Patient with macular hypopigmentation and no history of chronic disease ?
Vitiligo
Albinism
Psoriasis
Melanoma
Answer: ??A
91- Hypopigmentation and loss of sensation in forearm with ulnar nerve thickness?
Answer: Leprosy
92- Patient develop 2 cm dome shape mass in the dorsum of the hand , it's cover by keratin
What is the diagnosis ?
Answer: Keratoacanthoma
Medscape: Lesions typically are solitary and begin as firm, roundish, skin-colored or reddish papules that rapidly progress to dome-shaped nodules with a smooth shiny surface and a central crateriform ulceration or keratin plug that may project like a horn. Most keratoacanthomas occur on
sun-exposed areas. The face, neck, and dorsum of the upper extremities are common sites.
93- Patient has family history of allergy has scaling -skin , itching in the face and antecubital
fossa , what is the diagnosis ?
Answer: Atopic eczema
94- Child has itching and all student in his class got the same infection?
Answer : Scabies
95-Adult male on multiple drug came with violaceous maculopapular lesion in the trunk
Diagnosis?
Erythema multiforme
Toxic epidermal necrolysis
Answer : it could be toxic epidermal necrolysis .

smle ,2016

950

96- Female with pustules on her face, which type of acne is this?
Inflammatory.
Infectious.
Answer: A
From toronto note derma: inflammatory type 3 .
Type 1: comedonal, sparse, no scarring .
Type 2: comedonal, papular, moderate -+ little scarring.
Type 3: comedonal,papular, and pustular with scarring.
Type 4: nodulocystic acne, risk of severe scarring.

97- Case of lichen planus.


6 P's : Purple Pruritic Polygonal Peripheral Papules Penis (i.e. mucosa) small, polygonal, pruritic,
flat-topped, shiny, violet papules; resolves into hyperpigmented macules.
Common sites: wrists, ankles, mucous membranes in 60% (mouth, vulva, glans), nails, scalp
,distribution: symmetrical and bilateral
Wickhams striae: reticulate white-gray lines over surface; pathognomonic but may not be present
Mucous membrane lesions: lacy, whitish reticular network, milky-white plaques/papules; increased risk of SCC in erosions and ulcers
Nails: longitudinal ridging; dystrophic; pterygium formation
Scalp: scarring alopecia with perifollicular hyperkeratosis
Spontaneously resolves but may last for weeks, months or years (mouth and skin lesions)
Rarely associated with hepatitis C
Koebner phenomenon
98- Patient with psoriasis, took a medication then developed generalized psoriasis covering
all his body surface, what is the percentage of the involved body surface?
30%.
50%
70%
90%
Answer: D
Pustular psoriasis
http://www.kevinmd.com/blog/2014/05/mksap-64yearold-man-rapidly-spreading-rash.html
99- Case Necrotizing fasciitis treatment?
Imipenem and metronidazole
Ampicillin and gentamicin
Piperacillin and tazobactam
Penicillin and smth
Answer : C+ clindamycin as combination therapy

smle ,2016

951

100- Herpes keratitis of eye, scenario with picture, how to treat?

Answer:
The mainstay of therapy is antiviral treatment either in the form of topical therapy with trifluridine
1% eight to nine times a day or oral administration of acyclovir or valacyclovir for 10 to 14 days.
If trifluridine drops are used, care is to be taken to ensure antiviral drops are discontinued within
10-14 days due to corneal toxicity. Epithelial debridement of the dendrites may also be utilized in
conjunction with antiviral therapy to help reduce viral load. Topical corticosteroids are contraindicated in the treatment of active HSV epithelial keratitis.
http://eyewiki.aao.org/Herpes_Simplex_Virus_Keratitis#Management
101- Child with hairless spot, mother noted she was pulling her hair when stressed, what to
give her?
Lithium
Lorazepam
Other antiepileptic mentioned
Answer: SSRI
Citalopram
Fluvoxamine

smle ,2016

952
Escitalopram
Paroxetine
Sertraline
Fluoxetine
Treatment modalities vary in childhood and adult varieties. Apart from psychotherapy, the drug
treatment involves several agents including selective serotonin reuptake inhibitors (SSRIs) and
domipramine. Trichobezoar/Rapunzel syndrome requires surgical intervention.
102- Case of herpes type 1, what to give?
Oral antiviral
Topical steroids
Answer: A
Reference:
http://emedicine.medscape.com/article/218580-medication#2
http://www.uptodate.com/contents/treatment-of-herpes-simplex-virus-type-1-infection-in-immunocompetent-patients#H26
103- Child with itchy scalp and scales, other classmates affected, Diagnosis?
Tinea capitis
Scabies
Answer : A
104- Case of multiple myeloma
http://emedicine.medscape.com/article/204369-overview#a1
105- HIV PT come with diffuse pasutle in skin and mouth treatment is ?
Topical steroid
Oral AB
Topical AB
Chemo and radiotherapy
Answer: B
106- Case of thickened skin of forearm ask about test to do?
Answer : scl 70 for Scleroderma
107- Hypopigmentation on trunk ?
Antibiotics
Steroid
Selenium
Answer: C
108- Shingles at umbilicus level what is the dermatome ?
Answer: T10
109- Old lady with migraine on medication stable for 3 years went to ophthalmology to treat
blepharitis with eye drops and developed rash big nose and big mouth ?
A . Seborrheic dermatitis

smle ,2016

953
B . Contact dermatitis
C . Rosacea
Answer : B
110- Dew drops on rose petals vaginal lesions, what is the diagnosis ?
Herpes simplex
Syphilis
Chancroid lesion
Herpangina
Answer: A
111- Patient was started on Carbamazepine. Presented with large area of skin peeling with
blisters. He looks toxic. There is eosinophilia. What is the problem here?
Steven Johnson syndrome.
Toxic epidermal necrolysis.
Answer: B
http://emedicine.medscape.com/article/229698-overview#a5
112- Loose cornified fragments of the skin what is it ?
A. Scales
B. Crusts
Answer : A
113- What is the difference between Scale and Crust?
Scale is an accumulation of loose cornified fragments of the Stratum corneum.
Crust is the presence of a dried exudate (serum, blood, pus) on the skin surface.
114- Alopecia and eyelashes fall
Answer: Alopecia totalis
115- Contact with moulds in a new apartment her son develop a Rash in his hand and resolve completely After a while he develop multiple rash ?
Maculo papular
Wheal
Plaque
Cup...
Answer : Question is not clear !
116- DM patient presented with Hx of itching and rash like lesions with white center in
inguinal region but with sparing of the folds (picture) What is the cause ??
Answer: Candida
117- skin lesion "ringworm" which stain should be used ?
Potassium chloride
Answer: Potassium hydroxide
http://www.uptodate.com/contents/dermatophyte-tinea-infections

smle ,2016

954
118- Planus Case with hypopigmentation on the arm ( one area only ) + with symptoms on
it (Can't remember them well it's like paresthesia or something like this but i'm sure it not
vitiligo?
Vitiligo
Leprosy
Answer: B
119- Female with history of lichen sclerosis present with lesion what will you do?
Answer: Take biopsy
120- Fever, headache, with macules, papules, pustules and vesicles over the face, scalp & the
trunk ; what is the causative organism ?
Herpes type 6
CMV
Herpes zoster
EBV
Answer: C
121- Farmer have cyst on his dorsal of his right hand after 4 month his Dom-like cyst have
keratin growth?
BCC
keratoacanthoma
Answer: B

122- Patient with hypopigmented areas on trunk , the same lesion in his uncle , his wife pregnant , what the possible mode of transmission ?
Autosomal trait
X- linked trait .
No dominant trait
Answer: ?

123- Best test to diagnose syphilis ?


Answer: fluorescent treponemal antibody-absorption

124- Rash start in face and then spread ?


Answer: Rubella
125- Measles incubation period (or varicella) can't recall ?
Answer: Measles : 10-14 days
Varicella: 10-21 days
126- HIV had skin lesion violaceous, bx showed spindle , what is the diagnosis?

smle ,2016

955
Answer: kaposi sarcoma
127- Patient came with skin lesion , wood lamp pink, what is the diagnosis ?
fungal infection
Can't recall other choices
Answer: Erythrasma
Corynebacteria bacteria cause a pigmented rash in skin folds that fluoresces a coral-pink colour.
128- Best treatment of Pyoderma Gangrenosum?
Plasmapheresis
Steroid
Oral Antibiotics
Methotrexate
Answer: B
129- Long case of patient with salivary patches?
Answer: Psoriasis
130- Patient with Hx of unprotected sex 8 weeks ago, came with rash all over his body except
the face, what is the organism ?
Syphilis
Chlamydia
Chancroid
Answer : A
131- Black spot at the sole of foot that pared with scalpel, what is the diagnosis?
Verruca
Heloma
Tyloma
Answer: A

132- Patient presented to hospital with 5 days of generalized skin eruption including soles
and palms, what is the cause?
Drug Induced Reaction
Pityriasis
Erythema Nodosum
Erythema Marginatum
Erythema Multiforme
Answer: Most probably A
There is no enough data it could be A or E

smle ,2016

956
Drug eruption : Most commonly maculopapular (95% of cases); common in patients taking allopurinol (Zyloprim), beta-lactam antibiotics, sulfonamides, anticonvulsants, angiotensin-converting enzyme inhibitors, nonsteroidal anti-inflammatory drugs, hypoglycemics, and thiazide diuretics, but can occur with almost any drug; usually appears within 1 to 4 weeks of initiating drug;
key to diagnosis is timing of rash appearance in relation to drug use.
Erythema Multiforme : Round, dusky red lesions that evolve into target (iris) lesions over 48
hours; starts on backs of hands and feet and on extensor surfaces of arms and legs; symmetric;
may involve palms, soles, oral mucous membranes, or lips; key to diagnosis is presence of target
lesions.
133- Rash band like distribution, what is the Diagnosis?
Answer: Shingles
134- Patient with fever, mouth ulcer, and other symptoms, what is the Diagnosis?
HSV 1
HSV 2
Answer: A
135- Patient with flushing of face and amp and neck, which antibiotic she is using ?
No enough data but may be it is Jarisch-Herxheimer reaction
Read about it.
136- Single Red lesion on shoulder that keeps growing?
Strawberry Nevus
Pyoderma gangrenosum
Answer : ??
137- Athlete after exercise presented with hypopigmented lesion what is the treatment ?
Answer: Topical antifungal medications containing selenium
sulfide are often recommended (tinea versicolor)
138- Bee sting leads to erythema and swelling what is the treatment ?
2 Questions with different options.
First step in treatment following a bee sting is removal of the stinger itself. The stinger should be
removed as quickly as possible without regard to method: Once the stinger is removed, pain and
swelling should be reduced with a cold compress. A topical anesthetic containing benzocaine will
kill pain quickly and menthol is an effective anti-itch treatment.Itching can also be relieved by
antihistamine or by a steroid cream.

139- Hypopigmented macule shin on sun exposure how will treat him ?
Answer :
No answers !
May be it is post inflammatory hypopigmentation which mostly resolve spontaneously
140- Diagnosis of penile painless ulcer ?

smle ,2016

957
Answer: Darkfield microscopy
141- Itching in lower limbs , otherwise normal ?
Tinea
Scabies
No enough data , Both answers could be correct ! http://www.ihealthblogger.com/2013/03/ItchyLegs-Causes-Symptoms-Treatment-Prevention.html
142- Well circumscribed lesion on erythematous base , arthritis ?
Answer: Rheumatological disease
143- Patient with skin slightly elevated and mild itching. What's diagnosis?
Answer: Lichen planus
144- Patient with penile lesion that is caused by using sulfa drugs describe the
lesion ?
Erythema
Ulcer
Fixed drug eruptions of the skin blistering
Answer: C
Refrence : http://www.dermnetnz.org/reactions/fixed-drug-eruption.html
145- Boy have lesion on his forearm erythematous and silvery scaling , what is the diagnosis
?
Answer: Psoriasis
146- Fever, malaise , maculopapular rash over the body and behind the ear what is the causative organism ?
A- Rubella
B- Measles
C- Mumps
Answer: B
147- Child who has to have itchy papules, started as 1 papule then spread to the whole body,
what is the treatment?
Steroid
Acyclovir
Antibiotics
Antiseptic
Answer: A
Pityriasis rosea (also known as pityriasis rosea Gibert ) is a skin rash. It is benign but may inflict
substantial discomfort in certain cases. Classically, it begins with a single "herald patch" lesion,
followed in 1 or 2 weeks by a generalized body rash lasting up to 12 weeks. Treatment topical
steroids.
148- Which consider as Atypical moles (dysplastic nevi)?

smle ,2016

958
Irregular border
Smaller than 6 mm
Uniformity of color
Answer : A
Refrence: http://emedicine.medscape.com/article/1056283-clinical#b1
149- Dermatomyositis associated with ?
Generalized morbilliform eruption
Distal muscle weakness
Malignancy
Answer: Malignancy
150- Regarding Retinoid ( for acne ) side effect ?
Answer: Avoid sun exposure
153- Redness and itching between the toes , what is the Diagnosis ?
Answer: Scabies ( sure )
154- Xeroderma pigmentosum defect in?
Answer: DNA break repair gene
155- Picture of patient lower limbs has rash on medial thigh , written that it was pinkish in
color and itchy , and the patient is diabetic?
Answer:
Tinea
cruris

156-Visceral leishmania ?
L.donovani
L.tropica
Answer : A
157-Patient have skin eruption affected all body including palms and I think soles only thats
the details, what's the diagnosis?
Scleroderma
Something medication
smle ,2016

959
Answer: B

Not sure drug eruption is generalized and involve sole and palms

158- 7 years boy soccer player' have hypopigmentation on the trunk and over the arm and
the the pigmentation get lighter with sun what you will give him ?
Oral antibiotic
Topical antibiotic
Topical steroid
......... oxide
Answer: ?
159- Child came with swelling in the scalp with loss of hair Something sebum ?
160- Band of neutrophil under skin?
There is no answers , it could be Leukemia cutis !! CML
There were also missing questions about the following:
Hidradenitis suppurativa of the buttocks.
http://www.nhs.uk/conditions/hidradenitis-suppurativa/Pages/Introduction.aspx
Management of Acne & Scabies

smle ,2016

960

smle ,2016

961

Basic

Embryology
1- Failure of joining renal collecting duct with distal tubules.
smle ,2016

962
Answer: ?
Multicystic Renal Dysplasia?
Reference: http://emedicine.medscape.com/article/982560-overview#showall
Polycystic Kidneys? Reference: Maternal, Fetal, & Neonatal Physiology

2- What is the origin of smooth part of right atrium?


A- Right sinus venosus
B- Left sinus venosus
Answer: A
Sinus venosus (R horn): smooth part of right atrium (sinus venarum) and the "valve" of the superior vena cava. The sino-atrial node.
Sinus venosus (L horn): coronary sinus, valve of coronary sinus.
Reference: http://www.nervenet.org/embryo/hderiv.html

3- Lung Embryology (about alveolar?


Answer: ?
Development of the lung can be divided into two phases, lung growth (structural development)
and lung maturation (functional development). Lung growth can be influenced by a host of physical factors. Lung maturation and the achievement of functionality is primarily a biochemical
process and is under the control of a number of different hormones. Lung growth proceeds
smle ,2016

963
through gestation. There is progressive branching of the airways and finally development of alveolar spaces capable of gas exchange in the last trimester. The surfactant system, composed of
phospholipids that decrease surface tension within the alveoli and prevent alveolar collapse during exhalation, develops in the last trimester, and reaches maturity by approximately 36 weeks.
Lung growth continues after birth as alveolar number continues to increase. The end result of the
development of the lung is an organ with a tremendously large surface area that is approximately
50-100 m2, capable of exchanging oxygen and carbon dioxide across a very thin membrane.
Reference: http://www.columbia.edu/itc/hs/medical/humandev/2004/Chpt12-LungDev.pdf
9th update Our link Qs up to 28th of December
4. Embryology question about inferior vena cava renal segment
posterior cardinal
supracardinal
saccrocardinal
subcardinal
Answer: D
Reference: radiopaedia website
Normal IVC has a complex embryological development with many embryological veins contributing to different parts :
Right vitteline vein: forms supra-hepatic and hepatic segments of IVC
Right subcardinal vein: forms supra-renal segment
Right subsupracardinal anastomosis: forms renal segment
right supracardinal vein: forms infrarenal segment
right posterior cardinal vein: forms distal most IVC and its bifurcation into common iliac veins
5- The embryological origin of tongue?
a lingual
b occipital
Answer: B , Originating in occipital somites .
Reference: Langmans Medical Embryology
http://www.embryology.ch/anglais/sdigestive/gesicht04.html
6. Organ that originate from midgut and hindgut?
A- duodenum
Answer : transverse colon
7.What arises from the caudal part of the foregut and cranial part of midgut?
A- Esophagus
B- Duodenum
Answer: B
All snellen

smle ,2016

964

NEW 12 update
8. Origin of right atrium embryo ?
A- Cordis in heart
B- change of transposition of GA
C-ectopia cordis
Answer: non of the above the origin of the right atrium is Sinus venosus
Primitive heart tube -has 5 dilatations.
1.truncus arteriosus-gives aorta and pulmonary trunk
2.bulbus cordis-smooth parts of rt & left ventricles
3.primitive ventricle-trabeculated parts of rt &left ventrcles.
4.primitive atrium- gives trbeculated parts of rt & left atria.
5.sinus venosus- gives the smooth part of rt atrium, coronary sinus, oblique vein of left atrium
9. Follicular cell of ovary what it is orign embryologically ???
Answer:
Cortical Cords , which split into isolated cell clusters ,with each surrounding one or more primitive germ cells . Germ cells subsequently develop into oogonia , and the surrounding epithelial
cells , descenants of the surface epithelium , form Folicular cells .
Reference: Langmans Medical Embryology .
10. failure of 5th branchial arch development with 1st ,2nd and 4th something produce ?
Answer: ???

smle ,2016

965
11.failure of obliteration of pharyngeal arch 2,3,4 lead to ?
A- cervical cyst
B- branchial fistula
C- ectopic thymus
D- Parathyroid
Answer: A

Anatomy
1- Dorsalis pedis pulse location?
A- Beside extensor hallucis longus tendon
Answer: A
Reference: Moore - Clinically Oriented Anatomy, 7th ed, p590
2- Internal Carotid artery branches?
Answer:
There are no branches in the neck. Many important branches are given off in the skull.
Ophthalmic artery
Posterior communicating artery
Anterior cerebral artery
Middle cerebral artery
Reference: Clinical Anatomy By Systems, Richard S. Snell, P180
3- Which part of the spermatic cord originates from the internal oblique abdominal muscle?
A- internal spermatic sheath
B- External spermatic sheath
C- Tunica vaginalis
D- Cremaster muscle
Answer: D
Reference: Clinical Anatomy By Systems, Richard S. Snell, P703-704.
4- The radial pulse can be palpated lateral to which tendon?
A- Flexor carpi ulnaris

smle ,2016

966
B- Flexor carpi radialis
C- Flexor digitorum profundus
Answer: B
Reference: Clinical Anatomy By Systems, Richard S. Snell, P198
5- Posterior vagal trunk supplies:
A- Esophagus
B- Jejunum
C- Descending colon
Answer: A
Reference: Clinical Anatomy By Systems, Richard S. Snell, P718-721
6- What is the position of the femoral vein to the artery?
A- Medial to anterior
B- Lateral
C- Posterior
D- Anterior
Answer: ?
The choices must be wrong, the femoral artery is related posteriorly and medially to the femoral
vein. Posteriorly: the femoral vein intervenes between the artery and the adductor muscle. Medially: it is related to the femoral vein in the upper part of its course.
Reference: Clinical Anatomy By Systems, Richard S. Snell, P242
7- Which of the following muscles initiate unlocking of the knee during walking?
A- Plantaris
B- Tibialis anterior
C- Sartorius
D- Popliteus
Answer: D
Reference: Moore - Clinically Oriented Anatomy, 7th ed, p600-601
8- Which of the following nerves is responsible for adduction of fingers?
Answer: deep branch of ulnar nerve.
Reference: Clinical Anatomy By Systems, Richard S. Snell, P481
9- Peroneal artery (right fibular) is a branch of which artery?
A- femoral
B- popliteal
Answer: Popliteal artery > Posterior tibial artery > Peroneal artery.
Reference: Clinical Anatomy By Systems, Richard S. Snell, P248-255
10- Which nerve supplies the frontal belly of the occipitofrontalis muscle?
Answer: Temporal branch of facial nerve (CN VII).
Reference: Moore - Clinically Oriented Anatomy, 7th ed, p845
11- What is the narrowest part of the urethra?

smle ,2016

967
A- Prostatic urethra
B- Membranous urethra
C- External meatus
Answer: C
The narrowest part of the urethra is the external meatus.
Membranous urethra is the second narrowest part.
Prostatic urethra is the widest part.
Reference: Clinical Anatomy By Systems, Richard S. Snell, P819-822
12- Where does the lymph drained from the testicles?
A- Superficial inguinal
B- Deep inguinal
C- Para-aortic
D- Thoracic duct
Answer: C
Reference: Clinical Anatomy By Systems, Richard S. Snell, P278
13- What is the type of joint between the bodies of vertebra?
A- Synovial
B- Cartilaginous
C- Suture
Answer: B
Reference: Clinical Anatomy By Systems, Richard S. Snell, P 386
14- What is the blood supply to the posterior compartment of leg?
A- Tibial
B- Common fibular
C- Superficial fibular
Answer: A
Reference: Moore - Clinically Oriented Anatomy, 7th ed, p602
15- Which nerve is responsible for gag reflex?
Answer: CN IX and CN X, glossopharyngeal afferent, vagus efferent.
When the posterior part of the tongue is touched, the individual gags. CN IX and
CN X are responsible for the muscular contraction of each side of the pharynx. Glossopharyngeal branches provide the afferent limb of the gag reflex.
Reference: Moore - Clinically Oriented Anatomy, 7th ed, p949
16- Which nerve is responsible for tongue movement:
Answer: Hypoglossal.
Reference: Grays anatomy 2nd ed, ch 8, p 1043.
17- Patient with medial loss of sensation of one and half finger (Palmar and dorsal surfaces). Which nerve is affected?
A- Median N
B- Ulnar N
C- Axillary N

smle ,2016

968
D- Radial N
Answer: B
Reference: Grays anatomy 2nd ed, ch 7, p 772.
18- Muscle passing through the lesser sciatic foramen:
Answer: ?
The following pass through the foramen:
- The tendon of the Obturator internus
- Internal pudendal vessels
- Pudendal nerve
- Nerve to the obturator internus
Referance: https://en.wikipedia.org/wiki/Lesser_sciatic_foramen
19- In which scalp layer does vessels run:
Answer: Connective tissue (superficial fascia).
Reference: grays anatomy 2nd ed, ch 8, P 873.
20- Origin of gluteal artery ( scenario: ischemia in gluteus area then ask for origin of gluteal artery):
Answer: Internal Iliac artery.
Reference: grays Anatomy.
21 - Which ligament pass inside inguinal canal
A- broad ligament
B- round ligament
C- uterosacral ligament
D-transverse cervical ligament
Answer: B
The inguinal canal is a tubular structure that runs inferomedially and contains the spermatic cord
in males and the round ligament in females
Reference: medscape.
22 - inferior alveolar n from facial nerve injury, during dental procedure? What's the manifestation?
Answer: The main symptoms for the inferior alveolar nerve injury are: sensory paralysis of the
lower lip on the affected side, the mental region and the gingivae; stiffness, persistent pain; neuropathic pain such as allodynia; and pain and discomfort with occlusion.
Reference: http://www.aqb.jp/english/file/ClinicalpracticePart3-6.pdf
23 - Surgery in posterior triangle, then develop loss of sensation in lower mandible ipsilateral which nerve is affected?
A- lesser occipital nerve
B- greater occipital nerve
C- great auricular nerve
D- Dont remembers
Answer: C
smle ,2016

969
Reference:
Grays anatomy 2nd ed, ch 8, p 903.
http://www.nysora.com/files/2013/chapter-11/pic2.gif
24 - Knee trauma then can not fully extend the knee so which muscle is affected
A- Quadriceps femoris
B- Biceps femoris
(Others I don't remember)
Answer: A
Extensors: quadriceps femoris
Reference: Greys anatomy
25-Patellar knee jerk reflex maintained by:
A- L1-L2
B- L2-L3
C- L3-L4
D- L5-S1
Answer: C
Reference: USMLE step 1
26 - Gluteal muscle what the nerve intervention :
Answer:
1- Gluteus Maximus >> inferior gluteal nerve .
2- Gluteus Medius >> Superior gluteal nerve.
3- Gluteus minimus >> Superior gluteal nerve.
Reference: http://teachmeanatomy.info/lower-limb/muscles/gluteal-region/
27 - brain tumor compressed third portion of maxillary artery. Which of following artery
will affect it:
A-Temporal artery
B- sphenopalatine artery
Answer: B
Reference: Grays Anatomy.
28- Question about the thyroid gland and fascia related..
A-deep cervical fascia
Answer: A
Neck fascia:
Superficial fascia >> contains platysma Ms
Deep cervical fascia : 4 layers :
- investing layer >> surrounds all neck structures.
- prevertebral layer >> surrounds vertebral column + deep Ms associated with the back.
- pretracheal layer >> surrounds trachea, esophagus, thyroid gland.
- carotid sheath >> surround the 2 major neurovascular bundle (common carotid A,
internal carotid A, internal jugular V, vagus N).
Reference: grays anatomy 2nd ed, ch 8, p 948.

smle ,2016

970

29.Patient has trouble dorsi flexing his ankle joint which structure is affected (1 question nerve and 1 muscle)
- Muscle affected :
A- Ant tibialis.
B- Extensor digitorum longus.
C- Extensor hallucis longus
D- Peroneus tertius
Answer: A
2nd opinion : ( not sure since all choices are ms in the ant compartment of the leg so all cause
dorsiflexion, try to look for another movment problem :
Ant tibialis : dorsiflexion + inversion + medial arch dynamic support.
Extensor hallucis longus: + extension of great toe + dorsiflexion.
Extensor digitorum longus: extension of lateral four toes + dorsiflexion.
Peroneus( fibularis) tertius: eversion + dorsiflexion.
If no other movement affection I think A is the main Ms for dorsiflexion)
-Nerve affected:
Answer: Deep peroneal ( fibular) N.
All the leg ant.compartment Ms is innervated by: deep Fibular N.
Reference: grays anatomy 2nd ed, ch 6(lower limb), p 598.
30- Which of the following is not supplied by the inf. Mesenteric artery:
A- splenic flexure.
B- descending colon.
C- transverse.
D- cecum
Answer: D
Reference: greys anatomy.
32- artery in prepare inguinal hernia:
Answer: testicular, cremasteric, and deferential A.
33- 3rd layer of scalp:
Answer: Epicranial Aponeurosis
The scalp consists of 5 layers (seen in the image below):
1- S : the skin.
2- C : connective tissue= superficial fascia
3- A : epicranial aponeurosis = galea aponeurotica
4- L : loose areolar tissue
5- P : pericranium.
Reference: Medscape.
34- tabia move for femur which ligament ?
Answer: ACL?
(No enough information in the Q)

smle ,2016

971
35- medial 1/3 loss of sensation in hand, where is the deformity ?
Answer: ulnar n.
Reference: grays anatomy 2nd ed, ch 7 ( upper limb), p 772.
36- Unstable gate which artery response ?
Answer: if the gait hes talking about is waddling gluteus medius and minimus affected
superior gluteal A.
37- which ligament will be tearing during LP ?
Answer: Penetrating layers in order:
1- Skin
2- Facia and SC fat
3- Supraspinous ligament
4- Interspinous ligament
5- Ligamentum flavum
6- Epidural space and fat (epidural anesthesia needle stops here)
7- Dura
8- Subarachnoid membrane into subarachnoid space.
Reference: http://www.oxfordmedicaleducation.com/clinical-skills/procedures/lumbar-puncture/
38. Accessory nerve present in which triangle?
A- Posterior
B- Mental
C- Mandibular
D- Muscular
Answer: A ( Posterior cervical triangle).
Reference: grays anatomy 2nd ed, ch8 (head and neck), p 968-973.
39- Phrenic nerve paralysis:
A. increased heart rate
Answer:
Clinical manifestations: Patients with unilateral diaphragmatic paralysis are usually asymptomatic at rest, but may have exertional dyspnea and decreased exercise performance . However,
patients with underlying or intercurrent lung disease may experience dyspnea at rest. Orthopnea
can also occur, but is not as intense as with bilateral diaphragmatic paralysis. Unilateral diaphragm paralysis may also be associated with sleep-disordered breathing during rapid eye movement sleep
Reference: uptodate.
40- during laparoscopic surgery of inguinal hernia you find artery superficial going upward ?
Answer: Inferior epigastric artery.
41- Popliteal artery branch of?
A. Peroneal

smle ,2016

972
Answer: Popliteal artery is the continuation of Femoral a external iliac common iliac
abdominal aorta
Reference: grays anatomy 2nd ed, ch 6 (lower limb), p 585.
42- Injury to temporal area and superficial temporal artery bleed which layer of the skull where vessels are ?
A- pericranial.
B- epicranial aponeurosis.
Answer: Subgaleal hemorrhage? galeal aponeurosis (epicranial aponeurosis).
2nd opinion: the superficial temporal artery is in the connective tissue layer as well as other arteries. (not skull)
Reference : http://emedicine.medscape.com/article/834808-overview#a3
43- upper outer mass in breast skin look like bekkering what is the cause ?
A- pectoralis major.
B- crourp ligament.
Answer: Cooperligment.
I think he means puckering >> dimpling and puckering are due to contraction of ligament of
cooper.
Reference: Passing the USMLE: Basic Science.
44.Pt after RTA , no abduction and lateral rotation of the arm.. . What is the origin of the
affected nerve ?
A- medial plexus
B- lateral plexus
C- lower plexus
D- root
Answer: C ??
Deltoid M ..action : abducts arm ( 18 - 90 degree) teres minor M .. Action :- lateral rotation of
arm. Both supplied by axillary nerve from posterior it is a part of the brachial plexus . It consists
of contributions from all of the roots of the brachial plexus ( trunks )
Also, the upper trunk give suprascapular N which supply supraspinatus and infraspinatus
Supraspinatus action :- abducts of arm
Infraspinatus action :- lateral rotation of the arm
Negative Apleys scratch test - Suprascapular nerve (innervate supraspinatus and infraspinatus)
and it is a branch of the upper trunk.
Reference: Snells Clinical Anatomy.
45. what is the level of cardiac notch?
A- 3rd
B- 4th
C- 5th
D- 6th rip
Answer: B
Reference: Handbook of Cardiac Anatomy, Physiology, and Devices, page 58

smle ,2016

973
46. where do feel the pulse of factual artery?
Answer: located on the mandible (lower jawbone) on a line with the corners of the
mouth.
Reference: (wiki).
47. Action of Anterior compartment of the forearm muscle :
Answer: movement of the wrist, flex finger including the thumb and pronation.
Reference: Grays anatomy 2nd ed, ch 7( upper limb), P 736.
48. Pt with right arm numbness and tingling in thumb and index fingers, symptoms increased with hands raised up. The cause:
A- Thoracic inlet.
B- Thrombus
Answer:
I think its Thoracic outlet syndrome: neck, shoulder, and arm pain, numbness in the fingers, or
impaired circulation and flushed sensations to the extremities (causing discoloration). The involved upper extremity can feel weak. Often symptoms are reproduced or worsened when the
arm is positioned above the shoulder or extended.Pains can extend to the fingers and hands,
causing weakness.{ I do not think that its C6 compression ( thumb and index >> C6 dermatome)
because the pain should be relieved not worsened by lifting the arm over and behind the head}.
Reference: http://www.medicinenet.com/thoracic_outlet_syndrome/page2.htm
49. trauma to the fibular head what is the nerve injured?
A- common peroneal N.
Answer: A
Reference: http://www.nytimes.com/health/guides/disease/common-peroneal-nerve-dysfunction/overview.html
50.After surgery ( pregnant after delivery, with epidural anesthesia)? , pt loss of sensation
of medial thigh, which nerve is affected?
A- Femoral
B- Obturator
Answer: B {obturator ( L2 - L4 )}
51.Which ligament comes before you reach epidural space?
A- Flavum
Answer: A
Reference: http://www.oxfordmedicaleducation.com/clinical-skills/procedures/lumbar-puncture/
52. Indirect inguinal hernia weakness occur in which muscle ?
A- transverse facialis.
Answer: A
The protruding peritoneal sac enters the inguinal canal by passing through the deep inguinal
ring>> which sometimes referred to as defect or opening in the transversalis fascia.
Reference: Grays anatomy 2nd ed, ch 4, P 284 + 288.

smle ,2016

974
53.stab wound lateral to the middle of posterior border of sternocleidomastoid. Patient unable to do adduction of the shoulder and elevation over his head . What is the nerve injured?
A- long thoracic
B-Axillary nerve
Answer: A
54. fecal incontinence which level is affected ?
A- above C2
B- below C2
C- above T12
D- below T12
Answer: D (Agree)
Reference: http://eprimarycare.onf.org/NeurogenicBowel.html
Additional Questions ( 8th update)
55.Muscle responsible of internal rotation of left lower leg:
A- maximums
B- gluteus medius or minimus
C- rectus femurs
Answer: B (agree)
Reference: http://www.physio-pedia.com/Hip_Anatomy
56-pts injury at gluteal area affect walking which nerve affected:
A- sup. gluteal n.
B- infer gluteal n.
C- obturator n.
Answer: A (Agree)
Gluteal area ( gluteus medius and minimus) by Superior gluteal n.
injury to superior gluteal nerve will use positive Trendelenburg sign which affect walking. Reference: First aid step 1 (musculoskeletal chapter)
57.after trauma , loss of sensory sensation in the foot ( big toe and 2nd toe)+ drop foot .
Which nerve is affected:
A- saphenous nerve
B- deep peroneal nerve
C- pudendal nerve
Answer: B
Deep peroneal nerve :
Motor: Ms of the ant.compartment of the leg + extensor digitorum brevis + first 2 interossei Ms
Sensory: skin between great and 2nd toe.
Reference: Grays anatomy 2nd ed, ch 6(lower limb), P 599.
58.pressure on lesser omentum , what artery is compressed ?
Answer: Hepatic artery (agree)
Reference: First aid step 1 (GI chapter)
smle ,2016

975

59.name of triangle between hyoid bone an two anterior belly of diagastric ?


A. Submental triangle
B. Submandibular
C- carotid
Answer: A (Agree)
Boundaries of submental triangle of the neck:
-Inferiorly: Body of hyoid bone
-Laterally: Right and left anterior bellies of digastric Floor: Two mylohyoid muscles.
-Apex: At inferior end of the symphysis menti.
Reference: http://www.wesnorman.com/lesson5.htm
60.44 years lady with hyperlipidemia C/O of gluteal muscle atrophy ,what is the most likely
artery that affected by atheromatous plaque?
A- internal iliac artery
B- internal pudendal
c- external iliac
I dont remember other choices
Answer: internal iliac artery (am not sure)
(Agree) superior gluteal artery is a branch of internal iliac artery
Reference: http://teachmeanatomy.info/pelvis/vasculature/arterial-supply/
61.Mountain climber who has hypoxia, which of the following liver zones is most affected
by hypoxia ?
A- Central of acini zone II
B- Peripheral of acini zone II
C- Sinusoidal
Answer: zone 3 (Agree)
-zone1:peripheral zone: affected 1st by viral hepatitis.
-zone2: intermediate zone
-zone3: pericentral vein (centrilobular zone)
affected 1st by ischemia, contain P-450 system, most sensitive to toxic injury, site of alcoholic hepatitis
Reference: First aid step 1 (GI chapter)
62- long thoracic nerve is damaged, winged scapula is the result. Where does the long thoracic nerve originate from:
A- Root C5,C6,C7
Answer: A (Agree)
Reference: first aid step 1.
63.Loss of sense of smell which lobe affected ?
A- Frontal
B- Occipital
C- Partial
D- Temporal
smle ,2016

976
Answer : D
Reference: UQU , ENT chapter
2nd opinion: Cannot answer depend on this information:
-Frontal lobe injury is often associated with damage to the olfactory bulbs beneath the frontal
lobes. Patients may note reduced or altered sense of smell.
Reference: http://www.braininjury.com/symptoms.shtml
-the smell and taste center in the middle part of the temporal lobe
https://www.merckmanuals.com/home/ear,-nose,-and-throat-disorders/symptoms-of-nose-andthroat-disorders/overview-of-smell-and-taste-disorders
64- Indirect hernia sac relation to cord?
A- Anterior medial
B- Anterior lateral
C- Posterior medial
D- Posterior lateral
Answer: Anteromedial Recall

9th update .. Our link Qs up to 28th of December

65. For LP you will insert the needle in:


A- L1- L2
B- L3 - L4
C- L5- S1
Answer: B (Agree)
L3/4 or L4/5 interspace uptodate
Reference: http://emedicine.medscape.com/article/80773-technique
66- 3rd branch of maxillary artery :
Answer: Sphenopalatine (Agree)
The maxillary artery, the larger of the two terminal branches of the external carotid artery, arises
behind the neck of the mandible, and is at first imbedded in the substance of the parotid gland; it
passes forward between the ramus of the mandible and the sphenomandibular ligament, and then
runs, either superficial or deep to the lateral pterygoid muscle, to the pterygopalatine fossa. It
supplies the deep structures of the face, and may be divided into mandibular, pterygoid, and pterygopalatine portions. Sphenopalatine artery is from third portion.
Reference: https://en.wikipedia.org/wiki/Maxillary_artery
67. Patient was running and then felt pain in his left leg. The pain gets better with stretching the leg What is the muscle affected:
A- Posterior tibial
B- Gastrocnemius
C- Soleus
Answer: B (Agree)
Calf strains are most commonly found in the medial head of the gastrocnemius.
smle ,2016

977
Reference: http:// www.ncbi.nlm.nih.gov/pmc/articles/PMC2697334/
68. Child on picnic with family, presented to the ER with high suspicion of foreign body inhalation. The most common location in the chest will be in?
A- Right Main Bronchus
B- Left Main Bronchus
C- Carina of Trachea
D- Inlet of Larynx
Answer: A (Agree)
Reference: first aid step 1 (respiratory chapter)
69. Pt c/o pain in 2nd digit, which tendon affected?
Answer: ??? ( Q is not completed)
70.Blood supply in the hand, what make ant arch ?
Answer (Agree and revised) :
-Superficial Arch: predominant supply is ulnar artery. minor supply from superficial branch of
radial artery.
-Deep Arch: predominant supply is the deep branch of the radial artery. Minor supply from the
deep branch of the ulnar artery.
Reference: http://www.orthobullets.com/hand/6007/blood-supply-to-hand
71.Patient present with signs of increased intracranial pressure: we will do CT scan to confirm it. What nerve examination can help in diagnosis:
A- Optic nerve
B- facial nerve
Answer :
ICP leads to CN III and CN VI deficits (CN 3 & CN 6)
Reference: FA Step 2 CK- 8th Ed.
(not sure)
72.Loss of sensation around the mouth and mandible, which nerve affected:
A- trigeminal nerve
B- facial nerve
Answer: A (Agree)
Reference: Grays anatomy 2nd ed, ch8, P 865-866.
73. Absent gag reflex, loss of sensation posterior of the tounge:
A- Glossopharyngeal nerve
B- vagus nerve
C- facial nerve
Answer: A (Agree)
Reference: First aid step 1 (neurology chapter)
74.Muscle responsible for plantar flexion?

smle ,2016

978
A- rectus femoris
B- quads
C- plantaris
Answer: C (Agree)
Reference : BRS gross anatomy
http://www.ideafit.com/fitness-library/ankle-joint-anatomy
75.Patient who cannot flex his knee, absent ankle reflex, where is the level of injury?
A- L4-5
B- L5-S1
Answer: B
Reference: BRS gross anatomy
(Not sure)
76-Pain or numbness of 4 first fingers of hand. Which nerve affected?
A- Ulnar
B- Median
C- Radial
D- brachial plexus
Answer: B (Agree)
-Median. If the last finger and a half (pinky) then it would be ulnar. Main action of anterior muscles of forearm? Answer: Flexion and pronation
Reference: first aid step 1.
77-Male pt complain of weakness in flexion of both Rt knee and Rt hip which muscle affected?
A- Sartorius
Answer: A
Hip flexion + knee flexion (Sartorius), knee flexion + hip extension ( hamstring muscle ) , knee
extension + hip flexion (quadriceps)
Reference: Grays anatomy 2nd ed, ch 6 (lower limb), P 561.
78-Patient complains of severe chronic constipation. Which muscle is important to be relaxed?
A- Puborectalis
B- Pubococcygeus
C- Iliococcygeus
Answer: A
Paradoxical (Nonrelaxing) puborectalis syndrome: At rest, the puborectalis is contracted and
pulls the rectum forward; creating a sharp angle in the rectum that helps to prevent passive leakage of stool. During the normal process of defecation, the puborectalis reflexively relaxes and
straightens out, allowing stool to pass more easily through the rectum into the anal canal. Paradoxical puborectalis syndrome occurs when the muscle does not relax In some cases, it actually
contracts harder, creating an even sharper angle in the rectum, resulting in difficulty emptying
the rectum, a term sometimes referred to as obstructed defecation.
Reference: https://www.fascrs.org/patients/disease-condition/pelvic-floor-dysfunction-expanded-version
smle ,2016

979

79-What is the type of the pelvic bone:


A- Pivot
B- Ball and socket
Answer: ???
80-The Ankle joint consists of what bones?
A- Distal tibia, fibula, and talaus.
B- Lateral malleolus, medial malleolus, and talaus.
C- Calcaneus, tibia, and fibula.
Answer: A (Agree)
Reference: BRS gross anatomy
http://www.scoi.com/specialties/anatomy-ankle
81-Which ligament prevent uterine prolapse?
A- Round ligament
B- Broad ligament
Answer: ???
Prolapse happens when the ligaments supporting the uterus become so weak that the uterus cannot stay in place and slips down from its normal position. These ligaments are the round ligament, uterosacral ligaments, broad ligament and the ovarian ligament. The uterosacral ligaments
are by far the most important ligaments in preventing uterine prolapse.
2nd opinion:
Uterus is directly supported by:
1- the cardinal or transcervical ligaments
2- uterosacral ligaments.
3- two relatively weak round ligaments
3- the broad ligaments: They attach lateral sides of the uterus to the pelvic wall (preventing side
to side uterine movement but not vertical support)
The round and broad ligaments have less significant support.
Reference:
http://www.zen104556.zen.co.uk/Medicine/ObsGyn/Gynaecology/Surgery/UVProlapse.html
Oxford American Handbook of Obstetrics and Gynecology, ch 17, p 598 :
https://books.google.com.sa/books?id=rIfK0fbe6Q8C&pg=PA598&lpg=PA598&dq=uterova
ginal++prolapse+broad++ligaments+weakness&source=bl&ots=Ni3iYh7VA&sig=ev4UnbL4Ax-wt3iLPirKB6xH3CU&hl=en&sa=X&ved=0ahUKEwjM5KLO9_HNAhVCJcAKHZCVC7QQ6AEIMTAE#v=onepage&q=uterovaginal%20%20prolapse%20broad%20%20ligaments%20weakness&f=false
82-Patient have blunt in right 4 intercostal space affected:
A- upper loop of lung
B- lower loop of lung
C- horizontal fissure.
D- ?
Answer: C

smle ,2016

980
The horizontal fissure follows the 4th intercostal space from sternum until it meets the oblique
fissure it crosses rib 5.
Reference: Grays anatomy 2nd ed, ch 3, P 166.
83-Nerve to gluteus:
A- Femoral
B- Popliteal
C- ......Obturator
D- .....Obturator
Answer: ???
Gluteus maximus: inferior gluteal nerve.
Gluteus medius + gluteus minimus: superior gluteal nerve.
Reference: Grays anatomy 2nd ed, ch 6, P 548.
84-Stand on toes ..which nerve ?
A-Tibial n .
Answer: A
Standing on toes (plantar flexion) is performed by: gastrocnemius, plantaris and soleus, all three
muscles are innervated by tibial nerve
Reference : Grays Anatomy-The Anatomical Basis of Clinical Practice - 41st Edition (2015)
Ch 83 p 1409-1410
85-about kidney vein that drain to IVC (Part of kidney)?
answer:
-Both right and left renal veins drain into IVC.
-Right suprarenal vein drains into IVC.
-Left suprarenal vein drains into Left renal vein.
Reference: Gray's Anatomy-The Anatomical Basis of Clinical Practice - 41st Edition (2015)
Ch 74, p 1244.
86-Superior rictus muscle >> eye movement:
A- In down
B- Out down
C- Up in
D- Up out
Answer: C
action of SR at the eyeball >> (elevation)Up + In (adduction)
movement of the eye when testing the muscle >> Up + out
Reference: Grays Anatomy-The Anatomical Basis of Clinical Practice - 41st Edition (2015)
Ch 41 , p 671.
87-Femoral neck his leg was rotated laterally which muscle responsible:
A- Rectus femoris
B- Gluteus maximus
Answer: B
Gluteus maximus is continuously active in strong lateral rotation of the thigh.

smle ,2016

981
Reference: Grays anatomy
2nd opinion: Femur often positioned in flexion and external rotation (due to unopposed iliopsoas)
Reference: http://radiopaedia.org/articles/femoral-neck-fracture
88-What is the artery supply posterior inferior nasal septum ?
Answer: Sphenopalatine artery
The sphenopalatine branch of the maxillary artery supplies the mucosa of the turbinates, meatuses and posteroinferior part of the nasal septum
Reference: Grays Anatomy-The Anatomical Basis of Clinical Practice - 41st Edition (2015)
Ch 33, p 563.
89-Pt with forehead multiple fractures which nerve is involved ?
A- optic
B- olfactory
C- ophthalmic
Answer: A?
Traumatic optic neuropathy (TON)
Closed head injury may damage the optic nerve, usually affecting the intraorbital or intracanalicular segments
Reference: http://www.uptodate.com/contents/optic-neuropathies?source=search_result&search=optic+neuropathy&selectedTitle=1~150#H23
90-Pt with breast cancer and axillary LN involvement ..what muscle might be involved ?
A-Pectoralis major
B-pectoralis minor
C-latissimus dorsi
Answer: B?
Axillary nodes receive more than 75% of the lymph from the breast. Surgically, the nodes are
described in relation to pectoralis minor.
Reference: Grays Anatomy-The Anatomical Basis of Clinical Practice - 41st Edition (2015)
Ch 53 p 948
91-Patient with pain in hands and fingers during typing, decrease of blood flow to which of
the following will cause this pain?
A- Radial
B- Ulnar
C- Posterior interosseous
D- Anterior interosseous
Answer: B Ulnar (superficial palmar arch)
92-If there was inferior mesenteric artery thrombosis. Which artery will not be affected?
A- descending colon.
B- sigmoid.
C- splenic.
D- cecum.
smle ,2016

982
Answer: C or D??
Inferior mesenteric artery supplies the distal third of the transverse colon, descending and sigmoid colon, rectum and upper anal canal (hindgut derivatives)
Reference: Grays Anatomy-The Anatomical Basis of Clinical Practice - 41st Edition (2015)
Ch 66 p 1138
The caecum receives its supply from branches of superior mesenteric artery, and the splenic artery arises from the coeliac trunk.
So both C and D are correct unless "splenic" refers to the splenic flexure, in that case D would be
the right answer.
93 -What type of radial nerve injury ?
A- neuropraxia
B- neurotmesis
C- axonotmesis
Answer: A
Radial nerve injury can be any of these three !
TYPES OF NERVE INJURY:
There are three major types of nerve injury based upon the severity of nerve disruption and loss
of axonal continuity: neuropraxia, axonotmesis, or neurotmesis. Each type has its own healing
process, consequences, and management.
Neurapraxia Neurapraxia is segmental demyelination without interruption of axonal continuity. It is the most superficial type of nerve injury and may be caused by ischemia, blunt
trauma, compression, or traction. Neurapraxia causes a temporary conduction block. Wallerian degeneration (degeneration of the axon and myelin sheath distal to the injury site)
does not occur. Spontaneous recovery within weeks to a few months is the rule.
Axonotmesis In axonotmesis, the axon is disrupted and Wallerian degeneration occurs.
Axonotmesis is commonly seen in crush and stretch injuries. Functional recovery is expected through axonal regeneration. The Schwann cells and an intact endoneurial tube
play a key role in the regeneration process. Recovery is generally achieved over several
months.
Neurotmesis Neurotmesis usually involves complete nerve transection, including myelin,
axon, and various layers of connective tissue, with ensuing Wallerian degeneration.
Spontaneous recovery is unlikely without surgical repair . The risk of synkinesis (involuntary activation of a muscle when another muscle is activated) is highest with neurotmesis because of loss of nerve conduit integrity, which normally guides accurate nerve regeneration.
Reference: http://www.uptodate.com/contents/anatomic-danger-zones-in-cutaneous-surgery-ofthe-head-and-neck?source=machineLearning&search=neuropraxia&selectedTitle=1~1&sectionRank=1&anchor=H9766775#H9766775
94-loss of sensation of anterior 2/3 of tongue which nerve affected?
A- facial
B- trigeminal
C- glossopharyngeal
D- hypoglossal
Answer: B

smle ,2016

983
Lingual nerve arises from the mandibular division of trigeminal nerve
The nerve of general sensation to the anterior two-thirds is the lingual nerve. Taste sensation
travels in the chorda tympani branch of the facial nerve. The nerve supplying both general and
taste sensation to the posterior third is the glossopharyngeal nerve.
Reference: Grays Anatomy-The Anatomical Basis of Clinical Practice - 41st Edition (2015)
Ch 31 p 514
95-External laryngeal nerve action
A- abduct cord
B- adduct cord
C- upper cord area sensation
Answer: B
-The cricothyroid muscle produces tension and elongation of the vocal folds by drawing up the
arch of the cricoid cartilage and tilting back the upper border of the cricoid cartilage lamina; the
distance between the vocal processes and the angle of the thyroid is thus increased, and the folds
are consequently elongated, resulting in higher pitch phonation. - This muscle is the only laryngeal muscle supplied by the branch of the vagus nerve known as the external branch of the superior laryngeal nerve
Reference: Grays Anatomy-The Anatomical Basis of Clinical Practice - 41st Edition (2015)
Ch 35 p 597
96-A patient with a stabbed wound to the Gluteus. Examination: The patient tilt to the unaffected side while walking. Which nerve is affected?
A- Femoral N
B- Obturator N
C- Superior Gluteal N
D- Inferior Gluteal N
E- Peroneal N
Answer : C
The Trendelenburg gait pattern (or gluteus medius lurch) is an abnormal gait (as with walking)
caused by weakness of the abductor muscles of the lower limb, gluteus medius and gluteus minimus. People with a lesion of superior gluteal nerve have weakness of abducting the thigh at the
hip. Several dysfunctions can produce a positive Trendelenburg Test, including [6] Superior Gluteal Nerve Palsy Lumbar disk herniation Weakness of gluteus medius Advanced degeneration of
the hip Legg-Calv-Perthes Disease
Reference: http://www.physio-pedia.com/Trendelenburg_Test
In paralysis of the glutei ,medius and minimus (both are supplied by superior gluteal nerve), the
pelvis sinks on the unsupported side when the patient tries to stand on the affected limb. This is
known clinically as Trendelenburgs sign.
Reference: Grays Anatomy-The Anatomical Basis of Clinical Practice - 41st Edition (2015)
Ch 80 p 1358
97-Sparing and dissecting which artery during flap of rectus muscle ?
A- Inferior epigastric
B- Superior epigastric
smle ,2016

984
Answer: A
In free TRAM (transverse rectus abdominis muscle) flaps, the skin and fat of the lower abdomen
are connected via the deep inferior epigastric artery and vein to the blood supply in the axilla
Reference: Sabiston Textbook of Surgery - The Biological Basis of Modern Surgical Practice,
19E (2012) Ch 37 p 875
98- Arterial division of the tibial arteries :
Answer:
Anterior tibial: The named branches of the anterior tibial artery are the posterior and anterior
tibial recurrent, muscular, perforating, and anterior medial and lateral malleolar arteries.
Posterior tibial: The named branches of the posterior tibial artery are the circumflex fibular, nutrient, muscular, perforating, communicating, medial malleolar, calcaneal, lateral and medial
plantar, and fibular arteries.
Reference: Grays Anatomy-The Anatomical Basis of Clinical Practice - 41st Edition (2015)
Ch 83 p 1413
99-Phrenic nerve (preserve) for diaphragmatic operation Where?
A- Ant to anterior scalenus
B- ant to mid scalenus ms
C- Lateral to ant scalenus
Answer: A
Scalenus anterior forms an important landmark in the root of the neck because the phrenic nerve
passes anterior to it
Reference: Grays Anatomy-The Anatomical Basis of Clinical Practice - 41st Edition (2015)
Ch 29 p 451
100- can not extend forearm and wrist? nerve and site of injury?
Answer: Radial nerve injury due to humerus fracture
Extensors of the elbow and wrist are all supplied by radial nerve and its branches
Reference: Grays Anatomy-The Anatomical Basis of Clinical Practice - 41st Edition (2015)
Ch 46, p 786.
101- nerve responsible for elevation of arms:
Answer: Vertical abduction is achieved by
0-15: supraspinatus (Suprascapular N ) , 15- 90: middle deltoid (axillary N) , > 90 : trapezius
(spinal part of accessory N ).
Reference: Grays Anatomy-The Anatomical Basis of Clinical Practice - 41st Edition (2015)
Ch 46, p 786.
102-Obturator nerve supply which muscle?
Answer: Anterior branch supplies adductor longus, gracilis, usually adductor brevis and often
pectineus, Posterior branch supplies obturator externus and adductor magnus.
Reference: Grays Anatomy-The Anatomical Basis of Clinical Practice - 41st Edition (2015) Ch
80 p 1372.
103-Nerve Supply of parotid gland?

smle ,2016

985
Answer:
The sensory innervation is provided by the auriculotemporal nerve deriving from the mandibular
branch of the trigeminal nerve.
The parasympathetic innervation which stimulates the saliva production is carried from the glossopharyngeal nerve to the otic ganglion via the lesser petrosal nerve. From there the parasympathetic postganglionic neurons reach the gland via the auriculotemporal nerve.
Reference: https://www.kenhub.com/en/library/anatomy/the-parotid-gland
104-Strongest ligament of hip prevent injury ?
A- Iliofemoral
B- Ischiofemoral
Answer: A
The thick capsule is reinforced by the three major ligaments: iliofemoral, pubofemoral and ischiofemoral.The iliofemoral ligament is the strongest . also the strongest ligament in the human
body.
Reference:
Grays Anatomy-The Anatomical Basis of Clinical Practice - 41st Edition (2015)
Ch 81 p 1379
https://en.wikipedia.org/wiki/Iliofemoral_ligament
105- artistes cannot abduct his fingers, which ms is affected ?
A- Thenar ms
B- anterior interosseous
C- posterior interosseous
Answer:
The palmar interosseous muscles adduct the fingers towards the middle finger. This is in contrast
to the dorsal interossei, which abduct the fingers away from the middle finger.
the answer is dorsal interossei
Reference:
https://en.wikipedia.org/wiki/Palmar_interossei_muscles
Grays Anatomy-The Anatomical Basis of Clinical Practice - 41st Edition (2015)
Ch 50 p 885
106- During adrenalectomy you injures structure passing anterior to it:
A- Inferior vena cava
B- Duodenum
C- Rectum
D- Pancreas
Answer: A or D depending whether Rt or Lt.
The right suprarenal gland lies posterior to the inferior vena cava, separated from it by only a
thin layer of fascia and connective tissue. A small lower part of the anterior surface of the left suprarenal lies adjacent to the pancreas
Reference: Grays Anatomy-The Anatomical Basis of Clinical Practice - 41st Edition (2015)
Ch 81 p 1194
107- You are palpating artery between symphysis pubis and anterior superior spine
smle ,2016

986
A- Femoral artery
B- Internal iliac
Answer: A
Reference: https://en.wikipedia.org/wiki/Femoral_artery
The femoral artery begins behind the inguinal ligament, midway between the anterior superior
iliac spine and the pubic symphysis
Reference: Grays Anatomy-The Anatomical Basis of Clinical Practice - 41st Edition (2015)
Ch 80 p 1369
108-Someone had a dislocation of mandible and the doctor tried to retract the mandible inferiorly. Which muscle retract the muscle to its normal position?
A- Medial pterygoid
B- Lateral pterygoid
C- Masters
D- Temporalis
Answer : B
Lateral pterygoid has assumed a specialized role in mandibular opening that is mediated by its
horizontally orientated fibres. When left and right muscles contract together, the condyle is
pulled forwards and slightly downwards
Reference: Grays Anatomy-The Anatomical Basis of Clinical Practice - 41st Edition (2015)
Ch 32 p 548
109- muscle stretch thigh and leg ?
A- sartorius
B- biceps femoris
Answer: A
-Sartorius assists in flexing the leg, and the thigh on the pelvis.
Reference: Grays Anatomy-The Anatomical Basis of Clinical Practice - 41st Edition (2015)
Ch 80 p 1362
-The posterior thigh muscles(involving biceps femoris) flex the knee and extend the hip joint.
Reference: Grays Anatomy-The Anatomical Basis of Clinical Practice - 41st Edition (2015)
Ch 80 p 1367
110-On palpation of lateral fornix which organ you will feel?
A- Rectum
B- Ovaries
C- Perineal body
Answer: B
The ovaries, ischial spines, ureters, uterine tubes, and the uterine arteries can be palpated through
the lateral fornices of the vagina.
Reference: Lippincott's illustrated Q & A review of anatomy and embryology 1st Edition
(2011)
Q 35 p 89
111-Injury in 1st layer of palmar foot, which the structure may be affected?
A- Musculocutaneous nerve
smle ,2016

987
B- Abductor hallucis
C- Flexor digitorum longus
Answer: B
Reference: http://www.orthobullets.com/foot-and-ankle/7003/layers-of-the-plantar-foot
Plantar muscles of the foot first layer:
This superficial layer includes abductor hallucis, abductor digiti minimi and flexor digitorum
brevis
Reference: Grays Anatomy-The Anatomical Basis of Clinical Practice - 41st Edition (2015)
Ch 84 p 1441
112-if you take blood sample from thigh how to prevent sciatica nerve?
A- upper medaila
B- lower medial
C- upper lateral
D- lower lateral
Answer: C
Intramuscular injections into the buttock should be avoided to prevent iatrogenic damage to the
sciatic nerve. If the buttock is to be used, a relatively safe area is the true upper and outer quadrant.
Reference: Grays Anatomy-The Anatomical Basis of Clinical Practice - 41st Edition (2015)
Ch 78 p 1332
113- Valgus what ligament responsible about it?
A- Anterior cruciate ligament
B- Posterior cruciate ligament
Answer: MCL injury
The MCL is the most commonly injured knee ligament and valgus stress test is the primary tool
for assessing the integrity of its deep and superficial fibres.
Reference: Special tests in musculoskeletal examination An evidence based guide for clinicians -Ch 6 p 184
114-where we can palpated the facial nerve?
Answer:
Reference: http://emedicine.medscape.com/article/835286-overview
The facial nerve trunk lies approximately 1 cm inferomedial to the tip of the tragal pointer
Reference: Grays Anatomy-The Anatomical Basis of Clinical Practice - 41st Edition (2015)
Ch 26 p 412
115-Where you can palpate the facial artery in the face?
Answer: Facial Artery Pulse can be palpated as the artery crosses the mandible just anterior to
the masseter muscle (anterior inferior angle of masseter muscle against the base of mandible)
Reference: Grays Anatomy-The Anatomical Basis of Clinical Practice - 41st Edition (2015)
Ch 30 p 498
116- Location of saphenous vein:
A- Anterior to medial malleolus.

smle ,2016

988
B- Posterior to medial malleolus.
C- Posterior to lateral malleolus.
D- Anterior to lateral malleolus.
Answer: A
The long saphenous vein (great saphenous vein) ascends immediately anterior to the tibial (medial) malleolus
Reference:
Grays Anatomy-The Anatomical Basis of Clinical Practice - 41st Edition (2015)
Ch 80, p 1370
http://emedicine.medscape.com/article/80393-overview
117- scrotal layer anatomy ?
Answer: The scrotum is composed of multiple layers of tissues, including skin, dartos muscle
and external spermatic, cremasteric and internal spermatic fascia
Reference: Grays Anatomy-The Anatomical Basis of Clinical Practice - 41st Edition (2015),Ch
76 p 1285
118- cranial nerve supply skin of maxilla and mandible:
A- 3
B- 4
C- 5
Answer: C
Reference: grays anatomy 2nd ed, ch 8, p 851-852.
119-Superficial palmar arch artery of :
A- Ulnar
B- Radial
C- Anterior
D- Posterior
Answer: A
Superficial palmar arch: ulnar A.
Deep palmar arch: radial A.
Reference: Grays anatomy 2nd ed, ch 7, p 767-769.
120-most common nerve inj in humerus Fx? not mention which part of humerus fx.
A- radial
B- axillary
C- ulnar
Answer: A
proximal humerus fracture 3rd most common fracture after hip
Reference: Grays anatomy 2nd ed, ch 7, p 724.
121-area of nail fold which nerve supply it :
A- median.
B- ulnar.

smle ,2016

989
C- radius.
Answer: A
Reference: grays anatomy 2nd ed, ch 7, p 773.
122-which nerve supply the pain when you chew ?
A- facial
B- vages
C- auriculotemporal and masseter branch of mandibular nerve
Answer: C ?
123- pt with internal iliac artery injury, which part of body affected :
A- abdominal wall muscle
B- front of thigh muscle
C- bladder
Answer: C
Internal iliac A:
posterior trunk : lower posterior abdominal wall, posterior pelvic wall, gluteal region.
Anterior trunk: pelvic vecira, perineal,gluteal region, the adductor region of the thigh and placenta in fetus.
Reference: Grays anatomy 2nd ed, ch 5, p 471.
124-long scenario about post surgery in lacrimal gland , what is the site to put tube ?
A- Lacrimal duct
B- Lacrimal itchim
C- lacrimal gland
D- puncta
Answer: D
125-There was injury to the head exactly to the jugular foramen ,so which of the following
muscle is intact ?
A- Stylopharyngeus
B- Styloglossus
C- Sternocleidomastoid

Answer: B
Structure passing through the jugular foramen:
1- Glossopharyngeal N: stylopharyngeus Ms
2- Vagus N: palatoglossus, muscles of the soft palate except tensor veli palatini, pharynx except ,
larynx.
3- Accessory N: sternocleidomastoid + trapezius.
4- Internal jugular V
5- inferior petrosal sinus.
(styloglossus innervated by hypoglossus N).

smle ,2016

990
Reference: Grays anatomy 2nd ed, ch 8, p (812)+ (853-854)+(1039).
126-basal skull fracture, injury including the foramen ovale, which muscle will not be affected:
A- sternocleidomastoid
B- styloglossus
C- Trapezius

Answer: I think all correct. See ms intervention in Q above.


More info : Structures passing foramen ovale:
Mandibular n(V3): muscles of mastication ( temporalis, masseter, medial and lateral pterygoid),
tensor tympani, tensor veli palatini, anterior belly of digastric and mylohyoid Ms.
lesser petrosal N: parasympathetic for parotid gland.
Reference: Grays anatomy 2nd ed, ch 8, p 812+852+935.
127-after MVA the patient walk toward the unaffected side :
A- Superior gluteal nerve .
Answer:

128-Loss of taste sensation of the anterior 2/3 of the one side of the tongue and decreased
hearing on the ear on the same side , so where is the level of the injury of the facial nerve ?
A- After the nerve to stapedius muscle
B- Proximal to greater petrosal nerve
C- Distal to greater petrosal nerve
C- (I did not remember)

Answer: D ??
Reference: grays anatomy 2nd ed, ch 8, p852-853.
129-Which of these joint is hinge joint ?
A- Knee joint
B- Ankle joint
C- Elbow joint
D- Hip joint
Answer: A hinge joint is a common class of synovial joint that includes the ankle, elbow, and
smle ,2016

991
knee joints. Hinge joints are formed between two or more bones where the bones can only move
along one axis to flex or extend.
(Hip + shoulder joint is ball and socket)
Reference: http://www.innerbody.com/image_skel07/skel31.html
130-Lymph nodes of breast
A- pectoralis major
B- pectorals minor

Answer: B
Lymph nodes is divided into 3 levels according to its relation pectoralis minor muscle.
Reference: 1st aid for the Surgery clerkship, the breast Ch, p 281.
131-

Orbital mass with loss of sensation over lower eyelid , mandible , maxilla or zygomatic areas which nerve ?
A- Infra orbital
B- Maxillary

Answer: ???
maxillary N (v2) branches:
Zygomaticotemporal: anterior temple above zygomatic arch
Zygomaticofacial: area over zygomatic bone
Infraorbital: lower eyelid, cheek, side of the nose, and upper lip.
The answer is B if mandible area (by mandibular N (v3)) was not mentioned.
Reference: Grays anatomy 2nd ed, ch 8, p 867.
132-

Pain at rt, hand palmar arch test showing insufficient blood flow which could be injured
A- Radial.

smle ,2016

992
B- Ulnar
.
Answer: ??? ( no enough information)
133-child with hydrocephalus progressively increasing in between the 3rd and 4th ventricle
- which area is blocked ?
(I chose cerebral aqueduct)
Answer: Non-communicating hydrocephalus also called obstructive; hydrocephalus occurs when the flow of CSF is blocked along one or more of the narrow passages connecting the
ventricles. One of the most common causes of hydrocephalus is &quot;aqueductal stenosis.&quot; In this case, hydrocephalus results from a narrowing of the aqueduct of Sylvius, a
small passage between the third and fourth ventricles in the middle of the brain.
Reference: http://www.ninds.nih.gov/disorders/hydrocephalus/detail_hydrocephalus.htm
134-the foramen than present between the 3rd and 4th ventricle is?
Answer: aqueduct of sylvius
The third ventricle also communicates with the fourth ventricle through the narrow cerebral (sylvian or mesencephalic) aqueduct.
Reference: http://www.cerebromente.org.br/n02/fundamentos/ventriiii_i.htm
135. At inguinal canal base, what artery you can find?
A- iliac
B- femoral
Answer: if they ask about artery at the base i think its femoral , but if the ask about which artery
pass through the canal the answer will be 3 arteries: artery to vas deferens (or ductus deferens),
testicular artery, cremasteric artery. But if they just asked about the base of the canal the answer
will be >> The Canal Boundaries :The anterior wall is formed by the aponeurosis of the external
oblique, and reinforced by the internal oblique muscle laterally.The posterior wall is formed by
the transversalis fascia.The roof is formed by the transversalis fascia, internal oblique and transversus abdominis.The floor is formed by the inguinal ligament (a rolled up portion of the external oblique aponeurosis) and thickened medially by the lacunar ligament
Reference: http://teachmeanatomy.info/abdomen/areas/the-inguinal-canal/
136- which nerve carries the referred pain of the parotid to the ear?
A- vagus
B- facial
C- auriculotemporal
D- trigeminal
Answer: C or D
C is more accurate .The parotid branches of the auriculotemporal nerve mediate referred otalgia.
Auriculotemporal N branch from > mandibular N (v3) branch of > Trigeminal N.
Reference:
http://www.ajnr.org/content/30/10/1817.full
Grays anatomy 2nd ed, ch 8, p 867.
137-Rt dominant coronary artery, branch of it ?

smle ,2016

993
A- Marginal
Answer:
Right coronary artery
Acute marginal branch (AM)
AV node branch
posterior interventricular artery (PIV) =Posterior descending artery (PDA)
Left Main or left coronary artery (LCA)
Left anterior descending (LAD)
diagonal branches (D1, D2)
septal branches
Circumflex (Cx)
Marginal branches (M1,M2)
Reference: http://www.radiologyassistant.nl/en/p48275120e2ed5/coronary-anatomy-and-anomalies.html
138-Action of superior rectus muscle?!
Answer: elevation, adduction and medial rotation.
Reference: Grays anatomy 2nd ed, ch 8, p 888.
139- disease of spine got anterior chest abscess which nerve carry the infection:
A- lateral cutaneus
B- anterior cutaneus
C- posterior cutaneus
D- diaphragmatic
Answer: B
140. hemiplegic pt come with something in the eye( may be nystagmus or something )where
is the lesion in the Brain:
A- Pons
B- Medulla oblongata
C- internal capsule
D- Midbrain
Answer: ???

smle ,2016

994

Reference:
https://teddybrain.wordpress.com/2013/01/16/what-nystagmus-can-tell-you-in-neurology-clinical-approach-to-nystagmus/
http://patient.info/doctor/nystagmus

smle ,2016

995
141. Thyroid move with swallowing b/c :
A- Pretracheal fascia
B- Carotid sheath
C- Prevertebral fascia
Answer: A
Reference: Grays anatomy 2nd ed, ch 8, P 950.
142-Pt has pain on distal palmar aspect of finger and ask about blood supply ;
Answer: Proper palmar digital artery.
Reference: Grays anatomy 2nd ed, ch 7, p 769.
143-fracture in the humerus and ulna , the pt cant extend his arm , wrist and fingers what
the nerve affected ?
A- Radius nerve in the spiral groove
B- Median nerve in supepichondile region
C- Median nerve in .
D- Ulnar nerve in the medial epicondyle
Answer: A
Reference: Grays anatomy 2nd ed, ch 7, P 724.
144-pt 3 years old came to ER with pronated arm flexed wrist which ligament affected?
Answer: annular ligament ?
I think its pulled (nursemaid's) elbow other name radial head subluxation.
Pulled elbow is subluxation of the radial head into the annular ligament, encountered in young
children (typically between 6 m 3 yrs), as a result of pulling on the arm longitudinally as in pulling a child away from something the parents would rather they not touch, or lifting the child in
play. Children with a pulled elbow will hold the elbow flexed and the forearm in the prone position, unwilling to supinate it.
Reference: http://radiopaedia.org/articles/pulled-elbow-syndrome
145-in examination doctor touch posterior pharynx he want to test which nerve :
A- Vagus
B- hypoglossal
Answer: A
Explanation: Gag reflex:
Afferent >> glossopharyngeal IX (sensory part) + Efferent >> Vagus X(motor part).
Reference: 1st aid.
146- A patient had a dislocation in his jaw, the doctor wanted to retract the mandible to his
position which of the following retracts the mandible in his normal position?
A- temporalis
B- lateral pterygoid
C- medial pterygoid
D- masseter
Answer: A or B ??? ( not sure)

smle ,2016

996
147- femoral neck # his leg was rotated laterally which muscle responsible
A- Rectus femoris
B- Gluteus maximus
Answer: B , Both Bices femoris , obturator externus and Gluteus maximus >> Lateral leg rotation
148- Big and 2ed toe cannot dorsiflexion
A- deep peroneal N.
B- post tibial N.
C- sural N.
Answer: A
Dorsiflexion: by Ms of the anterior leg compartment >> all innervated by deep peroneal N.
Reference: Grays anatomy 2nd ed, Ch 6, p 598
149-Inferior ( temporal) horn of lateral ventricle which affect
A- Hiccup ( I think they mean Hippocampus)
B- Putman
C- caudate nucleus
Answer: C or A ?
Roof: is formed chiefly by the inferior surface of the tapetum of the corpus callosum, but the tail
of the caudate nucleus and the stria terminalis also extend forward in the roof of the temporal
horn to its extremity; the tail of the caudate nucleus joins the putamen.
Floor: the hippocampus, the fimbria hippocampi, the collateral eminence, and the choroid plexus.
Reference: https://en.wikipedia.org/wiki/Lateral_ventricles
150-pt when walking and lifting his Right leg , his left hip drops , the doctor told the or that
he has Trendelenburg sign , which muscle is affected ?
A- Left gluteus medius
B- Rt gluteus Magnus
Answer: right gluteus medius
151-what is posterior to the mid part of inguinal ligament or canal not sure
Answer: femoral a.
If the Q means >> Mid inguinal point: is halfway between the pubic symphysis and the anterior
superior iliac spine. The external iliac A becomes the femoral A as the vessel passes under the
inguinal ligament to enter the femoral triangle.
Reference:
http://teachmeanatomy.info/abdomen/areas/the-inguinal-canal/
Grays anatomy 2nd ed, ch 6, p 540.
152-Patient lost sensation at the level of umbilicus after MVA. the lesion will be at the level
of
A- T4
B- T10
C- T11
D- T 12

smle ,2016

997
Answer: B
Reference: Grays anatomy 2nd ed, ch 4, p 280.
153-Pt had injury to the soles of left foot what is the first structure u can see;
A- tibialis posterior
B- extensor hallucis longus tendon
C- extensor digitorum longus
Answer:
- first thing when u open soles u will see plantar faciitis then tibialis
posterior tendon (answered by orthopedic dr)
- 2nd opinion:
You will see plantar aponeurosis then 4 layers of muscles:
1s layer: abductor hallucis, abductor digiti minimi, flexor digitorum brevis.
2nd layer: Quadratus plantae Ms, lumbricals, and associated with the tendon of flexor digitorum longus.
3rd layer: flexor hallucis brevis, flexor digiti minimi brevis, adductor hallucis.
4th layer: the dorsal and plantar interossei.
Reference: Grays anatomy 2nd ed, ch 6, p 617-621.
154- Related hernia to cord ? Anatomy
A- Internal pudendal artery
B- Internal iliac artery
Answer: B ?
155- hyperextension injury developed pain in distal phalanx , tender palms , cant flex distal phalanx :
A- Rupture superfascialis tendon
B- Rupture profundus tendon
C- Extraarticular fracture of distal phalanx
D- Intra Articular fracture of middle phalanx
Answer:
156- soldier walks 1000miles developed pain on foot :
Answer: Spring ligament
2nd opinion:
I think its March foot ( fatigue fracture or stress fracture of metatarsal bone):
is a fracture of the distal third of one of the metatarsals occurring because of recurrent stress. It is
more common in soldiers, but also occurs in hikers, organists, and even those, like hospital doctors, whose duties entail much standing. March fractures most commonly occur in the second
and third metatarsal bones of the foot.
Reference: https://en.wikipedia.org/wiki/March_fracture
157-Case head trauma on parietal lobe subdural hematoma which artery is injured?
A- superficial temporal.
B- mid cerebral
C- Rt.cerebral .......

smle ,2016

998
Answer: B
MCA supply: the lateral surface of the hemisphere, except for:
the medial part of the frontal and the parietal lobe, which are supplied by the ACA
inferior part of the temporal lobe, which is supplied by the PCA.
Reference: http://radiopaedia.org/articles/cerebral-vascular-territories

158- Flexion and pronationlling worse at morning. Dx?


Answer:
159-A woman complaining of lt hand tingling mainly at thumb and index on exam there
was mild atrophy of thinner muscle Tinel's test was positive which nerve may be affected?
A- radial nerve
B- median nerve
C- musculocutaneous nerve
Answer: B
Median N:
Motor supply: flexor compartment of the foreman, thenar and intrinsic hand muscles.
Sensory supply: palmar aspect of the thumb, index, middle and radial half of the ring fingers.
Reference: http://radiopaedia.org/articles/median-nerve
160- Renal segment of inferior vena cava drain from the right side by which branch ?
A- Sub cardinal ...
B- Supra
Answer:
161- Injury in knee , anterior displacement of leg bone which ligament affected?
A- Ant cruciate ligament
B- Post cruciate
Answer: A (Agree)
Reference: Master the board step2 (surgery chapter)
162-Q Read about:
-Big artery and branch what supply ? ( brain , colon , spleen , appendix ) ?
-Nerve and dermatome of upper and lower limbs.

smle ,2016

999

Physiology
1- The corrected sodium of a patient is 138, what is the anion gap? (no further informations)
I think the question must be incomplete. ( we can suppose others values as normal and do the
equation (Na) - ( HCO + Cl ) . Na 135 -145 meq\l , HCO3 22-26 meq\l , Cl 98-107 meq\l .
2- ph 7.1, HCO3 18, CO2 25, dx:
a- Metabolic alkalosis
b- Metabolic acidosis
c- Respiratory alkalosis
d- Respiratory acidosis
Answer: B
Metabolic Acidosis (respiratory compensation)
Reference: Guyton and Hall textbook of physiology, 12th ed, p393

smle ,2016

1000

3- Acid-base balance:
A question about it.(read)
Reference: Guyton and Hall textbook of physiology, 12th ed, p394
4- Whats the normal anion gap?
a- 13
b- 26
c- 95
d- 123
Answer: A (normal range: 8-16 mEq/L)
Reference: Guyton and Hall textbook of physiology, 12th ed, p395
5- History of metabolic acidosis, lab results showing decreased ph and HCO3, how to compensate?
A- CO2 washout
Answer: A
In simple metabolic acidosis, the PCO2 is reduced because of partial respiratory compensation.
Reference: Guyton and Hall textbook of physiology, 12th ed, p394
6- Which system or organ will work in stress?
A- Respiratory
B- Renal
C- Sympathetic
D- Parasympathetic
smle ,2016

1001
Answer: C
Reference: Guyton and Hall textbook of physiology, 12th ed, p739
7- Which cell in the stomach is responsible for production of vit B12?
A- Parietal cells
B- Chief cells
C- Global cells
Answer: The question must be incorrect, it could be which cell in the stomach is responsible
for production of intrinsic factor that is responsible for vitamin B12 absorption? Then the answer
will be Parietal cells.
*Please read about the absorption of vitamin B12.
Reference: Guyton and Hall textbook of physiology, 12th ed, p417
8- A boy is fighting with two boys, which system is activated?
A- Sympathetic
B- Parasympathetic
Answer: A
Reference: Guyton and Hall textbook of physiology, 12th ed, p739
9- High PCO2, low PH indicates?
A- Metabolic acidosis
B- Metabolic alkalosis
C- Respiratory acidosis
D- Respiratory alkalosis
Answer: C
Reference: Guyton and Hall textbook of physiology, 12th ed, p394
10- Normal anion gap in mmol/L?
A- 3
B- 95
C- 123
Answer: A
The reference range of the anion gap is 3-11 mEq/L (3-11 mmol/L)
The normal value for the serum anion gap is 8-16 mEq/L (8-16 mmol/L)
1 mmol/L of AG = 1 mEq/L of AG
Reference: medscape, http://emedicine.medscape.com/article/2087291
11- Orthostatic Hypotension:
A- Decrease extracellular fluid
B- Decrease intracellular fluid
Answer: A
Postural (orthostatic) hypotension is a drop in systolic blood pressure of 20 mmHg or more on
standing from a sitting or lying position. Usually, reflex vasoconstriction prevents a drop in pressure but if this is absent or the patient is fluid depleted or on vasodilating or diuretic drugs, hypotension occurs.
Reference: Kumar & Clarks Clinical Medicine, 8th ed, p676

smle ,2016

1002

12- Renin is secreted from?


A- Renal afferent arterioles
B- Renal efferent arterioles
C- Distal tubules
D- Proximal tubules
Answer: A
Renin is synthesized and stored in an inactive form called prorenin in the juxtaglomerular cells
(JG cells) of the kidneys. The JG cells are modified smooth muscle cells located in the walls of
the afferent arterioles immediately proximal to the glomeruli.
Reference: Guyton and Hall textbook of physiology, 12th ed, p220
13- Which of the following increases the absorption of iron?
A- Folic acid
B- Vitamin C
Answer: B, It helps the body absorb iron from nonheme sources.
Reference: umm.edu/health/medical/altmed/supplement/vitamin-c-ascorbic-acid

Microbiology
1- Organism from animal bit?? no rabies in answers
A- Streptococcus mutalis<< not sure
Answer: Pasteurellaanimal bites
Reference: First Aid step 1 :

2-Patient with ventilator associated pneumonia. Culture showed lactose non-fermenting,


gram negative motile not the same Q but they asked about an organism!!!
Answer:
bacilli producing greenish colony + Oxidase positive. What is the organism?
A) Haemophilus Influenzae
B) Streptococcus pneumoniae
C) Klebsiella or other gram negative bacteria
D) Pseudomonas aeruginosa
Answer:D
Pseudomonas aeruginosa:
Aerobic gram-negative rod. Non-lactose fermenting, oxidase . Produces pyocyanin (blue-green
pigment); has a grape-like odor. Water source. Produces endotoxin (fever, shock) and exotoxin A
(inactivates EF-2).

smle ,2016

1003
PSEUDOmonas is associated with wound
and burn infections, Pneumonia (especially in cystic fibrosis), Sepsis, External otitis (swimmers
ear), UTI, Drug use and Diabetic Osteomyelitis, and hot tub folliculitis. Malignant otitis externa
in diabetics.
Ecthyma gangrenosumrapidly progressive, necrotic cutaneous lesions caused by Pseudomonas
bacteremia. Typically seen in immunocompromised patients.
Treatment: aminoglycoside plus extended- spectrum penicillin (e.g., piperacillin, ticarcillin,
cefepime, imipenem, meropenem
Reference: First Aid step 1.
3-HIV patient, presented with SOB and productive cough bronchoalveolar lavage showing
((something)) . Similar Q but no Pneumocystis in answers !!
A) Pneumocystis jiroveci
B) Aspergillus Fumigatus
C) Cryptococcus neoformans
Answer: C
The is bubble soape appearance so the answer is c
Cryptococcus neoformans: Cryptococcal meningitis, cryptococcosis. Heavily encapsulated yeast.
Not dimorphic. Found in soil, pigeon droppings. Acquired through inhalation with hematogenous
dissemination to meninges. Culture on Sabouraud agar. Stains with India ink and mucicarmine.
Latex agglutination test detects polysaccharide capsular antigen and is more specific. Soap bubble lesions in brain.
Reference: First Aid step 1
4-C.perf >> ??
A-cruptis under skin
Answer: A
C. perfringens :
Produces toxin (lecithinase, a phospholipase) that can cause myonecrosis (gas gangrene) and
hemolysis.
Reference: First Aid step 1
5-ventilator associated pneumonia in icu patient Gram negative oxidase postive
What is the organism?
Answer: Psuomonas agina
Pseudomonas aeruginosa
Aerobic gram-negative rod. Non-lactose fermenting, oxidase . Produces pyocyanin (blue-green
pigment); has a grape-like odor. Water source. Produces endotoxin (fever, shock) and exotoxin A
(inactivates EF-2).
PSEUDOmonas is associated with wound and burn infections, Pneumonia (especially in cystic
fibrosis), Sepsis, External otitis (swimmers ear), UTI, Drug use and Diabetic Osteomyelitis, and
hot tub folliculitis. Malignant otitis externa in diabetics.
Ecthyma gangrenosumrapidly progressive, necrotic cutaneous lesions caused by Pseudomonas
bacteremia. Typically seen in immunocompromised patients.

smle ,2016

1004
Treatment: aminoglycoside plus extended- spectrum penicillin (e.g., piperacillin, ticarcillin,
cefepime, imipenem, meropenem
Reference: First Aid step 1
6-Q - Child diagnosed to have Giardiasis (Giardia Lamblia). What is the best investigation
that confirm the diagnosis ?
A- 3 stool analysis in consecutive days
B- 3 stool analysis in separated days
Answer :B
Stool examination may be performed on fresh specimens or after preservation with polyvinyl alcohol or 10% formalin (with appropriate staining). Ideally, 3 specimens from different days
should be examined because of potential variations in fecal excretion of cysts. G intestinalis is
identified in 50-70% of patients after a single stool examination and in more than 90% after 3
stool examinations.
Reference: http://emedicine.medscape.com/article/176718-workup
7--Scraping used in diagnosis which organism:
A- scabies
B- toxo... (not toxoplasmosis)
Answer : A
Reference: http://emedicine.medscape.com/article/1109204-workup#c8
Advanced Health Assessment & Clinical Diagnosis in Primary Care - Elsevieron Page 418

8- (long scenario) man traveled for business, went to his flat to relax, second day he noticed
mold on his flat, he developed rash all over his body. We took sample of mold and injected
him, inflammation occurs after 30 minutes. What does it indicate?
a.
Immediate immunity.
b.
Late immunity.
Answer: A?

smle ,2016

1005
9- Someone with cancer, took cells from him inject with cancer, then re inject to patient so
they can be antigen for cancer. What type of immunity?
A Passive
B Active immunotherapy.
Answer: B ( you can read here for active and passive immunotherapy of cancer
Reference: http://www.cel-sci.com/active_immunotherapy.html , http://www.cel-sci.com/passive_immunotherapy.html
10- man done vasectomy, change his mind want to reproduce again, they found antisperm
antibodies what is the cause ?
A- antigen release or something like that?
B- cross reactivity with viral infection
C- inappropriate response of MHC II to antigen presenting
Answer: A.
Anti sperm antigen has been described as three immunoglobulin isotopes (IgG, IgA, IgM) each
of which targets different part of the spermatozoa. If more than 10% of the sperm are bound to
anti-sperm antibodies (ASA), then infertility is suspected. The blood-testis barrier separates the
immune system and the developing spermatozoa. The tight junction between the Sertoli cells
form the blood-testis barrier but it is usually breached by physiological leakage. Not all sperms
are protected by the barrier because spermatogonia and early spermatocytes are located below
the junction. They are protected by other means like immunologic tolerance and immunomodulation.
Infertility after anti-sperm antibody binding can be caused by autoagglutination, sperm cytotoxicity, blockage of sperm-ovum interaction, and inadequate motility. Each presents itself depending on the binding site of ASA.
Additional Questions ( 8th update)
11-patient having ear discharge bacteria is non fermentar, oxidase + gram negative bacilli
A- pseudomonas
B- moraxella
answer: A

smle ,2016

1006
12-UTI patient, results showed, lactose + non fermenter gram - bacilli:
A- klebsiella
B- proteus
answer: b?
13-Ventilator acquired pneumonia with greenish discharge:
- pseudomonas
Answer: A
7-Gram ve bacilli, lactose fermenting in urine culture?
Answer E. Coli?
14-UTI urease +ve:
A- proteus
Answer: A

15-Allergic reaction (Serum sickness) to animal protein derivative injection what will be
found in blood of patients?
A- IgE
B- IgG..
Answer : A
16-indole positive?
a- E coli
answer: A
17-about type 1 hypersensitivity ( easy )
18-patient who is sexually active came with non specific urethritis, what is the most likely
cause?
A- Chlamydia
B- Treponema pallidum
Answer: A ( also known as non-gonococcal urethritis.
Reference: Microbiology in Clinical Practice 2ed edition by D. C. Shanson page 466 )

smle ,2016

1007

19-treponema pallidum microscopy, to confirm:


A- Do dark field microscopy
Answer: A
20-food poisoning with abdominal cramps, nausea, vomiting is most likely caused by:
A- Salmonella
B- Staph
C- Enterococcus faecalis
Answer: B
21-patient with barking cough, red epiglottis is caused by:
A- Pertussis
B- HiB
Answer: B
22-.Side effect of DTP vaccin:
A) Fever
B) Generalized body pain
C) Erythema at injection site (local reaction )
Answer: C
Reference: Information Sheet Observed Rate Of Vaccine Reaction DTP (may 2014)
23-What does it mean when the patient has positive Hbs Ag?
Answer: Antigen found on surface of HBV; indicates hepatitis B infection \ if only HbsAg is
+ve that means early acute HBV infection
Reference: USMLE step 1 \ rapid review pathology goljan
24-Varicella vaccine in adult.
A) 1 dose once
B) 2 doses 4 weeks
C) 2 doses 6 weeks
Answer: B ( 2 doses at least 4 weeks apart )
Reference: CDC
25-RNA hepatitis?
A) hep C"
Answer: A (all hepatitis virus A,C,D,E RNA except HBV is DNA )
Refernce: 1st aid usmle step 1
26-man done vasectomy, change his mind want to reproduce again, they found antisperm
antibodies what is the cause ?
A-antigen release or something like that?
B-cross reactivity with viral infection
C-inappropriate response of MHC II to antigen presenting cell
answer : A ?

smle ,2016

1008

27-Pt dx with cutaneous leishmanial what is the organism?


A) Lishmenia kala azar (visceral Leishmania caused by L .donovani )
Answer: cutaneous leishmania by L.tropica , Mucocutaneous leishmaniasis by L. braziliensis
Reference : rapid review pathology by goljan
28-Pt asymptomatic come to you with lab result : HBsAg +ve (only this data they give me
)Your Dx:
A- Acute infection
B- Acute chronic infection
Answer: A early Acute infection
29-the best test for hypersensitivity type 1 :
1- subdermal skin injection ( my answer)
Answer:A
30-Cold agglutination test use which antibodies?
A-IgA
B-IgM
C-IgG
D-IgE
Answer: B
31-long scenario where the investigation shows high IGg :
monoclonal something << I choose this
Answer: Mutiple myloma z
32. DiOxyNucleTides used in which investigation ?
A- western blot
B- DNA suquence ( my answer not sure)
(and other 2 options)
9th update
Our link Qs up to 28th of December
33- TB culture media?
Answer:
LwensteinJensen medium, more commonly known as LJ medium, is a growth medium specially used for culture of Mycobacterium, notably Mycobacterium tuberculosis.
34- PPD positive ,tx
Answer: Isoniazid
35- Boy with sever abdominal pain, guarding, CT show, gas in the wall of intestine
Undercooked meet infected with:
A- Entero Coli

smle ,2016

1009
B- entamiba histolitica
Answer: ?
Organisms that can be found in raw meat:
Beaf: E. coli O157:H7, Salmonella, Shigella, Staphylococcus aureus and Listeria monocytogenes
Poultry: Salmonella and Campylobacter
Shellfish: Vibrio gastroenteritis, Salmonellas, Plesiomonas shigelloides, Staphylococcus and
Bacillus cereus
36- Isolated microorganism from urine, diplococci, kidney shape
A- Nisserrhea Gonnorrhea
Answer : A
Reference: http://microbeonline.com/characteristics-shape-of-pathogenic-bacteria/
37- male What is type of dysuria after urination bladder still full and palpable
Maculopapular rach on face and some inside buccal white?
A- mumps
B- herpis zoster
C- measls
Answer: Herpes simplex virus
Clinical features in men: Herpetic vesicles appear in the glans penis, the prepuce, the shaft of the
penis, and sometimes on the scrotum, thighs, and buttocks. In dry areas, the lesions progress to
pustules and then encrust. Herpetic urethritis occurs in 30%-40% of affected men and is characterized by severe dysuria and mucoid discharge. The perianal area and rectum may be involved
in persons who engage in anal intercourse, resulting in herpetic proctitis.
Reference: medscape
38- How to differentiate between positive Gram Cocci ?
A- Catalase
B- Coagulase
Answer : A 1st AID USMLE step 1
39-parasite in soil contamination:
A- tenia saginatm
B- ascaris-bancrofti ..
Answer: b
40- A child with chronic diarrhea, endoscopy with biopsy showed Sickled shaped parasite.
Whats the dx?
A. Entamyba histolytica
B. Giardia
C. Ancylostoma
Answer: B
refrence : CDC : http://www.cdc.gov/parasites/sth/

smle ,2016

1010
41-(Micro): Gram +ve Bacilli non aerobic :
Answer: Clostridium perfringens (spore forming)
42-(micro) gram +ve cocci catalase +ve coagulase +ve grape like cluster
Answer: Staph aureus

43- what to see under microscope in bacterial vaginosis


A-cell with multiple inclusion
B-giant cell
Answer : Demonstration of clue cells on a saline smear is the most specific criterion for diagnosing BV. Clue cells are vaginal epithelial cells that have bacteria adherent to their surfaces. The
edges of the squamous epithelial cells, which normally have a sharply defined cell border, become studded with bacteria. The epithelial cells appear to be peppered with coccobacilli.
Reference: http://emedicine.medscape.com/article/254342-workup
New 12
44-- most common organism in uti.
Answer: 1st most common E coli , 2ed common staph.. Saprophyticous
Reference: 1st AID USMLE step 1
45-Infection that is more severe in immunocompromised rather than immunocompetent:
a. crypto.... can't recall the rest
Answer :
It is now well known that people who are immunosuppressed secondary to HIV infection are at
higher risk for Cryptosporidium infection and that carriage of the parasite is associated with diarrheal disease in most cases. Furthermore, in those with diarrhea, the disease is much more severe
and prolonged than in otherwise healthy individuals
Reference: http://www.ncbi.nlm.nih.gov/pmc/articles/PMC118064/

smle ,2016

1011
46-Boy swimming in river , after one day , onset of fever and weakness , what the most
common organism cause infection ?
Answer: (Katayama's fever) Acute schistosomiasis
47-Monospot test +ve what is the diagnosis
Answer: EBV ( infectious mononucleosis )
Reference: http://www.cdc.gov/epstein-barr/laboratory-testing.html
48-Organism undercooked beef ?
A.Entamoeba
B.Tinea
Answer : b
Organisms that can be found in raw meat:
Beaf: E. coli O157:H7, Salmonella, Shigella, Staphylococcus aureus and Listeria monocytogenes
Poultry: Salmonella and Campylobacter
Shellfish: Vibrio gastroenteritis, Salmonellas, Plesiomonas shigelloides, Staphylococcus and Bacillus cereus
49-Bacteria sexual like behavior ?
A-Conjunction
Answer : A
Reference: http://www.ncbi.nlm.nih.gov/books/NBK21942/
50- bacteria grow in antiseptic and detergent
a-E.coli
b-...
Answer: klebsiella
All the bacterial contaminants of these antimicrobial substances were Klebsiellaspecies of which
all K. pneumoniae species were multidrug
Refernce: http://pubs.sciepub.com/ajnr/2/3/3/
51-scenario of diarrhea choices about which type of clustered :
difficil
itinus....
Answer: difficil
52-asymptomatic trichomonas when will you treat:
Answer: start immediately
right way to do pap smear
three sample from the endocervix
2samble from .....

smle ,2016

1012
53-Most common gonococcus infection in females :
A- urethra
B- cervix
Answer: cervix
54-the most common organism in whooping cough
A- Bordetella
B- pertussis
C- H. Influenza
Answer: B
55-Mouth ulcer :
A- HSV 1
B- HSV2
Answer: A
56-gram positive cocci in clusters :
A- Indole
B- urea
C- coagulase
Answer: C
57-Young , vesicle ,pustule on back like a band :
A- Shingles
B- Chicken box
C- Herpes
D- Coxsackievirus
Answer: A
58-vaginal discharge fishy odor.??
Answer: bacterial vaginosis
59-STD case with culture gram -ve diplococci , wt is organism ?
A- N . Gonorrhea
B- chlamydia
C- E.coli
Answer: N . Gonorrhea
Reference : 1st AID USMLE step 1
60-10 days neonate present with lethargy , irritability , fever , signs of meningitis which organism is causative :
A- listeria monocytogenes
B- streps pneumonia
smle ,2016

1013
C- staph aureus
D- N-meningitis
Answer: A
Immune new 12
61-We can't (or can"not sure) give vaccine against which gastroenteritis viruses
A- Adenovirus
B- Rotavirus or something like that
62- gene of (copper transport atpase ) on which chromosome
a-13
b-9
c-14
Answer: A
Reference: High-yield Genetics by Ronald W. DudekJohn E. Wiley page 100 chapter 14
63-You perform a skin hypersensitivity test for molds which came strongly positive within
30 minutes, what is the type of his hypersensitivity reaction?
A- Immediate hypersensitivity reaction
B- Delayed hypersensitivity reaction
C- Contact dermatitis
D- immune-complex reactions (serum sickness)
Answer: A. Immediate
64-male just moved to new apartment in humidified area and there was dust , he was injected with mites and 30 mints later , he developed allergy , what type of hypersensitive he
has ?
A- Immediate
B- Delayed
65- What types of hypersensitive rapid onset of ?
Fast acting antihypertensives:
A- captopril
B- clonidine
C- labetalol
D- nifedipine
E- sodium nitroprusside
66-long scenario for pt with contraindication to have BCG due to family history with defect
in interleukin receptor . What receptors?
A. INF gamma receptor
B. INF alpha- beta receptor
Answer A
Reference : Textbook of Clinical Pediatrics By A. Y. ElzoukiH. A. HarfiF. B. StapletonH. NazerWilliam OhR. J. Whitley page 930

smle ,2016

1014
67-A 4 months old kid has to take vaccine, mother stated that he had fever when he took 2
months vaccine. What shall u tell the mother?
68-wound healing in DM
Read about it
69-What is blood cell responsible for blockage :
A- Basophil
B- neutrophil
C- eosinophile
D- macrophage
70- Case of pt with malignancy , dr is planning to take the malignant cells and
implant it in the pt dendritic cells to enhance immunity against tumor cells what is this
called :
A- Active immunotherapy
B- Passive immunotherapy
C- Weird name can't recall !!
Answer : Active immunotherapy ( see previous Q )
71- Old man started a year a go on medication for peptic ulcer developed gynecomastia
what is the drug :
Answer: cimetidine
Reference: https://en.wikipedia.org/wiki/Cimetidine
72-girl after eating seafood developed rash with severe itching and diarrhea , the mechanism of reaction
A- complement
B- cell mediated
C- immediate
Answer : C
73- Humoral immune protection (immunoglobulins) is supplied to the neonate by a process
of transfer of passive immunity from the mother to the neonate??

74-Allergic rhinitis symptoms caused by chemicals produced from?


A- Mast cells
B- NK cells
C- CD4
Answer: mast cell
75-girl after eating seafood developed rash with severe itching and diarrhea , the mechanism of reaction:
A- complement
B- Cell mediated
C- antibody mediated

smle ,2016

1015
Answer:
Most likely its Missed >> type I igE mediated
76-pt want to get married she is hiv positive what will we do to decide ??
A-ELISA
B-western blot(my ans because its specific)for conformation
77-What is the Immunological reaction to peanut allergy ? It is a type one
78-hypersensitivity reaction to dietary substances from peanuts
Type 1
79-patient with sx persistent nesal obstruction and discharge since he transferred to this
new house where the molds are on the wall , in the hospital skin test was done and it was
positive in less than 30 min whats this type of allergy:
A- Immediate >>type I
B- Delayed
C- Serum sickness reaction
D- Anaphylaxis
Answer: A
80-Which Of the following increased in viral infection?
1. Basophils
2. Monocytes
3. Eosinophils
4. Lymphocytes
Answer : Lymphocytes
81-Defective regulation of immunosuppression
A- Activated CD+4 cells activate other inflammatory cells like macrophages & B-cells or recruit
more inflammatory cells by stimulation of homing receptor on leucocytes & vascular epithelium.
Answer: ?
82- Recurrent viral & fungal infection cell deficient ?
A- T cell
Answer: A
Refernce: http://www.antimicrobe.org/e33.asp Essentials of Rubin's Pathology 5th edtion page 56
83-RH -ve mother and +ve baby what is the complication at delivery
Answer : acute hemolysis (hypersesetivity type 2 )
84-pregnant women has HIV the cD was 400 and now it is CD200 how to deliver the pt :
A- c.s
B- spentnous vagenal dlivery
C- scheduled c.s
Answer : B

smle ,2016

1016
85- girl after eating seafood developed rash with severe itching and diarrhea , the mechanism of reaction
A- complement
B- cell mediated
C- immediate
86-what of the following is poor prognosis?
A- high Ig M
B- low Ig A
C- high ca
Answer:

Biochemistry
1.How does the protein enter the peroxisomes?
Folded with the help of C-terminal
Unfolded with the help of T-terminal
Folded with the help of T-terminal
Unfolded with the help of C-terminal
Answer: A
Peroxisomal proteins are fully folded in the cytoplasm and enter the organelle in folded form.
The most common signal sequence which directs proteins to peroxisomes is a C-terminal SerLys-Leu tripeptide. Reference: Biochemistry and Molecular Biology Textbook.
http://global.oup.com/uk/orc/biosciences/molbiol/snape_biochemistry5e/student/mcqs/ch27/
2- How does the body get rid of Lactic acidosis?
a.
Excretion by kidney
b.
Liver metabolism
Answer: B
In basic terms, lactic acid is the normal endpoint of the anaerobic breakdown of glucose in the
tissues. The lactate exits the cells and is transported to the liver, where it is oxidized back to glucose.
Reference: http://emedicine.medscape.com/article/167027-overview
3 - from rest muscle status to sudden contractions. What change in cellular level.
Answer:

smle ,2016

1017

4- which of the following made in nucleolus:


DNA
mRNA
rRNA
Answer: rRNA
5- Protein after transfer sequence what is last destination
A. endoplasmic retuculum
B. Cytosol
Answer:A
6- ALP which enzyme confirms that elevation from liver
Answer: gamma-glutamyl transpeptidase (GGT) or 5'-nucleotidase (5'-NT),
Ref: http://www.liver.ca/liver-disease/diagnosing-liver-disease/liver-function-tests.aspx
7- Mendles Laws:
Answer: MENDEL'S LAWS OF INHERITANCE Mendel noted that many physical characteristics of seeds and flowers were transmitted from parental strains to offspring in a predictable
and reproducible manner. From these observations, Mendel proposed that traits manifest through
the joint effects of two paired elements (or genes), with each parent contributing one element (allele) to each offspring at conception

smle ,2016

1018
Law of segregation The law of segregation states that paired parental copies of genes are separated from each other during gamete formation, with each copy (ie, allele) segregating into separate gametes. This process of gamete formation is now known as meiosis. (See "Principles of
molecular genetics", section on 'Meiosis and sustained genetic diversity'.)
Law of independent assortment The law of independent assortment states that genes segregate
into gametes independent of other genes such that the allelic status at one locus does not determine segregation of alleles at other loci. This is only true for genes that are not linked to each
other.
Law of dominance The law of dominance distinguishes dominant, recessive, and co-dominant
traits.
A trait is considered dominant when it is manifest in heterozygous carriers (termed heterozygotes).
A trait is considered recessive when it is observed only among those homozygous for the disease allele or with two different mutations in the same disease-causing gene (compound heterozygotes). Heterozygotes for a recessive trait (carriers) are typically phenotypically indistinguishable from non-carrier homozygotes. In X-linked disorders, however, carrier females can sometimes be affected.
A co-dominant or semi-dominant trait is one where both alleles contribute equally to phenotypic expression. Heterozygotes manifest an intermediate phenotype between those of the two
homozygous classes.
Ref : uptodate
8-Athletic come for check up all thing normal except Xanthelasma on achilles tendon and cholesterol?
A.Ldl receptor
B. Apo ll
C. Apo c
Answer: A
9- organic phosphate , found in or effect of :
A. liver
B. kidney
C. lung
Answer: C
Mnemonic devices used to remember the muscarinic effects of organophosphates are SLUDGE
(salivation, lacrimation, urination, diarrhea, GI upset, emesis) and DUMBELS (diaphoresis and
diarrhea; urination; miosis; bradycardia, bronchospasm, bronchorrhea; emesis; excess lacrimation; and salivation). Muscarinic effects by organ system include the following:
Cardiovascular - Bradycardia, hypotension
Respiratory - Rhinorrhea, bronchorrhea, bronchospasm, cough, severe respiratory distress
Gastrointestinal - Hypersalivation, nausea and vomiting, abdominal pain, diarrhea, fecal incontinence
Genitourinary - Incontinence
Ocular - Blurred vision, miosis
Glands - Increased lacrimation, diaphoresis
Ref : Medscape

smle ,2016

1019

10. Bond between DNA and tRNA:


A Covalent
B Ionic
C Hydrophiclic
Answer: A
11-type of bond bet tRNA and mRNA:
A.
Covalent bond
B.
Ionic bond
C.
hydrogen bond
answer: C
reference :
How does tRNA bind to codons in the mRNA?
The complementary bases on the codon and anticodon are held together by hydrogen bonds,
the same type of bonds that hold together the nucleotides in DNA.
The ribosome only allows the tRNA to bind to the mRNA if it is carrying an amino acid.
Once the amino acid is added to the polypeptide chain, the binding is no longer favored.
12-in glycolysis when convert pyrevet to lactate :
A.
Incre NPH
B.
Decrease AMP
Answer : not know
Lactic Acid Fermentation
Reaction occurs in cells without mitochondria (RBC) or in cells when O2 is limited (muscle cells
during
exercise). The entire purpose of this reaction is to convert the NADH produced in step 6 of glycolysis back to
NAD+ so that glycolysis can continue. This is simply a redox reaction in which pyruvate is reduced to lactate by the enzyme lactate dehydrogenase.
Lactate can then enter the bloodstream and travel to the liver where it is converted back to glucose through the gluconeogenesis pathway.
www.science.marshall.edu/frost/chapter14.pdf
13. Which is acidophilic:
A Lace
Additional Questions ( 8th update)
14- Residual ribosomal RNA in which cell :
A- RBC
B- Reticulocyte
ANSWER: reticulocyte
Reference: https://en.wikipedia.org/wiki/Reticulocyte
15- Which of the following enzyme in not functional ?
smle ,2016

1020
A- HMG Coa..
ANSWER:
plasma non functional enzymes:
http://nazimrmc.blogspot.com/2012/09/plasma-non-functional-enzyme.html

16- What is move the molecules in cell in and out


a. ATP cytoplasmic (ans by DR)
b. ATP nuclear
c. GTP cytoplasmic
d. GTP nuclear
answer : a , by Dr. of genetics
17-Ionised radiation what will happen ?
A- deamination
B- de purantion
C- DNA break strin
Answer: c Biochemistry for Health Professionals

by Laura BatmanianSimon WorrallJustin

18-Sx of hydrop fetalis or HB h


-normal 2 beta abnormal 4 alfa
- oppiste above
Answer: abnormal 4 alpha
19- melanin pigment from what cell :
Somatotrop
Answer : Corticotropes (or corticotrophs) are basophilic cells in the anterior
pituitary that produce ,melanocyte-stimulating hormone
20- Which one has powerful molecular energy in the cell ?
1- ATP hydrolysis move component from cytoplasmic to neuclus
2- GTP hydrolysis move component from cytoplasmic to neuclus (ans by
Dr)
3- ATP hydrolysis move component from nucleus to cytoplasmic
4- GTP hydrolysis move component from nucleus to cytoplasmic
21-Which one has inhibitory action?
Phosphatase A2
cyclooxygenase
Answer????
22-Molecule made in neuclolus ?
DNA,mRNA,rRNA ,RNA
Answer : mRNA
23-what gene induce the apoptosis of the cells.
smle ,2016

1021
-p53
-CRP
-nitric oxide
-TNF
Answer : P53
24-Which of the following is considered in estimating the response of nucleoside reverse
transcriptase inhibitors
A.HIV RNA level in the serum ( < 50 copies \ml)
B.genotype of HIV
answer : A

Reference

Atlas of Sexually Transmitted Diseases and AIDS by

Stephen A. MorseKing K. HolmesRonald C. Ballard

25- Ionizing radiation effect on DNA?


Pyrimidine ..., deamination, depurination (no oxidization)
Ionizing radiation = damage or alteration to DNA purine by oxidation\ROS and non ionizing radiation damage to DNA pyrimidine to form cyclobutane bi-pyrimidine dimer and pyrimidine pyrimidone photoproducts . Reference : Radiation-induced and oxidative DNA damages by Antonio MonariElise

26-Hydrochlorothiazide Furosemide What is the type of that something That is bind to ca


in psudogout ?
Answer : Pyrophosphate
27- Which one of these is considered as sugar pentose?
A- Mannose
B- Arabinose*
C- Erythrose
D- Glucose
Answer : B (There are a number of other types of pentoses aside from ribose. Xylose,
ribulose, arabinose and deoxyribose all contain five carbon atoms and thus are classified as pentoses) Reference : https://www.reference.com/science/example-pentose4bc6e97e31de80dd
28-mechanism of action Pyruvate kinase Electron movement:
Answer : phosphoenolpyruvate to pyruvate and release ATP in Glycolysis .
Reference : Biochemistry 5th ed By Richard A. Harvey (Ph. D.)Richard A. HarveyDenise R. Ferrier ( page 101)
29-Rbcs deficient in lactic dehydrogenase What is the enzyme responsible Pyruvate kinase
?
Answer : ??????????

smle ,2016

1022
30-What is the longest cell cycle?
Prophase.
Cytokinesis.
Interphase (G1 , S phase and G2 among them G1 is longest )
Metaphase
Answer: C Interphase is usually longest, followed by prophase and telophase; metaphase/anaphase is usually shortest.
40. How does radiation damage DNA?
A.Deamination
B.double strand DNA breaks*"
Answer: B
http://explorecuriocity.org/Explore/ArticleId/2212/radiation-effects-on-cells-dna-2212.aspx
41.comparison between NF type I and type II ?

42-After endoplasmic reticulum protein moves to :


A golgi apparatus
B mitochondria
C neocleas
D
answer: A Reference: USMLE step 1
43-Type of protein form in neocleas:
A mrna
B trna
Answer: A mRNA similar to question 4 page 286
44-What is the source of energy to transfers molecules in and out of the nucleus ?
A-ATP within cytoplasm
B-ATP within nuclus
C-Dtp or something else!!
answer: A

smle ,2016

1023
45-Patient irriagate , and weakness in lower and upper extremities , what is the cause ?
- deficincy in vit D
- deficincy in vit A
- deficincy in B1
- deficincy in B3
answer : C 1st AID USMLE step 1 2015 page 89
46-THE DRIVING FORCE IS GTP IN CYTOSOL
Answer
47- Which one has powerful molecular energy in the cell?
A:a - ATP hydrolysis move components from Cytoplasm to Nucleus
b - ATP hydrolysis move components from Nucleus to Cytoplasm *
c - GTP hydrolysis move components from Cytoplasm to Nucleus
d - GTP hydrolysis move components from Nucleus to Cytoplasm
Answer :
48-Case senario the result was (High Lactic acid ) : defecincy of :
Pyruvate kinase
Pyruvate dehydrogenase
Pyruvate carboxylase
Answer: pyruvate carboxylase

Reference : Atlas of Metabolic Diseases 2ed edtion

William L NyhanBruce A BarshopPinar T. Ozand

49-rejection of transportation liver mechanism


answer:
there are three types of graft rejection that may occur. They include hyperacute rejection, acute
rejection and chronic rejection. Hyperacute rejection is caused by preformed anti-donor antibodies. It is characterized by the binding of these antibodies to antigens on vascular endothelial
cells. Complement activation is involved and the effect is usually profound. Hyperacute rejection
happens within minutes to hours after the transplant procedure. Unlike hyperacute rejection,
which is B cell mediated, acute rejection is mediated by T cells. It involves direct cytotoxicity
and cytokine mediated pathways. Acute rejection is the most common and the primary target of
immunosuppressive agents. Acute rejection is usually seen within days or weeks of the transplant. Chronic rejection is the presence of any sign and symptom of rejection after 1 year. The

smle ,2016

1024
cause of chronic rejection is still unknown but an acute rejection is a strong predictor of chronic
rejections.
Source: https://en.wikipedia.org/wiki/Liver_transplantation#Graft_rejection
50-something produced by the nucleus
A) Mrna
B) tRNA
Answer: mRNA

Genetics
1-what is genetic beta thalassemia?
A. Inseration
B. Mutation
C. Depleting
D. Fharm shaft
Answer: B The -thalassemias are caused by mutations that diminish the synthesis of -globin
chains . mostly consisting of point mutations
Reference : Robbins Basic Pathology 8th ed page 1207
Mutations in the HBB gene cause beta thalassemia. The HBB gene provides instructions for
making a protein called beta-globin. Beta-globin is a component (subunit) of hemoglobin. Hemoglobin consists of four protein subunits, typically two subunits of beta-globin and two subunits of
another protein called alpha-globin.
https://ghr.nlm.nih.gov/condition/beta-thalassemia#
2-Mode of transmission of polycystic kidney in adults:
Answer: Polycystic kidney disease
a. Autosomal-dominant (adult) polycystic disease
b. Autosomal-recessive (childhood) polycystic disease
Reference :Robbins Basic Pathology 8th ed page 1830
The two major forms of polycystic kidney disease are distinguished by the usual age of onset and
the pattern in which it is passed through families. The autosomal dominant form (sometimes
called ADPKD) has signs and symptoms that typically begin in adulthood, although cysts in the
kidney are often present from birth or childhood. Autosomal dominant polycystic kidney disease can be further divided into type 1 and type 2, depending on the genetic cause. The autosomal recessive form of polycystic kidney disease (sometimes called ARPKD) is much rarer and is
often lethal early in life. The signs and symptoms of this condition are usually apparent at birth
or in early infancy.
Reference: https://ghr.nlm.nih.gov/condition/polycystic-kidney-disease#genes
Additional Questions ( 8th update)
3. Progressive familial intrahepatic cholestasis, mode of inheritance?
a-Autosomal recessive
b-X-linked
smle ,2016

1025
c-Mitochondrial
d-Multifactorial
Answer: a Reference: http://www.uptodate.com/contents/inherited-disorders-associated-withconjugated-hyperbilirubinemia
https://ghr.nlm.nih.gov/condition/progressive-familial-intrahepatic-cholestasis
4.Case of neurofibromatosis, has axillary freckles (no dx), mode of inheritance.
a-Autosomal dominant -- neurofibromatosis genome **Ch17q11.2
Answer : A Reference : Robbins Basic Pathology 8th ed page 274
http://www.uptodate.com/contents/neurofibromatosis-type-1-nf1-pathogenesis-clinical-featuresanddiagnosis
https://ghr.nlm.nih.gov/condition/neurofibromatosis-type-1#inheritance
5.which gene is affected in IBD ( chromosomal )
A.13
B.14
C.15
D.16
Answer: D : CD on chromosome 16q the affected genes are NOD2, ATG16L1 and IRGM \ UC
on chromosome 12q . Reference : Robbins Basic Pathology 8th ed page 1537 and
http://www.ncbi.nlm.nih.gov/pubmed/12557156

http://www.uptodate.com/contents/genetic-factors-in-inflammatory-bowel-disease
https://ghr.nlm.nih.gov/condition/crohn-disease#inheritance
6. Which one has powerful molecular energy in the cell
1- ATP hydrolysis move component from cytoplasmic to neuclus
2- GTP hydrolysis move component from cytoplasmic to neuclus
3- ATP hydrolysis move component from nucleus to cytoplasmic
4- GTP hydrolysis move component from nucleus to cytoplasmic
Answer: 2, by Dr. of genetics
7.what is the gene for ductal carcinoma of breast?
a) P53
b) Bcl2
c) C-myth

Answer: A Reference : Ductal Carcinoma In Situ and Microinvasive/Borderline Breast Cancer


http://link.springer.com/chapter/10.1007%2F978-1-4939-2035-8_5

8.Human papilloma virus cause cervical cancer:


-16,
-18

smle ,2016

1026
Answer: There are 15 high oncogenic risk HPVs that are currently identified. From the point of
view of cervical pathology, HPV 16 and HPV 18 are the most important. HPV 16 alone accounts
for almost 60% of cervical cancer cases, and HPV 18 accounts for another 10% of cases
Reference : Robbins Basic Pathology 8th ed page 1967
9.Thalassemia Hb F 40 Hb A2 20
A- alfa thalassemia
B- beta thalassemia minor
C- beta thalassemia major
D- thalassemia intermediate
Answer:B Reference: http://enotes.tripod.com/thalassemia.htm
read about: down syndrome
New12
10-female with ductal carcinoma Doctor want treat her, what is the gene responsible for
that cancer? (No BRCA1 in options)
a) P53
b) Bcl2**
c) C-myth
Answer: A Changes in other genes (OTHER THAN BRACA GENES):
TP53: The TP53 gene makes a protein called p53 that helps stop the growth of abnormal cells.
Inherited mutations of this gene cause Li-Fraumeni syndrome. People with this syndrome have
an increased risk of breast cancer, as well as other cancers such as leukemia, brain tumors, and
sarcomas (cancers of bones or connective tissue). This is a rare cause of breast cancer.
CHEK2: The Li-Fraumeni syndrome can also be caused by inherited mutations in the CHEK2
gene. Evenwhen it doesnt cause this syndrome, it can increase breast cancer risk when its mutated.PTEN: The PTEN gene normally helps regulate cell growth. Inherited mutations in this
gene cause Cowden syndrome, a rare disorder in which people are at increased risk for both benign and malignant breast tumors,as well as growths in the digestive tract, thyroid, uterus, and
ovaries. Defects in this gene can also cause a different syndrome called Bannayan-Riley-Ruvalcaba syndrome thats not thought to be linked to breast cancer risk. The syndromes caused by
mutations in PTEN can be grouped together as PTEN Tumor Hamartoma Syndrome.
CDH1: Inherited mutations in this gene cause hereditary diffuse gastric cancer, a syndrome in
which people develop a rare type of stomach cancer at an early age. Women with mutations in
this gene also have an increased risk of invasive lobular breast cancer.
STK11: Defects in this gene can lead to Peutz-Jeghers syndrome. People affected with this disorder develop pigmented spots on their lips and in their mouths, polyps in the urinary and gastrointestinal tracts, and have an increased risk of many types of cancer, including breast cancer.
PALB2: The PALB2 gene makes a protein that interacts with the protein made by the BRCA2
gene. Defects in this gene can lead to an increased risk of breast cancer. It isnt yet clear if
PALB2 gene mutations also increase the risk for ovarian cancer and male breast cancer.
Reference:http://www.cancer.org/cancer/breastcancer/moreinformation/breastcancerearlydetection/breast-cancerearly

smle ,2016

1027
11-mode of inheritance for Wilson's disease The answer :A-autosomal recessive
Answer: A Reference: https://ghr.nlm.nih.gov/condition/wilson-disease#inheritance
12-Autosomal recessive disease :
Both parents are carrier and phenotype normal what the chance they have
a kid with a disease ?
A- 25
Answer: A Reference: http://anthro.palomar.edu/practice/mendqui2.htm
http://www.biologyjunction.com/apgenetics_solutions.htm
13-1st degree cousins came for pre-marriage counseling, they are worried about hereditary
disease so they must be screened for:
Answer: a-thalassemia
14-chromosome in DM type 2 ?
Answer: chromosome 20 Reference: http://www.ncbi.nlm.nih.gov/pubmed/12475768
15-Polycystic kidney inheritance?
Answer: Autosomal Dominant
See above Q #2
16- kallman syndrome how to dx
A- Gonadotropin
B- Urinary something
Answer : We can dx KS by exclusion , physical Ex ( delay puberty + anosmia ), The main biochemical parameters in men are low serum testosterone and low levels of the gonadotropins LH
and FSH, and in women low serum oestrogen and low levels of LH and FSH.
Reference: http://emedicine.medscape.com/article/122824-workup
17-scenario about short stature, webbed neck what&#39;s chromosome?
Answer: 45X ( Turner Syndrome )
Read about turner
18-long scenario.. What is the gene mutation for it? (( gilbert syndrome))
Answer : UGT1A1 on Chromosome 2q37.
19- scenario about Down syndrome: single palmar crease...etc what&#39;s the diagnosis ?
Down syndrome Read about it
20) In SCA what is the genetic basic ?
A- point mutation
B- Deletion
Answer : A Sickle cell disease is caused by a point mutation in the sixth codon of -globin that
leads to the replacement of a glutamate residue with a valine residue.

smle ,2016

1028
The abnormal physiochemical properties of the resulting sickle hemoglobin (HbS) (autosomal
recessive )
Reference : Robbins Basic Pathology 8th ed page 1199
https://ghr.nlm.nih.gov/condition/sickle-cell-disease#statistics
21-0 yo with high cholesterol what is the most common gene defect
A. Apo cII
B. Apo B100
C. LDL receptor
Answer : C
22-Mass in upper part of kidney what's the gene
Ref : Robbins Basic Pathology 8th ed page11848

23. In Thalassemia:
A. 4 abnormal beta chain genes, normal 2 alpha chain genes .
B. 4 normal beta chain genes, abnormal 2 alpha chain genes
C. 2 abnormal beta chain genes, normal 4 alpha chain genes
D.2 normal beta chain genes, abnormal 4 alpha chain genes
Answer: c
24-The ATP7B gene is located on the long (q) arm of chromosome 13 at position 14.
-viral knockout which of the following
A- IL2
B- IL3
C- IL4
D- IL5
Answer: C Reference: https://ghr.nlm.nih.gov/gene/ATP7B#synonyms

smle ,2016

1029

Histology
1- What is the strongest part of the vein?
Tunica intima
Tunica media
Tunica adventitia
Serosa (Not sure if it was there)
Answer: C
2- Whats the origin of juxtaglomerular apparatus?
a.
Macula densa cell
b.
Bowman capsule
Answer: A The juxtaglomerular apparatus consists of three cells:
The macula densa, a part of the distal convoluted tubule of the same nephron.
Juxtaglomerular cells, which secrete renin.
Extraglomerular mesangial cells.
3- In which phase the endothelial lining of alveoli came in contact with blood vessels ?
a.
Alveolar phase.
b.
Canalicular phase.
c.
Pseudoglandular phase.
d.
Saccular phase.
Answer: D Saccular phase (terminal sac period - 26 weeks to birth)
4- calcitonin secreted from ?
A Parafollicular
Answer: parafollicular cell ( C cell )
5- Non keratinized stratified squamous epithelium:
Small bowel
Large bowel
Esophagus
Answer: C
Additional Questions ( 8th update)
6- melanin pigment from what cell :
a. Somatotroph
Answer: Melanocyte, there are other cells able to produce melanin but of different embryonic
origin (pigmented epithelium of retina, some neurons, adipocytes)

smle ,2016

1030

NEW 12 update
7 -Gfr 80 in this stage what happen?
Mesangial proliferation and thickening of basement membrane???? Couldnt
remember the other choices
-Deepest layer of vein & tunica intima
8-Amylotropic lateral scelrosis which horn is infected&gt;&gt;
anterior horn
lateral horn
posterior horn
Answer : A
9- which phase the alvulo endo..epithelium of the lung become mature ?
Alvular
calvcalus,
terminal sac
Answer : canalicular phase http://www.embryology.ch/anglais/rrespiratory/phasen04.html
10- Case of giardiasis , in biopsy sickle shape parasite
11- what is ther Glomular basement membrane barier:
answer is: podocyte http://www.ncbi.nlm.nih.gov/pmc/articles/PMC3839671/19
12-tenia coli which layer
A- adventitia
B- musculosa
Answer: Muscularis Externa
The colon has the same 4 layers that are present in most parts of gastrointestinal tract: the mucosa, submucosa, muscularis propria, and serosa.The mucosa includes a columnar epithelium
with a large number of mucus-secreting goblet cells (villi, which are present in the small intestine, are absent in colon), lamina propria, and muscularis mucosa. The submucosa contains the
blood vessels and Meissner nerve plexus. The muscularis propria contains the inner circular and
outer longitudinal muscles and myenteric (Auerbach) nerve plexus; teniae coli are formed by
outer longitudinal muscles. The serosa of the colon is visceral peritoneum.

No tenia coli in appendix and in rectum.


Reference: medscape
13- pt had HAV (its type A ) after 6 month he came to check up what hisopathology will
see? peripheral necroso-central necrosis- normal-interseptal necrosis

14- abundant cells in peripheral smear ?


Lymphocyte
Basophils

smle ,2016

1031
Neutrophil
Eosinophils
Answer: C
15- H.pylori infection cause antral gastritis :H.pylori stimulate which cell?
chief cell
gastrin
parietal
mucous
Answer: B gastrin specific products of H. pylori, such as ammonia, release gastrin from G cells
Reference: http://www.ncbi.nlm.nih.gov/pubmed/9394759

16- about minimal change GN..u should see in the biopsy ?


Answer: LM: normal glomeruli
17.which hepatic cell produce extra matrix component ?
A.Stellate cell
B.Hepatocytes
Answer: a ( Stellate cell = Ito cell )

Pharmacology
1- Mode of action of aspirin:
a.
inhibit platelet aggregation
b.
acts on antithrombin
Answer: A
Aspirin inhibits platelet aggregation by irreversibly inactivating cyclooxygenase, a key enzyme
in platelet prostaglandin metabolism.
Reference: Pubmed.
2- Clonidine decrease the effect of which of the following drugs?
Answer: Levodopa
Clonidine affects Levodopa by inhibiting its antiparkinsonism effect.
Reference: Basic and Clinical Pharmacology - Bertram G. Katzung, 12th ed. P1159
3- What is the mechanism of action of denosumab anti-resorptive properties:
A. cases osteoclastic apoptosis
B. inhibit RANKL
Answer: B
Denosumab is a fully human monoclonal antibody that binds the cytokine RANKL (receptor activator of NFB ligand), an essential factor initiating bone turnover. RANKL inhibition blocks
osteoclast maturation, function and survival, thus reducing bone resorption.
In contrast, bisphosphonates bind bone mineral, where they are absorbed by mature osteoclasts,
inducing osteoclast apoptosis and suppressing resorption. These differences in mechanism influence both the onset and reversibility of treatment.
smle ,2016

1032
Reference: Pubmed.
4- What is the side effect of thalidomide in pregnancy?
Answer: Phocomelia
Teratogenic effect of thalidomide include phocomelia (shortened or absent long bones of the
limbs) and many internal malformations.
Reference: Basic and Clinical Pharmacology - Bertram G. Katzung, 12th ed. P104
5- What is the mechanism of polio vaccine (Both types; oral and IV)?
Answer: Infection with poliovirus causes IgM and IgG responses in the blood, but mucosal IgA
is vital for blocking infection. This antibody can neutralize poliovirus in the intestine, the site of
primary infection. The live attenuated Sabin poliovirus vaccine is effective because it elicits a
strong mucosal IgA response and provide intestinal immunity. In contrast, the injected (Salk)
polio vaccine does not produce intestinal immunity, and therefore is less effective at preventing
spread of poliovirus in a population.
Reference: Pubmed.
6- What is the action of Atropine?(SYMPATHETIC)
A- Increase heart rate.
Answer: A
Reference: Basic and Clinical Pharmacology - Bertram G. Katzung, 12th ed. P118
7- What is the mechanism of action of Orlistat?
A- decrease cholesterol
B- increase satiety
C- decrease pancreatic enzymes
Answer: C
Orlistat: gastrointestinal lipase inhibitor, reduces fat absorption by 30% by inhibition of pancreatic lipase.
Reference: Toronto Notes.
8- Side effect of anticholinergic drugs?
A- Diarrhea
B- Excess Salivation
C- Blurred vision
Answer: C
Dry mouth, urinary retention, constipation, blurred vision, toxic-confusional states.
Reference: Toronto Notes.
9- Which of the following medications used in treating osteoporosis and can cause heartburn sensation?
A- Denosumab.
B- Risedronate.
C- Raloxifene.
Answer: B
Reference: Kaplan Pharmacology for Step 1.
smle ,2016

1033

10- Regarding mechanism of action of Aspirin on enzymes :


A. Stimulate cyclooxygenase.
B. irreversible inactivation of the cyclooxygenase (COX) enzyme
Answer: B
Irreversibly inhibits cyclooxygenase (both COX-1 and COX-2) enzyme by covalent acetylation.
Reference: FA STEP1
11- Mechanism of action of inhaled antiviral?
A- Inhibit DNA
B- Inhibit polysaccharide capsule.
C- Inhibit replication
Answer: C
Inhaled antiviral (Zanamivir; A neuraminidase inhibitors) is used for prevention and treatment of
influenza A and B. It inhibits influenza neuraminidase lead to decrease the release of progeny virus and inhibit virus replication.
Reference: FA STEP1
12- Which medication decreases the effect of OCPs?
A- Antiepileptic
B- Anticoagulant
Answer: A
Antiepileptics (barbiturates, phenytoin, and carbamazepine) are general inducers, they increase
the metabolism of OCPs thereby decreasing their effect.
Reference: kaplan Pharmacology for Step 1.
There were questions about the following: *Please read about these topics!
Digitalis side effects
Ca channel blockers side effects
Warfarin MOA
13- Which antidepressant can cause constipation:
Answer: TCAs Tricyclic antidepressants
Reference: Basic and Clinical Pharmacology - Bertram G. Katzung, 12th ed. P535
14- Aspirin inhibit which product formation?
A- Thromboxane A2
Answer: A
Aspirin irreversibly inhibits platelet cyclooxygenase 1 preventing the formation of prostaglandin
H2, and therefore thromboxane A2.
Reference: Clinically Oriented Pharmacology - J. G. Buch, P134
15- Aspirin can be used as an antipyretic but why does it cause hyperthermia if given in
high doses?

smle ,2016

1034
Answer: Salicylates also uncouple cellular oxidative phosphorylation, resulting in increased oxygen consumption, increased heat production, and fever with simultaneous decreased ATP production.
Reference: Critical Care Medicine - Perioperative Management
16- Idiopathic anovulatory drug to enhancing ovulation?
Answer: Clomiphene
Clomiphene is a weak estrogen-like hormone that acts on the hypothalamus, pituitary gland, and
ovary to increase levels of follicle-stimulating hormone (FSH) and luteinizing hormone (LH,
which is also important in the process of ovulation).
Reference: uptodate
17- Osteoporosis drug causes retrosternal pain or heart pain :
A- Raloxifene
B- Denosumab
Answer: A
it causes chest pain
Reference: uptodate
18- Mode of action of Montelukast?
A- Inhibit leukotriene
Answer: A
Montelukast is a selective leukotriene receptor antagonist that inhibits the cysteinyl leukotriene
CysLT1 receptor. This activity produces inhibition of the effects of this leukotriene on bronchial
smooth muscle resulting in the attenuation of bronchoconstriction and decreased vascular permeability, mucosal edema, and mucus production.
Reference: uptodate
19- What antihypertensive medication increase uric acid and glucose?
Answer: Thiazide
Reference: Basic and Clinical Pharmacology - Bertram G. Katzung, 12th ed. P261
20- Aspirin side effect :
A- Dry mouth
B- Constipation
C- Diarrhea
Answer: C
Reference: uptodate
21- What anticoagulant can distract the clot?
A- Enoxaparin
B- Heparin
C- Aspirin
D- none of the above
Answer: None of the above .

smle ,2016

1035
Correct answer is fibrinolytics (Streptokinase, Urokinase, Alteplase (t-PA), Reteplase (Analogue
of alteplase), Tenecteplase).
Reference: Rang & Dale's Pharmacology
22- Man got a bee sting then his wife trying look for the epinephrine what it is going to inhibit?
A- Leukotriene release from macrophages
B- Cross reactivity with the cardiac
C- Inhibit immunocomplex formation
Answer: Epinephrine is a direct-acting sympathomimetic drug that acts as an agonist at alpha
and beta-adrenergic receptors. It produces vasoconstriction to counteract the vasodilation and resulting hypotension associated with anaphylaxis. The bronchodilatory effects of epinephrine and
its ability to reduce mucosal edema relieve bronchoconstriction and improve respiratory effort.
Epinephrine also down-regulates the release of histamine, tryptase, and other inflammatory mediators from mast cells and basophils, improving respiratory function and reducing the pruritus,
urticaria, angioedema, and gastrointestinal symptoms which occur after allergen exposure.
Reference: Medscape
23- Elderly patient with low back pain, what is the drug cause relaxation of muscle ?
A. Diazepam
B. Pilocarpine
Answer: A
Reference: http://www.ncbi.nlm.nih.gov/pmc/articles/PMC4103716/
24- Mechanism of action of combined oral contraception & progesterone?
Answer: Progestagen negative feedback decreases the pulse frequency of gonadotropin-releasing hormone (GnRH) release by the hypothalamus, which decreases the release of follicle-stimulating hormone (FSH) and greatly decreases the release of luteinizing hormone (LH) by the anterior pituitary. Decreased levels of FSH inhibit follicular development, preventing an increase in
estradiol levels. Progestagen negative feedback and the lack of estrogen positive feedback on LH
release prevent a mid-cycle LH surge. Inhibition of follicular development and the absence of a
LH surge prevent ovulation.
Reference: Basic and Clinical Pharmacology - Bertram G. Katzung, 12th ed. P716-730
25- Type 2 diabetic patient on gliclazide and other oral hypoglycemic agent -I forgot what
is was- heared about incretins in the news and he wants more about it, what is its mechanism of action?
A- Increases sensitivity of the receptors to insulin
B- Delay gastric emptying
C- Decrease gluconeogenesis
D- Increase insulin secretion
answer: D
Reference: http://www.ncbi.nlm.nih.gov/pmc/articles/PMC2696340/
26- what is the contraindication for the use of prostaglandin F2 alpha which is used in induction of labour?

smle ,2016

1036
A- Asthma
B- Hypertension
C- Diabetes mellitus
Answer: A
Reference: http://www.ncbi.nlm.nih.gov/m/pubmed/6961113/
27- A teacher who suffers from headache on temporal side, CT normal Blood pressure normal he was taking paracetamol but not improve well, what can you give ?
A- Aspirin
B- Ibuprofen
C- Triptans
Answer: C
If the scenario go with tension type; give ibuprofen. If it goes with temporal arteritis; give steroid
with aspirin. If it goes with migraine; give triptans.
Reference: Medscape
28- Atypical antipsychotic with least EPS risk :
A- Clozapine
answer : A
EPS is more with risperidone > olanzapine > quetiapine > clozapine and aripiprazole.
Risperidone was associated with more use of antiparkinson medication. Clozapine had significantly less use of antiparkinson medication than risperidone. Quetiapine showed significantly
less use of antiparkinson medication than the 3 other SGAs it was compared with (olanzapine,
risperidone, and ziprasidone).
Reference: Medscape
29- Atypical antipsychotic that causes weight gain :
A- Quetiapine
B- Risperidone
C- Aripiprazole
Answer: A+B
Almost all AP showed a degree of weight gain after prolonged use, except for amisulpride, aripiprazole and ziprasidone
Reference: http://www.ncbi.nlm.nih.gov/pmc/articles/PMC3998960/#!po=17.6802
30- Patient with acute MI typical scenario, the analgesia he was given will work on :
A- Inhibition of COX
B- Work on P-peptide or receptor can't remember exactly.
Answer: Morphine is the analgesic used in this case, it acts on the mu-opioid receptors. The
mechanism of respiratory depression involves a reduction in the responsiveness of the brain stem
respiratory centers to increases in carbon dioxide tension and to electrical stimulation. It also
binds to and inhibits GABA inhibitory interneurons.
Reference: Medscape, DrugBank
31- Long scenario pt on metformin and sulfonylurea and not controlled!
A- Acarbose

smle ,2016

1037
Answer: Certainly not Acarbose, in case if two oral agents didnt control hyperglycemia, switch
to Insulin!
Reference:http://www.uptodate.com/contents/management-of-persistent-hyperglycemia-in-type2-diabetes-mellitus
32- Drug that reduces heart rate and peripheral resistance in HTN :
A- Carvedilol
B- Hydralazine
C- Amlodipine
D- Thiazide
Answer: A
Carvedilol (Nonselective Beta blocker will decrease the heart rate)
Reference: Basic and Clinical Pharmacology - Bertram G. Katzung, 12th ed. P223
33- Patient diagnosed with duodenal ulcer he was prescribed medication 1 month ago now
he have gynecomastia which medication?
a- Cimetidine
b- lansoprazole
answer: A
. Cimetidine. Long-term use of cimetidine at high dosesseldom used clinically todaydecreases testosterone binding to the androgen receptor and inhibits a CYP that hydroxylates estradiol. Clinically, these effects can cause galactorrhea in women and gynecomastia, reduced sperm
count, and impotence in men.
Reference: goodman Gilman
34- symptoms of URTI in patient, keeping in mind current flu season you prescribed antiviral which works on:
A- protease
B- DNA polymerase
Answer: none of the above.
The antiviral is likely to be Oseltamivir or Zanamivir, thus the answer will be neuraminidase inhibitor (Flu virus surface protein).
35- female patient on anti-epileptic drugs presented with gum hypertrophy and excessive
hair growth on face, chest back?
A- Phenytoin
Answer : A
The major systemic side effects of phenytoin are gingival hypertrophy, body hair increase, rash,
folic acid depletion, and decreased bone density.
Reference: Uptodate
36- Whats given with NSAIDs to decrease its side effects?
A- Cimetidine
B- Pseudoephedrine
Answer: The side effect (I believe) that we are concerned about here is Peptic Ulcer Disease. If
PPIs or Misoprostol is the answer then choose either (PPIs are generally safer but no consensus).

smle ,2016

1038
Dont choose H2s as such an option is not supported by medical evidence. Bear in mind that you
might have to avoid PPIs, for example when the patient is on clopidogrel (Coronary Artery
pt..etc), Youd also avoid Misoprostol in pts who are pregnant or expected to be pregnant as it is
an abortifacient medication.
Reference : UpToDate
37- TCA most associated with weight gain:
Answer : Amitriptyline is associated with a great tendency to gain weight. Doxepin, and
Nortriptyline all have intermediate tendency.
Reference : Sadock and Kaplan psychiatry
38- Antidepressant that causes constipation?:
Answer : Amitriptyline. Urinary retention, blurred vision, dry mouth and constipation are common anticholinergic side effects associated with TCAs. Amitriptyline Is the most anticholinergic.
Reference :Sadock and Kaplan psychiatry, P1106
38- A patient with heartburn taking antacids. She had rheumatic fever 1 week ago and was
started on aspirin. What side effect she can develop?
A- Constipation
B- Diarrhea
C- Dry mouth
D- Galactorrhea
Answer: B
Reference : Goodman Gilman
39- Antacid causing constipation :
A- Sodium chloride
B- Calcium
Answer: Aluminium Hydroxide.
Reference : MedLine
40- Most common side effect of Atropine : (No Dry Mouth in options)
Answer: Decrease secretions (Sweating, salivary, bronchial).
Reference: Kaplan USMLE step 1 Pharmacology lecture notes, P50
41- Most common side effect of Anticholinergic :
Answer: Frequency not always defined. Severity and frequency of adverse reactions are
dose related.
Effects in order of increasing dose are: Decreased secretions > Mydriasis and Cycloplegia > Hyperthermia with resulting vasodilation > Tachycardia > Sedation > Urinary retention and constipation > Behavioral excitation and hallucination.
Reference: Kaplan USMLE step 1 Pharmacology lecture notes, P50
42- Absolute OCP contraindication :
A- undiagnosed breast mass.
B- previous hx of DVT

smle ,2016

1039
Answer: B
Reference:http://www.rch.org.au/rch_gynaecology/for_health_professionals/Contraindications_to_the_OCP/
43- A female using OCP developed black discoloration on her face, what's your diagnosis?
Answer: Melas
Reference :http://www.dermnetnz.org/colour/melasma.html
44- Side effect of nitroglycerin :
A- Bradycardia
B- Headache
C- Sexual dysfunction
Answer: B
Reference: Basic and Clinical Pharmacology - Bertram G. Katzung, 12th ed. P208
45- Patient ate a lot of aspirin tablets, How will you manage?
Answer: No options given, likely answer NaHco3. Read below.
Decontamination - Activated charcoal (AC) effectively absorbs aspirin, and at least one initial
dose (1 g/kg up to 50 g PO) should be given to all alert and cooperative patients and all intubated
patients via orogastric tube who present within two hours of ingestion
Alkalinization of serum and urine Alkalinization with sodium bicarbonate is an essential component of management of the aspirin-poisoned patient.
Reference : UpToDatev
*For further informations please read about Salicylate Toxicity Treatment & Management.
46- Patient on lovastatin, his HDL and Cholesterol level in lower abnormal levels. What to
add?
A) Omega 3
B) Cholestyramine
Answer: A
Omega 3 because it increases HDL.
Reference:https://login.medscape.com/login/sso/getlogin?urlCache=aHR0cDovL3d3dy5tZWRzY2FwZS5vcmcvdmlld2FydGljbGUvNDc5NDk5XzU=&ac=401
47- Mechanism of aspirin in pregnant lady with history of abortions :
Answer: Low doses (typically 75 to 81 mg/day) are sufficient to irreversibly acetylates serine
530 of cyclooxygenase (COX)-1. This effect inhibits platelet generation of thromboxane A2, resulting in an antithrombotic effect.
Reference : UpToDate
48- Mechanism of aspirin in old man with pains :
Answer: Intermediate doses (650 mg to 4 g/day) inhibit COX-1 and COX-2, blocking prostaglandin (PG) production, and have analgesic and antipyretic effects.
Reference : UpToDate
smle ,2016

1040

49- Glaucoma drugs side effect:


Answer: Insufficient info, refer to the picture

50. Tx of chronic pain , all except :


A- Naloxone
B- Salicylate
C- Ibuprofen
D- Acetaminophen
Answer: A
all of them are correct except for Naloxone which has a reverse action opioid antagonist
51. Clonidine decrease the effect of which of the following drugs?
Answer: ?Clonidine may decrease the effectiveness of levodopa to control the symptoms of Parkinson disease.
indicated for the treatment of hypertension in adults, but is also used for anesthetic premedication, spinal anesthesia, opioid detoxification, alcohol withdrawal, smoking cessation, and amelioration of postmenopausal hot flashes [ 5-9 ]. In children, it is used in the treatment of attention
deficit disorder with hyperactivity, refractory conduct disorder, and Tourette's syndrome [ 10-12
]. In a mail survey of pediatricians, clonidine was also the second-most commonly prescribed
(off-label) medication for treating sleep disturbances in children [ 13 ].
Reference: uptodate
52- What is the mechanism of action of denosumab antiresorptive properties?
smle ,2016

1041
A. cases osteoclastic apoptosis
B. inhibit RANKL
Answer: B
Denosumab is a fully human monoclonal antibody that binds the cytokine RANKL (receptor activator of NFB ligand), an essential factor initiating bone turnover. RANKL inhibition blocks
osteoclast maturation, function and survival, thus reducing bone resorption.
In contrast, bisphosphonates bind bone mineral, where they are absorbed by mature osteoclasts,
inducing osteoclast apoptosis and suppressing resorption. These differences in mechanism influence both the onset and reversibility of treatment.
Reference: Pubmed.
53.What is the side effect of thalidomide in pregnancy?
Answer: Thalidomide can cause embryofetal harm when given to a pregnant female; it is a human teratogen. The use of thalidomide in humans has produced a high rate of severe and
lifethreatening birth defects such as phocomelia, amelia, hypoplasticity of the bones, absence of
bones, external ear abnormalities (including anotia, micropinna, small or absent external auditory
canals), facial palsy, eye abnormalities (anophthalmos, microphthalmos), and congenital heart
defects.
thalidomide was a widely used drug in the late 1950s and early 1960s for the treatment of nausea
In pregnant women. It became apparent in the 1960s that the thalidomide treatment resulted in
sever birth defects in thousands of children.
Reference: pubmed
54- What is the mechanism of polio vaccine (Both types; oral and IV)?
Answer: Infection with poliovirus causes IgM and IgG responses in the blood, but mucosal IgA
is vital for blocking infection. This antibody can neutralize poliovirus in the intestine, the site of
Primary infection . the live attenuated polio virus vaccine is effective because it elicits as strong
mucosal IgA response and provide in intestinal immunity . in contrast , the injected (salk) polio
vaccine does not produce intestinal immunity, and therefore is less effective at preventing spread
of poliovirus in a population.
Reference: Pubmed.
55- What is the action of Atropine?(SYMPATHETIC)
A.Increase heart rate.
Answer: A
atropine blocks vagal nerve activity on the heart and increases heart rate.
Reference:https://login.medscape.com/login/sso/getlogin?urlCache=aHR0cDovL3d3dy5tZWRzY2FwZS5jb20vdmlld2FydGljbGUvODI5MjI3XzI=&ac=401
56-What is the mechanism of action of Orlistat?
A-decrease cholesterol
B-increase satiety
C-decrease pancreatic enzymes
Answer: C

smle ,2016

1042
Orlistat gastrointestinal lipase inhibitor or reduces fat absorption by 30% by inhibition of pancreatic lipase .
Reference: Toronto Notes.
57- Side effect of anticholinergic drugs :
A-Diarrhea
B-Excess Salivation
C-Blurred vision
Answer: C
Dry mouth, urinary retention, constipation, blurred vision, toxic confusional states.
Reference: Toronto Notes.
58- Which of the following medications used in treating osteoporosis and can cause heartburn sensation?
A-Denosumab.
B-Risedronate.
C-Raloxifene.
Answer: B
Reference: Kaplan Pharmacology for Step 1.
59-Regarding mechanism of action of Aspirin on enzymes:
a. Stimulate cyclooxygenase.
b .irreversible inactivation of the cyclocoxygenase (cox ) enzyme .
Answer: B
Irreversibly inhibits cyclooxygenase (both COX1 and COX2) enzyme by covalent acetylation.
Reference: FA STEP1
60. Mechanism of action of inhaled antiviral :
A-Inhibit DNA
B-Inhibit polysaccharide capsule.
C-inhibit replication
Answer: C
Inhaled antiviral (Zanamivir; A neuraminidase inhibitors) is used for prevention and treatment of
influenza A and B. It inhibits influenza neuraminidase lead to decrease the release of progeny virus and inhibit virus replication.
Reference: FA STEP1
61- Which medication decreases the effect of OCPs :
A-Antiepileptic
B-Anticoagulant
Answer: A
Antiepileptics (barbiturates, phenytoin, and carbamazepine) are general inducers, they increase
the metabolism of OCPs thereby decreasing their effect.
Reference: kaplan Pharmacology for Step 1.
There were questions about the following: * Please read about these topics :
smle ,2016

1043
Digitalis side effects
Ca channel blockers side effects
Warfarin MOA
62- Which antidepressant can cause constipation:
Answer: TCAs Tricyclic antidepressants
Reference: Basic and Clinical Pharmacology Bertram G. Katzung, 12th ed. P535
63- Aspirin inhibit which product formation?
A. Thromboxane A2
Answer: A
Aspirin irreversibly inhibits platelet cyclooxygenase 1 preventing the formation of prostaglandin
H2, and therefore thromboxane A2.
Reference: Clinically Oriented Pharmacology J. G. Buch, P134
64- Aspirin can be used as an antipyretic but why does it cause hyperthermia if given in
high doses?
Answer:Salicylates also uncouple cellular oxidative phosphorylation, resulting in increased oxygen consumption, increased heat production, and fever with simultaneous decreased ATP production. Reference: Critical Care Medicine Perioperative Management.
65- Idiopathic anovulatory drug to enhancing ovulation:
Answer:Clomiphene is a weak estrogenlike hormone that acts on the hypothalamus, pituitary
gland, and ovary to increase levels of folliclestimulating hormone (FSH) and luteinizing hormone (LH, which is also important in the process of ovulation).
Reference : uptodate
66- Osteoporosis drug causes retrosternal pain or heart pain :
A. Ralixofin
B. Densomub
Answer: A
Reference: uptodate
67- action of heparin :
Answer: Heparin : act at multiple sites in coagulation process : bind to antithrombin III catalyzing to IIa,IXa,Xa,XIa,and XIIa resulting in rapid inac .
68: Which of the following well break acute coronary thrombi, causing MI?
A. ASA
B. Heparin
C. Alteplase
D. Enoxaparin (LMWH)*
Answer :
69. A patient presented to the ER with respiratory depression and pinpoint pupils :
smle ,2016

1044
Answer : Opioid as cocaine , morphine toxicity.
70-What is the mechanism of warfarin?
Answer : inhibit Vit K-dependent coagulation factor synthesis (II , VII , IX , X , protein C, S).
71- Which one of these drugs will cause hyperpigmentation?
A- ccb
B- amiodarone
C- digoxin
Answer: B
amiodarone cause blue-grey skin discoloration in long term use .
pigmentation may be induced by a wide variety of drugs; (NSAIDs), phenytoin(anticonvulsant ) ,
antimalarials, amiodarone, antipsychotic drugs,dexamethasone.
72-patient with HTN complaining of cough and he wants to change the drug to another
one with the same effect without cough as a side effect :
answer:
probably he is using ACEI .. We give ARB instead .
73-which of the following is a side effect of atropine ?
1- vasoconstriction
2- decrease IOP
3- decrease urine output
4- dry of mouth
Answer:D
74-Medication for acidity and heartburn that causes constipation :
Answer: aluminum hydroxylase as Gaviscon.
75- you need to know All The Antidepressant side effects
A) SSRI
B) MOI
C) TCA
Answer: Side effect of :
SSRIs :Nausea , Nervousness, agitation or restlessness , Dizziness , Reduced sexual desire or
difficulty reaching orgasm or inability to maintain an erection (erectile dysfunction) , Drowsiness
,Insomnia , Weight gain or loss , Headache , Dry mouth ,Vomiting , Diarrhea
MOI : The most common side effects : Dry mouth
Nausea, diarrhea or constipation
Headache , Drowsiness, Insomnia ,Skin reaction at the patch site
Dizziness or lightheadedness .
76-Ribavirin side effect :
A- Lactate
B- liver damage
smle ,2016

1045
C- renal damage
D- anemia
Answer: D
77-what is the mechanism of action of oxybutynin ?
A-blocks muscarinic cholinergic receptors
B- block b 2
C- activate a adrenergic
D- activat
Answer: A
78.hypertensive patient with BPH , what to give ?
A-prazosin
Answer:A
alpha blocker
79-pt had MI , he was given sublingual nitrate and analgesia , the MOA of the analgesic
that was given :
Answer: If analgesia morphine , it interacts with opoid receptor .
80-what medication that causes hypertensive crisis with Tyramine :
A) MAOI
B) TCA
Answer: A
81-What is the best drug to prevent postoperative thromboembolism?
A) LW heparin
B) Uf heparin
C) Warfarin
D) Enoxaparin
Answer: A
82. primary respiratory alkalosis and a primary metabolic acidosis is characteristic of salicylate poisoning,
83-Which one of these drugs can cause seizure ?
A- isoniazid
B- ethambutol
C- ripaficin
D- Pyrazinamide
Answer : A
84-Epinephrine works on which cytokines?
Answer:it Stimulates alpha and beta adrenergic receptor.epinephrine down-regulate the release
of histamine, tryptase

smle ,2016

1046
85-Q about side effect of aspirin and antacid :
Answer: Aspirin SE: Conditions of Excess Stomach Acid Secretion, Nausea, Vomiting, Heartburn,
Irritation of the Stomach (Cramps)
Antacid SE: cause nausea, constipation, diarrhea, or headache. Diarrhea is more common with
this product than constipation .
86- Regarding the previous Q , what should be given ?
Misprostol
Answer: A
Prevents stomach ulcers caused by nonsteroidal anti-inflammatory drugs (NSAIDs).
87- A woman on phenobarbital, what you will do while breastfeeding ?
A- stop drug
B- continue drug
C- ween child 3 week before starting breastfeeding.
D- stop it one month before starting breastfeeding.
Answer : B
89- which antithyroid drug can cause sore throat and pancytopenia ?
A-Methimazole
B-propranolol
Answer: A
90-Affective half life of fluextine :
A- 18 hr
B- 2 day
C- 4 day
D- 9 day
Answer: C
91-case of digoxin toxicity :
Cardiovascular: Accelerated junctional rhythm, asystole, atrial tachycardia with or without
block, AV dissociation, first-, second- (Wenckebach), or third-degree heart block, facial edema,
PR prolongation, PVCs (especially bigeminy or trigeminy), ST segment depression, ventricular
tachycardia or ventricular fibrillation
Central nervous system: Dizziness (6%), mental disturbances (5%), headache (4%), apathy, anxiety, confusion, delirium, depression, fever, hallucinations
Dermatologic: Rash (erythematous, maculopapular [most common], papular, scarlatiniform, vesicular or bullous), pruritus, urticaria, angioneurotic edema
Gastrointestinal: Nausea (4%), vomiting (2%), diarrhea (4%), abdominal pain, anorexia
Neuromuscular & skeletal: Weakness .Ocular: Visual disturbances (blurred or yellow vision)Respiratory: Laryngeal edema
92-Q about association between taking aspirin for viral illnesses and the development of
Reye syndrome in children.

smle ,2016

1047

93- Angioedema is a side effect of which drug?


A. BB
B. CCB
C. ACEI
Answer: C
94: How calcium channel blocker lead to edema ?
Answer:Increase hydrostatic pressure
95- Metformin mechanism of action :
Answer:The main effect of this drug from the biguanide family is to acutely decrease hepatic
glucose production, mostly through a mild and transient inhibition of the mitochondrial respiratory-chain complex 1. In addition, the resulting decrease in hepatic energy status activates the
AMP-activated protein kinase (AMPK), a cellular metabolic sensor, providing a generally accepted mechanism for metformin action on hepatic gluconeogenic program. The demonstration
that the respiratory-chain complex 1, but not AMPK, is the primary target of metformin was recently strengthened by showing that the metabolic effect of the drug is preserved in liver-specific
AMPK-deficient mice.
Reference: http://www.ncbi.nlm.nih.gov/pmc/articles/PMC3398862/
96- What is the mechanism of prophylactic antiviral that given against flu?
A .DNA polymerase
B . RNA1
C. RNA 2
D .RNA3
97-Patient is taking Sublingual Isosorbide Dinitrate for myocardial infarction. What is the
side effects of the drug?
A. Hypotension
B. Hypokalemia
C. Heart Block
Answer:A
It is a nitrate.
98-A Pregnant woman with past history of depression on Paroxitine for long time, She is
asking the physician if she can use this medication or not during pregnancy . What do you
think ?
A- It is not safe because of the risk of cardiac congenital malformation
B- It is not safe...
C- It is safe...
D- It is safe...
Answer: A
99-What is the rule of metformin in PCOS?
A. Decrease glucose level

smle ,2016

1048
B. Decrease insulin resistance
C. Anti-androgenic
D. Menstrual regulation
Answer : B
100. the antidote of morphine :
Answer: Naloxone
101- Which one of these drugs will cause hyperpigmentation?
Answer : Phenytoin
102-patient with asthma exacerbation, Which drug will decrease the mucous secretion
more than broncho dilation :
A-oral steroids
B-ipratropuime
C-luekot
D-omalizumab
Answer : D
omalizumab in Asthma , ipratropuime in COPD
103-ADHD case what is the treatment?
answer : Atomoxitine
104-patient presented to ER with history of drug overdose and coma for the last 8 hours,
on examination the gag reflex was absent .the best manegement is :
A-iv naloxone
B-gastric lavage
C-immediate endotracheal intubation
D-coracol
Answer : C
105- Case of Absent seizure, whats the effect of injecting Phenytoin intrathecal?
A- inhibit secretion of substance P
B- seizure secondary of toxic metabolite
C- increase the seizure frequency
Answer : C
106-Medication for acidity and heartburn that cause constipation :
Answer:Alamonium hydroxide
107- which of the following will cause sedation in low dose ?
A- midazolam
B- lorazipam
Answer: A

smle ,2016

1049
108-Treatment of hyper TG :
A- NA
B- fibrate
C- pyt
Answer:B
109-Nitate sublingual side effect :
A-headache
B-hypokalemia
C-bronchospasm
Answer: A
if no hypotension in the choices >> then choose Headache .
110- Old pt with recent memory loss ,poor self-care and social withdrawal, what to give
him?
A- Neostigmen
B- Rivastigmen
Answer:B
111-Which of the following is side effect of morphine ?
A- dry cough
B- nausea , vomiting
C- tachypnea
D- anxiety
Answer: B
112-Aspirin toxicity resulting in :
A- metabolic acidosis followed by respiratory alkalosis
B- Respiratory alkalosis followed by Metabolic acidosis
Answer : B
113-What is the Side effect of postmenopausal hormonal therapy :
A- Breast cancer
B- Uterine cancer
C- DVT
Answer : A
114-Pt developed imbalance, tinnitus and decreased hearing. what drug did he take:
A. vancomycin
B. Isoniazid
C. Ethambutol
D. Pyramedizine
Answer : A

smle ,2016

1050
115-Male k/c Hyperthyroidism on drug ,Has infection , His CBC result shows neutropenia .
which antithyroid therapy has these this side effect?
A- methimazole
B- iodate
Answer : A
Reference: UpToDate
116-Methotrexate Overdose, treatment :
A- Folic Acid
B- Folonic Acid (leucovorin)
C- Cobalamine
Answer : B
Reference : http://www.ncbi.nlm.nih.gov/pubmed/15019303
117-What is the following drug causes erectile dysfunction :
A- sertolin
B- olanzapine
Answer : B
118- Tricyclic antidepressant amitriptyline Side effect :
A- Dystonia.
B- Hypersalivation.
C- Hyperpigmentation.
Answer : A
119- parenteral morphine most common Side effect :
ANSWER : side effects of Morphine >10 % pruritis
urinary retention (epidural /IT) 15-70 %
VOMITING (7-70%)
REFERENCE :MEDSCAPE
120- case of acetaminophen toxicity , the sequels of liver damage :

smle ,2016

1051
121- Patient with drug overdose presented with diaphoresis and dilated pupil
What is the most likely substance:
A . Parasympathomimetic
B. Organophosphate
C. Anticholinestrase
Answer: B
122- What are the things that dont need gastric lavage
A- Paracetamol
B- Aspirin
C- Cleaning products
Answer: C
123- which drug is safe in pregnancy ?
A- Paracetamol
B- Asprin
C- ibuprofen
Answer: A
Reference : Medscape
124- Patent took over dose of medication to suicide he is on some medication , when he
came develop high anion gap metabolic acidosis ..
What is the medication?
A- Aspirin
B- Chloro.
Answer : A
125- side effect of - Topical retinoids, derivatives of vitA
A- Photosensitivity
B- Moist skin
Answer : A
one of the side effects of isotretoin is dryness of skin
126- child with swelling in the leg developed skin manifestations .What is the side effect of
medication you well use ?
127-Pt taking sympathomimetics what's the Side effect?
A- increase HR
B- Hypotension
Answer : A
128. OCP effect on liver:
A. Hepatoma adenoma
B. HCC
C. hepatic hyperplasia
Answer : A

smle ,2016

1052
Hepatic adenomas are more common in women and may be caused by oral contraceptives
Reference: Davidson's principles & practice of medicine 22nd Edition (2014)
Ch 23 p 970
129-Resting to vigorously excersie
A- Pyrvate to lactate
B- Pyrvate to co2+o2
C- Decrease AMP
D- Decrease NDH/NADH
Answer : A
In exercising skeletal muscle, NADH production exceeds the oxidative capacity of the respiratory chain. This results in an elevated NADH/NAD+ ratio, favoring reduction of pyruvate to lactate. Therefore, during intense exercise, lactate accumulates in muscle.
Reference: Lippincott Illustrated Review of Biochemistry 6th Edition (2013)
Ch 8 p 103
130-Aerobic Metabolism :
Pyruvate is the starting molecule for oxidative phosphorylation via the Krebb&#39;s or citric
acid cycle. In this process, all of the C-C and C-H bonds of the pyruvate will be transferred to
oxygen. The pathway can be seen in the figure below
131-medication least to cause GI upset (NSAIDs)
A_aspirin
B-celecoxib
C-indomethacin
Answer : B
Celecoxib, when used without concomitant aspirin therapy, is associated with less GI bleeding
and dyspepsia than other NSAIDs.
Reference: Lippincott Illustrated Review of Pharmacology 6th Edition (2015)
Ch 36 p 455
132-Side effect of antacid :
- diarrhea
-constipation
Aluminum hydroxide tends to cause constipation, whereas magnesium hydroxide tends to produce diarrhea.
Reference: Lippincott Illustrated Review of Pharmacology 6th Edition (2015)
Ch 31 p 406
133- Pt with acute asthma given drug that works by inhibition of phophodiestrase enzyme
which drug was given ?
A- Salmeterol
B- Beclomethasone
C- AminoPhyllin
Answer: C

smle ,2016

1053
Aminophylline is a compound of the bronchodilator theophylline with ethylenediamine
Theophylline bronchodilates via inhibition of phosphodiesterase (PDE)
Reference: Kaplan Step 1 Lecture Notes 2016 (Pharmacology)
Ch 8 p 242
134-which of the following drugs lead to generalized muscle pain ?
medications of dyslipidemia
Answer : simvastatine
135-Acetaminophen toxicity , what will happen or what we afraid of
A- Depletion of glutathione in liver
B- NADPH
Answer: A
At normal therapeutic doses, acetaminophen is virtually free of significant adverse effects. With
large doses of acetaminophen, the available glutathione in the liver becomes depleted
Reference: Lippincott Illustrated Review of Pharmacology 6th Edition (2015)
Ch 36 p 456
136- Which oral contraceptive causes hyperkalemia?
Answer :Drospirenone ( A synthetic progestogen) may raise serum potassium due to antimineralocorticoid effects
Reference: Lippincott Illustrated Review of Pharmacology 6th Edition (2015)
Ch 26 p 357
137- Case of old patient with signs and symptoms of Parkinson Disease, where is the lesion?
Answer: basal ganglia
Reference : UpToDate
138-mechanism of action of propylthiouracil :
Answer : inhibit thyroperosidase enzyme centrally and inhibit conversion of T4 to active T3 peripherally.
Propylthiouracil (PTU) inhibits both the oxidative processes required for iodination of tyrosyl
groups and the condensation (coupling) of iodotyrosines to form T3 and T4
PTU also blocks the peripheral conversion of T4 to T3.
Reference: Lippincott Illustrated Review of Pharmacology 6th Edition (2015)
Ch 24 p 332

139-treatment for polymyalgia rheumatica


Answer: corticosteroid
140-treatment for dysmenorrhea
Answer:

smle ,2016

1054

141-treatment of acute myloid leukemia:


Answer : induction of remission by cytarabine
142-side effect of nitroglycerin :
Answer: headache
Headache is the most common adverse effect of nitrates. High doses of nitrates can also cause
postural hypotension, facial flushing, and tachycardia
Reference: Lippincott Illustrated Review of Pharmacology 6th Edition (2015)
Ch 21 p 286
143- tamoxifen SE or Complications :
Answer :The most frequent adverse effects of tamoxifen and toremifene are hot flashes and nausea. Due to its estrogenic activity in the endometrium, endometrial hyperplasia and malignancies
have been reported with tamoxifen therapy
Reference: Lippincott Illustrated Review of Pharmacology 6th Edition (2015)
Ch 21 p 286
144-Q about drug interaction :
Answer: viagra and sublingual nitrate
inhibits PDE-5 to increase intracavernosal cyclicGMP level in case of erectile dysfunction .
145-Drug used for hypertriglyceridemia in addition to statins:
Answer: Fibrates
The fibrates are used in the treatment of hypertriglyceridemias. They are particularly useful in
treating type III hyperlipidemia (dysbetalipoproteinemia), in which intermediatedensity lipoprotein particles accumulate.
Reference: Lippincott Illustrated Review of Pharmacology 6th Edition (2015)
Ch 23 p 318
146-Pt with acetaminophen overdose sinc 24 min ,conscious and cooperative, the best initial management?
A- Oral Acetylcysteine
B- IV Acetylcysteine
C- Charcoal
Answer : A
NAC (oral N-acetylcysteine) is the most effective when initiated 8 to 10 hours post ingestion.
Reference: Lippincott Illustrated Review of Pharmacology 6th Edition (2015)
Ch 48 p 633
147-Which of the following drugS from bisphosphonate work as ATP analog to suppress
osteoclasts ? All the medication given end with drone
Answer: Clodronate + Etidronate+ Tiludronate

smle ,2016

1055
Non-nitrogen containing bisphosphonates (tiludronate,clodronate and etidronate) induce osteoclasts to undergo apoptosis by forming a toxic adenosine triphosphate (ATP) analogue
Reference:http://www.orthobullets.com/basic-science/9058/bisphosphonates
148-white vaginal discharge pseudohyphae what is the treatment
A- metronidazole
Answer: A
Pseudohyphae = Vulvovaginal candidiasis
First line treatments include topical or oral antifungal (imidazole) agents
Reference: Hacker & Moore's Essentials of Obstetrics and Gynecology 5th Edition (2010)
Ch 22 p 269
Imidazoles are azole derivatives, which currently include butoconazole, clotrimazole, econazole,
ketoconazole, miconazole, oxiconazole, sertaconazole, sulconazole, terconazole, and tioconazole.
Reference: Lippincott Illustrated Review of Pharmacology 6th Edition (2015)
Ch 42 p 545
149-Case about a diabetic who takes metformin which resulted in correction of Hba1c,
Metformin lowers glucose by?
Answer:The main mechanism of action of metformin is reduction of hepatic gluconeogenesis.
Metformin also slows intestinal absorption of sugars and improves peripheral glucose uptake and
utilization. Weight loss may occur because metformin causes loss of appetite.
Reference: Lippincott Illustrated Review of Pharmacology 6th Edition (2015)
Ch 25 p 343
150-patient presented with typical symptoms of angina relieved by using sublingual drug
what is the mechanism of action of this drug?
Answer: relax smooth muscle and dilation of vein and artery and decrease resistance
Reference : UpToDate
151- Which medication used for heroin withdrawal symptoms?
Answer: Methadone 10 mg IM or 20 mg oral is usually sufficient to relieve symptoms of withdrawal without producing intoxication.
Reference: UpToDate
152-Boy with hypopigmented lesion in the back and extremities becomes lighter with sun
exposure ,Treatment:
Answer: antifungal
153-What is usually given with analgesics to reduce side effects?
Answer: Metoclopramide
154-Obsessive compulsive disorder drug mechanism of action:
A- Increase serotonin reabsorption
B- Decrease serotonin reabsorption
smle ,2016

1056
C- Increase activity of sertonin
D- Decrease activity of sertonin
Answer: B
A number of other psychiatric disorders also respond favorably to SSRIs, including obsessive
compulsive disorder.
Reference: Lippincott Illustrated Review of Pharmacology 6th Edition (2015)
Ch 10 p 136
155-Obese, acid reflux symptoms ,2week diagnosed with RF ,he is on Aspirin , the treatment is :
A- Cimetidine
Answer:
156-Anticonvulsant used in treatment of post herpetic neuralgia:
A- carbamazepine
B- phenytoin
Answer: A
Anticonvulsant medications are useful in the treatment of neuropathic pain, most notably in reducing the lancinating component of painful syndromes such as PHN. Drugs that have been evaluated in randomized trials include gabapentin, pregabalin, and valproic acid .
Reference: UpToDate
157-Bone marrow with megakaryocytes , What is the treatment?
Answer: prednisolone
158-child with barking cough after giving racemic Ebenverin, what is the next step ?
A- Antibiotics
B- steroid
Answer: B
Reference : UpToDate
159-patient on nebulized steroid , developed white patches on tongue ( moth) , what is the
diagnosis ?
Answer: oral thrush
160-case of DM with HTN , what is the treatment ?
Answer: ( ACEI )
ACE inhibitors slow the progression of diabetic nephropathy and decrease albuminuria and, thus,
have a compelling indication for use in patients with diabetic nephropathy
Reference: Lippincott Illustrated Review of Pharmacology 6th Edition (2015)
Ch 17 p 232
161-a pregnant lady , has +ve protein in urine and hypertension , she received mg sulphate
smle ,2016

1057
, what is prevent ( or treat ) :
Answer: seizure
Reference : UpToDate
162-patient with DM and HTN controlled on ACEI , furosemide and spironolactone ,electrolyte balance is normal, what you should do:
A- stop furosemide
B- stop spironolactone
C- add digoxin
Answer: B
163-child in the school K/c of DM loss of consciousness , last insulin dose not known , what
should you do:
A- give IV Dextrose
B- SC insulin
C- urgent transfer to hospital
Answer: A
164-Antidot of digoxin overdose :
Answer: Digibind
Early recognition of digitalis toxicity and prompt administration of Fab fragments is essential for
the successful treatment of severe poisoning
Reference : UpToDate
165-Old patient with Alzheimer's dementia, became agitated and have hallucinations and
delusions. What is the appropriate drug in his case ?
Answer: Haloperidol
166-the Best treatment for depression in adolescents and children :
Answer: Fluoxetine
167-Nasal drop vasoconstrictor associated with :
Answer: Rebound phenomena
168-patient with Hx of 4 time of URTI within 2 months on antithyroid medication , which
medication he is taking ?
Answer: Methimazole
169-elderly patient with right hip pain tenderness ,the joint with normal adduction, flexion
and abduction. the best next management is:
A- NASID
B- Physiotherapy
smle ,2016

1058
Answer: A
Reference: Lippincott Illustrated Review of Pharmacology 6th Edition (2015)
Ch 36 p 452

170-If you give a patient ACE inhibitors for hypertension but there was no effect ,what is
the drug you are going to add ?
A- Thiazide diuretic
B- Beta blocker

Answer :A
Thiazides are useful in combination therapy with a variety of
other antihypertensive agents, including -blockers, ACE inhibitors,
ARBs, and potassium-sparing diuretics
Reference: Lippincott Illustrated Review of Pharmacology 6th Edition (2015)
Ch 17 p 229
171-Which of the following may present with hypoglycemic attack :
A- Metformin
B- Glitazone
C- Glyburide

Answer :

C
Major adverse effects of the sulfonylureas are weight gain, hyperinsulinemia, and hypoglycemia.
Reference: Lippincott Illustrated Review of Pharmacology 6th Edition (2015)
Ch 25 p 342
172- the mechanism of action of incretin :
A- increase insulin secretion
B- increase insulin sensitivity

smle ,2016

1059

Answer :A
The incretin mimetics are analogs of GLP-1 that exert their activity by acting as GLP-1 receptor
agonists. These agents improve glucose dependent insulin secretion, slow gastric emptying time,
reduce food intake by enhancing satiety
Reference: Lippincott Illustrated Review of Pharmacology 6th Edition (2015)
Ch 25 p 340
173-DM patient, his blood sugar not controlled by metformin and acarbose and he is allergic to sulpha, what you should add ?
A- Rivoglitazone
B- Acarbose
Answer

:
174- which of these drugs cause weight gain?
A- Risperidone
B- Olanzapine

Answer:A
The second-generation antipsychotic drugs (including Risperidone) are associated with a higher
risk of metabolic side effects, such as diabetes, hypercholesterolemia, and weight gain
Reference: Lippincott Illustrated Review of Pharmacology 6th Edition (2015)
Ch 11 p 148
175-Patient with renal stones was given a diuretic to lower the ca then he developed Gout ,
what medication was used :
A- Hydrochlorothiazide
B- Furosemide

smle ,2016

1060

Answer : B

unlike thiazides, loop diuretics increase the Ca2+ content of urine, whereas thiazide diuretics decrease it.
Reference: Lippincott Illustrated Review of Pharmacology 6th Edition (2015)
Ch 17 p 230
Both Thiazide and loop diuretics can cause hyperuricemia
Reference: Davidson's principles & practice of medicine 22nd Edition (2014)
Ch 25 p 1088
176-prophylactic of rheumatic disease in normal patient :
A-penicillin
Answer : A
177-what is the antithyroid used in pregnancy?
A- propylthiouracil
Answer : A
PTU (propylthiouracil) is recommended during the first trimester of pregnancy due to a greater
risk of teratogenic effects with methimazole.
Reference: Lippincott Illustrated Review of Pharmacology 6th Edition (2015)
Ch 24 p 332
178- patient with gout you will not use:
A. spironolactone
Answer : A
Both Thiazide and loop diuretics can cause hyperuricemia
Reference: Davidson's principles & practice of medicine 22nd Edition (2014)
Ch 25 p 1088
179-Patient on statin for HTN, lab showed high triglyceride, what will you add?
A- fibrate
Answer : A
180-dexamethasone toxicity is reverse by :
Answer:
181-Which antiparkinson cause hepatotoxicity?
Answer:tolcapone (Tasmar)
smle ,2016

1061
fulminating hepatic necrosis is associated with tolcapone use.
Reference: Lippincott Illustrated Review of Pharmacology 6th Edition (2015)
Ch 8 p 113
182-Which of those medication cause anxiety
Answer:SSRI
Reference : USMLE step 2 ck
183-Which of bisphosphonate causes inhibition of osteoclast activity?
Answer:Alendronate
Alendronate inhibits osteoclasts -mediated bone-resorption. Like all bisphosphonates, it is chemically related to inorganic pyrophosphate, the endogenous regulator of bone turnover. But while
pyrophosphate inhibits both osteoclastic bone resorption and the mineralization of the bone
newly formed by osteoblasts, alendronate specifically inhibits bone resorption without any effect
on mineralization at pharmacologically achievable doses.
184-Which of the following should be avoided during pregnancy?
A. Amoxicillin
B. Cephalosporins
C. Fluoroquinolones
Answer: C
Articular cartilage erosion (arthropathy) has been observed in immature animals exposed to fluoroquinolones. Therefore, these agents should be avoided in pregnancy and lactation and in children under 18 years of age.
Reference: Lippincott Illustrated Review of Pharmacology 6th Edition (2015)
Ch 40 p 516
185-Q about the clinical presentation of phenytoin side effect StevensJohnson syndrome;
186-post MI developed palpitation, ECG showed (VT). the managment :
A- Metoprolol
B- Adenosine
Answer: A
They are also used for atrial flutter and fibrillation and for AV nodal reentrant tachycardia. In addition, -blockers prevent life-threatening ventricular arrhythmias following a myocardial infarction.
Metoprolol is the -blocker most widely used in the treatment of cardiac arrhythmias
Reference: Lippincott Illustrated Review of Pharmacology 6th Edition (2015)
Ch 20 p 275
187-What antihypertensive decreases preload as well as causing vasodilatation:
Answer: ACEI , CARVEDILOL
The ACE inhibitors lower blood pressure by reducing peripheral vascular resistance without reflexively increasing cardiac output, heart rate, or contractility.
ACE inhibitors also reduce both cardiac preload and afterload, thereby decreasing cardiac work.
Reference: Lippincott Illustrated Review of Pharmacology 6th Edition (2015)

smle ,2016

1062
Ch 17 p 231-232
188-What is the antidote for paracetamol toxicity?
A- N-acetylcysteine
Answer: A
189-Newly diagnosed Hypertensive patient, came to the primary clinic complaining of dry
cough and shortness of breath. Which medication he
used ?
A- Pernidopril
B- Valsartan
C- Atenolol
D- Thiazide
Answer : A
Reference: Lippincott Illustrated Review of Pharmacology 6th Edition (2015)
Ch 17 p 233
190- A scenario of a boy who developed aplastic anemia after receiving an antibiotic. What
is the antibiotic?
A- Tetracycline
B- chloramphenicol
Answer : B
Patients taking chloramphenicol may experience dose-related anemia, hemolytic anemia (seen in
patients with glucose-6-phosphate dehydrogenase deficiency), and aplastic anemia. [Note:
Aplastic anemia is independent of dose and may occur after therapy has ceased.]
Reference: Lippincott Illustrated Review of Pharmacology 6th Edition (2015)
Ch 39 p 509
191-case of a guy with high phosphate , Organophosphate elevated in which organ?
A-Liver
B- Kidney
C- Lung
Answer : C
192-case of urine incontinence . what is the mechanism of action of oxybutynin?
Answer: competitive antagonist of acetylcholine at postganglionic -muscarinic receptors
Oxybutynin is a synthetic atropine-like drug that is used to treat overactive bladder. By blocking
muscarinic receptors in the bladder.
Reference: Lippincott Illustrated Review of Pharmacology 6th Edition (2015)
Ch 5 p 69
193-the treatment of acute gout is :
Answer: indomethacin

smle ,2016

1063
NSAIDs, corticosteroids, or colchicine are effective alternatives for the management of acute
gouty arthritis. Indomethacin is considered the classic NSAID of choice
Reference: Lippincott Illustrated Review of Pharmacology 6th Edition (2015)
Ch 36 p 462
194- Which one of the following can be given to a patient with beta-lactam-resistant organisms?
A- Azithromycin
B- Vancomycin
C- Gentamicin
Answer: B
Reference : First Aid step 1
195- What are the antibiotics can be given safely during breast
feeding?
A- chloramphenicol,
B- cimetidine,
C- erythromycin,
D- tetracycline
answer: C

Reference : First Aid step 1


196-Long scenario of patients labs show low hg low platelet with normal reticulocyte which Antibiotic cause this ?
A- Tetracycline
B- cloroampincoal
C- cefepime
Answer
chloramphenicol cause aplastic anemia ( low reticulocyte )
Reference : First Aid step 1

smle ,2016

1064
197.case of UTI methicillen sensitive , treatment is :
A-Cloxacillin
Answer : A
Reference : First Aid step 1
198-DM pt. with necrotizing fasciitis . they asked about antibiotic combination
A- penicillin/gentamycin
B- amoxil/erythromycin..
C- piperacillin/tazobactam
D- penicillin/gentamycin
NB..read about antibiotic and its combinations piperacillin/tazobactam instead of carbapenem
Answer :C
carbapenem+clindamycin +anti MRSA vancomycin
Reference : First Aid step 1
199-Ciprofloxacin mechanism of action is :
A-Inhibits RNA
B-Inhibits DNA gyrase
C-Inhibits cell wall synthesis
D-Other
Answer:B
Reference : First Aid step 1
200-What is the treatment of shigella?
A-Cefuxime.
B-Ampicillin.
C-Metronidazole
Answer: B
Reference : First Aid step 1
201- treatment of positive oxidase bacteria ?
Answer:
Antipseudomonal penicillins include ticarcillin (3 g every four hours) and piperacillin (3 g every
four hours or 4 g every six hours).
-*Cephalosporins with antipseudomonal activity include:
Ceftazidime 2 g every eight hours
Cefoperazone 2 g every 12 hours (not available in the United States)
Cefepime 2 g every eight hours
*Monobactam:
Aztreonam 2 g every eight hours
*Fluoroquinolones:
Ciprofloxacin
Reference : First Aid step 1 , uptodate

smle ,2016

1065
202-bacterial meningitis in 14 month child I think? Gram positive cocci, what is the management?
A-amoxicillin
B-amoxicillin and gentamicin
C-ceftriaxone and vancomycin
D-vancomycin
Answer : C
Reference : Toronto note , First Aid step 1
203-enterococcus faecalis antibiotic and the pt is allergic to ampicillin , what to give ?
Answer:Erythromycin
Reference : First Aid step 1
204-Which of the following antibiotics is DNA gyrase and it works on what organism ?
Answer:Bacterial DNA gyrase is the target of many antibiotics, including nalidixic acid, novobiocin, and ciprofloxacin.
Quinolone inhibition of A subunit . Novobiocin inhibit B sub unit of DNA .
205-Which of the following medication causes vertigo?
A- Ethambutol
B- Streptomycin
Answer: B
Common side effects of streptomycin have included vestibular toxicity (nausea, vomiting, vertigo), paresthesia of the face, rash, fever, urticaria, angioneurotic edema, and eosinophilia. Side
effects may be more likely and more severe in patients with underlying renal insufficiency.
206-side effect of silver sulfadiazine :
Answer:common side effect : pain, burning , rash , pruritus , skin necrosis , skin discoloration.
Serious Reaction as :hypersensitivity reaction , steven johnson syndrome , aplastic anemia .
Reference : First Aid step 1
207. Bacteroids in gunshot wound abdomen, what antibiotics:
Answer:
208-which drug is safe for pregnant women:
A-cimetidine
B- cefodizime
C- erythromycin
D- ciprofloxacin
Answer:C
Ampicillin,Clindamycin,Erythromycin,Penicillin,Gentamicin,Ampicillin-Sulbactam
,Cefoxitin,Cefotetan,Cefazolin
Reference : First Aid step 1
209- best antibiotic for gram( - )bacilli:
Answer : cephalosporins

smle ,2016

1066

210- A woman has +ve Hep B surface antigen delivered a baby in a hospital and resaved
immunoglobulin and B vaccine within the first 12 hour ,, what are the recommendation for
breastfeeding ??
A- no breast feeding
B- breast feeding is okay
C- breast feeding after 12 hours
Answer: B
211-positive culture of budding yeast in urine, what is the management ?
A-Fluconazole
B-caspofungin
Answer :A
Reference : First Aid step 1
212-What antibiotic is contraindicated in a patient with culture showing sensitivity to
many antibiotics (vancomycin, Fluoroquinolones...) but resistant to Beta lactamase??
A- Gentamycin
B- Azithromycin
Answer:
213-Tx for fever, cough, bilateral infiltration :
A- Cipro
B- azithromycin
Answer: B.
IF they meant legionella in the question.
Reference: First Aid step 1
214-Which of the following antibiotics is DNA gyrase and it works on what organism ?
Answer: pseudomonas
215- Which of the following medications cause vertigo?
A- Ethambutol
B- Streptomycin
Answer: B
216-gastroenteritis due to salmonella ,which Antibiotic to use ?
Answer : Azithromycin is likely to be the preferred empirical treatment, often given together with ceftriaxone, in
developed countries where chloramphenicol is usually reserved for life-threatening situations, for which no alternatives are available, and physicians are reluctant to use fluoroquinolones in children and lack easy access
to gatifloxacin
Reference: http://emedicine.medscape.com/article/228174-treatment

217-long caseof patient with +ve giemsa stain, treatment :


A- minocycline
B- benznidazole
Answer : A
smle ,2016

1067
218- 27w pregnant woman ( with Hx of treated UTI with Abx on 12 w ) now she has symptoms of UTI , management is:
A- Referral to cystoscopy
B- Start abx
C- Cont abx then do culture
D- Referral to surgery
Answer: B
219 - patient has Pneumonia on amoxicillin, what you'll add ?
A- Vancomycin
B- Erythromycin
C- Azithromycin
220- thalidomide side effects in pregnancy:
A- spina bifida
B- teeth defects
C- kidney defects
D-limb defects
Answer: D
221-penile painless rash after use of sulfa drug description of the rash ?!
Answer: Blisters or a hive-like rash on the lining of the mouth, vagina, or penis that can spread all over the body
222-which antibiotic inhibits calcium and aluminium salt ?!
A- tetracycline
B- chloramphenicol
Answer : A
Reference :First Aid step 1
223-patient get beta lactamase organism , on amoxicillin , what to add :
Answer: clavulanic acid
Reference :First Aid step 1
224-an elderly with low hemoglobin and high lymphocyte and the flow cytometry showed
different types of CD ,What is the treatment ?
A- cyclophosphamide B- Rituximab +prednisolone C- Rituximab
225-TB medication causes red urine :
Answer: Rifampin
Reference :First Aid step 1

smle ,2016

1068
226-Pregnant woman with n. gonorrhea, what is the Treatment ?
Answer: ceftriaxone
227-pregnant female with UTI, which drug if safe during all trimesters:
A- ampicillin
B- nitrofurantoin
Answer:A
228-Pt with rheumatic fever on aspirin developed heartburn , what you will add?
Answer :Misoprostol
229-treatment of enteric fever :
A- ciprofloxacin
B- chloramphenicol
Answer: A
Reference :First Aid step 1
230-neonate with signs of sepsis, which empirical antibiotic to use ?
A- ampicillin
B- gentamicin
C- cefotaxime
Answer:C
Reference :First Aid step 1
231-Stab wound in the abdomen with bacteroid fragilis ,what is the best antibiotic to use ?
Answer: ampicillin
Reference :First Aid step 1
232-Effective half life of fluoxetine :
A- 18 hr
B- 2 day
C- 4 day
D- 9 day
Answer: C
233-Pneumococcal 13 conjugate type:
A- Toxoid
B- conjogoid
C- inactivated
D- -live
Answer:B
234.A culture was taken from the patient, it was shown to be resistant to beta lactam and

smle ,2016

1069
sensitive to fluoroquinolones, aminoglycosides and chloramphenicol. Which of the following antibiotic is contraindicated?
A- levofloxacin
B- chloramphenicol
C- gentamicin
D- azithromycin
Answer : D
Most likely Azithro as it is excreted in the Bile and will not be used to treat a UTI. Even if it is
sensitive in vitro, that does not mean itll get to our focus of infection in the body.

smle ,2016

1070

smle ,2016

1071

smle ,2016

1072

smle ,2016

You might also like